Download as pdf or txt
Download as pdf or txt
You are on page 1of 635

一 信:liuxue119118 , 们 信免 供

CIVIL PROCEDURE -
DISCOVERY QUESTIONS
CIVIL PROCEDURE - DISCOVERY QUESTIONS 1.

CIVIL PROCEDURE - DISCOVERY QUESTIONS

Question 1 Question 2

In response to a massive request for produc- A pedestrian filed an action against a driver
tion of documents in a large antitrust case, in federal district court, alleging negligence.
the defendant produced to the plaintiff several The attorney for the driver has interviewed an
million pages of electronically stored documents eyewitness whose testimony will clearly indicate
and physical documents. After producing the that the driver was at fault.
information, the defendant discovered that
it had erroneously provided to the plaintiff Must the driver disclose the existence and
several hard-copy letters between the defen- identity of the eyewitness to the pedestrian?
dant’s attorney and some of its officers that were
protected by the attorney-client privilege. (A) No, because the identity of the eyewitness
is protected from discovery under the work
What, if anything, can the defendant do about product doctrine.
this error?
(B) No, because the driver is not likely to use
(A) The defendant may notify the plaintiff the eyewitness as part of her case.
of the error and the defendant’s claim of
privilege, and the plaintiff then must return, (C) Yes, because the driver must disclose all
sequester, or destroy the letters. witnesses who have discoverable informa-
tion, even without a specific request.
(B) The defendant may retrieve the letters only
if it demonstrates extraordinary circum- (D) Yes, but only in response to an appropriate
stances that will cause substantial prejudice interrogatory.
if the letters are not returned.

(C) Because the letters subject to the claim of


privilege are hard-copy letters rather than
electronically stored information, they are
not subject to the “claw-back” provision and
the defendant cannot retrieve them.

(D) Because the letters are already in the


possession of the plaintiff, any claim of
privilege is waived, and the defendant
cannot retrieve them.
2. CIVIL PROCEDURE - DISCOVERY QUESTIONS

Question 3 Question 4

The owner of a nursery sued a manufacturer An inventor sued a rival in federal court for
in federal court for damage to his growing violating the inventor’s patent. At a deposition
trees after one of the manufacturer’s factories attended by lawyers for both parties, the rival
negligently emitted toxic gas. In interrogatories gave convincing testimony that his product was
answered on May 1, the nursery owner truthfully completely different from that of the inventor.
stated that the leaves on his red maple trees are Just before trial, the rival began serving a four-
normally out by April 15, but this year the red year sentence in the state penitentiary. The
maples do not have any leaves. A week later, on rival’s lawyer considered the inventor’s suit to be
May 8, all of the nursery owner’s red maples a nuisance action, and was certain that the rival’s
have normal leaves on them. deposition testimony was all that she would need
to introduce at trial to win the case. The inven-
Is he under a duty to supplement the responses tor’s lawyer objects to having the deposition read
he gave in the interrogatory? into evidence, arguing that a party’s deposition
may not be used in this way.
(A) No, unless the manufacturer asks for an
update during the discovery period. Will the court allow the deposition to be used?

(B) No, because the information provided in the (A) No, because imprisonment does not make a
interrogatory was true. party “unavailable.”

(C) Yes, because the nursery owner knows the (B) No, because the rival is a party.
information provided in the interrogatory is
no longer true. (C) Yes, because the deposition of a party may
be used for any purpose.
(D) Yes, unless the manufacturer could find the
information on its own. (D) Yes, because the rival is unavailable.

一 信:liuxue119118 , 们 信免 供
CIVIL PROCEDURE - DISCOVERY QUESTIONS 3.

Question 5 Question 6

A plaintiff is suing a defendant in federal A worker was fired by her employer. The
court for personal injuries arising out of an worker filed an action in federal district court,
assault and battery. The defendant has confided alleging that she was fired on the basis of
in his therapist certain matters that the plain- race. The employer has an internal memo that
tiff believes would be relevant to her case and suggests that race in fact was the impetus for the
helpful in finding further information. worker’s firing.

On request by the plaintiff, will the federal Must the employer provide a copy of the
court order discovery of what the defendant told memo to the worker as part of the employer’s
his therapist over the defendant's objection? required initial disclosures?

(A) Yes, because the information from the (A) Yes, because the employer must produce
therapist will lead to other discoverable all documents in its possession that are rel-
evidence. evant to any claim or defense in the action.

(B) Yes, if the judge orders that the records (B) No, but the employer must provide the
from the therapist be sealed. worker a description of the memo, and the
worker may obtain a copy using a request
(C) No, because the communication with the for production of documents.
therapist is privileged.
(C) No, because the employer must produce in
(D) No, unless the therapist waives the privi- its required initial disclosures only informa-
lege. tion about witnesses having discoverable
information.

(D) No, because the employer must produce


in its required initial disclosures only
documents and other information that
it may use to support its allegations and
defenses in the action.
4. CIVIL PROCEDURE - DISCOVERY QUESTIONS

Question 7 Question 8

The roof on a city’s new assembly hall A customer slipped and fell in a store,
collapsed after a snowstorm. The city filed in suffering a severe injury. As required by a state
federal court a civil action against the archi- regulation, the store’s manager conducted an
tect and builder, claiming negligence in design, investigation into the cause of the accident and
construction, and supervision. The city’s attorney filed a report with the appropriate state agency.
retained an expert construction engineer to assist The customer subsequently retained an attorney,
with the trial and provide opinions. who filed a civil action against the store in
federal district court. The complaint alleged that
Which of the following statements is correct the store negligently left a spill on the floor of
regarding the defendants’ right to depose the the store, which caused the customer’s fall. The
engineer? customer’s attorney served on the store a request
for production of documents, which included a
(A) If the city expects to call the engineer as an request for all documents and reports prepared
expert witness at trial, the defendants may by the store that relate to the customer’s fall and
depose the engineer if they properly notice injury.
a deposition and have a subpoena served on
the engineer. Must the store produce the report it filed with
the state agency?
(B) If the city expects to call the engineer as an
expert witness at trial, the defendants may (A) No, because the report was prepared for
depose the engineer only if they obtain a purposes unrelated to litigation or trial and
court order. thus is not subject to discovery.

(C) If the city does not expect to call the (B) No, because the report is privileged as work
engineer as an expert witness at trial, then product.
under no circumstances may the defendants
depose him. (C) Yes, because the store prepared the
document to comply with state regulations
(D) Regardless of whether the city expects to rather than to prepare for litigation or trial.
call the engineer as an expert witness at
trial, the defendants may not depose him (D) Yes, because any document that may
because his opinions are protected as work provide information that is relevant to a
product prepared in anticipation of litigation party’s claim or defense must be produced
and thus not subject to discovery absent a in response to a proper request for
showing of substantial need for the infor- documents.
mation and inability to obtain its substantial
equivalent by other means.
CIVIL PROCEDURE - DISCOVERY QUESTIONS 5.

Question 9 Question 10

A patron at a private park was injured while A motorcyclist filed a negligence action
operating a small boat he rented from the park. against a van driver in federal district court,
The patron subsequently filed a civil tort action seeking compensatory damages for a traffic
against the park in federal district court, seeking accident involving the parties. In a deposition of
to recover compensatory damages for the injury. the driver, the motorcyclist’s attorney asked the
Prior to trial, the park took the deposition of driver the following question: “Did you tell your
a witness, another visitor to the park who saw attorney that the accident that is the subject of
the patron’s accident. The park properly served this lawsuit was your fault?” The driver refused
notice of the deposition on all parties, and all to answer the question on advice of counsel. The
parties were represented at the deposition. The motorcyclist now seeks an order compelling the
deposition took place in the witness’s home driver to answer the question.
town, which is 140 miles from the park and the
court where the action is pending. At the time Should the court order the driver to answer the
of trial, the witness is at home, and the park’s question?
attorney seeks to read the witness’s deposition
testimony into evidence. The attorney for the (A) No, because the question seeks hearsay,
plaintiff patron objects to the use of the witness’s which is not admissible.
deposition.
(B) No, because the question seeks privileged
May the park present the deposition into information.
evidence?
(C) No, because the motorcyclist’s attorney can
(A) No, because the deposition testimony may simply ask the driver directly if the accident
be used only if the witness is unavailable to was the driver’s fault.
testify live at the trial.
(D) Yes, because the question seeks information
(B) No, because the deposition testimony may that is relevant to the claim or defense of a
be used only to impeach the testimony of party.
the deponent.

(C) Yes, because of the witness’s distance from


the court.

(D) Yes, regardless of the witness’s distance


from court, because all parties were
properly notified and had an opportunity to
be represented at the deposition.
6. CIVIL PROCEDURE - DISCOVERY QUESTIONS

Question 11 Question 12

A passenger suffered severe injuries in the A man filed a negligence action against a
crash of a small private jet. Seeking compensa- woman in federal district court, alleging that
tory damages, the passenger filed a products the woman negligently caused a collision
liability action in federal district court against between their vehicles and seeking compensa-
both the manufacturer of the aircraft and the tory damages. The man served on the woman
manufacturer of the jet engine with which the a request that the woman produce a specified
aircraft was equipped. The aircraft manufac- electronic recording for inspection or copying.
turer served interrogatories on the manufacturer The woman made the recording immediately
of the engine, requesting it to list all reports after the accident while waiting for the police to
in its possession concerning accidents during arrive. She made the recording for the specific
the last 10 years involving the type of engine purpose of preparing for litigation that she knew
involved in the crash and to list the individual(s) would ensue. The recording contains the obser-
who conducted the investigation(s). The engine vations of three witnesses to the accident. The
manufacturer assigned the task of responding to woman objected to the man’s request for produc-
the interrogatory to one of its vice presidents of tion of the recording, and the man filed a motion
operations. Although the vice president oversees to compel production of the recording.
all of the engine manufacturer’s operations, she
does not have all the information sought by the Should the court order the woman to produce
interrogatory. Some of the information is located the recording?
in each of the engine manufacturer’s eight
locations in various parts of the country. (A) Yes, unless the woman files a timely mo-
tion for a protective order.
How should the vice president respond to the
interrogatory? (B) Yes, if the court finds that the recording is
likely to lead to the discovery of admissible
(A) She is not required to respond because the evidence.
interrogatory was not served by an oppos-
ing party. (C) No, unless the court finds that the man
has substantial need for the recording in
(B) She should conduct an investigation to preparing his case and is unable without
determine what relevant documents the undue hardship to obtain substantially the
engine manufacturer has in any of its same information that is on the recording
locations and answer the interrogatory by other means.
based on that investigation.
(D) No, because the Federal Rules of Civil
(C) She should respond to the interrogatory by Procedure do not provide a mechanism for
listing whatever documents she has in her the discovery of tangible things such as
own files regarding accidents involving the recording media.
engines.

(D) She should object to the interrogatory on


the grounds that the information sought is
covered by the initial disclosures required
by the discovery rules.
一 信:liuxue119118 , 们 信免 供

CIVIL PROCEDURE - DISCOVERY QUESTIONS 7.

Question 13 Question 14

A passenger train derailed, causing injuries A homeowner entered into a contract with
to many passengers. Immediately following the a termite control company under which the
accident, as ordered by the corporate owner of company agreed to treat the homeowner’s house
the train, the general counsel took statements to prevent termite infestation and to repair any
from the train engineer and other employees on damage if termites infested the house. The
the train regarding what occurred, and wrote homeowner later discovered termite infestation
down what the employees said. Later, one of the and damage in the house. When the termite
injured passengers properly sued the corpora- company refused to pay for the damage, the
tion in federal court and sent interrogatories homeowner filed a breach of contract action
requesting the contents of the written statements against the company to recover compensa-
taken by the corporate general counsel. tory damages. During discovery, the company
properly served written interrogatories on the
Is the corporation required to answer the homeowner. One of the interrogatories asked
interrogatories? the homeowner to state the date on which the
homeowner purchased the house and the price
(A) No, because the statements are protected that the homeowner paid for the house. The
work product. homeowner does not recall the exact date nor
the precise purchase price. The homeowner
(B) No, because the statements are commu- knows, however, that he has a copy of the house
nications protected by the attorney-client purchase contract and closing document in files
privilege. in his attic.
(C) Yes, because it is a party to the case. Must the homeowner provide the requested
information in his response to the interroga-
(D) Yes, because the statements are relevant to tory even though he does not independently
the issues in the case. remember the requested facts?

(A) No, because the requested information ex-


ists only in documents in the homeowner’s
possession, so the company must serve a
request for production of documents on the
homeowner to obtain the information.

(B) No, because, if the company wishes to


obtain information directly from the
homeowner prior to trial, it must take the
homeowner’s deposition.

(C) No, because written interrogatories may


be used only to obtain opposing parties’
opinions or the opinions of expert witnesses
that opposing parties intend to call to testify
at trial.

(D) Yes, but the homeowner may provide the


documents that contain the information in
lieu of answering the interrogatory.

一 信:liuxue119118 , 们 信免 供
CIVIL PROCEDURE -
DISCOVERY ANSWERS
CIVIL PROCEDURE - DISCOVERY ANSWERS 1.

CIVIL PROCEDURE - DISCOVERY ANSWERS

Answer to Question 1

(A) The defendant may notify the plaintiff of the error, and the plaintiff then must return, sequester,
or destroy the letters. If a trial party inadvertently discloses privileged material to an opposing
party, it may still invoke a claim of privilege by notifying the opposing party of the disclosure
and the basis for the claim of privilege. Once notified, the opposing party must promptly return,
sequester, or destroy the specified information and take reasonable steps to retrieve the material
if it disclosed it to others. The opposing party also may not use or disclose the privileged material
until the claim is resolved. [Fed. R. Civ. P. 26(b)(5)(B)] (B) is incorrect because a party who
inadvertently discloses privileged material does not have to demonstrate extraordinary circum-
stances to retrieve the material. (C) is incorrect because hard-copy information and electronically
stored information are not treated differently in this situation. (D) is incorrect because inadver-
tently disclosing privileged material does not waive the privilege.

Answer to Question 2

(D) The driver must disclose the identity of the witness in response to an appropriate interrogatory.
Federal Rule 26(a) requires, as an initial disclosure, a party to reveal the name and contact infor-
mation of individuals who are likely to have discoverable information and who the disclosing
party may use to support his claims or defenses (unless the use would be solely for impeach-
ment). After initial disclosures are made, discovery proceeds, and the parties may continue with
discovery of nonprivileged information that is relevant to any party’s claim or defense, including
the names and contact information of any person who knows of any discoverable matter. Here,
the eyewitness would not need to be disclosed as an initial disclosure because the driver obviously
will not use the eyewitness to support the driver’s claim or defense. However, the identity of the
eyewitness would need to be disclosed eventually, assuming the pedestrian submits a proper
discovery request. This makes (D) correct and (B) incorrect. (A) is incorrect because the work
product doctrine does not prevent the disclosure of the existence of the eyewitness. Any materials
generated by the attorney would probably be protected under the work product doctrine (unless
a showing of substantial need and undue hardship can be made); however, the eyewitness’s name
and contact information would not be protected. (C) is an overbroad description of the initial
disclosure requirements and is, thus, incorrect.

Answer to Question 3

(C) The nursery owner is under a duty to supplement his responses. A party is under a duty to amend
a prior response if he knows that the response, though correct when made, is no longer true
and the circumstances are such that a failure to amend the response is in substance a knowing
concealment. Thus, (B) is incorrect. The opposing party need not request such information,
making (A) an incorrect answer. [Fed. R. Civ. P. 26(e)(2)] The fact that a party may be able to
find the information without using discovery does not relieve a party from disclosing information
in his possession; thus, (D) is incorrect.

Answer to Question 4

(D) The court will allow the deposition testimony to be introduced. The deposition of a witness,
whether or not a party, may be used by any party for any purpose if the witness is unavailable due
to, among other things, imprisonment. [Fed. R. Civ. P. 32(a)(4)] (A) is incorrect because imprison-
2. CIVIL PROCEDURE - DISCOVERY ANSWERS

ment does make a party unavailable; it is specifically mentioned in Rule 32 as a potential cause
for unavailability. (B) is incorrect because a party’s deposition may be used as described above.
(C) is too broad. A party’s deposition may be used only by the adverse party for any purpose.
[Fed. R. Civ. P. 32(a)(3)] Here, the rival is seeking to admit his own deposition.

Answer to Question 5

(C) The federal court will not order discovery of what the defendant told the therapist. Federal Rule
of Civil Procedure 26 provides that discovery may be had of any nonprivileged matter that is
relevant to the subject matter involved in the pending action and proportional to the needs of the
case. Because the defendant’s conversation with the therapist is privileged, it is not within the
scope of discovery. (A) is incorrect. The rules also provide for discovery of inadmissible evidence.
However, this does not trump the doctor-patient privilege. (B) is incorrect because the privilege
would still be violated as to the parties to the litigation. (D) is incorrect because the patient, not
the doctor, must waive the privilege.
Answer to Question 6

(D) The employer does not have to provide a copy of the memo to the worker as part of the employer’s
required initial disclosures. Without waiting for a discovery request, a party must disclose copies
or descriptions of documents that the disclosing party may use to support its claims or defenses.
(C) is therefore incorrect. Because the employer would not be using the memo to support its
defenses, it does not have to provide the memo as part of its initial disclosures. (B) is incorrect.
(A) is incorrect because the employer must only produce documents relevant to its own claims or
defenses.

Answer to Question 7

(A) The defendants can depose the engineer if the city expects to call the engineer as an expert
witness at trial. A party must disclose the identities of expert witnesses expected to be used
at trial, and this disclosure must be accompanied by a report that states the expert’s opinions.
In addition, a party may depose experts who are expected to be called at trial. (B) is incorrect
because a party does not need a court order to depose experts who are expected to testify. (C) is
incorrect because a party can discover the opinions of experts not expected to testify at trial on a
showing of exceptional circumstances under which it is impracticable to obtain facts or opinions
by other means. (D) is incorrect because an expert’s opinion is not considered to be protected
work product.

Answer to Question 8

(C) The store must produce the report it filed with the state agency because the store did not prepare
the report for litigation or trial. Work product protection applies to documents that were made in
anticipation of litigation, and it therefore does not apply to the store’s report. (B) is therefore incor-
rect. (A) is incorrect because discovery may be had of any nonprivileged matter that is relevant to
any party’s claim or defense. (D) is incorrect because it is too broad; privileged matter may not be
subject to discovery even if it is relevant to a party’s claim or defense.

Answer to Question 9

(C) The park may present the deposition into evidence because of the witness’s distance from the
court. A deposition may be used at trial for any purpose if the deponent is at a distance greater
CIVIL PROCEDURE - DISCOVERY ANSWERS 3.

than 100 miles from the place of trial, which is the case here. (A) is incorrect because it describes
only one situation in which deposition testimony may be used as evidence; the deposition testi-
mony may also be used for impeachment when the deponent is testifying live at trial. Conversely,
(B) is incorrect because it states the rule when the deponent is testifying live at trial, but does not
address the testimony’s use when the witness is not at trial. (D) is incorrect because depositions
may only be used as evidence in certain situations, such as when the deponent is unable to testify
or is greater than 100 miles from the place of trial.

Answer to Question 10

(B) The court should not order the driver to answer the question because the question seeks privi-
leged information. In general, discovery may be had of any nonprivileged matter that is relevant
to any party’s claim or defense and that is proportional to the needs of the case. Here, the driver
is being asked about something he may have said to his attorney, which falls under the attorney-
client privilege. (A) is incorrect because information that would be inadmissible at trial may still
be discoverable. (As an aside, the driver’s statement is not considered hearsay under the Federal
Rules of Evidence because it would be a statement made by a party and offered against that party
(traditionally known as an “admission by a party-opponent”).) (C) is incorrect because the court
should not order the driver to answer because the question seeks privileged information, not
because it could be asked in another way. (D) is incorrect because information that is relevant to a
claim or defense is undiscoverable if it is privileged.

Answer to Question 11

(B) The vice president should conduct an investigation to determine what relevant documents the
engine manufacturer has and then answer the interrogatory based on that investigation. A party
who responds to interrogatories must respond with facts she herself knows and with facts that are
available to her. The interrogatory asks for all reports in the engine manufacturer’s possession
concerning accidents, and so the vice president should find all the relevant reports despite their
being in different locations. (A) is incorrect because a party may serve interrogatories on any
other party. (C) is incorrect because only listing the documents in her own files is an insufficient
response as long as the other reports are available to her. (D) is incorrect because the informa-
tion sought is not covered by the initial disclosures requirement of Rule 26(a)(1)(A) if the engine
manufacturer was not planning to use the reports to support its claims or defenses. Also, if the
reports were part of the required initial disclosures, the engine manufacturer should have already
provided the reports to the other parties.

Answer to Question 12

(C) The court should not order the woman to produce the recording unless it finds that the man has
substantial need for the recording and is unable to obtain the same information without undue
hardship. In general, a party’s work product made in anticipation of litigation is not discoverable.
However, the work product may be discovered if the opposing party can show substantial need
and that he cannot obtain the same information in an alternative way without undue hardship. (A)
is incorrect because the burden would be on the man to show that he has a substantial need for the
information and is unable to obtain the same information without undue hardship. (B) is incorrect
because it describes the standard for discovery of evidence that would be inadmissible at trial; the
issue here is work product. (D) is incorrect because the Federal Rules provide for the production
of tangible things.
4. CIVIL PROCEDURE - DISCOVERY ANSWERS

Answer to Question 13

(B) The corporation is not required to answer the interrogatories. The attorney-client evidentiary
privilege is an absolute privilege and protects confidential communications between an attorney
and client made during professional consultation. It only protects communications, not objects or
preexisting documents. The definition of a “client” can include a corporation, or any other organi-
zation or entity, public or private, seeking professional legal services, and will include statements
of any corporate officials or employees made to the attorney if they were authorized or directed
by the corporation to make such statements. Here, the corporation directed its general counsel
to interview the train company’s employees immediately following a train derailment to inves-
tigate what happened. Thus, the employees of the train company in this context would qualify
as “clients” because the interviews were obtained for professional consultation. Although the
communication was recorded into a written document, it was not a preexisting document. There-
fore, the written statements are protected from discovery under the attorney-client privilege. (A)
is tempting, but it is wrong because not all work product is protected. Instead, the work product
privilege protects documents and tangible things prepared by a party or representative of a party
(e.g., a lawyer) made in anticipation of litigation. However, it is generally a qualified privilege and
can be overcome by showing “substantial need” and to avoid “undue hardship” in obtaining
the materials in an alternative way. Although disclosures of mental impressions, conclusions,
opinions, or legal theories of a party’s lawyer are an exception and are absolutely privileged and
not discoverable, the facts state that the attorney merely wrote down what the train company’s
engineer and employees said after interviewing them. The facts do not say the written statements
included the lawyer’s mental impressions, conclusions, opinions, or legal theories. Therefore,
while it is true that the requested information is the work product of the attorney made in antici-
pation of litigation, this alone does not make it protected. Instead, the documents may still be
discoverable if the injured passenger can show substantial need and that it would cause undue
hardship for the passenger to obtain the materials in an alternative way. Note: Be sure to know
the differences between the work product and attorney-client privileges. Since they are similar,
this is an area ripe for testing! (C) is wrong because not all information is discoverable by a party
to the case. Instead, the scope of discovery includes any nonprivileged matter that is relevant to
any party’s claim or defense and proportional to the needs of the case. (D) is wrong because,
while it is true that the information sought is relevant, the attorney-client privilege extends to
protect even relevant evidence. In other words, privileged information falls outside the scope of
discovery.

Answer to Question 14

(D) The homeowner must provide the requested information. A party must respond to interroga-
tories not only with facts which he himself knows but also with facts that are available to him.
Since copies of the house purchase contract and closing documents are in his attic, informa-
tion regarding the house purchase price and sale are available to the homeowner. A party has
the option to produce business records if the answer to an interrogatory may be determined by
examining the party’s business records. [Fed. R. Civ. P. 33(d)] (A) is incorrect because an inter-
rogatory may ask a party about facts that are available to him. (B) is incorrect because a party
may serve interrogatories on another party to obtain information prior to trial. (C) is incorrect
because interrogatories may ask about facts and opinions.

一 信:liuxue119118 , 们 信免 供
一 信:liuxue119118 , 们 信免 供

CIVIL PROCEDURE -
PERSONAL JURISDICTION
AND VENUE QUESTIONS
CIVIL PROCEDURE - PERSONAL JURISDICTION AND VENUE QUESTIONS 1.

CIVIL PROCEDURE - PERSONAL JURISDICTION AND VENUE QUESTIONS

Question 1 Question 2

A plaintiff from State A filed a diversity A franchisee in State A entered into a 20-year
action in the federal court for State B against the franchise contract with a fast food company
defendant from State B, alleging that, 28 months whose offices and operations are all in State B.
before, the defendant’s negligence committed The contract gave the franchisee the right to
in State A caused the plaintiff’s damages.The operate one of the company’s fast food restau-
statute of limitations in State A is two years, rants at a designated location in the franchi-
whereas the statute of limitations in State B see’s hometown in State A. It also authorized
is three years. Each state would apply its own the fast food company to regulate in detail the
statute of limitations under its own choice of franchisee’s operation, including the menu, food
law principles.The defendant timely moved the purchases, food preparation, pricing, adver-
federal court for State B to transfer the case tising and logo display, and more. The contract
to the federal court for State A, alleging that obligated the franchisee to pay the fast food
it would be more convenient to the parties and company a designated percentage of the restau-
witnesses if trial were held in State A. rant’s monthly gross revenue. The contract
required the franchisee to send the payments,
If the court grants the motion, which statute of along with relevant financial data, to the compa-
limitations should the court apply? ny’s offices in State B. After several months
of operation, the franchisee fell behind on her
(A) The statute of limitations for State B, monthly payments to the company. The fast food
because the case was properly filed in the company filed a breach of contract action against
federal court for State B. the franchisee in a court in State B, seeking
payment of the sums due.
(B) The statute of limitations for State B,
because a statute of limitations is substan- Assuming the franchisee’s actions fall within
tive for Erie purposes. State B’s long arm statute, does the court in State
B have personal jurisdiction over the franchisee?
(C) The statute of limitations for State A,
because the accident occurred there. (A) No, because the franchisee lacks sufficient
contacts with State B.
(D) The statute of limitations for State A,
because a federal court is required to apply (B) Yes, but only if the court in State B is a
the statute of limitations of the state in federal court.
which it sits.
(C) Yes, but only if the court in State B is a
state court.

(D) Yes, because of the franchisee’s purposeful,


ongoing relationship with the fast food
company.
2. CIVIL PROCEDURE - PERSONAL JURISDICTION AND VENUE QUESTIONS

Question 3 Question 4

A small corporation manufactures and sells A manufacturer of widgets is incorporated in


widgets in State A and State B. It is a State A State A, although it has no operations, activities,
corporation and it operates three stores from employees, or offices in State A. Its principal
which it sells its widgets—two are in the District place of business and all of its manufacturing
of State A and one is in the District of State B. operations are in State B. The manufacturer sells
The corporation has no presence, sales, or opera- its widgets exclusively through its own stores,
tions anywhere else. A consumer who resides in and it operates stores in only State B and State
the District of State C purchased a widget from C. A consumer who resides in State D visited
the corporation’s State B store. The consumer one of the manufacturer’s State C stores and
was subsequently injured while using the widget purchased a widget. The consumer returned
at his home. The consumer intends to file a to his home to State D, where he was injured
products liability action against the corporation while using the widget. The consumer filed a
in federal district court. products liability action against the manufac-
turer in the United States District Court for State
In what district or districts is venue proper? A. The manufacturer filed a motion to dismiss
the action, or in the alternative to transfer the
(A) The District of State C only. action to the United States District Court for the
District of State C, for improper venue.
(B) The District of State B and the District of
State A only. How should the court rule?
(C) The District of State C, the District of State (A) Deny the motion to dismiss or transfer,
B, and the District of State A. because the District of State A is a proper
venue.
(D) The District of State A only.
(B) Grant the motion to dismiss for improper
venue.

(C) Grant the motion to transfer the action to


the District of State C.

(D) Deny the motion to transfer, because venue


is proper only in the District of State B.
CIVIL PROCEDURE - PERSONAL JURISDICTION AND VENUE QUESTIONS 3.

Question 5 Question 6

A used car dealer in State A is profiled in an A citizen of State A purchased life insurance
article in a newspaper with national circulation. by mail from a State B insurance company.
The article states that the dealer never sells a car The policy was the only one that the company
without rolling the odometer back at least 25,000 had ever sold in State A. The purchaser mailed
miles. In fact, the dealer is an honest business- premiums from State A to State B for five years,
person who never changes odometers, so he sued and then died. The insurance company refused
the newspaper for libel in State A state court. to pay the policy benefits. The purchaser’s
The newspaper’s entire operation is conducted administrator sued the company in State A
from its offices in State B, but it sells 5,000 state court. The state has a long arm statute that
copies in State A on an average day. In its initial grants a state court in personam jurisdiction
pleading, the newspaper argues for dismissal over a defendant who “contract[s] to insure any
based on lack of jurisdiction over the newspaper person, property, or risk located within this State
because of insufficient contacts. at the time of the contracting.” The insurance
company argued that its only contact with State
What is the likely ruling of the court on this A since it began its business was the purchaser’s
issue? insurance policy, and that this single contact
does not meet the minimum required for the
(A) Deny the motion because the newspaper’s exercise of in personam jurisdiction under Inter-
contacts with State A were sufficient so it national Shoe.
should reasonably anticipate being haled
into court in State A. How should the court rule on the minimum
contacts issue?
(B) Deny the motion, unless State A lacks a
“doing business” jurisdictional provision. (A) For the purchaser’s administrator, because
the State A statute authorizes jurisdiction.
(C) Grant the motion, because selling 5,000
copies of a newspaper per day is not signifi- (B) For the purchaser’s administrator, because
cant business. of the close connection between the contact
and the case.
(D) Grant the motion, unless State A has a
long arm statute covering the newspaper’s (C) For the insurance company, because the
conduct. exercise of jurisdiction would not be consti-
tutional.

(D) For the insurance company, because suit


must be brought in State B.
4. CIVIL PROCEDURE - PERSONAL JURISDICTION AND VENUE QUESTIONS

Question 7 Question 8

The plaintiff, a resident of the Northern The plaintiff is a resident of City A, located in
District of State A, sues the defendant, a resident the Northern District of State A. The defendant
of the Southern District of State B, for personal is a resident of City B, located in the Eastern
injuries he suffered when the defendant pushed District of State B. The defendant negligently
him off a ski lift in State C, which has only one ran a red light in City B, resulting in a collision
federal court district. between his car and the delivery van that the
plaintiff was driving. The result: 100 pounds of
In which of the three districts mentioned is cork that the plaintiff was delivering wound up
venue proper? striking him in the back of the head, causing
severe injuries. The plaintiff sued the defendant
(A) The Northern District of State A only. in the Northern District of State A. The defen-
dant moves to dismiss based on improper venue.
(B) The Southern District of State B only.
How should the court rule on the defendant’s
(C) The District of State C only. motion to dismiss?
(D) Either the Southern District of State B or (A) Grant the motion, because the Northern
the District of State C. District of State A is not a place of proper
venue.

(B) Grant the motion, because the court does


not have personal jurisdiction over the
defendant.

(C) Deny the motion, but it should transfer the


case to the Eastern District of State B.

(D) Deny the motion, because the Northern


District of State A is a place of proper
venue.
CIVIL PROCEDURE - PERSONAL JURISDICTION AND VENUE QUESTIONS 5.

Question 9 Question 10

A motorist was involved in a car accident with A town planned to build a new street through
the vice president of a large corporation while a parcel of vacant land on the edge of town.
on vacation in another state. Upon return to his The land records show a deed dated October 5,
home state, the motorist decided to sue the vice 1947, to the current property owner, for whom
president. The vice president has had absolutely an address is given in the capital city of the state.
no personal contacts with the motorist’s home The town instituted an in rem condemnation
state, although the company for which she works action in state court, with notice to all interested
does extensive business in the state. The motorist parties published in the town newspaper. The
read in a newspaper that the vice president was property owner, who still resides at the address
served with a subpoena to testify at a grand jury in the capital city, does not see the notice. The
proceeding in the state regarding some of her court allows the condemnation to proceed as
company’s activities in that state, so he arranged requested by the town.
for process to be served on the vice president
when she attended the hearing. Is the condemnation valid as to the property
owner?
Does the state court have jurisdiction over the
vice president? (A) Yes, because a court has jurisdiction over
the subject matter.
(A) Yes, because she was served with process
while in the state. (B) Yes, because notice by publication was
sufficient.
(B) Yes, because she is doing substantial and
continuous business in the state. (C) No, because the property owner was
required to be served by summons.
(C) No, because she was in the state solely to
testify at the grand jury proceeding, and (D) No, because the property owner should
she lacks minimum contacts with the state. have been mailed notice of the proceeding.

(D) No, because in-state service of process is


never a basis for asserting personal jurisdic-
tion.
一 信:liuxue119118 , 们 信免 供

6. CIVIL PROCEDURE - PERSONAL JURISDICTION AND VENUE QUESTIONS

Question 11 Question 12

The plaintiff, a citizen and resident of the A plaintiff filed a civil action against a defen-
Southern District of State A, filed a negligence dant in federal district court. The defendant filed
action against a citizen and resident of Canada, a motion to dismiss the action on the ground
alleging that the Canadian resident caused the that the court lacked subject matter jurisdic-
accident in which the plaintiff was injured while tion. Following an appropriate hearing, the court
driving in the Western District of State A. The ruled that it had subject matter jurisdiction and
action was filed in the United States District it denied the motion. The defendant then timely
Court for the Southern District of State A. The filed its answer.
defendant filed a motion seeking to dismiss the
action on the grounds of improper venue and May the defendant assert in its answer that the
lack of personal jurisdiction. action should be dismissed for lack of personal
jurisdiction?
How should the court rule?
(A) Yes, because the lack of personal juris-
(A) Dismiss the action, because the Southern diction is a defense that may properly be
District of State A is not a proper venue. raised in the defendant’s answer.

(B) Dismiss the action because the court lacks (B) Yes, because the defendant may raise its
personal jurisdiction over the Canadian objection to personal jurisdiction in its
defendant. answer, but here the court has discretion to
refuse to address the issue.
(C) Exercise its discretion by transferring
the action to a proper venue rather than (C) No, because the defendant failed to enter a
dismissing it, even though the Southern special appearance and to raise its objection
District of State A is not a proper venue. in a separate motion that does not address
the merits of the action.
(D) Deny the motion to dismiss because it has
personal jurisdiction over the Canadian (D) No, because the defendant waived any
defendant, and the Southern District of objection to personal jurisdiction by filing
State A is a proper venue. a pre-answer motion to dismiss without
raising the issue.
CIVIL PROCEDURE - PERSONAL JURISDICTION AND VENUE QUESTIONS 7.

Question 13 Question 14

The plaintiff filed a negligence action against A movie director sued an actor in federal
the defendant in the United States District court in the District of State A, alleging breach
Court for the District of State A after they were of a contract to star in a movie. Both parties are
involved in a car accident in State B. The defen- domiciled in State B, the contract was made
dant filed a motion objecting to venue. The there, and the movie was to be made there.
plaintiff is a citizen of State A. The defendant Nonetheless, the director prefers to sue in federal
owns a home in State C, where he lives most of court in State A where he is working. The actor
the year. The defendant also owns a beach house is also currently working in State A on another
in State A. He spends about a month and a half project and does not object to the federal court in
each year at the beach house, but never more State A hearing the case.
than a week at a time.
May the United States District Court for the
Is venue proper in the District of State A? District of State A hear the case?

(A) No, because the defendant is not domiciled (A) Yes, because the actor consented to the
in the District of State A and the act or court’s jurisdiction.
omission giving rise to the claim did not
occur there. (B) Yes, because courts in State A have
authority to hear claims arising from events
(B) No, because the defendant is not a citizen of in State B as long as the court has personal
State A. jurisdiction over the defendant.

(C) Yes, because the defendant resides in both (C) No, because the court lacks subject matter
the District of State A and the District of jurisdiction.
State C.
(D) No, because the court must transfer the case
(D) Yes, because the plaintiff resides in the to a proper venue in State B.
District of State A.

一 信:liuxue119118 , 们 信免 供
CIVIL PROCEDURE -
PERSONAL JURISDICTION
AND VENUE ANSWERS
CIVIL PROCEDURE - PERSONAL JURISDICTION AND VENUE ANSWERS 1.

CIVIL PROCEDURE - PERSONAL JURISDICTION AND VENUE ANSWERS

Answer to Question 1

(A) Under 28 U.S.C. section 1391, venue is proper in (i) a judicial district in which any defendant
resides, if all defendants reside in the same state in which the district is located; or (ii) a judicial
district in which a substantial part of the events or omissions giving rise to the claim occurred, or
where a substantial part of the property that is the subject of the action is located. (Note that there
is a fallback provision when (i) or (ii) do not apply.) Here, venue was proper either in State B (the
domicile of the sole defendant) or in State A (the place of the accident). Under 28 U.S.C. section
1404(a), when a case is properly filed in federal court, the federal district court may nonetheless
transfer the case, for the convenience of the parties and witnesses, to another district or division
where it might have been brought or to which the parties have consented. When transfer is made
under section 1404(a), the law of the transferring court ordinarily applies. (There is an exception
when transfer is based on a forum selection clause, but that exception is not applicable under these
facts.) Thus, (A) is the correct answer. (B) is incorrect because the fact that a statute of limitations
is substantive for Erie purposes is largely irrelevant. There are two questions that might arise in
an Erie situation: (i) whether federal or state law applies; and (ii) which state law applies. For the
first prong, a statute of limitations is “substantive,” meaning that the federal court must apply a
state statute of limitations. (C) is incorrect because the facts state that a State B court would apply
its own statute of limitations regardless of where the accident occurred. (D) is incorrect because,
although a very superficial correct statement of law that applies when a case is not transferred, the
rules on which state law applies change when a case is filed in a correct venue and is transferred
for convenience.

Answer to Question 2

(D) The court in State B has jurisdiction over the franchisee. By entering the comprehensive, long-
term contract with the fast food company, the franchisee created purposeful contacts with
Arkansas, and the claim asserted arises from those contacts. [See Burger King Corp. v. Rudze-
wicz, 471 U.S. 462 (1985)] As a result, the court has personal jurisdiction over the franchisee,
making (D) a correct answer choice and (A) an incorrect one. (B) and (C) are incorrect for much
of the same reason: a federal court is required to analyze whether it has personal jurisdiction over
a defendant in the same manner as a state court. Thus, a personal jurisdiction analysis is exactly
the same in federal court and in state court. In other words, it is irrelevant whether the court is a
state court or a federal court when it comes to a personal jurisdiction question.

Answer to Question 3

(C) Federal venue in civil actions is proper in (i) the district where any defendant resides, if all defen-
dants are residents of the state in which the district is located; and (ii) the district in which a
substantial part of the events or omissions giving rise to the claim occurred. If there is no district
anywhere in the United States that satisfies (i) or (ii), the action may be brought in a judicial
district in which any defendant is subject to the court’s personal jurisdiction with respect to such
action. The District of State C (place of the injury) and the District of State B (place of purchase)
are proper because substantial events giving rise to the claim occurred there. A corporation is
deemed to reside in any district in which it is subject to personal jurisdiction with respect to
the action in question. The facts state that the manufacturer is a State A corporation and has its
principal place of business in the District of State A. As a result, it is subject to personal jurisdic-
tion there based on a general jurisdiction theory. Thus, the District of State A is a place of proper
2. CIVIL PROCEDURE - PERSONAL JURISDICTION AND VENUE ANSWERS

venue because that is where the manufacturer (the only defendant) resides for purposes of venue.
Accordingly, (C) is the correct answer and (A), (B), and (D) are incorrect. An issue not raised
by the question is whether the District of State C has personal jurisdiction over the defendant
corporation. While that district is a proper venue under the federal venue statute, it may not have
personal jurisdiction over the defendant corporation absent additional facts. If it does not and the
defendant objects to that court’s jurisdiction over it, the court would dismiss for lack of personal
jurisdiction if the consumer had brought the action in the District of State C.

Answer to Question 4

(C) The court should grant the motion to transfer the action to the District of State C. Federal venue in
civil actions is proper in (i) the district where any defendant resides, if all defendants are residents
of the state in which the district is located; and (ii) the district in which a substantial part of the
events or omissions giving rise to the claim occurred. A corporate defendant is deemed to reside
in each district with which it has sufficient contacts to justify personal jurisdiction with respect to
the action. Thus, State A is a proper venue because, by virtue of it being incorporated there, the
manufacturer is subject to general jurisdiction (jurisdiction over all causes of action) there. This
makes (B) and (D) incorrect answer choices. However, federal courts may transfer to another
proper venue for the convenience of parties and witnesses and in the interests of justice. State C
would be more convenient for the parties and witnesses because the defendant’s operations are
there and relevant witnesses may be there, and State C is a proper venue because the defendant
manufacturer purposefully transacted business there (the sale at one of its stores), which gave rise
to the action, thus subjecting the manufacturer to specific jurisdiction in State C. In the instant
case, no witnesses, parties, or evidence are in State A. Thus, on the whole, the case should be
transferred to State C. (A) is not correct because a case may be transferred even though venue is
proper.

Answer to Question 5

(A) The court is likely to rule that the newspaper is subject to personal jurisdiction for a libel action
in State A because it has sufficient minimum contacts with the state. The publisher should reason-
ably anticipate causing injury wherever the newspaper is sold, and so should reasonably anticipate
being haled into court in each state. [Keeton v. Hustler Magazine, 465 U.S. 770 (1984); Calder v.
Jones, 465 U.S. 783 (1984)] (B) and (D) are incorrect. The exercise of jurisdiction over a partic-
ular defendant must be both authorized by statute and constitutional. These are separate require-
ments. The newspaper’s claim of insufficient contacts goes to the constitutionality of the exercise
of jurisdiction. Whether State A has a “doing business” statute or a long arm statute is irrelevant
to this determination. (C) is factually incorrect; selling 5,000 newspapers per day would probably
be considered sufficient contact, especially since the cause of action arose from that conduct.

Answer to Question 6

(B) The court should rule for the purchaser’s administrator on the minimum contacts issue and
exercise in personam jurisdiction over the insurance company. Generally, fewer contacts will
suffice for in personam jurisdiction if those contacts are directly related to the cause of action.
Here, even though the sale of the insurance policy constituted only a single contact, it is the
contact at issue in the case. Thus, the exercise of jurisdiction would be constitutional. (A) is incor-
rect because the exercise of jurisdiction, in addition to being authorized by statute, must also be
constitutional. (C) is incorrect because a single contact is sufficient if the case is based on that
CIVIL PROCEDURE - PERSONAL JURISDICTION AND VENUE ANSWERS 3.

contact, as discussed above. (This is called “specific jurisdiction.”) (D) is incorrect, because a suit
in State A would be permissible if it is authorized by statute and constitutional, which it is here.

Answer to Question 7

(D) Venue is proper in the district in which the defendant resides (here, the Southern District of State
B) or in the district in which the events giving rise to the claim occurred (here, the District of
State C). (A) is incorrect because the district in which plaintiff resides is not, without more, a
basis for venue in federal court. (B) is incorrect because venue is also proper in the district in
which the events occurred. (C) is incorrect because venue is also proper in the district in which
the defendant resides.

Answer to Question 8

(C) The court should deny the motion to dismiss and transfer the case. Venue in this case (a diversity
case) would be proper in a judicial district in which (i) any defendant resides, if all reside in the
same state or (ii) a substantial part of the events or omissions giving rise to the claim occurred.
Thus, venue would be proper in the Eastern District of State B because that venue is both the
defendant’s residence and the place where a substantial part of the events took place. The plain-
tiff’s choice of venue in the Northern District of State A must be corrected. To correct improper
venue, a court will usually transfer the case to a place of proper venue, not dismiss the case, with
the standard being whether transfer would be in the interests of justice. There are no facts here to
indicate that an injustice would be worked by transferring the case; as a result, the case should be
transferred rather than dismissed. Thus, (A) is incorrect. (B) is incorrect because an objection to
personal jurisdiction may be waived by failing to timely raise the issue. Here, the defendant has
not raised the issue, thus waiving it. (D) is incorrect, because the Northern District of State A is
not a place of proper venue.

Answer to Question 9

(C) States have limited power to assert in personam jurisdiction over defendants who are in the state
as parties or witnesses in another judicial proceeding. In effect, a civil defendant is given a “free
pass” into the state while she is testifying in a different proceeding. Thus, because the vice presi-
dent is essentially immunized from service of process while in the motorist’s state while she is
testifying in another proceeding, in-state service of process cannot form the basis for personal
jurisdiction over her. As a result, (A) is incorrect. Furthermore, it is her company that is “doing
business” in the state, not herself. The facts state that she has no contacts with the state. Thus,
(B) is incorrect. (D) is incorrect because in-state service of process may form the basis for a state
court to assert personal jurisdiction over an out-of-state defendant.

Answer to Question 10

(D) The condemnation is not valid because the property owner should have been mailed notice of
the proceeding. In an in rem action, persons whose interests are known to be affected and whose
addresses are known must be notified at least by ordinary mail. The town should at least have
sent notice to the property owner at the address it had for him, even if it seemed unlikely that he
would still be there. (B) is thus incorrect. (A) is incorrect because, although the statement is true,
it does not address the notice requirement. (C) is incorrect. It is well established that service by
mail meets the due process requirement of notice in certain cases (i.e., a summons is not always
required).
4. CIVIL PROCEDURE - PERSONAL JURISDICTION AND VENUE ANSWERS

Answer to Question 11

(D) The court should deny the motion to dismiss. All courts in State A (state and federal) have
personal jurisdiction over the defendant because the action arises from his purposeful activities
in State A. This makes (B) an incorrect answer choice. Venue is proper because the federal venue
statute provides that a nonresident of the United States may be sued in any district. Thus, (D) is
correct and (A) is incorrect. Although it is true that a court will ordinarily transfer a case in the
“interests of justice,” (C) is not correct because the Southern District of State A is a proper venue,
as explained above.

Answer to Question 12

(D) The defendant may not assert lack of personal jurisdiction in its answer. A defendant waives any
objection to personal jurisdiction if that defendant files a pre-answer motion to dismiss and does
not raise personal jurisdiction in that motion. A defendant must assert this defense in its first
response to the complaint—whether that response is a pre-answer motion or the answer. (A) is
incorrect. The defendant may assert the lack of personal jurisdiction in its answer only if that is
the first pleading filed with the court. (B) is incorrect because, here, the defendant has waived the
issue by not raising the matter in its first pleading or motion with the court. Since the issue has
been waived, the court does not have the discretion to hear the motion. (C) is incorrect. Many
states allow special appearances where the defendant can object to jurisdiction. Usually the defen-
dant must make the special appearance in its initial pleading to the court; otherwise, the defen-
dant will be deemed to have consented to jurisdiction. However, the Federal Rules require that a
defendant object to personal jurisdiction in its first response to the complaint, either a pre-answer
motion or the answer.

Answer to Question 13

(A) Venue is not proper in the District of State A. Federal venue in civil actions is proper in (i) the
district where any defendant resides, if all defendants are residents of the state in which the
district is located; and (ii) the district in which a substantial part of the events or omissions giving
rise to the claim occurred. If there is no district anywhere in the United States that satisfies (i)
or (ii), the action may be brought in a judicial district in which any defendant is subject to the
court’s personal jurisdiction with respect to such action. Here, the events giving rise to the action
occurred in State B. A person is deemed to “reside” where her domicile is located, i.e., at her true,
fixed, permanent home. Under these facts, the defendant resides in the District of State C—not
State A. Thus, (A) is the correct answer, and (D) is incorrect. (B) is incorrect because the answer
choice ignores the possibility that venue could be fixed based on where a substantial part of the
events occurred. (C) is incorrect because an individual’s domicile (i.e., her permanent home)
determines where she resides for venue purposes.

Answer to Question 14

(C) The court may not hear the case. This question highlights the difference between venue and
subject matter jurisdiction. Subject matter jurisdiction cannot be conferred on the court by a
failure of the parties to raise the issue. In fact, if the court determines on its own that it lacks
subject matter jurisdiction, it must dismiss the action even if the parties have not raised the issue.
Venue, on the other hand, may be waived by the parties. In the instant case, federal subject matter
jurisdiction is lacking because the claim does not arise under federal law and because the claim
is between two citizens of the same state (State B). As a result, the court may not hear the claim.
(A) is not correct because, as explained above, the parties may not confer subject matter jurisdic-
CIVIL PROCEDURE - PERSONAL JURISDICTION AND VENUE ANSWERS 5.

tion on the courts. The lack of personal jurisdiction, on the other hand, is a defense that may be
waived by failure to timely raise it. Here, it is the lack of subject matter jurisdiction that prevents
the court from hearing the case. (B) is not correct because, in addition to having personal jurisdic-
tion over the defendants, a court must have a valid basis for subject matter jurisdiction over the
case. Subject matter jurisdiction is lacking here. (D) is not correct because, as explained above, a
defect in venue may be waived.
CIVIL PROCEDURE -
POST-TRIAL MATTERS
QUESTIONS
CIVIL PROCEDURE - POST-TRIAL MATTERS QUESTIONS 1.

CIVIL PROCEDURE - POST-TRIAL MATTERS QUESTIONS

Question 1 Question 2

An employee sued her employer in federal A man owned a yacht and a sailboat. He
court alleging that she was discriminated against obtained two insurance policies, one for the
on the basis of her gender in violation of federal yacht and the other for the sailboat, insuring
law. At the close of the employee’s evidence, the each against loss or destruction. Thereafter, the
employer moved for judgment as a matter of law, man informed the insurer that his yacht sank
contending that the employee failed to submit and demanded payment from the insurer. When
adequate evidence to prove her case. At the close the insurer refused to pay, the man commenced
of all evidence, neither party made any motions, a proper diversity action against it for breach of
and the case was submitted to the jury. The jury contract. The man did not request a jury.In its
returned a verdict for the employee, finding that answer, the insurer denied liability, alleging that
she was discriminated against on the basis of the man obtained the policy with the fraudulent
gender. Twenty-five days after the judgment was intent to sink the yacht and collect the proceeds.
filed, the employer filed a renewed motion for In addition, the insurer asserted a counter-
judgment as a matter of law. claim seeking rescission of the sailboat policy,
asserting that it too was procured with intent
Should the court grant the motion? to defraud. After discovery, the insurer moved
for summary judgment dismissing the man’s
(A) No, because the employer did not move for claim. The court granted the motion. Thereafter,
judgment as a matter of law at the close of a bench trial was conducted on the insurer’s
all the evidence. counterclaim for rescission, and the court
granted the insurer’s counterclaim, finding that
(B) No, because the motion is untimely. the sailboat contract was procured by fraud. The
man appealed both of the court’s rulings.
(C) Yes, if the court finds a reasonable jury
would not have a legally sufficient basis to What is the standard of review on appeal for
find for the employee. the grant of summary judgment and the findings
of fact at the bench trial?
(D) Yes, if the court finds that the verdict is
against the weight of the evidence. (A) De novo for both.

(B) Clearly erroneous for both.

(C) De novo for the grant of summary


judgment and clearly erroneous for the
findings of fact at the bench trial.

(D) Clearly erroneous for the grant of summary


judgment and de novo for the findings of
fact at the bench trial.
2. CIVIL PROCEDURE - POST-TRIAL MATTERS QUESTIONS

Question 3 Question 4

A local newspaper published a story in which A plaintiff filed a proper diversity suit in
it reported that a pet trainer had physically federal district court against a defendant for
abused dogs in his care by beating them during battery stemming from an altercation the two
training sessions.The man properly commenced had in a bar. The two parties were former
a diversity action in federal court, alleging that business partners, and the defendant filed a
the story was defamatory. During discovery, counterclaim against the plaintiff for breach
the newspaper requested, among other things, of contract relating to their former business
copies of all videos of the trainer’s dog training dealings. The plaintiff filed a motion for
sessions. Although the trainer produced some summary judgment in the battery suit, and the
videos in response to the discovery request, court granted it. The court’s order simply stated:
he intentionally withheld all videos depicting “Plaintiff’s motion for summary judgment on the
incidents of animal abuse. At trial, the trainer battery claim is granted.” The defendant appeals
testified that he had never beaten or otherwise the grant of summary judgment.
abused dogs in his care. Although several dog
owners testified that they believed the trainer had Can the court of appeals hear the defendant’s
physically abused their dogs, the jury rendered a appeal?
verdict in favor of the trainer, awarding him both
compensatory and punitive damages. Six months (A) Yes, because the summary judgment ruling
after the judgment was entered, the newspaper was a final judgment on the merits.
obtained copies of the videos that the trainer
failed to produce. The videos showed that the (B) Yes, because the battery and contract
trainer repeatedly beat dogs in his care during claims do not share any common facts.
training.
(C) No, because a court’s ruling on a summary
What is the newspaper’s best course of action judgment motion is not a final judgment on
to avoid paying the judgment? the merits.

(A) Move for a new trial. (D) No, because there has not been a final
judgment on the merits since the contract
(B) File a notice of appeal. claim has not been adjudicated.

(C) Move for relief from the judgment based on


fraud, misrepresentation, or misconduct of
opposing party.

(D) Commence an independent action in equity


to set aside the judgment.
CIVIL PROCEDURE - POST-TRIAL MATTERS QUESTIONS 3.

Question 5 Question 6

A motorcyclist, a car driver, and a truck An airplane passenger sued an airplane


driver were involved in a three-vehicle accident manufacturer in federal court after the passenger
in a busy intersection. The motorcyclist filed was injured in a plane crash. The passenger
a negligence action against the car driver for alleged that a defective part made by the
personal injuries suffered in the accident. In the manufacturer had caused the crash. The case
car driver's pleadings, she denied that she was went to trial, and the jury found in favor of the
negligent and raised the motorcyclist's contribu- passenger. Judgment was entered on March 1.
tory negligence as a defense. The jury returned Six months later, an airplane parts subcontractor
a general verdict in favor of the car driver. The announced a recall of an engine part that had
motorcyclist then filed a negligence action been found to be defective and was the likely
against the truck driver. At the appropriate cause of two previous plane crashes. Until this
time, the truck driver filed a motion to dismiss announcement there had been no indication to
for failure to state a claim, asserting that the the public or aviation industry that this part was
motorcyclist was precluded from re-litigating his defective, because the subcontractor had been
contributory negligence. very secretive about its in-house investigation.
This engine part had also been in the passen-
Is the truck driver likely to be successful? ger’s plane. In light of this new evidence, the
manufacturer files a motion for relief from a
(A) Yes, because there was a final judgment on final judgment.
the merits in the first case.
Is the court likely to grant the manufacturer’s
(B) Yes, because the jury in the first case deter- motion?
mined that the motorcyclist was contribu-
torily negligent. (A) Yes, because there is newly discovered
evidence that by due diligence could not
(C) No, because it is not clear whether the jury have been discovered in time to move for a
in the first case found that the motorcyclist new trial.
was negligent.
(B) Yes, because the original judgment is now
(D) No, because only someone who was a party void.
in the first action can assert issue preclusion
in the second. (C) No, because motions for relief from a final
judgment must be brought within 28 days
of the entry of judgment.

(D) No, because the manufacturer should have


filed a motion to amend a final order.
4. CIVIL PROCEDURE - POST-TRIAL MATTERS QUESTIONS

Question 7 Question 8

After closing arguments were presented in a A plaintiff sued a defendant in federal court
federal civil trial, the judge gave all but one of for personal injuries resulting from a serious car
the approved jury instructions that the plaintiff accident. The plaintiff alleged that the defendant
had requested. The plaintiff did not object to had been intoxicated on multiple illegal drugs at
the rejection of the proposed instruction. After the time of the accident and had been speeding
deliberating, the jurors returned a verdict for the wrong way down a one-way street. The jury
the defense. The plaintiff filed a timely appeal, was so outraged by the defendant’s behavior
citing as error that the judge’s failure to give that they awarded the plaintiff $50 million in
the requested jury instruction caused the jury to compensatory damages. The trial judge believes
render its verdict in favor of the defense. that this award is so excessive as to shock the
conscience and that the plaintiff should be
Is the appellate court likely to hear the plain- awarded $5 million instead.
tiff’s appeal?
Can the trial judge order a new trial?
(A) Yes, because the judge’s failure to give
any requested instruction must be consid- (A) Yes, because a trial judge may order a
ered plain error if the judge turns out to be new trial when she does not agree with the
incorrect. jury’s damage award.

(B) No, because the plaintiff waived his right to (B) Yes, because a trial judge may order a
appeal by failing to object when the judge new trial when she believes that the jury’s
did not give the requested instruction. compensatory damages award is so exces-
sive as to shock the conscience.
(C) No, because the judge’s failure to give the
requested instruction was harmless error. (C) No, because a trial judge may not order a
new trial when the verdict is excessive or
(D) No, because the jury verdict was rendered, inadequate.
which makes the jury instruction issue
moot. (D) No, because the trial judge should lower the
award given by the jury and enter judgment
on that amount.
CIVIL PROCEDURE - POST-TRIAL MATTERS QUESTIONS 5.

Question 9 Question 10

A homeowner sued a contractor in federal The plaintiff sued the defendant in federal
court for damages resulting from an unsuc- court to recover damages after the defendant
cessful roof repair. The court dismissed the crashed his car into the plaintiff’s garage.
case for lack of personal jurisdiction over the Judgment was entered for the plaintiff in a valid,
contractor. The homeowner then filed her final judgment on the merits. The plaintiff had
complaint in a federal court that could properly originally thought that the damage done to his
assert jurisdiction over the contractor. In his yard was minimal, but he later discovered that
answer the contractor asserts the affirmative the yard’s elaborate landscaping would have
defense of claim preclusion. to be replaced at a high cost. He then files suit
against the defendant to recover damages for the
Should the court dismiss the case? property damage to his yard.

(A) Yes, because the first case involved the Is the plaintiff’s claim likely to be successful?
same claimant against the same defendant.
(A) Yes, because the two lawsuits involve dif-
(B) Yes, because there was a valid, final ferent causes of action.
judgment in the first case.
(B) Yes, because the plaintiff did not realize the
(C) No, because dismissal was based on lack of extent of the damage to his yard until after
personal jurisdiction. the conclusion of the first case.

(D) No, because the defense of claim preclusion (C) No, because the suit is barred by issue
should be raised at trial. preclusion.

(D) No, because the plaintiff is barred by


claim preclusion from bringing the second
lawsuit.
6. CIVIL PROCEDURE - POST-TRIAL MATTERS QUESTIONS

Question 11 Question 12

At a federal trial for negligence, the jury A manufacturer filed a diversity action in
returned a verdict in favor of the defendant. federal court for breach of contract against a
Shortly after the trial, one of the jurors told supplier. The case proceeded to trial, and the
the plaintiff that another juror researched a supplier’s sole defense was that it did not breach
confusing issue on the Internet during a recess in the contract. The jury returned a verdict for the
the trial and reported the results, which favored supplier, and a valid final judgment was entered.
the defendant, to the rest of the jury. Thereafter,
the plaintiff timely and properly filed a motion Thereafter, the supplier sued the manufac-
for a new trial based on jury misconduct. turer in state court for fraudulent misrepresenta-
tion for statements made by the manufacturer
Is the court likely to grant this motion for a during negotiations of the same contract that
new trial? was the subject of the initial federal lawsuit.
The manufacturer filed a motion to dismiss the
(A) Yes, because a juror conducting indepen- subsequent state claim, asserting claim preclu-
dent research during the trial on a matter sion.
being litigated amounts to jury misconduct,
which is a proper ground for granting a How should the state court rule?
new trial.
(A) The court should allow the subsequent
(B) Yes, because a juror communicating with state claim, because the supplier was the
the parties or their counsel after the case defendant in the first federal lawsuit, not
amounts to jury misconduct, which is a the plaintiff, and therefore is not precluded
proper ground for granting a new trial. from filing the claim.

(C) No, because, while jury misconduct is a (B) The court should allow the subsequent
proper ground for granting a new trial, state claim, because the issue of fraudulent
independent research by a juror during the misrepresentation was never litigated in the
trial does not amount to jury misconduct. initial federal lawsuit.

(D) No, because, while a juror conducting (C) The court should dismiss the subsequent
independent research during the trial on state claim because it was a compulsory
a matter being litigated amounts to jury counterclaim.
misconduct, the plaintiff failed to make a
timely objection at trial. (D) The court should dismiss the subsequent
state claim, because a final judgment on
the merits was entered on this breach of
contract action, which merged the suppli-
er’s state claim with the former federal
judgment under issue preclusion.
CIVIL PROCEDURE - POST-TRIAL MATTERS QUESTIONS 7.

Question 13 Question 14

A tourist from a foreign country rented a car A liquor store owner sued a woman for negli-
from a car rental company that is incorporated gence in federal court after she drove her car
and has its principal place of business in State through the front of his store. At trial, the store
A. The tourist then was in a traffic accident with owner did not present any evidence on the issue
another driver, a citizen of State B. The State of causation. No motions were filed during the
B driver filed an action in federal district court trial, and the jury returned a general verdict for
against both the tourist and the rental company, the plaintiff. Immediately after the verdict was
alleging that both were negligent and liable for read, the defendant filed a motion for judgment
the injuries the State B driver sustained in the as a matter of law and a motion for a new trial.
accident. The rental company believes that both
the tourist and the State B driver were negligent The court may:
and wants to recover from both for the extensive
damage done to its car. (A) Deny the motion for a new trial, but grant
the motion for judgment as a matter of law.
Are the rental company’s claims against the
State B driver and the foreign tourist compul- (B) Deny both the motion for a new trial and
sory such that it must assert them in the State B the motion for judgment as a matter of law.
driver’s pending federal action?
(C) Grant either the motion for judgment as a
(A) Both claims are compulsory. matter of law or the motion for a new trial,
but not both.
(B) The claim against the State B driver is
compulsory, but the claim against the (D) Grant the motion for a new trial, but not the
foreign tourist is not. motion for judgment as a matter of law.

(C) The claim against the foreign tourist is


compulsory, but the claim against the State
B driver is not.

(D) Neither claim is compulsory.


CIVIL PROCEDURE -
POST-TRIAL MATTERS
ANSWERS
一 信:liuxue119118 , 们 信免 供

CIVIL PROCEDURE - POST-TRIAL MATTERS ANSWERS 1.

CIVIL PROCEDURE - POST-TRIAL MATTERS ANSWERS

Answer to Question 1

(C) The court should grant the employer’s motion if it believes a jury would not have had a legally
sufficient basis to find for the jury. If a defendant moves for judgment as a matter of law under
Rule 50(a) at any point during the trial, it may renew that motion after the jury reaches a verdict,
regardless of whether it had moved for judgment as a matter of law at the close of all the evidence.
The motion to renew must be made within 28 days after the jury returns its verdict. (A) is there-
fore incorrect. (B) is incorrect because the motion was made within the 28-day period. (D) is
incorrect because it gives the wrong standard. A court may only grant a motion for judgment as
a matter of law if there is no legally sufficient evidentiary basis for it. In deciding the motion, the
trial judge is not free to weigh the evidence. Rather, she must view it in the light most favorable to
the verdict winner.

Answer to Question 2

(C) When an appellate court reviews a trial court’s grant of a motion for summary judgment, it
employs a de novo standard of review. It uses this standard because the denial or grant of a
summary judgment motion is an issue of law. In reviewing a trial court’s decision, the appel-
late court reviews the same paper record as the trial court and determines for itself whether the
movant is entitled to judgment as a matter of law. In contrast, findings of fact in bench trials,
whether based on oral or other evidence, may not be set aside unless clearly erroneous, and the
appellate court must give due regard to the trial court’s opportunity to judge the witnesses’ credi-
bility.

Answer to Question 3

(C) The newspaper’s best course of action would be to move for relief from the judgment on the
grounds that the dog trainer’s intentional failure to produce the requested videos amounts to
fraud, misrepresentation, or misconduct. Under the Federal Rules, after a judgment has been
rendered, a party may seek relief from it directly from the trial court itself. One of the grounds
for such a motion is that the opposing party obtained the judgment by fraud, misrepresentation, or
misconduct. The motion on this ground must be made within a “reasonable time” after entry of
the judgment but in no event more than a year after entry of the judgment. Here, the dog owner’s
intentional failure to produce the video constitutes fraud, misrepresentation, and misconduct
on his part, and the judgment was rendered within a year. It therefore is a viable option for the
newspaper and, as discussed below, the best one. (D) is incorrect because it offers a more diffi-
cult and therefore less efficient course of action than simply making a motion to the trial court
itself. The federal rules preserve the right of a party to commence an independent action for relief
from a judgment. However, a party’s right to commence such an action is strictly limited. Specifi-
cally, a party commencing the independent action must show that (i) the judgment against the
party should not, in equity and good conscience, be enforced; (ii) the party has a good defense to
the claim which gave rise to the judgment; (iii) the opposing party’s fraud, accident, or mistake
prevented the party from obtaining the benefit of its defense; (iv) the party’s fault or negligence
did not contribute to the judgment; and (v) the party does not have an adequate remedy at law.
Thus, while this course of action is possible, the newspaper’s best option is to bring the matter
to the attention of the trial court by making a motion in the case itself. (A) is incorrect because a
motion for a new trial must be filed within 28 days of entry of judgment, and the newspaper first
became aware of the photos six months after the judgment was entered. (B) is incorrect because,
2. CIVIL PROCEDURE - POST-TRIAL MATTERS ANSWERS

subject to exceptions that are not relevant to the question, a notice of appeal generally must be
filed within 30 days after entry of judgment.

Answer to Question 4

(D) The court of appeals cannot hear the defendant’s appeal because there has not been a final
judgment on the merits in this case. With the exception of some interlocutory orders, only
final orders are reviewable on appeal. A final order on the merits disposes of the whole case
by rendering judgment as to all the parties and all causes of action. When multiple claims or
parties are involved in an action, the court may enter a “final judgment” as to fewer than all of
the claims or parties only on an express determination that there is no just reason for delay. If the
court does not make such an express determination, a judgment as to fewer than all the claims
or parties is not a final judgment and is not appealable. Here, the court has not made an express
determination that there is no just reason for delay. Therefore, the summary judgment ruling is
not appealable until there is also a final judgment in the contract action. (A) and (C) are incor-
rect because a summary judgment ruling can be a final judgment on the merits, but there has not
been a final judgment here because of the outstanding contract dispute. (B) is incorrect because
whether the claims share any common facts is irrelevant to whether a judgment on one claim can
be appealed.

Answer to Question 5

(C) The truck driver is unlikely to be successful because it is unclear whether the jury in the first case
found that the motorcyclist was contributorily negligent. For issue preclusion to apply to an issue,
the issue must actually have been litigated and determined in the previous case. Here, the issue
of the motorcyclist’s negligence was not determined in the first case. The jury could have found
that the motorcyclist was contributorily negligent, or it simply could have found that the motorcy-
clist did not prove that the car driver was negligent. Because this issue was not decided, the truck
driver cannot assert issue preclusion (collateral estoppel) against the motorcyclist. (B) is therefore
incorrect. (A) is incorrect because issue preclusion requires more than a final judgment on the
merits in the first case. Issue preclusion applies to issues that were actually litigated and essen-
tial to the judgment in the first case and are relevant in the second case. (D) is incorrect because
nonparties may be allowed to assert issue preclusion and use a prior judgment to avoid liability in
a second suit.

Answer to Question 6

(A) The court is likely to grant the manufacturer’s motion. A court may relieve a party from a final
judgment or order on the ground of newly discovered evidence that by due diligence could not
have been discovered in time to move for a new trial. A motion for a new trial must be filed no
later than 28 days after judgment is entered. Here, the defendant discovered the new evidence
about the recalled engine part six months after the judgment was entered; therefore, it was too late
for a motion for new trial. (B) is incorrect because the original judgment is not void. A judgment
is void only if there is a fundamental flaw, such as lack of jurisdiction or due process. (C) is incor-
rect because motions for relief from a final judgment on the basis of newly discovered evidence
must be brought within one year. (D) is incorrect because, although a party may file a motion to
amend a final order on the basis of newly discovered evidence, it must be filed within 28 days
of the order. Because six months have passed since the judgment was entered, this motion is not
available to the manufacturer.
CIVIL PROCEDURE - POST-TRIAL MATTERS ANSWERS 3.

Answer to Question 7

(B) The appellate court will likely refuse to hear the appeal. Generally speaking, to preserve an error
at trial for appeal, a party must timely object at trial. In order to reserve its right to appeal an error
in an instruction given or a failure to give an instruction, a party must object on the record before
the instructions are given. If adequately preserved, the instructions are reviewed using an abuse
of discretion standard. However, if the objection is not made (and therefore not preserved for full
appellate review), the court’s review is limited to considering whether there was a plain error in
the instruction that affected substantial rights. Relief based on plain error is difficult to obtain.
(A) is wrong because the plain error doctrine does not always apply to the failure to give a jury
instruction, and note that the right affected must be a substantial one. Thus, the point of law in (A)
is stated incorrectly. (C) is wrong because although harmless error (i.e., an error that did not affect
the substantial rights of one of the parties) may not be the basis for reversal on appeal, in this
case the plaintiff did not preserve the issue for appeal because plaintiff’s counsel failed to state
a timely objection. (D) is wrong because it is a misstatement of law. Rendering a jury verdict by
itself does not make a procedural error moot for purposes of appeal.

Answer to Question 8

(B) The trial judge can order a new trial if she believes that the jury’s compensatory damages award
is so excessive as to shock the conscience, which is the situation here. (A) is incorrect because a
trial judge cannot order a new trial merely for disagreeing with the jury’s award; she must believe
it is so excessive as to shock the conscience. (C) is incorrect because a trial judge may order a new
trial when a verdict is excessive or inadequate. (D) is incorrect because it misstates the concept
of remittitur. A trial judge cannot lower a jury award and force the plaintiff to accept it. Rather,
using remittitur, the judge may offer the plaintiff the choice of a lower award or a new trial, but
she cannot simply lower the award.

Answer to Question 9

(C) The court should not dismiss the case because the dismissal based on lack of personal jurisdiction
is not a judgment on the merits. For claim preclusion to apply: (i) the earlier judgment must be a
valid, final judgment on the merits; (ii) the cases must be brought by the same claimant against
the same defendant; (iii) and the same cause of action must be involved in the later suit. Certain
involuntary dismissals, such as those based on lack of jurisdiction or improper venue, are not
judgments on the merits and do not have claim preclusive effect. Therefore, the case should not be
dismissed. (B) is therefore incorrect. (A) is incorrect because claim preclusion requires more than
just the same claimant bringing a case against the same defendant. (D) is incorrect because claim
preclusion is an affirmative defense that should be included in the defendant’s answer.

Answer to Question 10

(D) The plaintiff’s claim is not likely to be successful because it is barred by claim preclusion
principles. Claim preclusion requires that: (i) a valid, final judgment on the merits was entered in
the first case; (ii) the cases were brought by the same claimant against the same defendant; and
(iii) the same cause of action is involved in the later lawsuit. Generally, a claimant is required
to assert all causes of action arising out of the same transaction or occurrence that is the subject
matter of the claim. Here, the first case was resolved by a valid, final judgment on the merits.
Both cases involve the same claimant, same defendant, and claims arising from the same trans-
action or occurrence. The plaintiff should have sought damages for his yard in the first lawsuit

一 信:liuxue119118 , 们 信免 供
4. CIVIL PROCEDURE - POST-TRIAL MATTERS ANSWERS

because the second lawsuit is barred by claim preclusion. (A) is incorrect because the lawsuits
involve the same cause of action, as discussed above. (B) is incorrect because it is irrelevant
when the plaintiff realized the extent of the damage to his yard. (C) is incorrect because issue
preclusion involves binding a party in a different cause of action as to certain issues that were
litigated in the first action. Since the same cause of action is involved here, claim preclusion is
appropriate.

Answer to Question 11

(A) The court is likely to grant the motion for a new trial. Juror misconduct is a proper ground for
granting a new trial, and jurors are not allowed to engage in independent experiments or research
outside of the jury room. Therefore, the juror’s Internet research amounted to jury misconduct
for which the court may grant a new trial. (B) is wrong because it is a misstatement of the law. It
is true that jurors are not permitted to have any communications with the parties or their counsel
during jury deliberations; however, this discussion took place after the case was over. Although
many jurisdictions have specific rules about when, where, and how interested attorneys may talk
to jurors after trial, any violation of the rules likely would result in sanctions rather than a new
trial. (C) is wrong, because as previously stated, independent research by the juror during the
trial does amount to jury misconduct. (D) is wrong because, although typically a motion for a
new trial will not be granted on grounds not called to the attention of the court during the trial
itself, an exception occurs when failing to grant a new trial would result in a gross injustice.
Here, the plaintiff could not object at trial because she was not aware of the jury misconduct
until after the trial was over. Therefore, gross injustice would occur if the judge did not waive
this requirement.

Answer to Question 12

(C) The court should dismiss the subsequent state claim. If a counterclaim arises out of the same
transaction or occurrence as one of the plaintiff’s claims, it is a compulsory counterclaim and
must be pleaded or it will be barred. Here, the supplier’s claim that the manufacturer committed
fraudulent misrepresentation during contract negotiations arises out of the same transaction
or occurrence as the manufacturer’s initial claim, the breach of contract action. Therefore, the
supplier should have asserted the fraudulent misrepresentation counterclaim in the initial case.
Because the supplier did not, the claim is now barred. Claim preclusion (res judicata) does not
apply to this situation because the earlier case and latter case were not brought by the same
claimant against the same defendant. The earlier case was brought by the manufacturer against
the supplier, but the subsequent case was brought by the supplier against the manufacturer. (A)
is incorrect because it states that the subsequent state claim should be allowed. It is correct that
claim preclusion cannot be asserted when the same claimant is not suing the same defendant, but
the state claim should still be barred as an unasserted compulsory counterclaim. (B) and (D) are
wrong because they pertain to the rules of issue preclusion (collateral estoppel). Claim preclusion
focuses on something relatively large—the scope of a “cause of action.” If it applies, the result is
usually to bar the claimant from asserting a second case. Issue preclusion, in contrast, focuses on
something relatively narrow—an issue that was litigated and determined in the first case and that
is relevant in a second case. Under issue preclusion a judgment binds the plaintiff or defendant (or
their privies) in subsequent actions on different causes of action between them (or their privies) as
to issues actually litigated and essential to the judgment in the first action. This conclusive effect
of the first judgment is called issue preclusion (or collateral estoppel). Here, fraudulent misrepre-
sentation was not actually litigated in the first case, so issue preclusion does not apply.
CIVIL PROCEDURE - POST-TRIAL MATTERS ANSWERS 5.

Answer to Question 13

(B) The rental company’s claim against the State B driver is compulsory, but the claim against the
tourist is not. Counterclaims are compulsory if they arise from the same transaction or occurrence
as the claim asserted against the pleading party. Since the rental company’s claim arises from
the same transaction or occurrence as the State B driver’s claim against it, the counterclaim is
compulsory. Therefore (C) and (D) are incorrect. The rental company’s claim against the tourist is
a cross-claim, and cross-claims are not compulsory. Therefore (A) is incorrect.

Answer to Question 14

(D) A motion for a new trial may be granted because of an error during the trial (usually going to the
admissibility of evidence or the propriety of the jury instructions), because the verdict is against
the weight of the evidence (limited to cases where the judge finds the verdict seriously erroneous),
because of jury misconduct, or because the verdict is excessive or inadequate. A motion for
judgment as a matter of law (formerly known as a motion for directed verdict) may be made by
any party any time before submission of the case to the jury. Here, the court may grant the motion
for a new trial under these circumstances because there was absolutely no evidence presented on
the issue of causation, which is a fundamental element of a negligence claim that must be proven.
As such, the court may find that the verdict is seriously erroneous and against the weight of the
evidence and grant the motion for a new trial. However, the court cannot grant the motion for
judgment as a matter of law because it was filed after the case was submitted to the jury, so it was
too late for such a motion. For these reasons (A), (B) and (C) are all wrong because they either
partially or completely reach the wrong conclusion.
CIVIL PROCEDURE -
PRETRIAL MATTERS
QUESTIONS
CIVIL PROCEDURE - PRETRIAL MATTERS QUESTIONS 1.

CIVIL PROCEDURE - PRETRIAL MATTERS QUESTIONS

Question 1 Question 2

A State A plaintiff filed a breach of contract A plaintiff filed a claim against a defen-
action against a State B defendant in federal dant in the United States District Court for the
district court, seeking $100,000 in compensa- District of State A. The plaintiff served process
tory damages. Three months after the plaintiff on the defendant in State B via certified mail
filed and served the complaint, the plaintiff and as allowed by the State A Rules of Civil Proce-
defendant were in an automobile accident. dure. The defendant filed a motion to dismiss the
action on the grounds that the court in State A
May the State A plaintiff assert a negligence lacked personal jurisdiction. The court denied
claim against the State B defendant in the the motion. The defendant then filed an answer
pending breach of contract action to recover addressing the merits of the complaint. The
damages incurred in the automobile accident? answer also contained a motion to dismiss the
action for insufficient service of process.
(A) Yes, but only if the negligence claim seeks
more than $75,000. How should the court rule on the service of
process issue?
(B) Yes, regardless of the amount of the negli-
gence claim, because the court on motion (A) Deny the motion because the Federal Rules
may permit a plaintiff to file a supplemental of Civil Procedure expressly permit service
pleading setting out events that happened of process by certified mail.
after the pleading to be supplemented.
(B) Deny the motion because the defendant
(C) No, because the two claims do not arise waived any objection to service of process.
from the same transaction or occurrence or
series of transactions or occurrences. (C) Grant the motion because the Federal Rules
of Civil Procedure do not authorize service
(D) No, because the time for filing an amended of process by certified mail.
or supplemental complaint has expired.
(D) Grant the motion because service by certi-
fied mail violates due process under these
circumstances.
2. CIVIL PROCEDURE - PRETRIAL MATTERS QUESTIONS

Question 3 Question 4

A plaintiff filed a breach of contract action For the past 30 years, a motorist purchased a
against a defendant in federal district court, new model of a particular automobile every two
invoking the court’s diversity of citizenship juris- years. In the past year, the model was restyled
diction. The defendant filed an answer denying by its manufacturer. Although the motorist
the material allegations in the complaint. still liked the automobile for other reasons, the
Approximately two months after the answer was motorist believed that the automobile was now
served, the court entered a scheduling order that so ugly that he did not want to be seen in one,
required the parties to complete all discovery and he would be forced to buy a different brand.
within 10 months after the entry of the sched- The motorist was so angry that he sued the
uling order. Two months later (four months manufacturer in federal court for damages to
after service of the answer and two months into compensate him for having to drive something
the discovery period), the defendant sought to other than that model car.
amend his answer to add an affirmative defense
that the plaintiff’s claim was barred by the As an attorney for the manufacturer, which of
statute of limitations. the following documents should you file first?

Which of the following statements is correct (A) An answer denying the allegations in the
regarding the defendant’s right to amend his motorist’s complaint.
answer to add an affirmative defense?
(B) A motion for summary judgment.
(A) The defendant has a right to amend his
answer any time before trial. (C) A counterclaim for malicious prosecution.

(B) The defendant must obtain leave of the (D) A motion to dismiss for failure to state a
court to amend his answer, but the court claim upon which relief may be granted.
should freely grant leave.

(C) The defendant may amend his answer to


add the affirmative defense only if he can
show he could not, with due diligence, have
discovered the defense prior to serving his
answer.

(D) The defendant may not amend his answer


more than 21 days after serving it.
CIVIL PROCEDURE - PRETRIAL MATTERS QUESTIONS 3.

Question 5 Question 6

A corporation has been the subject of several In exchange for future royalty payments, an
news reports charging that the pollution put inventor sold a developer a license to use the
out by its factory endangers 5,000 residents of inventor’s patented technology. The inventor and
a subdivision. The corporation filed an action the developer are citizens of different states. A
naming all 5,000 residents as defendants and month later, the inventor and developer were
seeking a declaratory judgment that it is not in a traffic accident entirely unrelated to their
liable for damaging their health, and the court patent licensing transaction. A few months
properly certified the action as a class action. Per after that, the inventor filed a breach of contract
court order, detailed notice of the suit is posted action against the developer seeking $250,000
on every lightpole throughout the subdivision, in royalty payments. The developer then filed
but the residents are not notified individually. two counterclaims, one alleging that the inven-
tor’s licensed product did not work properly and
Was notice in this case constitutional? one alleging negligence and seeking $150,000
in damages for injuries sustained in the traffic
(A) Yes, because the posting was reasonably accident. The inventor was also injured in the
calculated to give notice to all 5,000 resi- accident and believes the developer’s negligence
dents. was the cause of the accident.
(B) Yes, if a state statute authorizes such May or must the inventor file a counterclaim
process. against the developer to recover $50,000 in
damages he sustained in the accident?
(C) No, because all residents could reasonably
be notified by mail. (A) The inventor must assert the claim as a
counterclaim or he will be barred from
(D) No, because individual defendants must later asserting it because the inventor’s tort
always be notified by personal service of a claim is a compulsory counterclaim.
summons.
(B) The inventor may assert the tort claim as a
counterclaim, or he may wait and assert it
in an independent action.

(C) The inventor may not assert a counterclaim


because the inventor is the original plaintiff
and only defendants may assert counter-
claims.

(D) The inventor may not assert a counterclaim


because the federal court lacks subject
matter jurisdiction over the inventor’s tort
claim since the amount of the claim is too
small.
4. CIVIL PROCEDURE - PRETRIAL MATTERS QUESTIONS

Question 7 Question 8

A consumer filed a products liability action While driving in the Western District of State
against a manufacturer in federal district court. A, a citizen and resident of State B caused an
The manufacturer timely filed a motion to accident that injured a citizen and resident of the
dismiss the action for lack of personal jurisdic- Southern District of State A. The State A citizen
tion. Following a hearing, the court found that it filed a negligence action against the State B
had personal jurisdiction over the manufacturer citizen in the United States District Court for the
and denied the motion. The manufacturer then Southern District of State A. The State B citizen
filed its answer, responding to the allegations in filed a motion seeking to dismiss the action on
the consumer’s complaint and raising affirmative the ground that the court lacked personal juris-
defenses. In its answer, the manufacturer moved diction over him. The court denied the motion.
to dismiss the action because the consumer The State B citizen then filed an answer as well
failed to state a claim upon which relief can be as a motion to dismiss the action for improper
granted. The court held a hearing on that motion venue.
and denied that motion as well. The parties then
conducted discovery for approximately a year, How should the court rule?
and the case was set for trial. As the trial was
about to begin, the manufacturer filed a motion (A) The court should grant the motion to dis-
to dismiss the action on the grounds that the miss, because the Southern District of State
court lacked subject matter jurisdiction because A is not a proper venue.
the parties actually were citizens of the same
state. (B) The court should grant the motion to
dismiss, because the court lacks personal
If the manufacturer’s assertion about citizen- jurisdiction over the State B defendant.
ship and a lack of subject matter jurisdiction are
correct, how should the court rule? (C) The court should deny the motion to
dismiss, because it was not timely asserted,
(A) Deny the motion because the manufacturer and any objection to venue has been
waived any objection to the court’s lack of waived.
subject matter jurisdiction by not raising
the objection in its pre-answer motion. (D) The court should deny the motion to
dismiss, because the Southern District of
(B) Deny the motion because the manufacturer State A is a proper venue.
waived any objection to the court’s lack of
subject matter jurisdiction by not raising the
objection in its answer.

(C) Deny the motion because it would be


prejudicial to the consumer to permit
the manufacturer to raise its objection to
subject matter jurisdiction so late in the
proceedings.

(D) Grant the motion.


CIVIL PROCEDURE - PRETRIAL MATTERS QUESTIONS 5.

Question 9 Question 10

A State A citizen filed an antitrust action A law firm entered into a contract with an
against an State B corporation in federal district office furniture supplier to purchase a substan-
court. Fourteen days after filing the complaint tial amount of office furniture. The contract
and before the State B corporation filed any provided that the supplier would deliver and
response, the State A citizen sought to file an install the furniture within 30 days. Six months
amended complaint, adding a breach of contract passed and the furniture had not been delivered,
claim against the same State B corporation. and the supplier informed the law firm that the
The breach of contract claim was completely furniture would not be available for at least
unrelated to the antitrust claim, and the facts another month. The law firm then filed a breach
underlying the two claims were completely of contract action against the supplier in federal
different and distinct. district court. The action sought recovery of lost
revenues and profits that the law firm alleged
May the State A plaintiff amend his were caused by the supplier’s failure to deliver
complaint? the furniture as agreed. The law firm properly
served process on the supplier, but the supplier
(A) No, because the amendment involves a failed to file an answer or otherwise respond to
claim that does not arise from the same the complaint. After several months passed, the
transaction or occurrence as the original law firm filed a motion requesting that the clerk
pleading. of court make an entry of default, and the clerk
did so.
(B) No, because a plaintiff may not join
unrelated claims in the same action. What procedure must the law firm follow to
obtain a default judgment against the supplier?
(C) Yes, but only with leave of the court or
written consent of the defendant. (A) File a motion to have the clerk of court
enter the default judgment, which the clerk
(D) Yes, even without leave of the court or may do without the supplier receiving any
written consent of the defendant. further notice of the motion.

(B) File a motion to have the clerk of court


enter the default judgment, which the clerk
may do as long as the supplier receives
additional notice of the motion for default
judgment.

(C) File a motion to have the judge enter the


default judgment, which the judge may do
without the supplier receiving any further
notice of the motion for default judgment.

(D) File a motion to have the judge enter the


default judgment, which the judge may do
as long as the supplier receives additional
notice of the motion for default judgment.
6. CIVIL PROCEDURE - PRETRIAL MATTERS QUESTIONS

Question 11 Question 12

A homeowner from State A filed a breach of A woman was severely injured in a car
contract and negligence action against a State accident when she was thrown through the
B architect in federal district court, seeking windshield of her car because her seatbelt failed
damages for problems with the homeowner’s to restrain her. She properly sued both the car
house. In designing the house, the architect and seatbelt manufacturers in federal court,
relied on calculations performed by a State B asserting a breach of warranty claim against
engineer whom the architect hired to assist him the car manufacturer and a products liability
with the project. The architect believes that claim against the seatbelt manufacturer. The
incorrect calculations by the engineer caused any car manufacturer filed a timely answer denying
defects in the design. liability. It also asserted an indemnification
claim against the seatbelt manufacturer based
May the architect assert and maintain an on a provision in the contract between the two
indemnity claim against the engineer in the manufacturers.
pending federal action?
Which of the following statements is correct
(A) No, because the federal court lacks sub- regarding the joinder of claims and parties in
ject matter jurisdiction over the architect’s this action?
claim against the engineer since they are
citizens of the same state and the claim is a (A) The car manufacturer and the seatbelt
state law claim. manufacturer cannot be properly joined as
defendants in this action because the claims
(B) No, because the architect’s claim does filed against each of them are based on dif-
not meet the requirements for a proper ferent legal theories.
impleader claim.
(B) The car manufacturer’s claim against the
(C) Yes, because the claim arises from the seatbelt manufacturer is not a proper cross-
same transaction or occurrence as the claim because it is based on a contract
homeowner’s original claims and therefore that is not logically related in time, place,
the architect may assert the claim even or facts to the claims the woman asserted
though it does not meet the requirements against each of the defendants.
for impleader.
(C) The car manufacturer and the seatbelt
(D) Yes, because the architect’s claim is a manufacturer are properly joined as defen-
proper impleader claim, and the court has dants in this action because the claims
supplemental jurisdiction over the claim. filed against each of them arise from the
same transaction or occurrence and have in
common a question of law or fact.

(D) The car manufacturer’s claim against the


seatbelt manufacturer is a proper third party
complaint that satisfies the requirements of
impleader.
CIVIL PROCEDURE - PRETRIAL MATTERS QUESTIONS 7.

Question 13 Question 14

A farmer brought a federal diversity action A woman filed a complaint in federal court
against a feed distributor, alleging that the feed against a man for both battery and negligence
deliveries sent by the distributor were less than for running over her with his golf cart. After
the agreed-upon amount bargained and paid for. the woman properly served the complaint on
Following discovery, the farmer filed a motion the man, he filed an answer denying all allega-
for partial summary judgment, seeking a ruling tions of negligence, but did not reply at all to the
solely on the issue that the contract had been battery claim. The woman would like to file a
breached. In support of the motion, the only motion to secure judgment in her favor on the
evidence the farmer cited was the distributor’s battery claim.
discovery responses, and noted that no evidence
was presented to establish that the distributor What type of motion should the woman’s
complied with the contract. The distributor lawyer file to try to obtain judgment on the
filed a response denying breach of contract, but battery claim?
did not provide any evidence in support of his
response. (A) A partial motion for summary judgment on
just the battery claim.
If no other evidence is offered by either party,
how should the court rule on the motion? (B) A motion for judgment on the pleadings for
failure to state a legal defense to the battery
(A) Deny the motion, because the farmer failed claim.
to carry his initial burden of establishing a
prima facie case for summary judgment. (C) A motion for a default judgment on the
battery claim.
(B) Deny the motion, because the distribu-
tor’s response created a dispute of fact that (D) A motion for a more definite statement on
precludes the grant of summary judgment. the battery claim.

(C) Grant the motion, because the evidence is


so one-sided that no reasonable jury could
find in favor of the distributor.

(D) Grant the motion, because the distributor


has not carried his burden of establishing
by affirmative evidence that there is a
genuine dispute of material fact.
8. CIVIL PROCEDURE - PRETRIAL MATTERS QUESTIONS

Question 15 Question 16

A citizen of State A sued a police officer A book dealer (who is a citizen of State A)
in federal district court pursuant to 28 U.S.C. sold a number of rare comic books to a collector
§1983, alleging violation of her civil rights. The (who is a citizen of State B) for $90,000. The
citizen sought to join a defamation claim against collector paid $10,000 upon delivery and agreed
the police officer’s wife arising from a public to pay the balance in eight monthly installments
altercation in which the two were involved. The of $10,000 each. Two weeks after the sale, the
claim against the police officer seeks $100,000 dealer and the collector were involved in an
in damages, and the claim against the officer’s automobile accident that was unrelated to the
wife seeks $85,000 in damages. All three comic book transaction. More than eight months
parties are citizens and residents of State A. The pass and the collector failed to pay the $80,000
operative facts applicable to the two claims are owed on the comic books.
unrelated. The police officer and his wife filed
a motion to dismiss the action both for lack of If the dealer files a contract action against
subject matter jurisdiction and improper joinder the collector in federal district court to collect
of parties. the $80,000 owed on the comic books, can he
join in the same action a negligence claim in the
How should the court rule? amount of $100,000 for injuries incurred in the
automobile accident?
(A) Deny the motion, because joinder is proper
and supplemental jurisdiction exists since (A) Yes, the dealer must join the two claims in
the State A citizen should expect to try both one action or else the omitted claim will be
claims in the same proceeding. waived.

(B) Deny the motion, because joinder of the (B) Yes, the dealer may join the claims in one
police officer’s wife is permissible, so action but is not required to do so.
subject matter jurisdiction over the claim
against her is not required. (C) No, because they do not arise from the
same transaction or occurrence.
(C) The court has discretion either to hear both
claims or to dismiss them. (D) No, because the court does not have supple-
mental jurisdiction over the negligence
(D) Grant the motion, because the claims do not claim.
arise from the same transaction or occur-
rence, so joinder is improper, and the court
lacks subject matter jurisdiction over the
claim against the police officer’s wife.
CIVIL PROCEDURE - PRETRIAL MATTERS QUESTIONS 9.

Question 17 Question 18

A rancher purchased cattle from a breeder. A plaintiff filed a negligence action against
As payment for the cattle, the rancher gave a defendant in federal district court, seeking
the breeder a promissory note that obligated damages for personal injuries suffered in a
the rancher to pay the breeder a lump sum of traffic accident. The plaintiff timely served on
$80,000 18 months after the purchase. During the defendant a request for production of certain
the course of the year that he had the cattle, documents. The defendant objected to part of
however, the rancher discovered that the cattle the request, claiming that it sought information
were diseased, so a year after the purchase the beyond the scope of discovery. The defendant
rancher filed an action against the breeder in honestly believed this to be true, but case law
federal district court for breach of warranty, clearly held to the contrary. After conferring
alleging that the cattle were diseased at the time with the defendant and not resolving the matter,
of sale the previous year. The breeder will file the plaintiff filed a motion to compel the defen-
his answer within three weeks. dant to produce the documents.

May the breeder assert in his answer a If the court orders the defendant to produce
counterclaim against the rancher seeking the documents, what sanctions may the court
payment of the promissory note? impose on the defendant at that time?

(A) Yes, the breeder may assert the claim on (A) The court may enter a default judgment
the promissory note as a counterclaim or he against the defendant.
may assert it in a separate action.
(B) The court may strike relevant pleadings of
(B) Yes, the claim on the promissory note is a the defendant or enter an order precluding
compulsory counterclaim because it arises the defendant from offering evidence on
from the same transaction or occurrence as certain claims or defenses.
the rancher’s initial claim.
(C) The court may not impose sanctions at this
(C) No, because it does not arise from the same time, but it must require the defendant to
transaction or occurrence as the rancher’s pay the reasonable expenses incurred by the
initial claim. plaintiff in making the motion to compel.

(D) No, because the claim will not yet be (D) The court at this time may neither impose
mature when the breeder files his answer. sanctions nor require the defendant to pay
any expenses incurred by the plaintiff.
CIVIL PROCEDURE -
PRETRIAL MATTERS
ANSWERS
CIVIL PROCEDURE - PRETRIAL MATTERS ANSWERS 1.

CIVIL PROCEDURE - PRETRIAL MATTERS ANSWERS

Answer to Question 1

(B) The plaintiff may assert the negligence claim if permitted by the court. The court on motion may
permit a plaintiff to file a supplemental pleading that relates to matters occurring after the date
of the original pleading. [Fed. R. Civ. P. 15(d)] (A) is incorrect because a plaintiff may aggregate
all his claims against a single defendant to meet the jurisdictional amount, regardless of whether
the claims are legally or factually related. Since the contract action already exceeds $75,000, the
amount in controversy in the negligence action is irrelevant. (C) is incorrect because a plaintiff may
bring all claims he has against a defendant in a single action regardless of whether they are related.
(D) is incorrect because there is no time limit on when a supplemental pleading can be filed.

Answer to Question 2

(B) The court should deny the motion. Federal Rule 12(h) provides that objections to service of
process must be asserted in the defendant’s first response to the complaint—whether that response
is a Rule 12(b) pre-answer motion or an answer—or else it is waived. Since the defendant filed
a Rule 12(b) pre-answer motion to dismiss and did not include his objection to service in that
motion, the defendant waived the objection and cannot later assert it in his answer. (A), (C), and
(D) are incorrect because the defendant waived his objection to service of process. It is irrelevant
whether service of process by certified mail is permitted by the Federal Rules of Civil Procedure
or violates due process.

Answer to Question 3

(B) The defendant may amend with leave of the court. A party may amend a responsive pleading of
right within 21 days after serving it. Thereafter, according to Federal Rule 15, the party may amend
only with consent of all parties or with leave of the court, but the “court should freely grant leave
when justice so requires.” With so much time left for discovery and before trial, a court would
almost certainly grant leave to amend. (A) is incorrect because, as stated, the time for amendment
as of right is 21 days after serving it, not any time before trial as (A) implies. (C) is incorrect in that
due diligence in discovering the defense need not be shown. (D) is incorrect. The 21-day period
applies to amendment as of right, but a court may grant leave to amend after that period.

Answer to Question 4

(D) As the manufacturer’s attorney, you should file a preanswer motion under Rule 12(b)(6) asserting
the defense of failure to state a claim upon which relief may be granted. This would almost
certainly dispose of the action quickly, avoiding even the need to file an answer, since there is no
apparent cause of action available to the motorist under which he could receive the relief he seeks.
(A) is incorrect because filing an answer would not dispose of the claims. (B) is incorrect. A
motion for summary judgment needs fact assertions upon which to base a motion. (C) is incorrect.
A claim for malicious prosecution would not dispose of the litigation. Additionally, such a claim
would not ripen until after the motorist’s claims are disposed of.

Answer to Question 5

(C) Notice here was not constitutional. Due process requires that the method of giving notice must
have a reasonable prospect of giving actual notice. Posting of notice is insufficient to notify
一 信:liuxue119118 , 们 信免 供

2. CIVIL PROCEDURE - PRETRIAL MATTERS ANSWERS

persons whose names and addresses are known. These persons must be notified at least by
ordinary mail. Thus, (A) is incorrect. (B) is incorrect because, even if authorized by statute, due
process requirements would not be met. (D) is incorrect because personal service is not always
required.

Answer to Question 6

(A) The inventor must file a counterclaim against the developer to recover the $50,000 in damages
from the accident. As between the plaintiff and the defendant, any claim that arises from the same
transaction or occurrence as a claim asserted against a party is a compulsory counterclaim—
even if the party asserting the counterclaim is a plaintiff. (C) is therefore incorrect. The court
also has jurisdiction over both claims. The way to think about these claims is that the plaintiff,
the inventor, is essentially asserting two claims against the developer, and a single plaintiff may
aggregate his claims against a single defendant. (Had the inventor included both claims in his
original complaint, the claims could have been aggregated, giving the court subject matter juris-
diction based on diversity.) (B) is incorrect because the inventor’s counterclaim is compulsory and
must be asserted. If he does not, he will be barred from doing so later. (D) is incorrect because
compulsory counterclaims generally do not have to meet the jurisdictional amount requirement
for diversity jurisdiction, and as explained above, the court does have subject matter jurisdiction
over both claims.

Answer to Question 7

(D) The court should grant the motion to dismiss the action. Federal subject matter jurisdiction is
never waived, and a federal court must dismiss whenever it determines that it lacks subject matter
jurisdiction. Thus, (D) is correct and (A) and (B) are incorrect. (C) is incorrect because prejudice
to the parties due to the fact that extensive discovery has been conducted is not a consideration.
The court does not have discretion to retain a case when it lacks subject matter jurisdiction.

Answer to Question 8

(C) The court should deny the motion because it was not timely asserted. Objections to venue must
be asserted in the defendant’s first Rule 12 response to the complaint—whether that response is a
pre-answer motion or an answer. If a defendant does not object to venue, the objection is waived.
Here, because the defendant did not include an objection to venue with his pre-answer motion to
dismiss based on a lack of personal jurisdiction, he waived the improper venue issue. Thus, (C) is
correct and (A) is incorrect. (B) is also incorrect. It is important to distinguish between personal
jurisdiction (i.e., the power of the court over a particular defendant) from venue (i.e., whether the
court is the proper place for a trial within the state). Here, the court correctly ruled that a State A
court would have personal jurisdiction over the defendant, because the defendant has purposeful
contacts with State A (driving on State A roads), and all states have long arm provisions granting
personal jurisdiction over a defendant who commits a tort within state boundaries. Plus, the facts
state that the court already denied the motion. (D) is not correct because the Southern District of
State A is not a place of proper venue because the accident did not occur there nor is the defen-
dant’s residence there.

Answer to Question 9

(D) The plaintiff may amend his complaint. A party may amend a pleading to which a responsive
pleading is required of right if the party does so within 21 days after the responsive pleading or
CIVIL PROCEDURE - PRETRIAL MATTERS ANSWERS 3.

other response is served. [Fed. R. Civ. P. 15(a)(1)(B)] (A) is incorrect because the amendments to
a complaint do not have to arise from the same transaction or occurrence as the original plead-
ings. (B) is incorrect because a plaintiff may bring all claims he has against a defendant regard-
less of whether they are related. (C) is incorrect because a complaint may be amended once as a
matter of course within 21 days of serving it or 21 days after service of a responsive pleading or
pre-answer motion. Thereafter, a pleading may be amended only by the written consent of the
defendant or by leave of the court.

Answer to Question 10

(C) The law firm may file a motion with the judge. A default judgment may be entered by the clerk
only if it is for a sum certain. Otherwise, a judge must hear and determine damages. Thus, (C) is
correct and (A) and (B) are incorrect. Furthermore, a defendant is entitled to notice only if she has
entered some sort of appearance in the case. Here, there has been no appearance or other pleading
in the case, so the supplier is not entitled to further notice in the action. As a result, (B) and (D)
are incorrect.

Answer to Question 11

(D) The architect may assert and maintain an indemnity claim against the engineer. The architect’s
claim is a proper impleader claim because the architect’s claim asserts that the engineer is liable
to the architect for any liability the architect has to the homeowner. (B) and (C) are therefore
incorrect. If the third-party claim does not meet the requirements for diversity of citizenship or
federal question jurisdiction, it will invoke supplemental jurisdiction because the claims will meet
the common nucleus of operative fact requirement of supplemental jurisdiction. Here, the court
has supplemental jurisdiction over the architect’s claim because it arises from a common nucleus
of operative fact as the homeowner’s original claim. The fact that the architect and engineer
share state citizenship does not defeat supplemental jurisdiction. A claim by a defendant against a
third-party defendant is not among the prohibited uses of supplemental jurisdiction specified in 28
U.S.C. section 1367(b). (A) is therefore incorrect.

Answer to Question 12

(C) The two parties are properly joined as defendants. Parties may join as plaintiffs or be joined as
defendants whenever (i) some claim is made by each plaintiff and against each defendant relating
to or arising out of the same series of occurrences or transactions; and (ii) there is a question of
fact or law common to all parties. Here, although the woman’s claims against the seatbelt and car
manufacturer are different, they arise from the same transaction—the car accident—and who is
at fault and how much are questions of fact and law that both claims have in common. Therefore,
choice (C) correctly states the rule of law. (A) is wrong because it incorrectly concludes that the
parties cannot be joined and misstates the rule of law. Whether the claims against each defendant
are different is not relevant in determining whether joinder is permitted. Instead, the correct rule,
as set forth above, is whether the claims arose from the same series of occurrences or transactions
with a common question of fact or law. (B) is wrong because the car manufacturer’s indemnity
claim against the seatbelt manufacturer is a proper cross-claim. Moreover, it states the wrong test
for a cross-claim. A party may file any claim against a co-party arising from the same transaction
or occurrence as either the original complaint or a counterclaim, or relating to property that is
the subject matter of the transaction. Here, the indemnity claim arose from the same transaction
as the original complaint—the car accident. Therefore, this is a proper cross-claim. (D) is wrong
because it does not satisfy the requirements of impleader. Impleader is available when a third
4. CIVIL PROCEDURE - PRETRIAL MATTERS ANSWERS

party (non-party) is derivatively liable to a defending party for all or part of a judgment that the
plaintiff may recover against the defendant. Here, both the car and seatbelt manufacturers are
already parties (co-defendants) to this action. Therefore, this is not an impleader situation.

Answer to Question 13

(A) The court should deny the motion. Summary judgment may be granted if, from the pleadings,
affidavits, and discovery materials, it appears that there is no genuine dispute of material fact
and the moving party is entitled to judgment as a matter of law. [Fed. R. Civ. P. 56] For the plain-
tiff to prevail on a summary judgment motion, he must satisfy the burden of production on every
element of its claim. If neither party offers any evidence on an issue, the party with the burden of
production loses. Here, the moving party is the farmer, who moved for partial summary judgment
on the issue of breach. As such, it is the farmer’s burden to present substantial evidence of breach
(lack of compliance), not the defendant-distributor’s burden to present evidence to prove compli-
ance. Therefore, the farmer failed to meet the prima facie standard for summary judgment. (B) is
wrong because, although it is true that the court may not decide disputed fact issues (meaning a
dispute backed by evidence on both sides of the issue) on a motion for summary judgment, simply
citing to the distributor’s lack of response regarding compliance without any evidence supporting
noncompliance (breach) does not meet the standard of showing that there is no genuine dispute
of a material fact. (C) is wrong because, although a plaintiff may prevail on a summary judgment
motion when the plaintiff presents substantial evidence on each and every element of its claim,
here the farmer failed to present any evidence to prove breach of contract. Instead, he only
presented the defendant’s failure to respond to show compliance. (D) is wrong because it incor-
rectly places the initial burden of production on the defendant-distributor to meet the standard for
summary judgment. It is the plaintiff-farmer’s burden to initially produce substantial evidence of
breach before the burden of production shifts to the defense to present.

Answer to Question 14

(B) The woman’s lawyer should file a motion for judgment on the pleadings. Once pleadings are
complete, any party may move for judgment on the pleadings. [Fed. R. Civ. P. 12(c)] A motion for
judgment on the pleadings may be filed only after all pleadings are complete, but early enough
so as not to delay the trial. In reviewing the motion, the court accepts all facts alleged in all the
pleadings as true, and a failure to deny allegations in the complaint constitutes an admission.
If the court concludes a party is entitled to judgment as a matter of law, it will grant the motion
and render judgment. Failure to state a legal defense to a claim is one listed reason for a motion
for judgment on the pleadings. [Fed. R. Civ. P. 12(h)(2)] Here, since the man never replied to the
battery assertions, they are deemed admissions. Therefore, the proper motion for the woman’s
attorney to file is a motion for judgment on the pleadings. Note that the court may allow the man
to amend his answer if the failure to defend was an oversight, but that is not what the question
asked. (A) is wrong because a motion for summary judgment looks at the evidence and facts
outside of the pleadings and also considers affidavits and discovery materials. Here, only the
pleadings are at issue. As such, a motion for summary judgment is not the proper motion to file.
(C) looks appealing, but is wrong because a default judgment is a judgment, with the same effect
as any other judgment, that is entered because the defendant fails to oppose (plead or otherwise
defend) the case at all. A party opposes a case by filing an answer or reply. Here, the man did
oppose the case by filing an answer to the negligence claim. Although he did not reply to the
battery claim, since a failure to deny constitutes an admission, the allegations are deemed admis-
sions. Therefore, a motion for default judgment is not the correct choice. (D) is wrong because a
party moves for a more definite statement before responding (by filing an answer or reply) to a
CIVIL PROCEDURE - PRETRIAL MATTERS ANSWERS 5.

pleading (a complaint) when it is so vague or ambiguous that a responsive pleading cannot reason-
ably be framed. The opposing party then has 14 days after notice of an order to obey unless the
court fixes a different time. If not obeyed, the court may strike the pleading or issue any other
appropriate order. [Fed. R. Civ. P. 12(e)] Here, the man already filed an answer to the complaint,
so this is an improper motion. Moreover, the facts state the woman is seeking a judgment on the
battery claim, not a correction to it.

Answer to Question 15

(D) The court should grant the motion to dismiss the defamation action both for lack of subject matter
jurisdiction and improper joinder of parties. Parties may be joined as defendants whenever: (i)
some claim is made by each plaintiff and against each defendant relating to or arising out of the
same series of occurrences or transactions; and (ii) there is a question of fact or law common
to all the parties. Here, the claims against the police officer and his wife are not related and do
not arise out of the same occurrence or transaction, and there is no common question of fact or
law involved in the claims. Therefore, joinder is improper. Even if joinder were proper, the court
would still need subject matter jurisdiction over both claims. (B) is therefore incorrect. Here,
jurisdiction based on diversity of citizenship is not available because all the parties are citizens of
State A. A court has federal question jurisdiction over a claim when it arises under federal law.
Here, the citizen is alleging a defamation claim against the police officer’s wife, which is not a
claim arising under federal law. Therefore, the court does not have subject matter jurisdiction over
the claim. (A) is incorrect because joinder is improper, and supplemental jurisdiction applies to
situations where the two claims derive from a common nucleus of operative fact and are such that
a plaintiff would ordinarily be expected to try them in one judicial proceeding. The two claims
in this case do not derive from a common nucleus of operative fact, so supplemental jurisdic-
tion cannot apply here. (C) is incorrect because the court does not have discretion to hear a claim
when the court lacks subject matter jurisdiction over it.

Answer to Question 16

(B) The dealer may join the claims in one action. FRCP 18 permits a plaintiff to join all claims the
plaintiff has against a given defendant regardless of whether they are related. (C) is therefore
incorrect. Diversity of citizenship jurisdiction also exists over both claims. The parties are diverse,
and the amount in controversy exceeds $75,000 for each claim. (A) is incorrect because the dealer
may join the claims but is not required to do so. (D) is incorrect because supplemental jurisdic-
tion applies to situations where a claim does not invoke federal question or diversity jurisdiction
but arises out of a common nucleus of operative fact as the original claim. Here, the negligence
claim invokes diversity jurisdiction, thereby making supplemental jurisdiction unnecessary.
Furthermore, for purposes of meeting the jurisdictional amount, a plaintiff may aggregate all her
claims against a single defendant. This aggregation is permitted regardless of whether the claims
are legally or factually related to each other. Thus, even if the claims did not individually exceed
$75,000, subject matter jurisdiction would exist if the claims together exceed $75,000.

Answer to Question 17

(D) At the time the rancher filed his complaint and at the time the breeder files his answer, the breeder
will have no claim. One year has passed since the purchase. The rancher has no obligation to pay
until the note matures 18 months after the purchase. In order for a counterclaim to be compul-
sory it must not only arise out of the same transaction or occurrence as the plaintiff’s claim, the
defendant must have the claim at the time the plaintiff serves his pleading. The breeder will have
6. CIVIL PROCEDURE - PRETRIAL MATTERS ANSWERS

no claim until 18 months after the purchase. (A), (B), and (C) are incorrect because, although
the promissory note claim arises from the same transaction or occurrence, it is not a compulsory
counterclaim because the breeder did not have the claim at the time the rancher filed his action.

Answer to Question 18

(C) The court may not impose sanctions on the defendant, but it must require the defendant to pay the
plaintiff’s reasonable expenses incurred in making the motion. If a motion to compel is granted,
the court must require the opposing party to pay the movant’s reasonable expenses incurred in
making the motion. However, the court may not order this payment if the movant filed the motion
before attempting to seek production without a court order, if the nondisclosure was substan-
tially justified, or if other circumstances exist that make an award of expenses unjust. [Fed. R.
Civ. P. 37(a)(5)(A)] None of these exceptions appear applicable here. (A) and (B) are incorrect
because they describe possible sanctions for a party who fails to comply with an order to provide
discovery. (D) is incorrect because the court may require the opposing party to pay the movant’s
expenses.
CIVIL PROCEDURE -
SUBJECT MATTER
JURISDICTION
QUESTIONS
CIVIL PROCEDURE - SUBJECT MATTER JURISDICTION QUESTIONS 1.

CIVIL PROCEDURE - SUBJECT MATTER JURISDICTION QUESTIONS

Question 1 Question 2

A homeowner from State A hired a builder A woman who had been living in a foreign
from State B to build a summer home for her in country for many years traveled to the United
State C. They negotiated and signed the building States to assist her brother, who had been
contract in the builder’s business office in State indicted for a serious crime committed in State
D. Shortly thereafter, the homeowner informed A. She retained a lawyer who resided in State A
the builder that she had decided not to build to represent her brother, and then she returned to
the summer home and would no longer need the foreign country. Thereafter, the lawyer sent
the builder’s services.The builder commenced the woman a bill for $100,000 for his services,
an action against the homeowner for breach of but she refused to pay. Before she moved to the
contract in the federal court for State D, seeking foreign country, the woman had resided in State
damages in excess of $75,000. The homeowner B. Although the woman is still a citizen of the
moved for a transfer of venue to the federal court United States, she had moved to the foreign
for State A. The court granted the motion, and country 10 years ago, intending to live there
the case was transferred to the federal court for permanently.May the lawyer commence an
State A.Assuming that the contract does not action against the woman for breach of contract
contain a choice of law provision, what substan- in the federal court for State A?
tive law should the federal court for State A
apply? (A) Yes, because the action is between citizens
of different states.
(A) Federal common law principles of negli-
gence, because this is a diversity action in a (B) Yes, because the action is between a citizen
federal court. of a state and an alien.

(B) State C law, because the summer home was (C) No, because the federal court for State A
to be built there. may not exercise personal jurisdiction over
the woman.
(C) The substantive law as determined by the
choice of law rules of State A. (D) No, because the court lacks subject matter
jurisdiction.
(D) The substantive law as determined by the
choice of law rules of State D.
2. CIVIL PROCEDURE - SUBJECT MATTER JURISDICTION QUESTIONS

Question 3 Question 4

A worker who lives in State A believes that A writer registered under federal copyright
her employer passed her over for a promotion law his copyright in certain song lyrics he wrote.
on the basis of her race. The worker filed an The writer later entered into a contract with an
action against the employer in federal district advertiser in which the writer granted the adver-
court, alleging a violation of federal employment tiser a license to use the lyrics in radio advertise-
discrimination statutes and seeking $50,000 ments. When the writer heard the advertisement
in damages and lost wages. The employer is using the lyrics, the writer was incensed at how
a corporation that was incorporated under the lyrics had been used. Believing that the
State B law, but all its operations, officers, and advertiser had lied to him about how the lyrics
employees are in State A. would be used, the writer filed an action in
federal district court claiming that the advertiser
Does the federal court have subject matter had made false representations that fraudulently
jurisdiction over the worker’s employment induced the writer into entering the contract to
discrimination claim? license the lyrics. The writer is a citizen of State
A. The advertiser is a partnership comprised of
(A) No, because the worker and the employer partners who are citizens of State A, State B,
are both citizens of State A. and State C. The partnership’s headquarters and
most of its operations are in State B.
(B) No, because the amount in controversy is
too small. Does the federal court have subject matter
jurisdiction over the action?
(C) Yes, because the worker is a citizen of State
A and the employer is a citizen of State B. (A) No, because the action does not arise under
federal law and the parties are citizens of
(D) Yes, because the claim arises under federal the same state.
law.
(B) Yes, because the plaintiff and defendant are
citizens of different states.

(C) Yes, because the action arises under federal


law.

(D) Yes, because the transaction involves inter-


state commerce.
CIVIL PROCEDURE - SUBJECT MATTER JURISDICTION QUESTIONS 3.

Question 5 Question 6

A State A consumer borrowed $10,000 from The plaintiff, a citizen of State A, filed suit
a bank in State A. The bank was incorporated against the defendant, also a citizen of State A,
in State A and operated only in that state. The in federal district court, alleging that the defen-
consumer filed an action in federal district court, dant had failed to perform a contract to provide
alleging that the bank violated federal truth in 1,000 fully automatic machine guns. The defen-
lending statutes in making the loan. The action dant claimed that a recently enacted federal
sought $1,000 as a result of the alleged violation. statute made the manufacture of fully automatic
machine guns illegal.
Does the federal district court have subject
matter jurisdiction over the action? Does the federal district court have subject
matter jurisdiction?
(A) Yes, because the action arises under federal
law. (A) No, because no federal question has been
presented and the requirements of diversity
(B) Yes, because the transaction involves inter- of citizenship jurisdiction have not been
state commerce. met.

(C) No, because the amount in controversy is (B) No, unless the plaintiff moves to another
too small. state after filing suit.

(D) No, because the consumer and the bank are (C) Yes, because a federal statute has been
citizens of the same state. enacted, thereby presenting a federal
question.

(D) Yes, if the value of 1,000 fully automatic


machine guns exceeds $75,000.
4. CIVIL PROCEDURE - SUBJECT MATTER JURISDICTION QUESTIONS

Question 7 Question 8

The plaintiff and the defendant are both A resident of State A recently purchased
citizens of State A. The plaintiff wished to sue a new home for $275,000 from a developer,
the defendant in a federal court on a $100,000 a limited partnership. Shortly after the buyer
claim. For this reason, and only this reason, she moved in, the house slid into a canyon and was
moved permanently to State B and filed suit in no longer habitable. The buyer sued the devel-
a federal court there, with jurisdiction being oper in federal court. The developer’s organiza-
based on diversity of citizenship. A few days tional structure consists of two general partners,
after being properly served, the defendant, after who are citizens of State B, and four limited
accepting a job offer from a company based in partners, who all are citizens of State A. The
State B, moved permanently to State B. developer moves to dismiss the action for lack of
subject matter jurisdiction.
For purposes of the plaintiff’s case, what are
the citizenships of the parties? Will the developer’s motion be granted?

(A) Both the plaintiff and the defendant are (A) No, because the citizenships of the limited
citizens of State B. partners are not considered when determin-
ing whether diversity exists.
(B) The defendant is a citizen of State A, and
the plaintiff is a citizen of State B. (B) No, because the general partners and
limited partners are of diverse citizenships.
(C) The defendant is a citizen of State B, and
the plaintiff is a citizen of State A. (C) Yes, because the buyer and the limited
partners share State A citizenship.
(D) Both the plaintiff and the defendant are
citizens of State A. (D) Yes, because actions regarding real estate
must be brought exclusively in state court.
CIVIL PROCEDURE - SUBJECT MATTER JURISDICTION QUESTIONS 5.

Question 9 Question 10

Thirty-five farmers along the State A-State B A plaintiff, a citizen of State A, sued a defen-
border incurred severe damages when a factory dant, a citizen of State B, on three claims: the
released toxic smoke that damaged their crops. first for $25,000 for breach of contract; the
Thirty-two farmers are on the State A side of second for $10,000 for fraud; and the third for
the border; three are on the State B side. All $50,000 for breach of a second contract.
landowners are citizens of their respective states.
They seek to bring a class action in federal court Does the federal district court in State B have
in State A against the factory owner, which is a subject matter jurisdiction?
citizen of State A.
(A) No, because no one claim exceeds $75,000.
If a class action is otherwise proper and the
three State B landowners are named as the class (B) No, because a citizen of a state cannot be
representatives, may the action be brought under sued in a federal court in that state.
diversity jurisdiction?
(C) Yes, because diversity of citizenship juris-
(A) Yes, because only the named representa- diction is available.
tives’ citizenship is considered in determin-
ing diversity of citizenship. (D) Yes, because federal question jurisdiction is
available.
(B) Yes, because the case presents a question
involving interstate commerce.

(C) No, because the unnamed class members


are State A citizens.

(D) No, because the three named representa-


tives are not of diverse citizenship.
6. CIVIL PROCEDURE - SUBJECT MATTER JURISDICTION QUESTIONS

Question 11 Question 12

Three years ago a woman’s husband Following a midair collision, a passenger in


abandoned her and their home in State A and one of the planes, a citizen of State A, sued the
moved to State B, where he is now a citizen. The Federal Aviation Administration in federal court
woman now files against her husband for divorce under the Federal Tort Claims Act, alleging
in federal court in State A, seeking a property negligence on the part of its air traffic control-
settlement valued at $200,000. The husband lers. The passenger also named the pilot of the
moved to dismiss for lack of jurisdiction. other plane, a citizen of State A, as a defendant
for a claim based on state law negligence. The
Will the court grant the husband’s motion? pilot moves for dismissal as to him.

(A) No, because diversity of citizenship is pres- If the court grants the motion to dismiss, what
ent and the amount in controversy exceeds is the most likely reason?
$75,000.
(A) The Federal Rules of Civil Procedure do
(B) No, if State A has a long arm statute not permit joinder under these facts.
granting personal jurisdiction over the
husband. (B) There is no basis for federal subject matter
jurisdiction.
(C) Yes, if the husband moves to State A after
suit is filed. (C) The court has declined to exercise supple-
mental jurisdiction.
(D) Yes, because federal courts do not have
subject matter jurisdiction over domestic (D) The claim against the pilot is not suffi-
relations cases. ciently related to the claim against the
Federal Aviation Administration.
CIVIL PROCEDURE - SUBJECT MATTER JURISDICTION QUESTIONS 7.

Question 13 Question 14

A citizen of State A was killed in a car A developer contracted with a general


accident caused by a motorist who was also contractor to build an office building, and
a citizen of State A. A citizen of State B is completion of the building was two years late.
executor of the decedent’s estate. The executor The developer filed a breach of contract action
files a complaint in federal court in State A in federal district court against the general
asserting a wrongful death claim against the contractor, seeking damages caused by the
motorist. The motorist files a motion to dismiss delay. The developer is a citizen of State A, and
on the basis of lack of subject matter jurisdiction. the general contractor is a citizen of State B.
The general contractor filed a third-party claim
How should the court rule? against a major subcontractor, claiming that the
subcontractor caused any delay and should be
(A) Grant the motion, because diversity of citi- liable to the general contractor for anything the
zenship does not exist. general contractor has to pay the developer. The
subcontractor believes that the developer inter-
(B) Grant the motion, because probate matters fered with the subcontract and that the devel-
are within the sole jurisdiction of the state oper’s interference caused not only the delay but
courts. also substantial cost overruns for the subcon-
tractor. Like the developer, the subcontractor is a
(C) Deny the motion, because diversity of citizen of State A.
citizenship exists.
May the subcontractor assert and maintain a
(D) Deny the motion, because a plaintiff is claim against the developer in the pending action
entitled to choose his forum. seeking payment for the cost overruns?

(A) No, because the federal court lacks subject


matter jurisdiction over the subcontrac-
tor’s claim since the subcontractor and the
developer are citizens of the same state and
the claim does not arise under federal law.

(B) No, because the subcontractor is a third-


party defendant and may not assert claims
against the original plaintiff.

(C) Yes, because the subcontractor’s claim


arises from the same transaction or occur-
rence as the plaintiff’s original claim, and
the court has supplemental jurisdiction over
the claim.

(D) Yes, because the subcontractor’s claim


arises from the same transaction or occur-
rence as the other claims in the action, so
it is a compulsory claim that the subcon-
tractor may assert only in the pending
action and therefore is within the federal
court’s subject matter jurisdiction.

一 信:liuxue119118 , 们 信免 供
8. CIVIL PROCEDURE - SUBJECT MATTER JURISDICTION QUESTIONS

Question 15 Question 16

A supplier of rivets contracted to supply An employee sued his employer in federal


rivets to an aircraft manufacturer. The aircraft court in State B, alleging that the employer
manufacturer cancelled the order, and the violated federal civil rights statutes by permit-
supplier intends to file a breach of contract ting a hostile work environment in which the
action against the aircraft manufacturer, seeking employee was afraid to go to work due to harass-
damages of $600,000. The supplier is a State A ment from a co-worker. The co-worker is from
corporation, and all its operations are in State A. State A, and both the employer and employee
The aircraft manufacturer is a State B corpora- are from State B. The employee properly joined
tion. Its corporate headquarters are in State C, the co-worker under the permissive party joinder
but its manufacturing operations are in State A. rule. The law in State B provided the basis for
The supply contract was signed in State A, and the employee’s claim against the co-worker, and
it provided that the rivets would be delivered and neither claim exceeded $20,000.
paid for in State A.
Does the federal court have supplemental
Can the supplier maintain an action based on jurisdiction over the employee’s claim against the
diversity of citizenship jurisdiction in a federal co-worker?
district court?
(A) Yes, because supplemental jurisdiction ex-
(A) No, because both corporations are citizens ists whenever the primary claim is based on
of State A. a federal question.

(B) No, because all relevant events giving rise (B) Yes, because supplemental jurisdiction may
to the claim occurred in one state. include claims that involve the joinder or
intervention of additional parties.
(C) Yes, because the supplier is a citizen of
State A and the aircraft manufacturer is a (C) No, because supplemental jurisdiction does
citizen of State B and State C. not include claims that involve the joinder
or intervention of additional parties.
(D) Yes, because the supplier is a citizen of
State A and the aircraft manufacturer is a (D) No, because supplemental jurisdiction does
citizen only of State B. not exist when complete diversity of citizen-
ship among all joined parties is lacking.
CIVIL PROCEDURE - SUBJECT MATTER JURISDICTION QUESTIONS 9.

Question 17 Question 18

An electrician from State A was hired by a A corporation manufactured a patented


commercial builder from State B to install all automatic potato peeler. Subsequently, that
the electricity in a new strip mall located in State corporation’s biggest rival manufactured a
A. Upon completion, the builder refused to pay similar potato peeler. Both are corporations of
the electrician, claiming the installation was not the same state. The state has a unique statute
up to the building code. The electrician filed a authorizing quadruple damages for unfair
diversity action in federal court sitting in State business competition relating to potatoes;
A against the builder for breach of contract. however, there is considerable debate whether
Although the action was filed within the appli- the law is constitutional under the state constitu-
cable statute of limitations period, the electrician tion. The corporation holding the patent sued
failed to serve the summons and complaint on its rival in federal court, alleging that the rival
the builder until after the statute of limitations corporation violated its patent and the unfair
on the claim expired. business law. Shortly after the pretrial meeting
of the parties required under the Federal Rules,
Under the law in State A, an action is the two companies reached a settlement of the
“commenced” for purposes of the state’s statute patent infringement claim.
of limitations when the defendant is served with
the summons and complaint. However, under the May the court dismiss the remaining claim?
Federal Rules of Civil Procedure, an action is
“commenced” in federal court when the plain- (A) No, because the federal court still has
tiff files the complaint in federal court. The supplemental jurisdiction over the case.
builder filed a motion to dismiss the complaint
for exceeding the statute of limitations period, (B) No, because the federal court already
arguing the state law governs. exercised jurisdiction over the case and thus
cannot now dismiss it.
How should the court rule?
(C) Yes, because the federal court never had
(A) Grant the motion, because statutes of jurisdiction over the claim.
limitations and rules for tolling statutes of
limitations are considered substantive rules, (D) Yes, because the federal court may decline
and therefore the state law will apply. to exercise its supplemental jurisdiction.

(B) Grant the motion, because the federal


rule will change the outcome of the case,
and when applying a federal rule that will
change the outcome, the federal court
must apply state law to discourage forum
shopping.

(C) Deny the motion, because the Federal Rules


of Civil Procedure control over conflicting
state law so long as they are procedural and
not unconstitutional.

(D) Deny the motion, because the statutes of


limitations and rules for tolling statutes of
limitations are considered procedural rules,
and therefore, the federal law will apply.
CIVIL PROCEDURE -
SUBJECT MATTER
JURISDICTION ANSWERS
CIVIL PROCEDURE - SUBJECT MATTER JURISDICTION ANSWERS 1.

CIVIL PROCEDURE - SUBJECT MATTER JURISDICTION ANSWERS

Answer to Question 1

(D) The court should apply the substantive law as determined by State D’s choice of law rules. Under
Erie, a federal district court sitting in diversity must apply the substantive law of the state in which
it sits. A state’s substantive law includes the choice of law rules of that state. When a case in a
federal court is transferred to a federal court in different state, the transferee court applies the
choice of law rules of the state in which the transferor court sits, provided that venue was proper
in the transferor court and the case is not being transferred due to a forum selection clause. If
venue was improper in the transferor court, then the choice of law rules of the transferee court
would apply. Here, venue was proper in the federal court for State D because the contract was
signed there, which is a “substantial part of the events or omissions giving rise to the claim.”
Thus, the federal court for State A (the transferee court) would apply the choice of law rules of
State D, where the transferor court sits. (C) is incorrect for the reasons explained above. (A) and
(B) are incorrect because under Erie, a federal court in diversity must apply the substantive law
(including the choice of law rules) of the state in which it sits. It may ultimately turn out that State
C law applies, but if it does, it will because State D law requires it under its choice of law rules.

Answer to Question 2

(D) The court lacks subject matter jurisdiction of the action because the lawyer’s state law claim is
supported neither by diversity of citizenship jurisdiction nor by alienage jurisdiction. Although
the woman is a citizen of the United States, she is not a citizen of any state of the United States.
A person is a citizen of the state or country where she is domiciled. The question tells us that the
woman moved from State B to a foreign country and has been living there for many years. Thus,
it is reasonable to conclude that she changed her domicile from State B to the foreign country.
However, the woman is not an alien; rather she is a United States citizen without a state citizen-
ship. (B) is incorrect because both the lawyer and the woman are citizens of the United States. A
United States citizen may become a citizen of a foreign country by renouncing her United States
citizenship and acquiring the citizenship or nationality of the foreign country by a formal process
called naturalization. Here, the woman is not an alien. (C) is incorrect because a court in State A
would have specific personal jurisdiction over the woman because she entered into a contract in
State A with the lawyer (her agreement to retain and pay him) and the lawyer’s claim arises out of
that contract.

Answer to Question 3

(D) The court has subject matter jurisdiction because a federal question has been presented. There are
two primary bases for federal subject matter jurisdiction: federal question jurisdiction and diver-
sity of citizenship jurisdiction. Federal question jurisdiction is available when the plaintiff, in his
well-pleaded complaint, alleges a claim that arises under federal law. Here, the worker alleges that
the employer violated federal employment discrimination statutes. Thus, the worker’s claim arises
under federal law. (A) and (B) are incorrect. Diversity of citizenship jurisdiction does not exist
because the employer is a citizen of State A (in which it must have its principal place of business)
and State B (in which it was incorporated), and the worker is also a citizen of State A, and also
because the worker’s claim is for less than $75,000. However, federal question jurisdiction does
not have an amount in controversy requirement and does not require complete diversity among
the parties. (C) inaccurately applies the citizenship requirement for a corporation, which may have
more than one state citizenship for diversity purposes, as explained above.
一 信:liuxue119118 , 们 信免 供

2. CIVIL PROCEDURE - SUBJECT MATTER JURISDICTION ANSWERS

Answer to Question 4

(A) The federal court does not have jurisdiction because neither diversity of citizenship jurisdiction
nor federal question jurisdiction exists. Diversity of citizenship jurisdiction is available when (i)
there is complete diversity of citizenship, meaning that each plaintiff is a citizen of a different
state from every defendant; and (ii) the amount in controversy exceeds $75,000. A natural
person’s citizenship is the state that is the person’s domicile. A partnership is a citizen of each
state of which its partners, both limited and general, are citizens. Here, the writer is a citizen of
State A, and the advertiser’s partners are citizens of State A, State B, and State C. Given the State
A-State A connection, complete diversity does not exist. Hence, (B) is an incorrect answer choice
as to diversity jurisdiction. Federal question jurisdiction is available when the plaintiff, in his well-
pleaded complaint, alleges a claim that arises under federal law. Anticipation of a federal defense
or the fact that federal law is implicated by the plaintiff’s claim do not give rise to federal question
jurisdiction; the plaintiff’s claim must arise under federal law. Here, although federal copyright
law is peripherally involved, the writer’s cause of action is actually based on state contract law. As
a result, no federal question has been presented by the writer’s complaint, making (C) incorrect.
Note too that federal question jurisdiction does not have a complete diversity requirement, making
(B) incorrect as to federal question jurisdiction. (D) is incorrect because federal question jurisdic-
tion does not arise merely because interstate commerce is affected.

Answer to Question 5

(A) The court has subject matter jurisdiction because a federal question has been presented. There
are two primary bases for federal subject matter jurisdiction: federal question jurisdiction and
diversity of citizenship jurisdiction. Federal question jurisdiction is available when the plaintiff,
in his well-pleaded complaint, alleges a claim that arises under federal law. Here, the consumer
alleges that the bank violated the federal truth in lending statutes in making the loan. Thus, the
consumer’s claim arises under federal law. (B) is incorrect because federal question jurisdiction
does not arise merely because interstate commerce is affected. (C) and (D) are incorrect. While
diversity of citizenship jurisdiction does not exist because the bank is a citizen of State A (in
which it was incorporated and has its principal place of business) and consumer is also a citizen
of State A, and because the amount in controversy is less than $75,000, federal question jurisdic-
tion does not have an amount in controversy requirement and does not require complete diversity
among the parties.

Answer to Question 6

(A) The federal district court does not have diversity jurisdiction because both parties are citizens of
State A. The court does not have federal question jurisdiction because the recently enacted federal
statute arises only in anticipation of the defendant’s defense. This is insufficient to confer federal
question jurisdiction. (Rationale: If, for example, the defendant relied on some other defense or
defaulted instead of defending on the basis of the new federal statute, no federal question would
ever be involved in the case. [See Louisville & Nashville Railroad v. Mottley, 211 U.S. 149
(1908)]) (B) is incorrect because citizenship of the parties is determined at the time suit is filed.
Assuming the $75,000 jurisdictional requirement could be met, the plaintiff would have had to
move prior to filing suit to be able to invoke diversity of citizenship jurisdiction. (C) is incorrect
because a federal question has not been presented, as described above. (D) is incorrect because,
even if the plaintiff’s claim exceeds $75,000, diversity of citizenship would not exist because the
plaintiff and defendant are both citizens of State A.
CIVIL PROCEDURE - SUBJECT MATTER JURISDICTION ANSWERS 3.

Answer to Question 7

(B) The defendant is a citizen of State A, and the plaintiff is a citizen of State B. Diversity is deter-
mined at the time the action is filed, not when the cause of action accrues or after the action
commences. Also, the plaintiff’s motive for moving to State B is irrelevant, as long as the change
of citizenship is genuine; i.e., she intends to remain there. The defendant’s motive is also irrel-
evant. Even though he moved to State B for reasons unrelated to the case and his move appears to
be enough to change his citizenship, citizenship is still determined at the time the action is filed,
and thus he is still considered to be a citizen of State A for purposes of this lawsuit.

Answer to Question 8

(C) The developer’s motion will be granted because diversity of citizenship is not present. In deter-
mining the citizenship of a limited partnership, the court will consider the citizenship of all
partners, both limited and general. Because the buyer and the four limited partners are citizens
of State A, adequate diversity does not exist. [See Carden v. Arkoma Associates, 494 U.S. 185
(1990)] (A) is incorrect because the citizenships of all partners, limited and general, are consid-
ered to determine if diversity exists. (B) is incorrect because diversity requires that all plaintiffs
be of diverse citizenships from all defendants. There is no requirement that defendants be of
diverse citizenships. (D) is incorrect because there is no such rule.

Answer to Question 9

(A) The action may be brought under diversity jurisdiction. In a class action brought under diver-
sity jurisdiction, only the citizenship of the named representatives is considered in determining
whether diversity is present. (C) is incorrect for that reason. (B) is incorrect. For a federal court to
have subject matter jurisdiction, diversity of citizenship jurisdiction or federal question jurisdic-
tion must be available, or there must be a statute providing for exclusive federal jurisdiction. (D)
is incorrect. The rule for diversity is that all plaintiffs must be of diverse citizenships from all
defendants. The fact that two plaintiffs may share a state citizenship would not defeat diversity
jurisdiction.

Answer to Question 10
(C) The federal district court has subject matter jurisdiction under diversity of citizenship jurisdiction.
Here, the plaintiff is a citizen of State A and the defendant is a citizen of State B, so the parties
are of diverse citizenship. Also, the jurisdictional amount for diversity jurisdiction is satisfied
because the total amount in controversy exceeds $75,000. A single plaintiff may aggregate several
unrelated claims against a single defendant to reach the amount in controversy minimum. (A) is
incorrect because the plaintiff may aggregate the claims, as discussed above. (B) is an incorrect
statement of law; a citizen of a state may be subject to suit in federal court in the citizen’s state.
(D) is incorrect because a federal question has not been presented.

Answer to Question 11

(D) The court will grant the husband’s motion. Even if diversity and the minimum amount in contro-
versy are present, the federal district courts will not exercise jurisdiction over domestic relations
or probate proceedings. For this reason, (A) is incorrect. (B) is incorrect because, even if State A
has a statute granting personal jurisdiction over the husband, that statute would not confer subject
matter jurisdiction over the case. (C) is incorrect. Citizenship is determined at the time suit is
filed; thus, the husband is a citizen of State B.
4. CIVIL PROCEDURE - SUBJECT MATTER JURISDICTION ANSWERS

Answer to Question 12

(C) If the court grants the motion, it will most likely be because it decided not to exercise supple-
mental jurisdiction over the passenger’s claim against the pilot. A defendant in the pilot’s position
is a “pendent party”; i.e., one who is involved in an action only with respect to a claim that does
not satisfy a basis of federal subject matter jurisdiction. (Here, there is no diversity or federal
question jurisdiction as to the claim against the pilot, and the claim is a state-law claim.) The
supplemental jurisdiction statute allows jurisdiction as to a claim that is transactionally related
to one over which the court has federal question jurisdiction. Note, however, that the exercise of
supplemental jurisdiction is discretionary with the court, taking into consideration whether the
pendent claims raise novel issues of state law, whether the claims based on state law predominate
over the federal law claims, or when the claims based on federal law have been dismissed. (A) is
incorrect because parties may be joined as defendants whenever: (i) some claim is made by each
plaintiff and against each defendant relating to or arising out of the same series of occurrences
or transactions; and (ii) there is a question of fact or law common to all the parties. Here, the
actions arise out the same collision and the question of who is at fault for the action would be the
common question of law or fact. (B) is incorrect because a basis for federal subject matter juris-
diction does exist. Supplemental jurisdiction exists over the claim (but the court has discretion to
decline jurisdiction). (D) is factually incorrect. For supplemental jurisdiction purposes, the two
claims arise from the same nucleus of operative fact (the same plane accident), and thus it would
be expected that the passenger would try them in the same action. For joinder purposes, the two
claims arise from the same series of transactions or occurrences.

Answer to Question 13

(A) The court should grant the motion because diversity of citizenship does not exist. Although the
general rule is that diversity is determined as of the time the action is commenced, the diversity
statute provides that the legal representative of the estate of a decedent is deemed a citizen of
the same state as the decedent. [28 U.S.C. §1332(c)(2)] Here, then, there is no diversity because
the deceased and the motorist were from the same state. (C) is thus incorrect. (B) is incorrect.
Although probate matters are not tried in federal court, this is an action for wrongful death, which
is not a probate matter. (D) is incorrect because it is too broad a statement. A plaintiff may choose
the forum, but that forum still must be able to validly assert personal jurisdiction over the defen-
dant.

Answer to Question 14

(C) The subcontractor may assert and maintain a claim against the developer in the pending action.
A third-party defendant is allowed to assert a claim against the plaintiff only if the claim arises
from the same transaction or occurrence as the plaintiff’s claim, which is the situation here. (B) is
therefore incorrect. (A) is incorrect because such claims by nonplaintiffs automatically fall within
the court’s supplemental jurisdiction, since the plaintiff’s claim and the third-party defendant’s
claim derive from a common nucleus of operative fact. (D) is incorrect because a third-party
defendant’s claim against the plaintiff is not compulsory.

Answer to Question 15

(C) The supplier can maintain the action. Diversity of citizenship jurisdiction requires complete diver-
sity—meaning that no plaintiff may share state citizenship with any defendant—and the amount
in controversy must exceed $75,000. Here, the issue is the citizenship(s) of the parties. For
CIVIL PROCEDURE - SUBJECT MATTER JURISDICTION ANSWERS 5.

purposes of diversity of citizenship jurisdiction, a corporation is deemed to be a citizen of every


state in which it is incorporated and the one state in which it has its principal place of business.
The supplier is a citizen of State A only, given that State A is both its state of incorporation and its
principal place of business. The aircraft manufacturer is a citizen of State B, its state of incorpora-
tion, and the state identified as its principal place of business. The United States Supreme Court
has held that the principal place of business is the corporate headquarters where the corporation’s
high level officers direct and control the corporation’s activities. Here, the aircraft manufacturer’s
headquarters are in State C, making State C its principal place of business despite the fact that its
manufacturing operations are in State A. (A) is not correct because, as explained above, a corpo-
ration’s principal place of business is the state from which the corporation’s high level officers
direct and control the corporation’s activities, not the state in which it has all of its physical opera-
tions. (B) is not correct because the place where the events took place has no role in diversity of
citizenship jurisdiction. (The events will determine where venue would be proper, but that is a
separate question from subject matter jurisdiction.) (D) is not correct because a corporation may
have two (or more) state citizenships, as explained above.

Answer to Question 16

(B) The court has supplemental jurisdiction. When jurisdiction is based on a “federal question” claim,
and diversity jurisdiction is not available, a nonfederal claim can be joined only if the court has
supplemental (pendent) jurisdiction over it. The court will have supplemental (pendent) juris-
diction over the claim if it arose from the same transaction or occurrence as the federal claim.
Here, although diversity jurisdiction is not available in the claim between the employee and the
co-worker because the amount in controversy does not meet the more than $75,000 jurisdictional
limit, the court will have supplemental jurisdiction because the hostile work environment claim
arose out of the same transaction or occurrence as the federal civil rights claim. (A) is wrong
because it is an incomplete statement of the law. While supplemental jurisdiction may exist
whenever the original claim is based on a federal question, to permit it, the state law claim also
must be part of the same transaction or occurrence. Choice (A) is incomplete because it requires
both elements for supplemental jurisdiction to exist. (C) is wrong because it presents an incorrect
statement of the rule, and intervention is not applicable to the facts of this question. As set forth
above, a nonfederal claim can be joined if the court has supplemental (pendent) jurisdiction over
it. Moreover, while intervention is not applicable to the facts, permissive intervention is avail-
able when the applicant’s claim or defense and the main action have a question of fact or law in
common; no direct personal or pecuniary interest is required. However, a claim in permissive
intervention is discretionary, and must not destroy complete diversity (if it does, intervention will
be denied), and must be supported by its own jurisdictional ground. (D) is wrong because it is
also a misstatement of the law, as set forth above.

Answer to Question 17

(A) The court should grant the motion. Under the Erie doctrine, when a state law-based claim is
brought in federal court based on diversity of citizenship, the federal court generally applies the
substantive law of the state in which it is sitting. However, where a specific federal statute or the
Federal Rules of Civil Procedure apply, the federal court must apply federal procedural law as
long as the federal rule is rationally deemed to be procedural and does not abridge, enlarge, or
modify any substantive rights. However, if there is no federal statute or rule on point, the federal
judge must determine whether the law on that issue is substantive or procedural. If it is a matter
of substance, the federal judge must follow state law in a diversity case. If it is a matter of proce-
dure, the federal judge may ignore state law. The Supreme Court has established that statutes of
6. CIVIL PROCEDURE - SUBJECT MATTER JURISDICTION ANSWERS

limitations and rules for tolling statutes of limitations are substantive for Erie purposes; there-
fore, a federal judge in a diversity case must follow state law on those issues. [Guaranty Trust Co.
v. York, 326 U.S. 99 (1945)] As a result, the State A law (including its choice of law principles)
will govern the statute of limitations. Since the law considers an action as “commenced” for the
purpose of the state’s statute of limitations when the defendant is served with the summons and
complaint, the statute of limitations has expired and the builder’s motion to dismiss should be
granted. (B) is wrong because it not only applies the wrong law to the facts, it also incorrectly
blends two distinct tests together. In cases where there is no federal directive on point and where
it is difficult to determine whether an issue is substantive or procedural for Erie purposes, the
Supreme Court has given different tests at different times, but has failed to integrate the tests
comprehensively. One such test is outcome determination, which holds that an issue is substan-
tive if it substantially affects the outcome of the case. [Guaranty Trust Co. v. York, supra] Another
test is forum shopping deterrence, which directs that the federal judge should follow state law on
the issue if failing to do so would cause litigants to flock to federal court. [Hanna v. Plumer, 380
U.S. 460 (1965)] Neither of these tests apply to the question because, as stated above, the Supreme
Court has established that statutes of limitations and rules for tolling statutes of limitations are
substantive for Erie purposes. (C) is wrong because, although it is a true statement of law, the
state law in this question is a substantive rule, not procedural. As such, the state law governs. (D)
is wrong because it is an incorrect statement of the law. As stated above, statutes of limitations
and rules for tolling statutes of limitations are substantive for Erie purposes; therefore, a federal
judge in a diversity case must follow state law on those issues.

Answer to Question 18

(D) The court may dismiss the remaining claim. The court may decline to exercise its supplemental
jurisdiction on one of four bases: (i) the case presents a novel or complex issue of state law; (ii) in
a federal question case, the nonfederal claim substantially predominates the case; (iii) the claims
over which the federal court had original jurisdiction have been dismissed (taking into consider-
ation the amount of time the court has spent on the case); or (iv) there are extraordinary circum-
stances for declining jurisdiction. Here, bases (i) and (ii) above would apply. (A) is incorrect,
because the court may dismiss the case even if it still has subject matter jurisdiction, as discussed
above. (B) is an incorrect statement of law. (C) is incorrect. The patent claim raised a federal
question and provided the court with federal question jurisdiction, and the claim based on the
unfair business competition statute arose from a common nucleus of operative fact as the federal
claim.
CIVIL PROCEDURE -
TRIAL MATTERS
QUESTIONS
CIVIL PROCEDURE - TRIAL MATTERS QUESTIONS 1.

CIVIL PROCEDURE - TRIAL MATTERS QUESTIONS

Question 1 Question 2

A homeowner from State A entered into a A driver collided with a bicyclist, severely
contract to sell his home to a woman from State injuring her. The bicyclist sued the driver in
B. When the homeowner refused to convey the the federal court for State A, properly invoking
property, the woman sued the homeowner for diversity of citizenship. In his answer, the driver
breach of contract in federal court based on raised the defense of contributory negligence,
diversity of citizenship. In her complaint, the which is a complete defense under the law of
woman sought $100,000 in damages and, alter- State A, where the accident occurred. The court
natively, specific performance of the contract. instructed the jury to return a general verdict
The woman did not include a demand for a with answers to written questions. In its written
jury trial in her complaint. The homeowner answers, the jury found that both the driver
timely served and filed his answer, denying and the bicyclist were negligent. Nonetheless, it
the principal allegations of the woman’s returned a general verdict awarding the bicyclist
complaint. Three weeks after the homeowner damages for her medical expenses and for her
filed his answer, the woman served an amended pain and suffering.
complaint, which restated her original claims
and this time included a demand for a jury trial. Which of the following options is NOT
available to the court to redress the inconsis-
Is the woman entitled to a jury trial? tency between the general verdict and the jury’s
finding of contributory negligence?
(A) No, because she waived her right to de-
mand a jury trial. (A) The court may dismiss the jury and order a
new trial.
(B) No, because specific performance is an
equitable remedy. (B) The court may enter a judgment in favor
of the driver, dismissing the plaintiff’s
(C) Yes, because the amended complaint was complaint.
served within 21 days of service of the
homeowner’s answer. (C) The court may direct the jury to further
consider its verdict and its answers to the
(D) Yes, because the issues raised by the court’s written questions.
complaint are primarily legal in nature.
(D) The court may enter a judgment in accor-
dance with the jury’s verdict, provided there
is a legally sufficient evidentiary basis for it.
2. CIVIL PROCEDURE - TRIAL MATTERS QUESTIONS

Question 3 Question 4

A homeowner hired a heating company A plaintiff filed a negligence action against


to install a new furnace in her house. A few a defendant in federal court, alleging that the
days after the installation, the furnace burst defendant caused a serious car accident in which
into flames and caused severe damage to the the plaintiff was injured. The defendant pleaded
house. The homeowner filed a lawsuit in federal contributory negligence as a defense. The court
court against the company to recover damages, decided to use a general verdict with special
alleging that the company was negligent. interrogatories in this case. The jury returned a
The homeowner filed a motion for summary verdict for the defendant, but its answers to the
judgment, supported by affidavits from two interrogatories were inconsistent with the verdict
neighbors who claimed that they overheard and showed that the jury had failed to follow the
heating company employees discussing that they court’s instructions when it came to contributory
had never before worked with the homeowner’s negligence.
type of furnace and were unsure how to install
it. Can the court set aside the verdict?

If the heating company objects to the (A) Yes, the court may either ask the jury to
homeowner’s affidavits, what is the likely reconsider its verdict or order a new trial.
reason?
(B) Yes, the court may order a new trial only.
(A) The affidavits do not set forth facts that
would be admissible in evidence. (C) No, the court must enter judgment
according to the jury’s general verdict
(B) The homeowner’s neighbors are not because a special verdict was not used.
credible witnesses because they have a
personal relationship with the homeowner. (D) No, a court can never set aside a jury’s
verdict.
(C) The homeowner should rely on only the
pleadings when moving for summary
judgment.

(D) A motion for summary judgment cannot be


supported by affidavits.
CIVIL PROCEDURE - TRIAL MATTERS QUESTIONS 3.

Question 5 Question 6

While driving a new car he recently A manufacturer sold to a consumer an expen-


purchased from an authorized dealer, a consum- sive laser printer that never worked properly.
er’s car caught fire for an unknown reason. The Therefore, the consumer never fully paid for
fire not only damaged the car, but also injured the printer. The manufacturer sued for specific
the consumer. The consumer filed a products performance of the contract of sale of the printer.
liability action against the manufacturer of the The consumer filed a counterclaim for a breach
car in federal district court, seeking to recover of warranty, asking for $85,000 in damages.
compensatory damages for his injuries. The The consumer demanded a jury trial, but the
complaint alleged that parts of the electrical manufacturer objected.
system in the car were defective and that the
defects caused the fire. The manufacturer filed Assuming that the demand for a jury trial
an answer that denied the existence of any was timely made, how will the court rule on the
defects and denied that any defects caused the availability of a jury trial?
fire, but stated that it lacked sufficient knowl-
edge and information to know what caused the (A) For the consumer, because the underlying
fire. During discovery, the consumer served an dispute is legal in nature.
interrogatory on the manufacturer that asked the
manufacturer to “identify and summarize all (B) For the consumer, because a defendant may
evidence of which the manufacturer [was] aware always request a jury trial.
that indicated that the fire was not caused by a
defect in the car.” The manufacturer’s response (C) For the manufacturer, because it filed suit
stated that it did not have, and was not aware first.
of, any evidence indicating that the fire was not
caused by a defect. Based on that interrogatory (D) For the manufacturer, because an action for
response, the consumer filed a motion for partial specific performance is equitable in nature.
summary judgment on the issue of causation to
establish that any fire was caused by defects.

How should the court rule on the motion?

(A) Grant the consumer’s motion for summary


judgment.

(B) Grant the consumer’s motion for summary


judgment unless the manufacturer files
affidavits or other evidence indicating that
the fire was not caused by a defect.

(C) The court should deny the consumer’s


motion, because it addresses an ultimate
issue in the case.

(D) The court should deny the consumer’s


motion, because, unless the consumer has
presented evidence that a defect caused
the fire, the manufacturer does not need to
present evidence regarding causation.
4. CIVIL PROCEDURE - TRIAL MATTERS QUESTIONS

Question 7 Question 8

A patient sued her doctor in federal court A storeowner was sued by a customer in
for medical malpractice. During the trial, the federal court for negligence after the customer
evidence heavily favored the doctor, but the slipped and fell on an alleged spill on the floor
jury returned a verdict in favor of the patient. and broke her neck. Immediately following
Judgment was entered for the patient on the close of discovery, the storeowner filed a
November 1. No motions were filed during the motion for summary judgment and argued that
course of the trial. The doctor believes that a the customer failed to present any evidence that
reasonable jury could not have found in favor of showed the storeowner breached his duty of
the patient and wants to set aside the verdict. On care. The customer responded by offering an
November 20, the doctor files a renewed motion affidavit from a witness who saw the fall. At
for judgment as a matter of law. trial, however, the witness did not testify. No
motions were made at trial, and the jury returned
Will the doctor’s motion be successful? a verdict for the customer. Thereafter, the store-
owner filed a motion for judgment as a matter of
(A) Yes, because he filed the motion within 28 law.
days of entry of judgment.
Will the court grant the motion for judgment
(B) Yes, because the evidence was heavily in as a matter of law?
his favor.
(A) Yes, because the court may deny a motion
(C) No, because he cannot use this motion for summary judgment, allow the case to
unless he moved for judgment as a matter proceed through trial, and still grant a mo-
of law at some time during the trial. tion for judgment as a matter of law on the
legal issues.
(D) No, because renewed motions for judgment
as a matter of law must be filed no later (B) Yes, because the storeowner previously filed
than 14 days after entry of judgment. a motion for summary judgment on the
same grounds, and the customer’s evidence
in response was not presented at trial.

(C) No, because the storeowner’s motion is


improper and will not be granted unless he
also moves for a new trial.

(D) No, because the storeowner did not move


for judgment as a matter of law during the
trial.
CIVIL PROCEDURE - TRIAL MATTERS QUESTIONS 5.

Question 9 Question 10

A landlord hired a builder to construct a A consumer properly sued a used car dealer-
10-unit apartment building on a single founda- ship for fraudulent misrepresentation and breach
tion. Within the first year after construction, the of contract in federal court. The case proceeded
floors and walls began to crack. The landlord through trial, and following closing arguments,
sued the builder in federal court, alleging that the judge presented the jury instructions to the
the builder was negligent in constructing the jury, but failed to give one of the jury instruc-
foundation. The builder denied those allega- tions requested by the consumer. The consumer’s
tions, and after a year of extensive discovery, the attorney, however, did not object. Instead, he
case went to a jury trial, where all issues were decided to wait and see how the jury would
litigated. decide the case. After deliberating, the jury
returned a verdict for the car dealership. The
After closing arguments, the judge presented consumer filed a timely appeal based on the
the jury with a verdict form to complete after judge’s failure to give the requested jury instruc-
deliberations. In addition to instructing the tion, alleging that this failure caused the jury to
jury to select which party would be granted render its verdict in favor of the car dealership.
the verdict, the form also asked several specific
questions regarding how the jury decided How will the appellate court likely rule?
specific facts in the case.
(A) Hear the consumer’s appeal, because the
With what form of verdict was the jury judge’s failure to give the requested in-
presented? struction amounted to plain error under the
“plain error doctrine.”
(A) A special verdict form.
(B) Deny the appeal, because the judge’s failure
(B) A general verdict form. to give the requested instruction amounted
to harmless error.
(C) A directed verdict form.
(C) Deny the appeal, because the consumer
(D) A general verdict form with interrogatories. waived her right to appeal when she did not
timely object to the judge’s failure to give
the requested instruction at trial.

(D) Deny the appeal, because once the jury


verdict was rendered, the jury instruction
issue was moot.
6. CIVIL PROCEDURE - TRIAL MATTERS QUESTIONS

Question 11 Question 12

An author of a book brought a copyright The plaintiff sued the defendant in federal
infringement action in federal court against a court for breach of contract. The plaintiff
publisher of her book and is debating on whether presented a detailed case in which she showed
she would like a jury to try her case. the existence of the contract, its breach, and
damages. The defendant then presented his
Which of the following statements is correct defense, which consisted solely of evidence that
regarding the right to a jury trial in a civil action he regretted dealing with the plaintiff. At this
in federal district court? point in the trial, the plaintiff feels she should
prevail.
(A) A properly filed and served jury trial de-
mand may be withdrawn if it is done before What should she move for at this point in the
the last pleading is served. litigation?

(B) A proper jury trial demand may not be (A) For a judgment on the pleadings.
withdrawn once it is served and filed.
(B) For a summary judgment.
(C) The right to a jury trial must be preserved,
and therefore, it cannot be waived. (C) For a judgment as a matter of law.

(D) A party will waive its right to a jury trial (D) For a renewed judgment as a matter of law.
unless it is demanded properly and timely.

一 信:liuxue119118 , 们 信免 供
CIVIL PROCEDURE - TRIAL MATTERS QUESTIONS 7.

Question 13 Question 14

A truck collided with a car in State A, A car owner sued a crane operator for negli-
injuring the driver of the car. The driver of the gence in a federal court because the crane
car filed a civil action in federal district court in operator backed over and crushed her car.
State A against the trucking company to recover During jury selection, the car owner’s lawyer
damages for the driver’s injuries. The trucking wished to exercise one of her peremptory
company filed a motion to dismiss the action challenges to excuse some potential jurors.
against it on the grounds that the court in State
A did not have personal jurisdiction over it. Which of the following reasons would provide
Although the driver of the truck was the compa- the best grounds for the crane operator’s lawyer
ny’s employee, the trucking company argued that to object to the peremptory challenge?
the driver did not have authorization to drive the
truck to State A. Following a hearing, however, (A) Excusing the only potential male juror
the court ruled that the trucking company was because he is a crane operator, just like the
subject to the court’s personal jurisdiction. defendant.

Another two months passed, and the trucking (B) Excusing a second African-American juror.
company did not file an answer. The driver of
the car then filed a motion asking that the clerk (C) Excusing the juror with multiple tattoos.
of court make an entry of default, and the clerk
did so. (D) Excusing the juror who has been
unemployed for two years.
What procedure must the driver follow to
obtain a default judgment against the trucking
company?

(A) File a motion to have the clerk of court


enter the default judgment, which the clerk
may do without the trucking company re-
ceiving any further notice of the motion.

(B) File a motion to have the clerk of court


enter the default judgment, which the clerk
may do as long as the trucking company
receives additional notice of the motion for
default judgment.

(C) File a motion to have the judge enter the


default judgment, which the judge may do
without the trucking company receiving
any further notice of the motion for default
judgment.

(D) File a motion to have the judge enter the


default judgment, which the judge may do
as long as the trucking company receives
additional notice of the motion for default
judgment.
CIVIL PROCEDURE -
TRIAL MATTERS
ANSWERS
CIVIL PROCEDURE - TRIAL MATTERS ANSWERS 1.

CIVIL PROCEDURE - TRIAL MATTERS ANSWERS

Answer to Question 1

(A) The woman is not entitled to a jury trial because she failed to demand it within 14 days after she
was served with the homeowner’s answer. Under Federal Rule 38, a party who desires a jury trial
(on some or all fact issues) must file a written demand with the court and serve it on the parties.
Failure to make such a demand within 14 days after the service of the last pleading directed to
the jury-triable issue constitutes a waiver by that party of any right to trial by jury. Here, the last
pleading on the issue was the answer, and more than 14 days elapsed before the woman demanded
a jury trial. (B) is incorrect because the woman would be entitled to a jury title on her legal
claim for damages even though she also had an equitable claim for specific performance. If legal
and equitable claims are joined in one action involving common fact issues, the legal claim is
tried first to the jury and then the equitable claim to the court. The jury’s finding on fact issues
will bind the court in the equitable claim. (C) is incorrect because, as previously discussed, the
woman’s right to demand a jury trial expired 14 days after she was served with the homeowner’s
answer. Although the woman could amend her complaint as of right within 21 days of service of
the homeowner’s answer, she did not have the right to include in it a demand for jury trial. (D)
is incorrect because the homeowner’s right to a jury trial on legal claims does not depend on
whether the issues are primarily legal.

Answer to Question 2

(D) Under the Federal Rules, when the jury’s answers accompanying a general verdict are consistent
with each other but are inconsistent with the general verdict, the court may (i) enter a judgment
that is consistent with the answers, disregarding the general verdict; (ii) instruct the jury to delib-
erate further regarding its verdict and answers; or (iii) order a new trial. These options are set out
in choices (A), (B), and (C). It may not, however, enter a judgment in accordance with the verdict
itself, which is set out in choice (D).

Answer to Question 3

(A) If the heating company objects to the homeowner’s affidavits, it is because the affidavits do not set
forth facts that would be admissible in evidence. Affidavits used to support motions for summary
judgment must (i) be made on personal knowledge; (ii) set forth such facts that would be admis-
sible in evidence; and (iii) show the affiant is competent to testify. Here, the affidavits contain
hearsay and therefore likely would not be admissible in evidence. (B) is incorrect because the
credibility of witnesses is not relevant to whether their affidavits can be used to support a motion
for summary judgment. (C) and (D) are incorrect because a motion for summary judgment can be
supported by discovery materials, including affidavits or other declarations made under penalty of
perjury. In fact, the general rule is that a party may not rely on pleadings to support the motion or
show that a genuine dispute exists (unless the pleadings contain admissions or are verified).

Answer to Question 4

(A) The court may set aside the verdict and ask the jury to reconsider the verdict or order a new trial.
When a verdict shows on its face that the jury failed to follow the court’s instructions, the verdict
may be set aside, and either the jury will be asked to reconsider its verdict or a new trial will
be ordered. (D) is therefore incorrect. (C) is incorrect because the court does not need to use a
special verdict to take this action. (B) is incorrect because the court can ask the jury to reconsider
its verdict as well.
2. CIVIL PROCEDURE - TRIAL MATTERS ANSWERS

Answer to Question 5

(D) The court should deny the consumer’s motion. The plaintiff must prove the elements of the prima
facie case for her claim. Absent proof of an element of the prima facie case, summary judgment
for the plaintiff is not appropriate. Thus, (A) is incorrect and (D) is correct. Furthermore, although
a party is generally required to respond to a motion for summary judgment with affidavits, the
facts here indicate that the consumer has not come forward with any evidence pertaining to the
manufacturer’s fault for the damage and injury. Thus, because the consumer has not properly
supported his motion for summary judgment with relevant, admissible information, there is no
need for manufacturer to produce any evidence to survive a motion for summary judgment. This
makes (B) incorrect. (C) is an incorrect statement of the law. A motion for summary judgment
may be granted even though the motion addresses an ultimate issue in the case.

Answer to Question 6

(A) The court will grant a jury trial because the underlying dispute is legal in nature. The court will
look to the basic substance of the case to see if a jury trial is appropriate. Although the manufac-
turer’s suit is equitable in nature, the consumer’s counterclaim for breach of warranty is an action
at law, in which a jury trial is available on demand. Thus, (D) is incorrect. (B) is an incorrect
statement of law, and (C) is incorrect because filing suit first would not guarantee a jury (or
nonjury) trial.

Answer to Question 7

(C) The doctor’s motion will not be successful because a party can only file a renewed motion for
judgment as a matter of law if he filed a motion for judgment as matter of law at some point
during the trial. The doctor did not do that here, and therefore the renewed motion for judgment
as a matter of law is not available to him. (A) and (B) are incorrect because the renewed motion
for judgment as a matter of law is unavailable to the doctor even though he filed the motion within
the proper timeframe and even though the evidence was in his favor. (D) is incorrect because
renewed motions for judgment as a matter of law must be filed no later than 28 days after entry of
judgment.

Answer to Question 8

(D) The court will not grant the motion for judgment as a matter of law. A motion for judgment as
a matter of law (“JMOL”—formerly known as a motion for a directed verdict) may be made by
any party any time before submission of the case to the jury. Here, the storeowner needed to
file this motion before the case was submitted to the jury. If the storeowner had done so, and the
motion was denied, and the jury returned the verdict for the customer, the storeowner could then
file a renewed motion for judgment as a matter of law (formerly known as a judgment notwith-
standing the verdict—“JNOV”). However, because the storeowner failed to file a JMOL before the
case went to the jury, the motion will be denied. (A) is wrong because, although a court may deny
a motion for summary judgment, allow the case to proceed through trial, and still grant a motion
for judgment as a matter of law on the legal issues, it is called a renewed motion for judgment as
a matter of law. Moreover, it will only be permitted if a motion for judgment as a matter of law
was filed before submission of the case to the jury, which did not happen in this case. (B) is wrong
because it incorrectly states that the motion for summary judgment is the precursor to obtaining
a renewed motion for judgment as a matter of law, which is not the case, as set forth above. (C) is
wrong because it is an incorrect statement of law. A party may file a renewed motion for judgment
CIVIL PROCEDURE - TRIAL MATTERS ANSWERS 3.

as a matter of law and a motion for a new trial in the alternative, but it is not required. Note that
if a party fails to move for either a renewed judgment as a matter of law or for a new trial on the
basis of insufficiency of the evidence, that party is precluded from raising the question of eviden-
tiary sufficiency on appeal, to support either judgment as a matter of law or a new trial.

Answer to Question 9

(D) The jury was presented with a general verdict form with interrogatories. In such a case, the jury is
asked to give a general verdict and also to answer specific questions concerning certain ultimate
facts in the case. The purpose is to ensure that the jury properly considered the important issues.
Interrogatories must be submitted with the general verdict to test the verdict’s validity. [See Fed.
R. Civ. P. 49(b)] Here, the verdict form the judge presented to the jury is a general verdict with
interrogatories because it asked the jury to decide ultimately which party prevailed, but also asked
specific questions regarding how the jury decided certain facts to test the verdict’s validity. (A)
is wrong because it incorrectly classifies the verdict form. In a special verdict, a jury is asked to
make a finding on all material conclusions of fact, and the court applies the law. The procedure
for a special verdict is to submit to the jury a series of questions regarding each ultimate fact. The
court then makes legal conclusions based on those facts. [See Fed. R. Civ. P. 49] Here, although
the verdict form did ask specific questions, it also asked the jury to render the verdict, instead of
having the court apply the law and reach the legal conclusion. As such, it is not a special verdict
form. (B) is wrong because, in a general verdict, the jury finds for the plaintiff or defendant and
gives the amount of damages or relief due. Here, the jury was asked to render a verdict for the
party, but was also asked specific questions of facts. As such, it was a general verdict with inter-
rogatories. (C) is wrong because a directed verdict is issued by the judge, not the jury, before the
case is submitted to the jury. Moreover, this type of verdict is no longer called a “directed verdict.”
It is now called a “judgment as a matter of law.” Clearly, this does not apply to the facts, as the
case was submitted to the jury.

Answer to Question 10

(C) The court will likely deny the appeal. If a party does not give the trial judge a chance to consider
and rule on an issue, the party generally cannot appeal the issue. Procedural mistakes (i.e.,
matters other than legal theories) are asserted by parties by raising a timely objection at trial.
Failure to timely object waives the right to raise that issue on appeal. Here, failure to present the
requested jury instructions is a procedural error, and the failure of the consumer’s attorney to
object to the issue at trial waived the issue from being raised on appeal. (A) is wrong because the
court of appeals will not hear the case for the reason stated above, and the procedural mistake
does not amount to plain error. Although some jurisdictions have an exception to the objection
requirement and do not require an objection in the case of “plain error,” an error only consti-
tutes “plain error” when it is so blatantly incorrect that it calls into question the competence or
partiality of the judge. Such errors are very rare, and failing to give a requested jury instruc-
tion is a procedural mistake and does not rise to the level to constitute plain error. (B) is wrong
because, although it reaches the correct conclusion that the appeal will be denied, it is for the
wrong reason. “Harmless error” is the term used to describe an error that is made but “does
not affect the substantial rights of the parties.” In other words, the mistake does not affect the
outcome of the case. Such errors cannot be the basis of a reversal on appeal. Here, failure to give
the requested jury instruction may in fact have amounted to harmless error (no facts are given to
know); however, it is not relevant because the consumer’s failure to timely object to this error is
what prevents the court of appeals from reviewing this issue. (D) is wrong because it is an incor-
rect statement of law. The procedural mistake is not moot simply because the jury rendered a
4. CIVIL PROCEDURE - TRIAL MATTERS ANSWERS

verdict. Instead the consumer failed to preserve the issue for appellate review by failing to timely
object to it.

Answer to Question 11

(D) A party will waive its right to a jury trial unless it is demanded properly and timely. Under Rule
38 of the Federal Rules of Civil Procedure, a party who desires a jury trial (on some or all fact
issues) is required to file a written demand with the court and serve it on the parties. Failure to
make such a demand within 14 days after service of the last pleading directed to the jury-triable
issue constitutes waiver by that party of any right to trial by jury. (A) and (B) are wrong because
they are incorrect statements of the law. Instead, a jury demand may be withdrawn only if all
parties consent. (C) is also wrong because it is an incorrect statement of law. While it is true that a
party must file a written demand to preserve the right to a jury trial, it is optional. If the party fails
to make such a timely demand, such failure constitutes waiver by that party of any right to trial by
jury.

Answer to Question 12

(C) At this point in the litigation, when the opposing party has been fully heard, the appropriate
motion is for judgment as a matter of law (formerly known in federal practice as a motion for
directed verdict). The evidence will be viewed in the light most favorable to the party opposing
the motion, and will be granted if there is no legally sufficient evidentiary basis for a reasonable
jury to find in favor of that party. Motions (A) and (B) are pretrial motions: a motion for judgment
on the pleadings considers no evidence outside the complaint, answer, etc., while a motion for
summary judgment considers affidavits, discovery material, and other evidence submitted with
the motion. (D) is incorrect because a renewed motion for a judgment as a matter of law is made
after the jury returns a verdict. To make a renewed motion for judgment as a matter of law, one
must have moved for a motion for judgment as a matter of law at the close of all of the evidence.

Answer to Question 13

(D) The driver must file a motion to have the judge enter the default judgment. A defendant against
whom a default is entered loses the right to contest liability unless the entry of default is set aside.
However, the amount of damages must still be determined before a default judgment may be
entered, and the defaulting party can be heard at the hearing for damages. If the defendant has
“appeared,” even though he has not answered, he must be notified of the request for a default
judgment by first-class mail at least seven days before the hearing on the application for a default
judgment. Appearance includes any actual formal appearance before the court and any other
action that clearly indicates that the defendant intends to contest the case on the merits. Here, the
trucking company made an appearance when it challenged the court’s jurisdiction. (A) and (B)
are incorrect because the clerk may enter a default judgment only in very limited cases. When
the amount of damages is not for a sum certain, the judge must hold a hearing to determine the
damages. That is the case here because there are personal injuries involved. (A) and (C) are incor-
rect because notice is required when the defendant has appeared in the case in some fashion.

Answer to Question 14

(B) Peremptory challenges allow an attorney to disqualify a potential juror because the juror displays
an attitude or some characteristic that appears unfavorable to the attorney’s client but that does
not rise to the level of bias that would present grounds for a challenge for cause. A party cannot
CIVIL PROCEDURE - TRIAL MATTERS ANSWERS 5.

use peremptory challenges if the court suspects the challenge is for race, national origin, religion
or gender, which violate the juror’s equal protection rights under the Fourteenth Amendment. If
the court detects a pattern of excluding jurors for any of these reasons, the opposing party can
object, and the party may be required to justify the challenge by providing other nonobjection-
able means. Here, exercising a peremptory challenge to excuse a second African-American juror
from the panel without any other reason appears that it may be based on race. Therefore, choice
(B) provides the best choice for the defense to object to the plaintiff’s peremptory challenge. (A) is
wrong because, although there may be an argument that this is a gender-based challenge, bias is a
better but nonobjectionable reason behind striking this juror in terms of a peremptory challenge.
Since both the defendant and the juror are crane operators and the negligence claim is based on
negligence in operating the crane, the juror may be biased in favor of the defense. While a poten-
tial bias raises grounds for a challenge for cause, this question specifically asked for grounds to
object to the peremptory challenge, not a challenge for cause. Therefore, choice (B) more directly
and correctly answers the question asked. (C) and (D) are wrong because neither is based on race
or gender grounds.
一 信:liuxue119118 , 们 信免 供

CONSTITUTIONAL LAW -
FIRST AMENDMENT
FREEDOMS QUESTIONS
CONSTITUTIONAL LAW - FIRST AMENDMENT FREEDOMS QUESTIONS 1.

CONSTITUTIONAL LAW - FIRST AMENDMENT FREEDOMS QUESTIONS

Question 1 solicitation isfreedom of Question 2

In response to speech
whichis
a tremendous increase in A city ordinance prohibits the distribution of
begging in the downtown area of a city, the pamphlets “on public sidewalks or other public
city council enacted an ordinance that required areas when foot traffic is sufficiently heavy and
anyone soliciting for charitable contributions of the manner of distribution of the pamphlets
any sort in any public place to wear an identity causes obstruction of the foot traffic so as to
card issued by the local police department. result in spillover onto public streets where
Identity cards could be obtained by completing vehicular traffic creates a danger to human
an affidavit providing the applicant’s identi- life.” The state fair is held at fairgrounds whose
fication and address information and further entrances lie along a busy multilane street.
affirming that the applicant was not soliciting A demonstrator was distributing pamphlets
for personal use and belonged to a recognized advocating repeal of the federal milk price
charitable organization. support program at the state fair. He attracted a
crowd of about 10 farmers and children outside
A member of an anti-tobacco charitable the fairgrounds entrance at which he stood, but
organization wishes to solicit contributions by most of the few fairgoers entering the fair at
similarly minded persons for use in his organi- that late afternoon time simply ignored him.
zation’s campaign against public smoking. He When one of the dairy farmers became irate and
does not want to comply with the identity card threatened to “knock his block off,” the demon-
ordinance. He comes to you for legal advice and strator was arrested by a fair security guard
asks whether he should challenge the ordinance and subsequently prosecuted under the city
in federal court. ordinance.

Should you advise that the ordinance is consti- Which of the following statements is correct
regarding the city ordinance and the demonstra-
E A
tutional?
tor’s prosecution?
(A) No, because it violates the First Amend-
ment’s prohibition of government infringe-
ment of the right of free speech.
AB
(A) The ordinance is void on its face and void
as applied to the demonstrator.

(B) No, because it prevents religious organiza- (B) The ordinance is valid on its face but void
tions from obtaining contributions from as applied to the demonstrator.
their members, and thus interferes with the
free exercise of religion. (C) The ordinance is valid on its face and valid
as applied to the demonstrator.
(C) Yes, because it represents a reasonable
balancing of the state’s police power (D) The ordinance is void on its face but valid
interest in protecting its citizens from fraud as applied to the demonstrator.
and annoyance against the right of people
to seek charitable contributions.
tteEa8b
(D) Yes, because preventing Efx
EE hiEfraud in the
solicitation for charitable contributions is a
compelling interest.
2. CONSTITUTIONAL LAW - FIRST AMENDMENT FREEDOMS QUESTIONS

Question 3 Question 4

To discourage the consumption of alcohol, A husband and wife were both professors at
a city adopted an ordinance prohibiting any the United States Naval Academy. The husband
advertisements of liquor prices except within made a speech criticizing United States foreign
the stores in which alcoholic beverages are sold. policy with respect to a Middle Eastern country
After the ordinance was adopted, a store located and was dismissed from his teaching position
within the city that had regularly advertised soon thereafter. Six months later, he accepted
its liquor prices experienced a 30% decrease new employment in another state.
in sales. The owner was not given personal
notice of the city council meeting at which The man’s wife has commenced suit in federal
the ordinance was adopted. If he had been, as court claiming that the Naval Academy violated
the owner of the city’s largest volume dealer her husband’s right to due process and his right
of alcoholic beverages, he would have spoken of free speech when it fired him.
against the adoption of the ordinance.
What is the most likely ground on which the
If the store owner brings suit to have the court will dismiss the suit?
ordinance declared unconstitutional, how should
the court rule? BO
(A) The political question doctrine, because
D
(A) Uphold the ordinance, because states have
foreign policy is in the domain of the ex-
ecutive branch.
the power to ban and regulate liquor sales
under the Twenty-first Amendment. (B) The case is moot, because the husband has
a new job.
(B) Uphold the ordinance, because the ban does
not favor local economic interests or unduly (C) The plaintiff has no standing.
interfere with interstate commerce. I
(D) There is no federal question presented.
(C) Declare the ordinance unconstitutional
as applied to the store owner, because the
failure to give the owner notice violated
the Fourteenth Amendment Due Process
Clause.

(D) Declare the ordinance unconstitutional,


because it amounts to an absolute ban on For font issueforthemself
advertising a lawful product in violation of
party his
the First Amendment.
hot for third
wife
CONSTITUTIONAL LAW - FIRST AMENDMENT FREEDOMS QUESTIONS 3.

Question 5 Question 6

O
A privately owned bus company operated
over fixed city routes under a franchise granted
A state passed a law requiring that anyone
holding himself out to be a private investigator in
to it by the city council. To enhance revenues,
the bus company rented advertising space on its
O
the state must be licensed by the state. Licensure
requirements included a thorough background
vehicles. Although the city received a certain check into the person’s criminal record and
fixed percentage of the fare revenues from the mental health. It also required passing a test
company, it received nothing from the advertising on ethical obligations of a private investigator.
fees. The management of the bus company had Finally, the investigator was required to sign a
learned through bitter experience that any adver- two-part oath. Part one was a loyalty oath, which
tising that could be characterized as “controver- stated: “I solemnly swear (or affirm) that I will
sial” led to complaints from riders and sometimes be loyal to the United States and to the state and
to losses in revenue. To that end, management will uphold their Constitutions.”
Te Part two stated:
employs a firm “no controversial advertising” “I solemnly swear (or affirm) that I am not now
policy. Thus, they refused to take ads for X-rated a member of any organization that advocates
movies, abortion clinics, political candidates, and illegal acts, nor will I become a member of any
anything else that might be deemed “controver- such organization while I am a licensed private
sial.” A candidate for city council was waging investigator in this state.”
a populist campaign for the job and among
his targets were the local utilities and the bus An experienced investigator with a master’s
company. The candidate demanded that he be degree in criminal justice administration applied
allowed to buy advertising on the buses, but the for a private investigator’s license. He easily
company’s advertising manager refused, citing passed both background checks, but he refused
the company ban on all political ads. The candi- to take the oaths, claiming that they inhib-
A
date filed suit in federal district court, asking that
the court require the company to accept his ads.
ited his freedoms of speech and association as
guaranteed by the federal Constitution. The state
professional licensure board denied him a private
Which of the following is the company’s best investigator’s license solely on the basis of his
argument against the candidate’s constitutional refusal to take the oaths. The investigator sued in
claim? federal court to require the state to grant him a
a
(A) The company is a privately owned com-
license and to strike down the oath requirements
in the licensure statute.
pany, and thus there is insufficient “state
action” to invoke the United States Consti-

Bob
What is the court’s likely ruling?
tution.

Y
(A) Both the loyalty oath and the membership
(B) Commercial speech receives less protection oath are constitutional.
under the First Amendment than most other
forms of speech. (B) The loyalty oath is constitutional, but the
(C) Bus riders have First and Fourteenth membership oath is unconstitutional.
Amendment rights to see the type of adver-
(C) The membership oath is constitutional, but
tising they desire, and past experience has
the loyalty oath is unconstitutional.
shown that there is substantial opposition
by riders to controversial ads. (D) Neither the loyalty oath nor the membership
(D) The candidate has many other sources of oath is constitutional.
advertising available and he does not need
ads on the company’s vehicles to get his
message to the public.

yo
4. CONSTITUTIONAL LAW - FIRST AMENDMENT FREEDOMS QUESTIONS

Question 7 Question 8

A city passed an ordinance prohibiting door- A state statute prohibited individuals from
to-door solicitation of contributions by chari- donating more than $1,000 per year to any

f
table organizations that did not use at least 75% group that lobbies for or against any matter up
of their receipts for “charitable purposes.” The for consideration before the state legislature. A
ordinance further provided that anyone wishing voter who wanted to donate $5,000 to a lobbying
to solicit for purposes of charity must obtain a group challenged the statute on constitutional
permit and present satisfactory proof that at least grounds in federal court.
75% of the proceeds of such solicitation will be

__
used directly for the charitable purposes of the
organization. An organization seeking to solicit
Is the court likely to uphold the statute?

(A) Yes, because the statute is reasonably


o
support within the city applied for and was
denied a permit because it did not meet the 75% related to the state’s legitimate interest in
requirement. controlling such contributions.

As applied to the organization, is the (B) Yes, because the statute does not restrict the

c
ordinance constitutional? core political speech right to donate directly
to legislative candidates.
(A) Yes, because the 75% rule serves a legiti-
mate state interest in preserving the integ- (C) No, because the statute places a restraint on
rity of charities. core political speech and association rights
without sufficient justification.
(B) Yes, because the right to solicit for a charity
is balanced against the interest of the state (D) No, because a state may not place any
in preventing fraud and crime. limits on the amount of money that may be
contributed to political campaigns.
(C) No, because the ordinance violates the
protections afforded by the First Amend-
ment.

(D) No, because the ordinance has as a purpose


the protection from undue annoyance and
the preservation of residential privacy.
CONSTITUTIONAL LAW - FIRST AMENDMENT FREEDOMS QUESTIONS 5.

Question 9 Question 10

An author who was about to release a book To protect ready access to its capitol building,
on military censorship in war zones was inter- a state statute provides:
viewed by a newspaper reporter. During the
interview, the author spoke generally of military The distribution and sale of any writ-
censorship in war zones, but would not give the ten material within 100 feet of the
reporter any specific instances of censorship State Capitol Building shall be prohib-
that he intended to include in his book. During ited except where:
the interview, the author received a phone call
and excused himself. While the author was out (i) The person or organization intend-
of the room, the reporter found a prepublica- ing to distribute the material first ob-
tion copy of the author’s book and quickly took tains a permit from the State Admin-
photographs of several pages. The reporter later istrator’s Office, which shall be issued
printed the pictures and published an article at no charge to the first 10 persons or
that included the photographed sections of the organizations requesting a permit each
book verbatim. The author brought an appro- day, and
priate action against the reporter for copyright (ii) The person or organization does
infringement. The reporter defended on First not distribute or sell written materials
Amendment grounds. Assume the court properly within 10 feet of any entrance to the
concludes that any fair use exception in the State Capitol Building.
copyright laws is not applicable.
A state citizen who was unhappy with a law
Is the reporter’s defense likely to be that was recently enacted went to the main
successful?
D
(A) Yes, because the book included matters of
entrance of the capitol building and handed
out leaflets criticizing the law. He was soon
approached by the building’s security guards,
public concern, and so the reporter had a who asked to see his permit. When he was
right under the First Amendment to publish unable to produce one, they cited him for
it. leafleting without a permit.

c
(B) Yes, because newspapers have an absolute If the citizen challenges his citation on the
right under the First Amendment to print basis that the permit statute is unconstitutional
whatever information they receive. under the First Amendment, how should the
court rule?
(C) No, because the author has a Fifth Amend-
ment property right in his book. (A) The statute is unconstitutional because 10
permits a day is too few.
(D) No, because the newspaper did not get the
author’s permission to print the pages from (B) The statute is unconstitutional because 10
his book. feet from a doorway is unreasonable.
(C) The statute is constitutional because it is
content neutral, narrowly tailored to serve
an important government interest, and
leaves open alternative channels of commu-
nication.
(D) The statute is constitutional because the
grounds and walkways of the capitol
building are not public forums.
6. CONSTITUTIONAL LAW - FIRST AMENDMENT FREEDOMS QUESTIONS

Question 11 Question 12

A city council ordinance provided that city To protect the minor children living in the
officials should automatically issue a parade area, a town council enacted an ordinance that
permit to any group filing the proper papers with prohibited advertisements that include a depic-
city authorities, except in situations where a prior tion of a nude person, whether male or female.
group had already received permission to parade The owner of a local video rental shop often
on the same street at the same time on the same posts large posters to advertise movies he has
day. Another city ordinance prescribed fines for available for rent. A substantial part of his
persons conducting a parade in the city without business consists of the rental of adult movies,
a permit. which are kept in a separate part of the shop. In
the adult room he hangs movie advertisements,
A religious leader filed appropriate papers many of them depicting nude or partially nude
with city officials to parade down the main street people. He does not allow minors to enter the
of the city at 1 p.m. on July 15. City officials adult room.
noted that they had already issued a parade
permit for another parade at the same place, If the owner challenges enforcement of the
date and time. The officials told the leader that ordinance against him, how will the court most
he could not have a parade permit for the time likely rule?
and place requested and suggested that he select
another day and/or location. The leader refused (A) For the town, because the ordinance is a
and insisted that he would conduct his parade valid exercise of the town’s power to pro-
when scheduled, even without a permit. On tect the morals of its minor citizens.
July 15 at 1 p.m., both organizations assembled
on the main street and began to organize into (B) For the town, because the posters may
parades. City officials asked the religious leader appeal to a minor’s prurient interest in sex.
and his followers to desist, but they refused and
were thereafter arrested, convicted, and fined (C) For the owner, because prohibiting the
under the city ordinance. posting of the movie advertisements
violates his First Amendment rights.
If the convicted members seek to have their
convictions overturned by the federal courts, (D) For the owner, because not all nudity is
they will: obscene.
(A) Lose, because the religious leader should
have gone to federal court to secure his
organization’s rights before violating the
ordinance.
(B) Lose, because the city ordinances are a
reasonable restriction on time, place, and
manner of speech and were not applied in a
discriminatory manner.
(C) Lose, because the city ordinances repre-
sented the will of the people as expressed
through the city council.
(D) Win, because the city ordinances, on their
face, violate the free speech guarantees of
the First Amendment.
CONSTITUTIONAL LAW - FIRST AMENDMENT FREEDOMS QUESTIONS 7.

Question 13 Question 14

A city ordinance makes it unlawful for any The legislature of a state passed a statute
group of individuals or organization in excess providing that “no newspaper in this state shall
of 20 persons to demonstrate, march, or picket publish a political endorsement or an edito-
in the city’s civic center without first posting a rial favoring one political candidate or party
bond with the police department and receiving over another on either the day of election or
a permit. The permit procedure takes at least the day preceding the day of election.” The
one working day, and costs $10. In addition stated purpose for the legislation was to prevent
to making a violation a misdemeanor, the unfair attacks on candidates and to ensure that
ordinance authorizes the police department to they would have time to respond to or rebut
terminate any demonstration if “any person in any published article prior to the election. A
the demonstration, without provocation, uses, newspaper published in the state, which was
in the presence of other persons not a party to distributed regionally and in states bordering the
the demonstration, annoying, disturbing, oppro- state, filed suit in federal court seeking to enjoin
brious words and abusive language in such enforcement of the statute.
a manner as tending to cause a breach of the
peace.” The newspaper’s best argument for the
invalidity of the statute is which of the
A group of demonstrators brings suit in the following?
state court to enjoin the city from preventing
their scheduled demonstration on Memorial Day (A) The statute unduly burdens interstate com-
without a permit, and to enjoin the city from merce, because the newspaper is circulated
using this ordinance to require them to have a in other states.
permit.
(B) The statute unduly interferes with the
The demonstrators’ strongest contention for newspaper’s property interest in distributing
finding the provisions of this statute unconstitu- newspapers.
tional is that:
(C) The statute unduly restricts the newspaper’s
(A) The city’s civic center is a place where freedom of speech.
demonstrations of this type normally occur,
and the city cannot prevent citizens from (D) The statute violates the Equal Protec-
demonstrating there. tion Clause, because restrictions are
imposed merely on the basis of the day
(B) There is no showing by the city that the the newspaper is printed, while the same
demonstrators are likely to become disrup- material can be printed on other days.
tive or unruly.

(C) The ordinance is overbroad and unduly


vague.

(D) The First and Fourteenth Amendments


ensure the right of association in public
places without interference.
CONSTITUTIONAL LAW -
FIRST AMENDMENT
FREEDOMS ANSWERS

一 信:liuxue119118 , 们 信免 供
CONSTITUTIONAL LAW - FIRST AMENDMENT FREEDOMS ANSWERS 1.

CONSTITUTIONAL LAW - FIRST AMENDMENT FREEDOMS ANSWERS

Answer to Question 1

(A) The ordinance is probably unconstitutional because it violates free speech rights under the First
Amendment. The Supreme Court has held that a charitable appeal for funds involves a variety
of speech interests protected by the First Amendment. In one case, an ordinance that prohibited
door-to-door solicitation by organizations that did not use at least 75% of their receipts for chari-
table purposes was struck down by the Court. The present ordinance would probably run afoul of
the same rule, because in effect it prohibits all charitable solicitation absent relatively burdensome
compliance with its registration provisions. The ordinance is also vulnerable because it limits the
right of solicitation to those who belong to “a recognized charitable organization.” (B), while a
correct statement of law, does not apply in these circumstances. The question is directed toward
the challenge the organization member would mount, not one that a religious organization might
pursue. (C) is incorrect. Because the First Amendment rights at issue here are fundamental, a
“reasonable balance” is not enough; the government ordinance is a direct, content-based regula-
tion, and will be subjected to strict scrutiny. (D) is incorrect because even if the prevention of
fraud is a compelling state interest, the statute is not narrowly tailored to achieve that interest,
which is required under the strict scrutiny standard.

Answer to Question 2

(B) Substantially overbroad or vague statutes regulating First Amendment rights are void on their
face, and persons may not be prosecuted for their violation even if their conduct might other-
wise be subject to valid regulation. Conversely, statutes that reasonably regulate the time, place,
and manner of speech in public forums may be unconstitutional if applied in situations where
the First Amendment activity is unreasonably infringed. Here, the statute is a valid time, place,
and manner restriction because it is content neutral, it is narrowly tailored to serve an important
government interest, and it leaves open alternative channels of communication. It is not too vague
when read as a whole, because pedestrian traffic is “sufficiently heavy” when the foot traffic spills
over into public streets because of the obstruction. However, it is being applied to circumstances
that do not provide any reasonable basis for regulation of the speech. Thus, (B) is correct: While
the demonstrator’s conduct can be regulated, it is the threat by the farmer that triggers the demon-
strator’s arrest, not his violation of the statute. (A) and (D) are incorrect because the statute itself
is valid on its face. (C) is incorrect, in turn, because it wrongly asserts that the demonstrator may
be prosecuted under these circumstances.

Answer to Question 3

(D) The court should find the ordinance unconstitutional. Regulation of truthful advertising of a
lawful activity is valid only if it serves a substantial government interest, directly advances that
interest, and is narrowly tailored to achieve that interest. The consumption of alcohol is a valid
activity and the ordinance here does not address deceptive advertising. While the city may argue
that discouraging drinking is a substantial government interest, it might have a difficult time
proving that the ordinance here advances that interest. More importantly, given that the ordinance
amounts to a total ban on advertising outside of stores selling liquor, the city would have a diffi-
cult time arguing that the ordinance is narrowly tailored. (A) is incorrect because the Supreme
Court has held that while the Twenty-first Amendment gives states power to regulate liquor
commerce, it does not give them power to override First Amendment protections. (B) is incorrect
because while it may be true that the ordinance does not favor local economic interests or unduly
2. CONSTITUTIONAL LAW - FIRST AMENDMENT FREEDOMS ANSWERS

interfere with interstate commerce (and so does not violate the Commerce Clause), it nevertheless
is unconstitutional for the reasons stated above. (C) is incorrect because the store owner did not
have a right to personal notice. Such a right arises only when there is personalized adjudication—
when the government seeks to deprive a specific individual of life, liberty, or property. Here, the
government adopted a city-wide ordinance affecting all liquor sellers.

Answer to Question 4

(C) The most likely ground for dismissal is lack of standing. Except for very limited exceptions, the
traditional rule is that a person has standing only to raise constitutional issues which affect her
personally. She cannot claim that a third person’s constitutional rights were violated. The husband
is the injured party and must assert his own rights. (A) is incorrect. The issue in this question
is not the substance of America’s foreign policy, which would be a political question, but the
husband’s First Amendment right to speak out about it, which is not a political question. There-
fore, the court is not barred from litigating this issue under the political question doctrine. (B) is
incorrect. There are damages to be assessed if the husband was improperly dismissed. Under Mt.
Healthy Board of Education v. Doyle (1977), a public employee cannot be fired without a hearing
for exercising the right to free speech, even if there is no property right in the job. The case is
not moot because the husband has lost his job permanently. The case would be moot only if the
husband had been reinstated to his former job at his former pay. (D) is incorrect. The subject
matter of the suit—the dismissal of an employee for exercise of his First Amendment rights—
presents a federal question.
Lizak
Answer to Question 5

(A) The Constitution is inapplicable because the bus company is a private company. The Supreme
Court has ruled that the grant of a franchise is not sufficient to create state action. [Jackson v.
Metropolitan Edison (1974)] Thus, there is no basis for a First Amendment claim against the bus
company. (B) is wrong; the Constitution is inapplicable because of the lack of state action. Also,
(B) is wrong based on the facts of the question. The speech restricted was political, not commer-
cial. Thus, even though commercial speech receives less protection than political speech, this
speech cannot properly be characterized as speech that seeks to induce business transactions. (C)
is wrong because the audience’s opposition to a message does not justify otherwise impermissible
censorship. Also, (C) wrongly assumes that the Constitution applies. (D) is wrong because the
availability of alternative avenues of expression does not justify otherwise impermissible censor-
ship. Moreover, it also wrongly assumes that the Constitution applies.

Answer to Question 6

(B) The loyalty oath is constitutional, but the membership oath is unconstitutional. The First Amend-
ment protects the rights of association and speech. State infringements on these rights must serve
a compelling government interest, unrelated to the suppression of ideas. In the area of public
employment, neither standards of conduct nor loyalty oaths may be vague or overbroad. Precision
is required because of the potential chilling effect on First Amendment rights. Here, the inves-
tigator is not applying for public employment, but the state is requiring him to take the oaths to
obtain a state license, which involves the same First Amendment issues. The loyalty oath is virtu-
ally identical to oaths that have been held to be constitutional. The state has a compelling interest
in seeing that both constitutions are upheld and there is nothing vague about the oath. Oaths
similar to the membership oath, however, have been struck down as overbroad. The state has a
compelling interest only in preventing knowing membership with the specific intent to further
CONSTITUTIONAL LAW - FIRST AMENDMENT FREEDOMS ANSWERS 3.

unlawful aims. Persons cannot be denied a license because of mere membership in a particular
group, and the state statute here addresses mere membership. Thus, (B) is correct and (A), (C),
and (D) are incorrect.

Answer to Question 7

(C) The Court should declare the ordinance unconstitutional, because charitable solicitations for funds
in residential areas are within the protection of the First Amendment. In Village of Schaumburg
v. Citizens for a Better Environment (1980), the Supreme Court held unconstitutional a municipal
ordinance that prohibited the door-to-door solicitation of contributions by charitable organiza-
tions that did not use at least 75% of their receipts for charitable purposes. After review, the Court
stated that the precedent of earlier decisions established clearly that the charitable appeal for
funds involves a variety of speech interests that are within the protection of the First Amendment.
The Court concluded that the ordinance unduly intruded on the rights to free speech because the
justifications for the restriction were not sufficiently compelling. (A) is wrong because the state’s
“legitimate interest” is not enough to justify violation of First Amendment rights. Similarly, (B)
and (D) are wrong because the First Amendment rights outweigh the state’s purposes and there
are less intrusive ways of accomplishing the state’s goals.

Answer to Question 8

(C) The federal court would likely find that the statute violates the First Amendment by restraining
political speech and association rights. While the government may limit the amount of contribu-
tions that an individual may contribute to a candidate’s campaign, it may not limit contributions
to groups that lobby for or against matters before the legislature, because the Supreme Court has
found that such a law does not serve a sufficiently important interest to outweigh the restraints
that it puts on the First Amendment freedoms of speech and association. (A) is incorrect because
it states an incorrect result (the statute likely will be found invalid for the reason stated above) and
because it states an improper standard (a statute limiting campaign contributions is tested under
intermediate scrutiny—it must be closely drawn to achieve a sufficiently important interest). (B)
is incorrect. As mentioned above, the government may limit the amount of contributions that an
individual may contribute to a candidate’s campaign, because the government has a sufficiently
important interest in preventing the corruption or the appearance of corruption from large contri-
butions that outweighs First Amendment issues. (D) is incorrect because it is too broad. While
states may not limit the amount that a person may spend to get a candidate elected, they may limit
the amount that a person may contribute to another’s campaign.

Answer to Question 9

(D) The reporter’s First Amendment defense is not likely to succeed because the newspaper did not
get his permission. The author had a property right in his manuscript that can be protected by
the copyright laws regardless of the public importance of the content. The best way to answer
this question is to eliminate the wrong choices. (A) is wrong because there is no First Amend-
ment exception to copyright laws. It does not matter that the author may have been a public
figure or that his book discussed a matter of public concern; magazines have no right to publish
copyrighted material without permission beyond the statutory fair use exception, not the case
here. [Harper & Row Publishers v. Nation Enterprises (1985)] (B) is wrong because it is too
broad; the press generally has no greater freedom to speak than does the public. (C) is wrong
because the Fifth Amendment is not applicable here. The Fifth Amendment prohibits government
from taking private property without due process or just compensation, and here a private party
4. CONSTITUTIONAL LAW - FIRST AMENDMENT FREEDOMS ANSWERS

has acted. Therefore, (D) is correct; the author will prevail in an appropriate action for copyright
infringement against the reporter because the reporter published portions of the author’s work
without permission.

Answer to Question 10

(C) The statute is a constitutional restriction on speech in a public forum. The conduct related to
speech can be restricted through reasonable time, place, and manner regulations. To be reason-
able, such a regulation must be content neutral and narrowly tailored to serve an important
government interest, and leave open alternative channels of communication. The statute here does
not have any content rules. The state administrator must issue the permits to the first 10 people
or organizations who apply, regardless of message; the administrator has no discretion here. The
statute also appears to be narrowly tailored to achieving the purpose of facilitating access to the
capitol building, and it allows other ways of protesting (e.g., standing more than 100 feet from
the building). (A) is incorrect. If the purpose of the statute is to facilitate access to the building,
a 10-permit limit seems reasonable, especially because permits issued to organizations could
result in many more than 10 people within 100 feet of the capitol doors. (B) is incorrect for a
similar reason. More than 10 people closer than 10 feet from the capitol doors could be a barrier
to access. (D) is incorrect; the rule is just the opposite. Capitol grounds and their walkways are
traditional public forums.

Answer to Question 11

(B) The religious leader and his followers will lose. The government can place reasonable restrictions
on time, place, and manner of speech in public forums. It would be difficult to envision more
reasonable ordinances than the ones involved here. Therefore, (B) is correct, because it states
the appropriate rule. (D) is wrong, since a facial challenge requires that there be no conceivable
circumstances under which the ordinance could be constitutionally applied, and that is clearly not
the case here. (A) is wrong; the unconstitutionality of the ordinance could be raised for the first
time as a defense in the criminal case. Under accepted principles of comity, the religious leader
does not have to go to federal court to litigate federal constitutional claims. (C) is wrong because
even if the law represents the “will of the people,” it could still be unconstitutional. The Supreme
Court has emphasized repeatedly that constitutional rights are neither subject to, nor can be
compromised by, a “vote of the people.”

Answer to Question 12

(D) The court will rule for the owner because the ordinance is too broad. The Supreme Court has
defined obscenity as a description or depiction of sexual conduct that taken as a whole, by the
average person, applying contemporary community standards, appeals to the prurient interest
in sex, portrays sex in a patently offensive way, and using a reasonable person standard, does
not have serious literary, artistic, political, or scientific value. Thus, under this definition, not all
nudity is obscene. (A) is incorrect because although the town may try to protect its minors, that
purpose does not allow it to prohibit all such advertisements. The ordinance is too broad to be
valid and its purpose will not save it. (B) is incorrect because the “average person” in the above
definition does not include children. (C) is incorrect because if the posters fall within the Court’s
definition of obscenity, they could be prohibited because obscenity is not protected by the First
Amendment.
CONSTITUTIONAL LAW - FIRST AMENDMENT FREEDOMS ANSWERS 5.

Answer to Question 13

(C) Although the First and Fourteenth Amendments severely limit the states’ right to regulate public
speech, reasonable regulations based on time, place, and manner are constitutionally permissible.
(D) is thus wrong. (A) is wrong because the fact that the civic center is a place normally used for
demonstrations of this type does not bar the city from restricting the demonstration; that is exactly
why the city sought to impose certain limitations on these demonstrations. (B) is also incorrect,
because a municipality does have the limited right to place certain limitations on a citizen’s right
of free speech in these circumstances, even when there is absolutely no reason to believe that the
demonstration will be anything but peaceful and quiet. However, any such regulation by the city
cannot be substantially overbroad (e.g., censor protected speech) or vague (i.e., be so unclear as to
what is prohibited that a demonstrator is required to censor himself). The statute’s provisions for
terminating a parade and arresting demonstrators are overbroad and too vague to meet any legiti-
mate state purpose. Absent any court decisions that have limited construction of the regulation
so as to remove the threat to constitutionally protected expression, the statute’s imprecise terms
could be applied to protected speech and do not provide persons with reasonable notice as to what
speech is prohibited.

Answer to Question 14

(C) The newspaper’s best argument is based on freedom of speech. The validity of a law that regulates
elections is determined by a balancing test. If the law regulates “core political speech,” rather
than the process surrounding elections, strict scrutiny is applied (i.e., the law must be narrowly
tailored to achieve a compelling interest). The Supreme Court has held that a state law prohib-
iting campaigning on election day is invalid as applied to a newspaper editorial urging voters to
vote a certain way, because the right to comment on political issues is an essential element of free
speech. The state statute at issue here is similar to the law that was held invalid by the Court;
thus, its enforcement should be enjoined on free speech grounds. (A) does not offer a very good
argument for the paper. It is true that the regulation affects interstate commerce because the
newspaper is circulated in other states. However, because the regulation does not directly burden
interstate commerce (i.e., it does not discriminate against interstate commerce by favoring local
economic interests), the newspaper must argue that the burden on interstate commerce outweighs
the promotion of legitimate local interests. Here, the local interest of promoting fair elections
is quite strong, and the burden on interstate commerce is very weak. Therefore, the Commerce
Clause argument is not very strong. (B) is incorrect because the statute in fact does not affect the
newspaper’s right to distribute the paper; it merely prohibits the paper from containing certain
material on certain days. (D) would not be a good argument because the Equal Protection Clause
only prohibits setting up classes to treat similar persons or groups in a dissimilar manner. Here,
all similar entities (newspapers) are treated alike—none is allowed to publish political editorials
the day before and the day of an election. The fact that the same material can be printed on other
days does not create an equal protection problem.
CONSTITUTIONAL LAW -
INDIVIDUAL RIGHTS
QUESTIONS
一 信:liuxue119118 , 们 信免 供

CONSTITUTIONAL LAW - INDIVIDUAL RIGHTS QUESTIONS 1.

CONSTITUTIONAL LAW - INDIVIDUAL RIGHTS QUESTIONS

Question 1 Question 2

Congress enacted a law requiring all civil A state was suffering from a near-depression
service employees to retire by age 75, except caused by layoffs in the tourist service industry.
when such employees are employed by the In an attempt to alleviate the problem, the state
armed services. Civil service employees of the enacted a statute providing for the immediate
armed services are required to retire by age 65. hiring of 100,000 employees to repair, maintain,
An employee of the armed services just turned and otherwise work at the discretion of the state
65 years old. He files suit in the federal district director of highways. The statute further stated
court seeking a declaratory judgment that would that preference would be given to persons who
prevent his employer from requiring him to had worked in the tourist service industry for
retire before age 75. five years and had been laid off.

What is the employee’s strongest argument Which of the following constitutional provi-
that the statute’s provision regarding civil service sions would be most relevant in determining
employees of the armed services is invalid? the constitutionality of the preference for tourist
service industry workers?
(A) The provision denies him the privileges and
immunities of national citizenship. (A) The Privileges and Immunities Clause of
Article IV.
(B) The provision denies him a property right
without just compensation. (B) The Equal Protection Clause of the
Fourteenth Amendment.
(C) The provision is invidious discrimination
on the basis of age in violation of the Fifth (C) The reserved powers of the state under the
Amendment. Tenth Amendment.

(D) The provision is not within the enumer- (D) The Privileges or Immunities Clause of the
ated powers of Congress under Article I, Fourteenth Amendment.
Section 8.
2. CONSTITUTIONAL LAW - INDIVIDUAL RIGHTS QUESTIONS

Question 3 Question 4

Congress has recently enacted legislation A state statute prohibits aliens from owning
that makes it a federal crime for any person to land and makes it illegal to sell land to aliens.
interfere with any right conferred by the Equal A landowner who entered into a contract to
Protection Clause of the Fourteenth Amend- sell property to an alien brought an action to
ment. challenge the statute. The alien, however, did not
participate in the action.
The statute may be applied constitutionally in
which of the following situations? What is the landowner’s strongest consti-
tutional argument against the validity of the
(A) A person who hates Asians bribes a federal statute?
official so that he fails to distribute free
dairy products to otherwise eligible Asians. (A) The statute denies the equal protection of
the laws to aliens.
(B) A person who believes women are inferior
to men persuades the dean of a private (B) The statute unconstitutionally impairs the
school licensed by the state to deny admis- landowner’s contract for the sale of land to
sion to otherwise qualified women because a buyer.
of their sex.
(C) The statute is a direct restraint on the alien-
(C) By threats of violence, a person coerces the ation of the landowner’s real property.
coach of a public high school’s basketball
team to exclude white athletes from the (D) The statute denies the landowner of a
team solely because of their race. property right without due process of law.

(D) A person persuades the members of his


church council to deny shelter and food
to homosexual men who seek aid at the
church-run downtown relief center.
CONSTITUTIONAL LAW - INDIVIDUAL RIGHTS QUESTIONS 3.

Question 5 Question 6

A citizen who is unhappy about a recent A federal district court judge was accused of
decision of his city council stood in front of city misconduct in office and was impeached by the
hall and gave an extemporaneous speech belit- House of Representatives. At trial in the United
tling each member of the city council. During States Senate, the judge was convicted and
the diatribe, the citizen made the following removed from office. Nevertheless, the Presi-
statement: “If there is a God, the city council dent directed the Attorney General to institute
members will surely burn in hell forever.” A criminal proceedings against the judge. After
state statute, enacted in 1898, prohibited “the presentation to a federal grand jury, an indict-
public utterance of any blasphemy or sacri- ment was issued against the judge and signed
lege,” and provided criminal penalties for its by the Attorney General. At the opening of his
violation. On hearing the citizen’s utterances, a trial, the judge moved to have the indictment
police officer arrested him for violating the 1898 dismissed.
statute. The local district attorney decided to
proceed with prosecution of the case, only the How is the trial judge most likely to rule?
third recorded such prosecution in the state’s
history. (A) Dismiss, because the President had told the
Attorney General to prosecute.
Which of the following arguments would
NOT be helpful for the citizen’s defense? (B) Dismiss, because the criminal proceeding
violates the Fifth Amendment’s proscription
(A) Application of the statute to the citizen against double jeopardy.
infringes his freedom of speech in violation
of the Fourteenth Amendment. (C) Deny the dismissal, because the federal
grand jury issued the indictment.
(B) Application of the statute to the citizen
denies him equal protection of the law in (D) Deny the dismissal, because the judge has
violation of the Fourteenth Amendment. not been previously tried in a criminal
proceeding.
(C) The statute violates the Fourteenth Amend-
ment because it is an establishment of
religion.

(D) The statute violates the Fourteenth Amend-


ment because it is vague.
4. CONSTITUTIONAL LAW - INDIVIDUAL RIGHTS QUESTIONS

Question 7 Question 8

A candidate for a state legislature brought an A woman decided to run for office as a park
action under the Fourteenth Amendment against district trustee in the next general election.
a privately owned cab company to compel the A city ordinance provided that all candidates
company to accept his political advertisements running for the office of park district trustee
on its vehicles. The cab company is licensed must present to the city clerk within 90 days
by a rural county within the state in which the of the election petitions signed by at least
candidate is running. As part of its taxi business, 500 qualified voters, a $500 filing fee, and a
the cab company uses pickup trucks to ferry its personal financial statement that will be open
largely rural-based customers and the agricul- to the public for inspection. The woman timely
turally related items they often carry around. filed the petitions with sufficient signatures, but
The company sells space on the wooden slats refused to file the financial statement or pay
surrounding the bed of each pickup truck to the filing fee. When the clerk refused to put
commercial advertisers, but it refuses to carry her name on the ballot, the woman filed suit,
any political advertising. claiming that the filing fee and personal finan-
cial statement requirements are unconstitutional.
Which of the following is the strongest justifi-
cation for the trial court’s denying the candidate Which of the following statements best
any relief? describes the likely outcome of the woman’s
suit?
(A) The sides of the cab company’s pickup
trucks are not public forums. (A) The filing fee requirement will be held
invalid if the woman is indigent, but the
(B) The candidate has reasonable alterna- financial statement requirement will be
tive methods of getting his message to the upheld.
public.
(B) The filing fee requirement will be upheld,
(C) The Fourteenth Amendment provides no but the financial statement requirement
basis on which to compel the cab company will be held invalid because it impairs the
to accept the candidate’s ads. woman’s right of personal privacy.

(D) The right of the cab company to choose (C) Both requirements will be held invalid
what messages to display outweighs the and the woman will be entitled to have her
candidate’s right to insist that the cab name placed on the election ballot.
company accept his advertising.
(D) Both requirements will be held invalid
unless the requirements are shown to serve
some legitimate state purpose.
CONSTITUTIONAL LAW - INDIVIDUAL RIGHTS QUESTIONS 5.

Question 9 Question 10

Citizen groups proposed the merger of a large State legislation established family counseling
town of 60,000 residents with an adjacent small centers throughout the state targeting poor
town of 30,000. To allay fears of residents of the single-parent households, which were typically
small town that control of the government would headed by a woman. Accordingly, the legisla-
be in the hands of the more populous town, the tion provided that all counseling would be free
proposal would establish three council districts for single mothers with an income of less than
of 20,000 persons each in the large town and $20,000 per year. Others would have to pay fees
three districts of 10,000 persons each in the for the counseling on a sliding scale depending
small town for a period of 20 years. Each district on income. A single father raising two children
would elect one representative to the six-person on an income of approximately $7,000 per year
town council, and the mayor would be elected at objected to paying the sliding scale fee. A public
large. The proposal carried by large majorities in interest law firm files suit in federal court on his
both towns, and representatives from both towns behalf, alleging discrimination.
proceeded to carve each former city into three
council electoral districts. Three taxpayers from Which of the following statements is most
the large town filed suit to enjoin the holding of accurate?
an election with council districts of such dispa-
rate proportions. (A) The plaintiff must show that the law is not
rationally related to a legitimate state inter-
Which of the following represents the plain- est favoring mothers over fathers.
tiffs’ best constitutional argument?
(B) The plaintiff must show that the state has
(A) The government of the merged cities is not no compelling interest in favoring mothers
a republican form of government. over fathers.

(B) The plaintiffs and other residents of the (C) The state must show that favoring mothers
large town have been denied equal protec- over fathers is substantially related to an
tion of the law. important governmental interest.

(C) The plaintiffs and other residents of the (D) The state must show that the law is ratio-
large town have been denied the privileges nally related to a legitimate state interest
and immunities of citizenship as guaran- favoring mothers over fathers.
teed by Article IV of the federal Constitu-
tion.

(D) The plaintiffs and other residents of the


large town have been denied due process of
law.
6. CONSTITUTIONAL LAW - INDIVIDUAL RIGHTS QUESTIONS

Question 11 Question 12

Congress provided funds to a group of young As part of a new environmental policy, a state
scouts. A person believed that this particular legislature enacted a statute creating a farmland
scouting group practiced racial discrimination development agency. The agency was organized
and brought suit to enjoin the federal provision along the lines of a private corporation. The
of funds to this group. agency’s purpose was to restore and cultivate
polluted land in the state, making the land
Assuming the factual allegation regarding suitable for farmers. Once the agency cultivated
racial discrimination is correct and that the the land, it would sell the land to new farmers at
person has standing to challenge the expenditure, a reduced price. The agency began cultivating
how should the court rule? and selling farmland in one county where the
pollution levels were high, but did not operate in
(A) Deny the injunction, because the discrimi- a neighboring county with slightly lower pollu-
nation is by a private entity. tion levels. A group of residents in the second
county bring an action in federal court against
(B) Deny the injunction, because the scouts the agency director, seeking to compel the
have a First Amendment right to associate. agency to begin operations in their county.
(C) Grant the injunction, because the scouts are What will be the probable outcome of this
violating the Equal Protection Clause. litigation?
(D) Grant the injunction, because by providing (A) The action will be dismissed, because the
the funds, Congress is promoting segrega- state is immune from litigation under the
tion. Eleventh Amendment.

(B) The action will be dismissed, because the


agency is organized as a private business
and thus does not engage in any state
action.

(C) The state will prevail, because it has a


rational basis for not operating in the neigh-
boring county.

(D) The citizens will prevail, because the state


cannot show a compelling state interest in
not operating in their county.
CONSTITUTIONAL LAW - INDIVIDUAL RIGHTS QUESTIONS 7.

Question 13 Question 14

Logging has long been one of the major A state law grants each county great
industries of a state. To protect the logging autonomy in setting the health standards
industry, which was being harmed by the too governing the preparation, packaging, transpor-
rapid depletion of the wood in the forests of tation, and sale of food items. A county council
the state, and to promote the general welfare of recently enacted an ordinance, valid under the
the state’s citizens, the state legislature enacted constitution and statutes of the state, prohib-
statutes for the first time requiring licenses for iting the packaging and sale of any food item in
commercial logging. To receive a license, the any non-biodegradable material; the ordinance
applicant must pay a $500 fee and establish by defines non-biodegradable and specifically lists
acceptable evidence that he has been engaged in as prohibited all forms of plastics, cellophane,
commercial logging in the forests of the state for or similar materials. The ordinance specifi-
the past 15 years. A limited number of special cally exempts from its terms sales of food to
licenses are available for those who do not meet public institutions such as hospitals, jails, and
the requirements of the regular licenses, and schools. A retail food seller in the county files
these special licenses are expressly reserved for an appropriate court action attacking the county
citizens who have resided in the state for at least ordinance on the grounds that it violates the
three years prior to the date of the application. A Equal Protection Clause of the Fourteenth
legally admitted alien who has been residing in Amendment.
the state for 10 years brings suit in federal court
to enjoin enforcement of the licensing statute as How should the court rule?
to himself and all other similarly situated noncit-
izen legal residents of the state. (A) For the food seller, because the state’s
interests could be effectuated by alterna-
Which of the following doctrines will tive methods less intrusive upon the food
probably be determinative of his claims? seller’s constitutional rights.

(A) The powers reserved to the states by the (B) For the food seller, because no compelling
Tenth Amendment to the federal Constitu- state interest is served by the challenged
tion. ordinance.

(B) The Equal Protection Clause of the (C) For the county, because the state may
Fourteenth Amendment. regulate in this area as Congress has not
entered the field.
(C) The Due Process Clause of the Fifth
Amendment to the federal Constitution. (D) For the county, because the ordinance
is rationally related to a legitimate state
(D) The Privileges and Immunities Clause of interest—the health and safety of its
Article IV. citizens.
CONSTITUTIONAL LAW -
INDIVIDUAL RIGHTS
ANSWERS
CONSTITUTIONAL LAW - INDIVIDUAL RIGHTS ANSWERS 1.

CONSTITUTIONAL LAW - INDIVIDUAL RIGHTS ANSWERS

Answer to Question 1

(C) The strongest possible argument here is (C), because equal protection claims are made against the
federal government pursuant to the Fifth Amendment’s Due Process Clause. The Supreme Court
has held that this provision implicitly includes a requirement for equal protection. Although the
employee has little chance of prevailing because only a rational basis test is used for age discrimi-
nation claims, this answer is the only possible basis for challenging the law. (A) is wrong because
both constitutional provisions referring to privileges and immunities apply to state government
conduct and are never used against the federal government. The Fourteenth Amendment provides
that no state shall deny any citizen the privileges or immunities of national citizenship. Article IV
provides that no state shall deny citizens of other states the privileges and immunities it accords
its own citizens. (B) is wrong because a “just compensation” claim will only provide him compen-
sation for his loss of property. Even if his job is considered to be a property right, the just compen-
sation claim would not provide him the relief he wants, which is reinstatement in his job. (D) is
wrong because even though there is no express provision in Article I, Section 8, concerning this
legislation, it certainly is within congressional powers.

Answer to Question 2

(B) The Equal Protection Clause of the Fourteenth Amendment is the most relevant. The Equal
Protection Clause is at issue when a law treats a person or class of persons differently from others.
Here, the classification scheme makes a distinction between individuals who worked in tourism
and those who did not, and those who worked for five years versus those with a lesser period of
employment. (A) is wrong, because the Privileges and Immunities Clause of Article IV has no
bearing: That clause prohibits discrimination by states against nonresidents, and here the state is
not treating residents of other states differently. (C) is not the best answer, because the reserved
powers doctrine helps, rather than hinders, the state by providing that all powers not delegated
to the federal government by the Constitution are reserved to the states. (D) is wrong because
employment is not within the narrow category of privileges protected by the Fourteenth Amend-
ment Privileges or Immunities Clause.

Answer to Question 3

(C) The facts of (C) are the only ones in which the person has compelled a state official to deny equal
protection of the law to some person. The Fourteenth Amendment prevents states from depriving
any person of life, liberty, or property without due process of law and equal protection of the law.
Because the Equal Protection Clause protects against state action only, the federal statute at issue
prohibits only behavior that causes or induces a state official, agency, or instrumentality to deny
the equal protection of the law to some person. In (C), the person coerced a state agent (the coach
of a public high school basketball team) to exclude people from participation in an activity at a
state institution solely on the basis of race. Because this is a state governmental act that classifies
people based on a suspect trait (race) in the absence of a compelling state interest, it violates equal
protection. For bringing about this violation, the person is guilty of violating the federal statute.
(A) is incorrect because the person there is inducing a federal official to discriminate in the distri-
bution of free dairy products. The Fourteenth Amendment, unlike the Fifth Amendment, does
not curtail the actions of the federal government. Therefore, the person acting has not interfered
with any right conferred by the Fourteenth Amendment Equal Protection Clause. (B) is incor-
rect because the person there has induced discriminatory action against women by the dean of a
2. CONSTITUTIONAL LAW - INDIVIDUAL RIGHTS ANSWERS

private school. The Fourteenth Amendment does not protect against the actions of private persons
or institutions. The actions of a private entity may constitute state action if the state is signifi-
cantly involved in the private entity. However, the mere granting of a license is not sufficient state
involvement with a private entity so as to convert its action into state action. Thus, the fact that
the private school in (B) is licensed by the state does not make the school’s discriminatory treat-
ment of women state action. (D) is incorrect because the person there has induced discriminatory
action by his church, which is a private entity with no apparent significant state involvement. As
explained above, the Fourteenth Amendment Equal Protection Clause does not protect against the
actions of such an entity. Thus, the person has not interfered with a right conferred by the Equal
Protection Clause.

Answer to Question 4

(A) The landowner’s strongest argument is based on the alien’s equal protection rights. Although the
state statute directly impairs the equal protection rights of the buyer rather than the seller, here
the seller can assert the buyer’s rights. A person who challenges the constitutionality of a govern-
mental action must have standing; i.e., she must demonstrate a concrete stake in the outcome of
the controversy and that the governmental action at issue impairs her rights. Generally, a claimant
must have suffered (or may presently suffer) a direct impairment of her own constitutional rights.
However, a plaintiff may assert a third party’s rights where the plaintiff has suffered injury and
the injury adversely affects her relationship with third parties, resulting in an indirect violation of
their rights. Here, the state statute causes injury to the seller by prohibiting her from contracting
to sell land to an alien buyer. Such an injury would adversely affect the seller’s relationship with
aliens by prohibiting her from selling land to them, and would thus indirectly violate their right
to equal protection. Consequently, the seller may assert the equal protection rights of aliens. (B)
is incorrect because under the Contract Clause, which limits the ability of states to enact laws
impairing the obligation of contracts, there is no impairment unless the law is retroactive. Here,
it appears that the statute was in existence when the buyer and seller entered into their contract.
Thus, the statute did not unconstitutionally impair that contract. (C) is incorrect because, even
if the statute does constitute a direct restraint on the alienation of the seller’s property, such a
restraint does not rise to the level of a constitutional violation. Thus, an argument based on (C)
would not be as strong as the assertion of aliens’ equal protection rights, as found in (A). (D) is
incorrect because nothing here indicates that the seller’s due process rights have been violated.
The procedural due process proscription against the governmental taking of property without
due process of law applies to individualized takings—i.e., when the government takes property
to which the individual has a legitimate claim. Here, the seller is not complaining about an action
against her individually, but rather is complaining about a law of general application. Thus, a
procedural due process argument is inapplicable. Substantive due process will not be helpful to
the seller either. If a law limits a fundamental right (i.e., voting, privacy, right to travel, or a First
Amendment right), the strict scrutiny test is used. However, if a fundamental right is not involved,
as here, the rational basis test is used. Under the rational basis test, it is unlikely that the seller
can succeed with her claim because she will have the burden of proving that the government has
no rational basis for its law. Because under this test almost any basis will support governmental
action, this statute is likely to be upheld, and so this is not the seller’s best argument.

Answer to Question 5

(B) A Fourteenth Amendment equal protection defense would be least helpful to the citizen. If a
law treats a person or class of persons differently than others, it is an equal protection question.
Here, the statute on its face does not treat different classes of persons differently, and there is
CONSTITUTIONAL LAW - INDIVIDUAL RIGHTS ANSWERS 3.

nothing in the facts to suggest that the law, although infrequently enforced, has been applied to
the citizen in a discriminatory manner. Hence, an equal protection argument would be weak.
(A) is incorrect because it is presumptively unconstitutional for the government to place burdens
on speech because of its content. To prevail, the government would have to show that the statute
was necessary to serve a compelling state interest and narrowly drawn to achieve that end. Thus,
free speech rights would provide an effective challenge to the statute. (C) is incorrect because
the citizen could argue that the statute violates the Establishment Clause. A statute will be valid
under that provision only if it (i) has a secular purpose, (ii) has a primary effect that neither
advances nor inhibits religion, and (iii) does not produce excessive government entanglement with
religion. The citizen can argue that a statute prohibiting the utterance of “blasphemy” and “sacri-
lege” violates that rule. (D) is incorrect because the citizen has a strong argument that the statute
violates the Due Process Clause. If a criminal law or regulation fails to give reasonable notice as
to what is prohibited, it violates the Due Process Clause. This principle is applied strictly when
First Amendment activity is involved to avoid the chilling effect that a vague law might have on
freedom of speech. Here, the citizen can defend by asserting that the statute does not define the
prohibited speech except in the most general terms, and therefore the statute is unconstitutionally
vague.

Answer to Question 6

(D) The trial judge will most likely deny the motion to dismiss the indictment. The Fifth Amendment
right to be free of double jeopardy for the same offense applies to subsequent criminal actions, but
not to civil actions or impeachment proceedings, which are distinct from criminal proceedings.
Article I, Section 3 of the Constitution specifically states that a conviction by impeachment does
not prevent the party convicted from being subject to indictment, trial, judgment, and punishment
according to the law. Hence, (B) is incorrect. (A) is incorrect because there is nothing in the facts
to show that the Attorney General was not acting within his prosecutorial discretion even if he
was complying with the wishes of the President. (C) is incorrect because the fact that the grand
jury issued the indictment is irrelevant. If double jeopardy did apply or if the Attorney General
had abandoned his prosecutorial discretion in instituting criminal proceedings, the fact that the
grand jury issued an indictment would not prevent the indictment from being dismissed.

Answer to Question 7

(C) The strongest basis for denying the candidate relief is that there is no state action on which the
Fourteenth Amendment can operate. The cab company is privately owned and operated. For state
action to be present, the government must be significantly involved in the private entity; mere
county licensing does not convert the private company’s operations into state action. (A), (B),
and (D) are incorrect because they ignore this critical factor. If state action were involved, these
justifications—based on the First Amendment prohibitions against government infringement of
speech—would tend to support the cab company’s position, but there is no need to apply First
Amendment doctrines because the government is not involved.

Answer to Question 8

(A) The filing fee is invalid if the woman is indigent, but the financial statement requirement is
valid. A state may not impose on candidates a fee that renders it impossible for indigents to run
for office. Even as applied to nonindigent candidates, an unreasonably high filing fee that is not
tailored to promote a substantial or overriding state interest might be held invalid. However, even
a reasonable and otherwise valid fee would have to be waived for an indigent candidate unable
一 信:liuxue119118 , 们 信免 供

4. CONSTITUTIONAL LAW - INDIVIDUAL RIGHTS ANSWERS

to pay the fee. If the woman is indigent, then a $500 filing fee would certainly preclude her
running for office. Other types of restrictions on the ability of persons to be candidates must be
examined to see if the restrictions violate either the First Amendment right of political associa-
tion or the Fourteenth Amendment Equal Protection Clause. A ballot access regulation must be
a reasonable, nondiscriminatory means of promoting important state interests (such as running
an honest, efficient election system). The financial statement requirement here is a reasonable
means of promoting the important state interest of disclosing possible conflicts of interest that
might compromise the integrity of elected officials. (It also might be a legitimate method for
determining whether a candidate is truly indigent, thus justifying waiver of the filing fee.) This
requirement is nondiscriminatory because it is applied to all candidates, rather than only some
candidates. Any impairment of a candidate’s right of privacy resulting from this requirement is
slight in comparison to the important governmental interest served thereby. Thus, the financial
statement requirement is valid. (B) is incorrect in stating the invalidity of the financial statement
requirement. Also, (B) is incorrect because it cannot be said with any certainty that the filing fee
requirement is valid. The fee requirement will certainly be invalid as applied to indigent candi-
dates, and it might be so high and lacking in promotion of a substantial state interest that it will
be invalid as applied to all candidates. (C) is incorrect because it states that the financial state-
ment requirement is unconstitutional in addition to the filing fee requirement. As noted above, the
requirement of a financial statement is valid. (D) is incorrect because the validity of restrictions
on the ability of persons to be candidates is determined by a balancing test: A severe restriction
such as a filing fee making it impossible for indigents to run for office would require a compelling
state purpose to be valid.

Answer to Question 9

(B) The plaintiffs’ best argument is a denial of equal protection. The Equal Protection Clause of
the Fourteenth Amendment prohibits state dilution of the right to vote by malapportionment of
electoral districts. This rule applies to electoral districts for local governmental bodies as well as
for the state legislature. When a local government establishes voting districts for the election of
representatives, it must establish districts that do not have a significant variance in the number of
persons in each district. Here, the large town districts have twice as many persons as the small
town districts, creating an unconstitutional dilution of the large town citizens’ right to vote. (A) is
incorrect. The clause in Article IV, Section 4 guaranteeing a “republican form of government” to
the states has been judged by the Court to involve a political question. The Court will therefore
decline to address that issue. (C) is incorrect because the Article IV Privileges and Immunities
Clause, which provides that citizens of each state shall be entitled to all privileges and immuni-
ties of citizens of the several states, only prohibits a state from discriminating in favor of its
own citizens and against citizens of other states with regard to “fundamental rights,” i.e., those
involving important commercial activities or civil liberties. Here, out-of-state citizens are not
being discriminated against, so the clause is not applicable. (D) is incorrect because the Due
Process Clause is usually used to review a law that limits the liberty of all persons to engage in
some activity, whereas the Equal Protection Clause is implicated when a law limits the liberty or
rights of some persons but not others. The option raising the equal protection argument is there-
fore the better choice.

Answer to Question 10

(C) A law containing gender-based classifications must be shown by its proponent, the state, to be
substantially related to important government interests. State laws that discriminate by treating
people in similar situations in dissimilar ways are subject to challenge under the Equal Protection

一 信:liuxue119118 , 们 信免 供
CONSTITUTIONAL LAW - INDIVIDUAL RIGHTS ANSWERS 5.

Clause of the Fourteenth Amendment. The Supreme Court usually applies one of three tests
for evaluating an equal protection challenge to a statute. Where the classification relates to who
may exercise a fundamental right or is based on a “suspect” characteristic, the classification
must be necessary to promote a compelling state interest. Where the classification relates only to
matters of economics or social welfare, it is valid if it is rationally related to a legitimate govern-
mental interest. For quasi-suspect classifications, the Court has applied an intermediate standard,
upholding the law if its proponent can show, by an exceedingly persuasive justification, that the
classification is substantially related to important governmental interests. A distinction based on
gender, such as the provision in the state statute for free financial counseling to single mothers
but not to single fathers in the same financial situation, is a quasi-suspect classification. Thus,
to win the case, the state must show that favoring mothers over fathers is substantially related
to an important governmental interest. (A) is incorrect because a classification based on gender
is subject to an intermediate level of scrutiny, requiring a “substantial” rather than “rational”
relation and an “important” governmental interest rather than merely a “legitimate” governmental
interest. (B) is incorrect because gender is not a suspect classification that requires the compel-
ling state interest test to be applied. Even if it were, the plaintiff would not have the burden of
proving absence of a compelling interest; the state would have to prove the existence of a compel-
ling interest for the law to be upheld. (D) is incorrect for similar reasons: Gender is an “almost
suspect” class that requires a showing of more than just a rational relationship, and, even if that
test were appropriate, it would require the plaintiff to bear the burden of demonstrating that the
law does not have a rational relationship to a legitimate state interest.

Answer to Question 11

(D) Under the Equal Protection provisions of the Due Process Clause of the Fifth Amendment,
Congress may not engage in any activity that promotes segregation. Here, by providing funds
to a scouting group that engages in segregation, Congress has violated the Due Process Clause
of the Fifth Amendment, and this would thus be the strongest argument. (A) and (B) are incor-
rect because, while the discrimination might be by a private entity and members of private
entities generally may associate with whom they like, by providing funding for the private entity,
Congress is supporting discrimination in violation of the Fifth Amendment Due Process Clause.
(C) is incorrect because the Constitution generally prohibits only action by the government. In
some cases, private action might be considered state action (e.g., where the government is entan-
gled with the private action). It is unclear whether the scouts’ actions here would be considered
government action. In any case, the Equal Protection Clause would not apply against the federal
government, making (D) a better answer choice.

Answer to Question 12

(C) The state will prevail. Because this is economic legislation not involving fundamental rights or
suspect classifications, it will be upheld if it has a rational basis. The higher pollution levels justify
intervention in certain areas, and so the legislation would probably be valid. (A) is wrong because
while the Eleventh Amendment immunizes states from suit in federal courts, that immunity
does not authorize a state to violate the Constitution; a suit alleging such a violation may be filed
against the responsible officials. (B) is an incorrect statement of law; as an agency of the state, the
agency’s operations are state action. (D) is wrong because it applies the wrong standard of review.

Answer to Question 13

(B) This situation presents a possible violation of the Equal Protection Clause. A state may not favor
established residents over new residents. To do so in an area that affects a person’s ability to
6. CONSTITUTIONAL LAW - INDIVIDUAL RIGHTS ANSWERS

engage in his livelihood impedes migration from state to state. Interstate travel is a fundamental
right, and a classification that burdens it would trigger a strict scrutiny analysis. In any case, the
classification would be subjected to something more than the mere rationality test. (A) is incorrect
because this is not an area reserved to the states, and even if it were, the United States Constitu-
tion would take precedence over state law. (C) is wrong because the Fifth Amendment applies
only to federal, not state, government action. (D) is wrong because the Privileges and Immunities
Clause does not apply to aliens.

Answer to Question 14

(D) The court should rule for the county because the ordinance is valid. Where no suspect or quasi-
suspect classification or fundamental rights are involved, the equal protection analysis uses
the rational basis test: if the statute is rationally related to a legitimate state interest, it is valid.
Because there appears to be a rational basis for the challenged legislation—protecting the
environment and the citizenry from the effects of potentially permanent artificial materials—the
law is valid. (A) is incorrect because it assumes that the state must use the least restrictive alterna-
tive, but that is not required except when strict scrutiny is used. There is no basis for such review
under the facts of this question because there is neither a suspect classification nor a fundamental
right. (B) is wrong for the same reason: under equal protection analysis, a “compelling purpose”
is required only when strict scrutiny is used. Challenges to economic regulations are subjected
only to a rational basis test. (C) is incorrect because whether Congress has entered the field is
irrelevant to an equal protection analysis. The call of the question specifically asks for the deter-
mination of the equal protection challenge; federal preemption issues are irrelevant here.
CONSTITUTIONAL LAW -
INTERGOVERNMENTAL
IMMUNITIES QUESTIONS
CONSTITUTIONAL LAW - INTERGOVERNMENTAL IMMUNITIES QUESTIONS 1.

CONSTITUTIONAL LAW - INTERGOVERNMENTAL IMMUNITIES QUESTIONS

Question 1 Question 2

A state legislator was the chairman of a A state enacted a sales tax on specified items
committee that disbursed funds to schools in the purchased within the state. A subdivision of the
state for various projects. The federal govern- federal government purchased from a dealer in
ment supplied a portion of the funds as part of a the largest city in the state 100 new automobiles
federal revenue sharing plan. The legislator was for use by federal agencies operating within the
charged with a violation of federal law when state.
he and his committee made a $10,000 grant for
textbooks to a private school for whites only. Must the federal government pay the sales tax
The legislator’s defense is that as chairman of applicable to the new auto purchase?
this committee he was acting in the course of his
legislative duties, and thus, immune from federal (A) No, unless Congress has consented to such
interference. a tax.

What is the best argument that would support (B) No, because the tax unfairly discriminates
the legislator’s constitutional claim? against interstate commerce.

(A) If the state law authorizes the legislator’s (C) Yes, because the tax is nondiscriminatory.
action, he cannot be prosecuted for viola-
tion of a federal law. (D) Yes, because there is a rational basis for the
tax and it does not appear to be a disguised
(B) The doctrine of federalism forbids the penalty.
federal government from restricting the
state’s rights regarding the education of
minor children within the state.

(C) The Tenth Amendment prevents the federal


government from interfering with a member
of the state’s legislature in the performance
of his legislative duties.

(D) As long as the private school is not a


parochial school, federal law cannot limit
a state’s rights regarding the education of
minor children within the state.
2. CONSTITUTIONAL LAW - INTERGOVERNMENTAL IMMUNITIES QUESTIONS

Question 3 Question 4

A state operates a nuclear power plant on a Under a program created by Congress, the
river. The plant uses river water for cooling and federal government allocated $5 million to
discharges water back into the river 10 degrees one of the nation’s leading energy producing
warmer than when it was taken in. While the states “to be used solely for the development
discharged water quickly cools, it has adversely of synthetic materials to replace petroleum.”
affected the business of a downstream ice cutting The state accepted the funds and distributed $3
operator, located in a neighboring state. The ice million from those funds to a state university,
cutter successfully urged his state to bring suit an agency of the state government. The univer-
in the United States Supreme Court against the sity applied the $3 million to its ongoing energy
state operating the power plant, alleging damage research program, which included projects inves-
to its environment and seeking an injunction tigating the creation of synthetic replacements
against the thermal discharge. for petroleum products, solar power, geothermal
power, biomass, and fusion power generation.
What should the United States Supreme Court The Justice Department brings suit in federal
do? district court seeking to enjoin the expenditure
of the funds in the university research programs.
(A) Dismiss the action, because the suit is re-
ally one by the ice cutter against the state How should the court rule?
operating the power plant and is barred by
the Eleventh Amendment. (A) The university expenditure is unconstitu-
tional because Congress has improperly
(B) Hear the matter on the merits, because the delegated its power to spend for the general
ice cutter’s state is suing in its own right welfare to a state government.
and jurisdiction is proper.
(B) The expenditure is unconstitutional as a
(C) Dismiss the action, because it does not have violation of the Supremacy Clause.
original jurisdiction.
(C) The expenditure is constitutional because
(D) Remand the case to the district court for it conforms substantially with the purpose
trial, because it does not have the time to and goals of the federal program.
function as a trial court.
(D) The expenditure is constitutional because
the federal government’s attempt to control
the actions of a state agency violates the
concepts of federalism.
CONSTITUTIONAL LAW - INTERGOVERNMENTAL IMMUNITIES QUESTIONS 3.

Question 5 Question 6

The director of a one-person field station of A state legislature recently enacted a statute
the United States Department of Agriculture imposing a “land utilization tax” on those
(“USDA”) in a small town was instructed by his operating businesses within the state who pay no
superiors to sell surplus government cheese and property taxes. The state seeks to apply the tax
butter to local low income residents at 10% of to the owner of a small restaurant and store that
market value. All sales were conducted at the is located in a national park within the state. The
USDA warehouse next to the field station. owner operates the restaurant and store pursuant
to a contract with the United States Government
Pursuant to state statutes allowing municipal and pays no property taxes.
governments to establish reasonable regulations
governing the retail sale of foodstuffs, the town Which of the following is the owner’s best
in which the field office is located requires any constitutional argument against application of
establishment for the retail sale of food to pass a the tax to her?
health and sanitation inspection and meet other
specified criteria for obtaining a city license. (A) The owner has been denied the privileges
The director of the field office failed to obtain a and immunities of national citizenship,
license from the town and was prosecuted for the protected by the Fourteenth Amendment,
failure. because the state tax impairs her funda-
mental right to conduct business on federal
Which of the following will provide the best lands.
defense for the director in this prosecution?
(B) The tax is unconstitutional as applied to
(A) The ordinance under which the director the owner, because it interferes with inter-
is being prosecuted is invalid as an undue state tourism in violation of the federal
burden upon interstate commerce. commerce power.

(B) The ordinance under which the director is (C) The owner is being denied equal protection
being prosecuted violates the Equal Protec- of law, because those operating businesses
tion Clause of the Fourteenth Amendment. on federal lands in other states are not
subject to the tax.
(C) The ordinance under which the director is
being prosecuted deprives him of property (D) The tax is unconstitutional as applied to the
without due process of law. owner, because under the property power
the federal government has plenary power
(D) The ordinance under which the director is to regulate federal lands.
being prosecuted violates the principles of
intergovernmental immunity as applied to
him.
4. CONSTITUTIONAL LAW - INTERGOVERNMENTAL IMMUNITIES QUESTIONS

Question 7 Question 8

A city’s airport board that oversees a large The federal government owned an old but
international airport in the United States has functional building in a warm southern state.
adopted a policy of reviewing current vendor Its furnace was quite outmoded, but because of
licensees every three years. During the process, the building’s location, the furnace was used
the board reviews customer comments, assesses only a few days a year. However, on the days
the utility of the vendor’s services, and reviews the furnace was used, it emitted a noticeable
the profitability of the vendor (licensees pay rent quantity of smoke and particulate matter. The
based on a percentage of their gross profit). The amount of pollutants emitted on those days
licensee is entitled to present evidence on all of exceeded the state’s stringent environmental
the issues reviewed. regulations. The state brings an action against
the federal government in federal court, seeking
The owner of a franchised currency exchange to enforce the state’s envcironmental standards.
on wheels had a license to operate the cart
within the airport until last week, when his Is the state likely to prevail?
license came up for review. After an appro-
priate hearing, the board refused to renew the (A) Yes, because a state may regulate the qual-
owner’s license mainly because of an excessive ity of its own air under its police power.
number of customer complaints. Nevertheless,
the owner continued operating his cart pursuant (B) Yes, because the states and the federal
to a license granted to him under the Federal government have joint responsibility for
Borders Act that allowed him to operate his cart clean air, and the federal government
at all borders or their functional equivalent. A should give full faith and credit to the
city police officer patrolling the airport asked to state’s laws and regulations in the area.
see the owner’s license. The owner showed his
federal license, but the officer issued the owner a (C) No, because the Supremacy Clause of
citation because he did not have a city license. Article VI prevents the state from imposing
its regulations on the federal government.
Which of the following is the owner’s best
constitutional defense? (D) No, because the emissions from the
building occur so seldom as to be minimal,
(A) The city deprived the owner of his license and the state cannot assert sufficient harm
without due process of law. to the health of its citizens to constitute a
compelling interest.
(B) The licensing scheme was arbitrary and
capricious.

(C) The licensing scheme substantially inter-


feres with interstate and foreign commerce.

(D) The license requirement here was super-


seded by the owner’s federal license.
CONSTITUTIONAL LAW - INTERGOVERNMENTAL IMMUNITIES QUESTIONS 5.

Question 9 (C) No, because, under recent Supreme Court


precedent, allowing a person who is 90% at
A man was injured when the automobile he fault to recover from a person who is 10%
was driving in the state of Green crossed over at fault violates due process.
the yellow line and crashed into an automo-
bile being driven by a woman. The man was a (D) No, because the Tenth Amendment reserves
domiciliary of the state of Green. The woman to the states the right to enforce their strong
was a domiciliary of the state of Orange. The public policies regarding torts and drunk
man suffered severe injuries in the crash, and his driving.
automobile was totally destroyed. The woman’s
automobile was severely damaged, but she Question 10
received only minor cuts and bruises as a result
of the crash. At the time of the crash, the man Congress passed a statute requiring that
was legally intoxicated. all full-time employees, including state and
municipal employees, be paid overtime for any
The woman brought suit against the man in a work over 40 hours per week, or be otherwise
proper Green state court to recover $20,000 for compensated through time off or vacation time.
the damages to her car. The man filed a counter- A small town located in a very warm climate
claim against the woman for $400,000 for his employs only three refuse collectors and cannot
personal injuries and damages to his automobile. afford to hire more. During the warmest times
The state of Green has adopted a pure compara- of the year, extra refuse collections are routine
tive negligence rule. The court found that the because of the fear that rotting garbage in the hot
man was 90% at fault and the woman was 10% climate could contribute to the spread of disease.
at fault and entered a final judgment against the During the hottest months, the three refuse
woman for $22,000 after offsetting her damages collectors in the town regularly worked well
against the man’s damages. beyond the 40 hours per week standard. Because
of normal vacation time and the normally heavy
The man then filed suit in the state of Orange garbage collection schedule, it would not be
to enforce his judgment against the woman. The possible to give the refuse collectors additional
state of Orange has adopted partial compara- time off during the cooler months without hiring
tive negligence by statute. The woman appeared additional refuse collectors. The town brings an
at the state of Orange proceeding and argued action in federal district court, asserting that the
that the Green judgment should not be enforced federal statute requiring overtime pay for munic-
because the state of Orange has a strong public ipal workers is unconstitutional.
policy against both drunk driving and allowing a
tortfeasor who was more than 50% at fault from Should the court find that the statute is consti-
recovering from someone who was less than tutional?
50% at fault.
(A) Yes, as a reasonable application of the Nec-
Must the Orange state court enforce the Green essary and Proper Clause.
state court judgment? (B) Yes, because regulation of state and local
employees is within the ambit of federal
(A) Yes, because the Green state court had ad- power over interstate commerce.
equate and independent grounds on which
to base its judgment. (C) No, because regulation of state and local
employees is an issue reserved to the states.
(B) Yes, because the Green judgment is entitled
to full faith and credit. (D) No, because the state police power to
promote health and safety precludes federal
action in this case.
6. CONSTITUTIONAL LAW - INTERGOVERNMENTAL IMMUNITIES QUESTIONS

Question 11 Question 12

A state passes a law allowing terminally ill A woman elected to a state legislature is
patients to use marijuana under limited circum- prosecuted in federal court for violating the civil
stances. Federal law, however, provides that rights of a member of the janitorial staff of the
marijuana may not be used under any circum- state legislature’s building in connection with a
stances. The state sues in federal district court, sexual assault that occurred in her office. The
arguing that marijuana is beneficial for the woman raises as a defense the claim that the
terminally ill and should be permitted under allegedly wrongful acts were committed in the
these circumstances, despite the federal prohi- course of legislative business, and she is immune
bition of marijuana use throughout the United from scrutiny by the federal courts.
States.
Which of the following is the strongest
How should the federal district court rule? constitutional argument in favor of the woman’s
defense?
(A) The issue should be certified to the court of
appeals immediately because of the impor- (A) The application of federal civil rights stat-
tance of the issue. utes to state legislators would violate the
Due Process Clause.
(B) The issue is a peculiarly local one that is
better addressed on a state level, and should (B) A federal court must follow state law
enjoin enforcement of the federal law. respecting the scope of legislative
immunity.
(C) The federal statute preempts the state
medical marijuana law, and the state law is (C) State legislators enjoy the protection of the
invalid pursuant to the Supremacy Clause. Speech and Debate Clause of the United
States Constitution.
(D) The federal court has no jurisdiction to
hear the case, because a state is a party and (D) Because of the doctrine of federalism,
original jurisdiction lies with the United federal law generally cannot be applied to
States Supreme Court. state legislators acting in the course of their
official duties.
CONSTITUTIONAL LAW - INTERGOVERNMENTAL IMMUNITIES QUESTIONS 7.

Question 13 Question 14

A state passed a law providing that legisla- The federal accounting office issued a call for
tive staff members in the state legislature were competitive bids for a contract to supply 3,000
to receive an hourly wage that was less than four-wheel drive utility vehicles without antipol-
the federal minimum wage. A legislative staff lution devices and with engines with a displace-
member filed suit in federal court challenging ment of 4,000 cubic centimeters. A supplier in a
this state law. state won the contract as low bidder and began
manufacture of the vehicles. However, the state’s
Which of the following is the strongest statutes require that automobiles manufactured
argument in support of the staff member’s in that state be equipped with antipollution
constitutional challenge? devices and have a maximum displacement of
2,500 cubic centimeters. The supplier files suit in
(A) Congress may extend federal minimum state court, seeking a judicial declaration that the
wage requirements to legislative staff mem- state statute may not be enforced as to it.
bers by virtue of the Necessary and Proper
Clause. How should the court rule?

(B) The minimum wage law applies to (A) The statute may not constitutionally be
private sector employees as well as state applied to the supplier because to do so
employees. would violate the Supremacy Clause.

(C) Congress may extend federal minimum (B) The statute may not constitutionally be
wage requirements to state legislative staff applied to the supplier because to do so
members as a means of guaranteeing a would violate the Contracts Clause.
republican form of government.
(C) The statute may not constitutionally be
(D) Congress has plenary power to extend applied to the supplier because to do so
federal minimum wage requirements under would violate the Commerce Clause.
the General Welfare Clause.
(D) The statute may constitutionally be applied
to the supplier.
CONSTITUTIONAL LAW -
INTERGOVERNMENTAL
IMMUNITIES ANSWERS
CONSTITUTIONAL LAW - INTERGOVERNMENTAL IMMUNITIES ANSWERS 1.

CONSTITUTIONAL LAW - INTERGOVERNMENTAL IMMUNITIES ANSWERS

Answer to Question 1

(C) The general principles of intergovernmental immunity prevent federal interference with state
governmental functions. Though there is not a great deal of bite left in the Tenth Amendment
under current case law, particularly with regard to congressional conditions on expenditures of
federal money, this is still the best argument presented. (B) is not the best answer because it
goes too far; the doctrine of federalism relates to the functions of state government, not to all
the actions of the state government. Clearly, the federal government could prevent the state from
exercising its power in such a manner as to deprive its citizens of their federal rights, including
those rights with regard to public education. (A) is obviously wrong, because the doctrine of
federal preemption prevents the states from interfering with the effect and purposes of federal law.
(D) is wrong because it is too narrow. Aside from the problem of separation of church and state,
there are many reasons why the federal government could limit a state’s rights in this area.

Answer to Question 2

(A) As a direct tax on the federal government, the sales tax is invalid unless Congress has consented
to such a tax. (B) is wrong because the tax is on all autos purchased in the state, regardless of
their source, and thus, there is no burden on interstate commerce. (C) is wrong because direct
state taxation of the federal government is invalid whether or not discriminatory, absent the
consent of Congress. (D) is wrong because, as stated, unless Congress consents, a direct tax on
the federal government is invalid. Thus, the fact that there is a rational basis and that the tax is
not a penalty does not matter: The question is one of the power to tax, not whether the tax itself is
appropriate.

Answer to Question 3

(B) The Court should hear the case on the merits. A state can sue another state to protect its natural
resources for the benefit of its own citizens. [Pennsylvania v. West Virginia (1923)] The ice
cutter’s state is suing the state operating the power plant under this doctrine. Under Article III,
the Supreme Court has original and exclusive jurisdiction over controversies between two states.
Because this is a controversy between two states, Supreme Court jurisdiction is proper. (A) is
incorrect. The ice cutter’s state is a real party in interest, and the Eleventh Amendment, which
bars suits by citizens of one state against another state in federal court, is inapplicable. (C) is
incorrect. Article III, Section 2(2) of the Constitution confers exclusive original jurisdiction to
the Supreme Court over all controversies between two or more states. (D) is incorrect. Article III,
Section 2 of the Constitution confers exclusive original jurisdiction on the Supreme Court in cases
between two or more states. In such cases, the Court must serve as a trial court and even Congress
may not limit that jurisdiction. [See Marbury v. Madison (1803)]

Answer to Question 4

(B) The court should rule that the expenditure is unconstitutional. The federal power to tax and
spend includes the power to attach conditions to the expenditures, and to the extent the univer-
sity research program conflicts with the federal legislation, the state action is superseded by the
federal statute. Here, because the federal program expressly limited use of the distributed funds
to development of synthetic materials to replace petroleum, the university projects involving
solar, geothermal, biomass, and fusion energy would be in conflict with the statutory prescription.
2. CONSTITUTIONAL LAW - INTERGOVERNMENTAL IMMUNITIES ANSWERS

“Substantial” conformity is not enough, so (C) is incorrect. (D) is an incorrect statement of the
law. The supremacy doctrine permits Congress to effectively control the actions of a state in
spending money provided by the federal government. (A) is incorrect. There is nothing unconsti-
tutional about the means by which the funds are disbursed.

Answer to Question 5

(D) The town licensing requirement unconstitutionally impinges upon a duly authorized federal
program. The United States Government, as well as its agencies and instrumentalities, is immune
from state regulation that interferes with federal activities, functions, and programs. To the extent
that state regulations substantially interfere with an authorized federal program, the state laws
must yield. Here, the director, as an agent of the federal government, was carrying out a duly
authorized program of the Department of Agriculture by conducting sales of surplus government
food at a federally owned warehouse. To sustain the power of the town to prosecute the director
for not having a retail food sale license would give the town overriding authority over the selection
of personnel to administer a federal program, as well as over the means by which this program is
to be implemented. Thus, the licensing requirement would substantially interfere with the proper
functioning of this federal program by directly interfering with a federal employee in the carrying
out of his orders. (A) is incorrect because the facts do not indicate that the licensing requirement
is in any way a measurable burden upon interstate commerce, much less an undue burden. The
licensing regulation appears to affect, almost exclusively, the peculiarly local concerns of health
and sanitation. (B) is incorrect because an equal protection violation exists where a law unrea-
sonably limits the liberty of some persons but not others, i.e., where a law treats similar persons
in a dissimilar manner. The licensing regulation at issue here is apparently being applied in an
evenhanded fashion, and the director is not being treated differently from anyone else who does
not have the required license. Thus, there is no equal protection problem. (C) is incorrect because
the director is not being deprived of any property or interest to which he has a legitimate claim;
e.g., he is not being deprived of employment to which he is entitled. Because there is no depriva-
tion of property, there is no due process issue.

Answer to Question 6

(D) The best argument is that the tax interferes with the plenary power of Congress to regulate federal
lands. Article IV, Section 3 of the Constitution gives Congress the power to make all necessary
rules and regulations concerning property belonging to the federal government. The owner would
argue that this power, when combined with the Supremacy Clause (which makes federal law the
supreme law of the land, superseding conflicting state law) would prevent any attempt by a state
to tax persons on federal land, absent congressional consent to the tax. In fact, this argument will
probably fail because the Court has indicated that while the states may not directly tax or regulate
the federal government, they may indirectly do so by adopting taxes or regulations on persons
dealing with the federal government, as long as the tax or regulation does not unduly burden the
federal government. The tax here seems to fall into the permissible category; nevertheless, this
is the owner’s best argument. (A) is not a good argument because the Privileges or Immunities
Clause of the Fourteenth Amendment does not protect the right to conduct business on federal
lands. The Clause protects a narrow range of privileges of United States citizenship, such as the
right to enter onto federal lands, the right to petition Congress for redress, and the right to vote
for federal officers. The Clause does not protect the entire Bill of Rights against infringement, and
the owner’s right here falls outside the scope of the Clause. (B) is incorrect because the commerce
power would not preempt the tax here. Generally, a state may adopt taxes that will affect interstate
commerce as long as the tax does not discriminate against interstate commerce (the tax here does
CONSTITUTIONAL LAW - INTERGOVERNMENTAL IMMUNITIES ANSWERS 3.

not), and (i) there is a substantial nexus between the state and the taxpayer (here, the owner does
business on land within the state); (ii) the tax is fairly apportioned (the tax here is fair since it
merely equalizes the tax burden on users of land within the state); and (iii) there is a fair relation-
ship between the tax and the benefits provided by the state (this requisite is met because the owner
gets the protection of doing business in the state in exchange for the tax). (C) is incorrect because
there is no equal protection violation here. The Equal Protection Clause merely prohibits the states
from treating similar people in a dissimilar manner without a valid reason. When, as in this case,
the law does not involve a suspect or quasi-suspect class or a fundamental right, the law will be
upheld as long as there is a rational basis for the discrimination. Here, it is questionable whether
the law discriminates, since it is applicable to all persons using land in the state. Assuming that
the law does discriminate on the basis of the location of land, the discrimination is permissible
because it is rationally related to a legitimate state purpose (a state may impose taxes to raise
revenues but may not impose a tax on land or activities outside of the state).

Answer to Question 7

(D) The owner’s best defense is that the license requirement was superseded by the owner’s federal
license. The Supremacy Clause makes federal law the supreme law of the land. This means
that whenever a valid federal law conflicts with a state law, the state law is inapplicable and the
federal law controls. The federal law granting the owner a license here would be valid pursuant to
Congress’s power to regulate interstate and foreign commerce. In addition to actually regulating
such commerce, Congress can adopt any law necessary or proper to implement its power, and
providing licenses to vendors to ensure that people can exchange money at the border and its
functional equivalents would certainly be within the scope of the broad federal power. Because
the city license requirement interferes with the federal licensing scheme, it cannot be enforced
against the owner. (A) is incorrect because the owner appears to have had all the process that was
due. It may be that the owner had a property interest in his license under the Fourteenth Amend-
ment Due Process Clause in that he could expect to keep the license as long as he performed
well. He could not be deprived of his property interest in his license without due process of law,
including a hearing. Here, he was given a hearing prior to revocation of his license at which he
was allowed to present relevant evidence, and nothing in the facts indicates that the board was
biased against the owner. Thus, due process was satisfied. (B) is incorrect because nothing in
the facts indicates that the licensing scheme was arbitrary. An arbitrary licensing scheme would
violate substantive due process, which requires at a minimum that laws be rational. Note also that
this choice is something of a red herring—the “arbitrary and capricious” standard is the standard
that courts use in reviewing determinations of fact by administrative agencies that are not “on
the record.” (C) is incorrect because nothing in the facts indicates that the city’s licensing scheme
interferes with foreign or interstate commerce—nothing in the scheme favors local economic
interests over out-of-state or foreign interests, so there is no unlawful discrimination, and the
scheme appears to place little if any burden on interstate or foreign commerce.

Answer to Question 8

(C) The United States will win because of the Supremacy Clause. Where the federal government and
the state government share power to regulate a particular subject matter, the Supremacy Clause
provides that federal law is the supreme law of the land, and consequently a conflicting state law
will be superseded by a federal law. A corollary of this rule is that the states have no power to
regulate the legitimate activities of the federal government unless Congress consents. Running a
government building is a legitimate governmental activity, and so the Supremacy Clause prevents
the state regulation here. (A) is incorrect because although regulation of air quality is certainly
4. CONSTITUTIONAL LAW - INTERGOVERNMENTAL IMMUNITIES ANSWERS

within the scope of a state’s police power (the power to legislate for the health, safety, and morals
of the community), it does not allow the state to regulate the legitimate activities of the federal
government. An immunity is afforded the federal government by the Supremacy Clause, and the
state’s police power cannot overcome that immunity. (B) is incorrect because while it is true that
the power to regulate air quality is shared, this means that when the federal government acts,
conflicting state law will be superseded by virtue of the Supremacy Clause. The federal govern-
ment does not have to defer to the state. (D) is incorrect because it is based on an inappropriate
premise—that the state’s interference would be justified if it were related to a compelling interest.
The strength of the state’s interest is irrelevant under the Supremacy Clause.

Answer to Question 9

(B) The court should enforce the Green state court judgment on full faith and credit grounds. Under
the Full Faith and Credit Clause of the United States Constitution, states must give full faith and
credit to the judgment of another state if the judgment is final, on the merits, and rendered by a
court with jurisdiction. These requirements appear to have been satisfied here. (A) is incorrect
because it is irrelevant. The doctrine of adequate and independent grounds comes into play when
the United States Supreme Court is asked to review a state court ruling in a case that might have
constitutional dimensions, but has been disposed of on state law grounds that are adequate to
support the judgment and independent of any federal law. (C) is incorrect because no such prece-
dent exists. (D) is incorrect because the Tenth Amendment provides that powers not delegated to
the United States Government nor prohibited to the states are reserved to the states. Here, opera-
tion of the Full Faith and Credit Clause effectively prohibits the state of Orange from enforcing its
public policy because the man obtained a final judgment on the merits of his counterclaim against
the woman in a Green state court with jurisdiction. There is no public policy defense to the Full
Faith and Credit Clause.

Answer to Question 10

(B) Congress may constitutionally regulate wages and hours of state and local employees under its
commerce power. The Commerce Clause of Article I, Section 8 vests in Congress broad powers to
regulate any activity, local or interstate, which either in itself or in combination with other activi-
ties has a substantial economic effect upon, or effect on movement in, interstate commerce. Under
this approach, Congress clearly has the power to regulate wages and hours of those employed by
private employers. [See United States v. Darby (1941)] This power has been held applicable to
state and local governments; Congress can therefore require state or local governments to follow
the provisions of federal legislation requiring a state or private employer to pay overtime wages
to its employees. [Garcia v. San Antonio Metropolitan Transit Authority (1985)] Thus, the town
will not succeed in its challenge to the statute. (A) is incorrect because the statute’s constitution-
ality is not dependent on the Necessary and Proper Clause, which permits Congress to exercise
auxiliary powers that are “necessary and proper” for carrying out Congress’s enumerated powers.
The power to require state and private employers to pay overtime wages to their employees comes
directly from Congress’s enumerated power to regulate commerce among the states. (C) is incor-
rect because the Court has abandoned former holdings that the Tenth Amendment precluded
Congress from regulating areas involving the traditional functions of state and local governments.
A court is unlikely to strike down on Tenth Amendment grounds a law such as the one here
that subjects state and local governments to regulations that are equally applicable to the private
sector. (D) is incorrect because the state police power does not prevail over a federal statute based
on Congress’s power over interstate commerce. The town’s health and safety concerns regarding
the garbage can only be resolved by paying overtime wages or hiring additional employees.
CONSTITUTIONAL LAW - INTERGOVERNMENTAL IMMUNITIES ANSWERS 5.

Answer to Question 11

(C) The court should rule that the federal statute preempts the state law. Under the Supremacy Clause,
a valid federal regulation supersedes any state action that conflicts with it, as is the case here. (A)
is wrong because there is no basis for certification under the facts presented. While (B) might
state an alluring policy choice, it is both outside the purview of the court and inappropriate in the
face of the express determination by Congress to preempt local control. (D) is incorrect because
while the Constitution gives the Supreme Court original jurisdiction over cases in which a state
is a party, that jurisdiction is not exclusive. Congress has given the lower federal courts concur-
rent jurisdiction over all cases in which the Supreme Court has original jurisdiction except those
between states. This is a case between the United States and a state. Therefore, the Supreme
Court’s jurisdiction is not exclusive and the case can be heard in a lower federal court.
Answer to Question 12
(D) The doctrine of federalism generally bars applying federal law to state legislators in the course of
their duties. The general principles of intergovernmental immunity prevent federal interference with
state governmental functions. In these facts, although the assault occurred in the state legislator’s
office, there is no indication that she was conducting state governmental duties at the time of the
assault and, therefore, while this would be her strongest constitutional defense, it would not be appli-
cable under these facts. There is no indication of any violation of due process, and federal courts
need follow state law only in cases where jurisdiction is based on diversity of citizenship or when a
federal statute specifically incorporates state law. Thus, (A) and (B) are incorrect. (C) is incorrect
because the protection of the Speech and Debate Clause extends only to federal legislators.
Answer to Question 13
(B) The strongest argument is that the minimum wage law applies to both private sector and state
employees. Generally, the Tenth Amendment provides that powers not delegated to the federal
government are reserved to the states. Under the Tenth Amendment, the court will usually uphold
federal legislation that applies both to the private sector and to state governments, but will gener-
ally not uphold attempts by Congress to regulate the states as states. Here, the federal minimum
wage law applies to all private and government employers. Thus, it will be upheld. (C) is therefore
incorrect because extending minimum wage laws to legislative staffers does not interfere with the
Constitutional guarantee of a republican form of government. (A) is incorrect because the Neces-
sary and Proper Clause alone does not furnish authorization for federal laws. Congress must be
acting in the exercise of one of its express powers, and (A) only deals with the Necessary and
Proper Clause. (D) is incorrect because the General Welfare Clause is a congressional taxing and
spending authorization and is not related to minimum wage requirements.
Answer to Question 14
(A) Application of the statute would violate the Supremacy Clause. Because the state statute conflicts
with the terms of the federal contract, the federal contract must take precedence, pursuant to the
Supremacy Clause and principles of federal immunity from state regulation. (B) is not a good
answer because there are no facts indicating that the statute was enacted after the contract was
formed; in fact, the problem implies that the reverse is true. (C) is not the best answer because,
although it is possible that a state’s regulation of the manufacture of goods might affect interstate
commerce so as to invoke the commerce power, the facts of this problem do not indicate any
likelihood of such a circumstance and are insufficient to be conclusive anyway. (D) is incorrect.
Thus, (A) is the only correct answer.
CONSTITUTIONAL LAW -
JUDICIAL REVIEW
QUESTIONS
CONSTITUTIONAL LAW - JUDICIAL REVIEW QUESTIONS 1.

CONSTITUTIONAL LAW - JUDICIAL REVIEW QUESTIONS

Question 1 Question 2

Congress has enacted a law providing that A man was arrested in a state for armed
all disagreements between the United States robbery. A combined preliminary hearing to
and a state over federal grant-in-aid funds shall determine probable cause and initial appearance
be settled by the filing of a suit in the federal was held within 20 hours of his arrest. Probable
district court in the affected state. The law cause was found, and bail was properly denied
further provides: “The judgment of the federal under the state’s Bail Reform Act. A state statute
court shall be transmitted to the head of the provided that when a defendant is in custody,
federal agency dispensing such funds, who, if his trial must begin within 50 days of his arrest.
satisfied that the judgment is fair and lawful, After 50 days had passed since the man’s arrest
shall execute the judgment according to its and no trial had been held, he filed a motion for
terms.” dismissal for violation of his right to a speedy
trial under the state constitution, which tracked
Is this law constitutional? verbatim the speedy trial provision of the United
States Constitution. The trial judge held that
(A) Yes, because disagreements over federal he was bound to follow federal interpretations
grant-in-aid funds necessarily involve fed- of the speedy trial provision and granted the
eral questions within the judicial power of man’s motion on that basis. On appeal, the state
the United States. supreme court agreed with the trial judge. The
state prosecutor seeks to challenge the ruling in
(B) Yes, because the spending of federal the United States Supreme Court.
monies necessarily includes the authority
to provide for the effective settlement of If the Supreme Court thinks that the state
disputes involving them. court wrongly decided that the man was
denied his right to a speedy trial under federal
(C) No, because it vests authority in the federal standards, how should it proceed?
court to determine a matter prohibited to it
by the Eleventh Amendment. (A) Reverse the decision because the state
speedy trial provision cannot be interpreted
(D) No, because it vests authority in a federal in a manner different from federal interpre-
court to render an advisory opinion. tations.

(B) Reverse the decision and remand it to state


court because the state speedy trial issue
was so intertwined with the federal question
that it would be difficult to determine on
which ground the state court relied.

(C) Decline jurisdiction because the Eleventh


Amendment prohibits a state from
challenging a decision of its supreme court
in federal court.

(D) Reverse the decision and remand the case


to be decided on the independent state
grounds only.
2. CONSTITUTIONAL LAW - JUDICIAL REVIEW QUESTIONS

Question 3 Question 4

To prevent the Supreme Court from whittling A pregnant woman wished to obtain an
away the protections that previous Supreme abortion, but she was poor and could not afford
Court decisions had created for individuals one. Her home state did not provide financial
accused of crimes, Congress passed a law elimi- assistance for abortions, but a neighboring state
nating from Supreme Court jurisdiction all cases did for women who had been living within the
in which a state supreme court has decided that state for at least three months prior to the proce-
a defendant’s federal constitutional rights have dure. The woman went to see a doctor in the
been violated. neighboring state. He told her of the residency
requirement. He also told the woman that he
If the statute is held unconstitutional, what is thought the residency requirement was unconsti-
the most likely reason? tutional and suggested that the woman bring an
action in federal court challenging the residency
(A) The determination of the extent of constitu- requirement. She complied, bringing an action
tional rights is precisely the domain of the in forma pauperis, naming the doctor as the only
Supreme Court. defendant. The doctor responded that he believed
that the requirement was unconstitutional and
(B) Congress’s power to limit jurisdiction would like to be able to perform the abortion for
applies only to cases originating in the the woman.
federal courts.
How should the court proceed?
(C) To be effective, the action taken by
Congress here would require a constitu- (A) Dismiss the action because the woman
tional amendment. lacks standing.
(D) Congress may not eliminate all avenues (B) Dismiss the action because there is no case
for Supreme Court review of issues vested or controversy.
within the judicial power of the federal
courts. (C) Abstain from hearing the action because
deciding whether to fund abortions is a
highly political issue.

(D) Issue a declaratory injunction upholding the


residency requirement because the state has
a compelling interest in preventing nonresi-
dents from draining local welfare funds.
一 信:liuxue119118 , 们 信免 供

CONSTITUTIONAL LAW - JUDICIAL REVIEW QUESTIONS 3.

Question 5 Question 6

In which of the following cases does Congress A state legislature adopted a statute requiring
have the power to restrict the jurisdiction of the that state school districts be funded equally on a
United States Supreme Court? per capita basis, because the previous method of
funding school districts based on the amount of
(A) A case involving an ambassador. taxes paid by residents of the district resulted in
schools in the state’s wealthiest district receiving
(B) A case involving a dispute between two twice as much funding per pupil as did schools
states. located in poorer districts. A resident of the
wealthy district whose children attend public
(C) A case involving maritime jurisdiction. schools brought an action in state court to have
the new statute declared invalid. He estab-
(D) A case involving a state and the federal lished at trial that the disparities in the previous
government. funding system were not based in any way on
racial or ethnic discrimination. Nevertheless,
when the case reached the state supreme court,
it ruled that, based on a provision in the state
constitution similar to the Fourteenth Amend-
ment Equal Protection Clause of the United
States Constitution, all school districts in the
state must be funded equally on a per capita
basis.

Subsequent to this decision, a taxpayer


in a neighboring state sued in federal court,
demanding equal per capita funding in his state’s
school districts. The taxpayer’s case eventually
reached the United States Supreme Court, which
ruled that the Fourteenth Amendment does
not compel equal funding, provided there is no
probable racial discrimination in the funding.
After that decision, the resident of the wealthy
district who had lost his state court case filed
a petition for a writ of certiorari to have the
decision by his state supreme court overturned.

Is the resident likely to prevail?

(A) Yes, because the Supremacy Clause renders


the state decision invalid.

(B) Yes, because the issue is res judicata.

(C) No, because the state decision turned on


state law grounds.

(D) No, because the resident lacks standing.

一 信:liuxue119118 , 们 信免 供
4. CONSTITUTIONAL LAW - JUDICIAL REVIEW QUESTIONS

Question 7 Question 8

A duly enacted federal statute sets aside a A state enacted a statute requiring the parents
$5 million grant to study whether after-school of every child to have the child vaccinated
dance classes could help prevent teenagers from for smallpox before the child’s third birthday.
joining gangs. A nonpartisan taxpayer’s group Failure to comply was a misdemeanor punish-
has filed suit in federal court against appropriate able by a fine of $500 or six months in the
federal officials to enjoin their expenditure of the county jail. The parents of a nine-month-old
$5 million provided by the statute. The group’s infant living within the state objected to applica-
primary purpose is to restore fiscal integrity in tion of the statute to their child on the ground
the federal government. The suit challenges the that any injections or vaccinations violated the
statute as an improper exercise of congressional tenets and beliefs of their religion. The parents
power. have commenced an action in federal court
to declare the statute unconstitutional on the
Which of the following best reflects the likely ground that it violates their right to the free
result of the case on a proper challenge? exercise of their religion under the First Amend-
ment to the United States Constitution.
(A) The court will dismiss the case because a
federal court lacks jurisdiction to enjoin What is the best argument to defeat their
expenditure of funds voted by Congress, action?
under the doctrine of separation of powers.
(A) A proceeding for declaratory judgment is
(B) The court will dismiss the case because not the proper vehicle for asserting this
the nonpartisan group lacks standing to claim.
maintain the action.
(B) The state legislature has repeatedly defeated
(C) The court will hear the case on its merits bills to repeal the statute.
and will find that the grant is a valid
exercise of the federal taxing and spending (C) There is no substantial threat that the
powers. statute will be enforced.

(D) The court will hear the case on its merits (D) The federal courts should abstain until the
and will find that the grant is an invalid state courts have had an opportunity to
exercise of the federal taxing and spending construe the statute.
powers.
CONSTITUTIONAL LAW - JUDICIAL REVIEW QUESTIONS 5.

Question 9 Question 10

A businessperson held a majority ownership The leader of a controversial religious sect,


interest in one of the three daily newspapers in which had been characterized by the govern-
a city, as well as numerous other media outlets ment as a “religious cult,” sought to broaden his
across the country. When a popular radio station influence by running for the state legislature.
in the city was put on the market, the buisness- The state had no provision barring members
person entered into negotiations to purchase it. of the clergy from serving as legislators. The
However, the Federal Communications Commis- leader took advantage of the state’s liberal voter
sion (“FCC”) blocked the sale on the basis of registration laws by having many of his followers
a regulation forbidding ownership of a radio or move into the district a month before the
television station to any entity owning a daily election. The leader’s subsequent victory in the
newspaper in the same city. The next year, race prompted a citizens’ group to file a lawsuit
however, the FCC suspended enforcement of in federal district court. The suit, invoking the
that regulation to permit the radio station to be Establishment Clause, sought to enjoin the state
sold to the owner of another daily newspaper in legislature from seating the leader in the legisla-
the city. The businessperson filed an action in ture or expending any state funds to provide him
federal district court seeking to enjoin the sale. with legislative privileges. The legislature moved
to dismiss the lawsuit.
What should the court do?
Should the court grant the legislature’s
(A) Decide the case on the merits, because motion?
the businessman can claim that the FCC’s
unequal treatment of the two transactions (A) Yes, because the Eleventh Amendment
violated the Due Process Clause of the bars this type of action against a state by its
Fifth Amendment. citizens.

(B) Decide the case on the merits, because (B) Yes, because a political question is
the businessman can claim that the FCC’s involved.
refusal to allow him to own the station
violated his freedom of speech rights under (C) No, because citizens have standing to
the First Amendment. challenge government expenditures that
may violate the Establishment Clause.
(C) Dismiss the action, because the
businessman cannot show that enjoining the (D) No, because the lawsuit is not seeking
transaction will eliminate any injury that he damages from the state.
might have suffered.

(D) Dismiss the action, because the federal


government has the power to regulate
ownership of the broadcast media.
6. CONSTITUTIONAL LAW - JUDICIAL REVIEW QUESTIONS

Question 11 Question 12

Congress adopted legislation prohibiting the A state statute made it unlawful to sell soft
federal courts from ordering busing as a remedy drink beverages in any type of container except
for past racial discrimination in a public school refundable glass containers. A violation of the
district. statute constituted a misdemeanor, subjecting the
violator to a fine of up to $1,000.
Which of the following is the strongest
argument that the federal legislation is unconsti- An out-of-state corporation manufacturing
tutional? and selling aluminum soft drink cans in the
above state brought suit in federal district court
(A) The courts, not Congress, have the primary contesting the constitutionality of the statute as
responsibility for defining the minimum re- an undue burden on interstate commerce under
quirements of the Equal Protection Clause the Commerce Clause.
of the Fourteenth Amendment.
What should the court do?
(B) The Privileges or Immunities Clause of the
Fourteenth Amendment prohibits Congress (A) Deny a hearing, because the Eleventh
from limiting the forms of relief afforded Amendment forbids the exercise of judicial
by federal courts. power by the federal government in a suit
prosecuted against any state by a citizen of
(C) Congress cannot limit the authority of another state.
federal courts to hear and decide cases
properly presented for decision. (B) Deny the hearing, because only state courts
may decide the constitutionality of state
(D) The legislation unduly burdens interstate laws.
commerce.
(C) Grant a hearing, because a federal question
is involved.

(D) Grant a hearing, because jurisdiction exists


on the basis of diversity.
CONSTITUTIONAL LAW - JUDICIAL REVIEW QUESTIONS 7.

Question 13 Question 14

Congress created a tribunal by statute, which During summer months, thousands of tourists
was empowered to review claims made by regularly visited a state’s capital to see its many
clients of a federal agency and make recommen- sites. Because the city had few outdoor water
dations to the agency regarding their merits. A fountains, a number of vendors had taken advan-
lawyer was properly appointed to the tribunal. tage of the situation by selling water out of
Six years later, the tribunal was abolished by coolers set up on the sidewalks of the city.
repeal of the authorizing legislation, and the
lawyer was offered an administrative position in After receiving several complaints regarding
the transportation department at a lower salary. the clutter associated with the water vendors,
The lawyer brought an action against the federal the city council passed an ordinance, effective
government on the grounds that Congress may immediately, prohibiting persons from selling
not remove a federal judge from office during bottled water out of coolers on the sidewalks
good behavior, nor decrease his salary during between the hours of 8 a.m. and 6 p.m., Monday
continuance in office. Counsel for the govern- through Friday.
ment made a motion to dismiss.
A vendor had been selling water on the
How should the trial court rule? sidewalks of the city for the past three years.
He sold his water for $1 less per bottle than
(A) For the government, because the lawyer other water vendors because his water bottles
was not an Article III judge and is not en- were adorned with religious messages, and his
titled to life tenure. reduced price helped the dissemination of his
religious message. Sixty percent of the vendor’s
(B) For the government, because the lawyer sales were made on weekdays between 8 a.m.
lacked standing to raise the question. and 6 p.m. On learning of the ordinance, the
vendor sought to enjoin its enforcement in
(C) For the lawyer, because he has established federal district court on the basis that it is uncon-
a property right to his position on the stitutional.
tribunal.
In this suit, the court will probably:
(D) For the lawyer, because of the indepen-
dence of the federal judiciary constitution- (A) Reach the merits of the vendor’s challenge
ally guaranteed by Article III. to the ordinance, because it interferes with
his right to earn a living under the Privileg-
es and Immunities Clause of Article IV.

(B) Reach the merits of the vendor’s challenge,


because enforcement of the ordinance will
interfere with his business, and the rights of
the public are linked to the vendor’s rights.

(C) Decline to hear the case, because the


ordinance has not yet been enforced against
him.

(D) Decline to hear the case, because the


ordinance constitutes a reasonable time,
place, and manner restriction on speech and
association.
CONSTITUTIONAL LAW -
JUDICIAL REVIEW
ANSWERS
CONSTITUTIONAL LAW - JUDICIAL REVIEW ANSWERS 1.

CONSTITUTIONAL LAW - JUDICIAL REVIEW ANSWERS

Answer to Question 1

(D) The law is unconstitutional because it gives the federal district courts authority to render advisory
opinions. The Supreme Court has interpreted the constitutional power of the federal courts to hear
“cases and controversies” to mean that federal courts may not render advisory opinions. Under
the law here, the district court’s decision is not binding on an agency dispensing funds, because
the agency head is given discretion to decide whether the court’s judgment is fair. Therefore,
the judgment is merely advisory and so is not within the jurisdiction of the federal courts. (A) is
incorrect because while it is true that the cases would present a federal question, it is not enough
merely that a federal question exists; there must be a case or controversy. (B) is incorrect for the
same reason. Congress certainly has authority to provide for the settlement of disputes under
federal grant programs, but that authority does not permit Congress to violate the case or contro-
versy requirement. (C) is incorrect because the Eleventh Amendment only prohibits the federal
courts from hearing an action by a citizen of a state against his state; actions between the federal
government and a state are not barred.

Answer to Question 2

(D) The best way to approach this question is to eliminate the wrong answers first. (A) is incorrect
because state constitutional provisions do not have to be interpreted exactly the same as federal
provisions; the federal Constitution provides the minimum rights that states must provide, but
states are free to grant broader rights. Thus, even though a 50-day delay may be constitutional
under the federal Constitution, it can still be held unconstitutional under a state constitution. (B) is
incorrect because the facts make it clear that the state court was relying on federal case interpre-
tations. Moreover, if the Supreme Court could not decide whether the case was based on federal
or state grounds, it would not reverse the case, because a federal court will not hear a case that
can be based on adequate and independent state grounds; rather, it would dismiss the case or
remand it to the state for clarification. (C) is incorrect because the Eleventh Amendment gener-
ally prohibits federal courts from hearing actions by a private party or foreign government against
the state government; it does not bar a state from appealing a ruling from its own court system.
Thus, (D) is correct. The Supreme Court had jurisdiction to hear the case, because it has jurisdic-
tion to hear appeals from a state’s highest court concerning the constitutionality of a state statute,
and as indicated above, the state court’s decision was not based on independent state grounds; the
decision was based on federal case law interpreting an identical federal provision. Thus, jurisdic-
tion was proper and the Court could reverse the state court decision and hold that a 50-day delay
does not violate the federal Constitution. However, the case should be remanded so that the state
may decide whether the delay was too long under state law, since a state is free to provide its
citizens with more civil protection than is required by the federal Constitution.

Answer to Question 3

(D) The most likely ground for holding the statute unconstitutional is that Congress may not limit all
avenues for Supreme Court review of federal constitutional issues. Article III, Section 2 provides
that the Supreme Court shall have appellate jurisdiction under such regulations as the Congress
shall make. Although Ex parte McCardle (1868) gives Congress broad power to regulate the
Supreme Court’s appellate jurisdiction, it has been suggested that Congress may not eliminate
all avenues for Supreme Court review of issues vested within the federal judicial power. This
argument is the most likely basis for finding the law here unconstitutional because none of the
2. CONSTITUTIONAL LAW - JUDICIAL REVIEW ANSWERS

other choices is viable. (A) is not the best argument. As indicated above, Congress has the power
to regulate the appellate jurisdiction of the Supreme Court, and since the Supreme Court’s
primary role as an appellate court is to determine constitutional issues, it follows that Congress
can eliminate from the Supreme Court’s review the determination of certain cases involving
constitutional issues, as long as jurisdiction remains in some lower federal courts. (B) is incor-
rect because Congress’s power is limited to regulating the Supreme Court’s appellate jurisdiction;
it has no power to restrict or enlarge the Court’s original jurisdiction. (C) is incorrect because
Congress has the authority to adopt laws modifying appellate jurisdiction; modification of the
Court’s original jurisdiction would require a constitutional amendment.

Answer to Question 4

(B) The federal court should dismiss the action because there is no case or controversy. The federal
courts will not issue advisory opinions and so will not hear collusive actions. The fear is that if
interested parties are not on both sides of an issue, the court will not have an opportunity to fairly
address all of the sides to each issue. Here, there is no interested party opposing the pregnant
woman, and so the federal court should dismiss. (A) is incorrect because the woman has standing.
To have standing, a person must have a concrete stake in the outcome of the controversy. A
plaintiff will have to show an injury in fact caused by the government that can be remedied by a
decision in her favor. The woman here would be able to get an abortion if the statute is stricken,
and so she has standing. (C) is incorrect because a political question is not involved. A political
question is an issue committed by the Constitution to another branch of the government or an
issue inherently incapable of resolution and enforcement by the political process. Whether a
residency requirement is constitutional is not a political question. (D) is incorrect. When a state
uses a durational residency requirement (a waiting period) for dispensing benefits, the govern-
ment usually must show that the requirement is tailored to promote a compelling interest because
it interferes with an individual’s fundamental right to migrate from state to state. However, even
assuming that the community interest in preventing nonresidents from draining local welfare
funds is compelling, the court should decline jurisdiction because there is no case or controversy
here.

Answer to Question 5

(C) Congress has the power to limit the Supreme Court’s jurisdiction in maritime cases. The Supreme
Court has original jurisdiction in all cases affecting ambassadors, other public ministers and
consuls, and those in which a state shall be a party. Congress may neither restrict nor enlarge the
Supreme Court’s original jurisdiction. Thus, (A), (B), and (D) are incorrect. In all other cases,
such as maritime disputes, the Supreme Court has only appellate jurisdiction, which Congress has
extensive power to regulate and limit. Hence, (C) is correct.

Answer to Question 6

(C) The resident will not prevail because the state decision was based on state law grounds. The
Supreme Court will hear a case from a state court only if it turned on federal grounds. If it finds
adequate and independent state grounds for the decision, it will refuse jurisdiction. Here, the
facts state that the state decision was based on a provision of the state constitution. The fact that
the state provision is similar to the federal Equal Protection Clause is irrelevant to determining
whether the decision here was based on state law, since the state court did not base its decision
on interpretation of the federal provision, but rather interpreted the state provision. Therefore, the
Supreme Court will refuse jurisdiction, and the resident will not prevail. (A) is incorrect because
CONSTITUTIONAL LAW - JUDICIAL REVIEW ANSWERS 3.

the Supremacy Clause renders state laws invalid under implied preemption rules only if they
actually conflict with federal law, they prevent achievement of federal objectives, or Congress
has preempted the field. Nothing in the facts indicates that Congress has preempted the issue of
school funding, and the state decision is not in conflict with the Supreme Court decision here and
does not prevent achievement of the federal objectives; the Supreme Court held that the federal
Equal Protection Clause does not require equal funding, not that equal funding violates equal
protection. Thus, the state court was free to grant its citizens more protection under the state
equal protection provision than is granted under the federal provision. (B) is incorrect because
res judicata is not a constitutional issue. Moreover, the Supreme Court did not decide whether
equal funding was invalid, only that it was not required. (D) is incorrect because the resident
has standing. To have standing, a party must have a concrete stake in the outcome of a contro-
versy. This requires an injury in fact caused by the government action that can be remedied by
a decision in the litigant’s favor. The resident has an injury in fact since his children are being
deprived of educational money under the new state statute. This injury could be remedied by a
decision in his favor, because if the statute is held invalid, the state would go back to the unequal
funding system.

Answer to Question 7

(B) The court will likely hold that the nonpartisan taxpayer’s group lacks standing to maintain the
action because its interest is too remote. As an organization representing the interests of its
members, the nonpartisan group would have standing to challenge government actions that injure
the organization or its members. A party must have standing to mount a constitutional challenge;
i.e., the party must demonstrate a concrete stake in the outcome of a controversy. To show the
existence of such a stake, the plaintiff must be able to assert that it is injured by a government
action or that the government has made a clear threat to cause injury to it if it fails to comply with
a law, regulation, or order. Also, the plaintiff must show that the injury in fact will be remedied
by a decision in its favor. As a general rule, people do not have standing as taxpayers to challenge
the manner in which the federal government spends tax dollars, because their interest is too
remote. Under an exception to this general rule, federal taxpayers have standing to challenge
federal appropriation and spending measures if they can establish that the challenged measure:
(i) was enacted under the taxing and spending power of Congress; and (ii) exceeds some specific
limitation on the power. The only such limit found by the Court to date on the taxing power is
the Establishment Clause. Here, the nonpartisan group of taxpayers is trying to challenge the way
in which the federal government is spending tax money, which falls within the general rule that
such people cannot demonstrate a sufficient stake in the outcome of the controversy as to confer
on them standing. In addition, the group cannot claim standing under the exception to this rule,
because the challenge raised by the group does not allege that the Establishment Clause, or any
other specific limitation on the taxing and spending power, has been exceeded. (A) is incorrect
because, although there is no explicit constitutional statement of the power of federal courts to
determine the constitutionality of acts of other branches of government, such judicial review of
other branches of the federal government was established in Marbury v. Madison. The Constitu-
tion is law, and the judiciary has the authority and duty to declare what the law is. For instance, if
a plaintiff can show that a particular spending measure exceeds the limitations of the Establish-
ment Clause, a federal court has the power to prevent such an unconstitutional expenditure of
funds. (C) is not as good an answer as (B) even though, if the court were to reach the merits of the
case, it would likely uphold the spending measure. Pursuant to the taxing and spending powers,
Congress may lay and collect taxes, as well as spend to provide for the common defense and
general welfare. Such spending may be for any public purpose, not merely for the accomplishment
of other enumerated powers. The statute here was enacted for the public purpose of deterring
4. CONSTITUTIONAL LAW - JUDICIAL REVIEW ANSWERS

gang membership. Consequently, the statute is a valid exercise of the federal taxing and spending
powers. Nevertheless, the court will never reach the merits on a proper jurisdictional challenge, as
discussed above. (D) is incorrect both because the court will never reach the merits and because
the statute would be upheld if the court did reach the merits.

Answer to Question 8

(C) If there is no substantial threat that the statute will be enforced, then there are no constitutional
issues ripe for review. A federal court will resolve only constitutional issues that are necessarily
presented, ripe for review, and unavoidable for decision of the case. Someone seeking a declara-
tion that a statute is unconstitutional must demonstrate that she has engaged (or will engage) in
specific conduct, and that the challenged statute poses a real and immediate danger to her inter-
ests. The court will not determine the constitutionality of a statute when the statute has not been
enforced and there is no immediate threat that it will be enforced. Thus, if the state statute is
not likely to be enforced, it is not a substantial threat to the parents and so they will be unable to
demonstrate any real and immediate harm (or threat thereof) to their interest. Here, the parents’
child is nine months old and the statute cannot be violated before the child turns age three. There-
fore, this case is not ripe, and this choice gives the state a chance to defeat the parents’ argument.
The other choices do not present the state with viable arguments. (A) is incorrect because, if
ripeness were present, an action for declaratory judgment would be proper. A federal court may
not issue advisory opinions, but where there is a real controversy, it may issue a final judgment
declaring the rights and liabilities of the parties even though no affirmative relief is sought.
Maintenance of such an action requires an actual dispute between parties with adverse legal
interests. If the parents were being prosecuted for violating the statute (or threatened with prose-
cution), the requisites for a declaratory judgment would be met and it would be an appropriate
means of determining the constitutionality of the state statute. (B) is incorrect because the refusal
of the state legislature to repeal the statute indicates a legislative intent that the statute remain in
effect, thus implying a greater likelihood that the statute would be enforced. This would make the
parents’ case ripe and so would help them rather than defeat their action. (D) is incorrect because
the grounds for abstention are absent. When a federal constitutional claim is premised on an
unsettled question of state law, the federal court should abstain, to give the state courts a chance
to settle the underlying state law question. Here, there is no unsettled question of state law. There-
fore, abstention by the federal court would be inappropriate.

Answer to Question 9

(C) The court should dismiss the action because the businessperson cannot show that he has an injury
that will be remedied by a decision in his favor. Even if a federal court has jurisdiction over
the subject matter of a case, it will not decide a constitutional challenge to a government action
unless the person challenging the action has “standing” to raise the constitutional issue, i.e., a
concrete stake in the outcome of the controversy. This requires the plaintiff to show an injury
in fact—caused by government conduct—that will be remedied by a decision in his favor. Here,
the businessperson cannot establish that whatever injury he might have suffered by having his
purchase blocked would be remedied by an injunction against the subsequent transaction; hence,
he does not have standing to obtain an injunction against the sale to the other newspaper owner.
(A) is incorrect. While the businessperson could claim that the Fifth Amendment Due Process
Clause forbids discriminatory treatment in the exercise of a First Amendment right to be a broad-
caster, that argument is unlikely to succeed (as discussed below). In any case, the court will not
consider the merits of his argument because he does not have standing to enjoin the purchase
by the other newspaper owner. (B) is incorrect because, even if the businessperson were able to
CONSTITUTIONAL LAW - JUDICIAL REVIEW ANSWERS 5.

establish standing, the FCC may regulate ownership of the broadcast media by forbidding owner-
ship of a radio or television station by a daily newspaper located in the same community, as a
means of promoting the diversity of information received by the public. (D) is incorrect even
though it is a true statement (as discussed above). The court would not even reach the merits
of the businessperson’s case before dismissing it; it would dismiss the action on the basis of the
businessperson’s lack of standing to challenge the sale of the station to the other newspaper owner.

Answer to Question 10

(A) The court should grant the legislature’s motion because the Eleventh Amendment generally
prohibits a federal court from hearing a private party’s claim against its own or another state
government. This jurisdictional bar includes actions against a state government for injunctive
or declaratory relief where the state itself, rather than state officials, is named as a party. Here,
the citizens’ group is seeking an injunction against the state legislature itself rather than a state
official. Thus, the action would be barred by the Eleventh Amendment. (B) is not as good an
answer as (A) because political questions generally involve issues committed by the Constitution
to one of the other branches of the federal government and issues lacking well-developed judicial
standards for resolution and enforcement. Here, the lawsuit is alleging that the state legislature
will violate the Establishment Clause if it allows the leader to take office. This issue does not
involve other branches of the federal government and, at the same time, allows the court to use
extensive Establishment Clause precedent to reach a decision in the case. Hence, the court is
not likely to dismiss the lawsuit on political question grounds. (C) is wrong because standing to
challenge an expenditure as a violation of the Establishment Clause is limited to specific enact-
ments under Congress’s taxing and spending power. Even though state funds might be expended
in this case as a routine part of the legislative privileges granted to the leader, it is not an expen-
diture that falls within the Establishment Clause exception. (D) is wrong because, as discussed
above, an action seeking injunctive relief against the state as a party is also barred by the
Eleventh Amendment.

Answer to Question 11

(A) The strongest argument against the legislation’s constitutionality is that Congress cannot usurp
the courts’ authority to define the scope of the Equal Protection Clause. If the Equal Protection
Clause requires a remedy for past racial discrimination, then Congress could not constitutionally
interfere with the fashioning of a judicial remedy to achieve constitutionally required conduct. (B)
is wrong because the Privileges or Immunities Clause protects individual rights against infringe-
ment by state government; it does not limit the powers of Congress vis-à-vis the federal courts.
(C) is wrong because Congress may limit the jurisdiction of federal courts. (D) is wrong because
Congress may burden interstate commerce, because it has very broad power on that subject.

Answer to Question 12

(C) The court should grant a hearing because a federal question is involved. Federal district courts are
courts of limited jurisdiction and only hear cases in which there is a federal question involved or
those in which there is diversity of citizenship and the amount in controversy exceeds $75,000. In
these facts, the company is alleging that the state statute violates the Commerce Clause by placing
an undue burden on interstate commerce. Thus, a federal question based on the United States
Constitution is involved here. (A) is wrong because the purpose of the Eleventh Amendment is
to protect the sovereignty of a state by preventing lawsuits against that state being brought in a
federal court; the appeal in this case deals only with the constitutionality of a state statute. (B) is
6. CONSTITUTIONAL LAW - JUDICIAL REVIEW ANSWERS

wrong because federal courts may also decide whether state laws violate the United States Consti-
tution. (D) is wrong under these facts; even if there is diversity of citizenship, there is no amount
in controversy alleged in these facts.

Answer to Question 13

(A) The trial court should rule for the government. Only Article III judges enjoy the constitutional
grant of life tenure, and the lawyer is not an Article III judge. The tribunal is an administrative
agency with limited jurisdiction, and while the lawyer exercises some adjudicatory powers, all
persons with some adjudicatory functions are not judges within the meaning of Article III. (A)
is thus correct and (D) is wrong. (C) is also wrong, because the Constitution does not recognize
property rights in employment. The lawyer could show some kind of wrongful discharge only if
he could claim that he was deprived of something “in the nature of a property right.” (B) is incor-
rect because the lawyer clearly has standing; he alleges a specific injury to himself and brings the
claim in his own right, alleging the violation of a constitutional right.

Answer to Question 14

(B) The court will reach the merits of the case because the vendor has standing. A person who
challenges governmental action in federal court on constitutional grounds must have standing
to raise the constitutional issue. A person has standing only if he can demonstrate a concrete
stake in the outcome of a controversy and that the governmental action at issue impairs his own
rights. A plaintiff may also assert third-party rights if he has suffered an injury and that injury
adversely affects his relationships with third parties, resulting in an indirect violation of their
rights. Here, enforcement of the ordinance will effectively destroy 60% of the vendor’s business
and cut down on the dissemination of his religious message. Thus, the vendor is faced with an
immediate and direct threat of injury. This injury will also affect the vendor’s relationship with
those who purchase water from him on the sidewalk, because now he will be unable to sell them
water with religious messages when they need it (physically, or, perhaps, metaphysically). Answer
(A) is incorrect. Although, as discussed above, the vendor has standing, the constitutional issue
involved here is not the Privileges and Immunities Clause of Article IV. That Clause prohibits
discrimination by a state against nonresidents that impacts a fundamental right, such as the
conduct of commercial activities. Here, while the ordinance involves commercial activity, nothing
in the ordinance treats nonresidents differently from residents. Answer (C) is incorrect because
the vendor does not have to be charged with violating the law to challenge its constitutionality.
Nothing suggests that the law will not be enforced against him if he tries to continue selling his
water; hence, he is faced with an immediate and direct threat of injury. Answer (D) is incorrect
because it goes to the merits and is a fact question that must be resolved at trial. The vendor will
argue that the water ordinance impinges on his First Amendment freedoms of speech and religion.
The city may defend such a claim by arguing that the water ordinance constitutes a reasonable
time, place, and manner restriction on First Amendment activities. However, such an argument
would not be a reason for declining jurisdiction; rather, it is a fact issue that must be determined at
trial.
CONSTITUTIONAL LAW -
SEPARATION OF POWERS
QUESTIONS
CONSTITUTIONAL LAW - SEPARATION OF POWERS QUESTIONS 1.

CONSTITUTIONAL LAW - SEPARATION OF POWERS QUESTIONS

Question 1 Question 2

Congress enacted a statute, over the Presi- To boost the slumping cruise ship trade,
dent’s veto, that granted Congress the power to Congress passed a bill providing that six
compel the President to remove United States outmoded United States Navy vessels would be
troops from foreign territory when such troops sold to a private cruise company for $1 each, on
have for 60 days been engaged in hostilities and condition that the company refurbish the ships at
there has been no formal declaration of war. The its own expense and operate the ships as cruise
statute also provided that Congress may force ships for at least four years. The conditions for
the President to withdraw the troops before the the refurbishing were highly specific, and it
60 days have elapsed by a two-thirds majority would cost the company at least $3 million to
vote of the Senate Foreign Relations Committee refit each ship as a modern cruise liner. The
to that effect. company agreed to the conditions, but a compet-
itor cruise company filed suit in federal court to
Which of the following statements best block the sale.
describes the likely result of judicial review of
the constitutional validity of this statute? Which of the following statements is most
correct concerning disposition of the case?
(A) The statute is a valid exercise of Congress’s
authority under the war power. (A) The federal court should treat the sale as
presumptively valid, because the Constitu-
(B) The statute is a valid exercise of Congress’s tion expressly gives Congress the power to
foreign relations powers. dispose of property of the United States.

(C) The statute is constitutionally suspect as an (B) The federal court should treat the sale as
infringement on the President’s exclusive presumptively invalid, because the Consti-
power, as commander in chief, over matters tution expressly denies Congress the right
relating to war. to deprive persons of property without due
process of law.
(D) The statute is constitutionally suspect,
because the committee decision is not (C) The federal court should rule the statute
subject to a presidential veto. constitutional only if the President or the
Secretary of Defense has certified that
the Navy ships are obsolete for defense
purposes.

(D) The federal court should rule the statute


unconstitutional, because it denies other
cruise ship operators the equal protection of
the laws.
2. CONSTITUTIONAL LAW - SEPARATION OF POWERS QUESTIONS

Question 3 Question 4

Which of the following clauses of the Consti- The United States entered into a treaty with
tution would most likely permit Congress to a foreign nation. Under the treaty, the foreign
impose on the states a uniform child custody nation agreed to give up its quest for a nuclear
law? arsenal and the United States agreed to allow
the foreign nation to export automobiles to
(A) The Commerce Clause. the United States. After the treaty was signed,
evidence was discovered indicating that the
(B) The Police Power Clause. foreign nation might not be living up to its end
of the treaty. Before an investigation could be
(C) The Privileges and Immunities Clause of completed, an enraged Congress enacted a
Article IV. statute that specifically terminated permission
for the foreign nation to export automobiles to
(D) The Taxing and Spending Power Clause. the United States.

Is the federal legislation effective?

(A) No, because the agreement with the foreign


nation was a treaty rather than an executive
agreement.

(B) No, because it interferes with the executive


power over foreign affairs.

(C) Yes, because it works to repeal the


substance of the treaty.

(D) Yes, but only if signed by the President


(rather than being passed over his veto) and
ratified by two-thirds of the Senate.
CONSTITUTIONAL LAW - SEPARATION OF POWERS QUESTIONS 3.

Question 5 Question 6

Congress passed a statute providing that if Congress passed a law mandating standards
the President deployed more than 1,000 United for hybrid automobiles in an effort to encourage
States Armed Forces personnel in a foreign more citizens to purchase hybrid cars so as to
country where “combat conditions” existed, the improve environmental conditions. The same
President was required to return such Armed law established a commission, consisting solely
Forces to the United States within 60 days of members of Congress, whose purpose was
or formally seek a declaration of war from to obtain information from manufacturers of
Congress. The statute further provided that, if hybrid automobiles regarding the number, type,
Congress failed to declare war within 30 days and costs to the consumer of hybrid automobiles
of the President’s request, the President was used in the United States and report its findings
required to return the Armed Forces to the on an annual basis to Congress. A manufacturer
United States. of hybrid cars objects to having to supply the
government with any information about its cars
What is Congress’s best argument for the and so resists a subpoena for its chief executive
constitutionality of the statute? officer to testify before the commission.

(A) In making decisions concerning foreign What is the general rule insofar as Congress’s
affairs, the President must first obtain the power to investigate is concerned?
advice and consent of the Senate.
(A) The power of Congress is limited only by
(B) The statute does not limit the President’s the concept of separation of powers.
power to repel invasions.
(B) Congress’s power to investigate is coexten-
(C) The President cannot institute military sive with its power to legislate.
actions in foreign countries without a decla-
ration of war, and only Congress has the (C) Congress’s power to investigate is limited
right to declare war. to the appointment of its own members to
make such investigations.
(D) The statute reflects the balance between the
legislature and the executive branch in their (D) There are no limitations upon Congres-
shared control over the Armed Forces. sional powers of investigation as to the
domestic economy.
4. CONSTITUTIONAL LAW - SEPARATION OF POWERS QUESTIONS

Question 7 Question 8

Congress passed a statute that established a A Congressional committee was formed


five-member commission to investigate issues to conduct an investigation. The committee
relating to gun control and to make recom- subpoenaed a man to appear before it to answer
mendations to Congress for new gun control certain questions. The man appeared before the
laws. All five members of the commission committee but refused to answer any questions.
were appointed by Congress. An entity that has The committee notified the Speaker of the
organizational standing brings a suit to enjoin House of the man’s refusal to cooperate. The
the commission from investigating and recom- Speaker called a special session of the House. At
mending new gun control laws to Congress. the special session, a majority of the members of
the House voted to order the Attorney General of
What should the court do? the United States to prosecute the man pursuant
to a federal statute that establishes the penalties
(A) Forbid the commission to take any action. for contempt of Congress.
(B) Order that all members of the commission Is the Attorney General constitutionally
be appointed by the President by and with obligated to prosecute the man pursuant to the
the advice and consent of the Senate. congressional order?
(C) Allow the commission to investigate but (A) No, if the man is an appointive official of
not make recommendations regarding gun the executive branch, because he would
control. then be immune from prosecution for acts
performed in the course of his duties.
(D) Allow the commission to continue inves-
tigating and making recommendations to (B) No, because the decision to prosecute is
Congress regarding gun control. exclusively within the discretion of the
executive branch.

(C) Yes, because the Attorney General may not


lawfully disobey a directive from Congress
to punish a contempt.

(D) Yes, because the Attorney General may not


refuse to prosecute one who has violated
federal law.
CONSTITUTIONAL LAW - SEPARATION OF POWERS QUESTIONS 5.

Question 9 Question 10

To help stimulate the economy, Congress The United States entered into a treaty with
passed a law proscribing loans with interest rates a neighboring country whereby each country
higher than 10% per annum. A bank that had agreed to ban hunting of the red-tailed raccoon,
been lending money to high risk recipients had a species of raccoon indigenous to both. The
been charging 15% interest on its loans. raccoon had been placed on the endangered
species list of various conservation groups.
If the bank challenges the federal statute, The raccoon freely crossed the international
claiming that Congress had no power to pass boundary between the countries, and state lines
such a statute, it will most likely: within the United States. Laws in three states
in the United States permitted hunting of the
(A) Lose, because the General Welfare Clause raccoon.
coupled with the Necessary and Proper
Clause gives Congress the power to pass After the treaty was fully ratified by both
legislation to limit interest rates. countries, a United States federal court would
most likely hold that the state laws permitting
(B) Lose, because the Commerce Clause gives hunting of the raccoons are:
Congress power to regulate interstate
commerce, and the interest rates banks may (A) Unconstitutional, because a treaty is the
charge affect interstate commerce. supreme law of the land.

(C) Win, because regulating interest rates of (B) Unconstitutional, because free-roaming
banks is not among the powers granted to wildlife is federal property.
Congress.
(C) Constitutional, because wild animals are
(D) Win, because the regulation interferes with natural inhabitants of the state, and the
the bank’s rights under the Privileges and federal government may not take state
Immunities Clause of Article IV. property without consent of the state.

(D) Constitutional, under the rights reserved to


the states by the Tenth Amendment.
6. CONSTITUTIONAL LAW - SEPARATION OF POWERS QUESTIONS

Question 11 Question 12

Federal legislation required that state and A federal statute provided for federal grants
local police departments receiving financial to cities that desired to rebuild inner-city areas
assistance from the Federal Law Enforcement for residential housing. A city applied for
Assistance Agency devote a specified amount funding to build housing and received a grant
of those resources to combatting “white-collar” of $2.5 million. After the area was prepared for
crimes. construction, however, the city council decided
that it would greatly benefit the inner-city
Which of the following would provide the best dwellers if, in addition to housing, commercial
constitutional underpinning for this legislation? property were built. Thus, the council decided
to use $1.5 million for housing and to “borrow”
(A) The power to enforce the penal statutes of the remaining $1 million from the housing fund
the United States. to build a commercial mall. The city resolution
provided that 30% of the rental from the mall
(B) The police power. each year would go to a fund for maintenance
of the housing and for funds to build additional
(C) The war and defense power. housing. Construction had started on residen-
tial buildings and the commercial mall when
(D) The power to tax and spend for the general the federal court, at the request of the federal
welfare. government, froze the construction accounts
containing the proceeds from the grant.

In a motion by the city to release the funds,


the court would most likely:

(A) Grant the motion, because the city’s plan


for a fund to build more residential housing
substantially complies with the terms of the
grant.

(B) Grant the motion, because the doctrine of


preservation of state sovereignty prevents
the federal government from interfering
with the state’s discretion in this situation.

(C) Deny the motion, because the federal


government can control the expenditure of
the funds because it provided the funds.

(D) Deny the motion, because the doctrine of


state sovereignty has no application in this
situation; the action was by the city council
and not the state legislature.
CONSTITUTIONAL LAW - SEPARATION OF POWERS QUESTIONS 7.

Question 13 Question 14

As part of the Domestic Security Act, In response to lobbying, Congress passed


Congress established a permanent commission legislation appropriating $200 million for grants-
to evaluate the security of federal government in-aid to domestic horse liniment manufac-
buildings located in the District of Columbia. turers and providing some degree of protection
The commission would have the authority to from foreign competition. However, because
evaluate current security conditions, estab- of concern about inefficiencies in the industry,
lish new security guidelines, and coordinate the legislation was amended to allow the Secre-
security procedures among various govern- tary of Commerce the authority to deny grants
mental agencies. The legislation provided that to horse liniment manufacturers who failed to
three members of the commission were to be meet certain “management efficiency standards”
appointed by the President, two members by outlined in the legislation.
a committee of the House of Representatives,
and two members by the United States Supreme One liniment manufacturer and a member
Court. The President had the authority to veto of the trade association petitioned the Secre-
any of the selections to the commission made tary of Commerce for a $15 million grant. This
by the congressional committee or the Supreme figure equaled the amount it would be entitled to
Court. under the legislation, based on the number of its
employees, plants, and upon its average produc-
If a party with the requisite standing tion of horse liniment over a 10-year period.
challenges the legislation and it is found uncon- The Secretary of Commerce refused to award
stitutional, it will most likely be because: the funds, because she determined that the
liniment manufacturer was making no attempt to
(A) The President’s power to veto a particular improve its management efficiency. The liniment
selection made by the congressional com- manufacturer filed suit against the Secretary of
mittee constitutes an improper exercise of Commerce, asserting that the power granted to
the veto power. the Secretary was unconstitutional.

(B) The commission will be exercising admin- Is the legislation constitutional?


istrative powers.
(A) Yes, because the Secretary of Commerce,
(C) The judiciary does not have the power to as a representative of the executive branch,
make appointments to an advisory commis- may be granted regulatory authority.
sion.
(B) Yes, because the executive branch, repre-
(D) The legislation does not provide for Senate sented by the Secretary of Commerce,
confirmation of the presidential appointees. shares power with Congress in the field of
foreign commerce.

(C) No, because there was an improper delega-


tion of legislative power.

(D) No, because the executive branch may not


impound funds appropriated by Congress.
CONSTITUTIONAL LAW -
SEPARATION OF POWERS
ANSWERS
CONSTITUTIONAL LAW - SEPARATION OF POWERS ANSWERS 1.

CONSTITUTIONAL LAW - SEPARATION OF POWERS ANSWERS

Answer to Question 1

(D) The statute is constitutionally suspect. Action having the purpose and effect of altering the legal
rights, duties, and relations of persons, including executive branch officials, must be subjected
to the possibility of presidential veto. [Immigration & Naturalization Service v. Chadha (1983)]
Although the President (or his predecessor) had the opportunity to veto the statute, the passage of
a Senate committee decision that shortens the time that the President may use the troops would
have the purpose and effect of altering the rights and duties of the President, which accrue to him
by virtue of his rather extensive military powers, and would not be subject to a presidential veto.
For this reason the statutory provision may be an unconstitutional legislative veto of executive
action. It follows that (A) and (B) are therefore incorrect. (C) is incorrect because the President
does not have exclusive power over matters relating to war. Such power is shared with Congress.

Answer to Question 2

(A) The court should hold the sale presumptively valid pursuant to Congress’s property power. Article
IV, Section 3 of the Constitution gives Congress the power to dispose of all property belonging to
the federal government. There are no express limits placed on this power, and a disposal has never
been invalidated on the ground that it places a competitor of the purchaser at a disadvantage. (B)
is incorrect because the Due Process Clause would not prevent the sale here. The Due Process
Clause prohibits the government from denying persons life, liberty, or property without due
process of law. For the restrictions of the Clause to apply, a person must have a legitimate property
interest in the property taken. “Property” includes more than personal belongings, but a mere
expectation or desire for the benefit is not enough. One must have a legitimate claim or entitlement
to the benefit before one may make a procedural due process challenge, and here the competitor
has no claim to a right to be offered the ships that were sold to the first cruise company. (C) is
incorrect because, as indicated above, there is no express limit on Congress’s power to dispose
of government property; nothing in the Constitution requires Congress to get the President’s (or
any other executive officer’s) permission to exercise the power. (D) is incorrect because there is
no denial of equal protection here. While the Equal Protection Clause is not applicable to the
federal government, equal protection guarantees are applicable through the Fifth Amendment
Due Process Clause. Nevertheless, there is no equal protection violation here: Because no suspect
class or fundamental right is involved, the action would be tested under the rational basis standard
(discriminatory government action is valid as long as it is rationally related to any legitimate
government interest) and would be upheld since the sale is rationally related to the government’s
interest in reviving the cruise ship industry. It does not matter that the government is not doing all
that it can; a first step is permissible.

Answer to Question 3

(D) The most likely method the United States could use to impose a uniform child custody law on all
the states is through the taxing and spending power, making an allocation of funds available to
each state that adopts the uniform law. No other selection would pass constitutional muster. The
Commerce Clause has not been applied to child custody matters, and it is unlikely that Congress
could use its commerce powers to regulate in this area. Under the Commerce Clause, Congress
can regulate channels of interstate commerce, instrumentalities of interstate commerce, and
economic or commercial actvities that in aggregate have a substantial economic effect on inter-
state commerce. It is doubtful that regulations of noneconomic activities, such as child custody
2. CONSTITUTIONAL LAW - SEPARATION OF POWERS ANSWERS

laws, can be grounded in the Commerce Clause. Thus, (A) is not the most likely basis for such a
statute. (B) is wrong; there is no general federal police power. (C) is wrong because the Privileges
and Immunities Clause of Article IV does not apply, since this is an action of the federal govern-
ment.

Answer to Question 4

(C) The federal statute is effective because it would work to repeal the treaty. Although a treaty
has supremacy over conflicting state law, a treaty is only on a supremacy parity with an act of
Congress. Any conflict between an act of Congress and a treaty is resolved by order of adoption;
i.e., the last in time prevails. Here, the congressional legislation conflicts with the treaty by specifi-
cally terminating the permission to freely export automobiles from the foreign nation that was
conferred by the treaty. Because this statute was enacted after the treaty, the terms of the statute
prevail, thus repealing the substance of the treaty. (A) is incorrect because the legislation has the
same effect in this case regardless of whether the agreement was a treaty or an executive agree-
ment; as discussed above, the legislation will prevail over the treaty. (B) is incorrect because
there is no unconstitutional interference with the executive’s powers here. Both the President and
Congress have some authority over foreign relations (e.g., the Commerce Clause allows Congress
to regulate foreign commerce), and Congress has a right to exercise its power here. As stated
above, the last expression of the sovereign (here, the statute) controls. (D) states the requirements
for adoption of a treaty (i.e., the treaty power is granted to the President by and with the advice
and consent of the Senate, providing two-thirds of the Senators present concur). Legislation that
has the effect of repealing a treaty need not be signed by the President (i.e., it may be passed over
the President’s veto), nor must such legislation be ratified by two-thirds of the Senate (unless an
attempt is being made to override a presidential veto, in which case a two-thirds vote of each
house is required). Therefore, (D) is incorrect.

Answer to Question 5

(D) The best reason to uphold the statute is that it reflects the balance of power between the President
and Congress over the Armed Forces. The Constitution makes the President the commander in
chief of the armed forces, but it gives Congress the power to declare war and raise an army and
navy. The Supreme Court has never delineated the extent of either branch’s power under these
clauses, but clearly the power over war is shared. Since none of the other answers is correct, (D)
is the only viable alternative. (A) is incorrect because it is too broad. The President need not
obtain the advice and consent of the Senate concerning all foreign affairs, but only with regard
to treaties. Moreover, nothing in the Constitution requires the President to obtain the advice and
consent of the Senate regarding how to wage a war. (B) is incorrect because it is too narrow.
The President’s power as commander in chief has historically extended beyond the power to
repel invasions. At the very least, it has also included the power to protect citizens abroad. (C) is
incorrect because the President has the power, as commander in chief, to send troops to another
country even in the absence of a declaration of war (e.g., to protect United States citizens).

Answer to Question 6

(B) The power of Congress to investigate is coextensive with the power to legislate created by Article
I, Section 8 of the Constitution. (A) and (D) are accordingly incorrect statements of the limitations
on this power. (A) is wrong because it postulates that “only” the separation of powers doctrine
limits the investigative power. (D) is wrong because it is too sweeping. Congressional investiga-
tion is limited to matters over which the Congress has jurisdiction. (C) is simply wrong. Congress

一 信:liuxue119118 , 们 信免 供
CONSTITUTIONAL LAW - SEPARATION OF POWERS ANSWERS 3.

may use appropriate means to investigate, and Congressional investigative committees may be
comprised of members appointed by the executive branch, provided that it is done in such a way
as to not violate separation of powers principles.

Answer to Question 7

(D) The court should allow the commission to continue investigating. Congress may establish legisla-
tive and investigative commissions. In these facts, the commission is not exercising any enforce-
ment powers but rather is merely investigating and making recommendations, and all of its
members are appointed by Congress; therefore, for the limited purposes indicated, the Constitu-
tion is not violated. Thus, (B) is incorrect because this commission does not have any enforcement
powers. Therefore, the President need not appoint any members of the commission. Furthermore,
the advice and consent of the Senate is not required here. (C) is incorrect because Congressional
commissions can both investigate and make recommendations to Congress. (A) is incorrect
because forbidding the commission to take any action is too strong a remedy and is not constitu-
tionally required.

Answer to Question 8

(B) The executive branch has sole discretion to prosecute. The Attorney General is not obligated
to prosecute the man, regardless of whether the man is an appointive official of the executive
branch. The doctrine of separation of powers prohibits legislative interference with the executive
discretion as to whether to prosecute. Here, the legislative branch is mandating that the Attorney
General—a member of the executive branch—prosecute the man. Thus, (C) and (D) are incor-
rect. (A) is incorrect because officials of the executive branch are not immune from prosecution
relating to their duties.

Answer to Question 9

(B) Congress may regulate, under the Commerce Clause, anything that has a substantial economic
effect upon interstate commerce. The rates that banks charge for interest certainly affect interstate
commerce or may simply constitute commerce in its broad sense. In either case, the regulation
is clearly within Congress’s commerce power. Thus, (C) is incorrect. (A) is incorrect because
the General Welfare Clause gives Congress the power to spend for the general welfare, and the
regulation here is not related to any spending measure. (D) is incorrect because the Privileges and
Immunities Clause of Article IV prohibits states from discriminating against nonresidents. Here,
we have a federal statute. So the Privileges and Immunities Clause is inapplicable.

Answer to Question 10

(A) The state laws will likely be found unconstitutional because a treaty is the supreme law of the
land, and state statutes that conflict with ratified treaties are invalid. (B) and (C) state incor-
rect rules of law. The federal government may exercise its property powers to acquire control of
free-roaming animals on public land. The federal government does not, however, have inherent
authority over or ownership of all “free-roaming wildlife.” A state may, in turn, assert some rights
over animals within its borders. That power must, nevertheless, give way in the face of a valid
exercise of federal power. (D) is wrong because a treaty, being the supreme law of the land, takes
precedence over the rights reserved to the state under the Tenth Amendment.
4. CONSTITUTIONAL LAW - SEPARATION OF POWERS ANSWERS

Answer to Question 11

(D) The strongest support for the legislation would be the power to tax and spend for the general
welfare. Prior to disbursement, the federal government may attach any reasonable condition to
expenditures under the general welfare power. (A) is not a power of Congress enumerated in
the Constitution. There is no general federal police power, the federal government being one of
limited, enumerated powers. Thus, (B) is incorrect. (C) is incorrect because the war and defense
power does not apply to the facts presented.

Answer to Question 12

(C) The court will likely deny the motion because Congress generally has the power to condition
federal spending if Congress reasonably finds that the spending program is for the general welfare.
(A) is incorrect because the federal government determines how the funds are to be expended, not
the recipient. Thus, while it may be arguable that the city’s plan is a better way to use the money,
the fact remains that the money was given for one purpose only and it must be expended for that
purpose. Hence, (B) is wrong because the federal government in this situation does have the
authority to control the spending of its funds and it is not an interference with the state’s sovereign
powers. (D) is wrong because the doctrine of sovereignty does not just extend to the state’s execu-
tive or legislative branch, but to all governmental activities and entities within the state.

Answer to Question 13

(B) If the legislation is found to be unconstitutional, it will be because the commission will be
exercising administrative powers. Under Article II, Section 2, Congress may not appoint members
of a body with administrative or enforcement powers. Such persons are “officers of the United
States” and must be appointed by the President with senatorial confirmation unless Congress has
vested their appointment in the President alone, in federal courts, or in heads of departments.
Here, selection of two members of the commission by a committee of the House of Represen-
tatives would violate the Appointments Clause because the commission has investigative and
administrative powers. (A) is incorrect because the problem with the legislation is not that the
President is infringing on the legislative branch’s power by being able to veto congressional
appointments, but rather that the congressional appointments are infringing on the executive
branch appointment power. (C) is incorrect because Congress may by law vest appointment power
of inferior officers in the federal courts. Hence, the judiciary has been properly granted the power
by Congress to appoint members of the commission. (D) is incorrect because only Cabinet-level
officers require Senate confirmation for their appointment. Inferior officers, which would include
members of this commission, may be appointed by the President without Senate approval if
provided for by Congress, as is the case here.

Answer to Question 14

(A) Congress may delegate many of its powers to executive agencies, provided adequate standards
are established to govern exercise of the delegated power. The power was properly delegated to
the Secretary of Commerce because the statute “outlines” the “management efficiency standards”
to be followed. (B) is incorrect because Congress has sole authority over foreign commerce;
the authority is not shared with the executive branch. (C) is incorrect because the delegation is
proper if adequate standards are established; as determined above, there are adequate standards
here. (D) is incorrect because there is no executive impoundment here, merely a refusal to grant
funds based upon standards established by Congress. Such standards are proper where, as here,
Congress acts pursuant to its spending power and the standards are specified in advance.
一 信:liuxue119118 , 们 信免 供

CONTRACTS -
CONDITIONS,
DISCHARGE, AND
BREACH QUESTIONS
CONTRACTS - CONDITIONS, DISCHARGE, AND BREACH QUESTIONS 1.

CONTRACTS - CONDITIONS, DISCHARGE, AND BREACH QUESTIONS

Question 1 (D) The builder will win because he attempted


to perform the contract as originally agreed.
On January 1, a builder and a landowner
agreed in writing that the builder would build Question 2
a house on the landowner’s lot according to
the landowner’s plans and specifications for The owner of a neighborhood tavern ordered
$160,000, the work to commence on April 20 kegs of domestic beer from a brewery. The
1. The landowner agreed to make an initial written contract between the parties provided
payment of $20,000 on April 1, and to pay the that the brewery would deliver the beer by
balance on completion of the work. January 31, and the tavern owner would pay
the brewery $1,400 upon delivery. The tavern
On February 1, the builder notified the owner was expecting a big crowd for Super Bowl
landowner that he (the builder) would lose Sunday, four days after the scheduled delivery
money on the job at that price, and would not date. The brewery did not deliver the beer to the
proceed with the work unless the landowner tavern owner until February 16, but the tavern
would agree to increase the price to $190,000. owner still accepted it.
The landowner thereupon, without notifying
the builder, agreed in writing with a competing Did a duty to pay the $1,400 arise upon
building company for the building company, the tavern owner’s acceptance of the beer on
commencing April 1, to build the house for February 16?
$175,000, which was the fair market cost of the
work to be done. (A) No, and the tavern owner retained the right
to sue for any damages incurred because of
On April 1, both the builder and the building the delay in delivery.
company showed up at the building site to begin
work, the builder telling the landowner that he (B) No, but the tavern owner waived the right
had reconsidered and would build the house for to sue for any damages incurred because of
$160,000 as originally agreed. The landowner the delay in delivery.
dismissed the builder and allowed the building
company to begin work on the house. (C) Yes, but the tavern owner retained the right
to sue for any damages incurred because of
In a contract action by the builder against the delay in delivery.
the landowner, which of the following would
the court decide under the prevailing American (D) Yes, and the tavern owner waived the right
view? to sue for any damages incurred because of
the delay in delivery.
(A) The landowner will win because the builder
in legal effect committed a total breach of
contract.

(B) The landowner will win because the


building company’s contract price was
$15,000 lower than the $190,000 demanded
by the builder on February 1.

(C) The builder will win because the landowner


did not tell him before April 1 about the
contract with the competing building
company.
2. CONTRACTS - CONDITIONS, DISCHARGE, AND BREACH QUESTIONS

Question 3 Question 4

A contractor agreed to build a plant for a An actor was hired by a director to perform
manufacturer for $5 million, with $1 million in a three-month run of a play that the director
paid in advance and the balance to be paid upon had written. Two weeks after the play’s opening
completion of the project. The contract required night, the actor fainted and could no longer
the contractor to use lighting fixtures from a perform because of a resulting concussion.
specific company. Inadvertently, the contractor After a week of treatment, the actor returned to
installed fixtures from a different company. the theatre to resume his role but the director
The installed fixtures are generally consid- refused him admission, having hired someone
ered to be of a slightly better quality than the else to replace the actor.
fixtures specified in the contract. The mistake
was not discovered until the manufacturer did In his action by the actor against the director
a final inspection of the building. As built, the for breach of contract, which of the following, if
plant is worth $10,000 more than it would have proved, would be the director's best defense?
been worth had the specified fixtures been
used. It would cost the contractor $100,000 (A) Most of the theatre members felt that the
to replace the fixtures with the ones specified replacement was a better performer than
in the contract. Because of a downturn in the the actor.
economy, the manufacturer no longer wants to
move into the new plant and refuses to pay the (B) The director had offered the actor a job in
contractor because of the breach regarding the the ticket office at a higher salary, but the
light fixtures. actor had declined.

If the contractor sues the manufacturer for (C) The director could not find any substi-
breach of contract, which of the following tute except for the replacement actor, who
doctrines will be most important to a court’s demanded a contract for a minimum of two
decision? and a half months if he was to perform at
all.
(A) Perfect tender.
(D) The actor did not really wish to continue to
(B) Divisibility. be employed by the director, but is merely
seeking money.
(C) Substantial performance.

(D) Quasi-contract.
CONTRACTS - CONDITIONS, DISCHARGE, AND BREACH QUESTIONS 3.

Question 5 Question 6

A man borrowed $5,000 from his colleague A candlemaker who owned a bee farm sold
to purchase stock and agreed in writing to repay the farm to a beekeeper. Because the candle-
the loan on or before August 1. maker was still interested in making beeswax
candles, the contract provided that the candle-
On August 1, the man notified his colleague maker reserved the right to purchase all of the
that he would be unable to pay back the $5,000. beeswax produced by the beekeeper during the
He told her that he could send her a check for next five years at the current market price at time
$2,500 and that, in addition, he could give her of delivery, and the beekeeper agreed to supply
an antique diamond ring that had been recently “in any event a minimum of 100 pounds of
appraised at $2,200. The colleague liked the ring beeswax per month during that period.” For one
and agreed to accept it plus $2,500 in cash as year, the beekeeper delivered to the candlemaker
payment for the loan. and the candlemaker paid for all of the beeswax
produced by the beekeeper. As the first year of
On August 2, a courier delivered the ring and the contract ended, the beekeeper was stung by
a certified check for $2,500 to the colleague. She a bee and, due to an allergy, became so seriously
took the check but told the courier to return the and permanently ill and impaired as to be
ring to the man. The man received the ring back unable to attend to the bees. He gave the candle-
the same afternoon. Meanwhile, the colleague maker prompt notice that he would be unable to
deposited the check in her bank, and the next continue, and from that time on he never made
day filed suit against the man for $2,500. The another delivery of beeswax to the candlemaker.
man consulted an attorney as to whether he has a
valid defense against his colleague’s suit. Assuming that contractual obligations existed
between the candlemaker and the beekeeper,
Assuming there are no Statute of Frauds was the beekeeper’s refusal to perform justifi-
issues, what advice should the attorney give the able?
man?
(A) Yes, because the parties had not agreed to a
(A) The man has no defense against his col- specific amount of beeswax in the contract.
league’s suit, because the amount of the
debt was undisputed. (B) Yes, because his performance was excused
because of his permanent disability.
(B) The man has no defense against his
colleague’s suit, because she properly (C) Yes, because he gave the candlemaker
exercised her right to enforce the original reasonable notice so that the candlemaker
agreement by refusing tender of the ring. could buy beeswax elsewhere.

(C) The man has no defense at law, but he (D) No, it constituted a breach of contract.
may successfully defend in equity under a
specific performance theory because the
ring is unique.

(D) The man has the option of defending in


equity under a specific performance theory
or waiting until his colleague obtains a
judgment against him and then suing her
for breach.
4. CONTRACTS - CONDITIONS, DISCHARGE, AND BREACH QUESTIONS

Question 7 Question 8

A brother and a sister made a written contract The manager of a monthly antique market
pursuant to which the brother promised to was looking to hire a professional appraiser who
convey a specified apartment house to his sister would tell patrons, for a fee of $10 per item, what
in return for his sister’s promise (i) to convey their antique is worth. Because the manager had
a 100-acre farm to the brother and (ii) to pay had problems at other antique markets he had
the brother $1,000 in cash six months after the run when appraisers built up a popular following
exchange of the apartment house and the farm. and then abruptly quit for a better job, he empha-
sized during the hiring interview the impor-
Which of the following statements concerning tance of honoring the contract to its completion.
the order of performances is LEAST accurate? When the manager offered the job to an experi-
enced appraiser and the appraiser accepted,
(A) The brother’s tendering of good title to the the written contract signed by the appraiser
apartment house is a condition precedent to contained, in addition to an agreed-to salary, a
the sister’s duty to convey good title to the liquidated damages clause for early termination
farm. of the contract. It also contained another clause
providing that the appraiser would receive 5% of
(B) The sister’s tendering of good title to the all gate receipts to be paid as a bonus at the end
farm is a condition precedent to the broth- of the contract, which ran for one year.
er’s duty to convey good title to the apart-
ment house. Eight months into the contract, the manager’s
worst fears were realized when the appraiser got
(C) The sister’s tendering of good title to a more lucrative offer and abruptly quit, leaving
the farm is a condition subsequent to the the manager scrambling to find a replacement for
brother’s duty to convey good title to the him. In response to the manager’s suit for breach
apartment house. of contract, the appraiser brings a countersuit
to recover 5% of gate receipts for the antique
(D) The brother’s tendering of good title to the markets at which he worked.
apartment house and the sister’s tendering
of good title to the farm are concurrent Is the appraiser likely to be successful in his
conditions. countersuit?

(A) Yes, because his breach was not substan-


tial.

(B) Yes, because the manager’s only remedy


was the liquidated damages clause.

(C) No, because working the entire year was an


implied condition of the contract.

(D) No, because working the entire year was an


express condition of the contract.
CONTRACTS - CONDITIONS, DISCHARGE, AND BREACH QUESTIONS 5.

Question 9 grower’s failure to deliver the balance of the


amount specified by the contract.
In July of last summer a grape grower
contracted with a winery to deliver “500 tons (D) Because the grape grower’s contract with
of premium quality pinot chardonnay grapes the winery was entered into before his
grown on my ranch.” The price was to be $1,000 contract with the vineyard, the grape
per ton and delivery was to be on or before grower is bound to deliver the entirety of
September 15. In August of the same year, the his grape crop to the winery.
grape grower entered into an identical contract
with a vineyard to sell 300 tons of premium Question 10
quality pinot chardonnay grapes. An amusement park entered into a contract
with an engineering firm to construct a new
The grape grower completed his harvest by
roller coaster for the park. The contract provided
September 10 and had 800 tons of premium
that the amusement park would make monthly
quality grapes. On September 11, an unexpected
payments to the engineering firm and that the
rain ruined 400 tons, and the grape grower
engineering firm would submit a monthly report
notified the winery and the vineyard on that
from an independent safety inspector that all
day that he would only be able to deliver 250
state-mandated safety guidelines were being met
tons to the winery and 150 tons to the vineyard.
during construction. Construction began and the
On September 14, the vineyard purchased an
amusement park made three monthly payments,
additional 150 tons of premium quality pinot
but the engineering firm failed to provide any
chardonnay grapes from a different grape
monthly safety reports. The amusement park
farmer, one of several other available sources for
did not object. Construction was then halted for
premium quality pinot chardonnay grapes. These
two months during hurricane season, and the
grapes along with the 150 tons from the grape
amusement park made no payments during that
grower gave the vineyard the 300 tons it needed.
period. The engineering firm did not object.
On September 15, what is the winery’s legal On the day construction was set to resume, the
position with regard to the grape grower’s failure engineering firm demanded a monthly payment.
to deliver the 500 tons of grapes required by his The amusement park refused on the grounds
contract? that no progress on the project had been made
in two months and no safety reports had been
(A) If the winery has given the grape grower a submitted. The engineering firm thereupon
written notice of termination, the winery abandoned work and repudiated the agreement.
will have the right to refuse to accept the
250 tons of grapes but will have no cause What was the probable legal effect of (i) the
of action for damages against the grape engineering firm’s failure to object to the amuse-
grower. ment park’s failure to make payments during
the hurricane season and (ii) the amusement
(B) Even if the winery has given the grape park’s making three monthly payments without
grower a written notice of termination, requiring submission of a safety report?
the winery must accept the 250 tons of
grapes and will have no cause of action for (A) Estoppel-type waiver as to both.
damages against the grape grower. (B) Waiver of delay in payment and revocable
waiver as to the safety reports.
(C) Because the vineyard’s purchase establishes
that it is possible for the grape grower to (C) Mutual rescission of the contract.
perform by obtaining additional grapes
from other available sources, the winery (D) Discharge of the amusement park’s duty
may accept the 250 tons from the grape to make the payments and estoppel-type
grower and recover damages for the grape waiver as to the safety reports.
6. CONTRACTS - CONDITIONS, DISCHARGE, AND BREACH QUESTIONS

Question 11 a builder to do the construction work in accor-


dance with the bookstore’s requirements. When
Buyers of a house with an old above- the premises were not ready by August 10, the
ground pool wanted it removed, so the current bookstore notified the manager that it would rent
homeowners agreed that they would arrange store space elsewhere. Five days later, on August
to have the pool moved to their new home, but 15, the manager sent the bookstore a letter by
since the sale was occurring in winter, the pool certified mail stating that the construction had
would have to be moved at a later date. The been delayed because the builder he had hired
parties therefore agreed that the buyers would had breached his contract by abandoning the job,
pay all but $5,000 of the home’s sale price at and he had to employ another builder. However,
closing, and then pay the final $5,000 six months the manager promised that the premises would
after the sale. The contract further stated: be completed no later than August 25. The
manager further stated that, under the circum-
It is understood and agreed that the stances, he believed he was entitled to this
purchasers’ obligation to pay the $5,000 six short period of time to complete the agreement
months after the sale shall be voided if the because the renovations, done especially for the
current homeowners have not, within three bookstore, had already cost him $15,000. On
months after the aforesaid sale, removed the August 25, the manager tendered performance
existing pool in the rear of the house. to the bookstore, but the bookstore refused to
accept the lease. The manager sought another
What is the homeowners’ removal of the pool tenant for the space, without success.
from the backyard of the house?
The shopping center brought suit against the
(A) A condition subsequent in form but prec- bookstore for breach of contract.
edent in substance to the buyers’ duty to
pay the $5,000. Which of the following facts will the court
consider in determining which party will
(B) A condition precedent in form but subse- prevail?
quent in substance to the buyers’ duty to
pay the $5,000. (A) A substitute tenant who was willing to rent
the renovated premises could not be found.
(C) A condition subsequent to the buyers’ duty
to pay the $5,000. (B) The shopping center could not have
foreseen that the builder would breach his
(D) Not a condition, either precedent or subse- contract.
quent, to the buyers’ duty to pay the $5,000.
(C) The shopping center manager’s letter of
Question 12 explanation arrived too late.
A large bookstore entered into a written (D) The contract provided that the store
agreement with the manager of a shopping center premises will be ready by August 10 “at the
to rent space that the manager agreed to fix up latest.”
to suit the bookstore’s purpose. The bookstore
explained to the manager that classes at the local
college begin on August 15 and in order to sell
textbooks to those students, the store must be
up and running before that date. The contract
between the parties provided that the store
premises would be ready by August 10 at the
latest. The manager immediately contracted with
CONTRACTS - CONDITIONS, DISCHARGE, AND BREACH QUESTIONS 7.

Question 13 Question 14

An automobile manufacturer entered into a A patient went to a highly reputable dentist,


contract with a well-known tire distributor. The who honestly told her that she needed a lot of
contract provided that the distributor would dental work, some of it involving complex proce-
deliver to the automobile manufacturer 1,000 dures. When she asked the dentist what the cost
tires on the 15th of each month for $50,000 per would be, he told her “about $3,500.” The patient
shipment, with payment due upon receipt. agreed to use him as her dentist and he began
her treatment. When the patient’s treatment was
The automobile manufacturer and the tire finished, the dentist sent her a bill for $4,100,
distributor properly fulfilled their contractual explaining that the higher bill was because
obligations for two months. The day after the more expensive inlays were used, and that he
third tire delivery, the tire distributor’s presi- carefully documented the cost of his materials
dent visited the automobile manufacturer and had sound medical reasons for his decision.
and found the automobiles produced with his The patient honestly believed that it was unfair
company’s tires to be a “disgrace.” To protect for the dentist to charge her $4,100. Therefore,
the tire distributor’s reputation, the president she sent the dentist’s invoice back to him, along
announced that he will not send any additional with a check for $3,500. On the check the patient
tire shipments. The automobile manufacturer had clearly written in large letters “Payment In
immediately brought a breach of contract action Full.” The dentist read the notation on the check
against the tire distributor. and deposited it at his bank. The dentist made
no notation of his own on the check other than
Which party is likely to prevail in the breach his signature on the back as an indorsement. Two
of contract suit? weeks later the dentist called his bank to make
sure that the patient’s check had cleared. He then
(A) The tire distributor, because it was not in immediately filed suit against the patient for
breach at the time the action was filed. $600.
(B) The tire distributor, if it can show that its Is the dentist likely to succeed in his action
reputation would be harmed by having its against the patient for $600?
tires associated with the automobiles.
(A) Yes, because the dentist merely estimated
(C) The automobile manufacturer, because the the cost of the dental work to be $3,500.
president’s action is an anticipatory repudia-
tion. (B) Yes, because the dentist can document that
the precious metal inlays were medically
(D) The automobile manufacturer, because the necessary and that he charged a fair price
president’s action amounts to a prospective for them.
inability to perform.
(C) No, because there has been an accord and
satisfaction of the original debt.

(D) No, because the patient's duty has been


discharged by an account stated.
8. CONTRACTS - CONDITIONS, DISCHARGE, AND BREACH QUESTIONS

Question 15 Question 16

In anticipation of pending domestic agricul- An owner and a builder executed a contract


ture regulations that would make growing corn providing that the builder was to construct a
less profitable, a candy company entered into a residence on a specified location according to
contract with a corn farmer who grew a particu- plans and specifications. The total contract price
larly good variety of corn, whereby the candy was $500,000. The lot on which the residence
company was given the right to purchase all high was to be built was located on the seashore and
fructose corn syrup refined by the corn farmer there was an existing wood frame structure that
for the next five years at a price set at 95% of the had to be demolished before the residence could
domestic market price at the time of delivery. be built. No date was included in the contract for
The candy company agreed to purchase no less completion of the home. The contract between
than 1,000 liters of corn syrup a week and to use the owner and the builder stated that construc-
its own trucks to transport the corn syrup to its tion would begin within two weeks after the
storage facilities. At the time this contract was existing structure was demolished and the
signed, the candy company gave written notice rubble removed from the lot. The day after the
to the corn farmer that it intended to buy all preexisting structure was demolished and the
high fructose corn syrup produced by the corn rubble removed, a hurricane eroded the seashore,
farmer until further notice. Thereafter, the candy resulting in the owner’s lot being now under
company continued to purchase all of the corn water.
farmer’s total corn syrup production until the
following year. At that time, the corn farmer, by The builder will not need to perform the
letter, notified the candy company that it could contract because:
no longer deliver corn syrup to it in accordance
with their agreement because the new domestic (A) The contract is void because the subject of
agriculture regulations had rendered growing the contract was destroyed through no fault
corn unprofitable for him. of the parties.

If the candy company sues the corn farmer (B) The builder is discharged of his obligation
for breach of contract, is the farmer likely to because of impossibility of performance.
prevail?
(C) The increased costs of construction would
(A) Yes, because corn is widely available. bankrupt him.

(B) Yes, because its performance was excused (D) The contract is void because of mutual
due to impracticability. mistake.

(C) Yes, because the candy company was


aware of the possibility of new government
regulations when it entered into the agree-
ment.

(D) No.
CONTRACTS -
CONDITIONS,
DISCHARGE, AND
BREACH ANSWERS
CONTRACTS - CONDITIONS, DISCHARGE, AND BREACH ANSWERS 1.

CONTRACTS - CONDITIONS, DISCHARGE, AND BREACH ANSWERS

Answer to Question 1
(A) The landowner will win because the builder’s notice constituted an anticipatory repudiation,
which can be treated as an immediate and total breach of contract. If a contract is executory on
both sides and one party unequivocally notifies the other party that he will not perform when his
duty is due, the nonrepudiating party has the option of treating the repudiation as an immediate
and total breach and suing for damages. Although the repudiator can retract the repudiation, this
must be done before the nonrepudiating party relies on the repudiation. Here, the contract was
wholly executory (because neither party had performed) when the builder said that he would
not perform at the agreed price. Because the builder’s statement was unequivocal, it constitutes
an anticipatory repudiation. Thus, the landowner was free to find someone to substitute for the
builder. Because the landowner did find a substitute, he relied on the repudiation and so the build-
er’s attempted revocation was invalid. (B) is incorrect because the difference in price between the
builder’s demand and the competing building company’s price is irrelevant. Under the common
law, the builder had a contractual duty to perform at the price he agreed to, and his statement that
he would not do so constituted an anticipatory repudiation regardless of the price the builder was
attempting to obtain. (C) is incorrect because the landowner had no duty to inform the builder of
the contract with the competing building company. The repudiating party is the wrongful party in
an anticipatory repudiation situation, and so the law does not impose a duty of notification on the
nonrepudiator. If the repudiator suffers harm by his repudiation, it is his own fault. (D) is incor-
rect because, as stated above, a repudiator can revoke the repudiation only if the nonrepudiating
party has not relied on the repudiation. Here, the landowner had relied (by hiring the competing
building company). Thus, it is too late for the builder to revoke the repudiation.
Answer to Question 2
(C) When the tavern owner accepted the beer on February 16, his duty to pay the $1,400 arose, but
he retained the right to sue for damages because of the late delivery. The UCC requires perfect
tender, which means that the goods and their delivery must conform to the contract in every way
or there is a breach. Here, the brewery breached the contract by delivering the beer late. When a
delivery is nonconforming, the buyer may: (i) reject the goods and cancel the contract or sue for
damages; or (ii) accept any commercial units, reject the rest, and sue for damages. Once the goods
are accepted, the buyer generally is bound on the contract and it is too late to cancel; however,
the buyer retains the right to sue for damages for any nonconformity. When the tavern owner
accepted the beer, he became bound under the contract to pay for the beer. Nevertheless, he can
sue for damages that arose from the late delivery. (A) is incorrect because the duty to pay for the
beer did arise when the tavern owner accepted the delivery. (B) is incorrect for two reasons: the
duty to pay arose upon acceptance of the delivery, and the tavern owner did not waive his right to
sue for damages. (D) is incorrect because the tavern owner retained the right to sue for damages
despite accepting the delivery.
Answer to Question 3
(C) Under the doctrine of substantial performance, contracts governed by the common law are
enforceable despite minor breaches. The contract here—to build a manufacturing plant—is
governed by the common law. In determining whether a breach is minor or substantial, courts
look to whether the party received the substantial benefit of the bargain. Here, the manufacturer
got a plant that was, perhaps, better than the one called for in the contract. Therefore, the contract
will be enforceable under the doctrine of substantial performance. (A) is incorrect because the
2. CONTRACTS - CONDITIONS, DISCHARGE, AND BREACH ANSWERS

perfect tender doctrine applies only to contracts for the sale of goods—not to common law
contracts. (B) is incorrect. Under the divisibility doctrine, if a party performs one of the units of a
divisible contract, he is entitled to the agreed-on equivalent for that unit even if he fails to perform
the other units. It is not a condition precedent to the other party’s liability that the whole contract
be performed. For a contract to be divisible: (i) the performance of each party must be divided
into two or more parts under the contract, (ii) the number of parts due from each party must be
the same, and (iii) the performance of each part by one party is agreed on as the equivalent of the
other party’s corresponding part. Here, the payment is broken into two parts, but the payments
do not correspond to units of work performed by the contractor, so this contract is not divisible.
In any case, divisibility is unlikely to be of much help in deciding this case unless the installation
of the lights was a separate task under the contract, with a corresponding payment. (D) is incor-
rect because quasi-contract provides a remedy in some situations in which a contract fails (or is
absent) and the defendant would be unjustly enriched as a result. The doctrine is not applicable
here because, as discussed above, the contract will not fail due to a minor breach.

Answer to Question 4

(C) The director is entitled to find a substitute to perform in the actor’s absence. If the only way the
director could acquire a substitute was to agree to the extended term, then the director’s actions
would be proper. (A) is wrong because this would not relieve the director from liability to the
actor. (B) is wrong because the actor does not have to accept any job under the contract, only the
job that was the subject of the contract or a similar job to avoid damages. (D) is wrong because
even if the actor did not wish to continue in the role, this factor would not excuse the director
from his liability under the contract.

Answer to Question 5

(D) The man has the option of defending in equity under a specific performance theory or waiting
until his colleague obtains a judgment against him and then suing her for breach. He has either
option available because his colleague is in breach of their accord agreement. An accord is an
agreement in which one party to an existing contract agrees to accept, in lieu of the perfor-
mance that she is supposed to receive from the other party, some other, different performance.
The accord must be supported by consideration, but the consideration is sufficient if it is of a
different type than called for under the original contract, even if the substituted consideration
is of less value. An accord suspends the right to enforce the original agreement. Performance
of the accord (i.e., satisfaction) cuts off the parties’ rights to enforce the original contract and
discharges the accord. Here, the accord was supported by sufficient consideration because the
man was giving a ring in lieu of some cash. The man’s duties under the accord were discharged
when he timely tendered delivery of the ring and cash. By refusing the ring and filing suit for
the part of the original debt that has not been paid, the colleague has breached the accord agree-
ment. If a creditor breaches an accord agreement, the debtor has the option of either raising the
accord agreement as an equitable defense in the creditor’s action and asking that it be dismissed,
or waiting until he is damaged (i.e., until the creditor is successful in an action on the original
contract) and then bringing an action at law for damages for breach of the accord contract. (A)
is incorrect because the amount of the debt does not have to be in dispute to have an enforceable
accord, as long as there was some alteration in the debtor’s consideration, as discussed above. (B)
is incorrect because the colleague would have the right to enforce the original contract only if the
man had breached the accord agreement. Here, the man’s tender of the ring discharged his duty
under the accord agreement, precluding his colleague from suing on the original contract. (C) is
incorrect because the man has both a breach of contract remedy and an equitable defense option
CONTRACTS - CONDITIONS, DISCHARGE, AND BREACH ANSWERS 3.

available to him. Also, whether the ring is unique does not affect his right to specific performance
of the accord agreement; he is simply raising the agreement as an equitable defense to prevent the
colleague from continuing with her suit on the original contract.

Answer to Question 6

(D) The beekeeper’s refusal to perform was a breach of contract. Discharge of contractual duties by
impossibility must be objective; i.e., the duties could not be performed by anyone. Generally,
physical incapacity of a person necessary to effectuate the contract only excuses performance
where the services are deemed “unique.” That would not be the case here. The beekeeper’s
allergy to the bees does not constitute objective impossibility of performance because his duties
are delegable. Hence, his failure to deliver is not excused. (B) is wrong because his performance
would be excused only if the parties had entered into a contract for personal services. (A) is wrong
because the beekeeper promised to supply at least 100 pounds per month. (C) is wrong because
such notice would not excuse a breach.
Answer to Question 7

(C) The least correct statement refers to a condition subsequent because a condition subsequent is one
the occurrence of which cuts off an already existing absolute duty of performance. The sister’s
tendering of good title would not cut off the brother’s duty to perform. (A) and (B) are wrong
because they are not inaccurate statements. A condition precedent is one that must occur before an
absolute duty of immediate performance arises in the other party. When conditions are concurrent
as they are here (as discussed below), it can be said that each condition is a condition precedent
to the other. (D) is wrong because conditions concurrent are those that are capable of occurring
together (which describes the brother and sister’s relationship regarding the exchange of the farm
and the apartment house) and the parties are bound to perform at the same time. In effect, each is
a condition precedent to the other.

Answer to Question 8

(C) The appraiser likely will not recover a percentage of the receipts from the manager. In construing
the contract, the court will attempt to give effect to the reasonable expectations of the parties.
Even though they did not expressly so provide, under the circumstances, the parties probably
intended the bonus to be incentive for the appraiser to stay for the entire year and that it be paid
only if the appraiser completed the term of the contract. Thus, such a condition will be implied.
(A) is contrary to the facts because there were four months left in the 12-month contract. Thus, the
breach was substantial. (B) is wrong because the manager’s remedy does not affect the appraiser’s
right to payment. (D) is wrong because the condition was implied, not express.

Answer to Question 9

(A) The winery may refuse the shipment if notice of termination is given but will not recover damages
for breach. This problem is governed by UCC sections 2-613, 2-615 and 2-616. When crops are
destroyed in the case of a farmer who has contracted to sell crops from a designated tract of land,
the loss may be governed either by UCC section 2-613 (casualty to identified goods) or section
2-615 (failure of presupposed conditions/commercial impracticability). The result is the same
under either section. Under section 2-613, if goods identified when the contract is made suffer
casualty without fault of either party before the risk of loss passes to the buyer, the contract may
be avoided. If the loss is partial, the buyer may treat the contract as avoided or accept the goods
with allowance from the price for the deficiency. The buyer does not have any further rights
4. CONTRACTS - CONDITIONS, DISCHARGE, AND BREACH ANSWERS

against the seller and thus cannot sue the seller for breach. Under section 2-615, a crop failure
resulting from an unexpected cause excuses a farmer’s obligation to deliver the full amount as
long as he makes a fair and reasonable allocation among his buyers. The grape grower has done
this by allocating pro rata between the winery and the vineyard. Nevertheless, under UCC section
2-616, the buyer may either accept the proposed modification or terminate the contract. Under
either provision, the winery is free to reject the 250 tons of grapes. Thus, (B) is wrong. (C) is
wrong because even though alternative sources are available, the grape grower is not obligated to
use them because the contract was tied to a designated parcel of land—“my [the grape grower’s]
ranch.” (D) is wrong because it is contrary to the provision of UCC section 2-615, which permits
the farmer to make an allocation.

Answer to Question 10

(B) The legal effect of the two circumstances is best stated by (B). When a condition or duty of
performance (i.e., payment) is broken, the beneficiary of the condition or duty has an election: it
may (i) terminate its liability; or (ii) continue under the contract. If it chooses the latter course,
it will be deemed to have waived the condition or duty. Because the engineering firm did not
terminate its liability to complete construction of the roller coaster, but rather treated the contract
as ongoing, it is deemed to have waived the delay in payment. The amusement park’s failure to
require a safety report would be deemed a revocable waiver. At any time the amusement park
could insist on the safety report before making a monthly payment. Because the safety report
represents that the work performed during a particular month met safety guidelines, the amuse-
ment park would always retain the right to condition its payment for a particular month on receipt
of the report. (A) is incorrect because there was no estoppel waiver as to either the failure to pay
or the failure to provide safety reports. Whenever a party indicates that it is “waiving” a condi-
tion before it is to happen, or some performance before it is rendered, and the person addressed
detrimentally relies on such an indication, the courts will hold this to be a binding (estoppel)
waiver. The engineering firm never indicated in advance that it was waiving the payments during
hurricane season and there was no detrimental reliance by the amusement park. Likewise, the
amusement park never indicated in advance that it was waiving the safety report requirement and
there was no detrimental reliance by the engineering firm. (C) is incorrect because mutual rescis-
sion requires an express agreement between the parties to rescind. The agreement to rescind is
itself a binding contract supported by consideration (the giving up by each party of its right to
counterperformance from the other). The parties did not rescind the contract here. (D) is incor-
rect because, as noted above, there was no estoppel waiver as to the safety reports. Also, there
was never any discharge of the amusement park’s duty to make the payments during the hurricane
season. (Although the amusement park’s duty might be excused because no work was done during
that time, this does not necessarily follow from the terms of the contract.)

Answer to Question 11

(A) A condition subsequent is one the occurrence of which cuts off an already existing duty of perfor-
mance. The form of the condition requiring removal of the pool is that of a condition subsequent
because, under the language of the contract, failure to do so will cut off the buyers’ duty to pay
the $5,000. A condition precedent is one that must occur before an absolute duty of immediate
performance arises in the other party. The substance of the pool removal provision is that of a
condition precedent because no duty to pay $5,000 arises until after the sellers have removed the
pool. (B) and (C) are wrong because the buyers are not under a duty to pay the $5,000 before the
sellers are required to remove the pool. (D) is wrong because removal of the pool is a condition
precedent to the buyers’ duty to pay the $5,000.
CONTRACTS - CONDITIONS, DISCHARGE, AND BREACH ANSWERS 5.

Answer to Question 12

(D) The most important fact in determining the resolution of this case is the provision requiring the
premises to be ready on August 10 “at the latest,” which is a time of the essence provision. To
relieve a party of its duty to perform, a breach must be material. Thus, if the shopping center’s
breach was material, the bookstore will prevail; if it was minor, the shopping center will prevail.
Failure to perform on time is a breach, but it is not a material breach unless time is of the essence.
Although the “at the latest” language does not on its own make time of the essence, a court will
look at the circumstances (i.e., the manager was aware when they entered into the contract that
opening by August 10 was crucial to the bookstore because of the sale of textbooks to college
students). Thus, the shopping center’s failure to perform within the time specified was a material
breach that gave the bookstore the absolute right to cancel the contract when the premises were
not ready by that date. (A) is incorrect because the ability to find a substitute tenant would bear
on the shopping center’s damages if the bookstore were in breach, but it would have no effect on
whether there was in fact a breach. (B) is incorrect because, regardless of whether the builder’s
breach was foreseeable, the shopping center’s duty to perform within the time agreed to was
not discharged. Where the nonoccurrence of the event was a basic assumption of the parties in
making the contract and neither party has expressly or impliedly assumed the risk of the event
occurring, contractual duties may be discharged. More specifically, duties may be discharged
for impracticability when a party has encountered: (i) extreme and unreasonable difficulty and/
or expense; and (ii) its nonoccurrence was a basic assumption of the parties. Here, the builder’s
performance of his contract was not a basic assumption of the parties in making the contract, and
in any case, the shopping center assumed the risk of the builder’s nonperformance. The shopping
center hired and oversaw the builder, and had the ability to hire and pay a substitute builder to
complete the project on time. (C) is incorrect because the letter of explanation has no legal effect,
regardless of when it was received.

Answer to Question 13

(C) The refusal of the tire distributor’s president to send any additional tire shipments to the automo-
bile manufacturer amounted to an anticipatory repudiation, which breaches the contract. An
anticipatory repudiation arises where a promisor, by words or conduct, unequivocally indicates
that he cannot or will not perform when the time comes. In such a case, the other party has
several options, including treating the repudiation as an immediate breach and suing for damages.
Thus, (C) is correct and (A) is incorrect. (B) is incorrect because the “disgraceful” quality of the
automobiles and the possible detriment to the tire distributor's reputation would not be a defense
under the contract because the contract made no provision regarding the quality of the automo-
bile manufacturer’s product. (D) is incorrect because the president’s actions were an anticipa-
tory repudiation and not a prospective inability to perform, which involves conduct or words that
merely raise doubts that the party will perform and do not rise to the level of an unequivocal
refusal to perform. Here, the president refused without equivocation to send any additional tire
shipments to the automobile manufacturer. A prospective inability to perform would not permit
the automobile manufacturer to treat it as an immediate breach. It would give it the right to
demand assurances.

Answer to Question 14

(C) The dentist will not prevail because the contractual duty was discharged by an accord and satis-
faction. An accord is an agreement in which one party to an existing contract agrees to accept,
in lieu of the performance that she is supposed to receive from the other party, some other,
6. CONTRACTS - CONDITIONS, DISCHARGE, AND BREACH ANSWERS

different performance. Satisfaction is the performance of the accord agreement. The effect of
this performance is to discharge both the accord agreement and the original contract. An accord
and satisfaction may be accomplished by a good faith tender and acceptance of a check conspicu-
ously marked “payment in full” where there is a bona fide dispute as to the amount owed. The
dentist and the patient agreed that the dentist would perform the dental work needed by the patient
in return for the patient’s promise to pay “about $3,500.” The facts indicate that, following the
actual dental work, there ensued a good faith dispute as to whether the patient owed only $3,500
or the additional $600 as well. The patient in good faith tendered to the dentist a check marked
“payment in full,” which notation the dentist saw before he deposited the check into his account.
The dentist’s acceptance of the patient’s check gives rise to an accord and satisfaction with regard
to the disputed original debt. Thus, the dentist is deemed to have accepted the amount tendered
by the patient as full payment for the dental services performed, and the patient’s duty to pay
for the services is discharged. (A) is incorrect because, even if the dentist could have success-
fully asserted that $4,100 was within the range of the amount “about $3,500,” his actions with
respect to the check constitute an acceptance of the amount tendered therein. (B) is incorrect
because, with the existence of an accord and satisfaction, the patient’s duty to pay is deemed to be
discharged. Thus, it is irrelevant at this point that the inlays may have been medically necessary
or that the dentist charged a fair price for them. (D) is incorrect because an account stated is a
contract whereby parties agree to an amount as a final balance due. Here, there was no agreement
of the parties as to a final balance due. Moreover, for an agreement to be an account stated, there
must be more than one prior transaction between the parties. The dispute between the patient and
dentist involves only one transaction. Thus, there is no discharge by account stated.

Answer to Question 15

(D) The farmer is unlikely to prevail because his refusal to deliver the corn syrup is nonjustifiable
and a breach of contract. (A) is wrong because the availability of corn from other suppliers does
not affect whether the farmer breached the contract. It may affect the candy company's damages.
(B) is wrong because the lack of profitability is not sufficient to excuse performance because of
impracticability. For performance to be excused, it must have become extremely and unreasonably
difficult due to an unanticipated or extraordinary event, the nonoccurrence of which was a basic
assumption of the contract. Here, it is not clear that performance would be extremely and unrea-
sonably difficult. More importantly, the pending regulations were not an unanticipated occurrence
and were the type of event that the farmer would assume the risk for. (C) is wrong because the
candy company’s knowledge of the new regulations does not have any bearing on the farmer's
performance.

Answer to Question 16

(B) Prevention of performance by an irresistible, superhuman cause is an excuse for nonperformance


of a contract, unless the parties stipulate to the contrary. The destruction of the lot by the forces of
nature rendered performance impossible, so the builder need not perform. (A) is wrong because
destruction of the property does not make the contract void; it merely discharges the builder. (C)
is wrong because the builder’s bankruptcy is irrelevant for this purpose. (D) is wrong because
there was no mutual mistake.
CONTRACTS -
CONSIDERATION
QUESTIONS
CONTRACTS - CONSIDERATION QUESTIONS 1.

CONTRACTS - CONSIDERATION QUESTIONS

Question 1 Question 2

The general partner of a two-person partner- A general contractor advertised in a trade


ship committed suicide because of gambling publication that she planned to bid on the
debts. His partner, who was a limited partner construction of a new building. The advertise-
and the general partner’s uncle, was besieged ment welcomed bids from subcontractors to
by creditors of the partnership who wanted perform various functions, such as plumbing,
their money. The jurisdiction statutorily limited electrical work, and masonry. The lowest
the liability of limited partners for debts of the plumbing bid she received was from a plumber
partnership or acts of the general partner to the for $10,000. The general contractor used that
extent of their investment in the partnership. bid in preparing her bid and submitted the bid to
However, the limited partner was unaware of the client. Shortly thereafter, the plumber called
this and believed that he was liable to all who her and explained that there was a mistake in
had claims against the partnership. The limited his bid and he could not perform the work for
partner told an unsecured creditor of the partner- less than $12,000. The general contractor was
ship that he would pay the partnership debt if awarded the contract and told the plumber she
the unsecured creditor would hold off filing was accepting his $10,000 bid, but the plumber
an involuntary bankruptcy petition against the reiterated that he would not do the work for less
partnership. In a bankruptcy action filed by than $12,000. The general contractor hired a
secured creditors, the assets of the partnership, different company to do the plumbing work on
which were very small, were consumed by the the building at a cost of $12,000. She now sues
costs of the proceedings and no creditor received the plumber for damages.
any payment. The general partner himself left no
assets. The unsecured creditor brought an action What is she entitled to recover?
against the limited partner for the amount of the
debt owed to him by the partnership. (A) $10,000.

Will the unsecured creditor be successful in (B) $2,000, which represents the difference
collecting the debt from the limited partner? between the plumber’s bid and the amount
she had to pay for the plumbing work.
(A) No, because his claim against the partner-
ship was worthless. (C) Nothing, because $12,000 was a reasonable
amount to pay for the work performed.
(B) No, because the limited partner’s only
obligation to pay the debts of the partner- (D) Nothing, because the general contractor did
ship was a moral one based on his relation- not accept the plumber’s bid before it was
ship to the general partner. withdrawn.

(C) Yes, because the unsecured creditor detri-


mentally relied on the limited partner’s
promise.

(D) Yes, because the limited partner’s promise


was supported by a bargained-for exchange.
2. CONTRACTS - CONSIDERATION QUESTIONS

Question 3 (D) The boutique owner conducted an adver-


tising campaign and made substantial
On April 1, the owner of a high-end boutique improvements to her store in reliance on the
and a dress designer signed a written agree- agreement.
ment wherein the boutique owner was appointed
the “sole and exclusive” retail distributor for Question 4
the designer’s clothing in an upscale commu-
nity. The contract provided that the designer Neighbors of an apparently destitute couple
was to have the absolute right to cease doing bought a month’s supply of food and gave it to
business with the boutique owner “at any time them. Later, the wife confided in the neighbors
and on 30 days’ notice.” On May 1, the boutique that she and her husband did have money and
owner handed the designer a written order for that, because they had been so kind, she was
$50,000 worth of his original gowns. To publi- leaving them money in her will. When the wife
cize her new line of merchandise, the boutique died, at the neighbors’ request the husband gave
owner conducted a large advertising campaign the neighbors the following signed instrument:
announcing to the community that she would “In consideration of my wife’s promise to our
have a wide selection of the designer’s originals neighbors, and their agreement not to sue her
on display. She also made substantial improve- estate, I agree to pay them the sum of $5,000.”
ments to the store to display these clothes in lush When the husband died of a heart attack several
and expensive settings. days later, the neighbors asked the administrator
of his estate to pay them the $5,000. The admin-
On June 15, the designer faxed the boutique istrator refused on the ground that there was no
owner advising her that her competitor had consideration for the agreement.
made a proposal that he simply could not pass
up, so he was invoking his right to terminate On which of the following theories would it be
their arrangement on July 15. On receipt of the most likely that the neighbors would recover?
message, the boutique owner filed suit against
the designer, seeking to prevent him from termi- (A) The husband’s written instrument was a
nating the exclusivity agreement. In defense, the binding unilateral contract.
designer contended, among other things, that the
April 1 exclusivity agreement with the boutique (B) The husband’s acceptance of the food was
owner is unenforceable for lack of consideration, fraudulent.
because the boutique owner did not obligate
herself under the contract in any way. (C) The husband is bound by promissory
estoppel.
What is the best argument to rebut this
contention? (D) The husband and the neighbors entered into
a valid compromise.
(A) Any want of consideration at the formation
stage was cured by the boutique owner’s
actual tender of a large order.
(B) The April agreement was, in effect, a “firm
offer” between merchants and therefore
binding on both parties even though there
was no consideration.
(C) There was sufficient consideration in
the boutique owner’s implied promise to
exercise her best efforts to promote the sale
of the designer’s gowns.
CONTRACTS - CONSIDERATION QUESTIONS 3.

Question 5 Question 6

A third-year college student telephoned his A manufacturer of a very popular, artisanal


father and asked for $1,000 for living expenses. red cheese enters into an agreement with a retail
The father agreed to send him the money seller of gourmet foods. The agreement provides
but told him that he needed to find a classier that the retailer will buy and the manufacturer
girlfriend or he would get no more money. The will sell all of the red cheese that the retailer
son thanked his father, and promptly went to a requires for two years at $500 per 10-wheel
jeweler and selected an engagement ring for his container. In the past two years, the retailer has
girlfriend priced at $5,000. The son, who was sold 10 containers per year but estimates that
21 years of age, signed a contract to purchase it could sell twice that amount with a steady
the ring. The contract required the son to make supply.
a $1,000 down payment and then to make a
series of installment payments. The son planned Under the above facts, what are the relative
to use the $1,000 check he was expecting from obligations of the manufacturer and the retailer?
his father for the down payment. The father
discovered the son’s plan to buy the engagement (A) The retailer must buy red cheese exclusive-
ring and refused to send the $1,000 check. The ly from the manufacturer, and the manufac-
jeweler is now demanding that the son make the turer must sell its red cheese exclusively to
$1,000 down payment on the ring and pay the the retailer.
first installment as well.
(B) The retailer must buy red cheese exclusively
Can the son legally enforce his father’s from the manufacturer, and the manufac-
promise to send the $1,000? turer may sell its red cheese to anyone.

(A) Yes, because the son relied on his father’s (C) The retailer may buy red cheese in excess
promise and the doctrine of promissory of 20 containers from another manufac-
estoppel applies. turer, and the manufacturer may refuse to
sell to retailer an amount in excess of 20
(B) Yes, because the son was an intended containers.
beneficiary.
(D) The retailer is not required to buy any
(C) No, because the father’s promise to pay for cheese from the manufacturer if it has a
his son’s expenses was not in writing. good faith reason, but the manufacturer
may sell its red cheese only to the retailer.
(D) No, because the father did not promise to
send the money with the expectation of
inducing the son to buy an engagement ring
for his girlfriend.
4. CONTRACTS - CONSIDERATION QUESTIONS

Question 7 Question 8

A pâté manufacturer entered into a written A homeowner was postponing shoveling the
agreement with a gourmet food store. The snow off her driveway one Saturday morning
manufacturer agreed to sell “all” its “output of because it was so cold outside. When she heard
liver pâté ” to the store and the store agreed to a scream and ran outside, she found the neigh-
sell the manufacturer’s pâté “exclusively.” The borhood newspaper delivery boy injured, lying
agreement between the store and the manufac- by his bicycle in her driveway. Feeling terribly
turer also contained the statement, “either party guilty, she took the boy to a local doctor and
may cancel this contract after two months on wrote a note to the doctor stating that, if the
giving reasonable notice to the other party.” doctor would treat the boy, the homeowner
After the manufacturer filled the store’s orders would cover all of the resulting costs.
for six months, the manufacturer determined
that it was becoming too costly to operate and The boy later admitted that he was attempting
maintain the special oven used to roast the pâté to ride the bicycle with his eyes closed, and that
loaves, and that it would be difficult and expen- his fall had nothing to do with the snow in the
sive to find a suitable substitute. The manufac- homeowner’s driveway. After treating the boy
turer, therefore, notified the store that it was for his injuries until he recovered, the doctor sent
getting out of the pâté business, and explained the homeowner a bill for his services. When the
why, and that it would stop shipping pâté to the homeowner refused to pay, the doctor brought an
store after 60 days. The store sued the manufac- action to recover the amount of his bill.
turer, demanding that the manufacturer continue
to ship pâté to the store or pay monetary In the doctor’s suit against the homeowner,
damages. which party is likely to prevail?

Will the store prevail? (A) The doctor, because he gave medical treat-
ment to the boy after receiving the home-
(A) Yes, because it was not impossible for the owner’s note.
manufacturer to perform.
(B) The doctor, because the homeowner’s
(B) Yes, because the manufacturer assumed the promise to pay the boy’s medical expenses
risk that making pâté would become expen- was in writing.
sive.
(C) The homeowner, because there was no
(C) No, because the expense of fixing the oven consideration for her promise to the doctor.
provides a good faith reason for stopping
production. (D) The homeowner, because she derived no
benefit from the medical services rendered
(D) No, because the cancellation provision to the boy.
made the contract illusory.
CONTRACTS - CONSIDERATION QUESTIONS 5.

Question 9 Question 10

A car manufacturer entered into contracts Several years ago, a lender lent a borrower
with various refineries to purchase the gasoline $1,000, and the parties agreed in writing that
they produced for the manufacturer’s dealer- the borrower would repay the lender within one
ships. One agreement with an oil dealer provided year. The borrower failed to repay the lender, but
that the car manufacturer was given the right to the lender took no action prior to the expiration
purchase all gasoline refined by the oil dealer of the five-year statute of limitations on suits for
for the next five years at a price set at 95% of the debt. Some time after that, the lender phoned
domestic market price at the time of delivery. the borrower and told him, “If you’ll pay me
The car manufacturer agreed to purchase no less $600 now, I’ll forget all about that unpaid $1,000
than 5,000 gallons a week and to use its own debt.” The borrower agreed orally and then sent
tankers to transport the gasoline from the oil the lender a signed letter, which stated, “I, the
dealer’s refinery to its storage facilities. At the borrower, agree to pay the lender $600.” The
time the contract was signed, the car manufac- borrower never paid the lender the $600 and the
turer gave written notice to the oil dealer that it lender sued the borrower shortly thereafter.
intended to buy all gasoline produced by the oil
dealer until further notice. What is the lender entitled to recover?

This agreement between the car manufacturer (A) $1,000, because the agreement by the bor-
and the oil dealer was: rower to pay the lender $600 revived the
original $1,000 obligation.
(A) Unenforceable because of the failure to set
a specific price for the gasoline. (B) $600, because the borrower’s moral obliga-
tion to pay the lender $1,000 became the
(B) Unenforceable because it was for an unrea- consideration for the borrower’s agreement
sonable period of time. to pay the lender $600.

(C) Enforceable as to price, but not as to the (C) Nothing, because the statute of limitations
amount of gasoline the oil dealer agreed to has run.
sell.
(D) Nothing, because no additional consider-
(D) Enforceable in all respects. ation was provided to support the borrow-
er’s agreement to pay the lender $600.
6. CONTRACTS - CONSIDERATION QUESTIONS

Question 11 (D) The student did not know that her boyfriend
would not be able to play basketball when
A student and her boyfriend were going away she offered to “make good any losses.”
to college and had quite a few personal belong-
ings to transport. The student drove in her car Question 12
while her father and her boyfriend rode in the
father’s van. About halfway to the college, An aunt agreed to pay for any lace shawl that
while the van and the car were driving down her niece purchased at a certain local shop if
the freeway, the van suddenly swerved out of the niece wore the shawl to the town’s ethnic
control and ran off the highway, ending up on festival. The niece despised shawls, but she
its side in the center divider. When the student really loved her aunt and did not want to hurt
ran to the van, she discovered to her horror that her feelings, so she purchased a $300 lace shawl
her father was dead, and her boyfriend appeared from the shop. She accompanied her aunt to the
to be injured, but not severely. Because her ethnic festival wearing the shawl, and the aunt
father previously had heart trouble, the student was very pleased. The aunt died shortly after the
assumed that he had had a heart attack while festival and her estate refused to reimburse the
driving. Filled with remorse, the student told niece for her purchase. The niece filed suit to
her boyfriend, “I’m so sorry about this. I’ll collect the $300 from the aunt’s estate.
make good any losses you suffer because of this
accident.” Which legal theory will offer the niece her
best chance of winning the case?
The boyfriend learned that he had suffered an
injury to his spinal column that would prevent (A) Quasi-contract.
him from ever playing basketball again. He
had been a scholarship athlete in basketball at (B) Bargain and exchange.
the college and was considered to be a certain
high draft selection for a professional basket- (C) Conditional gift.
ball league when he graduated. The boyfriend
brought an action against the student for several (D) Account stated.
million dollars in damages. A subsequent inves-
tigation revealed that the accident was caused
solely by a defect in the steering mechanism of
the van.

Which of the following is the best defense


the student could assert against her boyfriend’s
claim?

(A) There was no consideration supporting


the student’s promise to her boyfriend to
“make good any losses.”

(B) The student did not intend to offer to pay


the boyfriend for the loss of his professional
career when she said she would “make
good any losses.”

(C) The student was in error when she assumed


that her father’s heart attack was the cause
of the accident.
一 信:liuxue119118 , 们 信免 供

CONTRACTS - CONSIDERATION QUESTIONS 7.

Question 13 Question 14

A high school graduate was asked by his A professional wrestler entered into a written
elderly aunt to live with her in her large brown- agency contract with an agent, who agreed to try
stone and attend to the household activities for to get the wrestler’s picture on a variety of food
the rest of her life, in exchange for the house. products. The wrestler promised that the agent
The graduate agreed, and moved from his would have the exclusive right to promote the
parents’ home to the brownstone. For eight years, wrestler on food product lines. They agreed that
he attended to his aunt’s personal needs and the wrestler would receive 70% of the proceeds
maintained the household. No further discussion and that the agent would receive 30%. The
was ever had between the graduate and his aunt agent was able to persuade a breakfast cereal
regarding conveyance of the brownstone. company to put the wrestler’s picture on their
cereal boxes. Shortly after the agent confirmed
Shortly after his aunt died, the graduate was the cereal deal with the cereal manufacturer,
contacted by the aunt’s estranged daughter, the wrestler and the agent agreed orally that
who stated that in the aunt’s last will and testa- henceforth the wrestler would receive 50% of
ment, she had devised the house to her. She sent the proceeds, including proceeds from the cereal
proper notice to the graduate informing him that deal, and the agent would receive the other 50%.
he must vacate the premises by the first of the The wrestler received a $10,000 check from the
month. cereal manufacturer, and he promptly sent the
agent a check for $3,000. The agent demanded
If the graduate brings suit against the an additional $2,000, but the wrestler refused to
aunt’s estate for breach of contract, which of pay.
the following would be the graduate’s best
argument? If the agent sues the wrestler for the $2,000,
who will prevail?
(A) The doctrine of “unclean hands” prevents
the daughter from pressing her claim under (A) The agent, because consideration is not
the will. required for a modification.

(B) The Statute of Frauds need not be satisfied (B) The wrestler, because of the parol evidence
as between family members. rule.

(C) The aunt’s promise induced the graduate to (C) The wrestler, because the agent had a
perform, and injustice can be avoided only preexisting legal duty to secure food
by enforcement. product promotions for the wrestler.

(D) There has been part performance, and that (D) The wrestler, because an exclusive contract
performance can be explained only by the requires that the party given the privileges
existence of the asserted contract. of exclusivity use his best efforts.

一 信:liuxue119118 , 们 信免 供
CONTRACTS -
CONSIDERATION
ANSWERS
CONTRACTS - CONSIDERATION ANSWERS 1.

CONTRACTS - CONSIDERATION ANSWERS

Answer to Question 1

(D) The unsecured creditor will prevail because the limited partner’s promise was supported by a
bargained-for exchange. Legal detriment will result if the promisee does something he was under
no obligation to do or refrains from doing something that he has a legal right to do. The detriment
to the promisee need not involve any actual loss to the promisee or benefit to the promisor. Here,
there was a bargained-for exchange in the unsecured creditor’s forgoing of a claim in which he
had a good faith belief, even though the claim might have been worthless. Thus, (D) is correct,
and (A) is wrong. (C) is not a good answer because there is no need to rely on promissory estoppel
when an enforceable agreement, which can be enforced according to its terms, is present. (B) is
wrong because there was more than a moral obligation. Even though the limited partner may have
been motivated by a feeling of moral obligation, there was a bargained-for exchange between him
and the unsecured creditor.

Answer to Question 2

(B) The plumber can be held liable because his offer will be deemed irrevocable for a reasonable
length of time on a theory of promissory estoppel. Promissory estoppel renders an offer binding
as an option contract even without consideration if the offeror should reasonably expect it to
induce action or forbearance of a substantial character by the offeree before acceptance, and
such action or forbearance is in fact induced. [Restatement (Second) of Contracts §87] The
plumber offered to do the work for $10,000 if the general contractor was awarded the contract.
The plumber should have expected that the general contractor would use this figure to prepare
her bid and that if she were awarded the contract, she may be bound. This is what occurred. The
measure of the general contractor’s damages is the amount she reasonably paid for a substitute
performance, less the amount saved as a consequence of the breach; i.e., the $12,000 paid to the
other plumbing company minus the $10,000 not paid to the plumber, or $2,000. (A) is incor-
rect because the general contractor has not yet paid the plumber anything and has only suffered
$2,000 in damages. The general contractor, therefore, is only entitled to the measure of damages
described above. (C) is incorrect because it is irrelevant that the general contractor paid a reason-
able price for the substitute performance. In the absence of awareness on her part of the plumber’s
mistake, which is not indicated here, she was entitled to have the work done for the contract price,
or to receive damages for the difference between the contract price and the cost of the substitute
performance. (D) is incorrect because the general contractor is entitled to relief, even if she did
not “accept” the plumber’s bid, because she detrimentally relied on it by using it in her bid.

Answer to Question 3

(C) The best argument to rebut the designer’s contention that there was not sufficient consideration in
the boutique owner’s promise is that the boutique owner impliedly agreed to use her best efforts
to sell and promote the designer’s gowns. For a contract to be valid, consideration must exist on
both sides; i.e., promises must be mutually obligatory. Here, the designer has agreed to make the
boutique owner his exclusive retail distributor, but the boutique owner’s promise is less apparent—
what is she obliged to do? The facts do not show any specific obligation. However, in a case where
someone is to be the exclusive distributor, the court will imply a promise to use best efforts to
sell the product. This implied promise is valid consideration both under the common law and
the Uniform Commercial Code (applicable here because goods are involved). Thus, the contract
between the boutique owner and the designer includes an implied promise that the boutique
2. CONTRACTS - CONSIDERATION ANSWERS

owner will use her best efforts to promote and sell the designer’s gowns, and such a promise is
valid consideration. (A) is wrong because merely placing an order for gowns would not cure the
defect in the April 1 contract absent the boutique owner’s implied promise, because the boutique
owner still will not have promised to do anything under that contract (i.e., she is not bound to
do anything referable to the April 1 contract); rather, all she has promised to do is to pay for the
goods that she ordered, which is sufficient consideration only for the May 1 order, not for the
entire contract. (B) is wrong because it does not address the issue raised by the boutique owner. A
firm offer does not mean that the agreement is binding on both parties without consideration—it
just means that a merchant can create an option-type offer without consideration for the holding
open of the offer. (D) is wrong because the boutique owner’s subsequent reliance on the agree-
ment, while it may allow her to recover in restitution, does not address whether there was consid-
eration at the formation stage, which is the issue raised by the designer here. Promissory estoppel
(detrimental reliance) is an equitable doctrine used to enforce a promise when there is no consid-
eration. Here, as discussed above, there was consideration on both sides.
Answer to Question 4
(D) The most likely theory is that the husband and the neighbors entered into a valid compromise. If
the neighbors have given up a good faith claim, their agreement with the husband is a compromise
supported by valid consideration. Thus, there is an enforceable contract. (A) is wrong because
the husband was requesting a promise from the neighbors, not an act. (B) is wrong because the
husband was under no obligation to reject a gift if he did not deliberately induce the neighbors to
give it. (C) is wrong because there is no evidence that the neighbors gave the food in reliance on
any promise made by the husband.
Answer to Question 5
(D) The son cannot enforce the father’s promise on either contractual or promissory estoppel grounds.
An enforceable contract does not exist because there was no consideration for the father’s promise
to send $1,000. The son, the promisee, incurred no legal detriment by either refraining from
doing something that he had a legal right to do or doing something that he had no legal obliga-
tion to do, with such detriment being part of a bargained-for exchange with the father. However,
consideration is not necessary if the promisor should be estopped from not performing. A promise
is enforceable to the extent necessary to prevent injustice if: (i) the promisor should reasonably
expect to induce action or forbearance; and (ii) such action or forbearance is induced. The father
had previously sent the son money for his expenses, and promised to send him the money here at
issue for the same purpose. Given the father’s express disapproval of the girlfriend, he certainly
did not promise to send the money expecting to induce the son to buy the girlfriend a ring. The
father cannot be estopped from refusing to perform a promise where the result would be to
require him to pay for something that he not only was unaware would happen but also of which
he expressly disapproved. Thus, the father’s promise is not enforceable under a theory of promis-
sory estoppel. (A) is incorrect because, although the son relied on the father’s promise in entering
into the contract to purchase the ring, the promise was only made with the expectation of the
son’s incurring normal living expenses. Thus, the son was not justified in relying on the promise
for the purpose that he did, and promissory estoppel does not apply. (B) is incorrect because the
term “intended beneficiary” refers to a person not a party to a contract who is intended to have
rights conferred by that contract. Here, there was no contract between the father and someone
else, pursuant to which the son became entitled to the money. (C) is incorrect because the fact that
the father’s promise was not in writing is irrelevant to its enforceability. (This was not a surety-
ship promise.) Even if the father’s promise had been in writing, it would not be enforceable for the
reasons discussed above.
CONTRACTS - CONSIDERATION ANSWERS 3.

Answer to Question 6

(B) The retailer must buy all of its red cheese from the manufacturer, and the manufacturer may
sell its red cheese to anyone. Exclusivity is implied in a requirements contract; otherwise the
buyer’s promise would be illusory. If the retailer were free to purchase from other suppliers,
the promise would be one to buy as much as the retailer chooses. Consideration for a require-
ments contract exists because the promisor is suffering a legal detriment by parting with the
legal right to buy goods he may need from another source. On the other side, the manufacturer
has suffered legal detriment by agreeing to sell to the retailer any amount it requires at the stated
price. The manufacturer has parted with any discretion in the amount sold to the retailer and its
price. Therefore, there is no need to impose an exclusivity requirement on the manufacturer to
establish consideration. Thus, (A) is wrong. (C) is wrong because the retailer has promised to buy
all of the red cheese it requires from the manufacturer, not simply 20 containers. Likewise, the
manufacturer must sell to the retailer all of the cheese it requires. The only time a variation from
the contract is permitted is when a buyer demands a quantity unreasonably disproportionate to
a stated estimate or any normal prior requirements. In that case, the seller may seek relief from
a court. The estimate does not set the upper limit on the contract. (D) is wrong because, while
it is true that a buyer with a good faith reason need not purchase anything under a requirements
contract, the seller is free to sell its goods to anyone, as discussed above. Thus, the manufacturer
may sell its cheese to any buyer.

Answer to Question 7

(C) The extreme difficulty and expense of repairing or replacing the oven constitutes a good faith
reason for the manufacturer to cancel the contract, as it is entitled to do under the contract on
reasonable notice. Thus, the store will not prevail. Reservation of an unqualified right to cancel or
withdraw from a contract at any time may amount to an illusory promise. However, the promise is
not illusory, and there is a valid consideration, if the right to cancel is restricted in any way. Here,
the right of either party to cancel is restricted because it must be preceded by reasonable notice
to the other party, which was provided. Therefore, the promises are not illusory, and both parties
are bound. The manufacturer finds itself confronted with circumstances that present extreme
and unreasonable difficulty and expense in complying with its contractual duties of supplying
pâté. While this additional test might not be sufficient to discharge the manufacturer’s duties
on grounds of impracticability, it is clearly grounds for supplying the manufacturer with a good
faith reason for canceling the contract. (A) is incorrect because the manufacturer is entitled to
cancel the contract for a good faith reason on giving reasonable notice. It is not necessary to show
impossibility. (B) is incorrect because the fact that the parties agreed to a cancellation provision
indicates that the manufacturer did not assume all risks of increased expenses in making pâté.
The unreasonable amount of expense and difficulty involved with the maintenance and/or replace-
ment of the oven supports the manufacturer’s good faith decision to cancel the contract. (D) is
incorrect because, while some courts have found promises to be illusory if the contract provided
for an unqualified right to cancel the contract at any time, here the right to cancel is restricted by
requirements of good faith reasons for cancellation as well as giving reasonable notice.

Answer to Question 8

(A) The doctor is likely to prevail because he performed his duties under their contract and is entitled
to the homeowner’s performance. The homeowner offered to pay the doctor if he treated the
injured newspaper delivery boy. The doctor accepted by treating the boy; this treatment was also
his consideration for the contract. Thus, the homeowner was contractually bound to pay for the
4. CONTRACTS - CONSIDERATION ANSWERS

medical expenses and (A) is correct. (B) is incorrect because the homeowner’s promise need not
have been in writing to be enforceable. The requirement of a writing under the Statute of Frauds
for promises to pay the debt of another is not applicable here. The homeowner’s promise was not
a suretyship promise that was collateral to a promise of the boy; it was an independent promise
by the homeowner to be primarily liable for the boy’s medical expenses, and therefore need not
be in writing to be enforceable. (C) is incorrect because the doctor’s treatment is the consideration
for the homeowner’s promise to pay. It does not matter that the boy, rather than the homeowner,
received the benefit of the doctor’s services. Services provided to the newspaper boy was the legal
value that the homeowner bargained for. (D) is incorrect because the doctor’s treatment was the
benefit that the homeowner sought to derive from her bargain. As discussed above, it does not
matter that the boy, rather than the homeowner, benefited from the doctor’s services. That was
what the homeowner bargained for, and she received the benefit of her bargain. Also, remember
that the benefit need not have economic value; the homeowner’s benefit here is her peace of mind.

Answer to Question 9

(D) The contract is enforceable in all respects. This is an output contract, meaning that the seller has
promised to sell to the buyer all of the goods the seller produces, and the buyer has agreed to buy
that amount from the seller. (A) is wrong because there is a price term, although it is dependent
on future events. (B) is wrong because there are no facts that show the five years to be “unreason-
able” and even if there were, parties are free to enter into unreasonable contracts. (C) is wrong
because the car manufacturer agreed to buy at least 5,000 gallons a week by the original agree-
ment, and all the gasoline by its notice. Such output contracts are enforceable.

Answer to Question 10

(B) The lender is entitled to recover $600. When a past obligation would be enforceable except for
a technical defense to enforcement, a new promise in writing will be enforceable even in the
absence of any new consideration. As a general rule, a contract requires a bargained-for exchange
between the parties as consideration; “past” or “moral” consideration is usually insufficient.
Among the many exceptions to this rule is where a technical defense such as the statute of limita-
tions bars enforcement of the prior obligation and a new promise is made in writing. In such a
case, courts will state that the “moral” consideration is sufficient consideration for the new agree-
ment, or that the existence of the prior obligation is a substitute for consideration. Regardless of
how the courts characterize it, the new promise will be enforceable only according to its terms,
not the terms of the original obligation. Hence, the lender is entitled to recover $600 on the basis
of the borrower’s signed letter promising to pay that amount. (A) is incorrect because the new
agreement has no effect on the original obligation. The new agreement is what is being enforced,
not the original obligation. Had the borrower instead promised to pay “the debt I owed to the
lender,” the new agreement would be enforceable for $1,000. (C) is incorrect because the statute
of limitations just bars the judicial remedies for enforcing the prior agreement; it does not nullify
the agreement. Hence, the prior agreement may serve as a substitute for consideration if a new
agreement is made. (D) is incorrect because, as discussed above, no additional consideration is
needed for the agreement to pay $600.

Answer to Question 11

(A) The boyfriend did not promise to do anything he was not otherwise obligated to do in bargained-
for exchange for the student’s promise, so no consideration supported the latter’s promise. If the
agreement were otherwise enforceable, none of the other facts offered by the answers would
CONTRACTS - CONSIDERATION ANSWERS 5.

prevent it from being carried out. (B) is incorrect because a court will not examine the promisor’s
intent when making a promise. (C) is incorrect because mistake as to a fact that motivated the
contract is not a defense under these circumstances; the student had assumed the risk of mistake
by making the promise without being certain of the cause of the accident. (D) is similarly incor-
rect. Mistake as to the value of the promise is generally a risk assumed by the parties.

Answer to Question 12

(B) The legal theory of bargain and exchange would offer the niece her best chance of collecting
from her aunt’s estate. To be enforceable, a contract must be supported by consideration. Gener-
ally, consideration requires a bargained-for exchange of something of legal value from each party.
To be bargained for, the promise must induce the detriment and the detriment must induce the
promise. Here, the niece was induced to purchase the shawl and incur a debt in exchange for
her aunt’s promise to pay, and the aunt was induced to pay in exchange for her niece’s promise
to purchase the shawl and attend the festival. It does not matter that the aunt did not receive
an economic benefit—influencing her niece’s mind is sufficient to establish an exchange. Both
parties gave something of legal value because the aunt had no legal duty to pay for a shawl for
her niece and the niece had no legal duty to purchase the shawl and attend the festival. There-
fore, there was a bargained-for exchange of something of legal value and a contract was formed.
(A) is wrong because there was a contract that would be enforceable (see above). Restitution in
quasi-contract is available in cases where a contract is unenforceable and one of the parties has
performed, resulting in unjust enrichment to the other party, or in cases where there is no contract
but the plaintiff has conferred a benefit on the defendant with the reasonable expectation of being
paid, the defendant knew or had reason to know of the plaintiff’s expectation, and the defendant
would be unjustly enriched if she were allowed to retain the benefit without compensating the
plaintiff. This theory would not be appropriate because there is an enforceable contract and an
adequate legal remedy is available. (C) is wrong because a promise to make a gift is unenforce-
able for lack of consideration, and this result is not changed where the promise is conditional and
the conditions have been fulfilled. In fact, an argument that there was a conditional gift would
negate the bargain and exchange needed to establish an enforceable contract, because it would
involve arguing that the aunt did not intend to induce her niece’s detriment (i.e., the aunt was not
seeking to trade her promise to pay in exchange for her niece’s promise to attend the festival), but
rather was merely stating on what conditions the niece could receive her gift. Thus, (C) would
not be helpful to the niece’s case. (D) is wrong because the theory of account stated is used in
cases where the parties to a contract have had a series of transactions and agree to a final balance
due from one to the other. Here, the niece is seeking to enforce a single transaction and, thus, the
theory would not be appropriate.

Answer to Question 13

(C) Since the graduate's agreement with the aunt is for an interest in land and is not evidenced by a
writing, his best argument is for the application of promissory estoppel. When a party’s conduct
foreseeably induces another to change his position in reliance on an oral agreement, courts may
use promissory estoppel to remove the contract completely from the Statute of Frauds. Because
the graduate relied on his aunt’s promise by moving in with her and caring for her, and because
the monetary value of his services is difficult to determine, specific enforcement of the aunt’s
promise is necessary to avoid injustice. (A) is wrong because the daughter has done nothing
but assert her legal rights. She has done nothing wrong that would justify the application of the
unclean hands doctrine. (B) is a misstatement of law; there is no exception to the Statute of Frauds
for family members. (D) is wrong because the graduate’s performance of the oral agreement is not
6. CONTRACTS - CONSIDERATION ANSWERS

such as could be explained only by the existence of an oral agreement to convey the brownstone.
He could have been caring for the aunt for any number of reasons other than in exchange for the
brownstone, including doing it for free out of affection for the aunt.

Answer to Question 14

(C) The wrestler is likely to prevail because of the preexisting duty rule. At common law, a modifica-
tion of a contract required consideration because the parties were under a preexisting legal duty to
perform. Because there was no consideration for the modification, the agent is not entitled to the
$2,000. (The modern view permits modification without consideration if it is fair and equitable in
view of unanticipated circumstances, but no such circumstances exist here.) (A) would be correct
if the UCC applied, but it does not apply to this fact situation. (B) is wrong; the parol evidence
rule does not apply to subsequent modifications. (D) is wrong. It is a true statement but it is not
the reason the wrestler will prevail. There is nothing in the facts to indicate that the agent did not
use his best efforts.
CONTRACTS - DEFENSES
AND PAROL EVIDENCE
QUESTIONS
CONTRACTS - DEFENSES AND PAROL EVIDENCE QUESTIONS 1.

CONTRACTS - DEFENSES AND PAROL EVIDENCE QUESTIONS

Question 1 (D) No, because the phone call from the corpo-
ration’s general counsel to the owner’s
The owner of a gourmet food store entered attorney constituted a valid revocation
into oral negotiations with the president of a before acceptance.
food products corporation to secure an exclusive
distributorship in the state for a popular truffle Question 2
sauce. After some discussion, the parties agreed
on all salient points and shook hands on the A woman had a developmentally disabled
deal. They agreed further that the corporation’s brother who lived in a group home. The woman
general counsel would reduce the agreement to ran errands for her brother, took him to the
writing and that the agreement would become park, and generally made his life pleasant and
effective after it was drawn up and initialed comfortable. The siblings’ grandmother wanted
by the corporation’s general counsel and by to encourage her granddaughter to continue
the store owner’s attorney. The corporation’s helping her brother. Therefore, she called her
general counsel duly committed the agree- granddaughter and told her that if she continued
ment to writing and sent the writing to the store to take care of her brother for the next five years,
owner’s attorney, but without initialing it first. the grandmother would give the granddaughter
The attorney looked over the agreement, made her condominium in Hawaii.
no changes, initialed it, and mailed the agree-
ment to the corporation’s counsel on May 1. On The granddaughter continued to take care
May 2, the president of the corporation decided of her brother. However, two years after their
that a large national grocery chain would be a conversation, the grandmother sold her condo-
better distributor for the sauce than the gourmet minium in Hawaii and told her granddaughter
food store, so he had the general counsel call the that she would not be able to give her the condo-
store’s attorney to say that the deal was off. The minium as a gift, as she had promised. The
corporation’s counsel received the written agree- granddaughter continued to care for her brother.
ment with the attorney’s initials on it on May 3.
If five years after their initial agreement, the
Nevertheless, the corporation began to distribute
granddaughter brings suit against her grand-
its products through the national grocery chain.
mother for breach of contract, which of the
following will not be relevant to the grandmoth-
If the gourmet food store owner files suit er's defense?
against the corporation and its president for
breach, is she likely to prevail? (A) The contract involved the transfer of an
interest in real property.
(A) Yes, because the mailing of the written
agreement to the attorney constituted an (B) The contract involved services that could
irrevocable offer. not be performed within a year.

(B) Yes, because the owner and the corpora- (C) The granddaughter was caring for her
tion’s president entered into a valid oral brother before the agreement was entered
contract. into.

(C) No, because the corporation’s general (D) The grandmother sold the condominium
counsel never initialed the written agree- before the end of the five years.
ment and there was, therefore, failure of an
express condition.
2. CONTRACTS - DEFENSES AND PAROL EVIDENCE QUESTIONS

Question 3 the retailer on October 26. The president of the


retailer signed the letter on October 27 without
The order department of a machine tools making any changes to it and sent it back to the
manufacturing company received a phone call manufacturer by first class mail. The next day,
from a factory owner who placed an order having not heard from the retailer, the manufac-
for two of the company’s standard “Type-A” turer’s sales manager contacted the president of
machines. The factory owner and the company the retailer and informed him that, if the retailer
came to an oral agreement whereby the total accepted by the end of the month, it would
price for both machines was agreed to be receive a 1% discount on the total price. When
$10,000. The first machine was to be delivered the retailer’s president responded that he had
on May 1, with payment of $5,000 due 30 days already sent the acceptance, the sales manager
after delivery, and the second machine was to be assured him that the discount would still apply.
delivered on June 1 on the same terms (payment On October 31, the retailer received a circular
of $5,000 due 30 days after delivery). Although from a competing source offering comparable
the company did not carry the machine in stock, flash drives for 5% less than the manufacturer’s
no retooling was required because the Type-A price. The retailer immediately faxed a rejection
machine was a standard model. to the manufacturer. The manufacturer received
the rejection immediately but took no action on
The first machine was duly delivered on May it. The next day, November 1, the manufacturer
1. The second machine arrived on June 1, but the received the signed contract from the retailer.
factory owner refused to accept delivery and also
refused to pay for the first machine. What is the status of the parties’ agreement?

The company sued the factory owner on June (A) An enforceable contract was formed for
2. Assume that it cost the company $3,000 to $9,900 because the retailer accepted before
manufacture each Type-A machine, and that October 31 and needed no additional con-
the company could resell the machine for only sideration for the oral modification.
$3,000.
(B) An enforceable contract was formed
What damages should be awarded aside from for $10,000 because the parol evidence
any incidental damages? rule precludes the retailer from offering
evidence of the telephone conversation.
(A) $3,000.
(C) An enforceable contract was formed for
(B) $5,000. $10,000 because, even though the retailer
accepted before October 31, the Statute
(C) $7,000. of Frauds makes the oral modification
between the parties unenforceable.
(D) $10,000.
(D) An enforceable contract was not formed
Question 4 because the retailer’s rejection was received
before its acceptance.
A leading computer supply retailer contacted
a manufacturer of flash drives on October 25
to supplement the supply of flash drives at the
retailer’s area stores. In response, the manufac-
turer offered to supply 50 boxes of flash drives
at $200 a box, for a total price of $10,000,
delivery one week after acceptance, and sent
a letter containing those contractual terms to
CONTRACTS - DEFENSES AND PAROL EVIDENCE QUESTIONS 3.

Question 5 a tiny flaw in the stone that reduced the ring’s


value to $4,800. The jeweler noted that if the
A gambler lived in a state where gambling stone were perfect, the ring would be worth
was illegal. Nevertheless, he gambled on a $6,200. Although the ring’s owner was disap-
regular basis. He asked his friend to lend him pointed, he told his friend that she could have
$5,000 to bet on a football game. His friend it for its appraised value: $4,800. Unbeknownst
agreed to lend him $5,000 if the gambler would to the friend or the ring’s owner, the jeweler
bet half of it on the friend’s behalf. The gambler was suffering from a minor eye infection that
agreed, took his friend’s $5,000, and placed the caused him to find a “flaw” where none, in fact,
bet. The gambler won on 4-to-1 odds. He gave existed. After she purchased the ring from its
his friend his $5,000 back but refused to tender owner for $4,800, the friend sought her own
any winnings. appraisal for insurance purposes. A reputable
jeweler issued the friend an appraisal document
If his friend sues the gambler to recover accurately stating the ring’s value at $6,200. The
the winnings due under the contract, who will ring’s (now-former) owner learned of this and
prevail? demanded return of the ring in exchange for the
money she had paid for it. The friend refused.
(A) His friend, because he fully performed his
part of the bargain. If the original owner of the ring sues his
friend to replevy the ring, will the court rule in
(B) His friend, because the court will not allow his favor?
the gambler to unfairly profit from his
illegal contract. (A) No, because parties to a contract generally
assume the risk of determining value.
(C) The gambler, because the contract was
illegal and the court will not enforce an (B) No, because he should have known the ring
illegal contract. was worth more than $4,800 based on his
conversation with the gemologist.
(D) The gambler, because the contract was
illegal and the court will only act to put the (C) Yes, because both parties mistakenly relied
parties in the status quo ante, and his friend on an inaccurate appraisal in determining
already has his money back. the ring’s value.
Question 6 (D) Yes, because accuracy of the appraisal was
an express condition precedent.
Two years ago, an expert gemologist told
the owner of a sapphire ring that the ring was
probably worth about $6,000, although the
gemologist never gave a formal appraisal. The
ring’s owner now is in need of cash and offered
to sell it to his friend. She liked the ring but had
no idea of its value. She therefore suggested
that the owner take it to a reputable jeweler for
an appraisal, and if the price was not too high,
she would buy it from him. The owner did not
tell his friend about the gemologist’s opinion
because he hoped the ring’s value had increased.

The owner took the ring to a reputable jeweler,


who told the owner in good faith that there was
4. CONTRACTS - DEFENSES AND PAROL EVIDENCE QUESTIONS

Question 7 Question 8

The local school began accepting bids for the A girlfriend and boyfriend decided to get
construction of a new high school. Eager to get married. The girlfriend was one month short
the job, a local builder immediately advertised of her 18th birthday, and the boyfriend was
for bids from subcontractors. The bids from the 19. They went to a jeweler who made wedding
plumbing subcontractors came in at $165,000, rings to order and described what they were
$180,000, $195,000, and $200,000. interested in. The girlfriend and boyfriend then
signed a purchase order for two rings, a woman’s
After compiling all of the bids from the band for $500 and a man’s for $650. After the
subcontractors and calculating his costs based on girlfriend’s 18th birthday, the rings were ready.
the lowest bids, the builder submitted its general The girlfriend went to the jeweler and told him
bid to the school board. Pleasantly surprised by that they would not be needing the rings because
the builder’s low bid, the school board accepted. she and the boyfriend had broken up. When he
After the acceptance, the plumbing subcon- protested that they were custom-made and would
tractor who had submitted the lowest bid realized probably not sell to anyone else, the girlfriend
he had made a mistake in his calculations. His said, “All right, I’ve got $400 in my savings
bid should have been $200,000. The plumbing account. I’ll take my ring, but you’ll have to find
subcontractor immediately notified the builder of my boyfriend about the other one.” The jeweler
his mistake and stated that he was revoking his had the girlfriend sign another purchase order
bid. The builder informed the plumbing subcon- for the woman’s band at $400, payment to be
tractor that it was impossible for him to change made by the end of the month.
his bid, and the builder, having used the bid in
making his own bid, was accepting the $160,000 When the jeweler did not hear from the
bid. girlfriend after another month, he brought
an action for breach of contract against her.
In a suit for rescission of its contract with the Evidence produced at trial established that the
builder, is the subcontractor likely to prevail? market value of the rings was $500 and $650 for
the woman’s and man’s rings, respectively, and
(A) No, because once the school board accept- that the age of majority in the jurisdiction was
ed the builder’s bid, the subcontractor’s bid 18.
could not be changed or revoked for any
kind of mistake. Is the jeweler entitled to recover against the
girlfriend?
(B) No, because nothing indicates that the
builder actually knew or should have (A) Yes, in the amount of $1,150.
known of the mistake involved.
(B) Yes, in the amount of $500.
(C) Yes, because the mistake was part of the
basis of the bargain. (C) Yes, in the amount of $400.

(D) Yes, because the subcontractor revoked his (D) No.


bid before the builder accepted.
CONTRACTS - DEFENSES AND PAROL EVIDENCE QUESTIONS 5.

Question 9 Question 10

A buyer for a toy store emailed a toy distrib- A child lost her lunch on the way to elemen-
utor to place an order for 1,000 units of a video tary school. She walked to the local grocery
game. In the email, the buyer asked the distrib- store and explained her predicament to the
utor if the game was compatible with a certain manager. He told her that if she agreed to pay
popular video gaming system since the games him back the following day, she could have
were being ordered for resale. The distributor’s a pre-packaged lunch. The child accepted
reply said, “That gaming system’s name is listed the manager’s offer, picked up her lunch, and
on the packaging of the game.” Both parties promised to pay the manager the following day.
were unaware that the video game company had The child refuses to pay the next day, and the
just released a new generation of consoles. Upon grocery store sues her for the cost of the lunch.
receiving the shipment, the buyer discovered that
the game does not work with the latest console. If the grocery store prevails in its suit, what is
The buyer is uncertain whether it can re-sell the most likely reason?
all of the games since they only work on older
consoles. (A) A third party witnessed the child’s promise.

Which of the following is the most appro- (B) The child cannot return the lunch, so she
priate remedy for the buyer? cannot return the store to the status quo
ante.
(A) Specific performance of the contract,
requiring the manufacturer to ship video (C) The child needed the food.
games compatible with all generations of
consoles. (D) The contract did not have to be in writing.

(B) Rescission of the contract.

(C) A refund on any games that the buyer is


unable to re-sell.

(D) The difference between the purchase price


in the contract and the value of the games
as received.
6. CONTRACTS - DEFENSES AND PAROL EVIDENCE QUESTIONS

Question 11 Question 12

After negotiations, a painting contractor A 17-year-old boy walked into a medical


agreed with a homeowner to paint the exterior clinic and requested assistance with a deep cut
of her house for $20,000. The painter further he received when he fell off his bike. The doctor
agreed to use only a specific top-quality paint told the boy that if he agreed to work at the
because the homeowner wanted the paint to clinic for 45 hours a week for four weeks, he
last for many years. The painter filled out his would stitch the wound and apply a medicated
standard form contract, but deliberately did bandage. The clinic typically charges $225 for
not include a brand name for the paint that he such treatment. Although it seemed like a lot
would use. After telling the homeowner that of work for $225, the boy needed immediate
the contract contained the terms agreed to, she medical attention, so he accepted the offer and
signed without reading it. On completion of promised to report for work the next day, after
the job, the homeowner saw some empty paint which the doctor treated the boy’s injury. On
cans in the painter’s truck and realized that the his way home from the clinic, the medicated
top-quality paint had not been used. bandage fell into a ditch and was lost for good.
The boy refused to report for work the next day.
If the homeowner sues the paint contractor for
misrepresentation, would any statements made The medical clinic sues the boy to recover the
by the painter before the contract was signed be costs of the medical treatment.
admissible?
If the court rules in favor of the boy, what is
(A) No, they would be barred by the parol evi- the likely reason?
dence rule.
(A) The boy was a minor.
(B) Yes, they would not be barred by the parol
evidence rule. (B) The medicated bandage was destroyed and
thus there was a failure of consideration.
(C) Yes, they would be admitted if the contract
was proved to be an incomplete agreement (C) The contract was unconscionable.
of the parties.
(D) It was impossible for the boy to perform.
(D) No, they would not be admitted because
they occurred prior to the signing of the
contract.
CONTRACTS - DEFENSES AND PAROL EVIDENCE QUESTIONS 7.

Question 13 Question 14

An owner and a contractor executed a contract A gallery owner offered to sell a piano to
providing that the contractor was to construct a an acquaintance for $400. The acquaintance
three-story, castle-like structure on a specified had been to the gallery owner's house a few
location according to plans and specifications weeks prior to the offer and had seen a Stein-
drawn up by an architect. The total contract berg piano in his living room, so she accepted.
price was $900,000. No date was included in Unbeknownst to the acquaintance, the gallery
the contract for completion of the home, but owner also owned a Hairwin piano, which he
the builder was to begin construction one week kept at the gallery, and that was the piano that
after the contract was signed. The day after the he intended to sell. The acquaintance had never
contract was signed by the parties, the state been to the gallery, and the gallery owner was
development commission declared the land aware that the acquaintance had never seen the
encompassing the owner’s lot part of a natural Hairwin.
wilderness area, requiring that all residences
constructed therein be single story and have If the acquaintance sues the gallery owner
plans approved by the development commission. to obtain the Steinberg, which party is likely to
The original plans for the three-story structure prevail?
are totally incompatible with the commission’s
guidelines for residences in a wilderness area. (A) The acquaintance, because the gallery
owner knew of the ambiguity.
Must the builder perform the contract?
(B) The acquaintance, because that is the result
(A) No, the contract is void because of mutual under the objective test.
mistake.
(C) The gallery owner, because there was a
(B) No, and he may recover his lost profits in an mutual mistake.
action against the owner.
(D) The gallery owner, because he subjectively
(C) No, the contractor is discharged of his intended to sell the Hairwin instead of the
obligation because of supervening illegality. Steinberg.

(D) Yes, if the owner and the architect supply


new plans approved by the development
commission.
CONTRACTS - DEFENSES
AND PAROL EVIDENCE
ANSWERS
CONTRACTS - DEFENSES AND PAROL EVIDENCE ANSWERS 1.

CONTRACTS - DEFENSES AND PAROL EVIDENCE ANSWERS

Answer to Question 1

(B) The gourmet food store owner is likely to prevail, because she and the corporation’s presi-
dent formed a valid oral contract when they agreed on all salient points after negotiations. One
element of their agreement was that the terms would be put in writing, a process sometimes called
“memorializing” the agreement. The writing does not constitute the agreement itself, but is merely
a written record of it. Another element of their oral agreement concerned when it would take
effect: on completion of the “memorialization.” The general counsel’s failure to initial the writing,
whether deliberate or inadvertent, was a breach of the oral agreement that he would do so, but
cannot be used to avoid performance of the contract. Note that even though the contract involves
the sale of goods and may be for $500 or more, a violation of the Statute of Frauds does not affect
the contract’s validity, just its enforceability. Here, the memorialization of the contract makes it
enforceable under the Statute (even without his initials). (A) is incorrect because the oral agree-
ment was already in effect at the time the writing was mailed. The processes of offer and accep-
tance took place during the oral negotiations. Even if the mailing of the writing could somehow
be seen as an offer, there are no facts, such as detrimental reliance, that show it to be irrevo-
cable. (C) is incorrect because, as described above, the writing was a memorial of the existing
oral agreement. Even if the requirement of the general counsel’s initialing is characterized as an
express condition, the corporation’s president would not be permitted to prevent the occurrence of
the condition and then claim the benefit of its nonoccurrence. (D) is incorrect because acceptance
had already taken place on the parties’ reaching an oral agreement. This choice appears to invoke
the mailbox rule but gets it wrong, since a mailed acceptance is generally effective on posting
(which occurred prior to the phone call), not on receipt. In any event, the mailbox rule is not
applicable to these facts.

Answer to Question 2

(D) That the grandmother sold the condominium before the end of five years is not relevant to a
decision to deny the granddaughter a remedy. If there were an enforceable contract requiring the
grandmother to turn the condominium over to her granddaughter, and the grandmother sold the
condominium to a bona fide purchaser for value, that would cut off the granddaughter’s right to
specific performance, but she would still be entitled to a remedy. Thus, the sale does not support
a decision to deny the granddaughter a remedy. (A) is a basis for the court’s decision. The Statute
of Frauds makes certain contracts unenforceable unless they are evidenced by a writing signed by
the party to be charged and evidencing the material terms of the contract. Among the contracts
that are within the Statute are contracts for an interest in real property. Here the contract was oral
and it involved a transfer of real property. Thus, it is unenforceable. (B) would support the court’s
decision under a similar rationale. Contracts which by their terms cannot be performed within
one year are within the Statute of Frauds and are unenforceable without a writing, and, again, the
contract here was oral and required five years of performance. (C) would also support the court’s
decision. Where an oral promise is not enforceable under contract law because of the Statute of
Frauds and the promisor should foresee that it will induce the promisee to change position in
reliance on the oral promise, promissory estoppel may be used to take the contract out of the
Statute of Frauds entirely. If the court finds that the grandmother’s promise did not induce the
granddaughter to care for her brother (because she was already doing so before the promise was
made and after the promise was broken), then the promise is not enforceable under the doctrine of
promissory estoppel.
2. CONTRACTS - DEFENSES AND PAROL EVIDENCE ANSWERS

Answer to Question 3

(B) The company should recover $5,000 because the oral contract between it and the factory owner
is enforceable to the extent the factory owner received and accepted the goods. A promise for
the sale of goods of $500 or more is not enforceable unless evidenced by a writing signed by the
party to be charged. [UCC §2-201(1)] However, an oral contract for such goods is enforceable to
the extent of goods received and accepted by the buyer. [UCC §2-201(3)(c)] Oral contracts for
specially manufactured goods not suitable for sale in the ordinary course of the seller’s business
also are enforceable when the seller has begun substantially to perform. Here, the parties’ agree-
ment was oral. The factory owner accepted one machine, but neither machine was specially
manufactured. He is bound to pay the $5,000 contract price for the accepted machine but is not
bound to pay for the rejected machine. (A) is incorrect because the company is entitled to the
contract price for the machine the factory owner accepted, not just restitution. If a contract is
unenforceable because of noncompliance with the Statute of Frauds, a party can generally sue
for the restitution of any benefit that has been conferred. However, as discussed above, an oral
contract for the sale of goods of $500 or more is enforceable to the extent of goods received and
accepted by the buyer. Therefore, because the factory owner accepted one machine, the company
is entitled to the $5,000 contract price of that machine, not just $3,000 in restitutionary damages.
(C) is incorrect because the company is not entitled to any damages as to the rejected machine.
As indicated above, the company is entitled to $5,000 for the first machine. There is no enforce-
able contract regarding the second machine, and the company is not entitled to damages for that
machine. If there were an enforceable contract for the second machine, (C) would state a proper
measure of damages—if a buyer breaches by refusing to accept goods, the seller is entitled to
recover the difference between the contract price ($5,000) and the market or resale price ($3,000),
here, $2,000. Thus, the company would be entitled to $5,000 for the accepted machine and
$2,000 for the rejected machine, or $7,000. (D) is incorrect for the same reason that (C) is incor-
rect—the company is only entitled to the contract price of the machine accepted by the factory
owner. If the contract for the second machine were enforceable, (D) would still not be a proper
measure of damages. Under the UCC, the seller has a right to force goods on a buyer who has
not accepted them only if the seller is unable to resell the goods or if the goods have been lost or
damaged after the risk of loss passed to the buyer. [UCC §2-709] Because the company can resell
the second machine, which has not been lost or damaged, it could not recover the full price of the
second machine from the factory owner even if the contract was fully enforceable.

Answer to Question 4

(C) A contract was formed for $10,000 because the oral modification is unenforceable under the
Statute of Frauds. Under the UCC, a contract for the sale of goods priced at $500 or more is not
enforceable unless evidenced by a writing. Furthermore, contract modifications must also meet
the Statute of Frauds requirement if the contract as modified is within the Statute’s provisions.
Here, the Statute of Frauds is applicable to both the original contract and the contract as modified.
Since the modification was not in writing, it is not enforceable; hence, the terms of the original
contract, which satisfies Statute of Frauds requirements, are effective. (A) is wrong even though
the retailer did accept before October 31 and no additional consideration was needed for the oral
modification. Because the Statute of Frauds is applicable to the modification, the 1% discount
is not effective. (B) is wrong because the parol evidence rule is applicable only to oral expres-
sions made prior to or contemporaneous with the written contract; parol evidence can be offered
to show subsequent modifications of a written contract. Since the contract was created when
the retailer dispatched the letter (as discussed below), the agreement regarding the 1% discount
is a subsequent modification and therefore unaffected by the parol evidence rule. (D) is wrong
CONTRACTS - DEFENSES AND PAROL EVIDENCE ANSWERS 3.

because the mailbox rule is applicable to the facts here; the acceptance was effective on dispatch.
If the retailer had sent its rejection before its acceptance, the mailbox rule would not apply and the
rejection would take effect because it arrived before the acceptance. However, since the retailer
sent its acceptance first and then its rejection, and the manufacturer did not change its position in
reliance on the rejection, the mailbox rule is applicable and the acceptance was effective when it
was sent.

Answer to Question 5

(C) The gambler will prevail. The general rule is that a court will not enforce a contract if its subject
matter or consideration is illegal; the court will leave the parties as it finds them. Here, the subject
matter of the contract, placing gambling bets, is illegal in the state. Thus, (A) is wrong because his
friend’s performance is irrelevant. (B) is wrong because the court will refuse to help either party
to an illegal contract, even where one party has gained unfairly. (D) is wrong because the court
will not put the parties back into the position they were in prior to entering into the contract, but
rather will leave them where they stand.

Answer to Question 6

(C) The court should rescind the contract for mutual mistake and order the friend to return the ring to
its rightful owner. Mutual mistake of an existing material fact to a contract is a defense that allows
the adversely affected party to rescind as long as that party did not assume the risk of mistake.
Mutual mistake arises where both parties are mistaken as to an existing fact relating to a basic
assumption of the contract and the mistake has a material effect on the agreed-upon exchange.
Here, both parties were mistakenly operating under the assumption that the ring was worth only
$4,800 when in fact it was worth $6,200. While generally courts will not allow mistakes in value
to be the basis for rescission, because the courts will presume that both parties assumed the risk
of mistake, the facts here are sufficient to take this case out of the general rule—it appears that
neither party wanted to assume the risk of determining value, since both agreed to rely solely on
the valuation by an independent expert. (A) is incorrect because it does not take into account that
the facts here are sufficient to show that neither party assumed the risk of mistake, and so the
general rule does not apply. (B) is incorrect because the statement by the gemologist two years
ago does not indicate the value of the ring today, both because market conditions could have
changed in two years and because the gemologist gave a fairly equivocal and informal valuation.
(D) is incorrect because the appraisal was not a condition precedent—an event creating, limiting,
or extinguishing the absolute duty to perform—because when the friend requested the appraisal,
she did not obligate herself to buy the ring based on the appraisal. Her promise at that time—that
if the price was not too high, she would buy it from him—was too illusory to form a contract,
because what is “too high” is very subjective. Because there was no contract, the appraisal could
not be a condition.

Answer to Question 7

(B) The subcontractor is not likely to prevail because nothing indicates that the builder actually knew
or should have known of the computation error. Where only one of the parties entering into a
contract is mistaken about facts relating to the agreement, this unilateral mistake will not prevent
formation of a contract unless the nonmistaken party is or had reason to be aware of the mistake
made by the other party. Here, the computation error was the contractor’s mistake alone, and
the contract may not be rescinded unless the builder actually knew or should have known of the
mistake. While such errors may be canceled in equity if the nonmistaken party has not relied on
4. CONTRACTS - DEFENSES AND PAROL EVIDENCE ANSWERS

the contract, the builder clearly relied upon the bid when it submitted its own bid to the school
board. (A) is incorrect because the subcontractor would be able to avoid the contract if there were
either a mutual mistake or unilateral mistake of which the builder was aware. Generally, if a low
bid is significantly lower than all the other bids, the contractor has reason to know that a mistake
was made. Here, the plumbing subcontractor’s bid is not so much lower than the other bids that the
contractor should have known of the mistake. Thus, the contract cannot be avoided for unilateral
mistake. (C) is incorrect because the fact that the subcontractor’s error was part of the basis of
the bargain is not sufficient alone to avoid the contract. When both parties entering into a contract
are mistaken about existing facts relating to the agreement, the contract may be voidable by the
adversely affected party if, among other things, the mutual mistake concerns a basic assump-
tion on which the contract is made. Here, the computation error was a unilateral, not a mutual,
mistake. While there is modern authority indicating that a unilateral mistake that is so extreme
that it outweighs the other party’s expectations under the agreement will be a ground for cancel-
lation of the contract in equity, the error here is not sufficiently extreme to justify cancellation.
(D) is incorrect because in cases where the offeror would reasonably contemplate reliance by the
offeree in using the offer before it is accepted (as is the case with subcontracting bids), the offer
will be held irrevocable as an option contract for a reasonable length of time. The subcontractor
knew that the builder was relying on the subcontractor’s bid to make the bid to the school board,
and the builder did in fact rely on it. Thus, the subcontractor’s bid was irrevocable for a reason-
able time. Therefore, the subcontractor’s attempt to revoke the bid before the builder accepted was
ineffective.

Answer to Question 8

(C) The jeweler is entitled to recover $400 because, on attaining the age of majority, the girlfriend
affirmed the contract as to her ring at that price. Infants generally lack legal capacity to incur
binding contractual obligations. Contracts entered into by infants are voidable at this election
unless the contract is for a “necessity.” On reaching majority, an infant may affirm; that is, choose
to be bound by her contract. Here, at the time of entering into the contract for the two rings, the
girlfriend was not yet 18 years old (the applicable age of majority). Because a wedding ring is
not a necessity, the girlfriend’s contract was voidable at her option. However, when the girlfriend
went back to see the jeweler, she had reached the age of 18. At that time, she chose to be bound
by the contract, although her affirmation was limited to her ring (instead of both rings), and to
a lower price for her ring than that which was originally agreed upon. When a voidable promise
is reaffirmed, the promise will be enforced according to the terms of the reaffirmation rather
than the original obligation. Thus, the girlfriend is now bound by her contract, but only for the
purchase of her ring at a price of $400. (D) is incorrect because it ignores the fact that, having
affirmed the contract (albeit on different terms), the girlfriend, who is no longer an infant, is
bound to the terms of the reaffirmed contract. (A) is incorrect because $1,150 reflects the purchase
price of both rings as contained in the original contract. On reaching majority, the girlfriend
chose not to be bound by the original contract. Similarly, (B) is incorrect because $500 reflects
the purchase price of the girlfriend’s ring as contained in the original agreement (as well as the
market value of the ring). On reaching majority, the girlfriend could have refused to pay anything.
She is now bound only to the contract as affirmed by her, which is for $400.

Answer to Question 9

(B) The buyer is entitled to rescission of the contract on the ground of mutual mistake. Rescission
is a remedy that discharges the contractual duties of the parties and puts an end to the transac-
tion, leaving the parties as though the contract had never been made. Unilateral rescission results
CONTRACTS - DEFENSES AND PAROL EVIDENCE ANSWERS 5.

where one of the parties desires to rescind but the other party refuses to agree to a rescission. For
unilateral rescission to be granted, the party desiring rescission must have adequate legal grounds.
One ground for unilateral rescission is mutual mistake of a material fact. If the distributor reason-
ably believed that the games were compatible with the named gaming system when shipped to
the buyer, then both parties were operating under a mistake of fact. This mistake concerns a
basic assumption on which the contract is made, because the compatibility of the game with that
gaming system was a fact without which the buyer would not have entered into the transaction.
The mistake creates a material imbalance in the agreed exchange, and the party seeking avoid-
ance (the buyer) did not assume the risk of the mistake because she asked if the games were
compatible at the time of placing the order. Because of this mutual mistake of material fact,
which existed at the time the contract was entered into, the buyer is entitled to rescission of the
contract. (C) is not a standard remedy for a breach of contract. (D) only offers the buyer monetary
damages, which may be minimal. The value of the game for resale may be the same provided that
the buyer is able to re-sell the game to a consumer, but here the buyer is uncertain whether it can
re-sell this quantity of games. (A) is incorrect because a buyer has a right to specific performance
of a sales contract only where the goods are unique or in other proper circumstances. [UCC
§2-716] There is no indication that this video game is unique. Thus, a contract for the sale of such
goods is not an appropriate subject for specific performance.

Answer to Question 10

(C) The grocery store can recover the cost of the lunch from the child because food is a necessity,
and the child told the store manager that she had lost her lunch and needed the food. Although
an elementary-school-age child is a minor (the age of majority in most jurisdictions is 18), and
minors lack the legal capacity to contract, most states hold that a minor is bound to pay the
reasonable value of necessities. Food being a necessity, the child would be bound to repay the
grocery store for the lunch. (A) is wrong because the presence of a third party is not essential to
enforce the contract. (B) is irrelevant. (D) is wrong because even though it is true that the contract
did not have to be evidenced by a writing signed by the party to be charged to be valid—because
the contract was not for the sale of goods valued at $500 or more—that would not be the reason
the store could recover. Choice (D) does not address the child’s minority, the issue on which this
question turns.

Answer to Question 11

(B) Statements made by the painting contractor before the contract was signed would not be barred
by the parol evidence rule. This rule provides that a writing that constitutes a complete and final
expression of the bargain may not be varied by prior written or oral statements or contempora-
neous oral statements. However, the parol evidence rule does not bar prior statements when the
cause of action is for misrepresentation, which is attacking the validity of the agreement. Thus,
(A) and (D) are wrong. While (C) is true, it is not as good an answer as (B) because the state-
ments could be admitted even if the contract was not proved to be an incomplete agreement.

Answer to Question 12

(C) If the court rules in favor of the boy, it will be because the parties’ contract was unconscionable.
Unconscionability usually arises where one of the parties has substantially superior bargaining
power and can dictate the terms of the contract to the other party, who has inferior bargaining
power. Here, the boy told the doctor that he needed medical attention and the doctor forced him to
agree to work for, in essence, $1.25 per hour in exchange for care and a medicated bandage. These
6. CONTRACTS - DEFENSES AND PAROL EVIDENCE ANSWERS

facts would meet the requirements of an unconscionable contract. (A) is not a defense because,
although the boy was a minor (the age of majority in most jurisdictions is 18), the contract was
for necessities (i.e., medical care). Despite the general rule that minors lack the legal capacity to
contract and will not be bound by contracts they make, they will be held liable in restitution for
the benefits they receive under contracts they make for necessities, such as medical care. Thus,
the medical clinic could hold the boy liable for the reasonable value of the medical treatment
provided. (B) is wrong because the boy did in fact receive the medical care and the medicated
bandage and, therefore, there was consideration for his promise to perform. (D) is wrong because
nothing in the facts indicates that performance is impossible.

Answer to Question 13

(C) Performance is excused where prevented by operation of law, despite any stipulation to the
contrary. Because governmental interference made performance of the contract as contemplated
illegal, the contractor is excused from performance, even though some performance is possible.
Thus, (C) is correct, and (D) is incorrect. (A) is incorrect because there was no mutual mistake
when the contract was formed. (B) is wrong because the owner did not breach the contract.

Answer to Question 14

(A) The acquaintance will likely prevail because the gallery owner knew of the ambiguity. Contract
language with more than one possible meaning leads to different results depending on the aware-
ness of the parties. If one party was aware of the ambiguity and the other party was not at the
time of contracting, a contract will be enforced according to the intention of the party who was
unaware of the ambiguity. Here, the gallery owner knew he had two pianos, and the acquain-
tance did not. Thus, there is a contract on the terms as understood by the acquaintance; there is a
contract for the Steinberg. (B) is wrong because in the ambiguity situation, the court looks to the
subjective intention of the parties. The objective intent of the parties cannot be determined from
the contract in the case of ambiguity. (C) is wrong because mutual mistake requires that both
parties are mistaken about a basic assumption of the contract. Here, the gallery owner was never
mistaken. He knew which piano he was selling. (D) is a misstatement of the law. While it is true
that the subjective intent of the parties is used in the case of ambiguity, when only one party was
aware of the ambiguity, the other party's intent controls. Here, the acquaintance was unaware of
the ambiguity, so her intent controls.
CONTRACTS - OFFER
AND ACCEPTANCE
QUESTIONS
CONTRACTS - OFFER AND ACCEPTANCE QUESTIONS 1.

CONTRACTS - OFFER AND ACCEPTANCE QUESTIONS

Question 1 Question 2

A contractor with a contract to deepen a well On October 1, a siding installer sent a letter
in a drought-stricken area mistakenly entered to a builder stating, “I am offering to install
onto the wrong property and proceeded to siding on one or up to all of the 12 houses that
deepen the well there. The owner of the property you are currently building. This offer remains
saw the contractor at work but said nothing. open until November 1.” On October 13, the
When the contractor completed the job, the builder e-mailed the installer, “We accept your
property owner refused to pay his bill, and the offer with respect to the house on Main Street.”
contractor filed suit. In her answer, the property There was only one house on Main Street that
owner stated that she thought the contractor was the builder was constructing, so the installer
employed by the county and that the government promptly began the siding installation. On
was paying for the work because of the drought. October 25, the installer telephoned the builder
She knew, however, that two of her neighbors to inform him that she had accepted other work
had recently paid private contractors to deepen and so would not be able to install siding on the
their wells. builder’s other houses. The builder thereafter
faxed the installer, “We would like you to do the
Which of the following arguments offers siding on the other houses.”
the contractor his best chance for winning his
lawsuit? In an action by the builder against the installer
for breach of contract, will the builder succeed?
(A) Implied-in-fact contract.
(A) Yes, because the installer had promised that
(B) Promissory estoppel. the offer would remain open until Novem-
ber 1.
(C) Mutual mistake.
(B) Yes, because the installer’s attempted
(D) Unilateral mistake. revocation was by telephone.

(C) No, because the builder’s power of accep-


tance was terminated by the installer’s
contract with another party.

(D) No, because the builder’s power of accep-


tance was terminated by effective revoca-
tion.
一 信:liuxue119118 , 们 信免 供

2. CONTRACTS - OFFER AND ACCEPTANCE QUESTIONS

Question 3 Question 4

A retail store ordered 100 women’s swimsuits A landlord heard that a company was looking
at $10 each, as advertised in the catalog of a for old buildings to purchase and raze to make
swimwear manufacturer. The manufacturer room for new development. On July 1, the
shipped 40 swimsuits to the store, along with a landlord sent a letter to the company stating, “I
letter stating in relevant part: “We have shipped own two adjacent apartment buildings that might
you 40 swimsuits at $10 each in response to your meet your needs. One is located on 123 Main
recent order. Please remit $400. Be informed Street and the other property is right behind it on
that limited inventory will prevent us from 123 Wood Street. I will sell you one or both of
being able to ship any additional suits at this the apartment buildings for $250,000 each. This
time or at any time during this year’s beach offer remains open until August 1.” On July 15,
season.” The store took the 40 suits and began the company faxed the landlord, “Accept your
to sell them. The store immediately sought an offer with respect to the apartment building on
alternate supplier of swimsuits. The best price Main Street.” On July 18, the company faxed the
it could obtain was $11 per suit from a different landlord, “Will also take the building on Wood
company. The store ordered, received, accepted, Street.” On July 22, the landlord discovered that
and paid for 60 suits at $11 each from the other he did not have good title to the Wood Street
company. The store has refused to pay the property.
manufacturer the $400, and the manufacturer
has sued for payment. Which of the following would provide the best
legal support to the landlord’s contention that
What is the manufacturer entitled to recover? he was not liable for breach of contract as to the
Wood Street property?
(A) $400, the contract price for the 40 suits
delivered by the manufacturer and accepted (A) Impossibility of performance.
by the store.
(B) Unilateral mistake as to basic assumption.
(B) $340, the contract price for the 40 suits, less
the store’s cover damages. (C) Termination of the offer by the company’s
having first contracted to buy the Main
(C) The fair market value of the 40 suits, less Street property.
the $60 cover damages.
(D) Excuse by failure of an implied condition
(D) Nothing, because it failed to fully perform precedent.
the contract.

一 信:liuxue119118 , 们 信免 供
CONTRACTS - OFFER AND ACCEPTANCE QUESTIONS 3.

Question 5 games that you wanted for your business. I can


guarantee delivery until September 2 at $2,000
On August 1, a realtor mailed a written offer per machine.” On August 16, the distributor
to a developer for the sale of a large tract of land. sold the 20 electronic poker games to an arcade
The offer included the following terms: owner at a price of $2,300 per machine. On
August 17, the distributor faxed the retail seller:
This offer expires on September 1, if the “Am revoking offer regarding poker machines;
offeree has not caused an acceptance to be have already sold same at higher price.” The
received by the offeror on or before that date. retail seller received the fax the same day.
Early on September 1, the developer sent During the month of August, a special session
a written acceptance by messenger but the of the state legislature passed a bill declaring
messenger company negligently withheld the sale of electronic gambling machines to be
delivery to the realtor until September 2. On illegal in the state. The definition of electronic
September 4, the realtor entered into a contract gambling machines included the electronic
for sale of the tract to another buyer but did not poker games distributed by the distributor. The
inform the developer of the transaction. When governor signed the bill into law on August 28,
the developer contacted the realtor a few days and it became effective immediately. On August
later, the realtor said there was no contract 30, the retail seller faxed the distributor: “Accept
between them. your offer to sell me 20 poker games at $2,000
per machine.” The distributor received the fax
What is the most accurate statement regarding the same day.
the relationship between the parties?
If the distributor is not obliged to sell 20
(A) No contract between the realtor and the electronic poker machines to the retail seller at
developer arose on September 2. $2,000 each, what is the most likely reason?
(B) A contract would have arisen if a letter of (A) The distributor had already sold the ma-
acceptance were mailed on September 1. chines at a higher price.
(C) The realtor’s silence constituted an accep- (B) The state law banning the sale of such
tance of the developer’s written message on machines operated to revoke the offer.
September 2.
(C) The distributor revoked his offer before the
(D) A voidable contract arose on September 1. retail seller accepted.
Question 6 (D) The state law banning the sale of such
machines renders the performance of the
A wholesale distributor of electronic poker distributor’s duties impossible.
games served the entire state in which it had its
headquarters. There were no plants manufac-
turing the types of machines that the distrib-
utor sold within 1,000 miles of the state, and
electronic poker games were in scarce supply.
A retail seller sold mechanical and electronic
games to taverns and convenience stores in the
area. On August 3, the distributor sent a letter to
the retail seller, which the retail seller received
on August 5. The letter stated, in relevant part:
“I now have in stock those 20 electronic poker
4. CONTRACTS - OFFER AND ACCEPTANCE QUESTIONS

Question 7 (D) The homeowner, because she did not accept


the gardener’s offer to do the landscaping
A homeowner offered a landscape gardener job for $1,000.
$1,000 to trim and reshape the bushes on her
property if the gardener could finish the job Question 8
before her garden party on June 1. The gardener
told the homeowner that he would get back to The inventor of a laser-beam vegetable
her after he had checked his calendar. The next chopper ran a television ad that described the
day, the gardener phoned the homeowner, who chopper and said, “This chopper is yours for
was not at home, and left a message on her only $49.99 if you send your check or money
voicemail that he had the time, but could not do order” to a specified post office box. The owner
the job for less than $1,200. The gardener did of a retail specialty shop wrote to the inventor,
not hear from the homeowner for several days. “What’s your best, firm price for two dozen
choppers?” The inventor sent a written reply that
As June 1 drew closer, the gardener phoned said in its entirety, “We quote you for prompt
the homeowner again and left another message acceptance $39.99 per unit.” The shop owner
on her voicemail stating that “I’ll do the job subsequently mailed a check to the inventor in
for $1,000, this weekend, unless that would be the appropriate amount, with a memo enclosed
inconvenient.” The homeowner replayed the saying, “I accept your offer for 24 choppers.”
second message just as she was leaving town on
a business trip and did not contact the gardener. For a contract to arise from these communica-
That weekend, unbeknownst to the homeowner, tions, which of the following must be true?
the gardener took his tools to the homeowner’s
house and trimmed and reshaped the bushes (A) Both parties were merchants.
to the homeowner’s specifications. When the
homeowner returned from her trip several (B) The inventor had at least 24 choppers in
days later, the gardener presented her with a stock when the shop owner’s check and
handwritten invoice for $1,000. memo were received.

If the homeowner refuses to pay the gardener, (C) The shop owner’s check and memo were
and the latter brings an action solely for breach mailed within three months after his receipt
of contract to recover the $1,000 contract of the inventor’s letter.
amount, who will likely prevail?
(D) The shop owner’s check and memo were
(A) The gardener, because the homeowner mailed within a reasonable time after his
knew of the gardener’s plans to do the receipt of the inventor’s letter.
landscaping job over the weekend in ques-
tion, putting the burden on the homeowner
to call off the job if she did not want the
gardener to perform.

(B) The gardener, because this was a unilateral


contract, the terms of which the gardener
accepted by performing his duties under the
contract.

(C) The homeowner, because she revoked


her offer when she ignored the gardener’s
subsequent phone calls agreeing to do the
job at the original price.
CONTRACTS - OFFER AND ACCEPTANCE QUESTIONS 5.

Question 9 Question 10

A store selling secondhand office equipment The owner of a mint-condition classic car
advertised used desktop computers for sale wrote a letter to his trusted car mechanic
at its store, for $200 each. The administrator offering to sell him the car for $45,000, if he
of a college dormitory that needed additional bought it before April 15.
computers for its study area mailed the store a
check for $1,000 with a note ordering five used The mechanic researched the current
desktop computers, as advertised. market value of the car online and discovered
that comparable vehicles were being sold for
Upon checking his stock, the store owner $48,000. On April 1, he was just leaving his
found that he had only two desktop computers home to drive to the car owner’s house to give
left, but he had plenty of comparable laptops, him a check for $45,000 when he received a text
and therefore he sent the administrator two from the owner stating that he had changed his
desktop computers and three laptops by common mind and the car was no longer for sale. The
carrier. mechanic drove to the owner’s house anyway,
where the car was parked out front with a “for
What was the legal effect of the store owner’s sale” sign in its window. The mechanic knocked
shipment of the two desktops and three laptops on the owner’s door and, when he answered,
in response to the administrator’s order for five tendered the $45,000 certified check and
desktops? demanded the car. The owner refused.

(A) An acceptance of the administrator’s offer If the mechanic brings an action seeking
with respect to the two desktop computers, damages for breach of contract against the car
and a counteroffer as to the three laptops. owner, what is his likely recovery?

(B) An acceptance of the administrator’s offer (A) Nothing, because the offer to sell the car
to purchase five desktop computers, and a was withdrawn before the mechanic ac-
breach of the contract formed thereby. cepted.

(C) Neither an acceptance nor a counteroffer, (B) $45,000, because the car owner has failed
because a mere shipment of goods is not to perform under the contract of sale.
a manifestation of assent to any particular
terms. (C) $3,000, because his tender of the purchase
price was an acceptance of the car owner’s
(D) A mere offer to furnish substituted goods as offer.
an accommodation.
(D) $3,000, because the car owner’s letter
created an enforceable option.
6. CONTRACTS - OFFER AND ACCEPTANCE QUESTIONS

Question 11 Is the advertising agency correct in its asser-


tions?
For a number of years, a leasing company has
been in charge of leasing the luxury skyboxes (A) Yes, because the leasing company’s fail-
at a local basketball stadium. During this time, ure to reject the advertising agency’s offer
it annually sent area businesses personalized within a reasonable time constituted an
“invitations” to lease skyboxes for the season. The acceptance under the circumstances.
invitations, which were always sent out several
months before each season began, contained (B) Yes, because the leasing company’s “invita-
detailed price terms and language stating that the tion” to the advertising agency was an offer,
deadline for responding was 10 weeks before the which the agency accepted.
start of the season and that all leases were subject
to the approval of the management of the leasing (C) No, because the leasing company was
company. A local advertising agency had always entitled to reject the agency’s offer when it
responded to their invitation immediately by did.
registered mail because they found it very worth-
while to lease a skybox to entertain their clients. (D) No, but the advertising agency would be
During the five years that they had responded able to recover reliance damages from the
affirmatively to the invitation, they never received leasing company under a quasi-contract
any additional communications from the leasing theory.
company regarding approval, but the tickets and
an invoice would arrive about a week before the
season began.

Several months before the current season, the


advertising agency received and immediately
responded to its invitation. Two weeks before
the season began, a stunning trade brought the
league’s most popular star to the city’s basket-
ball team, prompting a dramatic increase in the
demand for tickets. A few days later, the adver-
tising agency, which had already scheduled in a
number of clients to attend games in its skybox,
received a notice from the leasing company
stating that management had not approved the
agency’s lease of the skybox for this season. In
a separate announcement to all area businesses,
the leasing company announced that all avail-
able skyboxes would be leased for three- or
five-year terms, and that an auction of the leases
would be conducted if the demand exceeded the
supply. The advertising agency decided that it
was not financially feasible to commit itself to
anything longer than a one-year lease. It sent
a letter to the leasing company, stating that a
contract was created between the parties and
that the leasing company will be in breach if it
does not perform.
CONTRACTS - OFFER AND ACCEPTANCE QUESTIONS 7.

Question 12 Question 13

A general contractor who wished to bid on a A sporting goods retailer whose tent stock was
construction project solicited bids from a variety running low saw a listing for the tent she wanted
of subcontractors. Four electrical subcontractors priced at $90 in the catalog of a large camping
submitted bids to the contractor in the amounts goods manufacturer. The retailer phoned the
of $75,000, $85,000, $90,000, and $95,000, manufacturer and placed her order for 10 tents
respectively. on May 1. The next day, the manufacturer
mailed the retailer a letter informing her that
As he was making out his company’s bid, the tents were now $92 and that they would be
which was higher than he wanted it to be, the shipped to her on May 16. The retailer received
contractor called the low bidder on the electrical the letter on May 4, but never responded. On
work and told him, “We won’t be able to do it May 15, the retailer received a catalog from
with your present bid, but if you can shave off another company showing tents similar to the
$5,000, I’m sure that the numbers will be there ones that she ordered, but for a cost of $70.
for us to get that project.” The low bidder told the She immediately called the manufacturer to
contractor that he could not lower his bid, adding cancel her order. Nevertheless, the manufacturer
that the bid he submitted was based on a $15,000 shipped the tents to the retailer on May 16.
error, and he could not do the job for less than
$90,000. The contractor lost the construction job Assuming that the parties’ communications
and subsequently sued the low bidder. were sufficient to form a contract, on what day
was the contract formed?
For what is the low bidder is liable?
(A) May 1, the day the retailer placed her order.
(A) Breach of contract, because the mistake
was not so unreasonably obvious as to (B) May 2, the day the manufacturer sent its
make acceptance of his bid unconscio- letter.
nable.
(C) May 4, the day the retailer received the
(B) Breach of contract, because the mistake letter.
was unilateral.
(D) May 16, the day the tents were shipped.
(C) Nothing, because the low bidder rejected
the contractor’s counteroffer.

(D) Nothing, because even though the low bidder


lacked authority to renege on its bid, the
contractor suffered no damages because
no bidder was willing to do the work for
$70,000.
8. CONTRACTS - OFFER AND ACCEPTANCE QUESTIONS

Question 14 Question 15

To encourage her neighbor to continue On November 3, an investor who had watched


pursuing her avocation of photography, an art his stocks sink to unprecedented low levels sent
patron promised to buy her first photographic a fax message to a dealer in precious metals:
work that wins a prize in a juried art show. To “Please quote your best price on 800 troy ounces
show that she was serious, the art patron gave platinum bars for immediate delivery at my
her neighbor a contract to sign which stated that bank.” At 10 a.m. the next morning (November
she agreed to buy and the neighbor agreed to 4) the dealer replied by fax, “My best price
sell the neighbor’s first framed photograph that is $475 per ounce.” The investor received the
wins a ribbon (first, second, or third prize) in dealer’s message later on that same day.
a juried art show, with payment due within 14
days of delivery. Price was not mentioned in What is the best characterization of the
the contract, which was enforceable in all other communications between the investor and the
respects. Both parties signed the contract. dealer?

Some months later, the neighbor won first (A) An offer and an acceptance.
prize for one of her photos set in an expensive
frame. The parties made arrangements for the (B) A request for an offer and an offer.
art patron to take possession of the piece. The
neighbor insisted on a price of $1,200, pointing (C) An offer and a price quotation.
to the expensive frame and the prestige of the
art show that awarded the prize. The art patron (D) A request for an offer and a price quotation.
instead tendered $700, a good faith approxima-
tion of the value of the photo display, and sued
when the neighbor refused to deliver it.

If the neighbor defends on the ground that no


price was fixed in the contract and the parties
have been unable to agree on a price, how should
the court rule?

(A) The art patron is entitled to purchase the


photograph at whatever price the court
determines to be reasonable.

(B) An arbitrator must be appointed to set the


price.

(C) The art patron is entitled to purchase the


photograph for $700.

(D) The art patron will have to pay $1,200 for


the photograph.
CONTRACTS - OFFER AND ACCEPTANCE QUESTIONS 9.

Question 16

Shortly after a series of rapes took place


within a city, the city council approved the
offering of a $25,000 reward for the arrest and
conviction of the perpetrator of the rapes. Infor-
mation concerning the reward was published in
the local newspaper.

In which of the following ways could the city’s


reward offer be effectively accepted?

(A) Only by an offeree’s making the arrest and


assisting in the successful conviction of a
rapist within the scope of the offer.

(B) Only by an offeree’s return promise to


make a reasonable effort to bring about the
arrest and conviction of a rapist within the
scope of the offer.

(C) By an offeree’s communication of


assent through the same medium (local
newspaper) used by the city in making its
offer.

(D) By an offeree’s supplying information


leading to arrest and conviction of a rapist
within the scope of the offer.
CONTRACTS - OFFER
AND ACCEPTANCE
ANSWERS
CONTRACTS - OFFER AND ACCEPTANCE ANSWERS 1.

CONTRACTS - OFFER AND ACCEPTANCE ANSWERS

Answer to Question 1

(A) The contractor’s best (and only) argument would be that the property owner’s silence while the
contractor deepened her well was an acceptance by silence of an implied-in-fact contract. An
implied-in-fact contract is formed by manifestations of assent other than oral or written language,
i.e., by conduct. Where a person knowingly accepts offered benefits, such conduct, viewed objec-
tively, may be said to manifest an agreement to the conferral of such benefits, resulting in a
contract implied in fact. While generally an acceptance must be communicated to an offeror to be
effective, courts will often find an acceptance where an offeree silently accepts offered benefits.
Here, the property owner’s purported belief that the work was being done by a county employee
at no charge is not plausible. The property owner had no prior notification from or contact with
any county employee, and it is not reasonable to believe that anyone, including an employee of
the government, would enter upon and disturb private property without prior consent. Moreover,
the property owner knew that her neighbors had not had their wells deepened by county workers
but had paid private contractors to do the work. Thus, the facts strongly suggest that the property
owner’s silence as she watched the contractor deepen her well was acceptance by silence of an
implied-in-fact contract. (B) is incorrect because promissory estoppel is inappropriate in this
case. Under the doctrine of promissory estoppel, as outlined in section 90 of the First Restate-
ment, a promise is enforceable to the extent necessary to prevent injustice if the promisor would
reasonably expect to induce action or forbearance of a definite and substantial character and
such action or forbearance is in fact induced. Here, the contractor and the property owner whose
well he deepened had never met or negotiated for services and had no communications with one
another prior to the contractor sending the property owner his bill, and the contractor did not even
realize that he was doing the work for that property owner’s benefit. Thus, promissory estoppel
is not proper. (C) and (D) are incorrect because mistake is a defense to formation of a contract
and is raised to render a contract voidable by the adversely affected party. Thus, mistake would
not be a ground on which relief could be granted. Even if the property owner’s purported mistake
were treated as credible by the court, there was no “mutual” mistake because the contractor and
the property owner were not mistaken about the same fact, but rather, each was mistaken about
a different fact (the contractor, that he was at the right address, and the property owner, that the
contractor was a county employee doing the work without charge). Nor could the facts be charac-
terized as a unilateral mistake, in which only one of the parties is mistaken about facts relating to
the agreement, because both parties were operating under a mistaken belief. Thus, both (C) and
(D) are incorrect.

Answer to Question 2

(D) The builder probably will not succeed because the builder was notified of the installer’s other
jobs. Notice to an offeree that the offeror has made an inconsistent contract with a third party
operates as a revocation of the offer. (A) is incorrect because there was apparently no consider-
ation given by the builder for the installer’s promise to keep the offer open until November 1.
Because this is considered a contract for services rather than goods, the UCC rules on merchant’s
firm offer do not apply. (C) is not as good an answer as (D) because it is the notice to the offeree
that constitutes the revocation (rather than the inconsistent contract with the third party). (B) is
incorrect because it is irrelevant that a telephone was used; an offer may be revoked by any means
informing the offeree that it has been revoked.
2. CONTRACTS - OFFER AND ACCEPTANCE ANSWERS

Answer to Question 3

(B) The manufacturer is entitled to $340. As a general rule, advertisements, catalogs, and other price
quotations are construed as invitations for offers rather than offers. If the language of the catalog
could be construed as a definite promise to specific offerees, a court might construe it as an offer,
but there is no evidence in this question for such an interpretation. The store’s order is therefore
an offer to purchase 100 swimsuits. Because goods are involved, UCC Article 2 applies. Under
UCC section 2-206(1)(b), an offer to buy goods for prompt shipment invites acceptance either by
a promise to ship or by prompt shipment of conforming or nonconforming goods. In this case,
the manufacturer shipped goods that did not conform to the quantity term of the store’s offer.
The Code provides that the shipment of nonconforming goods amounts to both an acceptance
of the offer and a breach of the newly formed contract unless the shipper precedes or accompa-
nies the shipment with notice that it is not accepting the offer and is offering the shipment as an
accommodation. Proper accommodation notice makes the shipment a counteroffer, rather than an
acceptance. The letter sent by the manufacturer stating that it will be able to ship only 40 suits is
not proper accommodation notice. Nothing indicates that the manufacturer is making a counter-
offer rather than an acceptance. In fact, the demand for payment suggests that the manufacturer
believes there is a contract. Without sufficient accommodation notice, a nonconforming shipment
is both an acceptance of the offer and a breach of contract, entitling the nonbreaching party to
damages. Thus, in this case, the store is entitled to deduct the $60 in damages from its payment.
[See UCC §2-711] (A) is incorrect because, as stated above, the letter was not proper accommo-
dation notice. If the manufacturer had stated that the shipment was intended as an accommoda-
tion and the store then accepted the suits, the manufacturer would have been entitled to the full
$400. In that case, the accommodation shipment would be a counteroffer, and the contract would
be for 40 swimsuits. (C) is incorrect because the store accepted the 40 swimsuits. Once a buyer
accepts goods, payment is due at the contract rate, not fair market value. The acceptance does not,
however, bar a claim for damages. (D) too is incorrect because the store accepted the suits. When
a buyer receives nonconforming goods, it may accept all, reject all, or reject some and keep some.
If, as here, the buyer chooses to accept the goods, payment is due at the contract rate—minus
damages.

Answer to Question 4

(C) Arguably, the company’s fax on July 15 could reasonably be interpreted as a rejection of the
landlord’s offer as to the Wood Street building, i.e., having accepted only the Main Street
property, the company was impliedly rejecting the rest. Once rejected, the offer is terminated and
the offeree’s power of acceptance is extinguished; thus, the July 18 attempt to accept would be
ineffective. None of the other alternatives makes sense. For impossibility of performance in (A)
to apply, the impossibility must be “objective”; i.e., the duties could not be performed by anyone.
Also, the impossibility must arise after the contract has been entered into. (D) is wrong because
a condition precedent must be distinguished from a promise. A condition is the occurrence of
an event that will create, limit, or extinguish the absolute duty to perform. In this case, it would
probably be determined that the intention of the parties was an exchange of promises. (B) is
wrong because a unilateral mistake in most cases will not prevent formation of a contract. Only
mutual mistake going to the heart of the bargain may prevent the formation of the contract.

Answer to Question 5

(A) No contract arose on September 2 because the realtor’s offer expired on September 1, when the
realtor did not receive the developer’s acceptance. If a period of acceptance is stated in an offer,
CONTRACTS - OFFER AND ACCEPTANCE ANSWERS 3.

the offeree must accept within that period to create a contract. Failure to timely accept terminates
the power of acceptance in the offeree (i.e., a late acceptance will not be effective and will not
create a contract). Under the mailbox rule, an acceptance generally is effective upon dispatch (i.e.,
the acceptance creates a contract at the moment it is mailed or given to the delivery company).
However, the mailbox rule does not apply where the offer states that acceptance will not be effec-
tive until received. In the latter case, acceptance is effective only upon receipt. Here, the realtor’s
offer specifically stated that the acceptance must be received by September 1 to be effective.
Thus, the realtor opted out of the mailbox rule, and no contract was created by delivery of the
acceptance on September 2. Note that the developer will not be able to successfully argue that
the acceptance was valid since the late delivery was the messenger company’s fault. This would
be a valid argument if the mailbox rule applied here, because the acceptance would have been
effective on September 1, when the message was given to the messenger company. However, by
opting out of the mailbox rule, the realtor put the burden of any negligence in delivery on the
developer. Thus, there was no valid acceptance. (B) is incorrect because of the requirement that
acceptance be received by September 1. This requirement obviates the general “mailbox rule,” so
that the mere mailing of a letter (or sending of a message) does not operate as an effective accep-
tance. (C) is incorrect because the realtor was not obligated to respond in any way to the message
received on September 2. Once the specified time passed without receipt of acceptance, the offer
(as well as the developer’s power of acceptance) was terminated. Thus, receipt of the message on
September 2 created neither a contract nor an obligation on the part of the realtor to respond to
the message. (D) is incorrect because no contract, voidable or otherwise, arose on September 1.
As explained above, there could be no contract because acceptance of the offer was not received
as specified by the offer. Also, the facts do not indicate circumstances under which a contract is
usually held to be voidable. A voidable contract is a contract that one or both parties may elect to
avoid (e.g., contracts of infants). The facts of this question provide no basis for concluding that
any contract that might have arisen between these parties would be voidable.

Answer to Question 6

(B) Because the subject matter of the contract became illegal after the distributor’s offer but before
the retail seller’s acceptance, the supervening illegality is deemed to revoke the offer. It will be a
defense to enforcement of a contract if either the consideration or the subject matter is illegal. If
the illegality is present at the time of the offer, there is no valid offer. If the illegality arises after
the offer but before acceptance, the illegality operates to revoke the offer. If the illegality arises
after formation of a valid contract, it discharges the contract because performance has become
impossible. Here, the retail seller received the distributor’s offer to sell the games on August 5.
Generally, an offer not supported by consideration can be revoked at will, even if the offeror has
promised not to revoke for a certain period. However, an offer by a merchant to buy or sell goods
in a signed writing that, by its terms, gives assurances that it will be held open is not revocable
for lack of consideration during the time stated. The distributor is a merchant who deals in goods
of the kind being sold. Its offer contained in the letter of August 3 assured the retail seller that
its terms would be open until September 2. Thus, the offer was not revocable until September 2,
and the distributor’s purported revocation of August 17 was ineffective. The legislation rendering
illegal the sale of the type of machines to be sold by the distributor to the retail seller took effect
on August 28, two days before the retail seller’s acceptance. Thus, the illegality arose after the
distributor’s offer and before the retail seller’s acceptance. Hence, the state law serves to revoke
the offer. (C) is incorrect because, as explained above, the distributor’s attempted revocation was
ineffective. (D) is incorrect because the state law would only create a condition of impossibility of
performance if it became effective after formation of a valid contract. Here, the law was effective
before the retail seller’s acceptance. (A) is incorrect because if the state law had not been enacted,
4. CONTRACTS - OFFER AND ACCEPTANCE ANSWERS

the distributor would have been bound by the terms of its offer until September 2. The fact that
the distributor was able to obtain a higher price for the machines would not permit it to revoke its
offer prior to expiration of the specified period.

Answer to Question 7

(D) The homeowner will likely prevail on the breach of contract claim because she did not enter into
a contract with the gardener. To form a contract, there must be a valid offer and acceptance. The
homeowner made an offer, but the gardener rejected the offer the next day with his first phone
call. Once an offer is rejected, the offeree’s power of acceptance is destroyed. Thus, the gardener’s
second call was not an acceptance, but rather a counteroffer. The homeowner did nothing to
accept the gardener’s counteroffer, and this is not the type of case where silence will be deemed
to be an acceptance (e.g., where the parties have so agreed or where that has been their course
of dealing). Thus, there was no acceptance and no contract to breach. (A) is incorrect because
an offeree cannot be forced to speak under penalty of having silence treated as an acceptance. If
an offeree silently takes offered benefits, the courts will often find acceptance, especially if prior
dealings between the parties, or trade practices known to both parties, create a commercially
reasonable expectation by the offeror that silence represents an acceptance; in such cases, the
offeree is under a duty to notify the offeror if she does not intend to accept. As discussed above,
the gardener rejected the initial offer and made a counteroffer, putting the homeowner in the
position of offeree. The homeowner’s silence cannot be construed as acceptance absent a showing
of prior dealings between the parties or trade practices known to both. Moreover, the gardener’s
last phone call to the homeowner was somewhat ambiguous, and the homeowner could argue
that she did not know with certainty that the gardener would still perform the job after getting
no response. Further, she was not home that weekend and, therefore, did not stand idly by and
knowingly accept the gardener’s work. (B) is incorrect because this was not a unilateral contract.
A unilateral contract exists only when an offeror makes acceptance possible only by performing
a stipulated act, whereas a bilateral contract contemplates an exchange of promises. Here, the
homeowner asked the gardener whether he would perform an act and expected a reply from the
gardener; indeed, the gardener told the homeowner that he would telephone her with his answer
the next day, which he did (in effect, rejecting the offer). Thus, this was not a unilateral contract
that could properly be accepted through performance. (C) is incorrect because the gardener’s
power of acceptance was destroyed when he rejected the homeowner’s offer, as discussed above,
and once that occurred, the offer was gone and could not be revoked.

Answer to Question 8

(D) An offer must be accepted within a reasonable time. The inventor’s reply letter constituted an
offer to sell 24 choppers for $39.99. The shop owner accepted this offer by a memo plus full
performance. If this occurred within a reasonable time, a contract was formed. (A) is incorrect
because contracts may be formed by nonmerchants. (B) is incorrect because the contract would
have already been formed by the shop owner’s acceptance; it is irrelevant whether the inventor
has enough stock on hand. (C) is incorrect because the inventor’s offer called for prompt accep-
tance and so did not constitute a firm offer, which must be kept open for a reasonable time, not to
exceed three months.

Answer to Question 9

(B) The store owner’s shipment constituted both an acceptance of the administrator’s offer and a
breach of the resulting contract. Under the UCC, an offer to buy goods can be accepted by the
CONTRACTS - OFFER AND ACCEPTANCE ANSWERS 5.

seller by either a promise to ship or by prompt shipment of conforming or nonconforming goods.


The UCC requires perfect tender, which means that the goods and their delivery must conform
to the contract exactly or it is a breach. Thus, shipment of nonconforming goods constitutes an
acceptance and a breach unless the seller notifies the buyer that the goods are offered only as
an accommodation. Here, the store owner shipped nonconforming goods without a notice of
accommodation; thus, the shipment was a breach. (A) is incorrect because the shipment was
not an acceptance of the conforming goods and a counteroffer as to the nonconforming goods.
The offeree has no power to partially accept an offer. The administrator ordered five desktop
computers and did not receive what he ordered. (C) is incorrect because under the UCC shipment
of requested goods, even nonconforming goods, is an acceptance, and it could be a counteroffer
only if notice of accommodation is given. (D) is incorrect because the result of the shipment is an
acceptance, not merely an offer to furnish substituted goods. For the shipment to function as an
offer to furnish substituted goods, notice must be given that the goods are intended as an accom-
modation. Here, no such notice was given, so the shipment was an acceptance (and a breach).

Answer to Question 10

(A) The mechanic will recover nothing because no contract was formed. To create a contract, there
must be an effective offer and acceptance. To be effective, an acceptance must be made before the
offer is terminated. An offer may be terminated in a number of ways, including directly informing
the offeree that the offer is being revoked. Here, the mechanic received a direct revocation of the
offer from the owner by text before he accepted the offer. If the car owner’s revocation was effec-
tive, the mechanic’s acceptance was ineffective. An offeror has the power to revoke an offer any
time before an acceptance has occurred, even if the offeror has promised to keep the offer open,
unless (i) consideration has been paid to keep the offer open (thus forming an option contract),
(ii) the offeree reasonably relies on the offer to his detriment, or (iii) the offer is a merchant’s firm
offer. Here, the mechanic did not pay the car owner to keep the offer open, there was no reason-
able detrimental reliance on the offer before the revocation (the mechanic did not even drive to
the car owner’s house until after receiving the revocation), and the car owner could not make
a merchant’s firm offer with respect to the car because he is not a merchant (i.e., he is not a car
dealer). Thus, the owner had the power to revoke, his revocation by text was effective, and the
mechanic’s acceptance was therefore ineffective. (B) is incorrect for two reasons: (i) there was
no contract between the parties (see above), and (ii) even if there were a contract, the mechanic
would not recover $45,000. In a contract for the sale of goods, the basic remedy for nondelivery
is the difference between the contract price and either the market price or the cost of cover, plus
consequential and incidental damages. Here, there is a $3,000 difference between the contract
price and the market price (and cover price), and so $3,000 would be the amount necessary to
put the mechanic in the position he would have been in had the contract been performed. (C) is
incorrect because, as indicated above, the mechanic’s power of acceptance was terminated by
the owner’s prior revocation of the offer. (D) is incorrect because, as indicated above, no option
contract was created, because the mechanic did not pay to keep the offer open.

Answer to Question 11

(A) The leasing company is contractually bound to lease the skybox to the advertising agency. Under
certain circumstances, an executory bilateral contract may be formed without any communication
of acceptance. A common example is where prior dealings between the parties, or trade practices
known to both, create a commercially reasonable expectation by the offeror that silence repre-
sents an acceptance. In such a case, the offeree is under a duty to notify the offeror if it does not
intend to accept. Here, despite the language in the invitation making leases subject to approval,
6. CONTRACTS - OFFER AND ACCEPTANCE ANSWERS

the leasing company never sent any notification of approval prior to sending out the tickets and
invoice right before the season would start. This course of dealing over the past five years gave
the advertising agency reason to expect that the leasing company’s silence after the invitation was
returned constituted an acceptance by the leasing company, regardless of the company’s actual
intent. [See Restatement (Second) of Contracts §69, illus. 5] (B) is incorrect because the language
stating that approval would be required precludes the invitation from constituting an offer. The
advertising agency’s response constitutes the offer and incorporates the term relating to approval
by management of the leasing company. Thus, the leasing company had the power to reject the
advertising agency’s offer if it acted within a reasonable time. Its failure to do so, given the course
of dealing between the parties, constituted an acceptance. (C) is incorrect because, as stated
above, the leasing company had a duty to reject the advertising agency’s offer within a commer-
cially reasonable time. Its delay of over two months before notifying the agency right before the
season began, after the agency had scheduled clients to use the skybox, was not commercially
reasonable under the circumstances. (D) is incorrect because the advertising agency would not
have to rely on a quasi-contract theory to recover its reliance damages. As discussed above, the
agency was correct in its assertion that a contract was created between the parties and that the
leasing company would be in breach if it did not perform. The advertising agency would then be
entitled to all appropriate contract remedies, including reliance damages.

Answer to Question 12

(C) The low bidder is liable for nothing because no contract was formed between the contractor and
the low bidder. Formation of a contract requires mutual agreement between the parties (offer and
acceptance) and consideration. There was no contract here because there was no acceptance. The
low bidder’s bid constituted an offer—a certain and definite promise, undertaking, or commit-
ment to enter into a contract communicated to the offeree. An offer gives the offeree the power to
accept and create a contract until the offer is terminated. An offer can be terminated in a number
of ways, including through a counteroffer from the offeree. A counteroffer serves as both a rejec-
tion terminating the original offer and a new offer from the original offeree, thus reversing the
former roles of the parties and giving the original offeror the right to accept or reject the new
offer. Here, the contractor’s call constituted a rejection and a counteroffer that the low bidder
rejected, and so no contract was formed. Therefore, the low bidder cannot be held liable. (A) and
(B) are incorrect because, as stated above, a contract was never formed between the contractor
and the low bidder. Thus, it is irrelevant whether the mistake was unilateral or obvious. (Note
that the general rule is that a contract will not be set aside for a unilateral mistake unless the
nonmistaken party either knew or should have known of the mistake. Thus if the contractor had
not called the low bidder but had instead accepted his bid, the low bidder would be liable on the
contract despite his mistake because (B) the mistake was unilateral and (A) it was not obvious.)
(D) is incorrect for several reasons: First, it relies on the existence of a contract, and as stated
above, there is no contract here. Second, the premise that the low bidder could not renege on its
offer is untrue. The general rule is that offers are revocable until accepted. In a subcontractor bid
situation, a bid is treated as irrevocable for a reasonable amount of time because of detrimental
reliance (i.e., the general contractor will rely on the mistaken bid in preparing his bid). However,
here the contractor learned of the low bidder’s mistake before any reliance on the bid. Moreover,
it is unclear whether the contractor is complaining about the low bidder’s reneging on his bid; the
contractor appears to be complaining that the low bidder refused to lower the mistaken bid. The
low bidder would have no duty to lower its bid in any case. The final premise in (D) is irrelevant.
If the low bidder lacked the power to renege, the lack of power goes to the $75,000 bid; the fact
that no one would do the job for $70,000 has no bearing on the issue.
CONTRACTS - OFFER AND ACCEPTANCE ANSWERS 7.

Answer to Question 13

(B) The contract was formed on May 2. An offer to buy goods for shipment is generally construed
as inviting acceptance either by a promise to ship or by shipment. Here, the letter constitutes a
promise to ship and thus is an acceptance. The rule for acceptances is that they are effective as
soon as they are dispatched, which was May 2. Thus, (B) is correct. (C) is incorrect because,
under the mailbox rule, a letter of acceptance creates a contract at the moment of dispatch, not on
the day that the offeree receives it. (A) is wrong because the order was an offer, not an acceptance
of the catalog listing. Catalogs containing price quotations are generally construed as invitations
to offer rather than offers. (D) is wrong because acceptance occurred before shipment; it occurred
when the manufacturer sent its promise to ship.

Answer to Question 14

(A) The art patron may purchase the photographic work at whatever price the court determines to be
reasonable. In a contract for the sale of goods, the failure to state the price does not prevent forma-
tion of a contract if the parties intended to form a contract without the price being settled. In such
a case, if the price is left to be agreed to by the parties and they fail to agree, a reasonable price at
the time of delivery will be supplied by the court. [UCC §2-305] Here, the parties had a contract
for the sale of goods (a piece of artwork), with the price of the artwork not fixed at the time of
contracting. Because they have failed to agree on a price, the art patron is entitled under the UCC
to purchase the artwork at whatever price the court determines to be reasonable. (B) is incor-
rect because there is no provision for arbitration in the agreement. As stated above, if the price is
left open in a contract for sale of goods, and the parties are unable to agree later on a price, the
court will supply a reasonable price. [UCC §2-305] (C) is incorrect because even though $700
was a good faith tender, it may or may not be the “reasonable” price determined by the court. As
discussed above, if the price in a contract for sale of goods is left to be agreed to by the parties at
a later point and they fail to agree, the court will supply a reasonable price. [UCC §2-305] (D) is
incorrect because, as discussed above, the UCC provides that if the price is left to be agreed to by
the parties and they fail to agree, the court will supply a reasonable price. [UCC §2-305] The art
patron would have to pay $1,200 only if the court determined that $1,200 was a reasonable price.

Answer to Question 15

(B) The investor’s communication was a request for an offer and the dealer’s response was an offer.
For a communication to be an offer, it must create a reasonable expectation in the offeree that the
offeror is willing to enter into a contract on the basis of the offered terms. The investor’s commu-
nication does not pass the test because it is clear on its face that he did not want to be bound by
whatever price the dealer came up with, but rather wanted to find out what the dealer would offer.
The dealer’s communication, on the other hand, passes the test. While it said nothing more than
the price, it was sent in response to a request containing specific delivery terms and a specific
quantity. Under the circumstances, the dealer’s response would have created a reasonable expecta-
tion in the investor that the dealer was willing to enter into a contract under the terms of the two
communications. (A) is wrong because, as indicated above, the investor’s communication was not
an offer because it did not indicate the requisite intent to be bound. Thus, the dealer’s communica-
tion could not be an acceptance, because an acceptance must be in response to an offer. (C) and
(D) are wrong because, as explained above, the dealer’s communication was not merely a price
quotation; given that it was sent in response to a very specific inquiry, it was sufficient to be an
offer.
8. CONTRACTS - OFFER AND ACCEPTANCE ANSWERS

Answer to Question 16

(D) Offers of this type normally require only that the offerees supply information leading to the
arrest and conviction of the culprit. The language of the reward offer should not be read with
total literalness, but rather in the context of what the city was seeking to obtain and the normal
duties required to accept an offer of this kind. Therefore, (A), which would require that the offeree
actually participate in the arrest, can be eliminated. (B) and (C) can be ruled out without diffi-
culty because they would envision a bilateral contract under which the offeree would be promising
or assenting to the city’s proposal. In this context the offer is obviously a unilateral proposal that
can be accepted only by performance.
CONTRACTS - REMEDIES
QUESTIONS
CONTRACTS - REMEDIES QUESTIONS 1.

CONTRACTS - REMEDIES QUESTIONS

Question 1 (D) In addition to other legally allowable


damages, an allowance for the landowner’s
A landowner nearing retirement hired a mental distress if the house cannot be
company to clear all the trees from a speci- completed in time for his retirement.
fied portion of a plot of land he owned in the
woods and to build a rustic log cabin on the Question 2
property with those trees. Under the contract,
the landowner was to make payments to the A franchised United States dealer of a very
company of $10,000 on the first of every month popular German car contracted with a doctor to
for nine months beginning August l, on monthly sell him the car for $29,000 cash, the sale to be
presentation of a certificate by an environmental consummated after delivery to the dealer of the
protection group certifying that construction was car, which the dealer ordered from the manufac-
proceeding in an ecologically sound manner. turer specifically for the doctor. The average
The company cleared the trees in July and began retail markup in such sales is 30%. The signed
working on the cabin. The landowner made retail contractual document was a form drafted
three $10,000 payments for the work done in by the dealer’s lawyer, and the doctor did not
July, August, and September, without requiring question or object to any of its terms. When the
a certificate. The company worked through car arrived from Germany, the doctor repudi-
October, but no work was done from November ated the contract. The dealer at once sold the car
1 to the end of February because of bad weather, for $29,000 cash to another buyer, for whom the
and the landowner made no payments during dealer had also ordered from the manufacturer a
that period. The company did not object. On car identical to the doctor’s.
March l, the company demanded payment of
$10,000; but the landowner refused on the In an action against the doctor for breach of
grounds that no construction work had been contract, how much will the dealer recover?
done for four months and the environmental
protection group had issued no certificate. The (A) $29,000 minus what it cost the dealer to
company thereupon abandoned work and repudi- purchase the car from the manufacturer.
ated the agreement.
(B) $29,000 minus the wholesale price of an
Assuming that the company committed a total identical car in the local wholesale market
breach on March l, what would be the probable among dealers.
measure of the landowner’s damages in an action
against the company for breach of contract? (C) Nominal damages only, because the dealer
resold the car to the other buyer without
(A) Restitution of the three monthly install- lowering the retail price.
ments paid in August, September, and
October. (D) Nothing, because the parties’ agreement
was an adhesion contract and therefore
(B) What it would cost to get the cabin unconscionable.
completed by another builder, minus
installments not yet paid to the company.

(C) The difference between the market value of


the partly built cabin, as of the time of the
company’s breach, and the market value of
the cabin if completed according to specifi-
cations.
2. CONTRACTS - REMEDIES QUESTIONS

Question 3 Question 4

A homeowner purchased a large recreational An automobile dealer agreed to sell a car to


vehicle. Local ordinances in the homeowner’s a buyer for $30,000, with a down payment of
suburb prohibited residents from parking recre- $6,000 due at the time the sales contract was
ational vehicles on the street or in an open signed and the balance payable in monthly
driveway, so the homeowner contacted a local installments over a period of five years. Under
contractor and explained his requirements for a the written contract, delivery would be made
garage. The contractor measured the vehicle and within 30 days. Two weeks after making the
then entered into a written contract to build a down payment, the buyer told the dealer that he
garage for the homeowner at a price of $5,000, lost his job and could not afford to go through
payable on completion of the job. When the with the purchase. The dealer, which could
garage was finished but before the contractor get as many of that model of car as it required
was paid, the homeowner drove the vehicle from the manufacturer for a wholesale price
into the garage, only to discover that the garage of $21,000, put the car in question back in its
was three inches too short to accommodate the inventory but refused to return the buyer’s down
vehicle. The homeowner was told that it would payment. A short time later, the dealer sold it to
cost $4,000 to partially dismantle the garage someone else for $28,500.
and rebuild it to fit the vehicle. The homeowner
refused to pay the contractor anything for the The buyer sues the dealer to get back his
job. The contractor consulted with several deposit, and the dealer counterclaims for
independent real estate appraisers, and they all damages. For purposes of this question, do not
agreed that the garage had enhanced the value of include incidental damages in your calculations.
the homeowner’s property by $6,000.
Who will prevail and for what amount?
If the contractor sues the homeowner, what
amount will the contractor likely recover? (A) The buyer will recover $6,000.

(A) $5,000, because a three-inch variation in (B) The buyer will recover $4,500.
length is, at most, a minor breach.
(C) The dealer will recover $9,000.
(B) $2,000, measured by the difference
between the amount that the garage (D) The dealer will recover $3,000.
enhanced the value of the property and the
cost to rebuild the garage to specifications.

(C) $1,000, measured by the difference between


the contract price and the amount it would
cost to rebuild the garage to specifications.

(D) Nothing, because the three-inch error was


a material breach, and the contractor will
be unable to successfully claim substantial
performance.
CONTRACTS - REMEDIES QUESTIONS 3.

Question 5 Question 6

A father had made a legally binding promise A contractor gave the low bid for some
to furnish his son and his fiancée with a house electrical repairs to a homeowner’s house. Based
on their wedding day, planned for June 10 of on this bid, the contractor and the homeowner
the following year. Pursuant to that promise, entered into a contract stating that the contractor
the father and a contractor-friend entered into a would perform the electrical repairs for $6,000.
contract for the building of a home on a piece Before beginning work on the project, the
of undeveloped land that the father owned. The contractor notified the homeowner that he would
contractor began building the house but breached lose money on the job at that price, and would
the contract by abandoning the house unfinished not proceed with the work unless the homeowner
on March 1. The contractor was aware when the would agree to increase the price to $9,000.
agreement was made of the purpose for which The homeowner thereupon, without notifying
the father wanted the completed house. the contractor, entered into a contract with
an electrician to make the repairs for $7,500,
Which of the following, if true, would best which was the fair market cost of the work to
support the father’s claim for consequential be done. The electrician finished the house on
damages on account of the house not being schedule and then showed the homeowner that
finished by June 10? he (the electrician) had spent $8,500 on the job.
The homeowner thereupon paid the electrician
(A) The son and his bride, married on June 10, the full balance of their contract price plus an
would have to pay storage charges on their additional $1,000, so that the electrician would
wedding gifts and new furniture until the not lose money on the job.
house could be completed.
In a contract action by the homeowner against
(B) The son’s fiancée jilted him on June 10 and the contractor, how much will the homeowner
ran off with another man who had a new recover?
house.
(A) The difference between the fair market cost
(C) The father was put to additional expense in of the repairs and the contractor’s original
providing the son and his bride, married on contract price.
June 10, with temporary housing.
(B) $3,000, the difference between the contrac-
(D) On June 10, the father paid a $5,000 tor’s original contract price and the amount
judgment obtained against him in a suit the contractor demanded.
filed March 15 by an adjoining landowner
on account of the father’s negligent excava- (C) $2,500, the difference between the contrac-
tion, including blasting, in an attempt to tor’s original contract price and the total
finish the house himself after the contrac- amount the homeowner paid the electrician
tor’s repudiation. for the repairs.

(D) $1,500, the difference between the contrac-


tor’s original contract price and the electri-
cian’s contract price.
4. CONTRACTS - REMEDIES QUESTIONS

Question 7 Question 8

A homeowner contracted with a builder for On March 15, a vineyardist entered into a
the remodeling of the homeowner’s bathroom written agreement with a winery that provided
and kitchen at a cost of $10,000. The contract that the vineyardist would sell 1,600 tons of
was in writing and specified that the work was to tokay grapes to the winery for $750 per ton,
be completed within two months after the date delivery to be no later than November 1 of the
of execution of the contract. Two weeks after same year. By November 1, the vineyardist
entering into the contract with the homeowner, had delivered only 700 tons of grapes and had
the builder was offered an extremely lucra- informed the winery by fax that she had used
tive job that would take all of his time and the remainder of her crop in the production of
effort for several months. The builder told the wine for her own shop. The winery purchased an
homeowner that he was not going to perform. additional 900 tons of tokay grapes from other
The homeowner diligently called many other growers at the then-prevailing market price of
contractors over a period of several weeks and $800 per ton. The vineyardist has submitted
none of them could offer a price anywhere near an invoice to the marketing department of the
as low as $10,000 for the remodeling work that winery for $525,000.
he wanted done. Two months after entering into
the contract with the builder, the homeowner Ignoring incidental costs of cover, how much
sued the builder for specific performance. should the winery pay the vineyardist?

What is the likely result of the suit? (A) $525,000, since by accepting delivery of
the 700 tons of grapes, the winery waived
(A) The court will order specific performance, an objection to the vineyardist’s breach.
because the homeowner was ready and able
to perform his part of the contract. (B) The market value of her 700 tons of grapes
as of November 1, less the cost of cover for
(B) The court will order specific performance, the remaining 900 tons.
because, despite diligent efforts, the
homeowner could find no one who could (C) $480,000, which represents the contract
perform the desired services at a competi- price for the grapes she delivered less the
tive price. cost of cover for the remaining 900 tons.

(C) The court will not order specific perfor- (D) Nothing, since she will be unable to enforce
mance, because the doctrine of laches any claim for payment in court.
applies.

(D) The court will not order specific perfor-


mance, because the remedy at law is
adequate.
CONTRACTS - REMEDIES QUESTIONS 5.

Question 9 Question 10

A hotelier planning to build a new hotel In a writing signed by both parties, a


estimated that first year profits would be about renowned architect agreed to design and super-
$10,000 per day. To encourage its contractor to vise construction of a new house for a buyer.
work in a timely manner, the hotelier included The architect’s fee was to be paid on comple-
in their contract a liquidated damages clause, tion of the house. When the design plans were
providing that the contractor would be liable to about two-thirds complete, the architect assigned
pay the hotelier $100,000 per day for each day to a newly licensed architect “all of my rights
that the contractor is late in completion of the and duties under my design and construction-
project. During the project, the contractor failed supervision contract with the buyer.” The novice
to place an order for the hotel’s elevators in a architect expressly promised the architect to
timely manner. As a result of that oversight, the carry out the work to the best of her ability. The
elevators were installed late and the hotel conse- buyer, on learning of the assignment, refused
quently opened 30 days later than scheduled. to allow the novice architect to proceed on the
project and brought an action against the archi-
The hotelier sued the contractor for damages. tect to compel him to resume and complete
At trial, an expert witness testified that the hotel performance of the contract.
would have received $300,000 in income during
that 30-day period and would have expended Is the buyer entitled to such relief?
$200,000, leaving a total profit of $100,000.
(A) Yes, because the architect’s services under
How much should the contractor be required the contract are unique.
to pay to the hotelier in damages?
(B) Yes, because the architect has personally
(A) $100,000, representing the hotelier’s lost completed two-thirds of the design work.
profits.
(C) No, because the architect-buyer contract is
(B) $300,000, representing the hotelier’s lost one for personal services by the architect.
income.
(D) No, because the architect effectively
(C) $3 million, representing damages provided delegated his remaining duties under the
in the contract. architect-buyer contract to the novice archi-
tect.
(D) $3.1 million, representing damages
provided in the contract plus lost profits.
6. CONTRACTS - REMEDIES QUESTIONS

Question 11 Question 12

A dealer sent a fax to a wholesaler stating A builder contracted with a landowner to


“Send 500 ‘Granny Rocker’ chairs at your usual construct a house on the landowner’s property
price.” The wholesaler responded, also by fax, for a contract price of $100,000. Before
“Will ship our last 500 ‘Granny Rocker’ chairs the builder completed his performance, the
at $100 per chair, our usual price. ‘Granny landowner informed the builder that he (the
Rocker’ line is being discontinued.” The whole- landowner) no longer wanted the house, and that
saler’s staff immediately began the paperwork he had no intention of paying the builder. At that
for processing the order and started preparing time, the builder had already incurred costs of
and packing the chairs for shipment. The dealer part performance of $30,000 and would have to
faxed back to the wholesaler, “Cancel order for spend an additional $60,000 to finish the job.
‘Granny Rocker’ chairs; your price is too high.”
The dealer had found a mill outlet that was the If the builder sues the landowner for breach
only other source of the chairs. Although the of contract, what is the builder’s most likely
outlet’s price was also $100 per chair, it was recovery?
more convenient for the dealer to buy the chairs
from the outlet. The day after receiving the (A) $10,000.
dealer’s cancellation, the wholesaler was able to
sell the 500 “Granny Rocker” chairs in its stock (B) $30,000.
to another party for $100 each.
(C) $40,000.
If the wholesaler sues the dealer for damages,
how much should the wholesaler recover? (D) $60,000.

(A) Nothing, because this was a contract be-


tween merchants and the dealer canceled
within a reasonable time.

(B) Nothing, because the wholesaler was able to


cover by selling the chairs at the same price
it would have received from the dealer.

(C) $50,000, the full contract price, because the


dealer breached the contract and $100 per
chair was a fair price.

(D) The wholesaler’s incidental costs of


preparing the paperwork and other office
costs connected with preparing and packing
the chairs for shipment to the dealer.
CONTRACTS - REMEDIES QUESTIONS 7.

Question 13 Question 14

A wholesaler who sold hair-care products to The owner of several radio stations throughout
beauty salons and spas contracted to purchase the Midwest purchased land in a small Midwest
20 hand-held ionic hair dryers to be delivered town for the purpose of building another radio
no later than May 1. She paid for the hair dryers station. He entered into a written contract with
at the time she placed her order. When the hair a contractor to build a studio and broadcast
dryers were delivered on May 1, the wholesaler transmitter on the property for $3 million. In
noticed that they were hot air, and not ionic, hair response to an engineering survey undertaken
dryers. She immediately contacted the manufac- before the contract was executed, the contract
turer, which refused to grant the wholesaler any contained a clause providing that the under-
remedy. ground cables be laid in a path that avoided an
identified area of magnetized rock, because
If the wholesaler decides to keep the hair under certain conditions, such as magnetic
dryers and brings a claim for breach of contract storms generated by sunspot activity, a higher
against the manufacturer, which of the following than normal level of interference with the radio
awards is most likely? broadcast signals would occur if the cables were
laid there.
(A) Nothing, because the wholesaler waived
her rights when she paid for the hair dryers After the contractor finished the job, the radio
in advance. station owner discovered that the contractor had
laid a portion of the underground cables through
(B) Nothing, because the wholesaler decided to a patch of magnetized rock. Despite the defect,
keep the nonconforming hair dryers. the studio and broadcasting transmitter are now
worth about $4 million. Without the defect, their
(C) The difference between the value of the value would be $100,000 more. The radio station
ionic hair dryers and the value of the hot-air owner contracted with an electrical contractor to
hair dryers. reroute the cables around the magnetized rock
for a cost of $50,000.
(D) The difference between the contract price
and the cost of buying conforming goods. How much is the original contractor entitled
to collect from the radio station owner?

(A) $2,900,000, because the actual value of the


property is $100,000 less than the value it
would have had if the contractor had not
breached.

(B) $3 million, because the value of the


property even with the defect is greater than
the contract price.

(C) $2,950,000, because the radio station owner


will have to pay $50,000 to correct the
contractor’s defective performance.

(D) Only the expenses he incurred, but not his


expected profit, because he breached the
contract with the radio station owner.
CONTRACTS - REMEDIES
ANSWERS
CONTRACTS - REMEDIES ANSWERS 1.

CONTRACTS - REMEDIES ANSWERS

Answer to Question 1

(B) Cost of completion minus installments is the correct measure of damages because the facts give
us the breach of a construction contract by the builder during construction. In such cases where
the builder breaches after partially performing, the owner of the land is entitled to the cost of
completion plus reasonable compensation for any delay in performance. Courts generally allow
the builder to offset or recover for work performed to date to avoid the unjust enrichment of the
owner. Hence, the unpaid installments should be deducted. Although this option does not mention
reasonable compensation for delay, it is clearly a more accurate statement of the correct measure
of damages than the other options. (A) is incorrect because the landowner’s damage relates to
the cost of having the cabin completed. This cost could either exceed or be less than the restitu-
tion of the installments. If the amount is less, the landowner would be unjustly enriched. (C) is
incorrect because damage relates to cost of completion and not market value. The house might
have minimal market value in its partially completed state, and to measure damages based on the
differences between such minimal value at the time of breach and market value when completed
according to specifications could dramatically overstate the landowner’s real damages. (D) is
incorrect because damages for mental distress are too speculative and are not awarded in a
contract situation.

Answer to Question 2

(A) The dealer can collect his lost profits, i.e., the difference between the contract price ($29,000) and
what the dealer paid to purchase the car from the manufacturer. In a contract for the sale of goods,
a seller can collect his lost profits when the buyer breaches if the seller cannot be made whole by a
subsequent sale of the item contracted for. This occurs where the seller can obtain or manufacture
as many goods as it can sell (e.g., a car dealership), because in such a situation, the seller would
have been able to sell to the subsequent purchaser anyway. This is known as a lost volume situa-
tion, and in such a situation, the UCC allows the seller to sue for his lost profits. Generally, lost
profits are measured by the difference between the cost of goods and the contract price, less the
seller’s saved expenses. (B) is incorrect because it uses the wrong measure for lost profits. The
cost of a similar car in the local wholesale market is irrelevant because the dealer did not purchase
the car on the wholesale market, but rather purchased the car from the manufacturer. The dealer’s
actual price will be used to determine his lost profits. (C) is incorrect because although the dealer
resold the car, the resale has not made him whole since he could have sold a car to the other buyer
anyway. Thus, the resale does not put the dealer in as good a position as he would have been in
had the doctor performed (the goal of contract remedies). Thus, the dealer will be allowed to
recover his lost profits, as explained above. (D) is incorrect because contracts of adhesion are not
unconscionable per se, and there is nothing in the facts to indicate a degree of unconscionability
that would render the contract voidable at the buyer’s option.

Answer to Question 3

(C) The contractor will recover $1,000 in a restitution or quasi-contract action. Despite having built a
garage, the contractor has breached the contract here. He was to build a garage that would fit the
homeowner’s recreational vehicle and the garage that he built is too short. Moreover, the breach
was material, even though the garage was a mere three inches short, because the homeowner did
not receive the substantial benefit of his bargain—a place for his vehicle. Therefore, the contractor
cannot recover in contract and (A) is incorrect. Nevertheless, courts agree that where the breach
2. CONTRACTS - REMEDIES ANSWERS

is not willful, the contractor can recover on the failed contract in restitution or quasi-contract to
prevent unjust enrichment. Here, it is clear that the breach was not willful. (D) is therefore incor-
rect. Regarding the amount of recovery, most courts would limit the recovery to the contract
price (rather than the benefit received by the nonbreaching party), offset by the damages to the
nonbreaching party (here, the $4,000 cost of rebuilding the garage as bargained for). Thus, (C) is
correct and (B) is incorrect.

Answer to Question 4

(D) The dealer will recover $3,000, which is the difference between its lost profits and the buyer’s
down payment. When the buyer repudiates or refuses to accept goods, the usual measure of the
seller’s damages is the difference between the contract price and the market price or the difference
between the contract price and the resale price of the particular goods. However, neither of those
measures of damages gives adequate compensation for the buyer’s breach where the seller has an
unlimited supply of the goods (i.e., a lost volume seller), because, but for the buyer’s breach, the
seller would have made two sales instead of one. In this type of case, lost profit is measured by the
contract price less the cost to the seller. Here, the dealer could have made two sales of that model
of car because it could get as many as it needed from the manufacturer. Hence, it lost a profit of
$9,000 as a result of the buyer’s breach. This amount is offset against the amount of the down
payment that the buyer made, resulting in a net recovery of $3,000 by the dealer. (A) is incorrect
because the dealer did suffer damages as a result of the buyer’s breach. (B) is wrong because it
represents the down payment minus the difference between the contract price and the resale price
for the goods. As discussed above, that does not adequately compensate the dealer for its damages
from the buyer’s breach. (C) is incorrect because $9,000 constitutes the dealer’s lost profits from
the buyer’s breach, but it must be offset against the down payment that the dealer received from
the buyer.

Answer to Question 5

(C) The father’s additional expense in providing temporary housing would be the best basis for a
claim of consequential damages. In addition to the standard measure of damages, consequential
damages may be awarded for further losses resulting from the breach that any reasonable person
would have foreseen would occur from a breach at the time of entry into the contract. Temporary
housing expenses would have been foreseeable at the time the contractor and the father entered
into the contract. (A) is incorrect because it would be consequential damage to the son rather than
the father, because it is the son who will incur the expense. (B) is incorrect because the fiancée’s
jilting of the son is not a foreseeable consequence of the breach. (D) is incorrect because at the
time the contractor and the father entered into the contract, both parties would have reasonably
assumed that the house would be built and completed by experienced construction personnel. The
father’s attempt to complete the house himself and the ensuing negligence were not foreseeable.

Answer to Question 6

(D) The homeowner can recover $1,500, the difference between the contractor’s contract price and
the contract price of the substitute performance. Here, while the homeowner actually paid $2,500
more than the contractor’s contract price to have the house repaired, he was obligated to pay
only $1,500 more because the electrician had a legal duty to make the repairs to the house for
his contract price and no more. The homeowner will not be able to recover the $1,000 difference
because he has a duty to mitigate damages, and paying more than he was actually obligated to
pay breaches the duty. (A) and (B) are incorrect because they do not apply the proper measure of
damages formula. (C) is incorrect because the “cost of completion” does not include the additional
一 信:liuxue119118 , 们 信免 供

CONTRACTS - REMEDIES ANSWERS 3.

$1,000 the homeowner gave the electrician to save the electrician from having performed the job
at a loss. As explained above, the homeowner was not required to pay the electrician the $1,000 to
complete the repairs and the homeowner’s paying the extra amount breaches his duty to mitigate
damages. Thus, the homeowner cannot recover the extra $1,000.

Answer to Question 7

(D) Specific performance is available only when monetary damages are inadequate. The court will
not order specific performance merely because the builder had a much lower price than anyone
else. Instead, the court will require the nonbreaching party to hire a different contractor, and the
builder would be liable for the difference between the new price for remodeling and the original
$10,000 price. It should be noted that nothing indicates that the builder was hired for his unique
talents; even if he were, specific performance would not be granted because of difficulties of
supervision and a reluctance to force one person to work for another. Thus, (A) and (B) are incor-
rect. (C) is also incorrect. Laches is a defense that would be asserted by the builder, contending
that the homeowner had not filed suit demanding specific performance in a timely fashion.
However, the facts tell us that the homeowner diligently called other contractors for many weeks
and sued the builder two months after the initial contract. Thus, the defense of laches does not
apply.

Answer to Question 8

(C) The vineyardist is entitled to the contract price for the grapes delivered and accepted, but the
winery is entitled to cover—to purchase grapes at the market price prevailing at the time of
performance and to deduct any increase over the contract price. (A) is wrong because the winery
is entitled to cover and does not “waive” the breach by accepting a part performance. (B) is wrong
because the price for the grapes delivered is the contract price, not the prevailing market price.
(D) is wrong because the vineyardist is entitled to payment for the grapes she delivered and would
be able to enforce her claim through litigation, if necessary.

Answer to Question 9

(A) The hotelier will be able to recover $100,000, its lost profits. The purpose of contract damages is
to put the nonbreaching party into as good a position as it would have been in had the breaching
party fully performed. This would be represented here by the hotelier’s lost profits, which is its
income ($300,000) minus its expenses ($200,000), or $100,000. The liquidated damages provi-
sion will not apply because it is unreasonable (see below). (B) is incorrect because giving the
hotelier its $300,000 income would put it in a better position than it would have been in if the
contractor had performed, because such an award does not take into account the expenses that the
hotelier avoided by not being in operation. (C) is incorrect because a court would not uphold the
liquidated damages clause here. Liquidated damages clauses are enforceable only if damages were
difficult to estimate at the time the contract was formed, and the amount agreed upon is a reason-
able forecast of the damages that would result from a breach. Here, the damages may have been
difficult to predict when the contract was formed because it is not known just how well a new
hotel will do; however, the amount agreed upon is too high. The facts indicate that a reasonable
estimate at the time of the contract was about $10,000 per day, but since the liquidated damages
amount ($100,000 per day) is 10 times that amount, this amount was unreasonable. (D) is incor-
rect because it seeks to combine the actual damages with the liquidated damages. Regardless of
whether the liquidated damages clause will be enforced here, this measure is improper because a
party may recover either liquidated damages, or, if not available, the actual damages, but not both.

一 信:liuxue119118 , 们 信免 供
4. CONTRACTS - REMEDIES ANSWERS

Answer to Question 10

(C) The buyer cannot compel the architect to resume performance. Contracts for personal services are
not subject to specific performance notwithstanding the fact that damages might be inadequate
or difficult to assess or the services to be performed are unique. The courts reason that specific
performance of personal service contracts is tantamount to involuntary servitude and would
present enforcement problems. At most, the buyer would be able to obtain an injunction to prevent
the architect from working on another project at the times the architect agreed to work for the
buyer. Thus, (C) is correct and (A) is incorrect. (B) is incorrect because it is irrelevant. Whether
specific performance is available at all generally depends on whether the subject matter is unique,
but even a contract for unique services cannot be enforced by specific performance. The fact that
a party has begun performance is irrelevant. (D) is incorrect because duties involving personal
skill and judgment may not be delegated absent consent by the obligee (the buyer). The architect
did not have the buyer’s consent, so there was no valid delegation.

Answer to Question 11

(D) The wholesaler will recover only its incidental damages, i.e., the costs of preparing to ship the
chairs. An offer calling for shipment of goods, such as the offer here, may be accepted by prompt
shipment with notice or by a promise to ship. Acceptance forms a contract. Here, the whole-
saler accepted the dealer’s offer by promising to ship (the warning that the wholesaler had no
more chairs was unimportant surplusage since it could fill the dealer’s order), and a contract
was formed. The dealer breached the contract by canceling its order. When a buyer breaches by
repudiating its offer, as the dealer did here, the seller has a right to recover its incidental damages
plus either the difference between the contract price and the market price or the difference
between the contract price and the resale price of the goods. If neither measure is adequate to put
the seller in as good a position as performance would have, she may recover lost profits. Here,
the contract price was the same as both the resale price and the market price; thus, that measure
of damages is not useful. Lost profits are not possible here because the wholesaler sold the chairs
and they were the last ones it had and would have, since the line was discontinued. Therefore,
the wholesaler lost no profits. It made what it would have made if the sale with the dealer had
gone through. Thus, the wholesaler would be limited to its incidental damages under the first two
measures of damages. (A) is incorrect because there is no rule under the UCC, which governs the
contract here, that makes contracts between merchants cancelable within a reasonable time. (B)
is incorrect because, as indicated above, the UCC allows the seller to recover incidental damages.
(C) is incorrect because the UCC seeks only to put the nonbreaching party in as good a position
as it would have been in had the other party performed, and here, awarding the wholesaler lost
profits would put it in a better position than performance would have, since it would give the
wholesaler a double recovery for selling the same goods. (The result would be different, however,
if the wholesaler had had more chairs to sell, because in that case, the breach would have cost the
wholesaler additional sales—i.e., the wholesaler could have sold to the dealer and to the other
party.)

Answer to Question 12

(C) The builder is most likely to recover $40,000 because he is entitled to his profit plus costs. Where
an owner breaches a construction contract after construction has been started but before construc-
tion is completed, the builder is entitled to recover any profit he would have derived from the
contract plus any costs he has incurred to the date of the breach. Here, the builder’s profit would
have been $10,000 and his costs up to the time of the breach are $30,000. Thus, he can recover
CONTRACTS - REMEDIES ANSWERS 5.

$40,000. Another way of saying this is contract price minus the cost of completion. Here, the
contract price was $100,000 and the cost of completion was $60,000. Thus, the builder is entitled
to $40,000. (A) is incorrect because that amount ($10,000) reflects only the builder’s profit and
he is also entitled to his costs. (B) is incorrect because that amount ($30,000) reflects only the
builder’s expenses, and he is also entitled to profits. (D) is incorrect because the builder is not
entitled to recover the cost of completion because he has not done the work.

Answer to Question 13

(C) The wholesaler is entitled to damages based on the difference between the value of the ionic hair
dryers and the value of the hot-air hair dryers that she received. If a buyer accepts nonconforming
goods, the buyer may recover warranty damages. The basic measure of warranty damages is the
difference between the value of the goods delivered and the value they would have had if they
had been as warranted in the contract, plus incidental and consequential damages. Here, the hair
dryers were not ionic, as specified by the contract, and the wholesaler notified the manufacturer
of the nonconformity immediately. The wholesaler is therefore entitled to warranty damages. (A)
is incorrect because paying for goods in advance does not constitute an acceptance and does not
waive any rights to inspect or reject nonconforming goods. (B) is incorrect because accepting
nonconforming goods does not mean that the buyer cannot recover damages for the breach. As
noted above, she is entitled to warranty damages. (D) is an incorrect measure of damages. It states
the rule for cover, which would apply if the buyer had rejected the goods. It is not available when
the buyer accepts the goods.

Answer to Question 14

(C) The contractor who did the original job is entitled to collect $2,950,000 because his breach of
contract entitled the radio station owner to offset the amount of money that will be needed to
compensate him for the breach. A failure to perform in accordance with contractual terms consti-
tutes a breach of the contract. A breach is minor if the obligee gains the substantial benefit of his
bargain despite the defective performance. In the case of a minor breach, the aggrieved party is
entitled to a remedy for the breach, such as damages. However, the aggrieved party is not relieved
of his duty of performance under the contract. The purpose of contract damages based on affir-
mance of the contract is to put the nonbreaching party where he would have been had the promise
been performed, i.e., sufficient damages to allow him to buy a substitute performance. Because
the nonbreaching party in a case involving a minor breach has a duty to tender counterperfor-
mance, his claim for damages is usually asserted as a setoff against his liability to the obligor.
Here, the contract imposed on the contractor a duty to lay the underground cables so as to avoid
magnetized rock. By failing to do so, the contractor breached the contract. Despite this breach, the
radio station owner received the substantial benefit of his bargain because the studio and broad-
cast transmitter have been built. Therefore, the breach is minor, entitling the radio station owner
to a remedy but not relieving him of his duty to pay the contractor under the contract. The radio
station owner has suffered damages of $50,000, because he will have to expend that amount to
reroute the cables in order to put him in the position he would have been in had the contractor
not breached. Thus, the radio station owner can assert the $50,000 claim as a setoff against the
$3 million he is under a duty to pay the contractor, leaving $2,950,000. (A) is incorrect because
a measure of damages based on the difference between the value of what the radio station owner
would have received if the contractor had properly performed the contract and the value of what
he actually received is appropriate only if having a third party properly complete the project
would be wasteful. Here, correcting the breach will cost only $50,000. (B) is incorrect because
the fact that the studio and transmitter are now worth more than the contract price does not put
6. CONTRACTS - REMEDIES ANSWERS

the radio station owner in the position he would have been in had the contractor not breached. The
higher value of the property is part of the radio station owner’s “benefit of the bargain” and does
not reduce the damages that he is entitled to collect from the contractor. (D) is incorrect because
it states a reliance measure of damages that is inapplicable here. Had the contractor’s breach been
material, the radio station owner would not have been under a duty to pay him under the contract,
but the contractor could have recovered in quasi-contract for the benefits he conferred in partially
performing. Here, however, because his breach was only minor, he can recover the amount he is
owed under the contract minus the setoff for the radio station owner’s damages.
CONTRACTS - THIRD-
PARTY BENEFICIARIES
AND ASSIGNMENTS
QUESTIONS
CONTRACTS - THIRD-PARTY BENEFICIARIES AND ASSIGNMENTS QUESTIONS 1.

CONTRACTS - THIRD-PARTY BENEFICIARIES AND ASSIGNMENTS QUESTIONS

Question 1 Question 2

A retailer and a supplier entered into a supply A businessman agreed with an accountant to
agreement. The supplier then assigned the lease to the accountant a small business computer
agreement to a wholesaler for consideration. system. Their oral agreement included that the
The retailer, unaware of the assignment, made lease would be for a term of five years, and that
payment to the original supplier. the accountant would pay a monthly charge
of $500 for the system. The businessman and
Which of the following is correct? the accountant further agreed that $200 of the
monthly charge would be paid by the accountant
(A) The retailer and the supplier are each liable directly to a third party, in satisfaction of a debt
to the wholesaler for half the amount paid. the businessman owed the third party.
(B) The supplier is liable to the wholesaler for The businessman prepared a written copy of
the full amount. the agreement on his computer, but accidentally
did not include the agreement that called for the
(C) The retailer is liable to the wholesaler for direct payment to the third party in the written
the full amount. copy. The businessman sent the copy of the
written agreement, signed by him, to the accoun-
(D) Neither the retailer nor the supplier is liable tant, who also signed it without noticing that the
to the wholesaler for any amount. provision calling for the direct payment to the
third party was missing.

If the accountant refused to make any


payments to the third party, and the latter brings
an action against the accountant for damages as
a consequence, which of the following, if true,
would be most harmful to the third party?

(A) The obligation of the businessman to


the third party was subject to a statute of
limitations that had run prior to the date the
agreement between the businessman and
the accountant was first discussed.

(B) The third party never notified the


businessman or the accountant that he
accepted their agreement to have the
accountant make payments to him.

(C) No consideration supported the obligation


owed by the businessman to the third party.

(D) Before the third party learned of the agree-


ment between the businessman and the
accountant, they agreed that the accountant
would pay the entire $500 directly to the
businessman.
2. CONTRACTS - THIRD-PARTY BENEFICIARIES AND ASSIGNMENTS QUESTIONS

Question 3 Question 4

A well-regarded paving contractor entered A homeowner entered into a written agree-


into a written contract with a store owner to pave ment with a contractor to remodel the homeown-
the parking lot behind his store. The contract er’s basement for a set amount. Among the
contained no provision regarding assignment. A contract’s specifications was a provision that
few days after they entered into the contract, the the contractor was to use an imported Brazilian
contractor encountered scheduling difficulties hardwood for the paneling. The contractor began
and assigned the job to another paving contractor the job, and then assigned his right to payment
who had a comparable reputation and would do under the agreement to a creditor. When the
the job for the contract price. job was a little more than half completed, the
homeowner sold the house to a buyer. In a
With regard to this assignment by the document separate from the deed of convey-
contractor, which of the following statements is ance, the homeowner assigned her right to the
true? contractor’s labor and delegated her duty to pay
under the contract to the buyer. The contractor
(A) The assignment is valid only if the store completed the job on time but used Wyoming
owner agrees to accept performance by the knotty pine instead of Brazilian hardwood for
competitor. the paneling. The knotty pine is considered to
be inferior to the Brazilian hardwood and is
(B) The contractor breached his contract with much less expensive, and the buyer was very
the store owner by assigning it to the displeased when he saw the result.
competitor without his prior consent.
Who can the buyer sue for breach?
(C) The store owner must accept performance
by the competitor. (A) The original homeowner only.
(D) The assignment is valid, even if the store (B) The contractor only.
owner objects, as long as the contractor
supervises the performance of the contract (C) The contractor and his creditor.
by the competitor.
(D) The homeowner, the contractor, and the
creditor.
CONTRACTS - THIRD-PARTY BENEFICIARIES AND ASSIGNMENTS QUESTIONS 3.

Question 5 Question 6

An orchard and a grocer had a written A contract between an investor and an


contract providing that the orchard would supply inventor stated that the investor will finance all
the grocer with a specified amount of apples and of the inventor’s expenses for the next six months
apple cider during the fall season. During the in exchange for half of the profits from any
summer, the orchard’s apple trees were damaged inventions that the inventor develops during that
by apple blight that diminished the fall apple time. During those six months, the investor runs
yield. Subsequently, the orchard assigned the into financial problems and borrows $200,000
contract to another orchard without notifying the from a bank. The investor executes a written
grocer. The grocer discovered the substitution instrument providing that the bank “is entitled to
after he had received two deliveries of apples collect the debt from my share of the profits, if
and cider, but said nothing. After the assignee any, under the inventor contract.” The bank gave
orchard made a third delivery, it failed to make prompt notice of this transaction to the inventor.
the last two deliveries called for in the contract
between the grocer and the first orchard. If the inventor thereafter refuses to account
for any profits to the bank and the bank sues
Does the grocer have a cause of action? the inventor for the investor’s share of profits
then realized, what is the inventor’s strongest
(A) Yes, only against the assignee orchard for argument in defense?
damages.
(A) The investor-inventor contract did not ex-
(B) Yes, against the assignee orchard and the pressly authorize an assignment of rights.
first orchard for damages.
(B) The investor and the inventor are partners,
(C) No cause of action against the assignee not simply debtor and creditor.
orchard because he and it are not in privity
of contract. (C) The bank is not an assignee of the investor’s
rights under the investor-inventor contract.
(D) No cause of action against either orchard.
(D) The bank is not an intended third-party
beneficiary of the investor-inventor contract.
4. CONTRACTS - THIRD-PARTY BENEFICIARIES AND ASSIGNMENTS QUESTIONS

Question 7 Question 8

An investor entered into a contract with a A vineyard and a wine distributor enter into
winery. The contract provided that the investor a valid written agreement whereby the vineyard
would invest $1 million in the winery and, in is to supply the wine distributor with all of the
return, the winery would produce and market distributor’s requirements of wine for a period
at least 500,000 bottles of wine each year for of five years. The agreement contains a clause
five years under a specified label. The contract prohibiting assigment of the contract. Nonethe-
included a provision that, if feasible, the wine less, a few months after the agreement is entered
would be distributed by the winery only through into, the vineyard assigned its rights under the
a certain wholesale distributor of fine wines. contract to one of its creditors. The vineyard
Neither the investor nor the winery had previ- continued to supply the distributor with wine,
ously dealt with the distributor. The distributor and the distributor paid the creditor. After one
learned of the contract two days later from year, the wine distributor decides that it wishes
reading a trade newspaper. In reliance thereon, to purchase its wines from a different vineyard
he immediately hired an additional sales execu- and notifies the vineyard that it is terminating its
tive and contracted for enlargement of his wine contract.
storage and display facility.
If the creditor of the vineyard sues the distrib-
If the winery refuses to distribute the wine utor, is it likely to prevail?
through the distributor and the distributor then
sues the winery for breach of contract, is it likely (A) Yes, because the distributor did not termi-
that the distributor will prevail? nate the contract in good faith, because it
still had a requirement for the wine.
(A) Yes, because the winery’s performance was
to run to the distributor rather than to the (B) Yes, because, by paying the creditor, the
investor. distributor waived the breach that resulted
from the improper assignment.
(B) Yes, because the investor and the winery
could reasonably foresee that the distributor (C) No, because the vineyard’s rights under
would change his position in reliance on the its agreement with the distributor were
contract. personal and therefore nonassignable.

(C) No, because the investor and the winery (D) No, because the original contract between
did not expressly agree that the distributor the vineyard and the wine distributor was
would have enforceable rights under their unenforceable by either party for want of
contract. a legally sufficient consideration for the
vineyard’s promise to supply the wine
(D) No, because the investor and the winery, distributor’s requirements of wine.
having no apparent motive to benefit the
distributor, appeared in making the contract
to have been protecting or serving only
their own interests.
CONTRACTS - THIRD-PARTY BENEFICIARIES AND ASSIGNMENTS QUESTIONS 5.

Question 9 Question 10

A famous singer contracted with a fashion The father of a girl whose horse crashed
designer to design all of her clothing for public through a neighbor’s fence told the neighbor that
appearances for the next year. The singer’s he would have the fence repaired immediately at
boyfriend was an aspiring astrologer. To promote his cost. The father hired a local contractor to do
her boyfriend’s career, the singer included a the work. The contractor did a poor job and did
clause in the contract that the designer would use not fix the fence properly.
the boyfriend as his exclusive astrologer during
the term of their agreement. Six months into the Assuming that there was an enforceable
agreement, the singer and her boyfriend broke agreement between the father and the neighbor,
up. She told the designer that he did not have to would the neighbor have an action against the
use the boyfriend’s services any longer, so the contractor?
designer began consulting another astrologer.
The boyfriend then learned of the agreement (A) No, because there was no agreement be-
between the singer and the designer. tween the contractor and the neighbor with
regard to the repairs on the fence.
Can he enforce the agreement in his favor?
(B) No, because the neighbor was only an
(A) Yes, because he was an intended benefi- incidental beneficiary of the contract
ciary of the agreement between the singer between the contractor and the father.
and the designer.
(C) Yes, because the neighbor was a creditor
(B) Yes, because he had an enforceable contract beneficiary of the contract between the
with the designer concerning the astrolog- contractor and the father.
ical readings.
(D) Yes, because the neighbor was a donee
(C) No, because the singer and the designer had beneficiary of the contract between the
a right to modify their agreement without contractor and the father.
the boyfriend’s permission.

(D) No, because the boyfriend was a gratuitous


beneficiary whose rights depended on his
status as the singer’s boyfriend.
6. CONTRACTS - THIRD-PARTY BENEFICIARIES AND ASSIGNMENTS QUESTIONS

Question 11 Question 12

A pet shop entered into a written contract A store sold office equipment and supplies to
with a distributor of pet products, whereby the various businesses in the area. The store entered
distributor agreed to supply the pet shop with into a written agreement with an electronics
whatever quantity of pet food it might order, at company to purchase all of its monthly require-
a mutually agreed price, for two years, with an ments of printers for a period of five years at a
option to renew. Also, the contract required that specified unit price. The agreement contained
the pet shop buy its pet food from the distributor a clause prohibiting assignment of the contract.
only. For the first six months of the contract, the Shortly thereafter, the electronics company
pet shop ordered from the distributor 10 cases assigned the contract to a finance company
of pet food per month. In the seventh month, the as security for a loan. The store subsequently
owner of the pet shop sold the shop, inventory, ordered the printers from the electronics
and accounts receivable to a chain operation. company, which were delivered on time.
As part of the sale, the pet shop assigned to the
chain operation its contract with the distributor. Which of the following accurately states the
The chain operation promptly notified the legal effect of the clause prohibiting assignment
distributor of the sale and assignment. That same of the contract?
month, looking to stock the pet food in stores
throughout its chain, the chain operation sent (A) The clause as properly interpreted was not
the distributor an order for 5,000 cases of pet breached, and the assignment was effective.
food “on the terms and conditions of the agree-
ment expressed in the contract which has been (B) The clause made the assignment to the
assigned to us.” The distributor did not have the finance company ineffective.
means to fill such a large order and refused to
deliver 5,000 cases. (C) The electronics company’s assignment
was a breach but was nevertheless effec-
If the chain operation brings suit to compel tive to transfer to the finance company the
the distributor to fill the order for 5,000 cases of electronics company’s rights against the
pet food, who will prevail? store.

(A) The chain operation, because it gave the (D) The clause is effective if the parties can
distributor prompt notice of the assignment establish a rational reason for including the
from the pet shop it purchased. covenant into their agreement.

(B) The chain operation, because it would not


be impossible for the distributor to acquire
the quantity of pet food that the chain
operation requires.

(C) The distributor, because the quantity


required by the chain operation is too great
compared to the pet shop’s needs and, thus,
the assignment is not enforceable.

(D) The distributor, because there was no


mutuality of obligation in the original
agreement between the distributor and the
pet shop and hence there was nothing to
“assign” to the chain operation.
CONTRACTS - THIRD-PARTY BENEFICIARIES AND ASSIGNMENTS QUESTIONS 7.

Question 13 Question 14

A housepainter and a landlord of a two-flat On December 10, a housepainter entered


entered into a written agreement whereby the into a contract with a homeowner to paint her
painter would paint both apartments by the end house for $8,000, to be paid on completion of
of May for $2,000. The painter told the landlord the work. On December 20, before the work
that after he finished the job, the landlord should was completed, the painter sent a letter to the
pay the $2,000 to the paint store on Main Street, homeowner telling her to pay the $8,000 to
where the painter has an open account and owes an assignee, whom he identified by name. The
the store several thousand dollars. The landlord painter sent a copy of the letter to the assignee,
agreed. The painter informed the store owner then completed the work. The homeowner
of the arrangement. The store owner responded refused to pay the assignee, and the assignee
by saying that he did not care who paid him, sued the homeowner for the $8,000.
but the painter’s account was seriously overdue,
and if he did not get paid by June 1 he would Which of the following, if true, would be the
sue. Shortly thereafter, the painter and landlord homeowner’s best defense?
modified their agreement to push back the work
for several months and, as a result, the landlord (A) The assignee was not the intended benefi-
would not be paying the paint store the $2,000 ciary of the painter-homeowner contract.
or any part of it by June 1. The store owner
learned about this latest turn of events the next (B) The assignee was incapable of performing
day. When June 1 rolled around and neither the painter’s work.
party had paid the store owner the $2,000, the
store owner sued the landlord for that amount. (C) The painter had not performed his work in
a workmanlike manner.
In the suit between the store owner and the
landlord, who is likely to prevail? (D) The assignment was made without consid-
eration.
(A) The store owner, because he was informed
of the original agreement and did not par-
ticipate in the modification.

(B) The store owner, because he assented to the


original agreement.

(C) The landlord, because the original agree-


ment was modified before the store owner’s
rights became vested.

(D) The landlord, because his contract was with


the painter and not with the store owner.
8. CONTRACTS - THIRD-PARTY BENEFICIARIES AND ASSIGNMENTS QUESTIONS

Question 15 Question 16

A car dealer entered into a written agreement A bicycle shop proprietor and a customer
with a mechanic for the restoration of a classic entered into a valid contract under which the
car in exchange for $5,000. Among the specifi- customer was to purchase a racing bicycle to be
cations in the agreement was a provision that the delivered on May 20. The contract provided that
mechanic was to use cream-colored leather on the bicycle was to be delivered to the customer’s
the seats. There was no provision in the agree- nephew as a birthday present. The proprietor
ment regarding assignment. The mechanic began sent a photocopy of the contract to the nephew,
the job, and then assigned his right to payment which the nephew received on May 4. In antici-
under the agreement to a bank. When the job pation of receiving the new bicycle, the nephew
was a little more than half completed, the dealer immediately donated his old bicycle to a chari-
sold the car to a car collector. In a document table organization. On May 15, the customer
separate from the bill of sale, the dealer assigned told the proprietor that he wanted to cancel
her right to the mechanic’s labor and delegated the contract for the bicycle. The proprietor
her duty to pay under the contract to the car assented and promptly sold the bicycle to another
collector. The mechanic completed the job on customer at twice the price. The nephew called
time but used bisque-colored leather instead of the bicycle shop when the bicycle was not deliv-
cream. The bisque leather was virtually identical ered on May 20 and was told of the cancellation
in appearance to the cream and is considered of the contract.
equivalent in value by classic car enthusiasts.
On learning of the substitution, the car collector Can the nephew compel the customer to honor
refused to pay the bank. the original contract or pay him damages?

If the bank files suit against the car dealer for (A) Yes, because the nephew is an intended
payment, will the court find the car dealer liable beneficiary of a valid contract.
to pay the bank?
(B) Yes, because the nephew reasonably relied
(A) Yes, because the contract between the car on the contract between the proprietor and
dealer and the mechanic did not contain a the customer.
nonassignment clause.
(C) No, because the nephew was merely an
(B) Yes, because the assignment did not release incidental beneficiary.
her from her duties.
(D) No, because the nephew was a donee
(C) No, because the car dealer delegated her beneficiary and the customer properly
duty to pay to the car collector and the car revoked the gift.
dealer is no longer liable on the contract.

(D) No, because the restoration work was not


performed according to specifications.
CONTRACTS - THIRD-
PARTY BENEFICIARIES
AND ASSIGNMENTS
ANSWERS
CONTRACTS - THIRD-PARTY BENEFICIARIES AND ASSIGNMENTS ANSWERS 1.

CONTRACTS - THIRD-PARTY BENEFICIARIES AND ASSIGNMENTS ANSWERS

Answer to Question 1

(B) When an assignment has been made, but the obligor on the contract (here, the retailer) has not
been informed of the assignment, it is still obligated to pay the party with whom it originally
dealt. Here, the retailer paid the original supplier in full, and therefore the original supplier is
liable to the wholesaler for the full amount of payment, and (A) and (C) are therefore wrong. (D)
is wrong because the original supplier is liable to the wholesaler.

Answer to Question 2

(D) The new agreement between the businessman and the accountant would be most harmful to the
third party. The parties to a contract that benefits a third party may validly modify the contract
before the beneficiary learns of the contract and relies on it to his detriment. (A) and (C) are
incorrect because, if the businessman intended to confer a benefit on the third party by entering
into the contract with the accountant, the third party could enforce the agreement as a donee
third-party beneficiary whether or not an enforceable obligation existed between the businessman
and the third party. (B) is incorrect because a third-party beneficiary is not obligated to notify the
parties that he accepts the benefit of their agreement, especially when the agreement merely calls
for the payment of money to him.

Answer to Question 3

(C) The store owner must accept the competitor’s performance. The assignment by the contractor of
the contract involved a delegation of her duties to the competitor. While generally all contractual
duties may be delegated to a third person, duties involving personal judgment and skill may not be
delegated. However, a contract for paving generally would not be regarded as involving a personal
subject matter; hence, the contractor could delegate the duties to another competent contractor
without the store owner’s consent and the store owner must accept performance. Thus, (A) and (B)
are wrong. (D) is a misstatement of law. The contractor need not supervise. The store owner may
sue the contractor if the competitor fails to perform under the contract.

Answer to Question 4

(B) The buyer can sue only the contractor for breach. The effect of a valid assignment of contract
rights is to establish privity of contract between the obligor and the assignee while extinguishing
privity between the obligor and assignor. The assignee then replaces the assignor as the real party
in interest and he alone is entitled to performance under the contract. As the real party in interest,
he may enforce his rights against the obligor directly. Here, the original homeowner (the assignor)
has assigned to the buyer (the assignee) the right to receive the performance of the contract
by the contractor (the obligor). Because the contractor has materially breached the contract by
departing from the specifications and using a cheaper paneling, the buyer can sue the contractor
for breach. (A) and (D) are incorrect because the buyer has no grounds for suing the original
homeowner (the assignor). The assignor does not guarantee that the obligor will perform the
contract for the assignee. The assignor only warrants that (i) she has the right to make the assign-
ment, (ii) the right is not subject to limitations or defenses other than those stated or apparent at
the time of assignment, and (iii) she will do nothing to defeat or impair the assigned right. Here,
the homeowner did nothing to impair the assignment and the contractor does not appear to have
any defenses against the homeowner. Thus, the buyer cannot sue the homeowner. (C) is incorrect
2. CONTRACTS - THIRD-PARTY BENEFICIARIES AND ASSIGNMENTS ANSWERS

because the creditor has no duties under the contract; he just has the right to receive payment.
Unless a contrary intention appears, courts will construe language assigning “the contract” as
including an assumption of the duties by the assignee. Here, however, the contractor assigned
only his right to payment to the creditor and continued to perform the duties under the contract
himself. Thus, no delegation of duties will be implied and the creditor cannot be sued for breach.

Answer to Question 5

(B) The grocer has a cause of action against both orchards. The delegator remains personally liable
for performance of the agreement even though the delegatee is performing the contract. Hence,
(A) is incorrect. (C) is incorrect because although the grocer and the assignee orchard are not
in privity of contract, the grocer is a third-party beneficiary of the agreement between the two
orchards. When there is a delegation of duties, both the delegator and delegatee are liable for
performance of the agreement. Hence, (D) is incorrect.
Answer to Question 6

(C) The only theory under which the bank can recover the investor’s share of the profits is that
the bank is an assignee of the investor’s rights. (D) would not provide a strong defense for the
inventor, because it is clear that the bank was not an intended beneficiary of the investor-inventor
contract. (A) is incorrect because there is no presumption that an assignment is invalid absent
express authorization in the contract. (B) is incorrect because, even if the investor and the inventor
are partners, the investor would still be able to assign his rights to profits. (C) is the best answer,
because the bank’s only hope of prevailing is for it to be considered an assignee of the investor’s
rights. It appears that the written instrument executed by the investor lacks the present words of
assignment necessary to manifest the intent of the assignor to transfer his rights under the contract
completely and immediately to the assignee. The instrument merely allows the bank to collect the
debt from the investor’s share of the profits. Thus, the inventor has a fairly strong argument that
the bank is not an assignee of the investor’s rights under the investor-inventor contract.

Answer to Question 7

(D) The distributor is not an intended beneficiary of the contract between the investor and the winery.
Although the distributor was mentioned in the contract, it seems clear that the investor and the
winery did not intend to confer any benefits or rights on the distributor. Rather, the parties seem to
have been simply expressing a preference as to the distributor of the wine, with no indication that
the validity of the contract depended on use of the distributor. (A) is incorrect because the winery
was contractually bound to perform only with regard to the investor, rather than to the distributor.
(B) is incorrect because the investor and the winery could not foresee that the distributor would
act in reliance on a contract that conferred no rights on him. (C) is incorrect because an express
agreement between the investor and the winery would not have been necessary to a determination
that the distributor had some rights under the contract. For example, if the distributor stood in such
a relationship to the investor that one could infer that the investor wished to make an agreement
for his benefit, then it would be more likely that the contract was primarily for the benefit of the
distributor. (D) is the best answer because the key factor is that the investor and the winery were
simply protecting their own interests, with no thought of conferring a benefit on the distributor.

Answer to Question 8

(A) The creditor is likely to prevail because the creditor is the real party in interest and can enforce
the contract. The effect of an assignment is to establish privity of contract between the obligor
CONTRACTS - THIRD-PARTY BENEFICIARIES AND ASSIGNMENTS ANSWERS 3.

and the assignee while extinguishing privity between the obligor and assignor. The assignee then
replaces the assignor as the real party in interest, and the assignee alone is entitled to perfor-
mance under the contract. Under the UCC, a buyer in a requirements contract must act in good
faith. Here, when the distributor terminated the contract in order to purchase wine from another
vineyard, it still had a requirement for wine. Thus, the distributor did not act in good faith. (B)
is wrong because the vineyard did not breach the nonassignment clause by assigning its rights
under the contract. Absent circumstances suggesting otherwise, a clause prohibiting the assign-
ment of “the contract” will be construed as barring only the delegation of the assignor’s duties.
Here, the vineyard assigned only its right to payment and thus did not breach the nonassignment
clause. Therefore, the payments to the creditor could not be considered a waiver of the breach.
(C) is wrong because the rights involved here are clearly assignable, since the payment of money
does not involve personal responsibilities. (D) is wrong because the UCC regards requirements
contracts as being supported by sufficient consideration.

Answer to Question 9

(C) Because the boyfriend’s rights as a beneficiary had not vested, the singer and the designer had
a right to modify their agreement without his permission. A contract that benefits a third party
may confer rights on that party if he is an intended beneficiary rather than merely an incidental
beneficiary. The boyfriend is an intended beneficiary of the agreement between the singer and
the designer because (i) he was expressly designated in the contract, (ii) some performance is to
be made directly to him, and (iii) he stands in a close relationship to the promisee (the singer),
suggesting that she intended for him to benefit. However, an intended beneficiary can enforce the
contract only after his rights have vested. Vesting will occur when the beneficiary (i) manifests
assent to the promise in a manner invited or requested by the parties, (ii) brings suit to enforce the
promise, or (iii) materially changes position in justifiable reliance on the promise. The facts for
this question do not indicate any action on the boyfriend’s part that would have caused his rights
to vest before the change in the contract extinguished his rights. (A) is incorrect even though it
is a true statement. An intended beneficiary can enforce the contract only after his rights have
vested, and here the boyfriend’s rights had not vested. (B) is incorrect because the boyfriend had
no contract with the designer. Any rights that he had were through the contract between the singer
and the designer. (D) is incorrect. The boyfriend can be classified as a donee beneficiary because
the promisee (the singer) is gratuitously conferring a benefit on the third party (the boyfriend).
However, a donee beneficiary is still an intended beneficiary under the Restatement (Second) of
Contracts section 302. Had the boyfriend’s rights in the contract vested before they were extin-
guished, his break-up from the singer would not have prevented him from enforcing the contract.

Answer to Question 10

(C) The neighbor, as third-party creditor beneficiary of the agreement to repair the fence, has an
action against the contractor. A third party is a creditor beneficiary of a contract if the promisee’s
primary intent in contracting is to discharge an obligation to the third party. Here, the father
bargained with the contractor to do repair work on the neighbor’s fence in discharge of an obliga-
tion that the father had to the neighbor. Therefore, the neighbor was the third-party creditor
beneficiary of the agreement to repair the fence and could sue the contractor on this agreement.
(A) is wrong because a third-party beneficiary who is an intended beneficiary (i.e., one to whom
performance is given under the contract and upon whom the right to performance had been
conferred by the promisee) may sue the promisor on the contract even though there is no agree-
ment between the third-party beneficiary and the promisor. (B) is wrong because the neighbor
was an intended, not an incidental, beneficiary; the contract between the father and the contractor
4. CONTRACTS - THIRD-PARTY BENEFICIARIES AND ASSIGNMENTS ANSWERS

provided that performance be made directly to the neighbor in discharge of an obligation that the
father had to the neighbor (i.e., to fix his fence, which the father’s daughter had damaged). (D) is
wrong because the neighbor was a creditor, and not a donee, beneficiary; the father’s promise to
have the neighbor’s fence fixed was not gratuitous, but was in discharge of an obligation that the
father had to the neighbor.

Answer to Question 11

(C) The chain operation cannot compel the distributor to fill the order for 5,000 cases of pet food,
because that amount was not contemplated by the original parties to the contract and is grossly
disproportionate to the number of cases ordered by the pet shop during the first six months of its
contract with the distributor. An assignment of rights is barred if it will substantially change the
obligor’s duty. At common law, the right to receive goods under a requirements contract generally
was not assignable because the assignment could change the obligation of the parties. Under the
UCC, such a right is assignable if the quantity requirement is not unreasonably disproportionate
to the quantity originally contemplated by the parties or, in the absence of a stated estimate, any
normal or otherwise comparable prior requirements. [UCC §2-306] Here, the owner of the pet
shop owned only one store and his requirements for the first six months of the contract were only
10 cases per month. The chain operation, on the other hand, owned multiple stores and its require-
ments are 5,000 cases per month, a disproportionately large increase. Therefore, the assignment
here would be unenforceable even under the Code. (A) and (B) are incorrect because prompt
notice of the assignment, or the ability to secure the quantity of pet food required by the chain
operation, does not obviate the problem of the unreasonably disproportionate requirements. (D) is
incorrect because there is mutuality of obligation (consideration on both sides) here. The parties
entered into a requirements contract whereby the distributor agreed to sell the pet shop all of its
pet food requirements and the pet shop promised to buy its pet food only from the distributor. (Its
promise to buy only what it “might order” is not illusory because it agreed to not purchase pet
food from any other source.) A requirements contract is not illusory because the UCC imposes a
duty to purchase requirements in good faith.

Answer to Question 12

(A) The clause was not breached, and the assignment was effective. Under the UCC, which governs
this sale-of-goods case, “unless the circumstances indicate the contrary, a prohibition of assign-
ment of ‘the contract’ is to be construed as barring only the delegation to the assignee of the
assignor’s performance.” Here, the electronics company assigned to the finance company the right
to receive payment on its contract with the store. There was no delegation of duties to the finance
company (the assignee). Therefore, when the electronics company “assigned the contract” to the
finance company, it assigned only the right to payments, and it did not breach its contract with the
store. (B) is wrong because the only time a nonassignment clause makes an assignment ineffective
is when the clause itself states that any assignment will be void or when the assignee has notice
of the nonassignment clause. (C) is wrong because the assignment was not a breach. (C) is stating
the rule for when the contract contains a clause prohibiting the assignment of "contractual rights."
In that case, the assignment is effective, but the obligor may sue for breach. Here, the contract
prohibits assignment of the contract, not contractual rights. (D) is wrong because the covenant
would not stand or fall on its rationale.

Answer to Question 13

(B) The store owner will prevail because he assented to the original agreement. The store owner is a
third-party beneficiary to the contract between the housepainter and the landlord. A third-party
CONTRACTS - THIRD-PARTY BENEFICIARIES AND ASSIGNMENTS ANSWERS 5.

beneficiary can enforce the contract if his rights have vested. A modification of the contract can
take place without the consent of the third-party beneficiary prior to the time the third-party
beneficiary’s rights become vested. The rights become vested when the third party assents in
a manner requested by the parties, detrimentally relies on the contract, or brings a lawsuit to
enforce it. Here, the painter asked the store owner to assent and, although his answer may have
been grumbling, he assented. Because the store owner’s rights were then vested, the contract
could not have been modified without his consent, and he will be able to enforce the original
agreement. (A) does not go far enough; mere knowledge of the original arrangement is not enough
to vest rights. (C) is wrong because the modification occurred after the store owner’s rights were
vested. (D) is wrong because a third-party beneficiary can enforce a contract against a promisor.

Answer to Question 14

(C) The homeowner’s best defense is that the painter had not performed his work in a workman-
like manner. The painter has assigned his claim against the homeowner to an assignee. In the
absence of an enforceable agreement to the contrary, an assignee would take subject to defenses
good against his assignor. Therefore, if the painter has not properly performed the work, the
homeowner could use this as a defense in a suit brought by the assignee. (A) is wrong because the
assignee is not claiming he is a third-party beneficiary, but rather he is an assignee of the rights of
the painter. (B) is wrong because the fact that the assignee is incapable of performing the painter’s
work is immaterial. The painter merely assigned his right to payment; he did not delegate his
painting duties to the assignee. (D) is wrong because consideration is not required for an assign-
ment to be valid.

Answer to Question 15

(B) The car dealer will have to pay the bank because she remains liable on the contract. A transfer of
contractual duties to a third party is called a “delegation” of duties. However, the party delegating
her duties (the delegator-obligor) remains liable on her contract, even if the third person (the
delegate) expressly assumes the duties (i.e., promises that he will perform the duties delegated).
When the delegate promises that he will perform the duty delegated and the promise is supported
by consideration or its equivalent, an assumption occurs; the nondelegating party (the bank)
becomes a third-party beneficiary of the assumption agreement and can sue either the delegate
who assumed the duty (the car collector) or the delegator who remains liable on the contract
(the car dealer). In this case, therefore, the car dealer (the delegator-obligor) is still liable on the
contract even though she delegated to the car collector her duty to pay for the restoration. On the
other side of the contract, the bank is the assignee of the mechanic’s right to payment under the
contract and therefore can seek enforcement of the contract directly against the car dealer. (A)
is incorrect because a clause prohibiting assignment of “the contract” is generally construed as
barring only the delegation of the assignor’s duties, not the right to receive payment from the
obligor; thus, a general prohibition on assignments would not prevent the bank from recovering
against the car dealer. Furthermore, a provision in a contract stating specifically that contractual
rights may not be assigned is usually ineffective. In other words, the assignor retains the power
to assign even though he may be liable for breach of contract, and the assignee may enforce the
contract. Thus, the existence of a nonassignment clause in the contract between the car dealer
and the mechanic would not have deprived the mechanic of the power to assign his rights to the
bank. (C) is incorrect because, as discussed above, one who delegates her duties under a contract
remains liable on the contract. (D) is incorrect because the breach by the mechanic would be
considered a minor breach and would not relieve either the car collector or the car dealer of
their duty to pay. A breach of contract is minor if the obligee gains the substantial benefit of her
6. CONTRACTS - THIRD-PARTY BENEFICIARIES AND ASSIGNMENTS ANSWERS

bargain despite the obligor’s defective performance. While the aggrieved party may have a right
to any provable damages for the breach, she is not relieved of her duty of performance under the
contract. Hence, the car dealer will be liable to pay the bank for the work.

Answer to Question 16

(D) Despite his reliance, the nephew has no recourse against the customer. The nephew is a third-
party beneficiary of the contract between the proprietor and the customer because he is expressly
designated in the contract and performance is to be made directly to him. Because the object was
to arrange a gift for the nephew rather than to bring about payment of a debt owed to him by the
customer, the nephew is a donee beneficiary. A third-party donee beneficiary has no cause of
action against the promisee, because the promisee’s act is gratuitous and he may not be held to it.
The only time a donee benficiary has recourse against the promisee is when the promisee himself
tells the donee beneficiary about the contract and should foresee that the beneficiary would rely
on it. If the beneficiary does reasonably rely on the contract to his detriment, the beneficiary can
sue the promisee directly under a promissory estoppel, not a third-party beneficiary, theory. Here,
because the proprietor, and not the promisee (the customer), told the nephew about the contract, the
nephew, as a third-party donee beneficiary, has no cause of action against the customer. Thus, (B)
is incorrect. (A) is incorrect because, although the nephew is an intended beneficiary of the valid
contract, he is a donee beneficiary and, as explained above, has no claim against the promisee. (C)
is incorrect because the nephew was designated in the contract as the party to whom the bicycle
was to be tendered. Thus, he was an intended, rather than an incidental, beneficiary.
CRIMINAL LAW - ARREST,
SEARCH AND SEIZURE
QUESTIONS
CRIMINAL LAW - ARREST, SEARCH AND SEIZURE QUESTIONS 1.

CRIMINAL LAW - ARREST, SEARCH AND SEIZURE QUESTIONS

Question 1 Question 2

A police officer often visited an art gallery Based on a tip from a reliable informant
on his lunch break and became friends with that an attorney was illegally selling automatic
one of the gallery’s salespersons, whom he weapons and ammunition from his storefront
found to be honest and forthright. After hearing office, the police obtained a warrant to search for
rumors that the gallery’s owner sometimes weapons at the office. When they arrived at the
dealt in stolen artwork, the officer decided to building, they saw a client exiting the attorney’s
investigate. Acting without a warrant, he snuck office and placing what appeared to be a weapon
into a back room and discovered various crates inside his jacket. The police stopped the client
addressed to and from a country that was a on the street and an officer patted down his outer
well-known conduit for stolen artwork. The clothing. The officer felt no weapon but did feel
officer questioned his salesperson friend about a bag with several small tube-shaped objects
the crates. She told the officer that she never saw in it. She immediately placed the client under
what was inside any of the crates, but that she arrest. The contents of the bag were later deter-
assumed they contained artwork for the owner’s mined to be marijuana cigarettes.
private collection that he maintained at home.
The officer obtained a search warrant for the Prior to trial on the narcotics charge, the client
owner’s home based on the foregoing informa- sought to suppress introduction of the marijuana
tion. Upon executing the warrant, the officer as evidence. The arresting officer testified at
found several pieces of stolen artwork at the the suppression hearing that, based on her long
owner’s home. The owner was subsequently tried experience as a narcotics officer, she concluded
for receiving stolen property. The salesperson immediately that the bag contained marijuana
was to be called to testify about what she knew cigarettes when she first touched it.
about the gallery owner’s activities related to the
procurement of artwork. If the officer’s testimony is believed, the
motion to suppress the marijuana evidence
Is the salesperson’s testimony admissible? should be:

(A) No, because it is the fruit of the poisonous (A) Denied, because the search was incident to
tree. a lawful arrest.

(B) No, because she is an unindicted (B) Denied, because the police had a reasonable
co-conspirator. suspicion that the client might be armed
and dangerous.
(C) Yes, because she is unindicted and therefore
has no privilege against compelled self- (C) Granted, because the scope of an officer’s
incrimination. patdown during an investigatory detention
is limited to a search for weapons.
(D) Yes.
(D) Granted, because the search warrant did
not authorize the police to search the client
despite the fact that he was just present at
the place to be searched.
2. CRIMINAL LAW - ARREST, SEARCH AND SEIZURE QUESTIONS

Question 3 Question 4

A defendant was indicted in state court for While on routine patrol late one night, a
bribing a public official. During the course of the police officer noticed that a car was weaving
investigation, police had demanded and received recklessly across several lanes of traffic. He
from the defendant’s bank the records of the stopped the driver, believing that he was driving
defendant’s checking account for the preceding while intoxicated. By state law, the officer was
two years. The records contained incriminating empowered to arrest the driver and take him
evidence. On the basis of a claim of violation of to the nearest police station for booking. As
his constitutional rights, the defendant moves he approached the vehicle, the officer saw the
to prevent the introduction of the records in driver put what appeared to be a bottle in the
evidence. glove compartment. The officer arrested the
driver and then searched his vehicle. In the
Should his motion be granted? glove compartment, the officer discovered a
vial containing a small amount of cocaine. The
(A) Yes, because a search warrant should have driver was charged with possession of cocaine.
been secured for seizure of the records. At a preliminary hearing, the driver’s attorney
moves to prevent introduction of the cocaine into
(B) Yes, because the records covered such an evidence on the grounds that the search violated
extensive period of time that their seizure his client’s federal constitutional rights.
unreasonably invaded the defendant’s right
of privacy. Will this motion most likely be granted?
(C) No, because the potential destructibility of (A) No, because the officer was acting under a
the records, coupled with the public interest fear for his personal safety.
in proper enforcement of the criminal laws,
created an exigent situation justifying the (B) No, because the search was incident to a
seizure. constitutionally valid custodial arrest.

(D) No, because the records were business (C) Yes, because the officer needed a warrant to
records of the bank in which the defendant search the glove compartment.
had no reasonable expectation of privacy.
(D) Yes, because there was no reasonable or
proper basis on which to justify conducting
the search.
CRIMINAL LAW - ARREST, SEARCH AND SEIZURE QUESTIONS 3.

Question 5 Question 6

The police suspected that an 18-year-old Two police officers noticed a car was weaving
living with his mother was selling marijuana to and generally being driven in an erratic manner.
his high school classmates. The police went to They pursued the vehicle and curbed it. When
the house without a search warrant and secured the driver emerged from the car, he was
the mother’s permission to search the teen’s obviously intoxicated. The officers arrested him
bedroom. In the bedroom, they found a locked and put the driver in their squad car to take him
suitcase under the teen’s bed. The mother said to the station house. The driver’s wife was in the
that it was her son’s suitcase but that they could passenger seat, and she followed the police in the
search it. The police then broke the lock, opened car to the local precinct.
it and found a substantial amount of marijuana.
The police charged the teen with possession of As the driver was being booked, one officer
marijuana with intent to distribute it. took a standard police inventory sheet and began
searching the car. Beneath the passenger seat
If the teen should bring a pretrial motion to he found the passenger’s purse. He opened the
suppress the marijuana, how should the court purse and found a plastic zip-lock bag containing
rule on the motion? a small amount of marijuana. The passenger was
charged with possession of marijuana. Prior to
(A) Grant it, because the mother was not in- trial, her attorney moved to suppress the admis-
formed of her right to refuse consent to the sion of the marijuana seized from the passenger’s
search. purse into evidence.

(B) Grant it, because the mother’s consent to Should the court rule favorably on the motion?
search the suitcase was invalid.
(A) Yes, because when conducting a search in-
(C) Deny it, because the mother had authority cident to an arrest the police may not open
to consent to a search of anything within a closed container.
her home.
(B) Yes, because the police lacked probable
(D) Deny it, under the doctrine of in loco cause to search the passenger’s purse.
parentis.
(C) No, because the search was incident to the
lawful arrest of the driver.

(D) No, because the marijuana was discov-


ered during the course of a valid inventory
search.
4. CRIMINAL LAW - ARREST, SEARCH AND SEIZURE QUESTIONS

Question 7 Question 8

Two police officers stopped an automobile A boarding house owner smelled what she
for improperly proceeding through a red light. believed to be a suspicious odor coming from the
When one of the officers approached the car, room of a young boarder. She reported it to her
he observed some hand-rolled cigarettes on the daughter who was a police officer. Her daughter,
dashboard. He ordered the driver out of the car, suspecting that it was marijuana, told her mother
examined the cigarettes, and determined that to inspect the room the next day when the
they were joints of marijuana. The officer then boarder was at work, and if she found anything,
arrested the driver and guarded him in the back to bring it to her. The next morning, after the
seat of the police cruiser while the second officer boarder had gone, the owner took the master key
searched the entire car, including the trunk. In and went into his room. In a drawer, she found
the trunk, he found two rare paintings that had a box of tablets and some white powder that
recently been stolen from the city’s art museum. she took to her daughter. The narcotics division
confirmed that they were drugs. The daughter
The driver was charged with possession of had her mother return the items and swear out
stolen goods and brought a motion to suppress an affidavit. A search warrant was issued and
the introduction of the paintings into evidence. the police arrested the boarder and searched
his room. A quantity of drugs was found, and
If the driver’s motion is denied, it will be the boarder was charged with possession of
because the second officer conducted which type narcotics for sale.
of search properly?
If the boarder moves to prevent evidence of
(A) Search incident to an arrest. the drugs from being introduced at trial, what
should the court do?
(B) Automobile search.
(A) Grant the boarder’s motion, because once
(C) Custody search. the drugs had been taken to the police
department, they had to be kept there and
(D) Inventory search. could not be returned to the boarder’s room
for the purpose of finding them there.

(B) Grant the boarder’s motion, because his


legitimate expectation of privacy had been
unreasonably invaded.

(C) Deny the boarder’s motion, because the


owner, as owner of the boarding house, had
a legal right to inspect the rooms.

(D) Deny the boarder’s motion, because even


without the drugs found by the owner, the
police had reasonable cause to seek a search
warrant.
CRIMINAL LAW - ARREST, SEARCH AND SEIZURE QUESTIONS 5.

Question 9 Question 10

A man and a woman were traveling in the The defendant boarded a bus in New York for
man’s car when they were stopped by the police a trip to Florida to deliver two bricks of cocaine
for running a red light. Before the police came to a drug dealer. En route, in Georgia, an officer
up to the car, the man told the woman, “You owe for the state motor vehicle department stopped
me a favor. Keep this package for me,” and gave the bus in order to conduct a safety check of the
the woman a small foil package. The woman put bus. The officer was accompanied by a state
the package in her backpack, saying, “O.K., but trooper. While the motor vehicle officer was
don’t tell me what’s in it.” Before the police even conducting the safety inspection, the trooper
began to question the occupants, the man blurted boarded the bus. The trooper announced that, for
out, “I’m clean, man, but she has a stash,” homeland security purposes and to combat drug
pointing at the woman. The officers searched the smuggling, he would be asking for identification,
backpack that the woman was holding and found for the passengers to identify their luggage, and
the foil package, which contained heroin. The for permission to search inside the luggage. The
woman was arrested, but the man was not. trooper stated that the passengers had the right
to leave the bus or to decline to show what was
Is the evidence found on the woman admis- inside their luggage. The trooper then walked to
sible? the back of the bus to avoid blocking the aisle.
The trooper pulled down a bag that another
(A) Yes, under the automobile exception. passenger indicated belonged to the defendant.
The trooper then asked the defendant if the bag
(B) Yes, because due process imputes knowl- was his and whether he could search the bag,
edge where there is willful ignorance. and the defendant responded by nodding his
head.
(C) No, because due process forbids granting of
immunity to the more culpable defendant. The trooper opened the bag, found the two
bricks of cocaine, and immediately placed
(D) No, because the woman did not know that thedefendant under arrest and advised him of
the package contained heroin. his rights under Miranda.Approximately one
hour later, the motor vehicle officer finished his
inspection, issuing the drivera citation for failing
to keep his log book up to date. The defendant
was charged with possession ofcocaine with
intent to deliver. Before trial, the defendant
moves for suppression of the cocaine.

The most likely outcome is that:

(A) The cocaine will be suppressed as a viola-


tion of the Fourth Amendment.

(B) The cocaine will be suppressed as a viola-


tion of the Fifth Amendment.

(C) The cocaine will be suppressed as viola-


tions of both the Fourth and Fifth Amend-
ments.

(D) The cocaine will not be suppressed.


6. CRIMINAL LAW - ARREST, SEARCH AND SEIZURE QUESTIONS

Question 11 Question 12

A plainclothes police officer who frequently An officer went to an apartment to execute


ate lunch at a certain deli heard rumors that the a properly obtained search warrant during an
deli’s owner often placed illegal bets on sporting investigation of an operation making counterfeit
events. Based on the rumors, the officer peeked watches. When he arrived he shouted, “Police,
into an envelope next to the register and saw open up,” but he did not wait before entering
betting slips. The officer asked his waitress about the apartment through the unlocked front door.
the envelope, and she told him that the owner The officer found the defendant half-asleep in
had given it to her and that a man in a brown cap a back room, with a workbench for assembling
was to pick it up at 2 p.m. The officer stayed at counterfeit watches nearby. Along with some
the deli until 2 p.m. and watched the waitress completed counterfeit watches, the officer found
hand the envelope to a man in a brown cap. The a toolset used for making watches and receipts
officer passed the above information on to a for various watch components in a drawer of
friend in the F.B.I. Several weeks later, based on the workbench, all of which the officer seized
that information, F.B.I. agents obtained a search as evidence. The defendant was charged with
warrant for the owner’s home, described in the illegal counterfeiting. Prior to trial, his attorney
warrant as a condominium in a large multi- moves to suppress the evidence obtained during
unit complex identified by the street address the search.
only. The agents went to the home in the early
evening, while the owner was at the deli. After Should the court grant his motion?
announcing their purpose to the owner’s wife,
they searched the home and found betting slips (A) No, because regardless of whether the
and other materials related to illegal gambling. search warrant itself was valid, the evi-
dence was in “plain view” upon entry.
The owner was indicted for conspiracy to
violate a federal statute prohibiting the use of (B) No, because the exclusionary rule does not
interstate phone lines to conduct gambling, and apply to the officer’s Fourth Amendment
for the possession of betting slips. violation.

Which of the following would be the best (C) Yes, because the officer failed to wait long
reason for excluding the evidence found at the enough prior to entering the apartment.
owner’s home?
(D) Yes, because an unlocked door is not an
(A) The owner was not home when the warrant invitation to enter.
was executed.

(B) The search was conducted in the evening


when it easily could have been conducted
during daylight hours.

(C) The warrant failed to specify which condo-


minium unit was to be searched.

(D) The waitress had never been used before as


an informant.
CRIMINAL LAW - ARREST, SEARCH AND SEIZURE QUESTIONS 7.

Question 13 Question 14

A detective obtained a valid search warrant The police entered a neighborhood suffering
for the home of a man suspected of manufac- from increased illegal drug activity, accom-
turing methamphetamine and counterfeiting. panied by a dog trained to sniff out cocaine.
The man was not at home, but his wife told the The police entered the backyard of a home and
detective that he was at their weekend cabin. brought the dog to an area immediately outside
After announcing his purpose, the detective the back door. The dog acted as if it smelled
searched the home and found a large sum of cocaine. The officers knocked on the back door
cash, along with a printing plate for $20 bills and and a man answered the door and let them
several large stacks of fresh $20 bills. There- in. He was immediately placed under arrest.
after, the detective went to the man’s cabin with After a brief search, the police officers found
an arrest warrant in hand. After knocking on the and confiscated a small quantity of cocaine
door and announcing his presence, the detective from the bedroom closet. The man is charged
found that the door was unlocked and entered with possession of cocaine. At trial, he moves
the cabin. The man was not in the cabin, but the to prevent introduction of the cocaine into
detective found several boxes of the over-the- evidence.
counter drugs and other ingredients for making
methamphetamine in a small footlocker. The Will this motion most likely be granted?
man was indicted for various criminal offenses,
including the illegal manufacture of metham- (A) Yes, because, under the circumstances, the
phetamine and counterfeiting. police activity violated the man’s reason-
able expectation of privacy.
Was the search at the man’s cabin following
the search of his home valid? (B) Yes, because this kind of detection by
a trained dog has not been scientifi-
(A) Yes, based on the wife’s consent. cally verified and cannot be the basis for
probable cause.
(B) Yes, as a search incident to arrest.
(C) No, because the man allowed the police
(C) No, because the detective lacked probable officers to enter his home.
cause.
(D) No, because the search was incident to a
(D) No, because the detective did not have a valid arrest.
warrant.
CRIMINAL LAW - ARREST,
SEARCH AND SEIZURE
ANSWERS
CRIMINAL LAW - ARREST, SEARCH AND SEIZURE ANSWERS 1.

CRIMINAL LAW - ARREST, SEARCH AND SEIZURE ANSWERS

Answer to Question 1

(D) The salesperson’s testimony will likely be admissible. Although the discovery of the crates as a
result of an illegal search may be said to have led to the salesperson’s testimony, her testimony
will not be excluded as the product of illegal police activity. Under the exclusionary rule, not
only must illegally obtained evidence be excluded, but also all evidence obtained or derived
from exploitation of that evidence must be excluded. Such derived evidence is called “the fruit
of the poisonous tree.” Despite this general rule, it is difficult for a defendant to have live witness
testimony excluded as the fruit of the poisonous tree, because a more direct link between the
taint and the evidence is required than for exclusion of other evidence. Among the factors that
a court considers in determining the existence of a sufficiently direct link is the extent to which
the witness is freely willing to testify. Here, the officer did not have a search warrant when he
looked into the back room. The gallery owner will argue that, because the officer’s questioning
of the salesperson followed the illegal search of the back room, the testimony of the salesperson
is derived from exploitation of the evidence found in the back room. However, there is no indica-
tion from the facts of such a direct link between the illegal conduct of the officer and the testi-
mony as would be required to exclude the testimony. Even prior to seeing the crates in the back
room, the officer was aware of rumors regarding the gallery owner’s illegal activity. Thus, it is
entirely possible that at some point the police would have questioned the salesperson as to her
knowledge of this matter and obtained from her the same information that now forms the basis
of her testimony. Consequently, the link between the proffered testimony and the illegal police
conduct is insufficiently direct to render the testimony inadmissible. Because the gallery owner
will be unable to have the salesperson’s testimony excluded as the fruit of the poisonous tree, (A)
is incorrect. (B) is incorrect for two reasons: First, there is no indication that the salesperson was
a co-conspirator with the gallery owner. Second, even if the salesperson were a co-conspirator,
there is no principle of law that prohibits an unindicted co-conspirator from testifying. (C) is
incorrect because a person has a Fifth Amendment right to refuse to testify whenever the testi-
mony may tend to incriminate. A nondefendant witness may not use the privilege to avoid being
sworn as a witness or to avoid being asked questions, but may refuse to answer specific questions
while on the stand. In any case, the rationale here—that the Fifth Amendment privilege does not
apply—is incorrect.

Answer to Question 2

(B) The client’s motion should be denied because the seizure of the marijuana was properly within
the scope of the stop and frisk. A police officer may stop a person without probable cause for
arrest if she has an articulable and reasonable suspicion of criminal activity. [Terry v. Ohio
(1968)] In such circumstances, if the officer reasonably believes that the person may be armed and
dangerous, she may conduct a protective frisk. The scope of the frisk is limited to a patdown of
the outer clothing for concealed instruments of assault, but the officer may reach into the suspect’s
clothing and seize any item that the officer reasonably believes, based on its “plain feel,” is a
weapon or contraband. [Minnesota v. Dickerson (1993)] Here, the officer believed that the client
put a weapon in his jacket as he was leaving a place where weapons and ammunition were being
sold illegally; thus, she had reasonable grounds to conduct both a stop and a frisk. If the court
accepts the officer’s testimony that she instantly recognized the marijuana cigarettes based on the
patdown only without any further conduct, they were properly seized and can be admitted into
evidence. (A) is incorrect because the client was not under arrest at the time the patdown disclosed
the marijuana. While the police may conduct a full search incident to a lawful arrest, they had
一 信:liuxue119118 , 们 信免 供

2. CRIMINAL LAW - ARREST, SEARCH AND SEIZURE ANSWERS

only detained the client for purposes of an investigatory detention at the time of the seizure. (C) is
incorrect because, as discussed above, a frisk for weapons also allows an officer to seize contra-
band if she immediately recognizes it as such. (D) is incorrect because the police did not need to
rely on the search warrant to search the client; for the limited stop and frisk that occurred here,
the police need only a reasonable suspicion of criminal activity and a reasonable belief that the
suspect is armed and dangerous.

Answer to Question 3

(D) The defendant’s motion should be denied. To have a Fourth Amendment right, a defendant must
have a reasonable expectation of privacy in the object seized. A person does not have such an
expectation in objects held out to the public, such as account records held by a bank. Thus, (D)
is correct. (A) is incorrect because a search warrant is not required where the defendant does not
have a Fourth Amendment right. (B) is incorrect because the defendant did not have a Fourth
Amendment right in the records. Also, the period covered by the records was not necessarily
overextensive. It could be that the defendant was suspected of being involved in bribery for the
preceding two years. (C) is incorrect because, even if the facts here presented describe an exigent
situation, such a situation is actually an exception to the warrant requirement; and, like the
warrant requirement, will not arise unless and until the defendant has a Fourth Amendment right.

Answer to Question 4

(B) The motion will be denied because the search was incident to a constitutional arrest. After
arresting the occupant of an automobile, the police may search the interior of the auto if the police
reasonably believe that evidence of the offense for which the person was arrested may be found in
the vehicle. The court is likely to find that, based on what he saw, the officer reasonably believed
that he would find evidence of intoxication (given that he put a bottle in his glove compartment
after being pulled over for weaving across lanes of traffic). Thus, (B) is correct and (D) is wrong.
(A) is wrong because the police officer need not fear for his safety to make a valid search incident
to a constitutional arrest under the circumstances above. (C) is wrong because a valid search
incident to arrest encompasses the area within the defendant’s “wingspan.” This includes the glove
compartment.

Answer to Question 5

(B) The teen’s motion should be granted. A search is valid if the police reasonably believe that they
obtained valid consent for the search. Generally, a homeowner has authority to consent to a
search of her own home, as long as she has apparent access to the place searched. Here, nothing
indicates that the teen’s bedroom door was locked. Thus, the mother had apparent authority to
search the teen’s room. However, it appears unreasonable for the police to believe that the mother
had authority to consent to a search of the suitcase because it was locked, and the police had to
break the lock to open it (indicating that the mother did not have a key for the lock and so likely
did not have access to the contents of the suitcase). The parent-child relationship does nothing to
change this analysis. A parent of an adult teenager does not have automatic authority to search
the adult teen’s locked things (the rule probably is different for young children). (A) is incorrect.
To be effective, a consent to search must be voluntary and given under no threat or compulsion.
However, the police are under no obligation to inform the defendant or other person of the right
to refuse entry. Therefore, this search is not invalid because the mother was not informed of her
right to refuse entry. The issue is whether her consent was sufficient to allow the police to search
the son’s suitcase. (C) is incorrect because, as explained above, a homeowner does not necessarily

一 信:liuxue119118 , 们 信免 供
CRIMINAL LAW - ARREST, SEARCH AND SEIZURE ANSWERS 3.

have authority to consent to a search of everything in her home. Where it is apparent that she does
not have access (e.g., when something is locked and she does not have a key), the police may not
reasonably believe she may consent to the search of the locked area. (D) is incorrect. The doctrine
of in loco parentis is a tort doctrine under which the state is charged with a parent’s responsibili-
ties over a child in its care. The doctrine does not apply here.

Answer to Question 6

(B) The motion to suppress the evidence will be granted because the police did not have probable
cause to search the car. When the police place the driver of an automobile under arrest, there are
a number of alternatives with respect to a search of the car: (i) They may conduct a search of the
passenger compartment incident to arrest if the search is contemporaneous with the arrest and
either the arrestee may still access the vehicle or the police reasonably believe that the vehicle
contains evidence of the crime for which the arrest was made. Here, the search was not contempo-
raneous and nothing indicates that either of the other two conditions was satisfied. (ii) If the police
have probable cause to search the car—i.e., reasonable grounds for believing that a legitimate
item of seizure is in the car—a search of the entire car can be made without a warrant. The search
based on probable cause can be made at the time of the arrest or at a later time. In this question,
the search of the car at the police station would have been valid if the police had probable cause
to search the car. The question does not provide any facts that could form the basis for probable
cause to search. Thus, the motion to suppress will be granted because of the lack of probable
cause. (iii) If the police take the car under their control for an administrative reason (such as to get
the car off the highway), they can “inventory” the items in the car under certain circumstances.
In this question, the car had not been impounded by the police, nor would it need to be because
the passenger could drive it home. (A) is wrong for two reasons: The search could not be justified
as a search incident to an arrest because the arrest had been completed. Also, when conducting a
valid search incident to an arrest, the police may open up a closed container within the arrestee’s
“wingspan.” (C) is wrong because, as stated, a search incident to an arrest must occur at the time
of the arrest. (D) is wrong. The police can conduct an administrative inventory if the car has been
impounded by the police, and the police are, in fact, conducting an inventory of the items in the
car. As discussed above, however, the police did not impound the car.

Answer to Question 7

(B) Because of a lessened expectation of privacy in a car (as compared to a home) and because of the
inherent mobility of a car which can prevent the police from easily obtaining a search warrant
and searching it, the police are permitted to make a complete search of an automobile if there is
probable cause to believe that the car contains the fruits, evidence, or instrumentalities of a crime.
[Carroll v. United States (1925)] When the police have probable cause to search an entire vehicle,
they may conduct a warrantless search of every part of the vehicle and its contents. [United States
v. Ross (1982)] While it is doubtful that the marijuana joints provided sufficient probable cause
to search the entire vehicle, this is the best reason to deny the motion, because all of the other
choices are clearly incorrect. (A) is incorrect because a lawful search of a vehicle incident to an
arrest includes a search of the defendant and, in limited circumstances, the area of the vehicle
within the defendant’s control. [Arizona v. Gant (2009)] In this case, as the trunk is outside the
defendant’s area of control, the search went beyond the scope of a proper search incident to an
arrest. Therefore, this was an invalid search incident to arrest and the driver’s motion would be
granted on this basis. (C) is incorrect. Technically, there is no such thing as a custody search.
“Custody” here may refer to custody of the person, in which case it is a search incident to arrest
discussed above, or custody of the vehicle, which is an inventory search. An inventory search
4. CRIMINAL LAW - ARREST, SEARCH AND SEIZURE ANSWERS

occurs when the officers properly impound a car and search it while it is in their control to make
sure that the assets of the owner of the car are fully accounted for. [South Dakota v. Opperman
(1976)] This choice is incorrect because it does not articulate a legal basis to conduct the search,
as the car was not impounded. (D) is incorrect. In this case, the police did not take possession of
the car, so this is not the best reason to deny the motion.

Answer to Question 8

(B) The court should grant the boarder’s motion, because the search violated his Fourth Amend-
ment rights. The boarder had an expectation of privacy with respect to the room he rented from
the owner, and the police had no reason to believe that the owner was authorized to invade his
privacy. Because the search took place without a warrant and violated his reasonable expectation
of privacy, he can have the evidence obtained as a result of the search excluded from trial. (C)
is wrong because although the owner, as the owner of the building, may have had limited rights
to inspect a boarder’s room, she is not authorized, at the instructions of the police, to go through
the private property of her boarders. Because she was acting at the directions of the police, her
conduct will be treated as government conduct for purposes of the Fourth Amendment. (D) is
wrong because the owner’s testimony concerning a “suspicious odor” would not be sufficient to
uphold a search warrant. (A) is wrong; there is no such rule of law. Nor would returning the drugs
to the room for the purpose of finding them there remove the taint from the original illegal search.

Answer to Question 9

(A) The evidence is admissible because the search was valid. Even though the police have validly
stopped an automobile, they cannot search the vehicle without meeting the requirements of one
of the exceptions to the warrant requirement, such as the automobile exception (which requires
probable cause) or consent. The automobile exception comes into play when the police have
probable cause to believe that the vehicle contains evidence of a crime. Under the exception, the
police may search anywhere in the vehicle in which the item for which they have cause to search
may be hidden, including packages in the vehicle. The statement of the man to the officers gave
them probable cause to believe that the car contained evidence of a crime (i.e., that the woman
had drugs somewhere in the car). Thus, the requirement for application of the automobile excep-
tion was present, providing validity for the warrantless search conducted by the police. Because
the search was valid, the evidence found on the woman is admissible. Besides being an incorrect
statement of law, (B) is incorrect because it focuses on the woman’s knowledge of the contents of
the package. Whether the woman knew that heroin (or some other illegal substance) was in the
package is irrelevant to the admissibility of the heroin. Even assuming that the woman knew of
the contents, the search would not be valid unless there was a ground for the warrantless search.
(D) similarly links the woman’s knowledge of the package’s contents to the admissibility of
the evidence. As noted above, the admissibility of the evidence is dependent on the validity of
the search that produced the evidence, rather than on the knowledge of the defendant as to the
existence of the evidence. (C) is incorrect for three reasons: First, due process does not prohibit
granting of immunity to a more culpable defendant. Second, there is no indication that immunity
was even granted here (immunity from prosecution may be granted to compel a witness to answer
questions. The facts merely state that the man was not arrested; this does not necessarily mean
that he was granted immunity). Third, the call of the question relates to the admissibility of
the evidence, and a grant of immunity does not relate to the question of the admissibility of the
evidence found on the woman; such admissibility is determined by the validity of the search of
the woman by the officers.
CRIMINAL LAW - ARREST, SEARCH AND SEIZURE ANSWERS 5.

Answer to Question 10

(D) The cocaine will not be suppressed. The issues are whether the defendant was unlawfully seized,
thus making the cocaine the fruit of an illegal seizure of the person, whether the trooper needed
a warrant before searching the baggage, and whether the trooper was required to advise the
defendant of his rights under Miranda prior to asking him if the bag belonged to him. Whether a
seizure occurs depends on whether, under the totality of the circumstances, a reasonable person
would feel free to decline the officers’ requests or otherwise terminate the encounter. In fact, the
Supreme Court has held that an officer boarding a bus does not constitute a seizure of the person
so long as a reasonable person would believe that he was free to leave. In the instant case, the
trooper boarded the bus, explained that the passengers were free to leave if they so desired, and
then moved to the back of the bus to avoid blocking the aisle. Under Supreme Court precedent,
this would not constitute a seizure of the person, as a reasonable person would know that he is
free to leave. Thus, the cocaine will not be suppressed as the fruit of an illegal seizure of the
person. The next issue is whether the trooper needed a warrant before conducting a search of
the defendant’s luggage. To be reasonable under the Fourth Amendment, most searches must be
pursuant to a warrant. A search conducted without a warrant will be invalid (and evidence discov-
ered during the search must be excluded from evidence) unless it is within one of the permissible
warrantless searches. One such exception is when the officers conducting the search have volun-
tary and intelligent consent to do so. In the instant case, the officer informed all bus passengers
that he might ask them to open up their luggage, and he informed them of their right to decline.
Thus, the defendant’s consent should be deemed “voluntary and intelligent.” As a result, no viola-
tion of the Fourth Amendment occurred, which makes (A) and (C) incorrect answers. As to the
Fifth Amendment argument, a person in custody must be informed that: (i) he has the right to
remain silent; (ii) anything he says can be used against him in court; and (iii) if he cannot afford
an attorney, one will be appointed for him if he so desires. The threshold requirement is that a
person be in custody before the Fifth Amendment privilege against compelled self-incrimination
is triggered. In the instant case, the trooper made it clear that each passenger was free to leave,
and the trooper took steps not to block the exit. Thus, the defendant here was not in custody, and
no Miranda warnings were required. Thus, even assuming that a court would exclude evidence
under the fruits of the poisonous tree doctrine (and it is not clear that they would have to do so)
based on a failure to give Miranda warnings, the warnings were not required under these facts.
Thus, (B) is incorrect, and this provides another reason for (C) being incorrect.

Answer to Question 11

(C) If the warrant was invalid, any evidence obtained thereunder will be excluded. A warrant must
be based on a showing of probable cause. When requesting a warrant, officers must submit to a
magistrate an affidavit setting forth sufficient underlying circumstances to enable the magistrate
to make a determination of probable cause, independent of the officers’ conclusions, that evidence
of a crime will be found at the premises. Also, a warrant must describe with reasonable precision
the place to be searched and any items to be seized. A finding that a warrant was invalid because
it was not supported by probable cause will not entitle the defendant to exclude evidence obtained
under the warrant if the police have acted in good faith and reasonable reliance on a facially valid
warrant. However, a police officer cannot in good faith rely on a defective search warrant if: (i) the
affidavit underlying the warrant is so lacking in probable cause that no reasonable police officer
would have relied on it; (ii) the warrant is defective on its face (e.g., it fails to state with particu-
larity the place to be searched or the things to be seized); (iii) the affiant lied to or misled the
magistrate; or (iv) the magistrate has wholly abandoned his judicial role. Because the deli owner
lives in a condominium, the warrant should have specified which unit in the multi-unit dwelling
6. CRIMINAL LAW - ARREST, SEARCH AND SEIZURE ANSWERS

was to be searched. Because the warrant did not so specify, it failed to describe with sufficient
particularity the place to be searched. Such an absence of precision renders the warrant defective
on its face, so that the F.B.I. agents cannot be said to have relied in good faith on a facially valid
warrant. Thus, the evidence obtained pursuant to this facially defective warrant will be excluded.
(A) and (B) each incorrectly imply that there was something wrong with the execution of the
warrant. A warrant must be executed by the police without unreasonable delay, with the police
knocking and announcing their purpose (unless they reasonably believe that such notice will
endanger them or lead to the destruction of evidence). The F.B.I. agents conducted themselves in
accordance with these standards. The fact that a warrant was executed in the evening or at a time
when a particular person was not on the premises will not invalidate a search conducted pursuant
to the warrant. Here, the agents announced themselves to the deli owner’s wife, and they were not
required to wait until the owner returned in order to conduct their search. (D) is incorrect because
it states a fact that will not, in and of itself, invalidate a warrant and a search pursuant thereto.
The sufficiency of a search warrant affidavit based on an informer’s hearsay is evaluated under
the totality of the circumstances. The informer’s reliability and credibility, as well as her basis
of knowledge, are all elements that may illuminate the issue of probable cause, but they are not
strictly separate requirements. Had the waitress been used before as an informant, and been previ-
ously found to be reliable, this would have been one factor in determining her present reliability.
However, the fact that the waitress had not previously served as an informer does not invalidate
the warrant. Given the specificity and accuracy of the waitress’s information (a man in a brown
cap would pick it up at 2 p.m.), there was enough of a basis for her information to establish suffi-
cient credibility to permit a magistrate to make a determination of probable cause.

Answer to Question 12

(B) The evidence is admissible because the exclusionary rule does not apply to violation of the “knock
and announce” rule. In executing a search warrant, police officers must knock and announce
their authority and purpose, and wait a reasonable time for an occupant to respond, except when
there is reasonable suspicion, based on facts, that the announcement would be dangerous or futile
or would inhibit the investigation (e.g., lead to destruction of evidence). [Richards v. Wisconsin
(1997)] Here, the officer knocked and announced his presence, but did not wait for the door to be
answered. Nothing in the facts indicates that the officer had any reason to suspect that waiting
would endanger him or lead to the destruction of evidence. Thus, the knock-and-announce
requirements were violated. However, the Supreme Court has held that the exclusionary rule does
not apply to knock-and-announce violations. [Hudson v. Michigan (2006)] Therefore, although
the knock-and-announce rule may have been violated, the evidence may nonetheless be admitted
into evidence because the search was otherwise valid. (C) is incorrect in that it states the rule
prior to Hudson v. Michigan. Although the officer may have violated the knock-and-announce
rule by failing to wait for a sufficient period before opening the door, the remedy for such a viola-
tion is not the suppression of the evidence. (A) is incorrect because the plain view exception to
the warrant requirement applies only when the police: (i) are legitimately on the premises; (ii)
discover evidence, fruits or instrumentalities of crime, or contraband; (iii) see such evidence in
plain view; and (iv) have probable cause to believe that the item is evidence, contraband, or a fruit
or instrumentality of crime. Because of the officer’s failure to knock and announce, he was not
legitimately on the seller’s premises. Thus, the seizure cannot be justified under the plain view
exception. (D) is incorrect because, while it might be a true statement, it is not relevant to the
suppression issue. The officer had a warrant and did not need permission (an invitation) to enter.
CRIMINAL LAW - ARREST, SEARCH AND SEIZURE ANSWERS 7.

Answer to Question 13

(D) The search that resulted in the seizure of the methamphetamine supplies at the cabin is invalid
because the search was executed without a warrant, and no exception to the warrant require-
ment is applicable. All warrantless searches are unconstitutional unless they fit into one of the
six recognized exceptions to the warrant requirement. To be valid, a warrantless search must
meet all the requirements of at least one exception. These exceptions include search incident to a
lawful arrest and search based on consent. When the officer conducted the search of the cabin, he
had no warrant for the search; he only had an arrest warrant. Thus, the validity of the search and
seizure depends on whether an exception to the warrant requirement is applicable. (A) is incorrect
because the wife’s statement that the man was at the cabin did not constitute consent to search
the cabin. Even though she had authority to give consent to the search, the search and seizure
was not justified under the consent exception. (B) is incorrect because the detective was not
engaged in arresting someone, which would have permitted him to search the arrestee and areas
into which he might reach to obtain weapons or destroy evidence, given that the man was not on
the premises. (C) is incorrect because it is irrelevant. It implies that if the detective had probable
cause, he could have executed the warrantless search. Probable cause alone is not grounds to
conduct a warrantless search.

Answer to Question 14

(A) The man’s motion will be granted because the police officers violated the man’s reasonable
expectation of privacy. Dog sniffs near the entry of a home or within the curtilage of a home are
considered searches within the meaning of the Fourth Amendment and therefore require a valid
warrant. The police officers here conducted the dog sniff in the man’s backyard and immediately
outside his back door. The arrest of the man was therefore invalid because it was based on an
unconstitutional warrantless search. Even if the arrest were valid, the search of the bedroom closet
would nevertheless be invalid. Incident to an arrest, the police may search the person and areas
into which he might reach to obtain weapons or destroy evidence (his “wingspan”). The police
may also make a protective sweep of the area beyond the defendant’s wingspan if they believe
dangerous accomplices may be present. Here, the bedroom closet was not within the man’s
wingspan (unless the back door was to the man’s bedroom), and nothing indicates a reason for the
officers to believe that dangerous accomplices were present. Therefore, the search could not have
been valid even if the arrest were valid. (B) is wrong. There is no such rule of law. (C) is wrong;
allowing the police to enter the house is no “consent” to search the bedroom closet. (D) is wrong;
as discussed above, the closet was outside the area of the immediate control of the defendant, and
there was no indication that there were accomplices present in the house. The search therefore
cannot be justified as a search incident to an arrest.
CRIMINAL LAW -
HOMICIDE QUESTIONS
CRIMINAL LAW - HOMICIDE QUESTIONS 1.

CRIMINAL LAW - HOMICIDE QUESTIONS

Question 1 Question 2

A gangster was threatening to inflict a “disfig- A hunter drove to one of his old spots for
uring injury” on a shopkeeper if the shopkeeper target shooting. When he arrived at the spot, he
didn’t accept “gang protection.” The shopkeeper noticed that an adjacent area that had formerly
refused to pay and told the gangster to get out been an open field now contained a new housing
of his store. The gangster pulled out a switch- tract with a playground where children were
blade and he flipped it at the shopkeeper. The playing. The hunter tacked a paper target to a
knife thunked into the wall a few centimeters tree that was located between the playground
from the shopkeeper’s left ear. “Oops. I slipped,” and the point from which he planned to shoot.
said the gangster. “This is your last chance. He fired a number of shots at the target, and all
Without my protection, you’re a dead man.” The but one hit the target. The one shot that missed
gangster turned and began to walk from the that went wide of the target, ricocheted off a
store. The shopkeeper took a revolver from under tree, and flew into the playground, striking a
the cash register, aimed it at the gangster, and child in the head, killing the child.
shot. However, the shopkeeper missed, killing a
delivery person instead. If the hunter is tried for the death of the child,
what is the most serious crime of which he can
At the shopkeeper’s subsequent trial for be convicted?
the murder of the delivery person, which of
the following is the strongest argument that (A) Murder, because the hunter’s intent to
the shopkeeper can only be convicted of shoot the child can be inferred from his
manslaughter? conduct.

(A) The shopkeeper did not intend to kill or (B) Murder, because the hunter acted with
seriously injure the delivery person. wanton recklessness when he chose to shoot
the gun in an area near the playground.
(B) The shopkeeper was acting in self-defense
when he shot at the gangster. (C) Manslaughter, because the hunter did not
intend to kill the child.
(C) The shopkeeper was acting under duress
when he shot the delivery person. (D) Neither murder nor manslaughter, because
the hunter’s shooting was negligent, and
(D) The shopkeeper was reacting to a sufficient mere negligence is insufficient to support
provocation to mitigate the homicide. conviction for a homicide crime.
2. CRIMINAL LAW - HOMICIDE QUESTIONS

Question 3 Question 4

While walking down the street, a pedestrian A defendant was charged with criminal
accidentally bumped into a street performer. homicide after shooting the victim, his former
The performer told the pedestrian to pay atten- lover, intending to kill him. The victim had
tion to where she was going. The pedestrian got decided to leave the defendant and move in with
mad and slapped the performer across the face. an old boyfriend. The victim was not severely
The performer, afraid that the pedestrian would wounded, but he was hospitalized for several
continue the assault and escalate it to the point days. During his stay at the hospital, it was
of murdering him, pulled out a gun and shot the discovered that the victim had developed a form
pedestrian, killing her instantly. of cancer that was invariably fatal. Three weeks
after being shot, the victim died of pneumonia
In what type of state is the performer most caused by an infection resulting from the bullet
likely to be convicted of manslaughter? wound in his neck. Two days after the defen-
dant shot the victim, a fire burned down the
(A) A state that recognizes the heat of passion boyfriend’s condo and killed him as he slept.
resulting from adequate provocation as a
mitigating circumstance. Is the defendant guilty of a criminal
homicide?
(B) A state that applies the doctrine of dimin-
ished responsibility. (A) No, because his actions merely constituted
attempt.
(C) A state that recognizes the misdemeanor-
manslaughter rule. (B) No, because the victim was already
suffering from an incurable disease.
(D) A state that recognizes the imperfect self-
defense doctrine. (C) No, because the defendant actually
prolonged the victim’s life because he
prevented his probable death in the fire that
destroyed the boyfriend’s condo.

(D) Yes, because the defendant’s act was the


proximate cause of the victim’s death.
CRIMINAL LAW - HOMICIDE QUESTIONS 3.

Question 5 Question 6

A woman had a history of epileptic seizures A father was terminally ill with a particularly
which came without warning over the last five painful form of cancer. His daughter visited him
years. One day, as she was driving to town, she every evening in the hospital and for several
had a seizure. She lost control of her car and months listened to his pleas to put him out of his
struck a pedestrian who was lawfully crossing misery. On her final visit, she gave her father a
the street. The pedestrian died immediately hug and then pulled a small revolver from her
thereafter as a result of his injuries. The woman purse. She fired a shot at her father, killing him
is charged with involuntary manslaughter in a instantly. The daughter immediately broke into
jurisdiction that follows common law principles. tears and surrendered to the police. The daughter
was charged with her father’s death.
Will she most likely be found guilty?
What is the most serious offense of which the
(A) Yes, because the law presumes that a per- daughter can be convicted?
son intends the natural and probable conse-
quences of her acts. (A) First degree murder, defined by the juris-
diction as premeditated and deliberate kill-
(B) Yes, because she knowingly and negligently ing of another human being.
chose to drive her car.
(B) Second degree murder, defined as any
(C) No, because her failure to control the car murder not classified as first degree murder.
was not a voluntary act.
(C) Voluntary manslaughter.
(D) No, because she had no intent to harm
anyone. (D) No crime.
4. CRIMINAL LAW - HOMICIDE QUESTIONS

Question 7 Question 8

A wife found some letters in her husband’s Two partners who operated an electronics
dresser that disclosed that her husband was retail store hired a thug to intimidate the owner
having an extended affair with her best friend. of a discount electronics store that was undercut-
Furious, the wife purchased a handgun and ting their prices and drawing customers away.
waited for the right moment. At a barbecue They told the thug to rob the store owner but
celebrating her friend’s birthday, the wife gave not to harm him; they only wanted to scare him
the gun to the friend’s 10-year-old nephew. out of town. The thug loitered near the discount
She told the nephew that it was a toy gun that store, waiting for it to close. When the lights in
would shoot out a Happy Birthday flag and that the store went out, an employee left by the back
he should shoot his aunt with it as a joke. The exit. The thug thought he was the owner and
nephew took the gun, pointed it at his aunt, and drew his gun, demanding money. The employee
pulled the trigger. However, the nephew’s aim resisted, and in the ensuing struggle was fatally
was faulty, and when the gun discharged the shot. The thug ran off, but both he and the
bullet struck the husband, killing him. partners were eventually arrested.

Which of the following best describes the In addition to conspiracy to commit robbery,
crime or crimes of which the wife can be are the partners guilty of any other crime(s)?
properly convicted?
(A) No other crimes.
(A) Murder of the husband and attempted mur-
der of the friend. (B) Robbery.

(B) Murder of the husband only. (C) Felony murder.

(C) Attempted murder of the friend only. (D) Robbery and felony murder.

(D) Attempted murder of the friend or murder


of the husband, but not both.
CRIMINAL LAW - HOMICIDE QUESTIONS 5.

Question 9 Question 10

The defendant entered a bank, planning to The defendant and his friend entered a conve-
rob it. An alert bank employee saw the defen- nience store wearing ski masks and demanded
dant brandishing her gun and pushed the silent all the money in the register, claiming they
alarm button to summon the police, and most had a gun. The clerk promptly complied with
of the employees and customers were successful that demand. The pair grabbed the money and
in fleeing the bank. However, when the police ran out the door. A police officer saw them
surrounded the bank, the defendant was still running through the parking lot, still wearing
inside with a hostage, one of the bank’s tellers, their masks, and surmised that a robbery had
whom the defendant had forced to turn over all taken place. Without any warning, the police
the money in her drawer. The police obtained officer drew out his gun and fired two shots,
only a vague description of the defendant from one of which shattered the defendant’s kneecap
the fleeing employees and customers as being a and sent him tumbling to the ground. The other
white female of average height, but all agreed bullet struck the friend in the head, killing him
that she was wearing a bright yellow scarf instantly. The defendant is placed on trial for the
around her neck. The defendant, hoping to friend’s death on a felony murder theory.
distract the police and escape out a back door,
forced the teller to put on the bright yellow scarf Which of the following is the best argument
and walk in front of a window. A rookie police for the defendant to make in order to gain an
officer, on seeing the yellow scarf, opened fire, acquittal?
killing the bank teller immediately. The defen-
dant was apprehended shortly thereafter trying (A) Both the defendant and his friend were
to escape out the back door. unarmed.

If the defendant is prosecuted and found (B) The police officer failed to warn the pair
guilty of the teller’s murder, what is the likely before firing.
reason?
(C) The felony had already been completed
(A) The defendant was still in the building when the friend was killed.
and had not yet run out at the time that the
teller was killed. (D) The friend was a co-felon.

(B) The police officer did not have legal justifi-


cation to use deadly force under the circum-
stances.

(C) The defendant caused the death of the


victim during the course of a felony.

(D) The defendant’s putting the victim in a


position of danger shows intent to kill.
6. CRIMINAL LAW - HOMICIDE QUESTIONS

Question 11 Question 12

A husband was very jealous of any contact his A worker on scaffolding several stories high
wife had with other men, and was particularly on a building took a loose brick from the build-
suspicious of his wife’s relationship with their ing’s façade and threw it off the scaffolding
neighbor, a plumber. Early one morning, the without looking, striking a pedestrian on the
shower in the couple’s master bathroom sprang sidewalk in the head and killing her instantly.
a leak while the wife was getting ready for the
day. Fearing permanent damage to their house, The homicide statute in this jurisdiction reads
and needing to get the repair done quickly, the as follows: “Murder is the unlawful killing of
wife quickly threw on some clothes and called a human being with malice aforethought. Such
the neighbor. The neighbor immediately went malice may be express or implied. It is express
over to the couple’s home, and went to the when there is manifested a deliberate inten-
master bathroom. At the same time, the husband tion to unlawfully take away the life of a fellow
unexpectedly came home because he had creature. It is implied when no considerable
forgotten something for work. The husband went provocation appears or when the circumstances
to the master bedroom and saw the neighbor and attending the killing show an abandoned and
his wife with her hair wet and clothes hastily put malignant heart.”
on. Enraged, he ran to his study, grabbed his gun
from the desk drawer, and shot and killed the The manslaughter statute in the jurisdiction
neighbor. reads as follows: “Manslaughter is the unlawful
killing of a human being without malice. It is
The jurisdiction defines murder in the first of two kinds: 1. Voluntary—upon a sudden
degree to include premeditated and deliberate quarrel or heat of passion. 2. Involuntary—in the
killings and all killings that take place during commission of an unlawful act, not amounting
the commission of a dangerous felony. Premedi- to a felony; or in the commission of a lawful act
tation and deliberation is defined as requiring that might produce death in an unlawful manner
some meaningful reflection prior to the killing. or without due caution and circumspection.”
All other common law murder is classified as
murder in the second degree. If the worker is charged with criminal
homicide, on which of the following theories
If the jury finds that the husband was unrea- could the trial judge properly give a charge to
sonable in his erroneous belief that the neighbor the jury?
and his wife were together for the purpose of
adultery, what crime did the husband commit (A) Involuntary manslaughter only.
when he killed the neighbor?
(B) Murder and involuntary manslaughter.
(A) Voluntary manslaughter.
(C) Murder and voluntary manslaughter.
(B) Involuntary manslaughter.
(D) First degree murder, second degree murder,
(C) First degree murder. and involuntary manslaughter.

(D) Second degree murder.


CRIMINAL LAW - HOMICIDE QUESTIONS 7.

Question 13 Question 14

Two teenagers who had recently gotten their A state statute defines all murders as second
driver’s licenses agreed to a game of “chicken,” degree murders unless deliberation and premedi-
with the one who turned his car away first tation can be shown, in which case the crime is
having to pay the other $25. The two cars raced elevated to first degree murder. Manslaughter is
toward each other, and when they were about 40 defined as at common law.
feet apart, the victim suddenly turned his car to
the right, but the defendant was going too fast to The defendant wanted to steal some papers
avoid the victim’s car. He crashed into the side of from a business associate’s office, and so he
the victim’s car, killing the victim instantly. arranged to have a meeting with her at her office.
When the associate left the room, the defen-
The homicide statute in this jurisdic- dant put a knockout drug in her coffee. After
tion reads in part as follows: “Murder is the she passed out from drinking the coffee, the
unlawful killing of a human being with malice defendant rummaged through her files, finding
aforethought. Such malice may be express or and stealing the papers that he wanted. Unfortu-
implied. It is express when there is manifested nately, the defendant miscalculated the dosage,
a deliberate intention to unlawfully take away and the business associate died.
the life of a fellow creature. It is implied when
no considerable provocation appears or when The crimes below are listed in descending
the circumstances attending the killing show an order of seriousness.
abandoned and malignant heart. All murder that
is perpetrated by willful, deliberate, or premedi- What is the most serious offense of which the
tated killing or committed in the perpetration of defendant can be convicted?
or attempt to perpetrate arson, rape, robbery, or
burglary is murder of the first degree. All other (A) Murder in the first degree.
kinds of murders are of the second degree.”
(B) Murder in the second degree.
The defendant is charged with first degree
murder. What is his most effective defense? (C) Voluntary manslaughter.

(A) An automobile is not a dangerous weapon, (D) Involuntary manslaughter.


and therefore there can be no deliberate
killing.

(B) The defendant, because of his youth, could


not have formed the necessary malice
aforethought to support a conviction for
murder.

(C) The defendant is guilty of murder, but not


first degree murder.

(D) The defendant cannot be guilty of any


degree of murder because he did not intend
to kill the victim.
8. CRIMINAL LAW - HOMICIDE QUESTIONS

Question 15 Question 16

While out walking one evening, a pedes- A stagehand decided to play a practical joke
trian was stopped at gunpoint by a robber who on an actor. The stagehand went to the storage
demanded all of her money. The pedestrian room where stage props were stored and took
hesitated in going for her wallet, and so the what he believed to be a stage gun from the
robber hit her over the head. In doing so, the locker where such guns were kept. In fact, a
robber accidentally dropped the gun, panicked, week before, an actress had put her real pistol in
and started to run. The pedestrian was stunned the stage gun locker and borrowed the stage gun
for a second by the blow to the head, but she for an amateur theatrical her church group was
recovered quickly, grabbed the gun from the putting on.
ground, and shot at the fleeing robber. The bullet
missed the robber, but hit a bystander, killing The actress had forgotten to remove the
him instantly. The pedestrian was arrested and bullets that her husband always kept in the gun.
charged with murder. The stagehand went into the actor’s dressing
room and yelled, “You’ve stolen the part that I
If her attorney asserts at trial that the always wanted to play, now die for it!” The actor
pedestrian should be charged with voluntary knew that the stagehand liked to play practical
manslaughter rather than murder, would this jokes, and, after an initial frightened reaction,
assertion be correct? the actor broke out laughing. The stagehand
laughed too, shouted, “Bang, you’re dead!” and
(A) Yes, because the pedestrian had no intent to pulled the trigger. A bullet hit the actor in the
kill the bystander. heart, killing him.

(B) Yes, because there was adequate provoca- Which of the following best describes the
tion for the pedestrian’s actions. stagehand’s criminal liability?

(C) No, because the pedestrian intended to kill (A) He is guilty of second degree murder.
the robber.
(B) He is guilty of voluntary manslaughter.
(D) No, because the pedestrian was in no
danger when she shot at the fleeing robber. (C) He is guilty of involuntary manslaughter.

(D) He has committed no crime.


CRIMINAL LAW -
HOMICIDE ANSWERS
CRIMINAL LAW - HOMICIDE ANSWERS 1.

CRIMINAL LAW - HOMICIDE ANSWERS

Answer to Question 1

(D) The strongest argument is that the shopkeeper was reacting to a sufficient provocation. This is a
very commonly tested MBE issue. (A) is incorrect. It is clear from the facts that the shopkeeper
manifestly intended to kill or seriously injure the gangster, and this intent is transferred to the
delivery person, who was killed by mistake. (B) is incorrect because any imminent threat of death
or serious injury to the shopkeeper had passed at the time he shot at the gangster. (C) is incorrect
because there is nothing in the facts that indicates duress. Therefore, (D) presents the strongest
argument—that the gangster’s intentional near miss with the knife was sufficient provocation to
mitigate the intended homicide from murder to voluntary manslaughter.

Answer to Question 2

(B) The hunter can be convicted of murder. Conviction for common law murder requires malice. At
common law, malice is: (i) the intent to kill (express malice); (ii) intent to inflict great bodily
injury; (iii) reckless indifference to an unjustifiably high risk to human life (“depraved heart”); or
(iv) intent to commit a felony (“felony murder”). The hunter did not intend to kill a child when he
fired at the target, so he did not meet the specific intent test. Thus, (A) is incorrect. Malice is also
shown when a person acts with a wanton and reckless disregard for human life. Based on these
facts, the hunter clearly acted with malice by firing a high-powered gun in front of an occupied
playground; thus, (B) is the correct answer. (C) is incorrect, given that specific intent is not the
only way to show malice. (D) is incorrect because the hunter’s actions go far beyond simple negli-
gence.

Answer to Question 3

(D) The imperfect self-defense doctrine applies to these facts. In states that apply the doctrine, murder
is reduced to voluntary manslaughter when the defendant unreasonably, but honestly, believed
in the necessity of responding with deadly force. In the instant case, the performer unreason-
ably, but apparently honestly, believed that the pedestrian would kill him. Thus, the performer
would be guilty of voluntary manslaughter in such a jurisdiction. (A) is wrong. In order to have
murder reduced to voluntary manslaughter based on a “heat of passion” theory, it must be shown
that: (i) the provocation must have been one that would arouse sudden and intense passion in the
mind of an ordinary person such as to cause him to lose self-control; (ii) the defendant was in
fact provoked; (iii) there must not have been sufficient time to cool off; and (iv) the defendant
did not in fact cool off. Adequate provocation generally consists of being subjected to a serious
battery or threat of deadly force, or of discovering one’s spouse in bed with another person. In the
instant case, not only is it unlikely that a slap on the face would be considered a serious battery
such that would provoke one to murder, the facts do not indicate that the performer acted in a fit
of rage. (B) is wrong because there is no evidence of any diminution of the defendant’s ability to
reason. (C) is wrong. Although some states would call a killing committed during a misdemeanor
“manslaughter,” in the instant case there are no facts to indicate that the performer was in the
process of committing a misdemeanor when he killed the pedestrian.

Answer to Question 4

(D) The defendant is guilty of a criminal homicide because his act was the proximate cause of the
victim’s death. The defendant shot the victim, intending to kill him, and the victim died. The
2. CRIMINAL LAW - HOMICIDE ANSWERS

only issue is whether the defendant’s act was the proximate cause of the victim’s death. Proximate
cause is found when the results are the natural and foreseeable results of the defendant’s acts.
Infections and negligent medical treatment are deemed to be a foreseeable risk in homicide cases;
since the victim died of pneumonia that resulted from the bullet, most courts would conclude that
proximate cause has been established. (A) would be correct if proximate cause could not be estab-
lished. (B) and (C) are wrong. The fact that the victim already had an incurable disease or would
likely have died from another cause will not negate proximate cause.

Answer to Question 5

(B) The woman will most likely be found guilty of involuntary manslaughter because she knowingly
and negligently chose to drive her car. Common law involuntary manslaughter is of two types:
(i) a killing committed by the criminal negligence of another; or (ii) a killing occurring during
the commission of an unlawful act. Criminal negligence requires a greater deviation from the
“reasonable person” standard for civil liability. Some states require that the defendant have had a
subjective awareness of the risk. Here, given that the woman was aware of the fact that she might
experience a seizure without warning while driving, her driving into town most likely constituted
criminal negligence. Thus, she will most likely be convicted of involuntary manslaughter. (A) is
incorrect. A jury can infer the required general intent from the performance of an act. However, it
would not relieve the prosecution of the burden of proving that the defendant acted in a criminally
negligent manner, an essential element of the crime of involuntary manslaughter. (C) is incorrect
because she voluntarily drove knowing that she might experience a seizure, which is sufficient for
involuntary manslaughter. (D) is incorrect because the state of mind for involuntary manslaughter
can be established by criminal negligence.

Answer to Question 6

(A) The daughter can be convicted of first degree murder for her father’s death. Therefore, (D) is
incorrect. Murder is the unlawful killing of a human being with malice aforethought. In the
absence of facts excusing the homicide or reducing it to voluntary manslaughter, malice afore-
thought exists if the defendant has (i) intent to kill, (ii) intent to inflict great bodily injury, (iii)
awareness of an unjustifiably high risk to human life, or (iv) intent to commit a felony (felony
murder doctrine). In this case, the daughter clearly had the intent to kill her father; her motive
for doing so is irrelevant for establishing malice aforethought. It is also quite possible under the
facts that the daughter acted with premeditation and deliberation, making her potentially liable
for first degree murder. “Deliberate” means that the defendant reflected on the crime in a cool and
dispassionate manner. “Premeditated” means that such reflection actually was undertaken, but it
need only be for a very brief period. Here, the daughter could have reflected on the crime before
she visited the hospital; the fact that she had a gun in her purse is circumstantial evidence of such
reflection. Even if she did not make the decision to kill until moments before she pulled out the
gun, she would still have been capable of premeditation and deliberation. (B) is therefore incorrect
because it is not the most serious crime of which the daughter can be convicted. (C) is incorrect
because of the absence of facts establishing adequate provocation that would reduce an intentional
killing from murder to voluntary manslaughter. Adequate provocation is most frequently recog-
nized in cases of (i) being subjected to a serious battery or threat of deadly force, and (ii) discov-
ering one’s spouse in bed with another person. While modern courts have broadened somewhat
the scope of what constitutes provocation, the law does not presently recognize feelings of mercy
toward the victim as an adequate provocation. Thus, the daughter can be convicted of murder for
her father’s death. (This question is a good illustration of why you must always treat the defendant
objectively in your analysis of Criminal Law questions.)
CRIMINAL LAW - HOMICIDE ANSWERS 3.

Answer to Question 7

(A) The wife is liable both for the attempted murder of her friend and, under the doctrine of trans-
ferred intent, for the murder of her husband. To be guilty of murder, one must unlawfully kill
another human being with malice aforethought, which is a term of art encompassing various
states of mind, including intent to kill, and the absence of facts excusing the homicide or reducing
it to voluntary manslaughter. Here, the wife had the intent to kill her friend. Under the doctrine of
transferred intent, this intent is transferred from the friend to the husband, the actual victim of the
murder. There are no facts here excusing the homicide or reducing it to voluntary manslaughter
on the basis of adequate provocation. Modern courts might allow a jury to consider whether
discovery of the love letters would arouse the intense passion required for the first element of the
provocation test; however, the third element, an absence of sufficient time between the provoca-
tion and the killing for the passions of a reasonable person to have cooled, is disproved by the
facts of the question. The act of killing, although done by the nephew, is attributable to the wife as
the principal because she caused an innocent intermediary (the nephew) to accomplish the result.
Hence, the wife is liable for the murder of the husband; (C) is therefore incorrect. At common law,
the crime of attempted murder required both a specific intent by the actor to kill the victim and an
act that puts the defendant in close proximity to completing the crime. The Model Penal Code and
most state criminal codes modify the “proximity” test for the act requirement, instead requiring
an act that constitutes a “substantial step” towards commission of the crime. Here, the wife had
the intent to kill the friend and intentionally gave the loaded gun to the nephew with instructions
to point it at the friend and pull the trigger. Under either of the tests used, her conduct satisfies the
act requirement of attempt. Thus, the wife is also liable for the attempted murder of the friend;
(B) is therefore incorrect. (D) is incorrect because the same act and intent can constitute both an
attempt as to one person and a completed crime as to another person. Although, if the nephew’s
aim had been better, the wife could not have been convicted of both attempted murder of the
friend and murder of the friend, the attempt against the friend and the murder of the husband are
separate crimes because more than one victim is involved.

Answer to Question 8

(C) The partners are guilty of felony murder in addition to conspiracy to commit robbery. A killing
(even if accidental) committed during the course of a felony is murder. If, in the course of a
conspiracy to commit a felony, a death is caused, all members of the conspiracy are liable for
murder if the death was caused in furtherance of the conspiracy and was a foreseeable conse-
quence of the conspiracy. Here, although there was to be no physical harm, and the thug mistook
the employee for the discount store owner, it was foreseeable that death would result from the
commission of a dangerous felony such as robbery. The employee’s death occurred in furtherance
of the conspiracy to rob the discount store owner. Because this death occurred in furtherance of
the conspiracy and was a foreseeable consequence thereof, the other conspirators (the partners) are
liable for felony murder. Robbery was not committed here because there was no taking of personal
property from the employee. Robbery consists of a taking of the personal property of another from
the other’s person or presence by force or intimidation with the intent to permanently deprive him
of the property. Although the thug demanded the employee’s money, the ensuing struggle prevented
him from actually taking it. This would be attempted robbery, because the thug committed an act
with the intention of committing robbery, but he fell short of completing the crime. However, this is
not robbery. Therefore, (A) is incorrect because it fails to account for the fact that the partners are
guilty of felony murder as well as conspiracy to commit robbery. (B) and (D) are incorrect because
they would find the partners guilty of robbery, despite the fact that there was no taking of personal
property. Also, (B) ignores the fact that the partners are guilty of felony murder.
4. CRIMINAL LAW - HOMICIDE ANSWERS

Answer to Question 9

(C) If the defendant is found guilty of murder, it will be through application of the felony murder rule
as stated in choice (C). Under the felony murder doctrine, a killing—even an unintentional one—
committed during the course of a felony is murder. Malice is implied from the intent to commit
the underlying felony. For the doctrine to apply, the death must have been a foreseeable result of
commission of the felony, but almost any death during an armed bank robbery would be deemed
foreseeable by a court. Courts following the “proximate cause” theory of felony murder extend it
to situations like the one in this fact pattern, where resistance by the victim or the police results
in the death of a third party who is not a co-felon. Even courts following the “agency theory” of
felony murder, which ordinarily requires that the killing be committed by an accomplice, would
apply it here because the victim was forced to occupy a dangerous place by the felon. (A) is wrong
because it is irrelevant that the defendant had not yet run out of the building at the time the teller
was killed; the defendant would be liable even if she had already fled from the building. Only
when the felon has reached a place of temporary safety after the felony has ended will the felony
murder rule cease to apply. (B) is wrong because the defendant’s guilt under the felony murder
rule is not dependent on whether the police were justified in using deadly force; it is the defen-
dant’s commission of the robbery, rather than the response by the police, that establishes her
liability for felony murder. (D) is wrong because the defendant’s putting the teller in a position of
danger, although it might be sufficient to imply malice based on reckless indifference to human
life, is not sufficient to establish an intent to kill (express malice). Here, malice is established by
the felony murder doctrine.

Answer to Question 10

(D) The defendant’s best argument is that, under the majority view, one co-felon is not crimi-
nally liable for the death of another co-felon from resistance of the victim or police. A killing
committed during the course of a felony is murder, with malice being implied from the intent to
commit the underlying felony. Liability under the felony murder doctrine requires that death have
been a foreseeable result of commission of the felony. Under the majority view, felony murder
liability cannot be based on the death of a co-felon from resistance by the victim or police pursuit.
In the instant case, the defendant’s friend and co-felon was killed by a pursuing police officer
following commission of the robbery. Under the majority rule, the defendant is not guilty of felony
murder for such a death. Thus, (D) represents his strongest argument for acquittal. (A) is incor-
rect because, despite the fact that the defendant and his friend were unarmed, they still committed
robbery of the store, in that they took the money from the clerk by means of threats of immediate
death or serious physical injury, with the intent to permanently deprive. Certainly, it was reason-
able for the clerk to not take a chance that they might not have guns, given that the defendant said
they did. Thus, the defendant is guilty of the underlying felony of robbery. Armed resistance by
the victim or police that results in death is a foreseeable result of robbery even when the robbers
are not armed. Consequently, the fact that the defendant and his friend were unarmed at the time
of committing the robbery will have no bearing on his guilt under a felony murder theory. (B) is
incorrect because the police officer’s failure to warn the fleeing felons before firing is irrelevant to
the defendant’s liability for felony murder. Even had the police officer given a warning, the defen-
dant would not be guilty of the felony murder of his co-felon killed by the police. The lack of a
warning might go to the issue of whether the police officer’s shooting was justified, but it does
not impact on the defendant’s guilt for felony murder. (C) is incorrect because, although death
must have been caused during the commission or attempted commission of a felony, the fact that
the felony was technically completed before death was caused does not prevent the killing from
being felony murder. Deaths caused while fleeing from the crime are felony murder. Although the
CRIMINAL LAW - HOMICIDE ANSWERS 5.

defendant and his friend had completed the robbery at the time the friend was killed, the killing
occurred while the felons were fleeing from the crime. Thus, if all other required factors were
present, this would be felony murder.

Answer to Question 11

(D) The husband’s killing would be second degree murder. The husband intended to kill his neighbor,
but he did not act in a deliberate and premeditated manner, nor was he acting under a provocation
that would have aroused a sudden and intense passion in the mind of an ordinary person (given
the jury finding). Thus, of the crimes listed, the husband can be guilty only of second degree
murder. Murder is the unlawful killing of a human being with malice aforethought. Malice afore-
thought exists if the defendant has: (i) intent to kill; (ii) intent to inflict great bodily injury; (iii)
awareness of an unjustifiably high risk to human life; or (iv) intent to commit a felony. Statutes
such as the one in this question often divide murder into degrees. Thus, first degree murder here
consists of a deliberate and premeditated taking of life, with all other murders relegated to second
degree status. An intentional killing is reduced from murder to voluntary manslaughter if: (i) the
defendant acts under a provocation that caused him to lose control; (ii) the provocation would
arouse sudden and intense passion in the mind of an ordinary person so as to cause him to lose
self-control; (iii) there is insufficient time between the provocation and the killing for the passions
of a reasonable person to cool; and (iv) the defendant in fact did not cool off between the provoca-
tion and the killing. Here, the facts are clear that the husband shot his neighbor with the intent to
kill him. This eliminates (B) as a correct answer. Involuntary manslaughter occurs when death
is caused by criminal negligence, rather than an intentional act. The husband’s shooting of the
neighbor goes far beyond mere criminal negligence. (A) is incorrect because of the unreason-
ableness of the husband’s belief that the neighbor and his wife were together for the purpose of
committing adultery. For purposes of voluntary manslaughter, the discovery of one’s spouse in
the act of adultery constitutes adequate provocation. However, as noted above, the provocation
must have been such as to arouse the passions of an ordinary person. If, as this question states,
the husband’s belief regarding the neighbor and his wife was unreasonable, the passions of an
ordinary person would not have been aroused on seeing them together. Therefore, the husband’s
killing will not be reduced to voluntary manslaughter. (C) is incorrect because the quickness and
anger with which the husband acted would preclude the notions that he meaningfully reflected
on the idea of killing his neighbor, as required by the definition of premeditation and deliberation
in the statute. This absence of deliberation and premeditation takes the killing outside the ambit
of first degree murder. Consequently, the husband’s intentional killing of the neighbor must be
considered second degree murder, as Choice (D) states.

Answer to Question 12

(B) The judge should instruct the jury on murder and involuntary manslaughter. First, a jury could
find the worker guilty of murder under the theory that throwing a brick from scaffolding high
above a sidewalk without looking shows an “abandoned and malignant heart.” The jury could
also find the worker guilty of involuntary manslaughter—either because throwing a brick off
scaffolding is an unlawful act not amounting to a felony or because throwing the brick off
scaffolding is a lawful act done “without due caution and circumspection.” Thus, (A) is wrong;
it would be proper to charge the jury on murder as well as manslaughter. (C) is wrong because
there is no way that the worker could be guilty of voluntary manslaughter, which would involve an
unjustified, unexcused, but mitigated intentional homicide (i.e., an intentional homicide committed
under “heat of passion”). The worker did not intend to kill or seriously injure; moreover, if even
assuming he did so intend, there is no factor of mitigation present. (D) is wrong because this
6. CRIMINAL LAW - HOMICIDE ANSWERS

question is based on a statute following the common law definition that does not divide murder
into degrees.

Answer to Question 13

(C) The most effective argument is that the defendant is not guilty of first degree murder. The only
theory of first degree murder applicable here is that the murder was “willful, deliberate, or [and]
premeditated.” This type of murder requires an actual intent to kill, which the defendant did not
have. (A) is wrong because an automobile can be a dangerous weapon when used as one, and the
intentional driving of an automobile at another can support and permit, though not require, an
inference that defendant intended to kill. (B) is wrong because, at common law, 14 is the cutoff
age for youthfulness to be a factor. The facts state that defendant was licensed and thus must be
over the age of 14. (D) is clearly wrong. Although the defendant might be able to avoid liability
for first degree murder because of his lack of intent to kill, he could be liable for second degree
murder.

Answer to Question 14

(B) The most serious offense of which the defendant can be convicted is murder in the second degree.
The jurisdiction defines murder in the first degree as deliberate premeditated murder, whereas
all other murders are second degree murders. Deliberation and premeditation requires some time
of cool reflection on the idea of killing. In the instant case, the defendant did not actually intend
to kill; thus, deliberation and premeditation are logically excluded. The facts indicate that all
other murders are classified as murder in the second degree. Because malice for murder may be
implied from the intent to commit the underlying felony, even an accidental killing committed
during the course of a felony is murder. Here, the defendant intended to, and did in fact, commit
a dangerous felony, robbery, which is the taking of personal property of another from the other’s
person or presence, by force or intimidation, with the intent to permanently deprive him of it. The
knockout drops constituted force sufficient to overcome the business associate’s resistance; hence,
the defendant has committed a robbery and his intent constitutes implied malice for purposes of
felony murder. Furthermore, an argument could be made that a drug that was sufficient to knock
someone out cold is a dangerous drug, and the use of such a drug would evidence an indifference
to an unjustifiably high risk to human life and thus constitute malice. This would provide a second
basis for a finding of murder in the second degree. Therefore, (B) is correct and (A), (C), and (D)
are incorrect.

Answer to Question 15

(B) The pedestrian’s actions were motivated by adequate provocation and therefore manslaughter is
the more appropriate crime. Voluntary manslaughter is a killing that would otherwise be murder
but is distinguishable from murder by adequate provocation. Provocation is sufficient to reduce
the killing from murder to manslaughter if it would arouse sudden and intense passion in an
ordinary person, and there has been insufficient time for the passions of a reasonable person to
cool. Also, the defendant must actually be provoked and have acted under that provocation (i.e.,
did not cool off). Here, the pedestrian’s use of a deadly weapon under these circumstances shows
that she intended to kill the robber or at least inflict great bodily harm. However, the commis-
sion of a violent felony (especially one that included a staggering blow) is provocation that would
arouse a reasonable person and apparently did arouse the pedestrian. She acted very quickly after
her attack, and thus a reasonable person would not have cooled off nor, apparently, had the pedes-
trian. Thus, she had the intent and provocation for manslaughter. However, she did not intend to
CRIMINAL LAW - HOMICIDE ANSWERS 7.

kill the bystander. Under the transferred intent doctrine, if a defendant intended injury to a person,
and in trying to carry out that intent caused similar injury to another, her intent is transferred
from the intended person to the one harmed. In addition, any mitigating circumstances that the
defendant could have asserted against the intended victim (such as provocation) will also usually
be transferred. Therefore, the pedestrian can be guilty of voluntary manslaughter for the death of
the bystander. (Note: It is possible that the pedestrian could try to escape conviction altogether in
some jurisdictions by claiming that the intent she formed was “justified” as an effort to prevent
the escape of a fleeing felon. However, the question asks only whether she has committed murder
or manslaughter.) (A) is wrong because it overlooks the transferred intent doctrine. If the pedes-
trian intended to kill the robber, this intent will be transferred to the bystander and she cannot
escape criminal liability simply because she hit the wrong person. (C) is wrong because, as
explained above, her intent was mitigated by adequate provocation, and thus manslaughter is the
more appropriate crime. (D) is wrong because even though the pedestrian was in no danger when
she shot at the robber, she was at least acting under adequate provocation so that she should be
convicted of voluntary manslaughter rather than murder.

Answer to Question 16

(D) The stagehand cannot be convicted of any of these crimes. Murder is the unlawful killing of a
human being with malice aforethought. Malice aforethought exists if the defendant has any of
the following states of mind: (i) intent to kill; (ii) intent to inflict great bodily injury; (iii) aware-
ness of an unjustifiably high risk to human life; or (iv) intent to commit a felony. Modern statutes
often divide murder into degrees. For instance, a deliberate and premeditated killing (i.e., one in
which the defendant made the decision to kill in a cool and dispassionate manner, and actually
reflected on the idea of killing) may be designated first degree murder. Second degree murder
is generally a killing with malice aforethought that is not specifically made first degree murder.
Here, the stagehand did not even realize that he was pointing a real gun at the actor. Thus, the
stagehand did not possess any of the states of mind that would constitute malice aforethought
(i.e., he did not intend to kill or to inflict great bodily injury, nor was he aware of a high risk to
human life). Consequently, the stagehand cannot be convicted of murder, either first or second
degree. Thus, (A) is incorrect. Voluntary manslaughter is a killing that would otherwise be murder
but is distinguishable from murder by the existence of adequate provocation. At the time of the
killing, the defendant must have been acting under a provocation that would arouse sudden and
intense passion in the mind of an ordinary person so as to cause him to lose self-control, with an
insufficient time between the provocation and the killing for the passions of a reasonable person
to cool. The stagehand cannot be convicted of voluntary manslaughter because: (i) his killing of
the actor was accidental, rather than intentional; and (ii) in shooting the actor, he was not acting
under any type of provocation. Therefore, (B) is incorrect. Involuntary manslaughter occurs when
a death is caused by criminal negligence. There is negligence when a person fails to be aware
that a substantial and unjustifiable risk exists or that a result will follow, and such failure consti-
tutes a substantial deviation from the standard of care that a reasonable person would exercise
under the circumstances. Criminal negligence requires a greater deviation from the “reasonable
person” standard than is required for civil liability. The stagehand’s firing the gun at the actor
was probably negligent. However, the stagehand’s conduct did not rise to the level of criminal
negligence, because he had insufficient knowledge of the true risk posed by his actions. If the
stagehand knew he was using a real gun but believed that it was not loaded, it would probably
have been criminally negligent to have pointed it at the actor and pulled the trigger. However, the
stagehand actually believed that he was using a fake gun. Thus, he cannot be said to have taken
the type of very unreasonable risk that would constitute criminal negligence. Therefore, (C) is
incorrect.
CRIMINAL LAW -
INCHOATE CRIMES
AND ACCOMPLICES
QUESTIONS
CRIMINAL LAW - INCHOATE CRIMES AND ACCOMPLICES QUESTIONS 1.

CRIMINAL LAW - INCHOATE CRIMES AND ACCOMPLICES QUESTIONS

Question 1 Question 2

A student activist suggested to her boyfriend A friend knew that a man and a woman cared
that they put a drug in the sugar bowls at their deeply for each other and encouraged the man
college cafeteria so that students would become to ask the woman to marry. Unbeknownst to the
sick and the activist’s antinuclear group could friend, the man was already married. Finally,
claim that toxic emissions from a nearby nuclear after continued encouragement by the friend, the
power plant were the cause. The boyfriend man went through a marriage ceremony with the
told the student that people could become very woman, and the friend was the man’s best man
ill, or even die, if they consumed too large a at the ceremony.
quantity of the drug. The student promised
that they would only put a small amount of the If the friend is charged with being an accom-
drug in each bowl and asked the boyfriend to plice to bigamy, should he be found guilty?
get the drug from his chemistry lab during his
chemistry class, which he did. After class, the (A) Yes, because he encouraged the man, and
boyfriend gave the bottle to the student. Before his mistake as to the existence of a prior
the boyfriend awoke the next day, the student marriage is no defense to a charge of
went to the cafeteria and put small quantities of bigamy.
the drug in several sugar bowls. She accidentally
put a much larger amount in the sugar bowl at (B) Yes, because he was present when the
one table. Four persons who sat at that table crime occurred and is thus a principal in
became extremely ill, and one of the four eventu- the second degree.
ally died.
(C) No, because he did not have the mental
In a jurisdiction applying the common law, state required for accomplice liability.
how will the boyfriend be liable for the death of
the student? (D) No, because his encouragement and assis-
tance was not the legal cause of the crime.
(A) As an accessory before the fact.

(B) As a principal in the first degree.

(C) As a principal in the second degree.

(D) As an accessory after the fact.


2. CRIMINAL LAW - INCHOATE CRIMES AND ACCOMPLICES QUESTIONS

Question 3 Question 4

A wife wanted to have her husband murdered A man was approached by his cousin, who
in order to collect the proceeds from a life insur- planned to rob a bank, to act as the escape
ance policy. The wife solicited the services of a driver. The man pretended to agree, and he
hitman, agreeing to split the insurance proceeds planned to report his cousin to the police before
with him. The plan was for the hitman to shoot the crime was committed. On the day appointed
the husband at home and make it look like a for the crime, the man picked up his cousin and
botched burglary attempt, while the wife would drove him to the bank, and they stopped at a
be at her church hall playing bingo that night. convenience store on the way. He left the car
However, when the hitman broke into the apart- running with his cousin inside and from the store
ment, the husband was able to subdue him and he called the police to warn that a bank robbery
call the police. The hitman was arrested and was about to occur. The police officer answering
implicated the wife, who was arrested in the the phone thought it was a prank call and hung
parish priest’s office just after she had confessed up. After another unsuccessful attempt the man
the plan to the priest. The wife is charged with went back to the car to talk his cousin out of the
attempted murder and conspiracy to commit bank robbery, but he found the car missing. The
murder in a state that retains the common cousin had grown nervous after a few minutes,
law definitions of these crimes. A psychiatric driven to the bank, and robbed it at gunpoint. He
evaluation of the hitman after he was arrested fled the scene, but was apprehended soon there-
revealed that he was legally insane, and he later after.
committed suicide while awaiting trial.
If the man is charged with conspiracy, is the
As to the conspiracy charge against the trial court likely to find him guilty?
wife, which of the following would be her most
promising defense? (A) Yes, because his withdrawal was ineffective
because it was not communicated to his
(A) She cannot be tried for both offenses; the cousin in time for him to change his plans.
prosecutor must choose between them.
(B) Yes, because he provided his cousin with an
(B) The hitman was insane when the instrumentality of the crime.
conspiracy was formed.
(C) No, because the man withdrew from any
(C) The hitman died in jail before trial. conspiracy.

(D) She had second thoughts about the plan (D) No, because the man lacked criminal intent.
while playing bingo and had revealed it to
her priest.
CRIMINAL LAW - INCHOATE CRIMES AND ACCOMPLICES QUESTIONS 3.

Question 5 Question 6

A man parked his car on a city street and After reading an article in a hunting magazine
entered a local tavern. While the man was detailing a state’s expanded season for the
inside, vandals broke the ignition motor on his hunting of grizzly bears, a hunter called his
car and smashed the taillights. Later that night nephew to see if he wanted to take a trip to the
the man left the tavern and attempted to start his state to hunt grizzly bears. His nephew agreed.
car, but was unable to because of the vandals’ Unknown to the hunter and his nephew, the
actions, which he had not been aware of. Police article in the magazine listed an incorrect ending
in a patrolling squad car noticed him entering date for the expanded grizzly bear hunting
the car and attempting to start it. When the man season; the hunting season had expired the day
got out of the car, the police approached him and before. While still in their pickup truck driving
asked him to identify himself. When he refused, to a campsite in the state, the hunter and his
he was arrested and charged with attempting to nephew were pulled over by a state trooper. They
drive an automobile at night without functioning volunteered that they were on their way to hunt
taillights. grizzly bears and were promptly arrested. A
state statute made hunting bears out of season a
What is the man’s best defense? strict liability offense.

(A) Entrapment. If the hunter and his nephew are charged with
conspiracy to hunt grizzly bears out of season,
(B) Legal impossibility. will they be acquitted?
(C) Factual impossibility. (A) Yes, because there was no agreement.
(D) Lack of requisite intent. (B) Yes, because they had not intended to
commit a crime.

(C) No, because hunting grizzly bears out of


season is a strict liability offense.

(D) No, because they agreed to hunt grizzly


bears on a date that was out of season.
4. CRIMINAL LAW - INCHOATE CRIMES AND ACCOMPLICES QUESTIONS

Question 7 Question 8

In order to track down “fences” and other As a wife was organizing files on the family
distributors of stolen goods, police officers, computer, she discovered emails and photos
with permission of the owners and insurers, showing that her husband was having an affair
forwarded stolen jewelry they have intercepted with his secretary. The wife was incensed,
on the way to the defendant, a reputed fence. The so she went to a drawer where she knew her
defendant took the jewelry, which he believed to husband kept his handgun and, assuming it
be stolen. was loaded because he always told her that it
was, headed off to her husband’s office with the
What crime has the defendant committed? intent to do away with his secretary. When the
wife arrived, she burst into the office, pulled
(A) Receipt of stolen property, because the in- the gun out of her purse, and pointed it at the
tercepted jewelry had not been returned to secretary. However, when the wife pulled the
its owners and therefore still had “stolen” trigger, nothing happened, because the handgun
status. contained no ammunition.
(B) Attempted receipt of stolen property, The jurisdiction’s criminal code defines
because his mistaken belief that the goods assault as “(1) an attempt to commit a battery; or
were stolen is no defense to attempt. (2) the intentional creation of a reasonable appre-
hension in the mind of the victim of imminent
(C) No crime, because his mistake as to the bodily harm.” The code uses the common law
legal status of the property constitutes definitions of homicide crimes.
a mistake of law, which is a defense to
attempt. Of which of the following crimes could the
wife be convicted?
(D) No crime, because the jewelry no longer
had “stolen” status. (A) Assault and attempted manslaughter, but
not attempted murder.

(B) Attempted murder only.

(C) Assault and either attempted murder or


attempted manslaughter.

(D) Attempted murder or attempted


manslaughter, but not assault.
CRIMINAL LAW - INCHOATE CRIMES AND ACCOMPLICES QUESTIONS 5.

Question 9 Question 10

When the defendant learned that his former A coal company developed a new extraction
wife, to whom he was paying $1,000 per month process that enabled it to produce coal at market-
in alimony, was dating someone else, the defen- able cost from “worn out” mines that it owned,
dant encouraged her to get married “for the sake and began operating the mines using the new
of the children.” The former wife said that she process. As a cost-cutting measure that increased
would consider it, but she also expressed concern profits, the corporate officers knowingly
that her boyfriend might already be married. permitted the new process to be utilized in such
The defendant told his former wife that he would a way that federally and state mandated methane
have an acquaintance run a computer check on gas detection measures were not complied with,
the boyfriend that would reveal whether he was and a consequent methane explosion severely
currently married. However, the defendant did injured several miners. In addition to being
not bother with the computer check; instead he prosecuted for felony violations of the methane
called the boyfriend and offered him $5,000 detection statutes, the corporation and its officers
if he would propose to the defendant’s former were prosecuted for attempted murder.
wife. The defendant then told his former wife
that, according to official records, the boyfriend What should be the outcome of this prosecu-
was single. The defendant’s former wife and the tion?
boyfriend went through a wedding ceremony
shortly thereafter. The boyfriend, however, was (A) Not guilty, because the corporate officers
already married to someone else, a fact that had not proceeded sufficiently beyond the
would have been disclosed by a routine check of planning stage for their actions to consti-
official records. tute an attempt.

If the defendant is charged with being an (B) Not guilty, because the corporate officers
accessory to bigamy, a strict liability offense in did not possess the requisite intent to consti-
the jurisdiction, should he be found guilty? tute the crime of attempted murder.

(A) Yes, because the defendant aided, abetted, (C) Guilty, because the corporate officers’
and encouraged the marriage. actions created a situation so dangerous to
human life or safety that their mental state
(B) Yes, because even though the defendant did would be considered an abandoned and
not know that the boyfriend was married, malignant heart.
bigamy is a strict liability offense.
(D) Guilty, because the violation of the federal
(C) No, unless the defendant’s action in failing and state methane detection statutes is a
to check the records was a breach of duty to felony.
his former wife.

(D) No, because the defendant did not have


the mental state necessary for aiding and
abetting.
6. CRIMINAL LAW - INCHOATE CRIMES AND ACCOMPLICES QUESTIONS

Question 11 Question 12

A student approached a reputed drug dealer In an effort to curb underage drinking, the
at a pool hall to purchase marijuana, although police staked out a liquor store near a college
he knew that it was a crime to possess or smoke campus that was suspected of selling liquor to
marijuana. The student bought a “marijuana minors in violation of state law. They recorded
cigarette,” which was in fact only an ordinary a liquor store clerk being told by a minor that he
tobacco cigarette, from the drug dealer. As the was an underage student at the local university
student left the pool hall, he lit the cigarette, and that he would pay triple the marked price for
whereupon he was immediately apprehended by a case of beer. The clerk readily agreed to the
a detective who was keeping all of the dealer’s deal and took the money, but both parties were
visitors under surveillance. After the student was arrested before they could complete the transac-
advised of his rights and admitted purchasing tion.
the “marijuana cigarette” from the dealer, the
police determined that there was no marijuana in In a jurisdiction following the unilateral
the cigarette. approach to conspiracy, which of the following
statements is correct?
If the student is charged with attempt to
smoke a marijuana cigarette, how should the (A) The minor can be convicted of solicitation
court rule? and the liquor store clerk can be convicted
of conspiracy to violate the statute.
(A) He is not guilty, because he accomplished
all acts he intended to do yet did not com- (B) Both the minor and the liquor store clerk
mit a criminal offense. can be convicted of conspiracy to violate
the statute.
(B) He is not guilty, because there was no
corroboration of his statement to the police. (C) The minor cannot be convicted of either
solicitation or conspiracy, but the liquor
(C) He is guilty, because had the attendant store clerk can be convicted of conspiracy
circumstances been as he believed them to to violate the statute.
be, he would have committed a criminal
offense. (D) The minor cannot be convicted of
either solicitation or conspiracy, and the
(D) He is guilty, because drug offenses are liquor store clerk cannot be convicted of
strict liability crimes. conspiracy.
CRIMINAL LAW - INCHOATE CRIMES AND ACCOMPLICES QUESTIONS 7.

Question 13 Question 14

A former employee and his friend agreed A 17-year-old high school student decided
to rob a grocery store. They entered the store it would be great fun to scare his teacher by
just before closing. The former employee held a playing some sort of prank on her. He told his
gun on the store manager and made him empty 16-year-old friend about his desire, and the
out the safe, and the friend directed a clerk friend, knowing the student’s propensity for
who was the only other person in the store to doing such stunts, encouraged him to go through
a storeroom in the back. Before tying her up, with his plan, giving him a toy rubber snake to
however, the friend raped her at gunpoint. The put in the teacher’s desk drawer. After opening
friend returned to the front of the store, where her desk drawer and having the rubber snake pop
the former employee had just finished putting out, the teacher jumped back and fell, injuring
the store’s money in a large sack. To prevent the her hip.
store manager from pursuing the pair or from
quickly notifying the police, the friend shot the A state statute provides that “anyone who
store manager in both knees. Neither the rape recklessly causes bodily injury to the person of
nor the shooting were part of the plan between another is guilty of battery in the third degree.”
the former employee and the friend.
What is the best theory for charging the friend
In a common law jurisdiction, of which of the with a battery in the third degree?
following crimes is the former employee guilty?
(A) The friend is an accomplice because he
(A) Robbery only. encouraged and provided assistance to the
student so that he could commit the crime.
(B) Robbery and rape only.
(B) The friend is a co-conspirator because he
(C) Robbery and aggravated battery only. agreed with the student to carry out the
plan.
(D) Robbery, rape, and aggravated battery.
(C) The friend is a principal and committed
acts of recklessness of his own that consti-
tuted the crime.

(D) By supplying the instrumentality of the


crime, he is vicariously liable for his
friend’s actions.
8. CRIMINAL LAW - INCHOATE CRIMES AND ACCOMPLICES QUESTIONS

Question 15 Question 16

The defendant is charged with the burglary of The police set up a sting operation targeting
a home. Evidence presented at the defendant’s a resale shop that had a reputation for selling
trial indicates that he talked another person into stolen goods. An undercover police officer
assisting him. The jury is instructed on burglary, approached the owner of the shop posing as
solicitation, conspiracy, and attempt. a truck driver who was down on his luck and
looking for a way to earn some extra money.
If the defendant is found by the jury to be The shop owner suggested that on his next load
guilty of burglary, which of the following is the officer should take a box of goods from the
true? back of his truck, bring them to the shop owner
to sell in exchange for cash, and then report the
(A) He also may be found guilty of conspiracy, box lost to the trucking company. The next day
but not of solicitation or attempt. the officer gave the shop owner an empty box
in exchange for money. Immediately after the
(B) He also may be found guilty of conspiracy exchange, the shop owner was arrested.
and solicitation, but not of attempt.
Which of the following crimes did the owner
(C) He also may be found guilty of conspiracy commit?
or solicitation but not both, and he may not
be found guilty of attempt. (A) Receipt of stolen property.
(D) He also may be found guilty of solicitation (B) Attempted receipt of stolen property.
or attempt but not both, and he may not be
found guilty of conspiracy. (C) Conspiracy and receipt of stolen property.

(D) No crime.
一 信:liuxue119118 , 们 信免 供

CRIMINAL LAW - INCHOATE CRIMES AND ACCOMPLICES QUESTIONS 9.

Question 17 Question 18

A 15-year-old girl who worked as a prosti- A statute in the jurisdiction makes it a crime
tute in a city approached a man and offered her to sell ammunition to a person under the age of
services. The man agreed, and the two engaged 18. The courts have interpreted this statute as
in sexual intercourse in the back seat of the creating a strict liability offense that does not
man’s car. These events were noticed by an require knowledge of the age of the purchaser
undercover police officer, who arrested the girl and as creating vicarious liability. A 16-year-old
and the man. The man admitted to having sex boy, who looked four or five years older, entered
with the girl, and he was charged with statutory a store and asked a clerk for a box of .22-caliber
rape. The girl was charged with being an accom- shells. The store owner had instructed her
plice to statutory rape. employees not to sell ammunition to anyone
under the age of 18 and to always ask for identi-
At the girl’s trial, what is her best defense? fication. The clerk asked the boy his age. The
boy said he was 20. The clerk then placed a box
(A) Because the man has not yet been con- of shells on the counter and asked, “Anything
victed, the girl may not be convicted as an else?” The boy said that was all he wanted but
accomplice. then discovered he did not have enough money
to pay for the shells, so the clerk put the box
(B) The statutory rape law is designed to back onto the shelf.
protect minors and therefore the girl cannot
be convicted as an accomplice. If the owner of the store is charged with
attempting to violate the statute, what would be
(C) As a minor, the girl does not have the her best argument?
capacity to be an accomplice.
(A) It was impossible for the sale to have oc-
(D) Because the girl is a prostitute and curred.
consented to the sexual acts, the man
cannot be convicted of statutory rape; (B) She had strictly instructed her employees
therefore, the girl cannot be an accomplice. not to sell ammunition to minors.

(C) The boy lied about his age.

(D) The clerk did not have the mental state


needed for the crime charged.

一 信:liuxue119118 , 们 信免 供
CRIMINAL LAW -
INCHOATE CRIMES AND
ACCOMPLICES ANSWERS
CRIMINAL LAW - INCHOATE CRIMES AND ACCOMPLICES ANSWERS 1.

CRIMINAL LAW - INCHOATE CRIMES AND ACCOMPLICES ANSWERS

Answer to Question 1

(A) The boyfriend is an accessory before the fact, and thus subject to the same penalty as the student.
In a jurisdiction applying the common law, an accessory before the fact is someone who provides
assistance before the crime is committed but who is not actually or constructively present during
the commission of the crime. Here, the boyfriend stole the drug from the chemistry lab but was
not present when the student laced the cafeteria’s sugar bowls with the drug. Thus, the boyfriend
is an accessory before the fact. (B) is incorrect because a principal in the first degree actually
perpetrates the crime, and here the boyfriend did not physically place the drugs in the sugar
bowls. (C) is incorrect because a principal in the second degree must be actually or constructively
present during the commission of the crime, and here the boyfriend was not present during the
commission of the crime. (D) is incorrect because the boyfriend’s aid was before, not after, the
fact. An accessory after the fact is one who helps the perpetrator elude capture; the boyfriend’s
conduct does not fit this description. Furthermore, accessories after the fact typically are not held
liable for the principal crime. Aiding one to escape prosecution or apprehension generally is a
separate crime.

Answer to Question 2

(C) The friend should not be found guilty as an accomplice to bigamy because he did not have the
requisite mental state. To hold a person liable as an accomplice to a crime, the state must show
that his assistance or encouragement was given with the intent to assist the principal and the intent
that the principal commit the crime. The facts indicate that the friend did not have this state of
mind. The friend did not know that the man was already married, and thus the friend is not guilty,
because he did not have the mental state required for accomplice liability. (A) is wrong because
it involves a mistaken legal premise insofar as liability as an accomplice is concerned. (B) is
accurate in part, but ignores the requirement of a mental state. (D) is wrong because the accom-
plice can be liable even if he did not “cause” the crime.

Answer to Question 3

(B) If the hitman was insane, the wife cannot be said to have conspired with him to commit the
murder. A conspiracy consists of: (i) an agreement between two or more persons; (ii) an intent to
enter into an agreement; and (iii) an intent to achieve the objective of the agreement. The parties
must agree to accomplish the same objective by mutual action. This agreement requires a meeting
of at least two “guilty minds”; i.e., two persons who are actually committing themselves to the
scheme. If the hitman was insane, then he lacked the capacity to intentionally enter into an agree-
ment to kill the husband and to intend to achieve the murder. Under this circumstance, the wife
was the only one who was actually committed to the crime and who intended to bring it about.
Because there was no meeting of the wife’s “guilty mind” with at least one other, the wife cannot
be convicted of conspiracy under the common law rule. (A) is incorrect because a defendant can
be tried and convicted of both conspiracy and the substantive offense (here, attempted murder).
(C) is incorrect because a conviction of conspiracy does not require that all parties be tried and
convicted. While an acquittal of all persons with whom a person is alleged to have conspired
precludes conviction of the remaining defendant, that is not the case here. The hitman was not
acquitted. Also, note that a state’s decision to discontinue prosecution would not be deemed to
be an acquittal, and thus most likely a state’s inability to prosecute because of a co-conspirator’s
death would not be an acquittal. (D) is incorrect for two reasons: (i) withdrawal from a conspiracy
2. CRIMINAL LAW - INCHOATE CRIMES AND ACCOMPLICES ANSWERS

is not a defense to a charge of conspiracy, and (ii) withdrawal is a defense to subsequent crimes
committed by other members of the conspiracy only if the withdrawing party performs an affir-
mative act notifying all members of the conspiracy in time for them to have the opportunity to
abandon their plans. Here, the wife notified the priest, but took no further action to thwart the
conspiracy. Thus, she has no defense to the conspiracy, and it is unlikely that merely telling a
priest would be a sufficient withdrawal as to subsequent crimes.

Answer to Question 4

(D) The man is not guilty of conspiracy because he never had the required mental state for conspiracy.
Conspiracy requires: (i) an agreement between two or more persons; (ii) the intent to enter into an
agreement; and (iii) the intent to achieve the objective of the agreement. The two intent elements
indicate that conspiracy is a specific intent crime. Here, because the man planned to contact the
police before his cousin could commit the crime, he never intended to achieve the criminal objec-
tive of robbing the bank. Thus, he cannot be found guilty of conspiracy. (A) is incorrect because
the man never entered into the conspiracy to begin with since he lacked the requisite intent; thus,
he did not need to withdraw. (B) is incorrect because merely providing the instrumentality to
commit the crime is insufficient by itself to establish liability for conspiracy. There also must
be some intent to facilitate the crime, which is absent here. (C) is incorrect because, as stated
above, the man never entered into the conspiracy. If he had, (C) would still be incorrect because,
at common law and in most states, withdrawal is not a defense to the conspiracy charge itself,
merely a limitation on liability for the subsequent acts of the co-conspirators.

Answer to Question 5

(D) The man’s best defense is lack of intent. The crime of attempt is a specific intent crime, requiring
the specific intent to carry out the crime in question. Since the man did not know that vandals
had smashed his taillights, he never intended to drive an automobile at night without functioning
taillights. (A) is incorrect because he did not attempt to drive the automobile at the instigation
of the police. (B) is incorrect because legal impossibility is not applicable to these facts. Legal
impossibillity is a defense when a defendant, having completed all the acts that he had intended,
would have committed no crime. Here, if the man had been able to start his car and drive away,
he would have been driving an automobile at night without functioning taillights. (C) is incorrect
because factual impossibility is not a good defense to a charge of attempt. It is no defense that
the substantive crime is incapable of completion due to some physical or factual condition that is
unknown to the defendant.

Answer to Question 6

(D) The hunter and his nephew will be convicted because they agreed to hunt grizzly bears on a
date that was out of hunting season. Conspiracy requires (i) an agreement between two or more
persons; (ii) an intent to enter into an agreement; and (iii) an intent to achieve the unlawful
objective of the agreement. Here, the hunter and his nephew, with the intent to do so, agreed to
hunt grizzly bears on a particular date when hunting was illegal. While they did not know that
hunting was illegal on that date, their ignorance of the state law on that point does not negate their
intent to commit the act and is not a defense. The majority rule is that the parties to a conspiracy
need not have been aware that their plan was an illegal one. While a few courts hold that the
conspirators must have known that their objective was criminal unless the crime was malum
in se (“corrupt motive” doctrine), most courts reject this exception. Hence, the hunter and his
nephew can be convicted of conspiracy. (A) is incorrect because the facts show an agreement
CRIMINAL LAW - INCHOATE CRIMES AND ACCOMPLICES ANSWERS 3.

between the two. (B) is incorrect because they intended to commit the act that constituted the
crime. As discussed above, the fact that they mistakenly believed that their objective was legal
is not a defense in most jurisdictions. (C) is incorrect because when a defendant is charged with
conspiracy, even conspiracy to commit a strict liability offense, an intent to achieve the objective
of the agreement must be shown.

Answer to Question 7

(B) The defendant is guilty of attempted receipt of stolen property. The general rule is that a defen-
dant cannot avoid liability for attempt on the basis that it would have been impossible to commit
the completed crime. It applies to cases where the defendant’s mistaken belief about the facts
(i.e., factual impossibility) or about the legal relationships or circumstances prevents him from
completing the crime that he intended to commit. Here, the defendant’s mistake about the legal
status of the goods he fenced does not constitute a defense to attempt. The legal status of the
goods, unlike the existence or nonexistence of a law proscribing the conduct, is one of the atten-
dant circumstances; this is treated like factual impossibility by most courts. By accepting the
goods he believes to be stolen, the defendant has purposely engaged in conduct that would consti-
tute receipt of stolen goods if the circumstances were as he believed them to be; hence, he is liable
for attempted receipt of stolen goods. (A) is incorrect. The jewelry in the fact pattern has lost
its status as “stolen” jewelry because it was being forwarded to the defendant with the owners’
permission; it was therefore impossible for him to be liable for the completed crime of receipt of
stolen goods. (C) is incorrect because the defendant’s mistake is not a mistake of law. If a defen-
dant otherwise has the kind of culpability required for the completed crime, his objective must
be proscribed by law. If those things the defendant does or intends to do would not actually be a
crime, this is legal impossibility and a defense, so the defendant would not be guilty of an attempt
even though he firmly believed that his goal was criminal. Here, however, the defendant’s mistake
was not a mistake of law, as discussed above. (D) is incorrect even though it correctly states that
the jewelry has lost its “stolen” status. As discussed above, the defendant is guilty of attempted
receipt of stolen property.

Answer to Question 8

(C) The wife could be convicted of assault and either attempted murder or attempted manslaughter.
To be liable for either attempted murder or attempted manslaughter, the defendant must have
acted with the intent to kill and have committed an act beyond mere preparation for the offense.
The fact that it is not possible to complete the intended offense (factual impossibility) is not a
defense to liability for attempt. Here, the wife clearly intended to kill the secretary and did every-
thing in her power to carry out the killing. The fact that the gun was not loaded is no defense.
(B) and (D) are wrong because the wife could also be convicted of assault. Assault is either: (i)
an attempt to commit a battery; or (ii) the intentional creation (other than by mere words) of a
reasonable apprehension in the mind of the victim of imminent bodily harm. Here, the wife has
committed both types of assault: She attempted to commit a battery against the secretary and
intentionally placed her in fear of imminent bodily harm. Although she could not be convicted of
the first type of assault if she were also convicted of attempted murder or attempted manslaughter
(because that type of assault is a lesser-included offense that merges into the greater offense), she
could be convicted of the second type of assault (it does not merge because it contains elements
not encompassed by attempted murder or attempted manslaughter). (A) is wrong because the
wife could be convicted of attempted murder instead of attempted manslaughter if she cannot
establish adequate provocation. An intentional killing can be reduced from murder to voluntary
manslaughter if (i) there exists a provocation that would arouse sudden and intense passion in the
4. CRIMINAL LAW - INCHOATE CRIMES AND ACCOMPLICES ANSWERS

mind of an ordinary person so as to cause her to lose self-control; (ii) the defendant in fact was
provoked; (iii) there was insufficient time for the passions of a reasonable person to cool; and (iv)
the defendant in fact did not cool off between the provocation and the killing. Many common
law courts recognized the existence of provocation in only two instances: exposure to a threat of
deadly force and discovery of one’s spouse in bed with another person. Furthermore, some provo-
cations, such as “mere words,” were defined as inadequate provocation as a matter of law. Modern
courts tend to be more reluctant to take such cases from juries and are more likely to submit to
the jury the question of whether “mere words” or similar matters constitute adequate provocation.
These principles apply in this case even though the wife’s liability is only for attempted murder
or manslaughter rather than the completed offense. Here, the wife’s discovery of the emails and
photographs might be sufficient to make it a jury question as to whether adequate provocation
existed. However, the jury might still conclude that the provocation was not sufficient and find the
wife liable for attempted murder rather than attempted manslaughter.

Answer to Question 9

(D) Although the defendant might have been indifferent with respect to the boyfriend’s marital status,
the facts do not show that the defendant intended to see the offense of bigamy committed. To
establish criminal liability under an accomplice theory, it must be established that the defendant
helped with the commission of the crime with the intent to assist the principal and the intent that
the principal commit the crime. A mental state of recklessness is not sufficient to justify a convic-
tion as an accomplice, regardless of the mental state required by the definition of the principal
crime. (A) is incorrect because, although the defendant did aid and encourage the marriage, he
cannot be held as an accomplice without the necessary intent, which was lacking. (B) is incor-
rect. Although bigamy is a strict liability offense when applied to the actual participants in the
bigamous marriage, the defendant is not a participant to the marriage. (C) is incorrect because,
even if the defendant’s actions amounted to a breach of duty, he still did not possess the mental
state necessary for accomplice liability.

Answer to Question 10

(B) The corporate officers cannot be guilty of attempted murder because criminal attempt requires a
specific intent to commit a particular completed crime. A criminal attempt is an act that, although
done with the intention of committing a crime, falls short of completing the crime. A defendant
must have the intent to perform an act and obtain a result that, if achieved, would constitute a
crime. Hence, attempt is a specific intent crime. Guilt of attempted murder requires that a defen-
dant have the specific intent to commit the crime of murder. In contrast to the malice aforethought
required for murder, which is satisfied not only by intent to kill but also by awareness of an unjus-
tifiably high risk to human life, intent to inflict great bodily injury, or intent to commit a felony,
attempted murder is satisfied only by intent to kill. Here, the corporate officers, in permitting the
mining to proceed without compliance with methane detection measures, subjected the miners
to an unjustifiably high risk to their lives. This “abandoned and malignant heart” demonstrated
by the officers would suffice to convict the officers of murder if the miners had been killed.
However, there were no deaths; the crime charged is attempted murder. Consequently, there must
be a showing that the officers specifically intended to cause the deaths of the miners. Because
the facts do not indicate such intent, the officers are not guilty of attempted murder. (C) is incor-
rect because, as explained above, the mental state of an abandoned and malignant heart will not
satisfy the intent required for attempted murder. (D) is incorrect because, even though a felony
was committed, the intent to commit a felony (although sufficient to constitute malice afore-
thought for a murder charge) will not satisfy the intent to kill that is necessary for a conviction of
CRIMINAL LAW - INCHOATE CRIMES AND ACCOMPLICES ANSWERS 5.

attempted murder. (A) is incorrect because the officers’ actions had gone well beyond the planning
stage; allowing the mining to proceed without compliance with methane detection measures satis-
fies the overt act requirement for attempt. Had the other requirements for attempt been satisfied,
their actions would be sufficient to support a conviction.

Answer to Question 11

(C) The student can be convicted of an attempt to smoke a marijuana cigarette. Whether impossibility
of success constitutes a defense to a charge of criminal attempt depends on the type of impossi-
bility at issue. It is no defense to a charge of criminal attempt that it would have been impossible
for the defendant to do all of those things that he intended to do, had the attendant circumstances
been as the defendant believed them to be. This is known as factual impossibility. It is this type
of impossibility that is present in the student’s case. Had the attendant circumstances been as he
believed them to be, i.e., that the cigarette contained marijuana, he would be guilty of the substan-
tive offense. Thus, impossibility is not a defense in this case. (B) is incorrect. Although corrobo-
ration of a defendant’s confession is required for a conviction, corroboration here could have
come from the detective’s testimony. (A) is incorrect because it ignores the effect of the attendant
circumstances. Under the better view, one must also consider the “facts” as the defendant believed
them to be. Here, the student believed that the cigarette contained marijuana. As a result, impos-
sibility here does not provide a defense. (D) is incorrect because attempt crimes are specific intent
crimes, not strict liability crimes. Thus, whether the drug offense in question here was a strict
liability crime is irrelevant.

Answer to Question 12

(C) The liquor store clerk can be convicted of conspiracy in a unilateral jurisdiction even though
the minor can be convicted of neither solicitation nor conspiracy. Under the unilateral (Model
Penal Code) approach, the crime of conspiracy requires only one party to have genuine criminal
intent, and may be shown by proof that the defendant agreed with another to commit a crime
(even if that other person does not share the commitment). Thus, the fact that no other party to
the conspiracy could be found guilty does not prevent the defendant from being convicted of
conspiracy. Here, the fact that the minor is a member of the class that the statute was designed to
protect prevents him from being found guilty of conspiracy; (B) is therefore incorrect. However,
unlike under the common law bilateral approach, under the unilateral approach this fact has no
bearing on the liquor store clerk’s liability for conspiracy. Thus, (C) is correct and (D) is incorrect.
(A) is incorrect because the crime of solicitation is treated the same as conspiracy. If the solicitor
could not be guilty of the completed crime because of legislative intent to exempt him, he cannot
be found guilty of solicitation of the crime. Because the minor is a member of the class intended
to be protected by the statute, he cannot be found guilty of soliciting the liquor store clerk to
commit it.

Answer to Question 13

(C) The former employee is guilty of robbery and aggravated battery. Robbery is (i) the taking (ii) of
the personal property (iii) of another (iv) from the other person’s presence (v) by force or intimida-
tion (vi) with the intent to permanently deprive the person of the property. The former employee
clearly committed a robbery by forcing the store manager, at gunpoint, to empty the contents of
the safe into a sack. Given that the former employee did not physically commit either the rape or
the aggravated battery, criminal liability would hinge on whether he could be held accountable
based on a conspiracy theory. Provided that a conspiracy exists, a person will be criminally liable
6. CRIMINAL LAW - INCHOATE CRIMES AND ACCOMPLICES ANSWERS

for the acts committed by a co-conspirator that were committed in furtherance of the objectives
of the conspiracy and were a natural and probable consequence of the conspiracy (i.e., foresee-
able). At common law, a conspiracy exists when there is (i) an agreement between two or more
persons; (ii) the intent to enter into the agreement; and (iii) the intent to achieve the objective
of the agreement. Here, the former employee and his friend intentionally entered into an agree-
ment to rob the grocery store. Furthermore, the fact that they armed themselves shows that they
considered the possibility that they might need to use force during the crime; thus, the fact that
the friend actually used force by shooting the store manager was foreseeable. The shooting was
also done in furtherance of the conspiracy, as it was intended to aid in the pair’s getaway. Thus,
the former employee may be found guilty of the aggravated assault on the store manager, and (A)
and (B) are incorrect. However, the rape was neither committed in furtherance of the conspiracy
(it did nothing to further the robbery or aid in the pair’s getaway), nor do the facts indicate that it
was foreseeable that the friend would commit such a crime during a robbery. Under these circum-
stances, criminal liability probably would not attach to the former employee for the acts of his
friend in raping the produce clerk, making (B) and (D) incorrect.

Answer to Question 14

(C) The best theory for charging the friend is that his own recklessness caused the bodily injury to
the person of another. It may be that the friend is not guilty of any crime, given that his actions,
and the actions of the student, may not have risen to the level of recklessness. Nonetheless, if he
is charged, it will be for his own conduct in bringing about the battery, as explained below. (A) is
incorrect. An accomplice is one who (i) with the intent to assist the principal and the intent that
the principal commit the crime (ii) actually aids, counsels, or encourages the principal before or
during the commission of the crime. When the substantive offense has recklessness as its mens
rea, most jurisdictions hold that the intent element is satisfied if (i) the accomplice intended to
facilitate the commission of the crime, and (ii) acted with recklessness. The accomplice is respon-
sible for the crimes he did or counseled and for any other crimes committed in the course of
committing the crime contemplated, as long as the other crimes are probable or foreseeable. Here,
it is clear that the friend is an accomplice of the student for the “crime” of playing a prank on the
teacher. However, the friend lacked the intent to facilitate the commission of the crime of battery
on the teacher, and thus he probably would not be liable as an accomplice for the crime of battery.
Even if playing the prank would be a crime in the jurisdiction, to be liable as accomplice to the
subsequent battery, it would have to be shown that it was probable or foreseeable that the teacher
would be battered as a result. It is doubtful whether such probability or foreseeability exists under
these facts. (B) is incorrect. Conspiracy requires (i) an agreement between two or more persons,
(ii) the intent to enter into an agreement, and (iii) the intent to achieve the objective of the agree-
ment. (Although not relevant here, most states also would require an overt act in furtherance
of the conspiracy.) Conspirators will be liable for all crimes committed in furtherance of the
conspiracy that were a natural and probable consequence of the conspiracy. Here, the “crime”
that the friend and the student “conspired” to commit was playing a prank on the teacher; they
did not conspire or intend to commit a battery. The battery was not committed in furtherance of
the conspiracy, and it is doubtful whether the battery would be foreseeable, as stated above. (D) is
incorrect. There is no such rule imposing vicarious liability on a defendant just for furnishing the
instrumentality of the crime.

Answer to Question 15

(A) The defendant may also be found guilty of conspiracy if he is found guilty of burglary. One who
solicits another to commit a crime cannot be convicted of both the solicitation and the completed
CRIMINAL LAW - INCHOATE CRIMES AND ACCOMPLICES ANSWERS 7.

crime. Likewise, one who completes a crime after attempting it may not be convicted of both the
attempt and the completed crime. However, if conspirators are successful, they can be convicted
of both criminal conspiracy and the crime they committed pursuant to the conspiracy (i.e.,
conspiracy does not merge with the completed offense). Thus, if the defendant is found guilty
of burglary, he cannot also be convicted of either attempt or solicitation. The defendant can be
convicted of conspiracy in addition to burglary (with conspiracy liability being based on the
apparent agreement between the defendant and the other person to bring about the burglary of
the home). (B) incorrectly states that the defendant may be found guilty of solicitation as well
as the principal offense of burglary. (C) and (D) make the same incorrect assertion. In addition,
(D) incorrectly states that the defendant may be found guilty of attempt as well as the completed
crime, and that he may not be found guilty of conspiracy.

Answer to Question 16

(B) The owner committed attempted receipt of stolen property. One commits the crime of receipt
of stolen property by receiving possession and control of “stolen” personal property, known
to have been obtained in a manner constituting a criminal offense by another person with the
intent to permanently deprive the owner of his interest in the property. The property must have
“stolen” status at the time it is received by the defendant. Here, the defendant did not receive any
actual stolen property, only an empty box. Thus both (A) and (C) are incorrect. (C) is also incor-
rect under the common law definition of conspiracy, which requires that there be an agreement
between two or more conspirators. Unless the jurisdiction follows the unilateral approach (which
does not require two or more culpable parties), the fact that the officer was only feigning agree-
ment would preclude the shop owner’s liability for conspiracy. (B) is the best answer. Criminal
attempt is defined as an act that, although done with the intention of committing a crime, falls
short of completing the crime. There must be a specific intent to commit the crime and an overt
act in furtherance of that intent. The shop owner intended to receive stolen property and delivered
a payment to the undercover officer in furtherance of that intent. While there was no actual receipt
of stolen goods, had the circumstances been as the shop owner believed them to be he would have
been guilty of receipt of stolen property. In most states and under the MPC, factual impossibility
is not a defense to an attempt charge, so it is not a defense that there was no actual stolen property
involved. Because (B) is correct, (D) is incorrect.

Answer to Question 17

(B) The girl’s best defense is that she cannot be convicted under the statute because it was designed
to protect minors. Statutory rape is the crime of carnal knowledge of a minor under a certain
age. In theory, a minor female could be convicted as an accomplice to statutory rape (such as by
being an accomplice to the statutory rape of another minor by an adult perpetrator). If, however,
the legislative intent is to protect members of a class from exploitation or overbearing, members
of that class are presumed to be immune from liability, even if they participate in the crime. Here,
the girl can argue that the crime of statutory rape was intended to protect minors from exploita-
tion, and therefore the legislative intent is that she, as the minor victim, be immune from prosecu-
tion. (A) is wrong because an accomplice may be convicted before the principal. Although the
common law rule required that the principal’s guilt be determined first, or at least at a joint trial,
this rule has been abandoned by most jurisdictions. (C) is wrong because it is too broad; as noted
above, a minor may be an accomplice to a crime. To be convicted as an accomplice, a person must
have acted with the intent to aid or encourage the principal in the commission of the crime. As
long as a child is old enough to be able to form this intent, she could be convicted as an accom-
plice. A 15-year-old is certainly capable of forming that intent. (Even under the common law
8. CRIMINAL LAW - INCHOATE CRIMES AND ACCOMPLICES ANSWERS

presumptions, children over age 14 are treated as adults.) (D) is wrong because the man could be
convicted despite the girl’s consent. Consent is not a defense to statutory rape; even if the female
willingly participates in the sexual acts, the male may be convicted. Thus, the girl’s consent, or
her status as a prostitute, is not the reason she may not be convicted as an accomplice.

Answer to Question 18

(D) The store owner will not be convicted of an attempt to violate the statute if her employee did not
have the requisite intent. Although the statute has been interpreted to create a strict liability crime,
which does not require proof of criminal intent, an attempt of a strict liability crime requires
proof that the defendant acted with the intent to bring about the proscribed result. Therefore, for
the store owner to be charged vicariously with attempt, her employee must have acted with the
requisite intent; he must have intended to sell the ammunition to a minor. If he did not so intend,
the store owner will not be convicted of attempt. (A) is incorrect because this is a case of factual
impossibility, which is not a defense to attempt. (B) is incorrect because careful instructions will
not, in and of themselves, absolve an employer from vicarious liability. (C) is incorrect because
the strict liability elements of the underlying offense make it clear that knowledge of the age of the
purchaser is not an element of the underlying offense. Thus, the clerk (and the store owner) can
be liable for selling ammunition to a minor no matter how old the purchaser looked or how old he
claimed to be. The boy’s lie may have bearing on the clerk’s lack of intent, but this is not as direct
an answer as (D).
CRIMINAL LAW - OTHER
CRIMES QUESTIONS
CRIMINAL LAW - OTHER CRIMES QUESTIONS 1.

CRIMINAL LAW - OTHER CRIMES QUESTIONS

Question 1 Question 2

A state has a modern theft statute that The defendant was walking down a street
combines such common law crimes as larceny, when he realized that a long-lost friend was
embezzlement, and receiving stolen property into walking in the opposite direction. The defendant
one comprehensive crime. The police arrested a started waving his arms in a desperate attempt
car dealer who had been dealing in stolen cars. to get his friend’s attention. The defendant did
When the police suggested strongly that it might not notice an old woman, who was walking past
be in his best interests to cooperate with a police him, and negligently struck her with his elbow
“sting” operation, the dealer readily agreed. The with sufficient force to cause her to stumble to
police told the dealer to continue to purchase the pavement. The fall did not, however, cause
stolen autos from thieves, but to inform the police her any major injury.
when a buyer was about to purchase a stolen car.
The police would then arrest the buyer. If the defendant is prosecuted for criminal
battery, what is the most likely verdict?
The defendant approached the dealer on his
car lot, asking for a late-model used car at a good (A) Not guilty, because his act did not cause
price. The dealer told the defendant that he had a serious bodily injury.
sedan that he was willing to sell, and he quoted
a price that was half “book value” of the car. (B) Not guilty, because he did not have the
The dealer told the defendant that the sedan was mental state required for criminal battery.
“hot,” to which the defendant responded, “It’s
none of my business where you got the car; I just (C) Guilty, because he failed to exercise due
asked for a late-model car at a good price.” The care in flailing his arms about near a public
defendant then gave the dealer the money and the sidewalk.
dealer gave the defendant the keys. The dealer
went to his office to complete the paperwork and (D) Guilty, because he caused an offensive
informed the police of the transaction. The defen- touching.
dant was arrested and charged with theft.

As an attorney assigned to the district attor-


ney’s felony review group, should you advise that
the charge should be dropped?

(A) Yes, because the defendant was entrapped.

(B) Yes, because the dealer was an agent of the


police, and when the stolen car came into
his hands it was as if it had been returned to
the rightful owner.

(C) No, because the police never recovered


possession of the car, and it retained its
character as stolen property when the
defendant purchased it knowing it had been
stolen.

(D) No, because the defendant had the specific


intent to purchase a stolen car.
2. CRIMINAL LAW - OTHER CRIMES QUESTIONS

Question 3 Question 4

A scientist overheard from some friends that A homeowner was trimming his sidewalk-
a young woman was suffering from a rare blood bordering hedge when a wasp began attacking
disease. The scientist told the young woman him. The homeowner attempted to hit the wasp.
that he could cure her by giving her a special During one of his swats, the homeowner struck
elixir he had invented. The scientist charged a jogger in the face. The jogger, reacting to the
the young woman $500 for the elixir. The elixir unexpected blow to his head, reached into his
had no effect on the blood disease. The scien- pocket and pulled out a knife. The homeowner
tist is charged with obtaining money by false tried to shield himself by raising his arms in
pretenses. front of his body, but the jogger was able to stab
the homeowner, seriously injuring him.
Which of the following, if true, would not
absolve the scientist of guilt for obtaining money If the jogger is prosecuted for aggravated
by false pretenses? battery, is it likely that he will be found guilty?

(A) The scientist was playing a practical joke (A) No, because he believed the homeowner
on the young woman and intended to return was attacking him.
the money.
(B) No, because he was adequately provoked by
(B) The scientist honestly believed that the the homeowner.
elixir would cure the blood disease, but his
belief was unreasonable. (C) Yes, because he used a deadly weapon.

(C) The young woman honestly believed that (D) Yes, because he intentionally created a
the elixir would cure the blood disease, but reasonable apprehension of imminent
her belief was unreasonable. bodily harm in the homeowner.

(D) The young woman was an undercover


police officer and did not believe that the
elixir would cure the blood disease.
CRIMINAL LAW - OTHER CRIMES QUESTIONS 3.

Question 5 Question 6

A con artist arranged with a local dog walker The victim was walking out of a store when
that he would pay the dog walker to give him she saw someone suddenly fall to the street
a dog owned by a rich client and then the dog suffering an apparent heart attack. However, that
walker would tell the owner that the dog ran person was in fact an accomplice of the defen-
away. The con artist would later return the dog to dant. With the victim’s attention momentarily
the owner, claiming he found the dog injured and diverted, the defendant removed the victim’s
had cared for it, and then demand reimbursement wallet from her purse. Another passerby shouted
for bogus veterinary bills. A week after getting to the victim, who turned and caught the defen-
the dog, the con artist contacted the owner. The dant by his sleeve. The defendant pushed her
owner invited the con artist into his home that hand away and started to run, but tripped over
evening to thank him for returning the dog, but a curb and dropped the wallet when he fell, and
the con artist grew frustrated when the owner the victim was able to recover it.
refused to pay the bogus vet bills. The con artist
impulsively swiped some cash he saw sitting The crimes below are listed in descending
on an end table when the dog’s owner was not order of seriousness.
looking.
What is the most serious crime for which the
The con artist is arrested. Based on the value defendant could be convicted?
of the dog and the dollar amount of the vet bills,
he is charged with misdemeanors for misappro- (A) Robbery.
priation of the dog and attempted theft by false
pretenses. (B) Attempted robbery.

Of what additional common law crime or (C) Larceny by trick.


crimes could the con artist be charged and
convicted? (D) Attempted larceny.

(A) Larceny.

(B) Burglary.

(C) Larceny and burglary.

(D) Larceny and conspiracy to commit


burglary.
4. CRIMINAL LAW - OTHER CRIMES QUESTIONS

Question 7 Question 8

A disgruntled customer of a business entered The defendant rented a room for two nights
its factory and announced that the products from at a motel. The room was equipped with a large
the factory were terrible. He set off a device that television set. The defendant decided to steal the
filled the factory with fumes, and then swiftly set, pawn it, and keep the proceeds. To conceal
ran out the main entrance. He rushed past a his identity as the thief, he contrived to make
visitor who was entering the factory, passing his room look as if it had been burglarized.
just inches in front of her. She was taken aback, However, he was traced through the pawnbroker
which caused her to trip and strike her head and arrested.
on the pavement. The customer was promptly
apprehended and charged with the misdemeanor On these facts, the defendant is guilty of what
of breach of the peace for his conduct in the crime?
factory and charged with assault against the
visitor. (A) Embezzlement.

Which of the following represents the custom- (B) False pretenses.


er’s best defense to the charge of assault?
(C) Larceny.
(A) The underlying offense was a misdemeanor
rather than a felony. (D) Larceny by trick.

(B) The customer made no physical contact


with the visitor.

(C) The visitor’s injury was not a foreseeable


consequence of the customer’s criminal
activity.

(D) The customer lacked the requisite mens rea.


CRIMINAL LAW - OTHER CRIMES QUESTIONS 5.

Question 9 Question 10

A borrower owed a lender $1,000. The debtor A student broke into her professor’s office to
had promised to pay the lender back in one look at examination questions without taking
week, but three months passed and no money them. Once in the office, the student was unable
was forthcoming. The debtor always managed to locate the examination questions, as they were
to avoid the lender’s calls, so the lender drove locked in a desk drawer. The student mistakenly
to the debtor’s house one night, intending to believed that looking at examination questions
demand repayment in person. The lender rang was a felony in the jurisdiction; in fact, such
the debtor’s doorbell, knocked on his door, and conduct would be a misdemeanor at most. The
screamed for the debtor to come out, but no one jurisdiction includes offices as structures that
responded. The lender then tried the doorknob could be burglarized.
on the closed front door. To her surprise the
door was unlocked, and she entered the debtor’s If the student were to be apprehended and
house. After yelling several times for the debtor, charged with burglary, is she likely to be
the lender concluded that the debtor was not at convicted?
home. Convinced that the debtor had run out
the back door to avoid her, the lender went to (A) No, because she did not complete the crime
the debtor’s living room, grabbed an overstuffed and she has not been charged with attempt.
chair, and carried it to the debtor’s front lawn.
The lender then doused the chair with lighter (B) No, because what she intended to do when
fluid and set it afire. Alarmed at the flaming she broke in was not a felony.
chair, one of the debtor’s neighbors called the
police. The police found the lender still standing (C) Yes, because she had the necessary mental
on the lawn next to the smoldering chair. state and committed the act of breaking and
entering.
Assuming that the jurisdiction has not statu-
torily amended the common law elements of the (D) Yes, because factual impossibility is not a
crimes below, with which of the following may defense.
the lender be properly charged?

(A) Arson only.

(B) Larceny only.

(C) Arson and burglary.

(D) Burglary and larceny.


6. CRIMINAL LAW - OTHER CRIMES QUESTIONS

Question 11 Question 12

An employee worked as a third-shift super- A husband’s and wife’s Social Security retire-
visor at a manufacturing plant. One of his duties ment benefits came in a single check payable to
was to ensure that all timekeeping records both each month, two-thirds of which was the
accurately reflected the time his crew actually husband’s retirement payment and one-third
worked. Workers, including the employee, were of which was the wife’s spousal benefit. Each
then paid for whatever hours the timecards month when the check arrived in the mail, the
reflected. The employee was also required wife would take it to their bank and cash it,
to assist in submitting budgets for payroll. receiving the entire proceeds in cash, which
Needing to leave work early for a second job she would use for her and her husband’s living
that he obtained, the employee had one of his expenses. After the husband died, the Social
trusted co-workers punch his card out at the Security check continued to come in the same
regular time every day of the week. At the end amount and made payable to both the husband
of the week, he signed the timecard with those and wife. The wife knew that she was no longer
hours included, and was paid accordingly. He entitled to her husband’s benefit, but that her
continued to do this for several weeks before own spousal benefit would increase greatly
being discovered. as a widow’s benefit. She also knew that she
would receive a one-time “death benefit.” She
What crime has the employee committed? concluded that the continued receipt of the
combined check reflected these increases, so
(A) Forgery. for several months after her husband’s death she
continued to cash the combined check, signing
(B) Embezzlement. both her and her husband’s names when she
negotiated it. When the federal government
(C) Larceny by trick. eventually processed the notification of the
husband’s death provided by the funeral home,
(D) False pretenses. it discovered that the wife had negotiated checks
containing $2,000 in benefits to which she was
not entitled.

Is she guilty of obtaining the payments by


false pretenses?

(A) Yes, because her actions constituted a pub-


lic welfare offense for which she is strictly
liable.

(B) Yes, because she should have known that


the combined check, payable in part to
a deceased person, contained benefits to
which she was not entitled.

(C) No, as long as she reasonably believed


that she was entitled to the funds in the
combined check.

(D) No, because she believed that she was


entitled to the total amount of the combined
check.
CRIMINAL LAW - OTHER CRIMES QUESTIONS 7.

Question 13 Question 14

A neighbor of a teenager purchased an expen- On the last play of a playoff football game,
sive sports car and parked it in his attached a game-winning touchdown was nullified by a
garage. To get into the garage, the teenager questionable penalty called by the referee. To
manufactured a radio device that could be register her displeasure but without intending
programmed to activate any automatic garage to hit anyone, a fan sitting in the stands threw a
door opener, and the teenager used the device bottle onto the field that just missed the head of
to open the neighbor’s garage one night. The the referee, who was looking in the other direc-
teenager hotwired the car and drove it away, tion and did not see the bottle being thrown. The
intending to cruise by a few of his friends’ fan was charged with assault.
favorite hangouts and then return it to its
owner with no one the wiser. After driving the Should the fan be convicted?
car around the town for a while, the teenager
decided to take his friends for a ride to the (A) Yes, because the throwing of the bottle was
nearby seacoast. When the party arrived at the a substantial step.
bluffs above the beach, the teenager impul-
sively decided to push the car over the cliff. The (B) No, because the referee did not see the
jurisdiction’s penal statutes have codified the bottle.
common law without alteration of any kind.
(C) No, because the fan did not intend to hit
Of what is the teenager guilty? anyone.

(A) No crime. (D) No, because the referee did not see the
bottle, nor did the fan intend to hit anyone.
(B) Burglary.

(C) Larceny.

(D) Both burglary and larceny.


8. CRIMINAL LAW - OTHER CRIMES QUESTIONS

Question 15 Question 16

The victim and another player regularly A worker was in the habit of carrying a lot
played backgammon together. The other player of cash with him after payday. His good friend
was entered in an upcoming backgammon was worried that someday the worker might
tournament and asked the victim if she could get robbed. To teach him to be more careful,
borrow his attractive custom-made backgammon and intending only to frighten him, the friend
set. He agreed, provided in return that he could purchased a realistic-looking toy gun and a
hold her backgammon computer until the face mask and hid in the bushes one night after
backgammon set was returned. After having payday, waiting for the worker to come home.
success at the tournament with the victim’s set, As the worker passed by, the friend jumped out
the other player decided to travel to the national of the bushes, pointed the toy gun at him, and
championship tournament, but she needed her took all of his money. The worker was badly
backgammon computer to hone her game. Not frightened by the incident. Shortly thereafter,
wanting to go to the nationals with her own the friend returned the money to the worker and
cheap set, she told the defendant, another weekly explained why he had staged the holdup.
club player, that her backgammon computer had
been borrowed by the victim, and asked him to The crimes below are listed in descending
get it from the victim’s car at the next meeting of order of seriousness.
the club. At the backgammon club meeting, the
defendant, who was not aware of the arrange- Which of the following is the most serious
ment between the victim and the other player, crime for which the friend can be convicted?
removed the computer from the victim’s car
without telling him. The next day, while waiting (A) Armed robbery.
to meet the player at the airport to give her the
computer, the defendant was playing a game (B) Robbery.
on it when a stranger stopped to admire it. The
stranger offered the defendant $400 for the (C) Assault.
computer, and the defendant readily agreed.
(D) Larceny.
The defendant is prosecuted for larceny of
the backgammon computer. Will the defendant
likley be found guilty?

(A) Yes, because the victim was rightfully in


possession of the computer when the de-
fendant took it.

(B) Yes, because the defendant sold the


computer to the stranger and pocketed the
proceeds.

(C) No, because the other player was the


rightful owner of the computer, and the
defendant was acting on the other player’s
behalf.

(D) No, because the defendant thought that the


other player was entitled to the computer
when he took it from the victim.
CRIMINAL LAW - OTHER CRIMES QUESTIONS 9.

Question 17 Question 18

During the nighttime, a woman broke into the An intoxicated airline passenger collecting
house of the victim with the intention of stealing his bags after passing through security grabbed
his diamond ring. When she could not find the a garment bag from its owner because he
diamond ring, she became angry, lit a match to mistakenly thought it was his own. The owner
a newspaper and threw it on the victim’s bed, of the garment bag tried to take it back from
setting the mattress on fire. The flames destroyed the passenger. During the tug of war over the
the bed and a portion of the floor under the bed. garment bag, the passenger knocked the owner
to the floor, took the garment bag, and ran to
In a common law jurisdiction, of which crime his gate. The passenger was later arrested and
or crimes is the woman guilty? charged with robbery.

(A) Burglary only. Should the passenger be found guilty?

(B) Arson only. (A) Yes, because his intoxication was volun-
tary.
(C) Burglary and attempted arson.
(B) Yes, because mistake is no defense to
(D) Burglary and arson. robbery.

(C) No, because he made no threats and was


intoxicated.

(D) No, because his mistake negated the


required specific intent.
CRIMINAL LAW - OTHER
CRIMES ANSWERS
CRIMINAL LAW - OTHER CRIMES ANSWERS 1.

CRIMINAL LAW - OTHER CRIMES ANSWERS

Answer to Question 1

(C) The charge should be pursued. The defendant is liable for theft by receipt of stolen property
because the car was still “stolen” when it was received by him. At common law and under modern
theft statutes, the crime of receiving stolen property requires (i) receiving possession and control
(ii) of “stolen” personal property (iii) known to have been obtained in a manner constituting a
criminal offense (iv) by another person, (v) with the intent to permanently deprive the owner of
his interest in the property. Here, all of the elements of the prima facie offense have been estab-
lished: the defendant “received” possession and control when he paid the dealer and received the
keys (the fact that he was arrested before he drove away is irrelevant because there is no aspor-
tation requirement as there is for larceny); the car qualifies as personal property; the defendant
knew that the car was stolen because the dealer informed him that it was “hot”; and the defendant
intended to keep the car and not return it to the original owner. In a “sting” operation such as the
one in this question, an attendant circumstance of the offense is that the property must still be
“stolen” at the time it is received by the defendant. Once stolen property is recovered by the owner
or by the police on the owner’s behalf, it loses its “stolen” status. Even if the owner consents to
the property’s use for the purpose of trapping a suspected recipient of stolen goods, the property
cannot be the basis of a receipt of stolen property charge because it is no longer “stolen.” This
situation is different, however. Here, neither the owner nor the police had recovered the stolen car
before the defendant took possession of it. The police have permitted the dealer to continue to
obtain stolen autos, but have not obtained the permission of the rightful owner of this car to use it
in their operation, because neither they nor the dealer know to whom this particular car belongs.
Therefore, the car retained its status as stolen property. [See, e.g., United States v. Muzii, 676 F.2d
919 (2d Cir.), cert. denied, 459 U.S. 863 (1982)] Thus, the dealer is liable for receipt of stolen
property. (A) is incorrect because, under the majority rule, entrapment requires the defendant to
prove that the criminal design originated with and was induced by the police and that he was
not in any way predisposed to commit the crime. Here, the police merely offered the opportu-
nity for the defendant to commit the crime. The defendant readily made the deal for the car even
though he knew it was stolen. (B) is incorrect. Even though the dealer arguably may have been an
agent of the police because he was cooperating with them, the police were not acting on behalf
of, and with the permission of, the rightful owner of the car that the defendant purchased. Thus,
that car never lost its status as stolen property even during the dealer’s possession of it. (D) is
wrong because intent is not required for the crime of receiving stolen property. While it is unclear
whether the defendant “intended” to purchase a stolen car, it is clear that the defendant knew he
was purchasing a stolen car, and knowledge satisfies the mens rea element of this offense.

Answer to Question 2

(B) The defendant will probably be found not guilty. Battery is an unlawful application of force to
the person of another resulting in either bodily injury or an offensive touching. The mental state
necessary for criminal battery is general intent, which is satisfied by intentional, reckless, or
criminally negligent conduct. Clearly, the defendant did not intend to have bodily contact with the
old woman, and in light of the circumstances it will be unlikely that his conduct will be consid-
ered such an extreme deviation from ordinary behavior as to establish criminal negligence. (A)
is wrong; a serious injury is not required for battery. (C) is wrong because criminal negligence
requires a greater deviation from ordinary behavior than simply a “failure to exercise due care,”
which is the tort standard. (D) is not as accurate as (B). It does not address the critical issue in the
case—the mental state of the defendant.
2. CRIMINAL LAW - OTHER CRIMES ANSWERS

Answer to Question 3

(C) It is of no help to the scientist that the young woman’s belief that the elixir would cure her disease
was unreasonable. It is not a defense to obtaining money by false pretenses that the victim unrea-
sonably relied on the defendant’s misrepresentation. The test for reliance is a subjective one. (A)
is incorrect because the obtaining of mere possession that the defendant expects to be tempo-
rary does not constitute false pretenses. (B) is incorrect because false pretenses requires that
the misrepresentation be knowingly false, and this is not the case when the defendant sincerely,
even though unreasonably, believes that the misrepresentation is true. (D) is incorrect because
false pretenses requires reliance by the victim to be actionable. (The scientist may be guilty of
attempted false pretenses, however.)

Answer to Question 4

(C) The jogger will likely be found guilty of aggravated battery. The jogger’s act constituted an
unlawful application of force to the person of another and is, thus, a battery. Use of a deadly
weapon in the commission of a battery elevates the crime to aggravated battery. (A) is wrong
because a person must reasonably believe that he is faced with imminent death or great bodily
harm in order to use deadly force. The accidental blow struck by the homeowner would not rise
to that level. (B) is incorrect. Although adequate provocation (e.g., being subjected to a serious
battery or a threat of deadly force) is enough to reduce a killing to voluntary manslaughter,
there is no rule justifying a battery based on adequate provocation. The fact that the jogger was
unexpectedly struck by the homeowner does not justify his battery of the homeowner. (D) is
wrong because it describes a type of assault, not battery. If there has been an actual touching of
the victim, a battery has been committed. In this case, the jogger actually touched the homeowner
when he stabbed the homeowner with the knife. Therefore, the jogger will be guilty of battery.

Answer to Question 5

(A) The only additional common law crime that the con artist could be convicted of is larceny.
Common law larceny consists of (i) a taking (ii) and carrying away (iii) of tangible personal
property (iv) of another (v) by trespass (vi) with intent to permanently (or for an unreasonable
time) deprive the person of an interest in the property. The con artist has committed larceny of
the cash because he took it from the dog owner by trespass (i.e., wrongfully) with the intent to
permanently deprive him of it. (B) and (C) are incorrect because the con artist has not committed
burglary. The elements of burglary at common law are (i) a breaking (ii) and entry (iii) of the
dwelling (iv) of another (v) at nighttime (vi) with the intent to commit a felony therein. While
the con artist had gained entry to the dog owner’s home with the intent to defraud the owner,
the facts do not suggest that a felony was involved (based on the dollar amount of the vet bills).
Furthermore, whether he committed a “breaking” is debatable. At common law, a breaking can
be constructive if the defendant gains entry by means of fraud, threat, or intimidation. The fraud,
however, is typically a misrepresentation of identity to trick the occupant into opening the door
of the dwelling. Here, while the con artist misrepresented how he had obtained the dog, the dog
owner invited him in as a gesture of thanks; i.e., he was not tricked into opening the door to let
him in. (D) is incorrect because there was no express or implied agreement between the con artist
and the dog walker to commit a burglary; hence, the con artist cannot be liable for conspiracy to
commit burglary.
CRIMINAL LAW - OTHER CRIMES ANSWERS 3.

Answer to Question 6

(A) The defendant could be convicted of robbery under these facts because of his use of force.
Robbery is larceny from the person or presence of the victim by means of violence or intimida-
tion. Larceny is the taking and carrying away of the tangible personal property of another by
trespass with the intent to permanently deprive the person of his interest in the property. The
use of force constituting a battery is sufficient for robbery, and the perpetrator’s use of force to
overcome the person’s resistance to the taking is also sufficient force to constitute a robbery. All
that is required for taking is that the perpetrator have possession of the item, and the slightest
movement will suffice for the carrying away. The fact that the defendant subsequently dropped the
wallet does not negate his completion of the crime; hence, (B) and (D) are incorrect. (C) is incor-
rect because larceny by trick involves the victim consenting to the defendant obtaining possession
of property as a result of a misrepresentation, which is not the case here.

Answer to Question 7

(D) The customer evidently did not have the mens rea for assault. An assault is either: (i) an attempt
to commit a battery; or (ii) the intentional creation (other than by mere words) of a reasonable
apprehension in the mind of the victim of imminent bodily harm. An attempt to commit a battery
would require a specific intent to unlawfully apply force to the person of another, resulting in
either bodily injury or an offensive touching. In this question, the customer was simply fleeing
from the factory at the time he darted past the visitor, who happened to be in his path. There
apparently was no intent on the part of the customer to bring about any sort of bodily injury or
offensive touching to the visitor. Thus, the customer did not possess the requisite mens rea (i.e.,
specific intent) for an attempt to commit a battery. Likewise, the facts indicate that the customer
did not intend to create in the mind of the visitor a fear of imminent bodily harm. Thus, the
customer also lacked the requisite mens rea for the second of the two types of assault. (A) states
a factor that is totally irrelevant to a charge of criminal assault. In determining possible criminal
liability for assault, there is no such thing as an “underlying offense,” as in felony murder, where
a killing occurring during the course of a felony (the underlying offense) is deemed to be murder.
Whether the defendant is guilty of assault on these facts does not depend on any other offense that
he committed or was in the process of committing. Thus, it is of no significance that the custom-
er’s conduct in the factory constituted a misdemeanor rather than a felony. (B) does not state a
defense to assault because, where there is an assault, there is no actual touching of the victim (a
touching would constitute a battery, not an assault). Consequently, the absence of contact between
the customer and the visitor is of no help to the customer. (C) is not helpful to the customer
because the foreseeability of the visitor’s injury as a consequence of the customer’s criminal
activity at the factory is irrelevant to a charge of assault. As detailed above, assault requires the
intent to either commit battery or to create fear of imminent bodily harm. “Foreseeability” speaks
to negligence rather than intent, and is not applicable to assault. Even if the visitor’s injury were
a foreseeable consequence of the customer’s criminal activity at the factory (and it may well have
been foreseeable that a passerby would be injured as the customer fled the scene), the customer
would not be guilty of assault based on mere foreseeability of injury, absent the presence of the
requisite intent on his part.

Answer to Question 8

(C) The defendant is guilty of larceny because, while having mere custody of the television set, he
carried it away from the motel intending to permanently deprive the motel owner of his interest
in the set. Larceny consists of the taking and carrying away of tangible personal property of
4. CRIMINAL LAW - OTHER CRIMES ANSWERS

another by trespass, with intent to permanently (or for an unreasonable time) deprive the person of
his interest in the property. Property must be taken from someone who has a possessory interest
superior to that of the defendant. If the defendant has custody of the property, rather than posses-
sion, his misappropriation of the property is larceny. Possession involves a much greater scope of
authority to deal with the property than does custody. Here, the defendant only had the authority
to use the television set for viewing purposes while he was staying at the motel. Thus, the defen-
dant had only enough authority to deal with the set as to indicate that he had custody of it rather
than possession. Consequently, the motel owner had a possessory interest in the set superior to
that of the defendant. The defendant took the set by trespass (without the consent of the owner)
and carried it away with the intent to permanently deprive the owner of his interest in the set.
Thus, the defendant is guilty of larceny. (D) is incorrect because larceny by trick occurs when the
victim consents to the defendant’s taking possession of the property, but such consent has been
induced by a misrepresentation. Here, the motel owner never consented to give the defendant
possession of the television set, through misrepresentation or otherwise. Instead, the defendant
simply took the set without the consent of the owner. Therefore, this is not larceny by trick. (B)
is incorrect for a similar reason. The offense of false pretenses consists of obtaining title to the
property of another by an intentional or knowing false statement of past or existing fact, with
intent to defraud the other. The defendant made no misrepresentations to the motel owner, nor
did the owner convey title to the television set to the defendant. Thus, the defendant is not guilty
of false pretenses. (A) is incorrect because embezzlement requires the fraudulent conversion of
property of another by a person in lawful possession of that property. The defendant never had
lawful possession of the television set. The taking of the set without the consent of the motel
owner was trespassory. Thus, the defendant has not committed embezzlement.

Answer to Question 9

(B) The lender can be charged with larceny, but not the other crimes. Larceny consists of a taking and
carrying away of the tangible personal property of another by trespass, with the intent to perma-
nently deprive the person of her interest in the property. Here, the lender took and carried away
the debtor’s personal property (her chair). The taking was trespassory because it was without
the debtor’s consent. At the time of the taking, the lender intended to deal with the chair in a
manner that involved a substantial risk of loss (i.e., burn it). Such intent is enough for larceny, as
the “intent to permanently deprive” includes a substantial risk of loss. Thus, the lender is guilty
of larceny. (Note that if the lender had simply taken the chair in the honest belief that she was
entitled to it as repayment of the debt, there would be no larceny. However, setting the chair on
fire indicates that the lender intended to permanently deprive the debtor of the chair rather than
take it as repayment.) (A) and (C) are wrong because common law arson has not occurred here. At
common law, arson is the malicious burning of the dwelling of another. Here, the burning was of
an item of personal property, not of the dwelling itself. (C) and (D) are wrong because the lender
did not commit common law burglary. Burglary consists of a breaking and entering of a dwelling
of another at nighttime with the intent to commit a felony therein. Here, the lender’s unauthorized
opening of the unlocked door is sufficient for a breaking, and the breaking and entering of the
debtor’s dwelling occurred at night. However, at the time the lender entered, she did not intend to
commit a felony; she simply wanted to confront the debtor and demand repayment of her money.
The lender did not decide to commit the larceny until after she was already in the house and
found that the debtor was not at home. Because she did not have the requisite intent when she
entered the house, she cannot be guilty of burglary.
CRIMINAL LAW - OTHER CRIMES ANSWERS 5.

Answer to Question 10

(B) The student is likely to be acquitted. Burglary requires a breaking and entering with the intent
to commit a felony therein. Because what the student intended to do when she broke in was not
a felony, it cannot be said that she entered with the intent to commit a felony. For this reason, she
should be acquitted of burglary. It follows that (B) is correct and (C) is incorrect. (D) is incorrect.
Although it is true that factual impossibility is not a defense to attempt, the student was charged
with burglary, not attempted burglary. The fact that the student was unable to retrieve the exams
does not affect the burglary charge because burglary requires the intent to commit a felony, not
the actual commission of the felony. If looking at the exam questions were a felony, the student
would have had the intent required for burglary regardless of her ability to actually look at the
exam questions. (A) is incorrect because the crime of burglary would have been complete at
the moment of breaking and entering with intent to commit a felony therein. However, because
looking at exam questions was not a felony, the student could not be convicted of either burglary
or attempted burglary.

Answer to Question 11

(D) The employee has committed theft by false pretenses. At common law, theft by false pretenses
occurs when a defendant (i) obtains title; (ii) to the property of another; (iii) by an intentional
(or knowing) false statement of past or existing fact; (iv) with the intent to defraud another. In
the instant case, the employee’s conduct meets all of the elements of the crime. The employee
obtained title to the property by falsely misrepresenting the number of hours that he worked,
with the intent that the company would pay him for the hours. Thus, he has committed the
crime of false pretenses. (A) is incorrect. At common law, forgery consisted of (i) a making or
altering; (ii) of a false instrument; (iii) with the intent to defraud. The falsity must be about the
instrument itself, not about the contents of the instrument. Here, the timecard is exactly what it
purports to be—a timecard containing the hours worked. Thus, the employee has not committed
forgery by submitting a false timecard. (B) is also incorrect. At common law, embezzlement was
(i) the fraudulent; (ii) conversion; (iii) of the property; (iv) of another; (v) by a person in lawful
possession of that property. In the instant case, it is the last element that is lacking. Although the
employee was required to maintain the timekeeping cards for his crew and to submit budgets,
he was never actually in lawful possession of any funds. Thus, he cannot properly be convicted
of embezzlement. Finally, (C) is also incorrect. At common law, larceny by trick occurred when
possession of property was conveyed rather than actual title. Here, the company intended to pay
the employee for the hours he purportedly worked; thus, it intended to pass title to the money the
employee received. As a result, the employee did not commit larceny by trick.

Answer to Question 12

(D) The wife is not guilty because she lacked the intent to defraud the government. The crime of false
pretenses consists of obtaining title to the property of another by an intentional false statement
of past or existing fact, with intent to defraud the other. This is a crime requiring specific intent,
i.e., intent to defraud. The wife believed that she was entitled to the total amount of the combined
check, because she thought that the amount of the check equaled what she would have received
as a death benefit and an increased spousal benefit. Because the wife believed that she was simply
receiving money to which she was entitled, rather than money actually belonging to the govern-
ment, she lacked the intent to defraud the government. Absent the requisite specific intent, the
wife is not guilty of obtaining money by false pretenses. (B) is incorrect because it would hold
the wife to a “reasonable person” standard; i.e., imposing liability if she should have known that
6. CRIMINAL LAW - OTHER CRIMES ANSWERS

she was receiving benefits to which she was not entitled. Even if the wife should have known this,
the fact remains that she sincerely believed that she was only receiving money to which she was
entitled. This sincere belief, even if unreasonable, will negate the existence of intent to defraud the
government. (C) is virtually identical to (B), stating in effect that the wife is guilty if her belief
as to her entitlement to the money is unreasonable. As explained above, the reasonableness of the
wife’s belief is not significant. As long as the wife truly believed that she was entitled to all of the
money, she did not intend to defraud the government. (A) is incorrect because the question asks
about the offense of false pretenses. As noted above, this offense clearly requires specific intent,
and is therefore not an offense to which strict liability would be properly applicable.

Answer to Question 13

(C) The teenager is guilty of larceny only. Larceny is the taking and carrying away of the tangible
personal property of another by trespass, with the intent to permanently deprive the person of
his interest in the property. Under the continuing trespass doctrine, if a defendant takes property
with a wrongful state of mind but without the intent to steal, and later, while still in possession
of it, forms the intent to steal it, the trespass involved in the initial wrongful taking is regarded as
“continuing” and the defendant is guilty of larceny. Here, the teenager’s initial wrongful taking
continued to the time the teenager intentionally destroyed the car, thereby permanently depriving
the owner of possession. Therefore, the teenager committed larceny and (A) is incorrect. Burglary
requires the breaking and entering of the dwelling of another at nighttime with the intent to
commit a felony therein. The teenager is not guilty of burglary because, when he entered the
attached garage, he did not intend to commit a felony therein; he merely intended to borrow the
car. Thus, (B) and (D) are incorrect.

Answer to Question 14

(D) The fan should not be convicted under either definition of assault. Criminal assault is either: (i)
an attempt to commit a battery, or (ii) the intentional creation, other than by mere words, of a
reasonable apprehension in the mind of the victim of imminent bodily harm. In the instant case,
the fan did not intend to hit anyone. Thus, the fan’s actions do not constitute an attempt to commit
a battery, which would require a specific intent to bring about bodily injury or an offensive
touching. In addition, the fact that the referee did not see the bottle being thrown at him means
that the defendant did not create in the referee a reasonable apprehension of imminent bodily
harm. Consequently, the fan is not likely to be convicted of either type of assault. This also makes
(B) and (C) incorrect, in that each answer considers only one type of assault. (A) is incorrect.
Although a substantial step, or some overt act showing the defendant’s intent to commit the crime,
is required to convict for an attempt of a crime, the facts here explicitly state that the fan had no
such intent. This makes (A) an incorrect answer choice.

Answer to Question 15

(D) Given that the defendant believed that, at the time he took the computer, the victim would have
permitted him to take it to return it to the other player, the defendant lacked the intent to perma-
nently deprive the victim of her interest in the computer, which is the necessary intent for larceny.
Common law larceny consists of a taking and carrying away of tangible personal property of
another by trespass with intent to permanently (or for an unreasonable time) deprive the person
of his interest in the property. Larceny is a crime against possession; thus, it is only necessary
that the property be taken from someone who has a possessory interest superior to that of the
defendant. At the moment of the taking, the defendant must have had the intent to permanently
CRIMINAL LAW - OTHER CRIMES ANSWERS 7.

deprive the person from whom the property is taken of his interest in the property. Here, although
the other player owned the computer, the victim had the right of possession of the computer
until the other player returned the victim’s backgammon set. Thus, when the defendant took the
computer from the victim’s car, he was taking and carrying away tangible personal property in
which the victim had a possessory interest. However, the defendant believed that, at the time
of the taking, he was merely returning the computer to the other player, and that the victim
would want him to do so. Thus, the defendant did not have the intent to deprive the victim of her
interest in the computer because he was unaware that the victim had any such interest. Because
the intent element of larceny is lacking, the defendant cannot be convicted of larceny. Alterna-
tive (A) is incorrect because, although it correctly states that the victim was rightfully in posses-
sion of the computer when the defendant took it, the defendant lacked the intent necessary for
larceny (as explained above). (B) is incorrect because the intent to deprive another of her interest
in the computer, demonstrated by the defendant’s sale of the computer to the stranger, did not
exist at the time that the defendant took the computer. Generally, if a defendant trespassorily
takes property without the intent to steal but later, while still in possession of it, forms the intent
to steal, the initial wrongful taking is regarded as “continuing” and the defendant is guilty of
larceny. However, the “continuing trespass” doctrine has no application if the defendant’s initial
taking of the property was not motivated by a “wrongful” state of mind. In this fact pattern,
when the defendant initially took the backgammon computer from the victim’s car, he lacked a
wrongful state of mind, as he was unaware of the arrangement between the victim and the player
and thus believed that he was retrieving the computer for its lawful owner. Therefore, because the
defendant’s initial taking was innocent, not wrongful, the “continuing trespass” doctrine cannot
be used to prosecute him for larceny, and the defendant’s intent at the airport does not change the
earlier innocent taking into larceny (although the defendant might be guilty of embezzlement for
intentionally converting the computer while in lawful possession of it). (C) is incorrect because,
despite the other player’s ownership of the computer, the victim had a superior right to possession
at the time that the defendant took the computer (i.e., she had the right to possess the computer,
even as against the other player—the owner—until the other player returned the backgammon
set). Thus, had the defendant known at the time of the taking that the victim’s right to possess the
computer was superior to that of the other player, the defendant would have been guilty of larceny
because he would have had the requisite intent.

Answer to Question 16

(C) The friend can be convicted of assault. There are two actions covered by the crime of assault: (i)
an attempted battery, and (ii) the intentional creation of a reasonable apprehension in the mind of
the victim of imminent bodily harm. The friend’s conduct fits within the second type of assault.
He intended to create an apprehension of imminent bodily harm in the worker’s mind, because
he used a realistic-looking toy gun and a mask and pretended to rob the worker. Thus, despite the
fact that it was only to teach the worker a lesson, the friend committed an assault. (A) and (B)
are wrong because the friend did not have the requisite intent for robbery. Robbery is a taking
of another’s personal property from the other’s person by force or intimidation with the intent
to permanently deprive him of his property. Here, the friend never intended to keep the money;
thus, he did not have the intent necessary for robbery. This also makes (D) wrong, as larceny also
requires the intent to permanently deprive the owner of his property.

Answer to Question 17

(D) The woman is guilty of arson and burglary. She is guilty of arson because she deliberately set a
fire that, in addition to burning the mattress, also burned part of the dwelling house of another,
8. CRIMINAL LAW - OTHER CRIMES ANSWERS

namely the floor. She is also guilty of burglary because she broke and entered into the dwelling
house of another during the nighttime to commit a felony. The fact that she was not successful in
committing the crime she intended or that she in fact committed another felony is irrelevant to
her guilt for burglary; it is the intent to commit a felony at the time of the breaking and entering
which is critical. (A) is incorrect. This choice correctly states that the woman is guilty of burglary,
but she is also guilty of arson; she deliberately set a fire that, in addition to burning the mattress,
also burned part of the dwelling house of another, namely the floor. Therefore, (B) and (C) are
incorrect.

Answer to Question 18

(D) The passenger should be acquitted. Robbery is larceny from a person by violence or intimidation.
Larceny requires a specific intent to take personal property from the possession of another with
intent to permanently deprive the other of his possessory interest. Because the passenger thought
that the property belonged to him, his mistake of fact is a valid defense to the crimes of larceny
and robbery. This is because the mistake negates the existence of a mental state (specific intent
to steal) essential to the charged crime. (C) is incorrect. Robbery is larceny by either violence
or intimidation. Knocking the owner to the floor constitutes the necessary violence, whether or
not the passenger used intimidation. Even though voluntary intoxication may be a defense to a
specific intent crime (in those circumstances where the intoxication negates the specific intent),
the better answer is (D), because the passenger’s mistake of fact is a defense to the specific intent
crime charged since he did not have the specific intent to steal. (A) is incorrect. As discussed
above, although voluntary intoxication can be a defense to a specific intent crime, (D) is the better
answer because the question does not establish that the mistake of fact was a product of the intoxi-
cation. (B) is incorrect. The passenger’s mistake of fact negates the specific intent to steal required
to convict him of robbery.
CRIMINAL LAW -
RIGHT TO COUNSEL
AND CONFESSIONS
QUESTIONS
CRIMINAL LAW - RIGHT TO COUNSEL AND CONFESSIONS QUESTIONS 1.

CRIMINAL LAW - RIGHT TO COUNSEL AND CONFESSIONS QUESTIONS

Question 1 Question 2

A defendant was arrested on suspicion of The defendant was arrested, given Miranda
running an illegal “moonshine” operation. warnings, and charged with burglary. At the
After taking the defendant back to the police police station, he telephoned his mother and
station, an officer began questioning the suspect, asked her to come to the station to post bail.
thinking that his partner had already given the Instead, his mother immediately called the
defendant a Miranda warning. The defendant family attorney. In the meantime, the police had
voluntarily confessed to each and every element begun questioning the defendant. Although he
of the crime. never told the police to stop the questioning, his
answers were at first vague or clearly unrespon-
At trial, the defendant took the witness stand sive. During the course of the questioning, the
and testified on his own behalf, declaring that family attorney phoned the station and told the
he was innocent and that a distillery that the police that she had been hired to represent the
officers found at his home belonged to someone defendant and would be there in half an hour.
else. The prosecution, on cross-examination, The police did not inform the defendant of the
produced the confession that the defendant gave attorney’s call. Ten minutes later, the defendant
concerning his illegal activities. The defense admitted to committing the burglary, and signed
counsel objected to the admission of the confes- a statement to that effect prepared by the police.
sion. The attorney arrived a few minutes later and
advised the defendant to remain silent, but he
How should the court rule on the defendant’s told her that he had already signed a confession.
objection?
How should the court rule on the attorney’s
(A) Sustained, because all evidence obtained in pretrial motion to exclude the confession as
violation of Miranda rights is inadmissible. evidence at trial?

(B) Sustained, because the prosecution did not (A) Grant the motion, because the police had a
get permission from the court in advance to duty to inform the defendant that an attor-
use the confession for any purpose. ney was coming to represent him.

(C) Overruled, because the prosecution may (B) Grant the motion, because the defendant
question the defendant on cross-examina- has been deprived of his Sixth Amendment
tion concerning any issue that was brought right to counsel.
out in his defense.
(C) Deny the motion, because the defendant’s
(D) Overruled, but the confession should be statement admitting the crime was volun-
admitted only for the limited purpose of tary.
impeachment.
(D) Deny the motion, because the defendant
waived his Miranda rights.
2. CRIMINAL LAW - RIGHT TO COUNSEL AND CONFESSIONS QUESTIONS

Question 3 Question 4

While investigating the most recent of a A state statute requires that a person who
series of murders, a homicide detective was is suspected of committing a crime must be
approached by an onlooker who seemed to have informed of the nature of that crime before
detailed knowledge of the murders. The detec- questioning may begin. The state supreme court
tive recalled the onlooker at some of the other has held that statements obtained in violation of
murder scenes, and immediately suspected that a suspect’s statutory rights may not be admitted
he knew something about the crimes. The detec- into evidence. The defendant, who was arrested
tive asked the onlooker not to leave until the on suspicion of committing an arson, was told
detective had the opportunity to ask him a few “You have the right to remain silent; anything
questions. After finishing with the evidence he you say can and will be used against you in a
was gathering, the detective started to question court of law; you have the right to the presence
the onlooker at the crime scene without giving of an attorney during questioning; and if you
him Miranda warnings. The onlooker eventually cannot afford an attorney, one will be appointed
revealed details of the crimes that were never for you.” In response to questioning, the defen-
made available to the public. As a result, the dant gave a statement implicating himself in the
onlooker was arrested and charged with several arson. He was charged and brought to trial in
murders. At a pretrial hearing, the onlooker testi- state court for arson.
fied that he believed that he could not leave until
he had spoken with the detective. The defense At trial, should the statement be excluded
counsel moves to suppress the statements made from evidence?
to the homicide detective.
(A) Yes, because the Miranda warnings were
What is the most likely result? not proper.

(A) The motion will not be granted because the (B) Yes, because the questioning violated state
onlooker was not in custody. law.

(B) The motion will not be granted because (C) No, because proper Miranda warnings were
the onlooker initiated the contact with the given in compliance with federal constitu-
homicide detective. tional requirements.

(C) The motion will be granted because the (D) No, because the requirement of informing
onlooker believed that he was not free to the suspect of the nature of the charges
leave. against him is not a state constitutional
requirement.
(D) The motion will be granted because the
detective was required to give the onlooker
Miranda warnings once the detective
suspected him of having committed the
crime.
CRIMINAL LAW - RIGHT TO COUNSEL AND CONFESSIONS QUESTIONS 3.

Question 5 Question 6

A suspect was arrested for a misdemeanor A police officer went to the defendant’s house
battery and was not provided Miranda warnings and placed him under arrest for operating an
at that time. He was placed in a “holding pen” auto theft ring. As the defendant was being
while waiting to post bond. Before the suspect arrested, he told his wife, “You had better
could do so, a detective in charge of investi- call our lawyer; I don’t want to sign anything
gating a string of burglaries noticed that the unless she’s with me.” The defendant was given
suspect matched the description of the serial Miranda warnings on the way to the police
burglar. The detective brought the suspect from station. Meanwhile, the defendant’s lawyer
the holding pen into a room specifically reserved called the station and told the desk sergeant that
for interrogation. The detective had the suspect she was on her way and to have the defendant
take a seat next to the door, which the detec- call her as soon as he arrived. The sergeant
tive left open. The detective gave him Miranda assured her that the defendant would be held
warnings and then began questioning him about without questioning for several hours until the
the burglaries. Caught by surprise that he was district attorney arrived. When the defendant
being questioned for the burglaries rather than arrived at the station, the arresting officer and
the battery, the suspect signed the rights waiver another officer immediately put the defendant in
and provided a statement confessing to the an interrogation room and questioned him about
burglaries. a bank robbery that had taken place two days
ago. They did not inform him of the call from
On proper motion, should the suspect’s state- his lawyer, but he agreed to talk as long as he
ment be suppressed? did not have to put anything in writing or sign
anything without her okay. He made incrimi-
(A) No, because the suspect was not arrested nating statements about the robbery, and he was
for burglary and thus was free to leave the eventually indicted for that crime as well. Prior
interview. to trial on the robbery charge, the defendant’s
lawyer moved to suppress the arresting officer’s
(B) No, because proper Miranda warnings were testimony about the defendant’s statements.
given.
The court should:
(C) Yes, because the detective failed to inform
the suspect that the questioning would be (A) Deny the motion, because the questioning
about the burglaries. was about a different crime from the one
for which the defendant was in custody.
(D) Yes, because he should have been provided
Miranda warnings when he was arrested. (B) Deny the motion, because the defendant’s
statements were made voluntarily after
receiving Miranda warnings.

(C) Grant the motion, because the defendant


was not informed that his lawyer was trying
to see him, and his lawyer was misinformed
that he would not be questioned right away.

(D) Grant the motion, because the defendant’s


refusal to write or sign anything indicates
that he did not knowingly and intelligently
waive his right to the assistance of counsel.
4. CRIMINAL LAW - RIGHT TO COUNSEL AND CONFESSIONS QUESTIONS

Question 7 Question 8

As a woman was walking home on a dark Acting on an anonymous telephone call, the
cloudy night, a man came upon her from behind police went to a woman’s apartment, pounded on
and stole her purse. A suspect was arrested the door, and demanded to search it for possible
nearby shortly thereafter. After the suspect had stolen property. The woman refused. The police
been booked, the police took his photograph. then kicked open the door and placed the woman
They then showed his photograph, along with under arrest. The woman then offered to give the
the photographs of four people who had the officers some inside information in exchange for
same general features as the arrested man, to the her release. Before she could say anything else,
woman. The woman identified the suspect as the the woman was given Miranda warnings by the
culprit. At trial, after the woman had identified police. Thereafter, she told the police that she
the suspect as the person who stole her purse, knew of a large supply of stolen property stored
the suspect’s attorney objects to the prosecution’s at a nearby warehouse and said that she and a
introduction into evidence of the photographic friend had been selling the stolen property out
identification. of the warehouse for years. The police raided
the warehouse and recovered the stolen property.
What is the most likely result? The woman was charged with conspiracy to
sell stolen property and for possession of stolen
(A) The objection will be overruled. property. At her trial, the woman moved to
suppress the statements.
(B) The objection will be sustained, because
the suspect’s right to counsel was violated Which of the following is the woman’s best
at the showing of the photograph to the argument in support of the motion to suppress?
woman.
(A) The woman was intimidated by the forced
(C) The objection will be sustained, because entry into her home and, because her state-
the woman did not have a good opportunity ments were involuntary and coerced, their
to observe the culprit. use against her would violate due process
of law.
(D) The objection will be sustained, because a
photographic identification must be consid- (B) The woman is entitled to know the identity
ered a critical stage of the proceeding. of her accuser, and the state cannot supply
this information.

(C) The woman’s statements were fruits of an


unlawful arrest, and although the Miranda
warnings may have been sufficient to
protect her right against self-incrimination,
they were not sufficient to purge the taint of
the illegal arrest.

(D) The police should have given the woman


Miranda warnings prior to entry into her
home, and the warnings were ineffectual
once the woman offered to give informa-
tion.

一 信:liuxue119118 , 们 信免 供
CRIMINAL LAW - RIGHT TO COUNSEL AND CONFESSIONS QUESTIONS 5.

Question 9 Question 10

The defendant broke into a woman’s house A suspect was captured in a high-speed police
one night. As he was collecting valuables, he chase five minutes after a bank robbery in which
was surprised by the woman. He struck her on the robbers had handed the teller a handwritten
the head with a candlestick and tied her up, then note demanding money. The suspect was taken
finished filling his sack and left. The police to the police station where, over his protests,
discovered the woman several hours later and he was required to write out the words of the
rushed her to the hospital. The defendant was note. He was then charged with the robbery.
apprehended by the police early the following At a pretrial suppression hearing, the suspect
morning with the loot still in his possession. He challenged the prosecution’s plan to offer in
was taken to police headquarters, given Miranda evidence the writing that he had been required
warnings, and asked if he wished to make a to make by the police so that the jury could
statement about the prior evening’s events. The compare it with the robbers’ note.
police did not mention that the woman had been
seriously injured and was in the hospital. The How should the court rule on the admissibility
defendant said he understood his rights and of the writing?
was willing to talk. He then admitted that he
committed the burglary of the woman’s house. (A) Admissible.
The following day, the woman died from injuries
caused by the blow to her head. The defendant (B) Inadmissible, because the suspect was not
was charged with murder. advised that his handwriting sample could
be admitted into evidence against him.
If the defendant moves to prevent introduc-
tion of the confession into evidence, should his (C) Inadmissible, because the suspect was
motion be granted? not advised of his right to refuse to give a
handwriting sample.
(A) No, because failure of the police to advise
the defendant of the woman’s condition (D) Inadmissible, because the suspect had
was harmless error since felony murder not been informed he had a right to have
does not require intent to kill or injure. counsel present.

(B) No, because the defendant’s waiver of his


rights did not depend on the nature of the
charges that were later filed against him.

(C) Yes, because the defendant could not make


a knowing and intelligent waiver unless
he was informed concerning the woman’s
condition.

(D) Yes, because the use of a confession to


burglary in a prosecution for murder
violates due process where the police
withheld information about the potential
seriousness of the offense.
6. CRIMINAL LAW - RIGHT TO COUNSEL AND CONFESSIONS QUESTIONS

Question 11 Question 12

A defendant was arrested for carrying a After being arrested on suspicion of murder,
concealed weapon in violation of his parole. a suspect was taken to the police station and
The police had reason to believe that the informed of his constitutional rights as required
defendant was part of a group that sold stolen by the Miranda decision. He immediately
automobile parts. The police interrogated the requested that a lawyer be provided because he
defendant continually for 10 hours. Finally, the had no money to hire one. The arresting officer
police threatened to have the defendant’s parole said that he would get the suspect’s lawyer
revoked if he did not cooperate. Fatigued from after he was booked, and the officer proceeded
the lengthy interrogation and afraid of going to book him. During the booking search, the
back to jail, the defendant confessed to being suspect said to the arresting officer, “I only
a member of the group involved in the theft of killed the bastard because he made a pass at
automobile parts. The defendant gave the police me.”
the location of a friend’s garage where the stolen
parts were located. Based on this confession, the If the suspect attempts to prevent introduc-
police obtained a search warrant to search the tion of the statement made by him to the officer
garage, where they discovered a large quantity during booking, will he most likely succeed?
of stolen automobile parts. The defendant was
prosecuted for conspiracy to sell stolen automo- (A) No, because booking is not a critical stage
bile parts. Before trial, he moved to prevent use of criminal proceedings requiring the as-
of the stolen parts as evidence. sistance of counsel.

The court will most likely rule that the (B) No, because the statement was not the result
evidence should be: of a custodial interrogation.

(A) Admitted, because it was obtained pursuant (C) Yes, because the statement was the product
to a warrant. of illegal police conduct.

(B) Admitted, because the defendant had no (D) Yes, because the request for an attorney
standing to object to the search of the should have been honored immediately.
garage.

(C) Suppressed, because the uncorroborated


statements of a criminal suspect are not a
sufficient basis for a warrant.

(D) Suppressed, because it is the fruit of the


defendant’s involuntary statements.
CRIMINAL LAW - RIGHT TO COUNSEL AND CONFESSIONS QUESTIONS 7.

Question 13 Question 14

The defendant was pulled over for speeding. Late one night, a young couple were killed
After a license plate check revealed that the car instantly when their car was struck by a speeding
was recently reported as stolen, the defendant truck as the couple’s car crossed an intersection
was arrested for car theft. At the police station, with the light green in their favor. Several weeks
the defendant received Miranda warnings and later, a burglar awaiting trial on burglary charges
agreed to answer questions regarding a local asked a jail officer to let him speak with a
auto “chop-shop” network. During a break for highway patrol officer. When the highway patrol
lunch, the police interrogators noticed that the officer came to the cell, the burglar told him that
defendant, who was otherwise rather shabbily he was the driver of the truck that had struck the
dressed, was wearing an obviously expensive car and had been speeding away from a burglary
watch. On their return to the interrogation room, when the accident occurred.
without giving any new Miranda warnings,
the officers began interrogating the defendant The burglar was charged with felony murder,
about a recent home invasion where a quantity on the theory that he had not yet reached a place
of valuable jewelry was taken, including a watch of temporary safety when the accident occurred.
similar to the one the defendant was wearing. At trial, the prosecution seeks to introduce the
The defendant confessed to the home invasion, burglar’s statements to the highway patrol officer
and was indicted on that charge, a felony. regarding the events of the night of the accident.
The burglar’s attorney objects.
If the defendant files an appropriate motion
to prevent the confession from being admitted at Which of the following is the strongest
his trial, is the defendant likely to succeed? argument for permitting the statements into
evidence?
(A) Yes, because the police did not repeat the
Miranda warnings before questioning the (A) The burglar had not been charged in con-
defendant about another crime. nection with the auto accident at the time
the statements were made to the highway
(B) Yes, because the police acted in a custodial patrol officer.
setting.
(B) The burglar made the statements spontane-
(C) No, because the defendant was not in ously, without inducement or interrogation
custody on a home invasion charge. by the police.

(D) No, because the original Miranda warnings (C) The highway patrol officer had no connec-
given the defendant were sufficient under tion with the burglary investigation for
the circumstances. which the burglar had been incarcerated.

(D) The burglar’s statements were not the


product of coercion by the police officers.
CRIMINAL LAW - RIGHT
TO COUNSEL AND
CONFESSIONS ANSWERS
CRIMINAL LAW - RIGHT TO COUNSEL AND CONFESSIONS ANSWERS 1.

CRIMINAL LAW - RIGHT TO COUNSEL AND CONFESSIONS ANSWERS

Answer to Question 1

(D) The defendant’s objection should be overruled. A confession obtained in violation of Miranda, but
otherwise voluntary, can be used for the limited purpose of impeaching a defendant who testifies
at trial. In contrast, an involuntary confession cannot be used to impeach. Here, there are no facts
to indicate that the defendant’s statement was involuntary. Thus, it can be used to impeach the
defendant. (A) is too broad a statement. Although a confession obtained in violation of Miranda
is inadmissible in the state’s case-in-chief as evidence of guilt, as discussed above, such evidence
is admissible for limited purposes. (B) is wrong. Advance permission from the court is not a
requirement if the confession is used to impeach. (C) is a correct statement but it does not speak
directly to the issue of whether the confession is admissible and to what extent.

Answer to Question 2

(D) The defendant’s confession should be admitted because he waived his Fifth Amendment privi-
lege against compelled self-incrimination after receiving Miranda warnings. Miranda v. Arizona
requires that a person in custody be informed of his right to remain silent and his right to the
presence of an attorney during questioning. A suspect may subsequently waive his rights by
making a confession, as long as the waiver was knowing and voluntary. In this case, the defendant
received proper Miranda warnings, and there is no indication that he did not understand what his
rights were. Although his answers during questioning were initially unresponsive, he never asked
for an attorney or indicated that he wished to remain silent, and he voluntarily confessed after a
relatively short period of interrogation. Hence, he validly waived his Miranda rights. (A) is incor-
rect because the police have no duty to inform the defendant that an attorney is attempting to see
him. The defendant’s ignorance of his attorney’s efforts has no bearing on whether he made a
knowing waiver of his Miranda rights. (B) is incorrect because the defendant’s right to counsel
was not violated. Although the defendant does have a separate Sixth Amendment right to counsel
under Escobedo v. Illinois because he has already been arrested and charged with the crime, this
right would only be violated if the defendant, after being informed of his right to counsel, had
requested an attorney or had been prevented from seeing his attorney. Here, he made no request
to see an attorney—even when he called his mother—and his attorney was allowed to see him
immediately upon her arrival. Thus, he has waived his Sixth Amendment right to counsel. (C)
is incorrect even though it is true that the defendant made a voluntary statement. Due process
requires that for confessions to be admissible, they must be “voluntary,” based on the totality of
the circumstances, and here all of the circumstances indicate that the defendant’s confession was
voluntary. However, even a voluntary confession will be inadmissible if it was obtained in viola-
tion of Miranda rights. (D) is therefore a better choice than (C).

Answer to Question 3

(A) The statements that the onlooker gave to the homicide detective most likely will not be
suppressed due to a failure to provide a Miranda warning. Prior to custodial interrogation, the
person being questioned must be informed that he has the right to remain silent, that anything
he says can be used against him in court, that he has the right to the presence of an attorney, and
that, if he cannot afford an attorney, one will be appointed for him if he so desires. The interroga-
tion must take place in a custodial setting. The test to determine whether a person is in custody
is an objective test; the subjective beliefs of the interrogator or the accused are not determina-
tive. Essentially, if a reasonable person under the circumstances would feel that he was free
2. CRIMINAL LAW - RIGHT TO COUNSEL AND CONFESSIONS ANSWERS

to terminate the interrogation and leave, he is not in custody. Here, the onlooker had not been
arrested, had not been placed in handcuffs, nor was he even at a police station. Being uncon-
strained at a crime scene probably would not constitute being "in custody" to a reasonable person;
as a result, Miranda warnings were not required and (A) is correct. (B) is incorrect because who
initiates the contact is not really relevant to determine whether the person is "in custody," although
it may be a factor to be considered when determining whether the defendant believed he was free
to leave. (C) is incorrect because it falsely states that the test for custody is a subjective one. The
test is an objective test. (D) is also incorrect. Miranda warnings are required prior to custodial
interrogation. The subjective beliefs of the interrogator as to who may have committed the crime
are irrelevant.

Answer to Question 4

(B) The statement should be excluded from evidence. In Miranda v. Arizona, the Fifth Amendment
privilege against compelled self-incrimination became the basis for ruling on the admissibility
of a confession. Miranda warnings and a valid waiver are minimum prerequisites to the admis-
sibility of any statement made by the accused in a custodial interrogation. Nonetheless, states
are free to provide greater protection to a defendant. Here, the state statute provides for greater
rights, in that it requires the interrogator to inform the defendant about the nature of the crimes he
wishes to discuss with the accused, and the state supreme court has held that statements obtained
in violation of a defendant’s statutory interrogation rights must be excluded. Thus, under state law,
the statement should be excluded, making (B) the correct answer and (C) incorrect. (D) is incor-
rect; a state statute is sufficient to provide the right to an accused under the state supreme court
ruling. (A) is substantively incorrect. The rights warning in this question would have complied
with Miranda.

Answer to Question 5

(B) The statement should not be suppressed. Under the Fifth Amendment, a suspect must be provided
with Miranda warnings prior to interrogation. The Miranda warnings and a valid waiver are
prerequisites to the admissibility of any statement made by the suspect during custodial interroga-
tion. Here, the detective provided the suspect with valid Miranda warnings; thus, the statement
should not be suppressed due to a failure to provide Miranda warnings. (A) is incorrect because
the facts state that the suspect had been arrested and was waiting to post bond. Thus, even though
it was for a different charge, he was not free to leave. (C) is also incorrect. Under Miranda, there
is no requirement for the police to advise the suspect of the offense they wish to question him
about. (D) is incorrect because Miranda warnings must be given prior to interrogation, not after
arrest.

Answer to Question 6

(B) The defendant’s motion should be denied because his interrogation did not violate his Fifth
Amendment right to counsel. At any time prior to or during interrogation, a suspect may invoke
a Miranda (Fifth Amendment) right to counsel. However, the request must be unambiguous and
specific. If the defendant agrees to answer questions orally, but requests the presence of counsel
before making any written statements, the defendant’s oral statements are admissible. The defen-
dant’s agreement to talk constitutes a voluntary and knowing waiver of the right to counsel,
even if it could be argued that it indicates a misunderstanding of the evidentiary effect of oral
statements. [Connecticut v. Barrett (1987)] Thus, (B) is correct and (D) is incorrect. (A) is incor-
rect because it is irrelevant to the defendant’s Fifth Amendment right to counsel that the officers
CRIMINAL LAW - RIGHT TO COUNSEL AND CONFESSIONS ANSWERS 3.

questioned him about a different crime. If the accused invokes his right to counsel under Miranda,
all questioning must cease, even about a totally unrelated crime, because the Fifth Amendment
right to counsel under Miranda, unlike the Sixth Amendment right to counsel, is not offense
specific. Here, the statements are admissible because the defendant did not effectively invoke
his right to counsel. (C) is incorrect because as long as Miranda warnings have been given and
adversary judicial proceedings have not commenced, voluntary statements are admissible even
if the police lie to the defendant’s lawyer about their intent to question him and fail to inform the
defendant that his lawyer is attempting to see him. [Moran v. Burbane (1986)]

Answer to Question 7

(A) The objection will likely be overruled. Under the Federal Rules of Evidence, after a witness has
testified, statements of prior identification are admissible to prove their truth unless the defendant
can show that the circumstances of the identification were unnecessarily suggestive and likely
to result in irreparable misidentification. There is no evidence of that in the facts. (B) is wrong
because there is no right to counsel at a photo display. (C) is wrong because it goes to the weight
to be given the evidence rather than its admissibility. (D) is wrong because the photograph identi-
fication would not be considered a critical stage.

Answer to Question 8

(C) The woman’s best argument is that her statements were fruits of an unlawful arrest and that the
Miranda warnings were not sufficient to purge the taint of the illegal arrest. The statements were
the fruits of an unlawful arrest because the police, without an arrest or search warrant, entered the
woman’s house without her consent and arrested her. Although the Miranda warnings may have
been sufficient to protect the woman’s right against self-incrimination, they were not sufficient
to purge the taint of the illegal arrest. Thus, the entry into the woman’s apartment and her arrest
without a warrant, probable cause, or circumstances permitting an exception from these require-
ments were illegal. The statements she made thereafter were fruits of the original illegality and
are to be suppressed unless the taint is purged. The giving of Miranda warnings is not sufficient.
(A) is wrong because the facts indicate that the statements were voluntarily made. (B) is a correct
statement of the law, but it is irrelevant under these facts. (D) is wrong because Miranda warnings
are only required prior to custodial interrogation.
Answer to Question 9

(B) The defendant’s motion should be denied because his waiver of his Miranda rights was valid.
Even though the prosecution must show, by a preponderance of the evidence, that a defen-
dant’s waiver of his Miranda rights was knowing and voluntary, the suspect need not have been
informed of all subjects of an interrogation to effect a valid waiver. The police were not required
to tell the defendant of the woman’s condition. (A) is incorrect because no error was involved.
(C) is incorrect for the reasons described above, i.e., the police need not inform the subject of
all aspects of the interrogation for the waiver to be considered valid. (D) is incorrect because
due process requires only that confessions be voluntary, i.e., not the product of official compul-
sion. Withholding information about the potential seriousness of the offense does not violate due
process.
Answer to Question 10

(A) The court should rule the writing to be admissible. The handwriting sample is relevant to the
issue of the identity of the bank robber and is admissible because it was properly obtained and
一 信:liuxue119118 , 们 信免 供

4. CRIMINAL LAW - RIGHT TO COUNSEL AND CONFESSIONS ANSWERS

violated no rule of privilege. A handwriting sample is not testimonial in nature and, therefore,
does not require Fifth or Sixth Amendment protections. (B) is incorrect. The handwriting sample
is evidence of physical characteristics and not testimonial in nature and, therefore, not subject
to Fifth Amendment protections. Therefore, there is no requirement that Miranda warnings
be given, advising the defendant that the sample could be used against him. (C) is incorrect.
Because the privilege against self-incrimination does not apply, there is no right to refuse to give
the handwriting sample, provided that the content of the writing is not used against the defen-
dant. (D) is incorrect. The presence of counsel is not required at a scientific identification made
by the police for the purposes of investigation, such as taking a handwriting sample. [Gilbert v.
California (1967)]

Answer to Question 11

(D) The court will most likely rule that the evidence should be suppressed. A statement is only consid-
ered to be voluntary if it is the product of a free and rational choice. If from the surrounding
circumstances it appears that the statement was produced by either physical or mental coercion,
it will not be considered to be voluntary. While it is a judgment for the court as to whether the
confession would be voluntary or involuntary, the lengthy interrogation would probably result in a
finding that the confession was involuntary. If the confession is determined to be involuntary, the
evidence obtained as a direct result of the confession will probably be suppressed as the fruit of
the poisonous tree. (A) is incorrect because evidence obtained pursuant to a warrant is inadmis-
sible if the warrant was improperly issued. (B) is incorrect because the defendant is not basing
his motion to suppress on the search of the garage but on the illegally obtained statement. (C) is
incorrect because a warrant may be issued on the basis of such uncorroborated statements.

Answer to Question 12

(B) The suspect will not be successful in preventing the introduction of his statement because it was
not the result of a custodial interrogation. If, after being given his Miranda warnings, the suspect
invokes his right to counsel, all interrogation must stop until counsel is present. “Interrogation”
includes any words or actions by the police that the police should know are likely to produce an
incriminating response. The suspect may volunteer information to the police at any time. (A) is
not the best answer. While it is true that booking is not a critical stage for purposes of the right to
counsel, that is not the reason the statement will be allowed. The statement would be excluded if it
had been the result of interrogation during the booking. (C) is incorrect; there is no illegal police
conduct set forth in the facts. (D) is an incorrect statement of the law.

Answer to Question 13

(D) The defendant’s waiver of his Fifth Amendment privilege against self-incrimination after
receiving Miranda warnings applies to the subsequent questioning about another crime. Miranda
v. Arizona (1966) requires that a person in custody be informed of his right to remain silent and
his right to the presence of an attorney during questioning. A suspect may then waive his Miranda
rights by answering an interrogator’s questions as long as the waiver was knowing and voluntary.
The suspect need not be informed of all potential subjects of an interrogation to effect a valid
waiver. If the suspect has waived his rights, there is generally no need to repeat the warnings
because of a break in the interrogation unless the time lapse has been so long that a failure to do
so would seem like an attempt to take advantage of the suspect’s ignorance of his rights. Here,
the defendant was given his Miranda warnings and apparently made a knowing, voluntary,
and intelligent waiver of his rights by agreeing to answer the officers’ questions. The break for
CRIMINAL LAW - RIGHT TO COUNSEL AND CONFESSIONS ANSWERS 5.

lunch does not invalidate the waiver, nor does the fact that the subsequent questioning involved
a different crime. Hence, the defendant probably will fail to prevent the confession from being
admitted at his trial. (A) is incorrect. If the defendant had asserted his Miranda rights during the
initial questioning, any subsequent interrogation about a different crime would be invalid without
a significant time lapse and a fresh set of warnings. However, because the defendant waived his
Miranda rights during the initial interrogation, subsequent interrogation about a different crime
does not require repetition of the warnings; the waiver applies to all potential subjects of an inter-
rogation. (B) is incorrect because the custodial setting requires only that the police give the defen-
dant the Miranda warnings at the outset, which they did. The valid waiver that followed permits
admissibility of the confession. (C) is incorrect because it is irrelevant what charge the defendant
was in custody for. Unlike the offense-specific Sixth Amendment right to counsel, a detainee’s
rights under Miranda are not offense specific. The defendant was in custody because he was not
free to leave; thus, any interrogation must satisfy the requirements of Miranda. If the police had
not given him Miranda warnings before he was questioned on the first charge or if he had not
validly waived his rights, any questioning regarding the second charge would have been improper.

Answer to Question 14

(B) The best argument is that the burglar made the statements spontaneously. Prior to a suspect’s
being charged with a crime, the Fifth Amendment privilege against compelled self-incrimination
is the usual basis for ruling on the admissibility of a confession. [Miranda v. Arizona (1966)]
Under Miranda, statements made during custodial interrogations are inadmissible unless the
defendant is first warned of his right to remain silent and his right to an attorney. Thus, Miranda
applies only when the defendant is in custody and only when the defendant’s statements are the
result of interrogation. Although almost any words or actions on the part of the police that they
should know are reasonably likely to elicit an incriminating response qualify as interrogation,
Miranda does not apply to spontaneous statements not made in response to interrogation. Here,
the police did nothing to solicit the statement from the burglar; it was spontaneous. Thus, (B) is
correct. (A) is incorrect because the defendant need not yet be charged for Miranda rights to apply
as long as he is in custody (i.e., not free to leave). Being in jail on another charge (as the burglar
was) satisfies the custody requirement. (C) is incorrect because the fact that the officer who took
the burglar’s admission had nothing to do with the investigation of the burglary does not alter the
rules of Miranda—questioning that is totally unrelated to the matter for which the accused is in
custody may still violate the accused’s Miranda rights. (D) is incorrect. Due process requires that
a confession be voluntary (i.e., not the product of police coercion). The Miranda rule, however,
goes beyond voluntariness. It makes inadmissible all statements obtained without Miranda
warnings or without a valid waiver of Miranda rights, not just statements actually coerced by the
police.
CRIMINAL LAW - STATE
OF MIND AND DEFENSES
QUESTIONS
CRIMINAL LAW - STATE OF MIND AND DEFENSES QUESTIONS 1.

CRIMINAL LAW - STATE OF MIND AND DEFENSES QUESTIONS

Question 1 Question 2

A state penal statute makes it a misdemeanor A father did not like his daughter’s boyfriend.
to willfully shut off the gas, electricity, or any One night the father came home and found the
other form of power to an inhabited dwelling, daughter and the boyfriend in the den with the
unless strictly outlined procedures for notice and lights out. The father grabbed the boyfriend
hearing are met. A landowner rented a furnished and told him to get out of the house and threat-
house to a young married couple for several ened to beat the boyfriend up if he did not leave
months when she failed to receive the monthly immediately. When the boyfriend did not leave,
rent check. The couple did not answer any of the father grabbed a poker from the den fireplace
the owner’s calls, and the neighbors confirmed and raised it above his head in a threatening
that they had not seen the couple for weeks. manner. The boyfriend threw a heavy ashtray at
The owner therefore concluded that the couple the father, hitting him in the side of the head and
had abandoned the rental. She called the power killing him. The boyfriend was charged with
company and had the electricity and gas shut off murder.
until she could find another tenant. A week later,
the couple returned from an overseas tour. They The jury should find the boyfriend:
reported to the authorities that their power had
been turned off, and the owner was prosecuted (A) Guilty, because he had refused to immedi-
under the misdemeanor statute. At trial it was ately leave the house.
established that, while they were abroad, the
couple had inadvertently failed to place the (B) Guilty, because his presence in the father’s
proper postage on the rent check, which was home with the daughter provoked the father
returned to their tour guide. to attack him.

What is the most probable outcome? (C) Not guilty, because a poker is a dangerous
weapon.
(A) The owner will be convicted, because the
charged crime is a violation of a public (D) Not guilty, because it was the daughter’s
safety statute, and she is strictly liable for home also, and she invited the boyfriend
her action in turning off the power. into it.

(B) The owner will be convicted, because she


did not undertake a more thorough inquiry
or wait a more reasonable length of time
before concluding that the house had been
abandoned.

(C) The owner will be acquitted, if the trier of


fact concludes that the owner was reason-
able in believing that the house had been
abandoned.

(D) The owner will be acquitted, if the trier of


fact concludes that the young couple was
negligent in not placing proper postage on
the rent check mailed from abroad.
2. CRIMINAL LAW - STATE OF MIND AND DEFENSES QUESTIONS

Question 3 Question 4

A husband was infuriated because he found A hunter saw what he thought to be a deer
out that another man was having an affair with in a copse of trees. To approach the trees, the
his wife. The husband saw a person whom he hunter had to climb over a fence posted with “no
believed to be that other man walking down hunting” and “no trespassing” signs. He stalked
the street ahead of him. The husband struck the the deer, fired, and discovered to his chagrin that
man with a staggering blow. The husband was he had just shot a large bull. The owner of the
arrested and charged with battery upon the man. property, a rancher, heard the shot and grabbed
It turned out that the man the husband struck his shotgun, fearing that hunters had shot
was only someone who strongly resembled the another one of his cattle. When he arrived at
man who was having the affair. the copse of trees and discovered that the hunter
had killed his prize bull, he became enraged
Which of the following best states the rule to and ordered the hunter to get off his property in
be applied? extremely abusive language. Instead of leaving
immediately, the hunter said, “It’s just a damn
(A) The husband is not guilty of battery if a cow!” The rancher raised his shotgun and fired
reasonable person would have made the at the hunter, narrowly missing him. When the
same mistake as to the man’s identity. rancher operated the pump-action of the shotgun
and raised it again, the hunter shot and killed
(B) The husband is not guilty of battery if a him.
reasonable person would have made the
same mistake as to the man’s identity and In the hunter’s prosecution for murder, he
also would have been so provoked under should be found:
the circumstances to strike him.
(A) Guilty, because he was a trespasser on the
(C) The husband is guilty of battery even if he owner’s land.
honestly and reasonably believed that the
person he struck was the man who was (B) Not guilty, because the owner attacked him
having an affair with his wife. with deadly force.

(D) The husband is guilty of battery only if he (C) Guilty, because the owner’s attack was
realized whom it was that he was striking. provoked by the hunter.

(D) Not guilty, because the owner had no right


to defend his property with deadly force.
CRIMINAL LAW - STATE OF MIND AND DEFENSES QUESTIONS 3.

Question 5 Question 6

While out drinking with friends one evening, The defendant demanded that her neighbor
a young man got into an argument with a soldier cut down bushes that blocked her view, but the
from the nearby Army base and was bested in a neighbor refused. Furious, the defendant slapped
brief exchange of punches. Vowing revenge on the neighbor. In response, the neighbor grabbed
the soldier who had beaten him, the young man, a pistol from her purse and fired a shot at the
after further drinking, went to the Army base defendant, but missed. Just as her neighbor
and surreptitiously approached what he believed cocked the pistol to fire another shot, the defen-
to be the barracks where the soldier slept. As dant grabbed a shovel and hit her neighbor over
he was climbing through the window he had the head, killing her instantly. The defendant
jimmied, a military police officer happened by was charged with the common law murder of her
and challenged him. In a tussle with the MP, the neighbor. At trial, the defendant testified that she
young man struck the MP with his own baton, hit her neighbor because she believed that her
killing him. Still extremely intoxicated, the neighbor would have shot and killed her if she
young man abandoned the idea of finding and did not.
severely beating the soldier, and staggered home.
The jurisdiction’s statutes define murder as “the If the jury believes the defendant, it should
premeditated and intentional killing of another find her:
or the killing of another in the commission of
robbery, rape, burglary, or arson.” The statutory (A) Guilty of murder, because she did not
definition of burglary is identical to the common retreat.
law. The jurisdiction’s statutes provide that
intoxication is not a defense to a crime unless it (B) Guilty of murder, because she was the
negates an element of the offense. original aggressor in the encounter and had
not withdrawn.
At the young man’s trial for the murder of
the MP, the court should instruct the jury on the (C) Not guilty of murder, because her neighbor
issue of the defense of intoxication that: was the first to resort to deadly force.

(A) Voluntary intoxication is a defense to the (D) Not guilty of murder, because she had no
crime of murder if the young man would opportunity to premeditate.
not have killed the MP but for the intoxica-
tion.

(B) The young man is guilty of murder despite


his intoxication only if the prosecution
proves by clear and convincing evidence
that he acted with premeditation and inten-
tionally.

(C) Voluntary intoxication is no defense to the


crime of murder.

(D) Intoxication is a defense to the crime of


burglary if it prevented the young man from
forming the intent to commit a crime inside
the barracks, in which case he could only be
convicted of murder on the requisite showing
of intentional action and premeditation.
4. CRIMINAL LAW - STATE OF MIND AND DEFENSES QUESTIONS

Question 7 Question 8

After becoming intoxicated one evening, The police had long suspected that an appli-
the defendant went to a used car lot to try to ance store owner was selling stolen merchandise,
find a car that he could use to drive home. On but were unable to prove it. One night, under the
discovering that none of the cars had keys in direction of the police, an informant approached
the ignition, he broke into the main office of the the store owner and offered to sell him a micro-
used car lot to get a key. As he left the office wave that was represented as stolen. In fact,
with a key, a security guard tried to apprehend the oven belonged to one of the officers. The
the defendant. The defendant pushed the guard store owner indicated that he was interested and
to the ground. The guard hit his head hard on suggested meeting at a local park to complete
the pavement and died. Relevant statutes extend the sale. The informant showed up at the sched-
burglary to include buildings not used as a uled time, but the store owner did not. When
dwelling. First degree murder is defined as “the the informant saw the owner again, he asked
premeditated and intentional killing of another why he had not shown up. The owner replied,
or a killing committed during the commission “I wanted to make sure you were not working
of a rape, robbery, burglary, or arson.” Second for the police.” The two men then set up another
degree murder is defined as all murders that are meeting in the park, at which time the owner
not first degree murder. purchased the microwave for $25. Shortly there-
after, the owner was arrested by the police with
If the defendant is charged with first degree the microwave in his possession.
murder for the death of the security guard, the
court should charge the jury on the issue of the On the issue of entrapment, the court should
defense of intoxication that: find:

(A) Voluntary intoxication is no defense to the (A) The conduct of the police amounted to
crime of first degree murder as defined by entrapment, because it directly implanted a
the statute. criminal intent in the mind of the owner.

(B) Voluntary intoxication is a defense to the (B) The conduct of the police amounted to
crime of first degree murder if the defen- entrapment, because the informant was not
dant would not have killed the security a member of the police department.
guard but for the intoxication.
(C) The conduct of the police amounted
(C) Voluntary intoxication is a defense to the to entrapment, because the informant
crime of first degree murder if it prevented performed a necessary act for the crime
the defendant from forming the intent to of knowingly receiving stolen property by
commit a burglary. making the property available to the owner
for purchase.
(D) Voluntary intoxication is a defense to first
degree murder if it prevented the defendant (D) The conduct of the police did not amount to
from forming the intent to kill the security entrapment, because the owner already had
guard. a preexisting criminal intent to buy stolen
property, and the action of the informant
merely afforded him an opportunity to
carry out his criminal intent.
CRIMINAL LAW - STATE OF MIND AND DEFENSES QUESTIONS 5.

Question 9 Question 10

A state statute reads in pertinent part: “Any A patient who was mentally incompetent and
licensed medical doctor who willfully neglects a ward of the state received extensive rehabilita-
to assist anyone with a life-threatening injury tion for leg and back injuries she suffered in a
shall be guilty of a crime, subject to punishment fall at a state institution. After her rehabilitation
of up to two years’ imprisonment in the state was nearly completed, she became entitled to
penitentiary or a fine of $15,000, or both.” state payments as compensation for her injury,
which her treatment center began applying to
A licensed physician, while jogging in a her outstanding rehabilitation bill. Thereafter,
park, heard the sounds of a man moaning in the legal guardian for the patient sought to
the bushes next to the jogging path. The physi- remove her from the center despite the fact that
cian stopped to investigate and found the man the patient had never shown any desire to leave.
bleeding in the bushes. The physician asked the The director of the center was aware that her
man if he needed help, and the man responded departure would prevent the center from directly
that he would be fine. Not having seen any applying the state payments to her bill. The
indications of a serious condition, the physician director was erroneously advised by his attorney
returned to her morning jog. The man subse- that judicial decisions would support his refusal
quently bled to death an hour later. The coroner’s of the guardian’s request until the bill was paid.
report made clear that the man could have been When the guardian arrived at the center, the
saved if he had received prompt medical atten- director refused to allow the guardian to remove
tion, even if only of a “first aid” variety until an the patient. A criminal statute in the jurisdic-
ambulance arrived. tion defines false imprisonment as knowingly
confining a person without valid consent and
The physician was charged with violating the without authority of law.
statute. At trial, the physician testified that she
was unaware of the statute, which had been in If the director is arrested and charged with
effect for one week when the incident took place. false imprisonment, can he be found guilty?

Should the physician be convicted of violating (A) Yes, if the director’s reliance on the advice
the statute? of his attorney was not reasonable.

(A) Yes, because she neglected someone whose (B) Yes, because the director kept the patient
life was in danger. without lawful authority.

(B) No, because the statute is too vague and (C) No, because the patient was not held
raises mere negligence to criminal status. against her will and was not harmed by the
confinement.
(C) No, because the physician was unaware of
the new statute. (D) No, because the director’s belief in the
lawfulness of his conduct precluded him
(D) No, because the physician believed that the from having the mental state required for
man’s condition was not life-threatening. the offense.
6. CRIMINAL LAW - STATE OF MIND AND DEFENSES QUESTIONS

Question 11 Question 12

A state statute contains the following provi- A statute provides “any person who
sions: knowingly sells intoxicating liquor to a person
under 21 years of age is guilty of a misdemeanor
Murder in the first degree is the killing of and may be fined $1,000 or sentenced to up to
a human being without justification and with six months in jail, or both.” A patron ordered
premeditation. Murder in the second degree is a drink at a bar. The bartender asked for some
any murder which is neither a murder in the first identification, and the patron produced a driver’s
degree nor a murder in the third degree. Murder license stating that he was 25 years old. In fact,
in the third degree is any killing that occurs the patron was only 18 years old, but he had
during and as a result of the commission of a some gray hair and was balding. A plainclothes
felony. police officer sitting at the bar witnessed the
bartender selling a drink to the patron. The
The jurisdiction also provides that robbery is a officer knew that the patron was under 21 years
felony for purposes of the felony murder rule. old, and demanded to see the patron’s driver’s
license. The officer determined it to be false and
A robber held up a convenience store with a arrested the bartender for violation of the statute.
knife. After the store clerk handed over all the
money in the register, she retrieved a pistol from Will the bartender likely be found guilty?
a drawer behind the counter. When the robber
turned to leave, the store clerk pointed the gun at (A) Yes, because this type of statute is a strict
the robber and shouted, “Stop or I’ll shoot!” The liability statute and the bartender’s knowl-
robber ducked behind the end of an aisle, and edge of the patron’s age is irrelevant.
the store clerk shot three times in the robber’s
direction. One of the bullets bounced off a pillar (B) Yes, because the bartender knew that the
and struck the only other customer in the store, patron was under 21 years old.
who was crouched behind a counter, killing him
instantly. (C) No, because the bartender made a reason-
able mistake concerning the age of the
The crimes below are listed in descending patron.
order of seriousness.
(D) No, because the patron produced a driver’s
What is the most serious homicide crime of license that stated that he was 25 years old.
which the store clerk can be convicted?

(A) Murder in the second degree.

(B) Murder in the third degree.

(C) Manslaughter.

(D) No homicide crime.


CRIMINAL LAW - STATE OF MIND AND DEFENSES QUESTIONS 7.

Question 13 Question 14

A robber approached a newsstand, with her Two men drinking at a local bar got into a
knife drawn, in an attempt to rob the attendant. heated argument. The small, slightly built man
The attendant gave the robber the small sum of knew that the large burly man had a short fuse,
money in his possession. As the robber turned yet continued to argue with him. The larger man
away, the attendant grabbed a gun and fired insulted the smaller man’s religion and national
several times. He intended only to wound the origin, whereupon the smaller man spat on the
robber but instead killed her. other, who responded by pouring a glass of
beer over the smaller man’s head. The smaller
If the attendant cannot be prosecuted for man then punched the larger man in the nose,
manslaughter, what is the likely reason? catching him off guard and knocking him to
the floor. The larger man got to his feet, pulled
(A) He only intended to wound the robber. out a knife, and advanced toward the other,
who was standing by the door. The smaller man
(B) The robbery constituted a provocation. reached inside his boot and drew out a small gun
and shot the larger man, killing him instantly.
(C) The attendant was trying to get his money The jurisdiction makes it a crime to carry a
back. concealed weapon. The smaller man is charged
with murder.
(D) He was apprehending a fleeing wrongdoer.
If the smaller man claims the killing was in
self-defense, which of the following is the most
helpful to the prosecution?

(A) The smaller man initiated the physical vio-


lence by spitting on the larger man, and his
punching the larger man in the nose is what
caused him to threaten the smaller man
with the knife.

(B) The smaller man was standing very close


to the door and could have broken off the
affray if he had chosen to do so.

(C) The use or possession of the type of gun


that the man used is a crime under state
law, and carrying any concealed weapon is
a separate crime.

(D) Before any violence erupted, the smaller


man was aware that the larger man was
becoming increasingly quarrelsome and
belligerent, and continued to drink and
argue with him notwithstanding.
8. CRIMINAL LAW - STATE OF MIND AND DEFENSES QUESTIONS

Question 15 Question 16

State law required all businesses in the state A thug approached the defendant in front of a
to report any deposits of toxic waste discov- bank, brandishing a gun. He told the defendant
ered on property owned by the reporting firm. that he was going to rob the bank and needed a
Another provision of the law required that the “bag man.” The defendant complied at gunpoint,
toxic waste reports be made on a specific state holding out a bag for the tellers to put money
form and that duplicate copies of the form be in while the thug waved the gun at everyone.
filed with both the state department of labor and The gun accidentally went off, killing one of
the state environmental protection agency. The the tellers. The defendant is charged with the
statute provided that after each report was filed, criminal homicide of the teller.
the relevant state agencies would ensure that any
dangerous area was cordoned off and work out What is the most likely result?
a plan with the reporting firm for the clean-up
of the waste deposit. The effective date of the (A) The defendant will be found guilty of
statute was June 15, but the state office did not murder.
produce any of the special forms until July 6.
(B) The defendant will be found guilty of
The defendant, a safety inspector employed by manslaughter.
a chemical company, was making her first visit
to one of the company’s plants on June 17 when (C) The defendant will be found not guilty
she noticed a strange substance on the premises. of murder, because duress is a defense to
The defendant filed no report to the state robbery.
agencies regarding the substance because she
was not aware of the requirement, nor did she (D) The defendant will be found not guilty
know that the substance was toxic. On June 20, of murder, because the defendant never
another employee of the chemical company was intended to kill anyone.
walking in an area near the substance when he
slipped and fell face first into it. The substance
turned out to be very toxic. The employee’s skin
contact with the substance was sufficient to kill
him in a matter of minutes. A later investiga-
tion revealed that it would have been impossible
to clean up the substance to make the area safe
before the employee encountered it.

If the defendant is charged with murder, which


of the following represents her best defense?

(A) She did not know that the substance was


toxic.

(B) She was unaware of the state reporting law.

(C) The state printing office did not produce


any copies of the special forms until July 6.

(D) It would have been impossible to clean up


the toxic waste deposit before the time that
the employee encountered it.
CRIMINAL LAW - STATE
OF MIND AND DEFENSES
ANSWERS
CRIMINAL LAW - STATE OF MIND AND DEFENSES ANSWERS 1.

CRIMINAL LAW - STATE OF MIND AND DEFENSES ANSWERS

Answer to Question 1

(C) The owner would be acquitted if the jury found that she was reasonable in believing that the house
was abandoned. Under the terms of the statute, the actor must willfully shut off the gas to an
inhabited dwelling. The requirement of an inhabited dwelling is a material element of the crime.
A reasonable mistake about a material element will negate criminal liability for all crimes except
strict liability offenses. This is not a strict liability offense because a mental state of willfulness is
required. Thus, if the owner reasonably believed that the house was abandoned, she made a reason-
able mistake about whether it was an inhabited dwelling, and she should be found not guilty. (A)
is wrong because, as mentioned, this is not a strict liability offense; willfulness is required. (B) is
not as good an answer as (C) because there is no specific amount of time that she must wait before
reasonably concluding that the house has been abandoned. (B) assumes that the jury will find
her actions unreasonable, but it is also possible that they will find her actions reasonable. (D) is
wrong because if all the elements of the statute were present, the owner would not be excused from
criminal liability simply because the couple had been negligent in mailing the rent check.

Answer to Question 2

(C) Clearly the boyfriend was acting in self-defense, and even if throwing a heavy ashtray could be
deemed to be deadly force, the boyfriend had the right to defend himself against the unprivi-
leged attack with the use of deadly force (i.e., the poker). (A) and (B) are incorrect because the
boyfriend’s presence in the house and failure to leave immediately did not give the father a privi-
lege to use deadly force against him. (D) is irrelevant.

Answer to Question 3

(C) The husband is guilty of battery regardless of his belief. Battery is an unlawful application of
force to the person of another resulting in either bodily injury or an offensive touching. Here, the
husband made a mistake as to the identity of the person whom he struck, but there was no mistake
as to the intention of the husband to strike this person. Mistake of fact affects criminal guilt only
if it shows that the defendant did not have the state of mind required for the crime. Although
battery need not be intentional (i.e., it can result from criminal negligence), it is clear from the
facts that the husband intended to strike the man. Thus, the mistake made by the husband (who
apparently believed that he was striking the man having the affair with his wife) does not show
the absence of the state of mind required for battery, and will therefore not affect the husband’s
guilt. (A) is incorrect because, even if a reasonable person would have made the same mistake
of identity, the fact remains that the husband committed a battery against the man he struck. A
case of mistaken identity provides no defense to the battery that was committed. (B) is incorrect
for the same reason as (A), and also because it implies that a battery against the man having an
affair with the defendant’s wife would be excused by adequate provocation (presumably resulting
from the affair with his wife). It is true that a killing that would otherwise be murder may be
reduced from murder to voluntary manslaughter if the defendant acted under a legally adequate
provocation, and finding one’s spouse in the act of adultery is deemed to be adequate provocation.
However, there is no rule of law that excuses a battery on the ground that the victim committed
adultery with the defendant’s wife. (D) is incorrect because it also is based on the premise that the
husband is not guilty if he thought that he was striking the man who was having an affair with
his wife. As explained previously, regardless of whom the husband thought he was striking, the
significant point is that he unlawfully struck a man, and this constitutes a battery.
2. CRIMINAL LAW - STATE OF MIND AND DEFENSES ANSWERS

Answer to Question 4

(B) The hunter is not guilty of murder because he acted in self-defense in response to the owner’s
deadly attack. At common law, murder is the unlawful killing of another with malice afore-
thought. Malice aforethought may be shown by (i) the intent to kill, (ii) the intent to inflict great
bodily injury, (iii) a reckless indifference to an unjustifiably high risk to human life (“abandoned
and malignant heart”), or (iv) the intent to commit a felony. In the instant case, it is clear that the
hunter intentionally shot and killed the rancher. However, a killing that otherwise might constitute
murder will be excused if the defendant was acting in self-defense. A person may use deadly force
in self-defense if he (i) is without fault, (ii) is confronted with unlawful force, and (iii) reason-
ably believes that he is threatened with imminent death or great bodily harm. All three elements
can be met here. Although the hunter was a trespasser and accidentally shot the rancher’s bull, he
did nothing to initiate or provoke the physical altercation, as the rancher would not be entitled to
use deadly force against a trespasser. The hunter was also confronted with unlawful force when
the rancher shot at the hunter and raised his rifle to take a second shot at the hunter. Clearly, this
would also represent a threat of imminent death. Thus, the hunter was entitled to use deadly force
in self-defense, making him not guilty of murder. This is not affected by the fact that the hunter
was a trespasser (choice (A)) or that the owner was enraged by the hunter’s negligence and attitude
(choice (C)). (D) is a correct statement of the law, but inapplicable to the given facts, given that
the hunter was not acting in defense of property, but rather in self-defense.

Answer to Question 5

(D) Burglary requires a specific intent to commit a crime and intoxication might negate the existence
of that intent. In the circumstances of the problem, the young man could argue with some force
that he did not intend to kill the MP or act with premeditation, but must account for the fact that
the killing occurred in what could be characterized as a burglary, which would be murder under
the statutes of the jurisdiction. Thus, if the young man did not have the requisite intent, he did not
commit burglary and so the killing does not fall under the felony murder part of the statute. The
young man can therefore be convicted only if he acted intentionally or with premeditation. (A) is
incorrect because voluntary intoxication is not a defense unless it negates intent; (A) concerns the
issue of causation. (B) is wrong because it states the wrong standard of proof—the prosecution
must prove all elements of the crime beyond a reasonable doubt. (C) is wrong because voluntary
intoxication may be a defense to the felony murder definition of murder.

Answer to Question 6

(C) The jury should find the defendant not guilty of murder. If the victim of the initial aggression
suddenly escalates a “minor” fight into one involving deadly force, and does so without giving the
initial aggressor the chance to withdraw or retreat, the aggressor may use deadly force in her own
defense. Here, even though the defendant was the initial aggressor, she reacquired her right to self-
defense because her neighbor responded to the defendant’s slap (nondeadly force) by shooting at
her (deadly force) without giving the defendant a chance to withdraw. Thus, the defendant regained
the right to use deadly force in self-defense. (C) is therefore correct, and (B) is incorrect. (A) is
incorrect because the majority of jurisdictions do not require a party to retreat before using deadly
force. Furthermore, even in those jurisdictions that follow the retreat rule, a person is not required
to retreat unless she can do so with complete safety. Here, given that the neighbor was cocking the
pistol to fire again at close range, the defendant would not have been able to retreat in safety. (D)
is incorrect because, under the common law, a defendant can be guilty of murder even if she did
not premeditate. At common law, malice aforethought includes (i) the intent to kill; (ii) the intent
CRIMINAL LAW - STATE OF MIND AND DEFENSES ANSWERS 3.

to inflict great bodily injury; (iii) a reckless indifference to an unjustifiably high risk to human life
(“abandoned and malignant heart”); and (iv) the intent to commit a felony for felony murder.

Answer to Question 7

(C) The court should instruct the jury that the defendant is not liable for first degree murder if he did
not form the intent to commit the crime of burglary, because a defense to the underlying felony
precludes a conviction for felony murder. Voluntary intoxication is a defense to specific intent
crimes, such as burglary, if it prevents the defendant from formulating the requisite intent. In the
absence of the defendant’s liability for burglary, he cannot be convicted of first degree murder
based on the facts presented here. (A) is incorrect because voluntary intoxication may be a defense
to first degree murder under the circumstances here. (B) is an incorrect statement of law; the
intoxication must negate a specific intent for the underlying felony for it to serve as a defense. (D)
is incorrect; while premeditated murder is a specific intent crime for which voluntary intoxication
may be a defense, nothing in the facts suggests that the charge is based on premeditation. Here,
the first degree murder charge is based on commission of a burglary.

Answer to Question 8

(D) The conduct of the police did not amount to entrapment. To prevail with the defense of entrap-
ment, the defendant must establish that: (i) the idea for the crime originated with the police, and
(ii) the defendant was not predisposed to commit the crime prior to solicitation by the govern-
ment. On the facts, the store owner will be unable to establish the second element. (A) is wrong.
The store owner was predisposed to committing the crime. (B) makes no sense. The fact that the
informant was not a member of the department would not be relevant as to the issue of entrap-
ment. (C) is wrong. Even if the police did perform an act necessary for the completion of the
crime, there will be no “entrapment” if the defendant was predisposed.

Answer to Question 9

(D) Because the physician believed that the man’s condition was not life-threatening, she should
not be convicted because she lacked the requisite intent for the crime. The statute here makes
it a crime for a licensed physician to willfully neglect someone with a life-threatening injury. A
requirement of willful action means that the person must have acted knowing that her conduct
would necessarily cause such a result. Thus, she must be aware of the circumstances and act with
the awareness of what the results of her conduct will be. To be convicted under this statute, the
physician must have been aware that the man’s condition was life-threatening and that leaving
him there without any assistance could cost him his life. Because the facts indicate that the physi-
cian believed that the man was not in a life-threatening situation, she did not violate the statute
by leaving the man unattended. She did not “willfully” neglect him, because she was not aware
of the status of his situation. (A) is wrong because it fails to account for the required mental state
(i.e., willfulness). Choice (A) would impose a strict liability; a doctor would be liable whether or
not she knew of the life-threatening situation. The statute here does not impose strict liability;
only willful neglect is punishable. (B) is wrong because the statute is not too vague. The statute
gives a person of ordinary intelligence fair notice of what sort of action or inaction is punishable
(i.e., the willful failure of a licensed medical physician to assist someone with a life-threatening
injury). (C) is wrong because ignorance of the law is no excuse. It is not a defense to a crime that
the defendant did not know that her acts were prohibited by criminal law. This is true regardless
of whether her ignorance was reasonable. Thus, despite the fact that the statute was in effect for
only one week, the physician may be convicted whether she was aware of it or not.
4. CRIMINAL LAW - STATE OF MIND AND DEFENSES ANSWERS

Answer to Question 10

(D) The director cannot be found guilty under the false imprisonment statute. Despite the general
rule that it is no defense to a crime that the defendant mistakenly believed that his conduct was
not prohibited by the criminal law, a mistake as to some aspect of law pertaining to the elements
of the crime rather than the existence of the statute making the act criminal may negate the state
of mind required for the crime. When a culpable state of mind is specified by an offense without
indicating to which element it applies, the state of mind applies to all material elements of the
offense unless a contrary purpose appears in the statute. Thus, the state of mind required here,
“knowingly,” requires not only that the director know of the patient’s confinement, but also that
the director know there is no valid consent and that he has no legal authority to confine her. His
mistake as to his legal authority to confine her negates the state of mind required for that element
of the false imprisonment statute. (A) is wrong because whether the director’s reliance on his
attorney’s advice was reasonable is irrelevant. As long as he believed that he had lawful authority
to hold the patient, the director did not have the state of mind required for the offense. (B) is
wrong because, as stated above, the director would have had to have knowledge of the absence
of lawful authority before he could be found liable under the false imprisonment statute. (C) is
wrong because the patient’s compliance with her confinement and her absence of harm are not
relevant. Valid consent to negate a charge of false imprisonment cannot be obtained from one
without capacity to give such consent, and the patient, being mentally incompetent, did not have
the capacity to consent to the confinement. Absence of harm may be a defense to a tort action for
false imprisonment where the victim is not aware of the confinement, but it is not a defense to the
crime of false imprisonment.

Answer to Question 11

(D) The store clerk can be convicted of no homicide crime because shooting at the robber was autho-
rized by law. A private person has the right to use deadly force to effectuate an arrest when the
felon appears to pose a threat to the person or to others and deadly force is necessary to prevent
his escape, as long as the felon was actually guilty of the felony. Similarly, a person has the right
to use deadly force in preventing the completion of a crime being committed if the crime is a
“dangerous felony” involving risk to human life. Here, the store clerk had the right to use deadly
force against the robber, even though she was not directly threatened with imminent death or
great bodily harm at the time she fired, because the robber was armed with a deadly weapon
and appeared to pose a danger to anyone who might try to stop him. Although this right to shoot
at the robber would not justify the killing of the customer if the clerk acted with malice afore-
thought as to the customer, in this case the customer was not in the line of the clerk’s fire. Thus,
the clerk’s conduct would not constitute reckless indifference to an unjustifiably high risk to
human life or any other state of mind constituting malice aforethought. The clerk therefore can
be convicted of no homicide crime. (C) is incorrect because the facts do not support a conviction
for manslaughter. The clerk is not guilty of voluntary manslaughter because she did not inten-
tionally kill anyone while under legally adequate provocation. She is not guilty of involuntary
manslaughter because her shooting at the robber was authorized by law and, given the low proba-
bility of anyone else being hit, was not such a great deviation from the conduct of a reasonable
person as to constitute criminal negligence. (A) is incorrect because, as discussed above, the clerk
did not act with reckless indifference to human life or any other state of mind constituting malice
aforethought as to anyone other than the robber. (B) is incorrect because the clerk did not cause a
death during the commission of a felony; she was trying to stop the completion of a felony when
she shot the gun.
CRIMINAL LAW - STATE OF MIND AND DEFENSES ANSWERS 5.

Answer to Question 12

(C) From the facts, it appears that the bartender made a reasonable mistake as to the age of the patron,
and thus he did not have the mental state necessary for the crime charged. In this question, the
statute requires a mental state of knowingly selling to a person under age 21; therefore, a mistake
concerning the age of the purchaser would negate criminal liability. (A) is wrong. In many juris-
dictions, selling liquor to a minor is a strict liability offense. In such jurisdictions, a person would
be guilty if he sold the liquor to a minor even if he made a reasonable mistake about age. The
statute in question, however, is not a strict liability offense because it requires a knowing state of
mind. (B) is wrong because it is simply an incorrect statement of the facts. There is nothing in the
question indicating that the bartender knew that the patron was younger than 21. (D) is wrong.
Although the driver’s license would be a consideration in deciding whether the bartender made a
reasonable mistake, it would not of itself negate criminal liability: The bartender would be guilty
if he knew that the patron was underage even if the patron produced a driver’s license stating
otherwise.

Answer to Question 13

(D) If the attendant cannot be prosecuted for manslaughter, it will be because he was privileged to use
deadly force to apprehend a fleeing felon. A private person may use deadly force to apprehend a
fleeing felon if the felon threatens death or serious bodily harm and deadly force is necessary to
prevent her escape. Also, for a private person to use force to effect an arrest, the person harmed
must actually be guilty of the felony for which the arrest was made; i.e., it is not sufficient that it
reasonably appeared that the person was guilty. Here, the robber actually committed the felony
of robbery. Because she was armed with a knife, the robber threatened death or serious bodily
harm, and deadly force was necessary to prevent her escape. Thus, the newsstand attendant’s best
argument is that he had the right to use deadly force to apprehend her, and this serves as a defense
to a charge of manslaughter. (A) is incorrect because acting with the intent to inflict serious
bodily injury is a sufficient mens rea for voluntary manslaughter. Thus, the fact that the attendant
intended only to inflict great bodily injury is no defense to manslaughter. (B) is incorrect because
the existence of provocation is not a defense to manslaughter. Rather, provocation must be found
to reduce a killing from murder to manslaughter. Thus, the fact that the attendant may have been
provoked by the robbery would, if anything, provide a basis for a manslaughter prosecution, rather
than a defense thereto. (C) is incorrect because deadly force may not lawfully be used merely to
regain possession of property. Deadly force could only be justified by another basis of privilege,
such as to effectuate an arrest.

Answer to Question 14

(B) The most helpful fact for the prosecution is that the smaller man had an opportunity to retreat
safely. The general rule is that one may use deadly force in self-defense even if the use of force
could be avoided by retreating safely. This rule, however, does not apply to one who is the initial
aggressor. Generally, one who is at fault for starting a confrontation has no right to use force in
his own defense during the confrontation. But the aggressor can regain his right to use force in
self-defense either (i) by attempting to withdraw and communicating the withdrawal to the other
person or (ii) when the other person suddenly escalates a minor fight into one involving deadly
force without giving the aggressor the chance to withdraw or retreat. Here, the smaller man was
the initial aggressor because he spat on the larger man and struck the first blow when he punched
him in the nose. The larger man then escalated the fight into one involving deadly force by pulling
a knife. However, the smaller man would not regain his right to self-defense unless “his back was
6. CRIMINAL LAW - STATE OF MIND AND DEFENSES ANSWERS

to the wall”; i.e., if he had a chance to withdraw rather than respond to the larger man’s deadly
force, he had a duty to do so. Thus, the prosecution could overcome the smaller man’s claim of
self-defense with the fact that he was very close to the door and could have withdrawn from the
confrontation. (A) is incorrect because the fact that the smaller man initiated the violence does
not necessarily extinguish his right of self-defense here, because the larger man escalated the
fight into one involving deadly force. To rebut the smaller man’s claim under these circumstances,
the prosecution must show that the smaller man had an opportunity to withdraw that he did not
use. (C) is incorrect because it is irrelevant to the smaller man’s right to use deadly force in self-
defense. The fact that the smaller man is guilty of weapons violations could be used in a separate
prosecution against him, but it would have no bearing on his right of self-defense. (D) is also irrel-
evant to the smaller man’s right of self-defense. The smaller man was under no duty to retreat or
to refrain from arguing with the larger man despite his belligerence. The only basis for the smaller
man’s losing his right to defend himself was his initiation of the physical contact.

Answer to Question 15

(A) The defendant’s best defense is that she did not know that the substance was toxic and therefore
did not have the state of mind necessary for murder. Ignorance or mistake as to a matter of fact
will serve as a defense to a crime if it shows that the defendant did not have the state of mind
required for the crime. If the mistake is offered as a defense to a malice crime, the mistake must
be reasonable; i.e., it must be the type of mistake that a reasonable person might make under
the circumstances. Although the statute itself may create a strict liability offense, for which the
defendant’s mistake as to the identity of the substance would not be a defense, the defendant
here is being charged with murder, which is a malice crime. For the defendant to be convicted of
murder, it must be shown that she had: (i) intent to kill; (ii) intent to inflict great bodily injury;
(iii) reckless indifference to an unjustifiably high risk to human life; or (iv) intent to commit a
felony. If she knew that the substance was toxic, the prosecution would have an argument that
she acted with reckless indifference to an unjustifiably high risk to human life by not taking any
action. However, if she did not know that the substance was toxic, her failure to act is far less
likely to establish reckless indifference. In other words, the defendant was operating under such
ignorance as to a matter of fact that she did not have the requisite state of mind for murder. This
would provide the defendant with a defense. (B) is incorrect because it is not a defense to crime
that a defendant was unaware that her acts were prohibited by the criminal law, or that the law
compelled her to do something. Thus, even though the defendant was unaware of the reporting
law, such ignorance of the law is no defense. Furthermore, regardless of her awareness of the
reporting law, the defendant, as a safety inspector for the chemical company, may have owed
a duty to the employee to act to clean up toxic waste. For this reason as well, the defendant’s
ignorance of the reporting law would not provide her with a strong argument. (C) does not provide
a strong defense argument because the requirement of filing the reports on a specified form is a
mere administrative detail. The fact that the forms were not yet available would not relieve the
defendant of her duty to report any known deposits of toxic waste or to otherwise act to safeguard
any employees from such waste. (D) is incorrect because, even if it would have been impossible
to clean up the waste deposit by the time the employee encountered it, an immediate report by the
defendant on the existence of the deposit would at least have resulted in warning the employees to
stay away from the area, perhaps leading also to the cordoning off of the area.

Answer to Question 16

(C) The defendant is not guilty of robbery (because he acted under duress) and therefore he is
not guilty of the teller’s homicide. A person is not guilty of an offense, other than intentional
CRIMINAL LAW - STATE OF MIND AND DEFENSES ANSWERS 7.

homicide, if he performs the otherwise criminal act under the threat of imminent infliction of
death or great bodily harm and his perception of the threat is reasonable. Here, the defendant
helped the thug rob the bank because he was ordered to do so or he would be shot. Because the
thug had a gun, the defendant’s perception of the thug’s threat of imminent death or great bodily
harm was reasonable. Therefore, the defendant acted under duress and is not guilty of robbery.
If the defendant is not guilty of robbery, then he cannot be guilty of murder. Murder requires a
mental state of malice aforethought (i.e., an intent to kill or inflict great bodily harm, an aware-
ness of an unjustifiably high risk to human life, or the intent to commit a felony). Here, the state’s
only basis for showing malice aforethought would be to use the felony murder doctrine. Under
that doctrine, a killing, even an accidental one, that occurs during the course of a felony is murder.
The only felony here is robbery, and because the defendant is excused from criminal liability
for that crime due to duress, he cannot be convicted of murder under the felony murder doctrine
because he did not commit the underlying felony. Thus, (C) is correct and (A) is incorrect. (B)
is incorrect because the defendant did not commit either voluntary or involuntary manslaughter.
Voluntary manslaughter is an intentional killing distinguishable from murder by the existence
of adequate provocation. As explained above, the defendant did not have the requisite intent for
murder (malice aforethought) and thus he cannot be guilty of voluntary manslaughter. Nor is the
defendant guilty of involuntary manslaughter. Involuntary manslaughter consists of either criminal
negligence or “unlawful act” manslaughter. Because the defendant acted under duress, he was
not criminally negligent, nor was his conduct an inherently wrongful act necessary for “unlawful
act” manslaughter. Therefore, there is no basis to find the defendant guilty of manslaughter. (D) is
incorrect because one can be convicted of murder without intending to kill (e.g., in the case of a
felony murder). Thus, if the defendant were guilty of robbery, he could be found guilty of murder
despite the fact that he did not intend to kill anyone.
CRIMINAL LAW - TRIAL
AND POST-TRIAL RIGHTS
QUESTIONS
CRIMINAL LAW - TRIAL AND POST-TRIAL RIGHTS QUESTIONS 1.

CRIMINAL LAW - TRIAL AND POST-TRIAL RIGHTS QUESTIONS

Question 1 Must the appellate court also overturn the


defendant’s conviction?
A defendant was charged with robbing and
murdering a store clerk. A security camera had (A) No, because the error in admitting the evi-
recorded the incident, clearly showing that the dence was harmless.
defendant shot the store clerk at point blank
range, and that the defendant dropped the (B) No, because the tainted evidence would not
handgun immediately after shooting the store have affected a judge’s decision as much as
clerk. The gun, with the defendant’s fingerprints that of a jury.
on it, was recovered. Two eyewitnesses also were
able to conclusively identify the defendant as the (C) Yes, because the police should have
robber/murderer. After the defendant had been obtained a warrant before searching his
validly arrested, the police conducted a warrant- home.
less search of the defendant’s home, and several
items from the store were found. The items from (D) Yes, because a violation of a constitutional
the store were introduced at trial over the defen- right is never harmless.
dant’s objection. The defendant was convicted
after a nonjury trial. On appeal, the appellate
court ruled that the warrantless search was
unconstitutional under the circumstances.
2. CRIMINAL LAW - TRIAL AND POST-TRIAL RIGHTS QUESTIONS

Question 2 The judge further instructed the jury that it


could return an acquittal by reason of insanity
Criminal statutes in the state define murder “if the defendant established by clear and
as “the unlawful killing of another human convincing evidence that he was unable to
being with malice aforethought, either express control his actions or conform his conduct to
or implied,” and define voluntary manslaughter the law.” The jury found the defendant guilty of
as “the unlawful killing of another human murder. The defendant appealed, claiming that
being under an extreme emotional disturbance his constitutional rights were violated by the
for which there was reasonable explanation court’s instructions as to homicide and by the
or excuse, without express or implied malice requirement that he prove insanity by clear and
aforethought.” Another statute provides that “it convincing evidence.
shall be an affirmative defense to the charge
of murder if the defendant proves by clear and Were the defendant’s rights violated?
convincing evidence that the defendant was
unable to control his actions or conform his (A) Yes, by the court’s instructions as to homi-
conduct to the law.” cide, but not the requirement of clear and
convincing evidence for insanity.
The defendant was charged with murder
and tried in state court. At trial, he introduced (B) Yes, by the requirement of clear and
evidence regarding his state of mind at the time convincing evidence for insanity, but not
of the homicide, including testimony from a the instructions as to homicide.
psychiatrist. At the conclusion of the case, the
court instructed the jury as follows: (C) Yes, by both the homicide instructions and
the clear and convincing evidence require-
There are two kinds of homicide in our ment.
state: murder and manslaughter. The common
elements of both are that the homicide be (D) No, neither the homicide instructions nor
unlawful—i.e., neither justifiable nor excus- the clear and convincing evidence require-
able—and that it be intentional. Malice afore- ment violated the defendant’s rights.
thought is an indispensable element of the
crime of murder. However, if the prosecution
can establish that the killing was both inten-
tional and unlawful, malice aforethought may
be presumed unless the defendant proves by
a preponderance of the evidence that he acted
under extreme emotional disturbance for which
there was reasonable explanation or excuse, in
which case he shall be liable only for volun-
tary manslaughter, because extreme emotional
disturbance is inconsistent with, and negates the
existence of, malice aforethought.
CRIMINAL LAW - TRIAL AND POST-TRIAL RIGHTS QUESTIONS 3.

Question 3 Question 4

A defendant was charged with murder. His The defendant was arrested and charged with
principal defense was that he had killed in hot possession of illegal narcotics and placed on trial
blood and should be guilty only of manslaughter. in a municipal court. The defendant demanded
The judge instructed the jury that the state must a jury trial, which was duly granted. After the
prove guilt beyond a reasonable doubt, that the jury had been sworn, selected, and impaneled,
killing was presumed to be murder, and that the the defendant’s attorney filed a motion with the
charge could be reduced to manslaughter, and presiding judge praying for a dismissal based on
the defendant accordingly found guilty of this a technical error in the bill of information drawn
lesser offense, if the defendant showed by a fair up by the city prosecutor. No witnesses had yet
preponderance of the evidence that the killing been sworn at the time the attorney filed the
was committed in the heat of passion on sudden motion. The judge ordered an immediate recess
provocation. The defendant was convicted of while he considered the motion. Two days later
murder. On appeal, he seeks a new trial and he ruled in favor of the defendant and dismissed
claims error in the judge’s instructions to the the charges against him.
jury.
A week later a state grand jury indicted the
Will the defendant’s conviction be affirmed? defendant for possession of illegal narcotics with
intention to distribute same. The charges arose
(A) Yes, because the judge carefully advised out of the same incident and arrest described
the jury of the state’s obligation to prove above. The defendant was ordered to appear
guilt beyond a reasonable doubt. in the superior court of the state to answer the
charges. The defendant’s attorney immediately
(B) Yes, because the defendant’s burden to filed a motion on the defendant’s behalf asserting
show hot blood was not one of ultimate that it would be unconstitutional to retry the
persuasion but only one of producing defendant in the state court.
evidence to rebut a legitimate presumption. The best argument against granting the
motion is which of the following?
(C) No, because the instruction put a burden on
the defendant that denied him due process (A) The city and the state are separate sover-
of law. eigns.
(D) No, because presumptions have a highly (B) The state charge requires the proving of a
prejudicial effect and thus cannot be used fact not required by the municipal charge.
on behalf of the state in a criminal case. (C) The defendant’s trial in municipal court had
not yet reached the stage where jeopardy
attaches.
(D) The dismissal of the case in municipal
court was based on a technicality that did
not go to the merits of the case.
4. CRIMINAL LAW - TRIAL AND POST-TRIAL RIGHTS QUESTIONS

Question 5 (C) The judge accepted the defendant’s guilty


plea despite his claim of innocence.
The defendant and two others were indicted
on felony charges for electronic theft of bank (D) The prosecution did not adhere to the plea
accounts. During plea negotiations, the district bargain after the defendant’s co-defendants
attorney indicated that he would also charge pleaded guilty.
the defendant with additional theft by computer
offenses if the defendant insisted on going to Question 6
trial, but that if he pleaded guilty to the one
charge and agreed to testify against his co-defen- The police suspected that a man was dealing
dants at their trial, the prosecution would recom- in illegal drugs. They gathered a sufficient
mend probation only. Because of the risk of jail amount of evidence and obtained a search
time if he was convicted of any of the additional warrant. They went to the man’s home and
offenses, the defendant agreed. conducted a thorough search. They found and
confiscated large amounts of cocaine, and then
When the defendant appeared in court, the placed the man under arrest.
judge informed him of the nature of the charge
and the potential punishment, and that the defen- At a suppression hearing, the man sought to
dant was waiving his right to trial by pleading suppress the evidence against him. The prosecu-
guilty. She asked if his plea was voluntary and tion informed the court that the warrant had
he replied that it was the best deal that he could been issued on the basis of information provided
get, but that he was really innocent of the crime by an informant. The man’s attorney requested
charged. The judge nevertheless accepted the the prosecution to produce the informant for
plea, reserving sentencing until the defendant questioning. When the prosecution refused to
testified against his co-defendants. Just before do so, the attorney made a further motion to
trial of the defendant’s co-defendants, they suppress all of the evidence against his client
pleaded guilty and provided additional informa- obtained from the search on the basis that his
tion of the defendant’s central role in the crime client was denied the right of confrontation by
and in additional computer theft offenses. At the the prosecution’s refusal to produce the infor-
defendant’s sentencing, the prosecution intro- mant.
duced the testimony of the defendant’s co-defen-
dants and asked the judge to impose a jail term. Should the attorney’s motion to suppress be
The judge sentenced the defendant to five years granted?
in prison for the crime to which he pleaded
guilty. (A) Yes, because the denial of the right of
cross-examination effectively prevents a
Which of the following arguments provides fair trial.
a valid basis for vacating the defendant’s guilty
plea and sentence? (B) Yes, because the rights of a defendant
override any right of the police to keep an
(A) The prosecution threatened to bring ad- informant anonymous.
ditional charges when the defendant indi-
cated that he was going to exercise his right (C) No, because defendants have no right to
to go to trial. know the identity of informants.

(B) Without the co-defendants’ testimony, the (D) No, because the prosecution is not required
prosecution would have been unable to offer to either name or produce the informant.
sufficient evidence of the defendant’s guilt
for a conviction.
CRIMINAL LAW - TRIAL AND POST-TRIAL RIGHTS QUESTIONS 5.

Question 7 If the second attorney files an appeal, what is


the appellate court most likely to do?
A defendant who was indigent was charged
with a crime for which the maximum punish- (A) Affirm both his conviction and the imposi-
ment is six months in prison and a fine of $500. tion of fees, because there is no reversible
At a hearing, the defendant told the judge that error, since the trial attorney competently
he wanted to plead not guilty and that he wished represented the defendant and the state has
to represent himself. The judge told the defen- a right to recoup costs from those able to
dant that the court would appoint an attorney pay.
to represent him if he needed counsel. He
further explained that the state has a policy of (B) Affirm the conviction because there was no
appointing private attorneys to defend indigents, reversible error, but reverse the imposition
and that if such defendants are acquitted or of fees, because the defendant could have
imprisoned, there is no charge for the court- gotten probation for himself as easily as the
appointed lawyer. If, however, the defendant attorney did.
is sentenced to probation, the defendant must
pay “reasonable attorneys’ fees,” which it is (C) Reverse the conviction, because the defen-
presumed the defendant will be able to pay out dant was denied the right to defend himself,
of job earnings while on probation. The defen- but affirm the imposition of fees because
dant told the judge that paying for an attorney the attorney was competent and the state
would be difficult, and that he still wished to has a right to recoup costs from those able
defend himself. The judge believed that the to pay.
defendant was competent to defend himself, but
nevertheless appointed an attorney with criminal (D) Reverse both the conviction and the imposi-
defense experience to defend him. The defendant tion of fees, because the defendant was
fully cooperated with the attorney, who did a denied the right to defend himself.
highly competent job, but the evidence heavily
favored the state. The defendant was convicted,
but the attorney’s plea for leniency was effective,
and the defendant received a suspended sentence
and probation.

Two weeks later, the defendant received a bill


for $500 for legal services, a figure that repre-
sented about half the sum a lawyer not appointed
by the court would have charged for similar
work. Although the defendant would be able to
pay the bill over a long period of time via install-
ments, he was angry that he had been billed at
all, and believed that he could have gotten proba-
tion if he had been allowed to argue his own
case. He consulted another attorney, and asked
her to appeal both his conviction and the imposi-
tion of the legal fees.
6. CRIMINAL LAW - TRIAL AND POST-TRIAL RIGHTS QUESTIONS

Question 8 Question 9

As part of a plea agreement, the defendant A husband and wife were charged with
pleaded guilty at a pretrial hearing to possession stealing credit cards and charging expensive
of stolen goods, because she knew that the police items on the misappropriated cards. An attorney
had recovered the goods and could thereby link was appointed by the court to represent the
her to the crime. Unbeknownst to the defendant, couple jointly. At the preliminary hearing, the
however, the search and seizure of the stolen judge found that the attorney would have no
property was conducted without a warrant, and conflict representing both defendants in the
no exception to the warrant requirement applied. joint trial. Halfway through the trial, however,
During the negotiation of the plea agreement, a conflict arose between the defenses of the
the defendant admitted her guilt and provided husband and wife. At the wife’s request, the
considerable factual details of the crime, but attorney moved that another attorney be
she was never provided with Miranda warnings appointed to represent the wife and that a
before making the statements. At the hearing, mistrial be declared. The trial judge moved
the defendant’s attorney failed to recognize that favorably on the attorney’s motion.
the search and seizure was unconstitutional or
request a copy of the warrant, and thus failed Another attorney was appointed to represent
to advise the defendant that the stolen property the wife, and as soon as the wife’s trial began,
could be suppressed. The defendant was her attorney moved to dismiss the case on the
convicted and sentenced in accordance with the ground that jeopardy had attached during the
plea agreement. wife’s first trial and that she was being retried in
violation of the United States Constitution.
Can the defendant move to withdraw her
guilty plea? Should the judge grant the wife’s attorney’s
motion?
(A) Yes, because no Miranda warnings were
given before the defendant provided details (A) Yes, because jeopardy attached when the
of the commission of the crime. jury began to hear evidence in the first trial.

(B) Yes, because the representation by the (B) Yes, because the judge incorrectly ruled
defendant’s attorney was ineffective. that there would be no conflict of interest
from the joint representation.
(C) No, because the defendant’s statement in
which she admits guilt would make the (C) No, because the wife requested the mistrial.
admission of the stolen property cumula-
tive. (D) No, because it is premature to move for a
dismissal based on double jeopardy until
(D) No, because a guilty plea may not be the defendant is convicted.
withdrawn after sentencing if the govern-
ment lives up to its end of the plea agree-
ment.
一 信:liuxue119118 , 们 信免 供

CRIMINAL LAW - TRIAL AND POST-TRIAL RIGHTS QUESTIONS 7.

Question 10 Question 11

A man was tried for the forcible rape of a A woman called the police to report that
woman. His defense was that the woman had she had been assaulted. She gave the police a
consented to have intercourse with him. Over detailed description of her attacker, and they
his objection, the trial judge charged the jury picked up a man who matched the descrip-
that while the prosecution must prove the overall tion who was found near the site of the alleged
case beyond a reasonable doubt, the man had the attack. The police took the man to the police
burden of proving his defense by a preponder- station and read him his Miranda warnings. The
ance of the evidence. man asked for a public defender to be appointed.
Before the public defender arrived, the woman
Were the judge’s instructions correct? came to the police station and was told there
would be a lineup as soon as the suspect’s lawyer
(A) Yes, if state law places the burden of prov- arrived. On the way to the viewing room, the
ing consent on the defendant. woman passed a holding cell where the man was
being held. She pointed at him and said loudly,
(B) Yes, as long as the judge emphasized that “That’s the man who attacked me!” The man did
the state must prove the entire case beyond not respond in any way. The woman later picked
a reasonable doubt. the man out of a lineup.
(C) No, because the instruction placed a burden At the trial, if the prosecutor wishes to intro-
on the defendant that denied the defendant duce evidence that the man said nothing when
due process of law. the woman confronted him, would such evidence
be admissible?
(D) No, because in a criminal trial the defen-
dant cannot be required to prove any (A) Yes, because it accurately describes how
critical issue by a preponderance of the the man responded.
evidence.
(B) Yes, because the man had been read his
Miranda warnings and knew that any
behavior could be used against him.

(C) No, because the man’s right against self-


incrimination would be violated if he were
required to speak.

(D) No, because counsel was not present at the


time of the incident.

一 信:liuxue119118 , 们 信免 供
8. CRIMINAL LAW - TRIAL AND POST-TRIAL RIGHTS QUESTIONS

Question 12 Question 13

Police inadvertently went to the suspect’s A husband was arrested for domestic violence
house when they were executing a search against his wife. His wife agreed to testify
warrant for drugs that had been issued for a against him. Shortly after the jury had been
different address. Despite the suspect’s protests, sworn in, but before any evidence could be
the police forcibly entered his home and discov- presented, the couple reconciled and the wife
ered a notebook lying on top of the desk in the refused to testify. Believing that the case was not
suspect’s study. The notebook listed several strong without the wife’s testimony, the prosecu-
prominent lobbyists with dates and amounts of tion dropped the charges and the husband was
money next to their names. The district attorney released. A few weeks later, the couple split
intends to ask the grand jury to indict the suspect up again and the wife filed a battery action
on charges of soliciting and receiving bribes. for damages against her husband based on the
A prominent politician who was formerly an original incident. The husband has filed a motion
associate of the suspect will testify that the asking the court to dismiss the wife’s case on
suspect is widely regarded in the political double jeopardy grounds.
community as being willing to do anything,
legal or not, if the price is right. How should the court rule?

Is a constitutional attack on an indictment (A) Deny the husband’s motion because jeopar-
against the suspect likely to succeed? dy had not yet attached during the first trial
since the prosecution had not yet presented
(A) Yes, because there will be no admissible any evidence against the defendant.
evidence before the grand jury on which to
base the indictment. (B) Deny the husband’s motion because this is
an action for damages and not a criminal
(B) Yes, because a portion of the evidence proceeding.
against the suspect was obtained in viola-
tion of constitutional guarantees against (C) Grant the husband’s motion because battery
unreasonable searches. is a lesser included offense of domestic
violence.
(C) No, because there is no constitutional right
to an indictment for a noncapital offense, (D) Grant the husband’s motion because the
and the suspect is therefore precluded from prior case was not discontinued due to a
raising any constitutional challenge to the manifest necessity.
indictment.

(D) No, because the grand jury may permissibly


consider evidence that would be barred
from a criminal trial by the exclusionary
rule.
CRIMINAL LAW - TRIAL AND POST-TRIAL RIGHTS QUESTIONS 9.

Question 14

A defendant, on trial for robbing the victim of


some jewelry, relied on the defense that he was
only trying to recover property that the alleged
victim had previously stolen from him. The trial
court instructed the jury that the prosecution
must prove guilt beyond a reasonable doubt, and
that if the jury should find that the defendant had
established by a preponderance of the evidence
that he was only trying to recover his property,
they should find him not guilty. After he was
convicted of robbery, the defendant asserts that
the instruction to the jury was error.

Is it likely that his conviction will be reversed?

(A) Yes, because the defendant need only con-


vince the jury of any defense to a reason-
able certainty, not by a preponderance of
the evidence.

(B) Yes, because the instruction put a burden on


the defendant that denied him due process
of law.

(C) No, because the defendant’s burden to show


that he was trying to recover his property
was not one of ultimate persuasion, but only
to produce evidence to rebut the legiti-
mate presumption that the robbery was
conducted with the intent to permanently
deprive the victim of the jewelry.

(D) No, because the instruction was an accurate


statement of the law.
CRIMINAL LAW - TRIAL
AND POST-TRIAL RIGHTS
ANSWERS
CRIMINAL LAW - TRIAL AND POST-TRIAL RIGHTS ANSWERS 1.

CRIMINAL LAW - TRIAL AND POST-TRIAL RIGHTS ANSWERS

Answer to Question 1

(A) The court need not overturn the conviction. A conviction will not necessarily be overturned
merely because improperly obtained evidence was admitted at trial. Rather, the harmless error test
applies, so a conviction can be upheld if the conviction would have resulted despite the admission
of improperly obtained evidence. On appeal, the government bears the burden of showing beyond
a reasonable doubt that the admission was harmless. In the instant case, there was plenty of other
evidence—a security camera recording, eyewitnesses, and physical evidence recovered from the
crime scene—that ties the defendant to the crime. Hence, the burden of proving harmless error
beyond a reasonable doubt would be met under these facts. Thus, (A) is correct, and (C) is incor-
rect. (B) is incorrect because it implies that some lesser standard applies when the judge makes
findings of fact rather than a jury. The standard remains the same—the government must show,
beyond a reasonable doubt, that the error was harmless. (D) is an overbroad and incorrect state-
ment of the law.

Answer to Question 2

(A) The defendant’s rights were violated by the homicide instructions. The Due Process Clause
requires in all criminal cases that the state prove guilt beyond a reasonable doubt. The prosecu-
tion must bear the burden of proving all of the elements of the crime charged. Thus, the Supreme
Court has held that if “malice aforethought” is an element of murder, the state may not require
the defendant to prove that he committed the homicide in the heat of passion, because heat of
passion negates malice and in effect requires the defendant to disprove the element of malice
aforethought. [Mullaney v. Wilbur (1975)] Here, the judge’s instructions create the same result
with “extreme emotional disturbance.” By instructing the jury that it may presume the existence
of malice aforethought from proof of an unlawful killing, and that the defendant bears the burden
of showing that he did not act with malice, the judge improperly imposed on this particular defen-
dant the burden to disprove an element of murder. Thus, (B) and (D) are incorrect. Insanity is an
affirmative defense for which it is constitutional to impose the burden of proof on the defendant.
Although many jurisdictions require a defendant to prove his insanity by a preponderance of the
evidence, federal courts require proof by clear and convincing evidence, and one Supreme Court
case upheld requiring a defendant to prove insanity beyond a reasonable doubt. [See Leland v.
Oregon (1952)] Thus, (B) and (C) are incorrect and (A) is correct.

Answer to Question 3

(C) The conviction should be reversed. The Due Process Clause has been interpreted as requiring the
prosecution to prove each element of the crime charged beyond a reasonable doubt. The “malice
aforethought” element of murder has traditionally been defined as encompassing the absence of
provocation engendering a passion. Putting the burden of persuasion as to the existence of provo-
cation and passion on the defendant relieves the prosecution of its burden as to their absence.
Therefore, (C) is the best answer and (A) is incorrect on the facts. (B) is incorrect because the
presumption of “malice aforethought” is not a legitimate presumption. (D) is incorrect as a matter
of law. Presumptions are permitted as long as they are not mandatory for the jury.

Answer to Question 4

(D) The state’s best argument is that the defendant procured dismissal of the original trial on a techni-
cality. The Fifth Amendment right to be free of double jeopardy for the same offense generally
2. CRIMINAL LAW - TRIAL AND POST-TRIAL RIGHTS ANSWERS

requires that, once jeopardy attaches in the first prosecution, the defendant may not be retried
for the same offense. Under certain circumstances, however, a defendant can be retried even if
jeopardy has attached. One such circumstance is that a trial may be discontinued and the defen-
dant reprosecuted for the same offense when the termination occurs at the behest of the defendant
on any grounds not constituting an acquittal on the merits. Double jeopardy does not bar two
trials, but only a retrial after a determination on the merits. Here, the municipal charge against
the defendant was dismissed, and the trial terminated, at the request of the defendant on the basis
of a technical error in the information. This was not an acquittal on the merits. Thus, there is no
violation of the prohibition of double jeopardy presented by the current state prosecution of the
defendant. Regarding (A), it is true that the constitutional prohibition against double jeopardy does
not apply to trials by separate sovereigns. A person may be tried for the same conduct by both a
state and the federal government, or by two states. However, a state and its municipalities are not
deemed to be separate sovereigns. Thus, the defendant may not be tried for the same conduct by
the city and the state. (B) is incorrect. The fact that the state charge requires proof of a fact not
required by the municipal charge does not satisfy the test for determining that the “same offense”
is not involved for purposes of double jeopardy. At best, it indicates only that the municipal charge
is a lesser included offense of the state charge. A lesser included offense is one that consists
entirely of some, but not all, elements of a greater crime. Attachment of jeopardy for a lesser
included offense usually bars retrial for the greater offense. Here, the municipal charge consists
entirely of all the elements of the state charge, except for the element of intent to distribute the
illegal narcotics. Therefore, the municipal charge is a lesser included offense of the state charge.
This fact would, if anything, provide an argument in support of the position that the trial in state
court constitutes double jeopardy. Thus, (B) does not present an argument against granting the
motion filed on behalf of the defendant. (C) is incorrect because it misstates the point at which
jeopardy attaches. Jeopardy attaches in a jury trial at the impaneling and swearing of the jury, and
in a bench trial when the first witness is sworn. This question deals with a jury trial, and we are
told that the jury has been sworn, selected, and impaneled. Thus, jeopardy has attached, despite
the fact that no witness has been sworn.

Answer to Question 5

(D) The prosecution’s failure to adhere to the plea bargain offers the defendant the only basis for
vacating his guilty plea and sentence. In accordance with the contractual view of plea bargains,
a defendant who agrees to a plea bargain has the right to have that bargain kept. If the prosecu-
tion does not keep the bargain, the court will decide whether the circumstances require specific
performance of the plea agreement or whether the defendant should be granted an opportunity
to withdraw his guilty plea. Here, the defendant agreed to testify against his co-defendants in
exchange for the recommendation of probation. His co-defendants’ decision to plead guilty rather
than go to trial may have been influenced by the fact that the defendant had agreed to testify;
hence, the fact that he did not testify at their trial does not allow the prosecution to avoid the
terms of the plea bargain. (A) is wrong because the fact that the prosecution threatened to bring
additional charges is not a basis for claiming that a guilty plea was involuntary as long as there
was some legal basis for bringing the additional charges. Consistent with the contract theory of
plea negotiation, the prosecution has the power to drive a hard bargain in reaching a plea agree-
ment. (B) is wrong because there is no general requirement that the record contain evidence of the
defendant’s guilt or other factual basis for the plea. Whether the prosecution could have proven its
case against the defendant without the plea is not at issue. (C) is wrong because the defendant’s
claim of innocence does not prevent the court from accepting his guilty plea. Before accepting a
guilty plea, the court must find that the plea was voluntary and intelligent. However, admission of
CRIMINAL LAW - TRIAL AND POST-TRIAL RIGHTS ANSWERS 3.

guilt is not a constitutional requisite to imposition of a criminal penalty. When a defendant pleads
guilty despite protesting his innocence, the plea may still be seen as an intelligent choice by the
defendant, and withdrawal of the plea will not be permitted when there is other strong evidence
of guilt in the record. Here, the defendant weighed the risks and benefits before pleading guilty,
and the statements by his co-defendants provide other evidence of his guilt; thus, his claim of
innocence is not a basis for attacking his guilty plea.

Answer to Question 6

(D) The man’s motion should be denied. The United States Supreme Court has held that when the
only issue is that of probable cause for issuance of a warrant, the name of the informer is a type
of privileged information. (Only if the informer were a material witness to the crime might his
identity have to be revealed at or before trial.) The validity of a warrant based on information
obtained from informers is based on the totality of the circumstances. (A) is incorrect because
although the right of cross-examination of one’s accusers has been deemed to be a fundamental
right, it does not apply when the issue is only that of probable cause for issuance of a warrant,
as compared to guilt or innocence. (B) is incorrect because, as noted above, unless the informer
were also a material witness to the crime, his identity need not be revealed. Here, the facts do
not indicate that the informer was a material witness to the crime. (C) is incorrect because it is
overbroad. As indicated above, there is a right to know the identity of an informant if the infor-
mant is also a material witness to the crime.

Answer to Question 7

(D) The appellate court would likely reverse both the conviction and the imposition of fees. Although
a waiver of the right to counsel will be carefully scrutinized to ensure that the defendant has
a rational and factual understanding of the proceeding against him, a defendant has a right to
waive counsel and represent himself as long as the waiver is knowing and voluntary and he is
competent to proceed pro se. Where the state provides counsel in cases of indigence, it may then
seek reimbursement from a convicted defendant who subsequently becomes able to pay. Here,
the judge thought that the defendant was competent to represent himself. Therefore, his waiver
of counsel should have been honored. Violation of the defendant’s right to represent himself
will result in a reversal of his conviction. Although, as stated above, the state may recoup costs
of appointed counsel from indigents who become able to pay, the state cannot recover from the
defendant because the attorney was appointed against the defendant’s will and in violation of his
right to represent himself. Had effect been given to the defendant’s right to represent himself,
there would have been no attorneys’ fees to be assessed against him. (A) is incorrect because the
denial of the defendant’s right to represent himself does constitute reversible error, regardless of
the competence with which the attorney represented him. Also, as explained above, the state’s
right to recoup costs from indigents who become able to pay does not extend to those defendants
who have had appointed counsel imposed on them against their will. (B) is incorrect because
it states that the conviction involved no reversible error, when denial of the defendant’s right to
represent himself is reversible error. (B) also incorrectly states that the imposition of fees should
be reversed because the defendant could have gotten probation for himself as easily as his attorney
did. There is no way to know that the defendant could have gotten probation for himself as easily
as the attorney did, and it is not really at issue whether the defendant could have done so. What
is at issue is that the costs were imposed as part of an unconstitutional forced representation
by appointed counsel. Thus, even if the attorney obtained probation for the defendant when the
defendant would have been unable to do so, the costs should be reversed, because the defendant
should have been free to proceed pro se, even if this meant risking imprisonment. Similarly, (C) is
4. CRIMINAL LAW - TRIAL AND POST-TRIAL RIGHTS ANSWERS

incorrect because it states that the attorney’s competence is a reason for affirming the imposition
of fees. As stated above, the defendant had the right to proceed pro se if his decision was knowing
and voluntary and he was competent to proceed pro se, regardless of the fact that appointed
counsel may have been highly competent. The state cannot force a defendant to forgo a constitu-
tionally protected right by accepting appointed counsel, and then further force him to pay for such
counsel.

Answer to Question 8

(B) The defendant can move to withdraw her plea. Ordinarily it is difficult to collaterally attack a
guilty plea after sentencing when the plea is seen as an intelligent choice among the defendant’s
alternatives. Notwithstanding, a plea may be attacked after sentencing in certain circumstances;
e.g., if there was ineffective assistance of counsel. Ineffective assistance of counsel undercuts the
assumption of an intelligent choice among the defendant’s alternatives on the advice of counsel.
Therefore, a defendant may successfully attack a guilty plea on the ground that she received
ineffective assistance of counsel if, but for the counsel’s errors, the defendant probably would
not have pleaded guilty and instead would have insisted upon going to trial. Here, the facts
indicate that the defendant pleaded guilty because she believed that the government lawfully
seized evidence that would implicate her in the crime. Had the defendant’s counsel requested a
copy of the search warrant, as most defense lawyers would have done, the defendant would have
discovered that the evidence might be suppressed, and thus would not as readily have pleaded
guilty. Hence, (B) is correct. Answer (A) is incorrect because any admissions made during plea
negotiations are inadmissible at trial. The fact that the defendant was not Mirandized before
such statements would not be a basis for overturning the plea agreement. Answer (C) is incorrect
because a defendant’s admissions during plea negotiations also would not save a plea agreement
for the government. Admissions during plea agreements would not cure a defective plea agree-
ment. Answer (D) is incorrect as an overbroad statement. A plea agreement may be attacked after
sentencing in certain circumstances.

Answer to Question 9

(C) Although jeopardy attached in the wife’s first trial, her retrial is not barred because she initiated
the grant of the mistrial in her first trial. As a general rule, the right to be free of double jeopardy
for the same offense bars a retrial for the same offense once jeopardy has attached in the first
trial. However, one of the exceptions permitting retrial even if jeopardy has attached is when a
mistrial is granted in the first trial at the request of the defendant on any ground not constituting
an acquittal on the merits. Here, the wife requested the mistrial because a conflict arose between
the defenses of her and her co-defendant in the joint trial, and the judge granted the mistrial
solely to allow the wife to obtain another attorney. Thus, no acquittal on the merits occurred
and the double jeopardy rule does not apply. (A) is incorrect for several reasons. Merely because
jeopardy attaches does not mean that the double jeopardy rule will apply; retrial will be permitted
under certain exceptions, one of which is applicable here. Furthermore, (A) is not a correct state-
ment of law. Jeopardy attaches in a jury trial when the jury is impaneled and sworn in, even if
it has not yet heard any evidence. (B) is incorrect because the judge’s finding at the preliminary
hearing stage appears to be an honest error rather than bad faith conduct. In the absence of bad
faith conduct by the judge or prosecutor designed to force the defendant to seek a mistrial, the
defendant’s securing of a mistrial does not preclude a retrial. (D) is incorrect because the right to
be free of double jeopardy creates a bar as soon as the defendant is retried for the same offense,
rather than on her conviction.
CRIMINAL LAW - TRIAL AND POST-TRIAL RIGHTS ANSWERS 5.

Answer to Question 10

(C) The judge’s instructions resulted in a violation of defendant’s due process rights. The Due Process
Clause requires that in all criminal cases the state prove guilt beyond a reasonable doubt. This
requirement means that each element of a crime must be proved beyond a reasonable doubt. The
man is charged with rape, which is the unlawful carnal knowledge of a woman by a man not her
husband, without her effective consent. Thus, lack of consent is an element of the offense, and the
state must prove lack of consent beyond a reasonable doubt. To require the man to prove consent
would violate the due process requirement that the state prove all elements of an offense beyond
a reasonable doubt. (B) is incorrect because lack of consent is part of the entire case that the state
must prove where the charge is rape. To say that the state must prove “the entire case” is really to
say that the state must prove each element of the offense. It is impermissible for a judge to segre-
gate one element of the offense from other elements, and to impose on the defendant the burden
of proving that element. (A) is incorrect because if state law were to place the burden of proving
consent on the defendant, such a law would violate the Due Process Clause and would therefore
be invalid. (D) is incorrect because a state may impose on a criminal defendant the burden of
proof regarding certain issues; e.g., an affirmative defense such as insanity or self-defense.

Answer to Question 11

(C) Introduction of the proffered evidence would in effect penalize the man for exercising his right
to be free from compulsory self-incrimination. Miranda warnings are given to safeguard the
Fifth Amendment right to be free from compelled self-incrimination. Prior to interrogation, a
person in custody must be clearly informed that he has the right to remain silent and anything he
says can be used against him in court. These warnings implicitly assure that silence will carry
no penalty. Thus, a prosecutor may not comment on a defendant’s silence after the defendant is
arrested and has received the Miranda warnings. To allow the prosecutor to introduce evidence
of the man’s silence in the face of the woman’s accusation would run counter to the very purpose
of the Miranda warnings, which is to allow the defendant to remain silent without fear of being
prejudiced by such silence. The man was no more required to respond to the woman’s accusa-
tion than he would have been to an accusation or question coming from the police. The man’s
privilege against compelled self-incrimination would be meaningless if he were required to either
respond to the woman or have his failure to respond introduced against him. (A) is wrong because
the mere fact that evidence is true does not render such evidence admissible. Evidence that runs
afoul of some rule of evidentiary exclusion (e.g., a statement that is hearsay not subject to any of
the exclusions to the hearsay rule) or that would violate a right of constitutional magnitude is not
admissible, regardless of whether it is true. Because evidence of the man’s silence would violate
his Fifth Amendment right to be free of compelled self-incrimination, this evidence is not admis-
sible, even though it accurately describes how the man responded. (B) is wrong because it turns
the Miranda warnings on their head. The warnings are to apprise the suspect that he has a right
to remain silent, and that if he chooses to say something, this statement can be used against him.
The warnings do not, as (B) suggests, apprise the suspect that he must respond to questions or
accusations, and that a failure to respond can be used against him. As has been explained above,
the crux of the Miranda warnings is the right to remain silent, and to be free of coercion to
speak at all. (D) is wrong for two reasons. (D) implies that the evidence would be admissible if
the man’s lawyer were present at the time of the incident. Regardless of the presence of counsel,
admission of evidence as to the man’s silence in the face of the woman’s accusation would violate
the man’s privilege against compelled self-incrimination. Also, events had not yet reached a stage
at which the man was entitled to counsel. At the time of this incident, the man was waiting to
take part in a lineup. The facts indicate that the man had not yet been charged with a crime. The
6. CRIMINAL LAW - TRIAL AND POST-TRIAL RIGHTS ANSWERS

right to counsel applies to post-charge lineups, but not to pre-charge lineups. Note also that the
woman’s accusation did not even occur in a lineup setting, but during an inadvertent viewing of
the man. Thus, the right to counsel had not yet attached.

Answer to Question 12

(D) A constitutional attack on the indictment will probably fail. There is no constitutional principle
that precludes grand juries from considering evidence obtained in violation of constitutional law;
the exclusionary rule is inapplicable to grand juries. Thus, (D) is correct, and (B) is incorrect. (A)
is incorrect because there will be admissible evidence. Even if the evidence is obtained in viola-
tion of the Constitution, the grand jury may consider the evidence. (C) is not the best answer,
because the lack of a right to an indictment for a noncapital offense does not mean that the defen-
dant cannot attack material constitutional irregularities. In fact, even the right to indictment by
a grand jury for capital crimes is not binding on the states, but that does not preclude defendants
from raising constitutional challenges to the proceedings against them.

Answer to Question 13

(B) The court should deny the motion because jeopardy generally does not attach in civil proceed-
ings, other than juvenile proceedings. Here, the wife is bringing a civil battery action for damages
against her husband. The prior trial was based on a criminal charge of domestic violence. Thus,
double jeopardy is inapplicable under these facts. Thus, even though simple battery may be a
lesser included offense of domestic violence, it would not prevent the wife from bringing a civil
action against the husband. Furthermore, even if the state were not allowed to retry the husband
on the domestic violence charge under the “manifest necessity” concept, that would not prevent
the wife from bringing a civil action against her husband. Thus, (C) and (D) are wrong. (A) is also
wrong because jeopardy attaches in a jury trial at the empanelling and swearing in of the jury.

Answer to Question 14

(B) The defendant’s conviction should be reversed. Robbery requires an intent to permanently deprive
the victim of her property. An intent to recover property that the defendant believes is his would
not be a sufficient intent. The prosecution must prove every element of the crime beyond a reason-
able doubt, and putting the burden of persuasion to show an innocent intent on the defendant
would deprive him of due process of law, since it would relieve the prosecution of its burden to
show the required intent for robbery. Thus, the instruction was improper and (D) is wrong. (A)
and (C) are wrong. The defendant does not carry any burden of proof with respect to an element
of the crime. The burden is on the prosecution to prove each element beyond a reasonable doubt.
EVIDENCE -
CHARACTER EVIDENCE
QUESTIONS
EVIDENCE - CHARACTER EVIDENCE QUESTIONS 1.

EVIDENCE - CHARACTER EVIDENCE QUESTIONS

Question 1 Question 2

A resident being interviewed live by a televi- In an accountant’s trial for filing fraudu-
sion reporter stated that, “The biggest problem in lent tax reports, the prosecution calls a former
this city is corruption in city government, partic- colleague of the accountant, and she testifies that
ularly the mayor.” The mayor has now brought the accountant’s reputation in the community is
an action for defamation against the resident. for frequently participating in very questionable
At trial, the mayor has produced testimony as reporting, often resulting in unnecessary risk for
to his honesty and good character. As part of his clients. She testifies further that she thinks
his defense, the defendant seeks to offer into the accountant is dishonest.
evidence the fact that the mayor was convicted
two years ago of taking a bribe to award a city Should the trial court admit this evidence over
contract for solid waste disposal. the accountant’s objection?

Is the evidence admissible? E A


(A) No, because the prosecution cannot initiate
evidence of the defendant’s character.
(A) No, because character evidence is not ad-
missible in civil cases. (B) No, because use of the colleague’s opinion
is improper.
(B) No, because character can be proved only
by opinion or reputation testimony. (C) Yes, because it is evidence of the defen-
dant’s character for dishonesty.
(C) Yes, because the mayor’s character is
directly in issue. (D) Yes, because it is evidence of habit.

(D) Yes, because there was an actual conviction


for the crime.
2. EVIDENCE - CHARACTER EVIDENCE QUESTIONS

Question 3 Question 4

A tourist was struck and killed at a street The plaintiff sued the defendant for bodily
crossing by an automobile. The accident was injuries suffered by the plaintiff as a result of
witnessed by a bystander and by the tourist’s a collision between the cars they were driving.
wife. The wife brought an action against the The accident occurred on a rainy day, when the
automobile driver for wrongful death, alleging defendant’s car skidded across the center line
excessive speed, failure to observe traffic signals, and struck the plaintiff’s car. A principal issue
and defective brakes. A witness for the driver is whether the defendant was driving too fast for
was called to testify that the driver enjoyed a the wet conditions. At trial, the plaintiff calls
reputation for being a safe and prudent driver a witness who is prepared to testify that he has
based on his personal knowledge of the driver’s lived next door to the defendant for 15 years,
driving habits. and that the defendant is notorious in the neigh-
borhood for driving his car at excessive rates of
How should the trial judge rule on the speed. The defendant’s attorney objects.
witness’s testimony?
C
(A) Admissible, because, where there are no
May the witness’s testimony be allowed?
C
unbiased eyewitnesses to an accident, repu- (A) Yes, because the defendant’s character as a
tation as a safe driver may be used to prove careless driver is in issue.
the driver acted in conformity with that
reputation at the time in question. (B) Yes, because character may be proven by
reputation evidence.
(B) Admissible, because the witness first testi-
fied that he has personal knowledge of the (C) No, because the testimony is improper
driver’s driving habits. character evidence.

(C) Inadmissible, because evidence of a reputa- (D) No, because the testimony is hearsay, not
tion as a safe and prudent driver cannot within any exception.
be used to prove that the driver acted in
conformity with that reputation at the time
in question.

(D) Inadmissible, because, in a civil case,


character evidence may be used only after
the plaintiff has attacked the character of
the defendant.
EVIDENCE - CHARACTER EVIDENCE QUESTIONS 3.

Question 5 Question 6

A businessperson filed a defamation suit A defendant is charged with the arson of a


against a newspaper for printing a column that house. At trial, the prosecution offers evidence
referred to him as “a nasty miser” and “worse that when the defendant was arrested, shortly
than Ebenezer Scrooge,” and accused him of after the crime had been committed, she had a
“never performing a real act of charity in his large amount of cocaine hidden in the trunk of
life.” During the presentation of the business- her car.

AD
person’s case, he wanted to put an agent of the
Internal Revenue Service on the stand. The agent This evidence should be:
is prepared to testify that the businessperson,
on his own initiative, reimbursed the IRS for an (A) Admitted to prove the defendant’s propen-
erroneous overpayment of a tax refund. Counsel sity to commit crimes.
representing the newspaper objects.
(B) Admitted to prove the defendant’s general
How is the court likely to rule on the agent’s bad character.
testimony?

BC
(A) Admissible, because the businessperson’s
character is at issue in the case.
(C) Excluded because such evidence may be
offered only to rebut evidence of good
character offered by the defendant.

(B) Admissible, because the businessperson has (D) Excluded because its probative value is
a right to defend his good character. substantially outweighed by the danger of
unfair prejudice.
(C) Inadmissible, because the agent’s testimony,
in and of itself, is not probative of any
material issue in the case.

(D) Inadmissible, because specific instances


of conduct are not admissible to prove
character.
4. EVIDENCE - CHARACTER EVIDENCE QUESTIONS

Question 7 Question 8

At the defendant’s trial for arson, a witness for During a defendant’s trial for murder, the state
the defense is asked whether the defendant has a introduced circumstantial evidence tending to
reputation for honesty and veracity. The prosecu- establish her guilt, but could not provide direct
tion objects. evidence that she had committed the murder. No

B
one had seen the killer going into or coming out
How should the court rule on the objection? of the victim’s office around 2:30 p.m., the time
of the victim’s death by strangulation. During
(A) Sustained, because the evidence is hearsay. her defense case, the defendant took the stand
and testified that she could not have committed
(B) Sustained, because the evidence is irrel- the charged murder, because on the day of the
evant. victim’s death she had been on a business trip
3,000 miles away. The prosecution then called a
(C) Overruled, because the prosecution put rebuttal witness. The witness will testify that, at
the defendant’s character at issue by filing 1 p.m. on the day of the charged murder, he had
charges against him. put his hand on the defendant’s knee while both
sat at a bar across town. The defendant had then
(D) Overruled, because the reputation evidence coquettishly removed his necktie and attempted
is admissible to establish a character trait. to strangle him with it until bystanders pulled
her away and she left the bar. The defendant
objects to admission of the witness’s testimony.

How should the trial judge rule?

(A) For the defendant, because she did not


place her character at issue in the trial. C
(B) For the defendant, because the testimony
is character evidence not directed at her
character for truth and veracity.

(C) For the state, because the testimony estab-


lished that the defendant had the opportu-
nity to commit the charged crime.

(D) For the state, because the testimony estab-


lished that the defendant has a character for
violence.
EVIDENCE - CHARACTER EVIDENCE QUESTIONS 5.

Question 9 Question 10

A plaintiff brought an action against a defen- A defendant is charged with theft of a car. On
dant for property damages, alleging that the cross-examination of the defendant, the prosecu-
defendant’s car nicked the side of the plaintiff’s tion asked her whether she had been convicted of
truck while the defendant was changing lanes on fraudulent business practices six months earlier.
an expressway. At trial, the defendant sought to
introduce evidence of her good driving record. Is this question proper?

B
Is the evidence admissible? (A) Yes, to show that the defendant has a bad

A
(A) No, because it is improper character evi-
dence.
character.

(B) Yes, to show that the defendant is inclined


to lie.
(B) No, because it is self-serving.
(C) No, because the probative value of the
(C) Yes, because it is proper character evidence. evidence is outweighed by the danger of
unfair prejudice.
(D) Yes, because it is habit evidence.
(D) No, because the conviction has insufficient
similarity to the crime charged.
6. EVIDENCE - CHARACTER EVIDENCE QUESTIONS

Question 11 Question 12

A defendant is on trial for aggravated assault. The defendant is on trial for embezzlement.
The defendant claims that he was merely a He does not take the stand.
bystander and that another person was the
actual aggressor. The defendant calls a witness Which of the defendant’s previous convic-
to testify that the defendant has a reputation for tions is most likely to be admitted into evidence
being a “good and loving person who would against him?

13
never hurt anyone.”
(A) A seven-year-old conviction for arson, a
Should the trial judge rule that the testimony felony.
is admissible?
(B) A 12-year-old conviction for embezzlement,

B
(A) Yes, because it is not hearsay. a felony.

(B) Yes, because this is a criminal matter, and (C) A six-month-old conviction for disorderly
it tends to show the defendant’s relevant conduct, a misdemeanor.
good character.
(D) A two-year-old conviction for felonious
(C) No, because the prosecution has not intro- sexual assault.
duced evidence concerning the defendant’s
reputation.

(D) No, because character evidence is not


permissible to show that an accused has
acted in conformity with the character.
EVIDENCE - CHARACTER EVIDENCE QUESTIONS 7.

Question 13 Question 14

The defendant was on trial for possession of Cars driven by the plaintiff and the defendant
marijuana with intent to deliver. The defendant collided, and the defendant was charged with
claimed that she had been set up and testified driving while intoxicated in connection with
that she was not the owner of the marijuana. In the accident. She pleaded guilty and was merely
support of her defense, defense counsel wishes fined, although, under the statute, the court could
to present testimony from the defendant’s long- have sentenced her to two years in prison.
time neighbor that the defendant has a reputa-
tion for being a law-abiding citizen, and that, in Thereafter, the plaintiff, alleging that the
his opinion, the defendant would never become defendant’s intoxication had caused the collision,
involved with drugs. sued the defendant for damages.

G
The prosecutor objects to the testimony in its
entirety. How should the court proceed?
I
At trial, the plaintiff offers the properly
authenticated record of the defendant’s convic-
tion.
(A) Admit the testimony regarding the defen-
Should the record be admitted?
B
dant’s reputation for being law-abiding,
but exclude the testimony regarding the
neighbor’s opinion that she would not be (A) Yes, as proof of the defendant’s character.
involved with drugs.
(B) Yes, as proof of the defendant’s intoxica-
(B) Exclude the testimony regarding the defen- tion.
dant’s reputation for being law-abiding, but
admit the testimony regarding the neigh- (C) No, because the conviction was not the
bor’s opinion that she would not be involved result of a trial.
with drugs.
(D) No, because it is hearsay not within any
(C) The testimony should be admitted in its exception.
entirety.

(D) The testimony should be excluded in its


entirety.
EVIDENCE -
CHARACTER EVIDENCE
ANSWERS
EVIDENCE - CHARACTER EVIDENCE ANSWERS 1.

EVIDENCE - CHARACTER EVIDENCE ANSWERS

Answer to Question 1

(C) The evidence is admissible because the mayor’s character is directly in issue. The general rule is
that evidence of character to prove the conduct of a person in the litigated event is not admissible
in a civil case. However, when a person’s character itself is one of the issues in the case, character
evidence is not only admissible, but in fact is the best method of proving the issue. Where the
plaintiff brings a defamation action for injury to reputation and the defendant pleads as an affir-
mative defense that his statements were true, the plaintiff’s character is directly at issue in the
case. Under the Federal Rules, any of the types of evidence (reputation, opinion, or specific acts)
may be used to prove character when character is directly in issue. [Fed. R. Evid. 405(b)] Here,
the mayor’s character is at issue and the resident is offering character evidence to show that his
assertion that the mayor is corrupt is a true statement. Thus, (C) is correct and (A) is incorrect. (B)
is incorrect because, as stated above, any of the types of evidence can be used to prove character
when it is directly in issue. (D) is incorrect because an actual conviction is required for impeach-
ment purposes, but not for the purpose of establishing character—evidence of an arrest or indict-
ment would have been equally admissible.

Answer to Question 2

(A) The court should not admit this evidence because the prosecution cannot initiate evidence of the
defendant’s bad character. The prosecution may offer such evidence only after the accused has put
his character in issue by either taking the stand (thus placing his credibility in issue) or offering
evidence of his good character. Thus, (C) is incorrect. (B) is incorrect because, under the Federal
Rules, character may be proven by opinion evidence. (D) is incorrect because this does not consti-
tute a regular response to a specific set of circumstances; it is merely reputation and opinion
evidence.

Answer to Question 3

(C) The judge should rule the witness’s testimony inadmissible. This question involves the admis-
sibility of propensity evidence. The driver’s witness is prepared to testify that the driver has a
reputation for being a safe and prudent driver, which the driver plans to use to prove that, because
he has a propensity for driving safely, he was in fact driving safely at the time of the accident.
This is a classic case of the impermissible use of propensity evidence. The general rule is that
evidence of character traits (here, safety and prudence) is inadmissible in a civil case to prove
that a party acted in conformity with those traits on a particular occasion. [Fed. R. Evid. 404(a)]
This case fits squarely within that general rule. Thus, (C) is correct. All the other answers are
wrong because they each assume that propensity evidence is admissible under some circum-
stances in a wrongful death action. (A) indicates that such evidence is admissible if there are no
unbiased eyewitnesses to the accident. (B) indicates that such evidence is admissible because
the witness has personal knowledge of the character traits (the driver’s propensity for safety and
prudence) that are the subject of his testimony. (D) indicates that such evidence is admissible
if the plaintiff has attacked the character of the defendant. All of these answers miss the basic
point—that propensity evidence is inadmissible in civil cases. Furthermore, (A) is wrong because
it presumes that there are no unbiased witnesses to the accident, but the question makes clear
that there was an unbiased witness: the bystander. (B) is also wrong because it indicates that
the witness must first testify that he has personal knowledge of the driver’s driving habits before
testifying about the driver’s reputation for safety and prudence. This answer confuses reputation
2. EVIDENCE - CHARACTER EVIDENCE ANSWERS

testimony with opinion testimony. The witness’s testimony pertained to the driver’s reputation. To
be a competent witness, the witness would have to have personal knowledge of the driver’s reputa-
tion. He would not have to have personal knowledge of the driver’s actual driving habits. Although
perhaps not as nonsensical as (A) and (B), (D) is just as inaccurate, as it bears no similarity to
anything in the rules of evidence pertaining to propensity evidence.

Answer to Question 4

(C) The testimony of the witness is inadmissible character evidence; i.e., it describes the defendant’s
general behavior patterns. Evidence of character to prove the conduct of a person in the litigated
event is generally not admissible in a civil case. Here, the plaintiff is trying to employ the circum-
stantial use of prior behavior patterns to draw the inference that the defendant drove at an exces-
sive rate of speed at the time of the incident here at issue. Such a use of character evidence is not
permitted. (A) is incorrect because character is not in issue. An exception to the general prohibi-
tion of character evidence is that, when proof of a person’s character, as a matter of substantive
law, is an essential element of a claim or defense in the case, character evidence is admissible. The
defendant’s character as a driver is not in issue; rather, his actions at a specific time and place are
in issue. Thus, this exception does not apply to these facts. Regarding (B), it is true that character
may be established by reputation evidence under the Federal Rules. However, as explained above,
this is not a proper case for the use of character evidence. Therefore, (B) is incorrect. (D) is also
incorrect. While it is true that testimony as to a person’s reputation in the community may in some
sense be considered hearsay (i.e., such testimony reflects what people are saying about a person),
reputation testimony is a permissible (and in fact, the most common) means of showing character.
Thus, (D) does not present a basis for refusing to allow the testimony.

Answer to Question 5

(C) The agent’s testimony is inadmissible because it is not probative of any material issue in the case.
Relevant evidence tends to make the existence of any fact that is of consequence to the determina-
tion of an action more probable than it would be without the evidence. [Fed. R. Evid. 401] While
evidence tending to prove the businessperson’s charitable nature, which is a material issue in
this case, would be relevant, the evidence here tends to prove only the businessperson’s honesty,
which is not at issue here. Therefore, it is not relevant and should not be admitted. (A) is incorrect
because even though the businessperson’s character has been called into question in this case, only
evidence that is probative of the particular character trait in issue may be admitted. When proof
of a person’s character, as a matter of substantive law, is an essential element of a claim or defense
in a civil action, it is said that character is “directly in issue.” In such a case, evidence of specific
acts may be used to prove character. [Fed. R. Evid. 405(b)] Because this is a defamation case, the
businesspersons’s character as to generosity is directly in issue, and specific acts may be used to
prove his generosity. However, as discussed above, the agent’s testimony is not probative of the
businessperson’s generosity. (B) is similarly incorrect. The businessperson has a right to prove his
good character, but only with regard to the particular character trait that has been defamed. (D) is
incorrect because specific acts may be used to prove character when character is directly in issue,
as discussed above. The businessperson’s generosity is directly in issue in this case and specific
acts of his may be used to prove his generosity. Therefore, the agent’s testimony is not inadmis-
sible on these grounds.

Answer to Question 6

(D) In a criminal case, other crimes and wrongs of the defendant may be admissible even though
they are not charged, but they are not automatically admissible. There are two basic principles:
EVIDENCE - CHARACTER EVIDENCE ANSWERS 3.

other crimes or wrongs are not admissible to show that the defendant is a bad person, nor are they
admissible to show propensity of the defendant to commit this crime. Other crimes or wrongs may
be admissible if they are relevant to show motive, intent, absence of mistake, identity or common
scheme or plan, unless the judge determines that the probative value is substantially outweighed
by prejudice. (A) and (B) are therefore wrong. (C) is wrong because extrinsic evidence of specific
crimes is not admissible to rebut evidence of good character. While (D) calls for a determination
on the part of the trial judge and could be debated, it is the best answer because (A), (B), and (C)
are incorrect statements of the law.

Answer to Question 7

(B) The objection should be sustained because the testimony of the witness is irrelevant. Generally,
a defendant may introduce evidence of his good character to show his innocence of the alleged
crime. However, the evidence must be relevant; i.e., it must have some tendency to make the
existence of any fact that is of consequence to the determination of an action more probable or
less probable than it would be without the evidence. Here, the defendant is charged with arson and
there is nothing to indicate that the defendant’s honesty is at issue. Therefore, the witness’s testi-
mony is inadmissible because it is irrelevant. (A) is incorrect. Hearsay is an out-of-court statement
offered to prove the truth of the matter asserted. However, reputation of a person’s character in the
community is an exception to the hearsay rule. Therefore, the witness’s statements regarding the
defendant’s reputation fall under that exception to the hearsay rule. (C) is incorrect because only
the defendant can put his character at issue; the prosecution cannot initiate evidence of the defen-
dant’s character. Furthermore, filing criminal charges against a person does not put his character
at issue. (D) is incorrect because reputation evidence is admissible to establish a character trait
only when the trait sought to be established is pertinent to the crime charged. Here, the defen-
dant’s reputation for honesty and veracity is not pertinent to the crime of arson.

Answer to Question 8

(C) The judge should rule for the state. This question is not about propensity evidence. Rather, it is
about evidence of prior bad acts to help prove something other than a person’s propensity to act
in a manner consistent with a character trait. Under Federal Rule 404(b), it is permissible to use
evidence of a criminal defendant’s prior bad acts to establish a motive for committing the crime,
to help to identify him as the perpetrator of a crime, or, as here, to show opportunity to commit
the crime. The defendant claims that she could not have committed the murder because she
was on a business trip the day it occurred. The witness’s testimony, however, pertaining to his
encounter with the defendant in a crosstown bar on the day of the murder, is clearly relevant to
show that the defendant had the opportunity to commit the murder. For this reason, (C) is correct.
All the other answers are wrong because they mistakenly assume that the witness’s testimony
involves propensity evidence rather than prior bad acts evidence. (A) correctly states that the
defendant did not place her character at issue by testifying that she was across the country when
the murder occurred. That would be a reason for excluding the witness’s testimony if his testi-
mony amounted to an attack on the defendant’s character (i.e., propensity evidence). However, his
testimony is to show opportunity; thus, it may be admitted. (B) correctly implies that propensity
evidence can be used to impeach the testimony of a witness only if it addresses the witness’s
character for truth and veracity. The defendant was a witness in the trial. If the purpose of the
witness’s testimony had been to impeach her testimony by attacking her character for something
other than truth or veracity, the testimony would be inadmissible. Because this evidence was
admitted not to impeach but to show opportunity, (B) is incorrect. (D) correctly implies that
the prosecution cannot introduce propensity evidence establishing a defendant’s bad character
4. EVIDENCE - CHARACTER EVIDENCE ANSWERS

until the defendant has placed her character in issue. The defendant’s testimony did not place
her character in issue. Thus, if the witness’s testimony amounted to an attack on the defendant’s
character, the testimony would be inadmissible. Because the testimony was not an attack on the
defendant’s character, however, (D) is incorrect.

Answer to Question 9

(A) The driving record is inadmissible because it is being offered as character evidence. In a civil
case, evidence of character to prove the conduct of a person in the litigated event is generally not
admissible. The slight probative value of character is outweighed by the dangers of prejudice and
distracting the jury from the main issues. Therefore, circumstantial use of prior behavior patterns
for the purpose of drawing the inference that a person has a particular character trait and that, at
the time and place in question, she probably acted in conformity with it is not permitted. Evidence
of the defendant’s good driving record is being offered to show that she is a careful driver and to
raise the inference that, when the accident occurred, she was acting in conformity with that trait.
This constitutes impermissible use of character evidence and is inadmissible. (B) is incorrect
because evidence is not excludable because it is self-serving. Virtually all evidence is self-serving
to the party offering it. (C) is incorrect because it is based on the mistaken assumption that
character evidence is admissible. As stated above, character evidence is generally inadmissible
in a civil case. It is admissible in a civil case only when proof of a person’s character, as a matter
of substantive law, is an essential element of a claim or defense (e.g., in a defamation case). The
defendant’s character is not in issue, so the driving record is inadmissible. (D) is incorrect because
this is not habit evidence. Habit describes one’s regular response to a specific set of circumstances.
Character describes one’s disposition with respect to general traits. The defendant’s good driving
record describes a general behavior pattern of careful driving, rather than a regular response to a
specific set of circumstances. Thus, this is character evidence, rather than habit evidence.

Answer to Question 10

(B) The question is proper to show that the defendant is inclined to lie. A witness may be impeached
with evidence of a prior conviction for (i) any felony or (ii) any crime involving dishonesty or false
statement. Because the conviction was for a crime of dishonesty (fraudulent business practices),
and the conviction is less than 10 years old, the question is proper on cross-examination because
it goes to the defendant’s credibility. (A) is wrong because evidence of prior convictions is not
admissible to show general bad character or propensity to commit crimes. (C) is wrong because
this crime has plenty of probative value on the issue of credibility and because most courts hold
that impeachment with a crime requiring proof or admission of an act of dishonesty or false state-
ment cannot be excluded as too prejudicial. (D) is wrong because similarity to the crime charged
is not required by the Federal Rules of Evidence.

Answer to Question 11

(B) The testimony is admissible because it tends to show the defendant’s good character for a perti-
nent trait. In a criminal matter, the accused is permitted, at any time during the trial, to introduce
evidence of his good character for the trait involved in the case to support the probability that
he did not commit the crime alleged. Here, the crime in question involves violence. Therefore,
evidence of the defendant’s good character for nonviolence may be admitted. Thus, (C) and (D)
are wrong as misstatements of the law. (A) is wrong because just the fact that evidence is not
hearsay is no reason to admit it. Choice (B) is a better option because it addresses the rules for
admissibility of character evidence, which is the issue raised by the question.
EVIDENCE - CHARACTER EVIDENCE ANSWERS 5.

Answer to Question 12

(B) The defendant’s conviction for embezzlement is most likely to be admitted. Because the defendant
did not take the stand, this evidence is not being offered for impeachment and, thus, the 10-year
time limit does not apply. Evidence of other crimes is admissible against an accused in a criminal
case if it is relevant to some issue other than the defendant’s character or disposition to commit the
crime charged. Where, as here, the crime charged is embezzlement, evidence that the defendant
committed embezzlement before might be admissible to establish fraudulent intent. The crimes
in (A), (C), and (D) are not relevant to any issue other than character and propensity to commit a
crime; therefore, they are inadmissible. The fact that (A) and (D) are felonies would be important
only if this were impeachment evidence, which it is not.

Answer to Question 13

(C) Both the neighbor’s testimony regarding the defendant’s reputation for being law-abiding and his
opinion that the defendant would not be involved with drugs should be admitted. The accused in a
criminal case may introduce evidence of her good character to show her innocence of the alleged
crime. However, when a defendant puts her character in issue by calling a qualified witness to
testify to her good character, the testimony must relate to the trait involved in the case, and is
limited to reputation or personal opinion testimony. Here, the defendant is on trial for a drug
charge and is claiming that the drugs were not hers, thereby putting at issue both her character
trait for obeying the law and her disposition towards drug involvement. Because the neighbor’s
testimony was limited to reputation and opinion evidence and did not mention specific acts, such
character evidence is admissible. Answer (A) is incorrect because the opinion evidence is admis-
sible. Answer (B) is incorrect because the reputation evidence is admissible. Answer (D) is incor-
rect because all of the testimony is admissible.

Answer to Question 14

(B) The record should be admitted because its direct relevance to the ultimate issue of the case
outweighs the danger of prejudice against the defendant. (A) is wrong because evidence of this
prior conviction would not be admissible as character evidence against the defendant. (C) is wrong
because the process leading to the conviction is irrelevant. (D) is wrong because an exception to
the hearsay rule applies. Federal Rule 803(22) provides that judgments of felony convictions are
admissible in both criminal and civil actions to prove any fact essential to the judgment. Felony
convictions are those for which the potential punishment is imprisonment in excess of one year;
hence, the hearsay exception applies here.
EVIDENCE - HEARSAY
QUESTIONS
EVIDENCE - HEARSAY QUESTIONS 1.

EVIDENCE - HEARSAY QUESTIONS

Question 1 Question 2

At a homicide trial, the prosecution presented Defendant was put on trial for robbery
evidence that, on the day of the fatal shooting, and aggravated battery. Despite having been
the defendant and several members of his gang seriously injured in the attack, the victim took
engaged in a running gun battle with the victim the witness stand and identified the defendant
and members of his gang. The prosecution as his attacker. The first trial ended in a hung
additionally presented evidence tending to prove jury on both charges, and the victim died shortly
that the defendant had been seen at the scene of thereafter from his injuries. The district attorney
the shooting around the time that the victim was then charged the defendant with felony murder.
killed. In his defense, the defendant proffered At the trial on that charge, the prosecution offers
testimony of a witness that, one day before the into evidence the victim’s testimony at the first
victim’s death, the defendant had said to him, trial identifying the defendant as his assailant.
“I’m flying to the state capital tonight for a The defendant’s attorney immediately objects.
two-day visit.”
Should the objection be sustained?
Should the witness’s testimony be admitted
over the objection of the prosecution? (A) Yes, because the victim’s testimony was
from a prosecution for a different offense
(A) Yes, because it is not being offered for the than the present trial.
truth of the matter asserted.
(B) Yes, because the use of the former testi-
(B) Yes, because it tends to prove that the mony of someone who has died violates the
defendant was in the state capital at the defendant’s constitutional right to confront
time the charged crime was committed. the witnesses against him in a criminal
case.
(C) No, because it is inadmissible hearsay.
(C) No, because the victim’s testimony is
(D) No, because it violates the propensity rule. nonhearsay under the Federal Rules as a
statement of prior identification.

(D) No, because the requirements of the excep-


tion to the hearsay rule for former testi-
mony have been satisfied.
2. EVIDENCE - HEARSAY QUESTIONS

Question 3 Question 4

A witness’s nephew was visiting her from In a medical malpractice action, a surgeon
a foreign country. One evening, the nephew was called as an expert witness by the plaintiff
went out with friends. At 11 p.m. that night, he and testified that the surgical procedure utilized
appeared back at the witness’s house, pounding by the defendant was so new and experimental
loudly on the door. She let him in, and noted that as to constitute negligence under the accepted
he was panting and out of breath. He immedi- standard of practice in the relevant medical
ately told her, “You won’t believe what I just community. On cross-examination by the defen-
saw! I was walking past your neighbor’s house dant’s counsel, the surgeon was asked whether
just now and the wife ran up to me with a gun Modern Surgical Procedures was a reliable
in her hand. She looked me straight in the eyes authority in his area of specialty. The surgeon
and said, ‘I killed the philandering fool’ before said that it was, and the defense counsel then
running off down the street.” After the nephew asked if the surgeon had relied upon the treatise
returned to his country, the wife was put on trial in reaching the conclusion that the defendant was
for the murder of her husband. The prosecution negligent. The surgeon stated that he did not.
wants to put the witness on the stand to testify Defense counsel now proposes to read a passage
regarding the nephew’s statement to her. The from the treatise stating that the surgical proce-
defense objects. dure at issue is widely accepted by responsible
medical practitioners. The plaintiff’s counsel
Can the witness testify to the nephew’s state- objects.
ment?
How should the court rule?
(A) Yes, because the nephew’s statement quali-
fies as an excited utterance. (A) For the defendant, but it should also cau-
tion the jury that the evidence may be
(B) Yes, because the nephew is not available to considered only in impeachment of the
testify. surgeon.

(C) No, because the wife did not make her (B) For the defendant.
admission to the witness.
(C) For the plaintiff, because the surgeon did
(D) No, because the witness’s testimony would not rely upon the treatise in forming his
constitute hearsay within hearsay. expert opinion.

(D) For the plaintiff, because the passage from


the treatise is inadmissible hearsay.
EVIDENCE - HEARSAY QUESTIONS 3.

Question 5 Question 6

An issue in the plaintiff’s action against A defendant was charged with the January 12
the defendant for causing the plaintiff’s back armed robbery of a grocery store in Texas. His
injury was whether the plaintiff’s condition had defense is that he was not in Texas on the date
resulted principally from a similar occurrence of the armed robbery and thus he could not have
five years before, with which the defendant had committed the crime. To show that he was not
no connection. in the area on January 12, the defendant wishes
to introduce into evidence a letter he wrote to
The plaintiff called as a witness his treating his sister stating, “I will see you in Vermont on
physician, who offered to testify that when she January 12.” The prosecution objects.
saw the plaintiff after the latest occurrence, the
plaintiff told her that before the accident he had Is the letter admissible?
been working full-time, without pain or limita-
tion of motion, in a job that involved lifting (A) Yes, as evidence of the defendant’s intent
heavy boxes. to go to Vermont on the date in question.

Should the physician’s testimony be admitted? (B) Yes, as a present sense impression.

(A) Yes, because it is a statement of the plain- (C) No, because the statement in the letter is
tiff’s then-existing physical condition. irrelevant.

(B) Yes, because it is a statement made for (D) No, because it is hearsay not within any
purposes of medical diagnosis or treatment. recognized exception to the hearsay rule.

(C) No, because it is hearsay not within any


exception.

(D) No, because the plaintiff is available as a


witness.
4. EVIDENCE - HEARSAY QUESTIONS

Question 7 Question 8

A pedestrian sued a driver, alleging that the A motorist who failed to stop at a stop sign
driver’s car ran a red light and struck the pedes- was struck by a car being taken for a test drive
trian in a crosswalk. At trial, the pedestrian’s by a mechanic who had repaired the car’s
attorney wishes to call an emergency room nurse brakes. The motorist sued the repair shop that
to testify that, when the pedestrian was brought employed her to recover for his injuries. At trial,
in, she asked him how he was injured and he the motorist called a bystander to testify that
replied, “I was hit by a car that ran a red light.” when the mechanic saw that the motorist was
injured, she ran over and told him, “I’m really
If the driver’s attorney objects, how should the sorry. I guess I didn’t fix the brakes as well as I
court rule regarding the nurse’s testimony? thought.”

(A) Admit it, as a present sense impression. Should the repair shop’s objection to the
bystander’s testimony be sustained?
(B) Admit it, because the statement was made
to the nurse for purposes of medical treat- (A) Yes, because the mechanic’s statement is
ment. inadmissible against the repair shop.

(C) Exclude it, as hearsay not within any recog- (B) Yes, because the motorist did not stop at the
nized exception to the hearsay rule. stop sign.

(D) Exclude it, because the pedestrian is avail- (C) No, because it is a declaration against
able to testify. interest.

(D) No, because it is a statement attributable to


a party-opponent.
EVIDENCE - HEARSAY QUESTIONS 5.

Question 9 Question 10

An airline passenger was on trial for A plaintiff brought a lawsuit against a depart-
attempting to smuggle illegal drugs into the ment store for injuries she suffered while riding
country. The prosecution is seeking to present an escalator that accelerated and then came to
evidence that when the passenger had passed the a sudden stop. At trial, a witness is prepared
entry area carrying a small suitcase, a police dog to testify for the plaintiff that he heard another
had sniffed at the suitcase and started barking, customer tell the store manager the day before
prompting the passenger to run into a men’s the plaintiff was injured that the escalator was
room, open his suitcase, and flush a bag down speeding up and stopping without warning. The
the toilet before anyone could apprehend him. customer still lives in the area but she has not
The dog’s handler is prepared to testify that been called to testify.
the dog has been trained not to bark unless it
detected illegal drugs, and that the dog started Should the court admit the witness’s testi-
barking after sniffing the suitcase. mony?

If the defense seeks to exclude the testimony (A) Yes, because it is a statement against inter-
by the dog handler describing the dog’s reaction est.
to the suitcase, how should the court proceed?
(B) Yes, because it is relevant evidence that the
(A) Admit the evidence, under a hearsay excep- department store was aware of the defect
tion. and did nothing to correct it.

(B) Admit the evidence, because the offered (C) No, because the customer is available to
evidence shows that the dog would not testify.
have barked unless illegal drugs had been
present. (D) No, because it is hearsay not within any
exception.
(C) Exclude the evidence, because it is hearsay
not within any recognized exception to the
hearsay rule.

(D) Exclude the evidence, because it violates


the Confrontation Clause of the United
States Constitution.
6. EVIDENCE - HEARSAY QUESTIONS

Question 11 Question 12

The plaintiff sued the defendant for injuries A man is suing a bakery to recover for
suffered when her car collided in an intersec- damage to the man’s garage, which burned down
tion with one driven by the defendant. At trial, from a fire that started in the bakery. The man
the plaintiff testified that she had had the right- alleged that the fire was started by one of the
of-way over the defendant to enter the intersec- bakery’s employees, who was known to come
tion. The defendant did not cross-examine her. into the bakery after an evening of drinking
The plaintiff then called a witness to testify at a nearby bar to experiment with recipes.
that, shortly after the collision, as she pulled The bakery denied the allegation and claimed
the plaintiff from the car, the witness heard the that the fire was the result of arson by a local
plaintiff say, “I think I’m dying! Didn’t the other gang member. At trial, the man called to the
driver see I had the right-of-way?” The witness’s stand a court reporter, who will testify that he
testimony was admitted over defense counsel’s recorded the testimony of the now-deceased
objections. On appeal from a verdict for the manager of the bakery at a preliminary hearing
plaintiff, the defendant challenges the admission on a criminal charge of arson against the gang
of the witness’s testimony. member, and the manager testified that the fire
was the result of the employee’s drunken experi-
Should the trial court’s ruling be upheld? ments. The bakery’s attorney objects to the court
reporter’s testimony.
(A) Yes, because the plaintiff’s statement was
made under belief of impending death. Should the court rule that the testimony is
admissible?
(B) Yes, because the plaintiff’s statement was
an excited utterance. (A) Yes, as substantive evidence of the cause of
the fire.
(C) No, because the plaintiff’s credibility had
not been attacked. (B) Yes, under the former testimony exception
to the hearsay rule.
(D) No, because the plaintiff’s belief that she
had the right-of-way had already been (C) No, because the court transcript would be
established without contradiction. the best evidence.

(D) No, because it is hearsay not within any


exception.
EVIDENCE - HEARSAY QUESTIONS 7.

Question 13 Question 14

A witness was stopped at an intersection A plaintiff suffered injuries when her car
when she saw a car run a red light, strike the was struck by the defendant’s car. The police
victim in the crosswalk, and proceed through the arrived on the scene and required the defen-
intersection. The witness gave a very detailed dant to take a breathalyzer test. The defendant
description of the driver to the police officer at was cited for driving while intoxicated, tried in
the scene. Based on this description, the police traffic court, and duly convicted. He received
apprehended the defendant and charged him the maximum sentence for driving while intoxi-
with several criminal counts for the accident that cated, which is 90 days’ imprisonment in the
seriously injured the victim. The witness testified county correctional facility. The plaintiff brought
at the trial, but the defendant was acquitted. The a civil action against the defendant, seeking
victim then filed a civil suit against the defendant compensation for her personal injuries. At the
to recover for her injuries. Before the trial of the trial of the plaintiff’s suit, the plaintiff’s attorney
victim’s suit, the witness died. In her suit against offers a properly authenticated photocopy of the
the defendant, the victim offers into evidence the court judgment showing that the defendant was
police report containing the witness’s description convicted of driving while intoxicated.
of the driver. The defendant objects.
Is the evidence admissible?
How should the court rule on the admissibility
of the report? (A) Yes, as a public record.

(A) Admissible, because the report is relevant, (B) Yes, as a final judgment offered to prove a
and it is not hearsay. fact essential to a point in controversy.

(B) Admissible, because the report falls within (C) No, because the crime was punishable by
the business records exception to the imprisonment of at most 90 days.
hearsay rule.
(D) No, because it is not the best evidence of
(C) Inadmissible, because the report is hearsay the defendant’s conviction.
not within any exception.

(D) Inadmissible, because the report is not the


best evidence.
8. EVIDENCE - HEARSAY QUESTIONS

Question 15 Question 16

A plaintiff brought a conversion action against The owner of a jewelry store brought a civil
a defendant, alleging that the defendant had action against a former clerk for the value of
wrongfully taken a necklace owned by the plain- various pieces of jewelry missing from the store.
tiff’s mother, who had recently died intestate. The defendant had been fired after another
The defendant’s defense is that the mother, who employee had reported that the defendant was
had lived next door to the defendant, had freely stealing jewelry. At the trial, the plaintiff calls
given her the necklace because she had often his employee as a witness. The witness testifies
given rides to and run errands for the mother, that he does not remember either having seen the
who did not drive and had trouble getting defendant take anything from the store or having
around. The defendant is cross-examined by the told the plaintiff that she had done so. The plain-
plaintiff’s attorney, who challenges the defen- tiff then takes the witness stand and proposes to
dant’s claim that the mother had given her the testify to what the witness had told him about
necklace. The defendant wishes to testify that, seeing the defendant stealing pieces of jewelry
before handing her the necklace, the mother told from the store.
her, “You’ve always been so good to me that I
want you to have this necklace.” Assuming appropriate objection by the defen-
dant, would such testimony by the plaintiff be
Is the defendant’s proposed testimony admis- admissible?
sible?
(A) Yes, as a statement against interest by the
(A) No, because it is hearsay not within any witness.
exception.
(B) Yes, as proper impeachment of the witness’s
(B) No, under the state’s Dead Man Act. testimony.

(C) Yes, because it is nonhearsay. (C) No, as irrelevant.

(D) Yes, because the declarant is unavailable to (D) No, as inadmissible hearsay if offered to
testify. prove theft by the defendant.
EVIDENCE - HEARSAY QUESTIONS 9.

Question 17 Question 18

The defendant was at work when her husband The defendant was prosecuted for bank
called her and said, “The neighbor just tripped robbery. At trial, the bank teller was unable to
over those roots I told you to take out. He’s identify the defendant, now bearded, as the bank
badly hurt and I’ll bet he sues us for all we’re robber. The prosecutor then showed the teller
worth.” The defendant then told her secretary, a group of photographs, and the teller testified
“The neighbor just got hurt because I forgot to that she had previously told the prosecutor that
do my yard work.” On returning home, however, the middle picture (concededly a picture of the
the defendant discovered that the neighbor had defendant before he grew a beard) was a picture
actually tripped over roots from his own tree of the bank robber.
in his own yard. The neighbor disagreed and
sued the defendant and her husband. At trial, Is the teller’s testimony admissible?
the neighbor called the defendant’s secretary to
testify as to the defendant’s statement to him. (A) No, because it is hearsay not within any
exception.
Will the secretary’s testimony be admitted?
(B) No, because it is a violation of the defen-
(A) No, because the defendant had no firsthand dant’s right of confrontation.
information when she made her statement
to the secretary. (C) Yes, as prior identification by the witness.

(B) No, because it is inadmissible lay opinion. (D) Yes, as past recollection recorded.

(C) Yes, because it is not hearsay.

(D) Yes, to impeach the defendant’s expected


testimony as to the result of her own inves-
tigation.
EVIDENCE - HEARSAY
ANSWERS
EVIDENCE - HEARSAY ANSWERS 1.

EVIDENCE - HEARSAY ANSWERS

Answer to Question 1
(B) The testimony should be admitted. When analyzing questions involving the admissibility of
hearsay evidence, ask two things: (i) Is the proffered evidence hearsay (i.e., an out-of-court state-
ment by a declarant, being offered to prove the truth of the matter asserted in the statement)? (ii)
If hearsay, is the proffered evidence nevertheless admissible because it fits within an exception
to the hearsay rule? The witness’s proffered testimony is hearsay because it matters whether the
defendant’s statement is true or false. If his statement is true (if the defendant actually intended
to go to the state capital, as stated), then the likelihood that he actually went there is increased.
(This is because people tend to act in a manner consistent with their previously stated inten-
tions.) Since the witness’s testimony would be used for the purpose of establishing the truth of
the defendant’s statement, it is hearsay evidence. However, the testimony fits within a hearsay
exception. A declarant’s statement of present intention to take an action in the future fits within
the “state of mind” exception contained in Federal Rule of Evidence 803(3). One’s intent is a part
of one’s state of mind. Thus, the testimony should be admitted. This two-step analysis of hearsay
evidence also makes clear that (A) and (C) are incorrect. (A) states that the witness’s testimony is
not being offered for the truth of the matter asserted, but, as stated above, it is offered for its truth.
(C) correctly characterizes the testimony as hearsay but incorrectly states that the evidence is
inadmissible; it is admissible under the state-of-mind exception. (D) is a nonsensical answer. The
witness’s testimony does not pertain in any way to the defendant’s character. Thus, the propensity
rule, which states that evidence of a person’s character trait is generally inadmissible to prove that
the person acted in conformity with the trait on a particular occasion, is not implicated by this
question.
Answer to Question 2
(D) The victim’s former testimony should be admissible. Under Federal Rule 804(b)(1), the testimony
of a now unavailable witness given at another hearing under oath is admissible in a subsequent
trial as long as there is a sufficient similarity of parties and issues so that the opportunity to
develop testimony or cross-examine at the prior hearing was meaningful. In criminal cases, the
accused or his attorney must have been present and have had the opportunity to cross-examine
at the time the testimony was given. Here, the victim’s testimony was given against the defen-
dant at a prosecution for the same conduct that constituted the felony murder offense for which
the defendant is now being prosecuted, and the defendant had the same motive to challenge the
victim’s identification of him then as he does now. Hence, the testimony can be admitted under
the former testimony exception to the hearsay rule. (A) is incorrect because it is not essential, even
in criminal cases, that the trials be on the same charge. The exception is satisfied as long as the
subject matter is sufficiently similar that the defendant had an opportunity and similar motive to
develop the declarant’s testimony at the prior hearing. (B) is incorrect because the Supreme Court
has rejected a Confrontation Clause challenge to the use of former testimony in this situation. As
long as the defendant had the opportunity to cross-examine at the prior hearing and the witness is
truly unavailable, the testimony is admissible. (C) is incorrect because it refers to prior statements
of identification by a witness at the present trial. This case involves the hearsay exception for
former testimony, which requires that the witness now be unavailable.
Answer to Question 3
(A) The witness can testify to the nephew’s statement because it qualifies as an excited utterance.
The statement of the wife that the nephew is relating is admissible as a statement of a party-
2. EVIDENCE - HEARSAY ANSWERS

opponent (commonly called an admission). The problem presented here is one of hearsay within
hearsay. Hearsay is a statement, other than one made by the declarant while testifying at the trial
or hearing, offered in evidence to prove the truth of the matter asserted. A hearsay statement to
which no exception to the hearsay rule is applicable must be excluded upon appropriate objection.
Hearsay included within hearsay is admissible only if each layer of hearsay falls within a hearsay
exception. Here, two separate statements are really being offered for the truth of the matter
asserted therein: First, the nephew’s statement is being offered to prove that he actually said that
the wife admitted killing the husband. Second, the wife’s statement is being offered to prove that
she killed the husband. However, the wife’s statement is a statement by a party-opponent, which is
traditionally treated as a hearsay exception and is treated as nonhearsay under the Federal Rules.
In either case, this statement alone would be admissible. The nephew’s statement relating the
wife’s admission is also admissible; it comes within the excited utterance exception to the hearsay
rule. Under this exception, a declaration made during or soon after a startling event is admissible.
There must have been an occurrence startling enough to produce a nervous excitement and thus
render the declaration an unreflective expression of the declarant’s impression of the event. Also,
the statement must have been made while the declarant was under the stress of the excitement.
Here, the nephew witnessed the wife running with a gun in her hand and declaring that she had
killed her husband. The nephew immediately ran back to the witness’s house and told her, “You
won’t believe what I just saw!” The occurrence, including the wife’s statement, was certainly a
startling event, and the nephew seems to have made his statement to the witness while he was
still under the stress of excitement caused by the occurrence. Thus, the witness can testify to
the nephew’s statement, including the part relating what the wife had told him. (B) is incorrect
because the exception to the hearsay rule for excited utterances such as the nephew’s statement
does not require the declarant to be unavailable to testify. (C) is incorrect because it does not
matter in this case that the party-opponent made her admission to someone other than the testi-
fying witness. The witness can repeat what the nephew said—including the wife’s admission—
because it was an excited utterance. (D) is incorrect because, as discussed above, both parts of the
nephew’s statement are admissible.
Answer to Question 4
(B) The court should rule for the defendant and allow the treatise to be read and considered by the
jury as substantive evidence. Although the treatise constitutes hearsay because it is an out-of-
court statement offered to prove the truth of the matter asserted (that the surgical procedure was
accepted), it falls within one of the exceptions to the hearsay rule. Under Federal Rule 803(18),
information in treatises can be read into evidence if the treatise is: (i) relied upon by the expert or
is called to his attention during cross-examination; and (ii) established as reliable by the witness,
another expert, or judicial notice. The treatise itself is not admitted into evidence, but rather the
relevant section is read in. Thus, (B) is correct and (D) is incorrect. (A) is incorrect because the
Federal Rules allow such evidence to be used substantively and do not limit the information to
impeachment, as a number of state courts do. (C) is incorrect because the information in a learned
treatise is admissible as long as it is established to be reliable; however, if the opposing party did
rely on the treatise, the offering party need not otherwise establish reliability.
Answer to Question 5
(B) The testimony of the physician should be admitted as a statement made for purposes of diagnosis
or treatment. The Federal Rules allow the admission of statements not only of past symptoms
and medical history, but also of the cause or source of the condition as reasonably pertinent to
diagnosis or treatment. This is an exception to the hearsay rule. (A) is incorrect because this is
not a statement of present bodily condition, which falls under a different exception to the hearsay
EVIDENCE - HEARSAY ANSWERS 3.

rule. (C) is incorrect because, as explained above, this testimony is admissible under an exception
to the hearsay rule. (D) is incorrect because this hearsay exception does not require unavailability
of the declarant.

Answer to Question 6

(A) The letter is admissible as evidence of the defendant’s intent to go to Vermont on January 12.
Hearsay is an out-of-court statement offered to prove the truth of the matter asserted. Upon
objection, hearsay must be excluded unless it falls within a recognized exception to the rule. The
letter is an out-of-court statement, and it is being offered to prove its truth; i.e., that the defendant
intended to be in Vermont on January 12. The letter, therefore, is hearsay. However, there is an
exception for declarations of state of mind, including declarations of intent offered to show subse-
quent acts of the declarant; i.e., a statement of intent to do something in the future is admitted
as circumstantial evidence that the intent was carried out. This principle was established in the
famous Hillmon case, which held that state of mind is admissible to show that the declarant acted
in conformity with his expressed declaration. The defendant’s letter was a statement of intent to
go to Vermont on January 12 and is admissible as circumstantial evidence that he did so. (B) is
wrong because it states the wrong hearsay exception. A present sense impression is a comment
made concurrently with the perception of an event concerning the event perceived. Clearly this
exception does not apply to these facts because the defendant was not perceiving an event and
describing it in his letter; rather, he was expressing his state of mind at the time he was writing.
(C) is wrong because the letter is relevant. Evidence is relevant if it tends to make the existence of
a material fact more probable or less probable than it would be without the evidence. The defen-
dant’s whereabouts on the day of the crime are certainly a fact of consequence to the determina-
tion of this action. The letter makes the fact that he was in Vermont on January 12 more probable
than it would be without the letter; thus, the letter is relevant. (D) is wrong because the letter falls
within the state of mind exception to the hearsay rule (see above).

Answer to Question 7

(C) The statement is inadmissible hearsay. This question involves interpretation of the hearsay excep-
tion for out-of-court statements made for purposes of medical diagnosis or treatment. Under
the Federal Rules, statements about the cause or source of a medical condition are admissible if
reasonably pertinent to diagnosis or treatment. [Fed. R. Evid. 803(4)] That a car struck the pedes-
trian is reasonably pertinent to medical diagnosis or treatment; a nurse or doctor would probably
want to know what degree of force was absorbed by his body. This information would provide
them with better insights as to the medical cause of the pedestrian’s injuries. On the other hand,
the color of the traffic light the car ran (red, yellow, or green) has no bearing on anything other
than fault; that information would not help medical personnel diagnose or treat the pedestrian’s
injury. Hence, (B) is incorrect. (A) is incorrect because a present sense impression is a statement
describing or explaining an event made while perceiving the event or immediately thereafter.
Here, the pedestrian’s statement to the emergency room nurse was not made immediately after
the accident. (D) is incorrect because the fact that the out-of-court declarant (the pedestrian) is
available to testify does not render his statement to the emergency room nurse inadmissible. This
hearsay exception, like most, does not require that the declarant be unavailable to testify before
evidence of his hearsay statement can be introduced.

Answer to Question 8

(D) The mechanic’s statement is admissible as a statement attributable to a party-opponent. The


Federal Rules treat statements by a party-opponent (commonly known as admissions) as
4. EVIDENCE - HEARSAY ANSWERS

nonhearsay (whereas most states consider them to be an exception to the hearsay rule). This rule
applies to any statement made by a party and offered against that party. Such a statement need not
have been against interest at the time it was made. Some statements are considered admissions
even if not made by the party against whom they are offered, as long as they are attributable to
the party. One such vicarious admission is a statement by an agent concerning a matter within the
scope of her agency, made during the existence of the agency relationship. [Fed. R. Evid. 801(d)
(2)(D)] Here, the bystander’s testimony as to the mechanic’s statement is offered to prove the truth
of the matter asserted therein; i.e., that the mechanic had not properly fixed the brakes. Thus, the
mechanic’s statement would normally be considered hearsay. However, the statement was made
while she was an agent of the repair shop, and the statement concerned a matter within the scope
of her agency (i.e., whether she had properly performed the job for which she was employed by
the repair shop). Consequently, her statement may be introduced against the repair shop as an
admission by a party-opponent of negligence in the repair of the brakes. (A) is wrong because the
statement is admissible against the repair shop as a vicarious admission. (B) is wrong because
it is irrelevant to the issue of whether the bystander’s testimony is admissible. The fact that the
motorist failed to stop at the stop sign, and thus was negligent, will not prevent introduction of
testimony that the mechanic admitted that she was negligent. (C) is wrong because the statement
against interest exception to the hearsay rule is applicable only where the declarant is unavailable
as a witness. Here, the mechanic is available as a witness. In addition, as noted above, a statement
by a party-opponent is admissible even if not against interest when made. Thus, the statement
would be admissible against the repair shop even if it were not against its interest when made.

Answer to Question 9

(B) The testimony is admissible with a proper foundation. Choice (B) essentially says that evidence
of the dog’s barking will be admissible because it is relevant; it is relevant because it can be
shown that, based on the dog’s training, it would not have barked unless drugs had been present.
(A) and (C) are incorrect because an animal cannot make a hearsay statement. For purposes of
the hearsay rule, a “statement” is limited to an oral or written assertion (or nonverbal conduct
intended to be an assertion) of a person. [Fed. R. Evid. 801(a)] Similarly, (D) is incorrect to
conclude that such evidence is inadmissible because it violates the Confrontation Clause of the
United States Constitution. For purposes of the Confrontation Clause, an animal would not be
considered a “witness”; thus, a defendant has no right to “confront” the animal in court. The
hearsay rule and the Confrontation Clause have a common rationale: human beings have a well-
documented capacity for deception and lying. Cross-examination is an extremely useful way to
test the veracity of a witness who might be lying. If the “witness” cannot be cross-examined, or
can only be cross-examined months after making an out-of-court statement, the ability to test the
truthfulness of the witness and the accuracy of his statements will be severely limited. However,
unlike human beings, animals do not consciously lie. It thus makes little sense to say that
evidence of a police dog’s barking, under circumstances where the bark indicates the presence of
drugs in a suitcase, should be inadmissible.

Answer to Question 10

(B) The court should find the witness’s testimony admissible as relevant evidence that the depart-
ment store was aware of the defect. Hearsay is a statement, other than one made by the declarant
while testifying at the trial or hearing, offered in evidence to prove the truth of the matter
asserted. [Fed. R. Evid. 801(c)] A hearsay statement to which no exception to the hearsay rule is
applicable must be excluded upon appropriate objection to its admission. [Fed. R. Evid. 802] A
statement that would be inadmissible hearsay may be admissible to show the effect of the state-
EVIDENCE - HEARSAY ANSWERS 5.

ment on the listener or reader. For example, in a negligence case where knowledge of a danger
is in issue, a third person’s statement of warning is admissible to show notice or knowledge
on the part of a listener. Here, the customer’s remark to the manager is an out-of-court state-
ment. However, the statement can be offered to show that the department store had notice of a
possible danger posed by the escalator. (Note that the remark would be inadmissible hearsay if
offered to prove the escalator was speeding up and stopping.) (A) is incorrect because it states
an exception to the hearsay rule, and the testimony offered is not hearsay (see above). Even if
the testimony were offered to prove the escalator was malfunctioning, this exception would not
apply because the declarant (the customer) made no statement against her interest. Also, she is
available to testify, which takes her statement out of the exception. (C) is incorrect because the
witness’s testimony is admissible nonhearsay. In addition, the unavailability of a declarant is only
significant with regard to certain hearsay exceptions that require unavailability (e.g., the state-
ment against interest). If the witness’s testimony is offered to show notice to the department store
of a potential problem with the escalator, there is no hearsay problem. The absence of a hearsay
problem precludes resort to a hearsay exception, and the availability of the customer is of no
significance. (D) is incorrect because the customer’s statement is not hearsay because it is not
offered to prove the truth of the matter asserted (i.e., that the escalator was malfunctioning), but
to show notice of the defect.

Answer to Question 11

(B) The testimony was properly admitted under the excited utterance exception to the hearsay rule.
Statements made under the stress of some exciting event and relating to that event are admissible
as an exception to the hearsay rule. (A) is incorrect because the Federal Rules require the maker
of a dying declaration to be unavailable for the declaration to be admissible. (C) is incorrect; even
though the statement bolsters the credibility of the plaintiff, it still qualifies as an exception to the
hearsay rule and is admissible substantive evidence. (D) is incorrect because it is up to the broad
discretion of the trial judge under Federal Rule 403 whether to exclude relevant evidence based on
needless presentation of cumulative evidence. There is nothing to indicate that admission of the
testimony constituted an abuse of discretion.

Answer to Question 12

(A) The court should rule that the testimony is admissible as substantive, nonhearsay evidence.
Hearsay is an out-of-court statement offered to prove the truth of the matter asserted, and is
inadmissible unless it falls within an exception to the hearsay rule. However, statements by an
opposing party (commonly called admissions) are not hearsay under the Federal Rules. An
admission is a statement made by a party and offered against that party. Certain statements by
nonparty declarants are attributable to the party as “vicarious” admissions because of the relation-
ship between the party and the declarant. For instance, the statement of a person authorized by a
party to speak on its behalf can be admitted against the party as an admission. Furthermore, state-
ments by an agent or employee concerning any matter within the scope of his agency or employ-
ment, made during the existence of the agency or employment relationship, are admissible against
the principal. Here, the bakery’s manager, an agent of the bakery, admitted during the preliminary
hearing that the bakery’s employee was at fault for the fire. This statement by the manager is
admissible against the bakery as a vicarious admission. Therefore, (A) is correct and (D) is incor-
rect. (B) is incorrect. Under the former testimony exception to the hearsay rule, the testimony of
a now-unavailable witness given at another hearing is admissible in a subsequent trial if there is a
sufficient similarity of parties and issues so as to provide sufficient opportunity for cross-exami-
nation. As discussed, the testimony of the court reporter is not hearsay and the exception does not
6. EVIDENCE - HEARSAY ANSWERS

apply. However, even if the testimony were hearsay, the former testimony exception would still
not apply because the criminal charges against the gang member are not related to the civil suit
between the man and the bakery. The bakery was not a party to the criminal case and thus never
had any opportunity to cross-examine the manager as to his statement. (C) is incorrect because
the best evidence rule applies only when the material contents of a document are sought to be
proved. The rule does not apply to these facts because the manager’s testimony exists indepen-
dently of the transcript.

Answer to Question 13

(C) The court should not admit the report because it is hearsay not within any exception. The report
contains an out-of-court statement being offered for its truth; i.e., that the person who hit the
victim fits the description given by the witness. Thus, (A) is incorrect. The report does not fall
within any exception to the hearsay rule. It is not a business record because the witness was not
under a business duty to convey the information to the police. (B) is therefore incorrect. (D) is
incorrect because the report is the original document, and the best evidence rule expresses a
preference for originals.

Answer to Question 14

(C) The evidence of the defendant’s conviction is inadmissible to prove that the defendant was intoxi-
cated because the crime was punishable by imprisonment of at most 90 days. Despite the fact that
copies of judgments are hearsay (because they are out-of-court statements used to prove the truth
of the matter asserted), the Federal Rules of Evidence provide that a judgment of a felony convic-
tion is admissible as an exception to the hearsay rule in both criminal and civil actions to prove
any fact essential to the judgment. The Rules define felony convictions as crimes punishable by
death or imprisonment in excess of one year. [Fed. R. Evid. 803(22)] Here, the crime for which
the defendant was convicted carries a maximum term of imprisonment of 90 days. Thus, the
defendant’s conviction is not a felony conviction for purposes of the Federal Rule, and the convic-
tion is inadmissible hearsay as proof of the fact asserted (i.e., that the defendant was driving his
car while intoxicated when he struck the plaintiff’s car). (A) is incorrect because the copy of the
judgment of conviction is not a record, report, statement, or data compilation of a public office
or agency, setting forth: (i) the activities of the office or agency, (ii) matters observed pursuant to
a duty imposed by law, or (iii) factual findings resulting from an investigation made pursuant to
authority granted by law. Therefore, the copy of the judgment is not deemed to be a public record
for purposes of the hearsay exception for public records and reports. (B) is incorrect because a
final judgment offered to prove a fact essential to a point in controversy is admissible under the
Federal Rules only if the judgment is a felony conviction. As noted above, the defendant’s convic-
tion is not a felony conviction. (D) is incorrect because a properly authenticated copy of a court
judgment would be the best evidence of the defendant’s conviction. The best evidence rule states
that, in proving the terms of a writing, where the terms are material, the original writing must
be produced. [Fed. R. Evid. 1002] One class of situations to which the rule applies is that in
which the writing is a legally operative or dispositive instrument such as a contract, deed, will, or
judgment. Such writings are viewed as essential repositories of the facts recorded therein, and as
such are within the rule. Furthermore, duplicates (e.g., photocopies) are admissible in the same
way as originals, unless: (i) the authenticity of the original is challenged; or (ii) circumstances
exist that would render it unfair to admit the duplicate in place of the original. [Fed. R. Evid.
1003] Here, a properly authenticated photocopy of the judgment of conviction would be as admis-
sible as the original judgment for purposes of proving that such a judgment has been entered.
Therefore, there is no violation of the best evidence rule.
EVIDENCE - HEARSAY ANSWERS 7.

Answer to Question 15

(C) The mother’s statement is admissible as a legally operative fact. Hearsay is a statement, other
than one made by the declarant while testifying at trial or a hearing, offered in evidence to prove
the truth of the matter asserted therein. Where an out-of-court statement is introduced for any
purpose other than to prove the truth of the matter asserted, the statement is not hearsay. One
type of out-of-court statement that is not hearsay is evidence of legally operative facts. These
are utterances to which legal significance is attached, such as words of contract, bribery, or
cancellation. Evidence of such statements is not hearsay because the issue is only whether the
statement was made. The defendant is defending the lawsuit on the basis that she received the
necklace as a gift, and the statement by the mother contains words that constitute an expres-
sion of donative intent, which is essential to a finding of a gift having been made. Thus, (C) is
correct and (A) is incorrect. (B) is incorrect because, assuming a Dead Man Act is applicable,
its protection has been waived. Dead Man Acts provide that a person interested in an event is
incompetent to testify to a personal transaction or communication with a deceased, when such
testimony is offered against the representative or successors in interest of the decedent. One who
claims under a decedent may waive the protection of the statute by cross-examining the inter-
ested person about the transaction. In such a case, the interested person may explain all matters
about which she is examined. Here, because the plaintiff’s attorney is questioning the defendant
about her dealings with the mother, the statute’s protection has been waived. (D) is incorrect
because unavailability of a declarant controls whether certain kinds of hearsay are admissible as
exceptions to the hearsay rule. Because the statement here at issue is not hearsay, the declarant’s
unavailability is irrelevant.

Answer to Question 16

(D) If offered to prove that the defendant stole the jewelry, the testimony by the plaintiff would be
hearsay and, thus, inadmissible. Hearsay is a statement, other than one made by the declarant
while testifying at the trial or hearing, offered in evidence to prove the truth of the matter
asserted. [Fed. R. Evid. 801(c)] A hearsay statement, to which no exception to the hearsay rule
is applicable, must be excluded upon appropriate objection. [Fed. R. Evid. 802] The proffered
testimony of the plaintiff relates to a statement made by the witness other than while testifying
at the instant trial. Therefore, if the witness’s out-of-court statement is offered to prove that the
defendant stole the pieces of jewelry, the statement is hearsay. Because no exceptions to the
hearsay rule apply, the statement is inadmissible. (A) is incorrect because the statement is not
against the interest of the declarant (the witness). Under the statement against interest exception
to the hearsay rule, statements of a person, now unavailable as a witness, against that person’s
pecuniary, proprietary, or penal interest when made are admissible. [Fed. R. Evid. 804(b)(3)]
Here, the witness may be deemed to be unavailable because he has testified to a lack of memory
of the subject matter to which his original statement to the plaintiff relates. However, the state-
ment contained in the proposed testimony of the plaintiff is not against any interest of the witness,
who is the declarant, but is rather against the interest, both penal and civil, of the defendant.
Therefore, the statement does not qualify for admissibility as a statement against interest. (B)
is incorrect because the witness has simply testified that he does not remember either seeing
the defendant take the jewelry or telling the plaintiff that she did so. Impeachment refers to the
casting of an adverse reflection on the truthfulness of a witness. One form of impeachment is to
show that a witness has, on another occasion, made statements that are inconsistent with some
material part of his present testimony. If the witness in his testimony had denied seeing the defen-
dant take anything or telling the plaintiff that she had done so, then the testimony of the plaintiff
as to the witness’s previous statements would be admissible as a prior inconsistent statement, thus
8. EVIDENCE - HEARSAY ANSWERS

serving to disprove the credibility of the witness. However, because the witness has merely testi-
fied to a lack of memory concerning these matters, the plaintiff’s testimony probably would not
be considered a prior inconsistent statement. Although (B) could be correct in some jurisdictions,
most would not consider introduction of a prior inconsistent statement an appropriate response to
a claim of lack of memory unless the court believed the witness was being deliberately evasive.
Because (D) is a completely accurate statement, it is the better answer. (C) is incorrect because the
plaintiff’s testimony is relevant. Evidence is logically relevant if it tends to make the existence of
any fact of consequence to the determination of an action more probable than it would be without
the evidence. [Fed. R. Evid. 401] The plaintiff’s testimony that he was told that the defendant stole
jewelry from his store would certainly tend to make it more probable that she took the jewelry
than would otherwise be the case. This fact is of great consequence to the determination of the
plaintiff’s action against the defendant for the value of the missing jewelry. Thus, the proffered
testimony is relevant. Although relevant, however, the testimony runs afoul of the hearsay rule,
and is thus inadmissible.

Answer to Question 17

(C) The testimony is not hearsay because the defendant’s statement constitutes an opposing party’s
statement—i.e., a statement by a party (the defendant) being offered against her. An opposing
party’s statement (commonly known as an admission) is not considered a hearsay statement under
the Federal Rules. [Fed. R. Evid. 801(d)(2)] Thus, (C) appears to be correct—that the secretary’s
testimony about the defendant’s statement will be admitted because it is not hearsay. (C) is not an
ideal answer, though, because it is so incomplete. The fact that an item of evidence is nonhearsay
does not automatically render it admissible. For example, if nonhearsay evidence is irrelevant,
it would not be admissible. Thus, deciding whether (C) is the best answer requires a thorough
assessment of the alternatives. (D) can be quickly discarded as incorrect. Impeachment evidence
is not admissible until after the witness to be impeached has testified. It would be improper to
admit the secretary’s testimony for the purpose of impeaching the defendant’s expected testi-
mony as to the result of her own investigation of the accident. (A) is tempting, but it is also incor-
rect. Normally, an out-of-court statement of a declarant, like the in-court testimony of a witness,
is admissible only if it was made with personal knowledge. This requirement, however, does
not apply to admissions. Thus, although the defendant’s statement (“The neighbor just got hurt
because I forgot to do my yard work”) was made without personal knowledge, it will be admis-
sible. As a party, the defendant will have ample opportunity to explain why she made the state-
ment even though she lacked personal knowledge about the accident that injured the neighbor. The
persuasiveness of her explanation will determine whether the jury takes her admission seriously.
(B) is also incorrect. The common law requirement that lay witnesses were not permitted to give
opinions, but only allowed to state facts, has been rejected in favor of a helpfulness standard. For
example, Rule 701 allows lay witnesses to give testimony in opinion form as long as it is helpful to
a clear understanding of the testimony or the determination of a fact in issue. Similarly, an admis-
sion is not rendered inadmissible merely because it is in opinion form. For example, a party’s
statement that “I was negligent” or “It was all my fault” will be admitted into evidence as an
admission. Therefore, although (C) is not an ideal answer, it is basically correct, and (A), (B), and
(D) are clearly incorrect.

Answer to Question 18

(C) The teller’s testimony is admissible as prior identification by the witness. Under the Federal Rules
of Evidence, a statement of prior identification of another person by the witness is nonhearsay.
Thus, (A) is incorrect. (B) is incorrect because the teller is testifying at the trial and is subject to
EVIDENCE - HEARSAY ANSWERS 9.

cross-examination. Thus, the defendant’s right of confrontation is not violated. (D) is incorrect
because past recollection recorded is an exception to the hearsay rule, and the statement here
at issue is nonhearsay. Also, past recollection recorded involves use of a writing made when the
events were fresh in the mind of the witness. There is no such writing involved here.
EVIDENCE -
IMPEACHMENT
QUESTIONS
EVIDENCE - IMPEACHMENT QUESTIONS 1.

EVIDENCE - IMPEACHMENT QUESTIONS

Question 1 Question 2

A defendant was on trial for burglary, and The defendant was charged with first degree
he took the stand in his own defense. On direct felony arson under the state’s arson statute. The
examination, the defendant vigorously denied indictment alleged that the defendant burned
having committed the burglary. Also on direct down his own building to collect the insur-
examination, the defense attorney asked the ance. At the trial, the defense called a witness
defendant questions about his employment who testified on direct examination that he
history in an attempt to portray him to the jury was with the defendant at the place of business
as a “solid citizen” who would not commit a when the fire started and that some cleaning
burglary. The defendant stated that his last solvent caught fire and spread out of control. The
regular employment was as a bookkeeper for a witness testified that the ignition of the fire was
corporation. On cross-examination, the prose- purely accidental. The defense also introduced
cutor asked the defendant if he had embezzled evidence that the witness is of good character.
funds from the corporation. The defendant On cross-examination, the prosecutor asked the
denied that he had embezzled from the corpora- witness if he is being prosecuted for first degree
tion or from anyone else. The prosecutor then felony arson in a separate trial for the burning of
wanted to call a police officer to the stand to the same building. The defense lawyer objects.
testify that when she arrested the defendant for
embezzlement, the defendant admitted to the Should the court allow the prosecutor’s
officer that he had embezzled money from the question?
corporation.
(A) Yes, because the defense lawyer has intro-
Assuming that the defendant has not yet been duced evidence tending to establish that the
tried on the embezzlement charges, may the witness is a person of good character.
prosecutor call the officer to the stand?
(B) Yes, because the question is appropriate
(A) Yes, but only for purposes of impeachment. to show bias or interest on the part of the
witness.
(B) Yes, both for impeachment of the defendant
and as substantive evidence. (C) No, because the witness has not been
convicted of the crime.
(C) No, because the defendant has not yet been
convicted of embezzlement. (D) No, because the question violates the
witness’s Fifth Amendment right to be
(D) No, because the evidence would be protected from self-incrimination.
extrinsic.
2. EVIDENCE - IMPEACHMENT QUESTIONS

Question 3 Question 4

A defendant is on trial in federal court, At the defendant’s trial for arson, the prosecu-
charged with having sold cocaine to an under- tion seeks to impeach the defendant through a
cover agent. The defendant calls a witness to the record of the defendant’s conviction of a misde-
stand who testifies that she was with the defen- meanor robbery five years ago. The defendant
dant in another state on the date of the alleged objects.
drug sale. Following her testimony, the prosecu-
tion seeks to introduce the record of judgment Should the court admit this evidence over the
of the witness’s seven-year-old embezzlement defendant’s objection?
conviction. The defendant’s attorney objects.
(A) No, unless the prosecution shows that its
The court should rule the record of judgment: probative value as impeachment evidence
outweighs its prejudicial effect.
(A) Admissible, as going to the witness’s cred-
ibility. (B) No, because the conviction is a misde-
meanor and the crime did not involve
(B) Admissible, provided no appeal is pending. dishonesty.

(C) Inadmissible, because the record is (C) Yes, because the conviction is less than 10
inadmissible hearsay. years old.

(D) Inadmissible, because, for impeachment, a (D) Yes, because a witness may be impeached
specific act of misconduct cannot be shown by any crime.
by extrinsic evidence.
EVIDENCE - IMPEACHMENT QUESTIONS 3.

Question 5 Question 6

In a college student’s civil suit for personal In a lawsuit over ownership of stereo equip-
injuries against her stepfather, arising from acts ment between two roommates, the plaintiff
of sexual abuse allegedly committed by him testified that he purchased the equipment one
against her when she was a minor, the student year ago for his DJ business. The defendant
calls as a witness a police officer who will testify offers the testimony of a clerk at the electronics
that, 11 years ago, the stepfather confessed store where the equipment was purchased. The
to the witness that he had committed the acts clerk testifies that she remembers that the sale of
complained of by the student. the equipment occurred 10 months ago, not one
year ago, because the purchase order was signed
Should the trial court admit the police with such a distinctive signature. However, she
officer’s testimony over the stepfather’s objec- cannot remember whether it was purchased by
tion? the defendant or the plaintiff.

(A) Yes, because past instances of misconduct If the plaintiff objects to this testimony,
may be used to impeach a witness. should the trial court admit it?

(B) Yes, because the stepfather’s confession is a (A) Yes, because the purchase order is a past
statement of a party-opponent. recollection recorded.

(C) No, because the best evidence of a convic- (B) Yes, because the purchase order is a
tion is the judgment of the court that business record.
convicted him.
(C) No, because the date of purchase is a collat-
(D) No, because the sexual assault that is the eral matter.
subject of the evidence is more than 10
years old. (D) No, because the content of the purchase
order is hearsay not within any exception.
4. EVIDENCE - IMPEACHMENT QUESTIONS

Question 7 Question 8

The plaintiff is suing the defendant for injuries In an action for breach of contract, which of
he suffered when his car was struck by the the following questions, if asked of a witness for
defendant's truck, allegedly because the defen- the plaintiff on cross-examination by counsel
dant had fallen asleep at the wheel after driving for the defendant, would most probably be ruled
all night. At trial, the defendant's girlfriend improper by the trial court?
testified that she had been with the defendant in
the truck and had taken over the driving duties (A) “Didn’t you use your employer’s computer-
for several hours that night while the defen- ized accounting system to divert company
dant napped. The plaintiff calls to the stand an funds to your personal bank account?”
acquaintance of the defendant's girlfriend, to
testify that the girlfriend told him that she had (B) “As a creditor of the plaintiff, isn’t it true
been unable to get out of bed the weekend the that you stand to gain several thousand
accident occurred because of severe back pain. dollars if the plaintiff prevails in this
lawsuit?”
The testimony of the acquaintance is:
(C) “Isn’t it true that you are currently a
(A) Admissible for impeachment purposes resident of the city detoxification hospital
only. recovering from addiction to Valium?”

(B) Admissible for impeachment purposes and (D) “Weren’t you convicted of forgery six years
as substantive evidence as a declaration of ago?”
physical condition.

(C) Inadmissible, because this means of


impeachment can be done only through
cross-examination.

(D) Inadmissible, because the plaintiff has not


first given the girlfriend an opportunity to
explain or deny the statement.
EVIDENCE - IMPEACHMENT QUESTIONS 5.

Question 9 Question 10

A vacant lot in a downtown area was A defendant is charged with having been one
condemned by the city council, which planned to of two men who robbed a bar and its patrons
establish a riverfront park on the site. During a at gunpoint at 5:30 p.m. on December 16. The
proceeding to establish condemnation damages, defendant calls a witness in his defense who
counsel for the city called the lot owner to the testifies that she drove to the defendant’s home
stand, expecting him to say that he had been at 10 a.m. on December 16 and picked up the
offered $250,000 for the property one month defendant and his wife, then took them to a
before it was condemned. However, on direct birthday party that lasted until 7 p.m. The prose-
examination, the lot owner testified that he had cutor asks on cross-examination, “What is your
received a $400,000 offer for his land a month relationship to the defendant’s wife?” Defense
before condemnation. Counsel for the city then counsel objects.
seeks to confront the lot owner with his state-
ment made at his deposition that the offer was How should the court respond?
for $250,000.
(A) Overrule the objection, because the ques-
The statement contained in the deposition is: tion attacks the witness’s truth and veracity.

(A) Inadmissible, because the city cannot im- (B) Overrule the objection, because the
peach its own witness. question is directed at discovering possible
bias in the witness.
(B) Inadmissible, because it is hearsay not
within any exception. (C) Sustain the objection, because the question
seeks to elicit irrelevant information.
(C) Admissible for impeachment and as
substantive evidence that the lot owner (D) Sustain the objection, because the answer
received an offer for $250,000. to the question would create prejudice that
would outweigh its probative value.
(D) Admissible, but only for the purpose of
impeachment.
6. EVIDENCE - IMPEACHMENT QUESTIONS

Question 11 Question 12

A pedestrian sued the defendant for injuries A decedent’s executor filed a wrongful death
suffered in a hit and run accident. The car that suit against a railroad company whose freight
struck the pedestrian was linked to the defen- train collided with the decedent’s automobile,
dant, but she denies that she was driving the car killing him instantly. After the accident it was
when the pedestrian was hit. The only eyewit- impossible for experts to tell whether one of the
ness, other than the pedestrian, was a six-year- crossing gates was broken because a vehicle
old child. During the trial, the pedestrian put the drove through it, or if it broke while being
child on the stand, and he testified that he saw lowered suddenly on top of a vehicle entering
the defendant driving the car that hit the pedes- the grade crossing. A motorist who did not see
trian. The defense elected not to cross-examine the accident but who arrived on the scene just
the child. The plaintiff’s next witness was a seconds later heard a pedestrian standing near
child psychologist who had never met the child the grade crossing exclaim, “That gate didn’t
before but was prepared to testify that, based come down on time!” The pedestrian collapsed
on his observations of the child on the witness after making his statement. An emergency
stand, it was highly probable that the child was medical technician tended to the pedestrian.
telling the truth. The psychologist was qualified She told the pedestrian that he had probably
as an expert witness with many years of experi- suffered a mild heart attack. The pedestrian
ence dealing with children. However, the defense told the medical technician, “Well, I guess I’m a
objected to any questions regarding the veracity lot luckier than that poor fool who rammed his
of the child. car right through that crossing barrier into the
train.” Two months later, the pedestrian suffered
Should the psychologist be allowed to testify another heart attack and died. Four months later,
about the child? the executor’s suit came to trial. The plaintiff
called the motorist to the stand, and the court
(A) Yes, because parties may put witnesses allowed him to testify to what he heard the
on the stand to reinforce or challenge the pedestrian say after the accident. The railroad’s
veracity of other witnesses. attorney wishes to call the medical technician to
the stand to testify as to the pedestrian’s state-
(B) Yes, because an expert may be needed to ment when she was treating him.
explain how children testify in order to
avoid confusing the jury. How should the court rule on the medical
technician’s testimony?
(C) No, experts may not testify regarding
veracity. (A) Admissible, but solely for the purpose of
impeachment.
(D) No, because the child’s veracity has not yet
been questioned. (B) Admissible, both for impeachment purposes
and as evidence of the positioning of the
crossing gate at the time of the accident.

(C) Inadmissible, because the pedestrian is not


available to be questioned about the incon-
sistent statements.

(D) Inadmissible, as hearsay not within any


recognized exception to the hearsay rule.
EVIDENCE - IMPEACHMENT QUESTIONS 7.

Question 13 Question 14

The plaintiff sued the defendant in an automo- The defendant was on trial for armed robbery.
bile collision case. At trial, the plaintiff wishes The defense placed the defendant’s friend on
to show by extrinsic evidence that the defen- the stand as an alibi witness. The friend testi-
dant’s primary witness is the defendant’s partner fied that, at the time of the armed robbery, the
in a gambling operation. defendant was engaged in other activities with
the friend. On cross-examination, the prosecutor
Is this evidence admissible? asked the friend, “Isn’t it true that the grand jury
has indicted you for the same crime as the defen-
(A) Yes, as evidence of the witness’s character. dant?” The defendant’s lawyer knew that the
friend had been indicted for the armed robbery
(B) Yes, as evidence of the witness’s possible and was to be tried separately in two weeks,
bias in favor of the defendant. but she vigorously objected to the prosecutor’s
question.
(C) No, because criminal conduct can be shown
only by admission or record of conviction. Should her objection be sustained?
(D) No, because bias must be shown on cross- (A) Yes, because the friend has been indicted,
examination and not by extrinsic evidence. but not convicted, of the armed robbery.

(B) Yes, because prior bad acts may not be


introduced to impeach a witness.

(C) No, because the friend’s indictment tends to


show his criminal propensities.

(D) No, because the friend’s indictment tends to


show bias on the part of the witness.
EVIDENCE -
IMPEACHMENT
ANSWERS
EVIDENCE - IMPEACHMENT ANSWERS 1.

EVIDENCE - IMPEACHMENT ANSWERS

Answer to Question 1
(D) The officer may not testify about the embezzlement because it constitutes impeachment by extrinsic
evidence of a specific instance of misconduct. A specific act of misconduct offered to attack the
witness’s character for truthfulness can be elicited only on cross-examination. If the witness denies
the act, the cross-examiner cannot refute the answer by calling other witnesses or producing other
evidence. Because the alleged embezzlement is admissible, if at all, only as impeachment evidence,
when the defendant denied it the prosecutor could not call the officer to testify. (A) is wrong
because extrinsic evidence, such as the officer’s testimony, of an instance of misconduct is not
admissible. (B) is wrong because when a person is charged with one crime, extrinsic evidence of
other crimes or misconduct is inadmissible to establish criminal disposition. Because nothing in the
facts indicates that such evidence is being offered to prove something other than disposition (e.g.,
motive, identity, common plan or scheme), the officer’s testimony is not admissible as substantive
evidence. As discussed above, for impeachment, the prosecutor is limited to inquiry on cross-
examination regarding the embezzlement. (C) is wrong because, even if the defendant had been
convicted of the embezzlement, the officer’s testimony would not be the proper way to introduce
it. The fact that a witness has been convicted of a crime usually is proved by eliciting an admission
on direct or cross-examination or by the record of conviction. A judge is less likely to allow proof
of conviction by testimony because it may be too time-consuming and more prejudicial than other
means of proof. Furthermore, this answer choice implies that evidence of a prior act of misconduct
may be introduced only if the act resulted in a conviction, and this is not the case.
Answer to Question 2
(B) The court should allow the prosecutor’s question because it is appropriate to show bias or interest
on the part of the witness. Evidence that a witness is biased or has an interest in the outcome of
the case tends to show that the witness has a motive to lie. Bias or adverse interest can be proved
by cross-examination or extrinsic evidence, and in some cases, both. Here, the fact that the
witness is being prosecuted for the same crime tends to show that he has a motive to lie in saying
that the fire started accidentally. Thus, it is proper impeachment for the prosecutor to cross-
examine the witness about his own prosecution. (A) is wrong because all witnesses are subject to
impeachment, and evidence (including character evidence) that bears on truthfulness is always
admissible (although the means of proof may be restricted). This choice confuses the basis for
the prosecution’s introduction of substantive character evidence against a defendant in a criminal
trial with valid impeachment of a witness for bias or interest. Nor would it matter if the defense
lawyer’s evidence of the witness’s good character was admitted erroneously (because generally a
witness’s testimony cannot be bolstered until the witness has been impeached). Regardless of the
defense’s introduction of that evidence, a witness may always be impeached by extrinsic evidence
of bias. (C) is wrong because conviction of a crime is not a requisite for introduction of evidence
showing bias or interest. A felony conviction or a conviction for a crime involving dishonesty is
an entirely separate method of impeachment. (D) is wrong for two reasons: (i) the answer to the
question could in no way incriminate the witness; and (ii) even if the answer could tie the witness
to the commission of the crime, he could invoke the privilege to refuse to answer—the question
itself would not be objectionable.
Answer to Question 3
(A) The court should find the record admissible. A witness other than the accused may be impeached
by (i) any felony conviction (unless the judge determines that its probative value is substantially
2. EVIDENCE - IMPEACHMENT ANSWERS

outweighed by Rule 403 considerations) or (ii) conviction of any other crime requiring proof or
admission of an act of dishonesty or false statement. [Fed. R. Evid. 609(a)] Embezzlement is
both. The fact that an appeal is pending does not affect the admissibility of the conviction; thus,
(B) is incorrect. The official record of judgment is always admissible proof that the judgment was
entered; (C) is therefore incorrect. While the proposition in (D) is correct, it does not apply to
proof of prior convictions. A prior conviction may be shown not only by direct or cross-examina-
tion of the witness but also by introducing a record of judgment.

Answer to Question 4

(B) Under the Federal Rules, a witness may be impeached by a record of conviction of any crime
(felony or misdemeanor) if it can be readily determined that conviction of the crime required
proof or admission of an act of dishonesty or false statement. A witness also may be impeached
by any felony, whether or not it involves dishonesty or a false statement. In a criminal case in
which the accused is being impeached, a felony conviction that does not involve dishonesty or a
false statement will be admitted only if the government shows that its probative value as impeach-
ment evidence outweighs its prejudicial effect. Here, the evidence the prosecution is seeking to
introduce is a misdemeanor that does not involve dishonesty or a false statement; thus, (B) is
correct and (A) is incorrect. (C) is a correct statement of law—a conviction is generally admissible
if it is less than 10 years old—but it is not as good an answer as (B). (D) is incorrect because, as
discussed above, a witness may not be impeached by any crime.

Answer to Question 5

(B) The officer’s testimony should be admitted because the stepfather’s confession is a statement by
a party-opponent. Generally, evidence is admissible if it is relevant, i.e., if it has a tendency to
prove any fact of consequence to the action. The testimony here is relevant because it makes it
more likely that the defendant committed the acts complained of. It is also competent because its
admission does not violate any exclusionary rule of evidence. Evidence law generally prohibits
admission of hearsay—out-of-court statements offered to prove the truth of the matter asserted—
but the rule does not apply to statements by party-opponents, which are treated as nonhearsay
under the Federal Rules. Here the stepfather’s statement is being offered against him by the
student. Thus, the testimony is admissible nonhearsay and (B) is correct. (A) may be a true
statement—past instances of misconduct may be used to impeach a witness, at least if they are
probative as to whether he is worthy of belief—but it is an incorrect answer because it focuses
on impeachment, and the defendant is not being impeached here. (C) is incorrect because the
best evidence rule is not applicable here. The rule requires that when the contents of a writing
are sought to be proved, the writing itself should be admitted if it is available. Here, however, the
witness is not trying to prove that the defendant has been convicted, but rather that he made the
admission. (D) is incorrect for reasons similar to why (C) is incorrect. It is true under the Federal
Rules that convictions generally are not admissible for impeachment of a witness if more than 10
years old, but the officer is not trying to admit the defendant’s conviction here (and is not trying to
impeach the defendant, who may not even have testified). Rather, the student is seeking to admit
the defendant’s confession. The confession is an opposing party’s statement and is admissible as
substantive evidence.

Answer to Question 6

(C) Testimony as to the date of purchase should not be admitted because its minimal relevance is
substantially outweighed by considerations of waste of time and confusion of the issues. The
EVIDENCE - IMPEACHMENT ANSWERS 3.

actual date of purchase has no bearing on the issue of ownership, but instead is offered to cast
doubt on the plaintiff’s credibility. However, when a witness makes a statement not directly
relevant to the issues in the case, the rule against impeachment on a collateral matter bars his
opponent from proving the statement untrue either by extrinsic evidence or by a prior inconsis-
tent statement. As noted previously, the clerk’s statement as to the date on which the defendant
purchased the equipment is not directly relevant to any other issue in the case. Thus, (C) is
correct. (A) is incorrect. The past recollection recorded exception to the hearsay rule permits a
party to offer into evidence a writing made by a witness whose memory cannot be refreshed. The
purpose of the writing would be to prove the facts contained therein to which the witness cannot
testify due to insufficient memory. Here, the clerk has not stated that she has insufficient recollec-
tion of the events to which she is testifying, she is simply referring to the purchase order due to
the unusual signature. Additionally, the defense is not attempting to introduce the purchase order
into evidence. Thus, the purchase order does not constitute a past recollection recorded. (B) is
incorrect for the same reason. A record of any act, transaction, occurrence, or event is admissible
as proof of that act, transaction, occurrence, or event if made in the regular course of business.
Here, the purchase order is not being offered into evidence and a passing reference to it does not
affect the clerk’s testimony. (D) is incorrect because the clerk is not testifying to the content of
the purchase order. Rather, she is simply testifying to the date of purchase from her own memory.
Thus, the hearsay nature of the contents of the purchase order is not at issue.

Answer to Question 7

(A) The testimony of the acquaintance is admissible, but only for impeachment purposes. For the
purpose of impeaching the credibility of a witness, a party may show that the witness has, on
another occasion, made statements that are inconsistent with some material part of her present
testimony. Here, the acquaintance is prepared to testify that the girlfriend stated that she had been
unable to get out of bed because of severe back pain at the time that she testified that she was with
the defendant and had taken over the driving, a fact that would be material to the allegations in
the lawsuit. Thus, the testimony is properly admissible for purposes of impeachment. (C) is incor-
rect because, under the Federal Rules, an inconsistent statement may be proved by either cross-
examination or extrinsic evidence. (D) is incorrect because, while extrinsic evidence is admissible
only if the witness is, at some point, given an opportunity to explain or deny the allegedly incon-
sistent statement, the opportunity need not come before the introduction of the statement under
the Federal Rules; the testimony may be admitted now and the girlfriend subsequently be given
an opportunity to explain or deny it. (B) is incorrect because the testimony is hearsay that is
not admissible for substantive purposes because it does not fall under the exception for declara-
tions of physical condition. Under the Federal Rules, declarations of present bodily condition are
admissible as an exception to the hearsay rule when made to anyone, not just a physician, whereas
declarations of past physical condition are admissible as a hearsay exception only if made to
assist in diagnosing or treating the condition. Here, the girlfriend's statement to the acquaintance
pertained to her past physical condition, and there is no indication that the statement was made to
assist in diagnosis or treatment. Hence, the girlfriend's statement does not fall within these or any
other exceptions to the hearsay rule, and is admissible for impeachment purposes only.

Answer to Question 8

(C) Federal Rule of Evidence 608(b) permits cross-examination as to prior bad acts if, in the discre-
tion of the trial court, they are probative of truthfulness. Thus, (A) is wrong. (D) is wrong because
Federal Rule 609 permits inquiry into a prior crime less than 10 years old if conviction of the
crime required proof or admission of an act of dishonesty or false statement. Because the bias
4. EVIDENCE - IMPEACHMENT ANSWERS

involved in a pecuniary interest is always relevant and admissible, (B) is wrong. Only the question
relating to drug abuse—a marginally relevant topic, relating possibly to the witness’s ability to
perceive or remember—appears improper.

Answer to Question 9

(C) Under the Federal Rules, depositions can be used more broadly than under the rules in many
states. Here, the prior inconsistent statement can be used for impeachment purposes and as
substantive evidence. (A) is wrong because the Federal Rules permit impeachment of your own
witness. (B) is wrong because under the Federal Rules, prior inconsistent statements, made under
oath and subject to the penalty of perjury in a deposition or prior hearing, are admissible as
nonhearsay. (D) is wrong because the statement can be used for more than impeachment.

Answer to Question 10

(B) The prosecutor’s question is aimed at discovering bias, which tends to show that the witness has
a motive to lie. Impeachment involves the casting of an adverse reflection on the veracity of a
witness, and it may take several forms. Evidence that a witness is biased tends to show that she
has a motive to lie, and is thus a well-recognized method of impeachment. Inferences of bias
may be shown by evidence of family or other relationship. Here, the prosecutor is attempting to
show that, due to a family relationship or friendship with the defendant’s wife, the witness may be
biased and would thus have a motive to lie on behalf of the defendant. Therefore, the prosecutor
is engaging in an accepted method of impeachment. (A) correctly states that the prosecutor’s
question attacks the witness’s truth and veracity. However, not all methods of attacking a witness’s
truth and veracity are admissible. (B) is a better answer than (A) because it identifies the specific
method of impeachment that the cross-examiner is using. (C) is incorrect because evidence that
tends to prove or disprove the credibility of a witness is relevant. The information sought to be
elicited by the prosecutor’s question will reflect on a possible motive that the witness may have to
lie. Therefore, such information is relevant. (D) incorrectly states that the answer to the prosecu-
tor’s question would create prejudice that would outweigh its probative value. Under Federal Rule
403, evidence may be excluded if its probative value is substantially outweighed by the danger
of unfair prejudice, confusion of the issues, or misleading the jury. While any material evidence
introduced by a party will probably be prejudicial to the adverse party’s case, it is only unfair
prejudice (i.e., suggesting a decision on an improper basis) that may be excluded under this rule.
Here, the answer to the prosecutor’s question would clarify the matter of whether the witness had
a motive to lie. This would not be unfairly prejudicial because it would tend to make the witness’s
testimony as to the defendant’s whereabouts at the time of the alleged crime more or less believ-
able, which is the proper basis on which the trier of fact should accept or reject her testimony.

Answer to Question 11

(D) The psychologist should not be allowed to testify because the child’s credibility has not been
impeached. Evidence of a witness’s character is admissible only after the witness’s character
for truthfulness has been attacked. [Fed. R. Evid. 608(a)] The psychologist’s testimony would
have been admissible if the child’s veracity had been attacked on cross-examination, but here
the defense did not cross-examine the child. (A) is incorrect because the rule for reinforcing a
witness’s veracity differs from the rule for challenging it. A party may put witnesses on the stand
to bolster another witness’s credibility only when the witness’s credibility has been attacked. (B)
is incorrect because there is no indication that the child’s ability to testify is in issue or that his
testimony may confuse the jury. Federal Rule 702 provides that expert testimony is admissible if
EVIDENCE - IMPEACHMENT ANSWERS 5.

the subject matter is one where scientific, technical, or other specialized knowledge would assist
the trier of fact in understanding the evidence or determining a fact in issue, and Rule 703 permits
an expert’s opinion to be based on observations made at trial. However, evaluating the reliability
of an eyewitness’s identification is traditionally within the province of the jury. Since the court has
determined that the child is competent to testify, the jury has the ability to decide on its own what
weight to give his testimony. (C) is incorrect because it is too broad. While courts generally allow
a jury to evaluate a witness’s veracity on its own, expert testimony would be allowed if the child’s
credibility had been attacked on cross-examination or by other witnesses on the ground that he
was too young to testify truthfully. Since this is an area where an expert’s specialized knowledge
would help the jury in evaluating the child’s testimony, the psychologist probably would have been
allowed to testify.

Answer to Question 12

(A) The medical technician’s testimony is admissible, but solely for purposes of impeaching the
pedestrian, a hearsay declarant. The pedestrian’s statement to the motorist, even though hearsay,
was admissible as an excited utterance or present sense impression because it was made immedi-
ately after the crash. Under Federal Rule 806, the credibility of a hearsay declarant may be
attacked by evidence that would be admissible if the declarant had testified as a witness. For the
purpose of impeaching the credibility of a witness, a party may show that the witness has, on
another occasion, made statements that are inconsistent with some material part of his present
testimony. The pedestrian’s statement to the motorist may therefore be impeached by proof that
he made the inconsistent statement to the medical technician. (B) is incorrect. The pedestrian’s
statement to the medical technician is hearsay because it is an out-of-court statement offered to
prove the truth of the matter asserted, i.e., that the decedent rammed the crossing barrier. [Fed.
R. Evid. 801(c)] If a statement is hearsay, and no exception to the rule is applicable, the evidence
is inadmissible for substantive purposes. [Fed. R. Evid. 802] The pedestrian’s statement does
not come within either the excited utterance exception or the present sense impression exception
to the hearsay rule because the statement to the medical technician was not made immediately
after the accident. Nor does his statement fall under Rule 801(d)(1)(A), which provides that where
a prior inconsistent statement was made under oath at a prior trial, hearing, other proceeding,
or deposition, it is admissible nonhearsay (i.e., it may be considered as substantive proof of the
facts stated). Because the pedestrian’s statement to the medical technician was not made under
oath, it is hearsay that may only be used to impeach and not as evidence of the position of the
crossing gate. (C) is incorrect because a hearsay declarant, such as the pedestrian, does not have
to be given an opportunity to explain or deny his alleged prior inconsistent statement. Gener-
ally, extrinsic evidence of the prior inconsistent statement of a witness is inadmissible unless the
witness was examined so as to give him an opportunity to explain or deny the alleged inconsistent
statement, but this foundation requirement may be dispensed with where “the interests of justice
otherwise require.” [Fed. R. Evid. 613(b)] The courts generally agree that inconsistent statements
by hearsay declarants may be used to impeach despite the lack of foundation. [Fed. R. Evid.
806] Therefore, the medical technician’s testimony may be admitted even though the pedestrian
is not available to be questioned about the inconsistent statements. (D) is incorrect because the
pedestrian’s statement to the medical technician, although hearsay, may be used for impeach-
ment purposes. As discussed above, a hearsay declarant may be impeached by evidence of his
prior inconsistent statements. The pedestrian’s statement to the motorist was properly admitted
hearsay, which can be impeached by evidence of his inconsistent statement to the medical techni-
cian. However, because the statement to the medical technician is hearsay not within an excep-
tion to the hearsay rule, it may be used only for impeachment purposes and not as evidence of the
position of the crossing gate.
6. EVIDENCE - IMPEACHMENT ANSWERS

Answer to Question 13

(B) The evidence is admissible as evidence of the witness’s possible bias in favor of the defendant.
Evidence that a witness is biased tends to show that the witness has a motive to lie. A witness
may always be impeached by extrinsic evidence of bias, provided a proper foundation is laid. Bias
may be shown by evidence of a business relationship or friendship with a party. Here, the fact
that the witness is the defendant’s partner in a gambling operation is admissible to impeach the
witness for bias. (A) is wrong because the only relevant facet of the witness’s character in this case
is his veracity, and extrinsic evidence of bad acts is not admissible to impeach a witness. Under
the Federal Rules, instances of a witness’s conduct may be inquired into on cross-examination
if they are probative of truthfulness (i.e., acts of deceit or lying), but extrinsic evidence is not
allowed. It is doubtful that running a gambling operation would be found to be probative of the
witness’s truthfulness. Even if it were, extrinsic evidence would be inadmissible. (C) misstates
the law. Criminal conduct can be shown by means other than admission or record of conviction.
As discussed above in (A), a witness may be impeached by instances of bad or criminal conduct
(even if it does not result in a conviction) if it is probative of truthfulness and it is brought out
on cross-examination (no extrinsic evidence). (D) is wrong because extrinsic evidence of bias is
admissible. (Note, however, that many states require that the bias be inquired into on cross-exami-
nation before extrinsic evidence is admissible.)

Answer to Question 14

(D) The prosecutor’s question was a proper means of impeaching the friend’s credibility. A witness
may be impeached through cross-examination by evidence that he is biased or has an interest in
the outcome of the case; i.e., evidence that shows that he has a motive to lie. Since the friend has
been indicted for the same armed robbery, it clearly helps his case for him to provide an alibi
for both of them. Furthermore, if the friend were to testify that he and the defendant committed
the robbery, his testimony could be used against him at his own trial as an admission. Thus, the
friend has a motive to lie and may be impeached by being asked on cross-examination about
the facts that show bias, as the prosecutor did here. (A) is incorrect because the prosecutor is
impeaching the friend on the basis of bias, not on the basis of poor character for truthfulness. For
the latter basis of impeachment, Federal Rule 609 requires a criminal conviction rather than just
an indictment (unless it was for a crime probative of truthfulness, such as perjury, which could be
the subject of cross-examination under Rule 608(b) even if there has not yet been a conviction).
Thus, the friend could not be impeached by being asked about an indictment (and (A) would then
be the correct choice) if the friend had been under indictment for a robbery other than the same
robbery for which the defendant is being tried (since it would not show a motive to lie for the
defendant). (B) misstates the law. A witness may be impeached by interrogating him as to specific
acts of misconduct that may affect his character and show him to be unworthy of belief. [See Fed.
R. Evid. 608(b)] (C) is incorrect because evidence is never admissible to show a person’s criminal
propensities. For impeachment purposes, the prosecution wants to attack the witness’s credibility
rather than his criminal propensity. The indictment, as opposed to the crime charged in the indict-
ment, generally shows nothing with respect to the witness’s character for truthfulness. Thus, to
impeach this witness on a ground other than bias, the prosecutor would ask about the robbery
itself, not the indictment.
EVIDENCE -
PRESENTATION OF
TESTIMONIAL EVIDENCE
QUESTIONS
EVIDENCE - PRESENTATION OF TESTIMONIAL EVIDENCE QUESTIONS 1.

EVIDENCE - PRESENTATION OF TESTIMONIAL EVIDENCE QUESTIONS

Question 1 Question 2

The plaintiff sued the defendant for damages Which of the following questions is most
and injuries after the defendant’s car struck the likely to be disallowed in response to an objec-
plaintiff’s car in an intersection. The plaintiff tion that it is leading?
wanted a witness who lived on the corner and
had seen the accident to testify on his behalf. (A) On direct examination, counsel asks his
However, the witness’s recollection of the own client: “You live at 555 Northward
accident was very fuzzy because the trial did Avenue, don’t you?”
not take place until almost three years after the
accident. The night before she was scheduled to (B) On cross-examination of an expert witness,
testify, the witness consulted her diary, in which counsel asks: “Isn’t it true that most family
she had noted at the time that the defendant’s practice attorneys routinely asserted a
car did not come to a complete stop at the stop community interest in military pensions at
sign and entered the intersection, striking the that time?”
car driven by the plaintiff after the plaintiff’s car
had already entered the intersection. The plain- (C) On direct examination of the victim, a four-
tiff’s attorney called the witness to the stand, and year-old child, the prosecutor asks: “Did the
she testified to those details. defendant touch you there?”

On cross-examination, the defendant’s (D) On direct examination of an eyewitness to


attorney asked the witness if she had consulted an accident, plaintiff’s counsel asks: “Was
any materials to prepare for her testimony. the light red when the defendant drove
The witness admitted that her recollection of through the intersection?”
the accident had been fuzzy and that she had
consulted her diary the night before her testi-
mony. The defendant’s attorney immediately
moved that the witness’s testimony be stricken
from the record.

How should the court rule?

(A) The witness’s testimony is admissible, be-


cause, after reviewing her notes, she had an
independent recollection of the event.

(B) The witness’s testimony is admissible,


because the contents of her diary are
protected under the work product rule.

(C) The witness’s testimony should be stricken,


because her diary was not made available to
the opposing party prior to trial.

(D) The witness’s testimony should be stricken,


because it is not the best evidence.
2. EVIDENCE - PRESENTATION OF TESTIMONIAL EVIDENCE QUESTIONS

Question 3 Question 4

A homeowner was helping his neighbor clean A beekeeper sued the manufacturer of a
out her garage. As the homeowner and the specially designed helmet and faceguard that
neighbor were moving some boards out of the was designed to protect the wearer from bee
garage, the neighbor ran into the overhead door’s stings. The beekeeper alleged that he was
hinge, causing it to fall and hit the homeowner wearing the helmet and faceguard when he
on his head, killing him. The homeowner’s was stung on the left cheek by a bee that had
wife sued the neighbor for the wrongful death entered through a small tear in the faceguard,
of her husband, seeking damages of $100,000. and that, one hour later, the beekeeper suffered a
The wife, who was present when the accident heart attack. The beekeeper’s physician, having
occurred, is called to testify that at the time of listened to the beekeeper testify to the foregoing
the accident, the homeowner was carrying the facts at trial, is called by the beekeeper and
boards on his shoulder, and he therefore was asked whether, assuming the truth of such testi-
unable to see that the neighbor had hit the door mony, the beekeeper’s subsequent heart attack
hinges. could have resulted from the bee sting.

Is the wife competent as a witness? The physician’s opinion is:

(A) Yes, if she is testifying as the personal rep- (A) Admissible, because the physician was the
resentative of her husband’s estate. beekeeper’s treating physician.

(B) Yes, in spite of the fact she is the plaintiff. (B) Admissible, as a response to a hypothetical
question.
(C) No, because she is unqualified to give
opinion evidence. (C) Inadmissible, because a proper foundation
has not been laid.
(D) No, because she cannot testify for her
husband in a civil case. (D) Inadmissible, because an expert’s opinion
may not be based solely on information
provided by laypersons.
EVIDENCE - PRESENTATION OF TESTIMONIAL EVIDENCE QUESTIONS 3.

Question 5 Question 6

In a suit between the drivers of two cars A plaintiff brought suit against a manufac-
involved in a collision, the plaintiff’s attorney turing company, seeking to recover for damages
calls a witness to the stand. On direct examina- he suffered when his car’s engine burst into
tion, the witness testifies that, while walking flames following the use of an engine additive
down the street, she heard a horn sounding, made by the company. The plaintiff contends
looked up, and saw two cars enter an intersec- that the manufacturing company was negligent
tion and collide. On cross-examination, the and in breach of warranty.
defense attorney goes over the same ground
with the witness. He asks her whether there was An automobile engineer sat in court while
any broken glass on the pavement, to which she the plaintiff testified to the events concerning
responds, “Yes, lots of it,” and before the defense the engine fire. The plaintiff’s testimony was
attorney can ask his next question, the witness not challenged or rebutted. The plaintiff calls
blurts out, “They had to be going over 50!” The the engineer to the stand and asks him whether,
defense attorney moves to strike the statement. based on the plaintiff’s prior testimony, it was
possible for a car engine to burst into flames as
How should the court rule? it did.

(A) Strike it, as unresponsive to any question Would the engineer’s testimony be admis-
asked. sible?

(B) Strike it, because the witness had no way of (A) No, because the engineer’s opinion was not
knowing how fast the cars were traveling. elicited by means of a hypothetical ques-
tion.
(C) Not to strike, because the defense attorney
“opened the door” to anything the witness (B) No, because the engineer was in the court
might say about the accident. while the plaintiff testified concerning the
engine fire.
(D) Not to strike, because the statement accuses
both drivers of going over 50, and is not (C) Yes, because the engineer was in the court
prejudicial to only one side. while the plaintiff testified concerning the
engine fire.

(D) Yes, as long as the engineer’s opinion is


based only on admissible evidence.
4. EVIDENCE - PRESENTATION OF TESTIMONIAL EVIDENCE QUESTIONS

Question 7 Question 8

A woman who was hospitalized with a serious A boy and his parents sued a driver for
case of salmonella poisoning after eating some $75,000 for injuries they claim were caused
chicken sued the grocery store that sold the when the driver’s car hit the boy one night when
chicken and the chicken plant that processed the boy was out delivering papers. The boy was
it. At trial, the woman called a biochemist and knocked unconscious in the accident, and the
asked him whether, in his opinion, based on driver claims that it was not his car that hit the
laboratory reports made concerning her illness boy. Except for damages, the main issue in the
and tests on the remainder of the chicken in suit is whether it was the driver’s car that hit the
her freezer, it was possible that the chicken was boy. The driver’s own attorney asks him, “Could
the source of the salmonella. The reports are the boy have mistaken your car for another?”
relied on by biochemists, and the biochemist had
the reports with him. Counsel for the chicken Is this question objectionable?
production plant objected to the biochemist’s
testimony on this point. (A) Yes, because the answer would be hearsay.

The court should rule that the testimony is: (B) Yes, because the answer would be an
opinion.
(A) Admissible, because such reports are rea-
sonably relied upon by biochemists in the (C) No, because the answer would be relevant
course of their profession. to the issue of whose car hit the boy.

(B) Admissible, because the biochemist has the (D) No, if a proper foundation has been laid.
reports with him and can offer them into
evidence.

(C) Inadmissible, because the biochemist’s


testimony embraces the ultimate issue in
the case.

(D) Inadmissible, because the contents of the


laboratory reports are hearsay and not
admissible under any exception.
EVIDENCE - PRESENTATION OF TESTIMONIAL EVIDENCE QUESTIONS 5.

Question 9 Question 10

While driving east on a two-lane highway, a A plaintiff sued a defendant for personal
pickup truck collided with a westbound motor- injuries sustained when the plaintiff slipped
cycle. The accident was observed by a cab driver. and fell on the floor in the defendant’s office.
Because the cab driver had a passenger, she did The plaintiff called a witness to testify that he
not stay at the scene of the accident but made was on duty in the hospital emergency room
some notes shortly thereafter. She made her when the plaintiff was admitted and that he saw
statement to the police the next day, in which she a doctor treat the plaintiff’s skull. As he was
stated that just before the accident, the pickup getting ready to testify, the witness refreshed his
swerved into the westbound lane, hitting the recollection by studying the plaintiff’s copy of
motorcycle. At the trial of the motorcyclist’s the hospital records. These records had not been
personal injury action against the truck driver, admitted into evidence.
the cab driver was having difficulty remem-
bering some of the facts. The motorcyclist’s Is the witness’s testimony concerning the
attorney sought to let her review the notes she treatment admissible?
had made.
(A) Yes, as evidence of the extent of the plain-
Should the court permit the cab driver to tiff’s injury.
review her notes?
(B) Yes, as past recollection recorded.
(A) Yes, because it is a present recollection
refreshed. (C) No, because it is not the best evidence.

(B) Yes, because it is a past recollection (D) No, because it is based on hearsay not
recorded. within any exception.

(C) No, because the cab driver is required to


report all accidents to her employer, and the
record is thus privileged.

(D) No, because there is no showing that the


cab driver had used her notes when she
gave her statement to the police officer.
6. EVIDENCE - PRESENTATION OF TESTIMONIAL EVIDENCE QUESTIONS

Question 11 Question 12

A plaintiff was injured when a portion of a An expert witness, who has an advanced
spiral stairway in a shopping mall collapsed. degree in engineering, is testifying at a jury
The plaintiff filed suit against both the owners trial to the possible causes for the failure of an
of the mall and the designers of the staircase. enclosed pedestrian bridge in a shopping mall.
At the trial of the case, the plaintiff wishes
to call a highly qualified civil engineer to the Which of the following data, if relied on by
stand to testify as an expert witness. The expert the expert witness in forming his opinion, is
is prepared to testify that the spiral staircase admissible on direct examination as substantive
was improperly designed, and the design defect evidence?
caused a portion of the staircase to collapse
under the plaintiff. The expert’s proposed testi- (A) Statements told to him by witnesses to the
mony is based in part upon a series of photo- collapse, as long as such statements are
graphs taken by a structural engineer hired by reasonably relied on by experts in his field.
the expert immediately after the expert was
engaged by the plaintiff’s attorney, and the (B) Statements that he reads from a text on
accompanying report by the structural engineer. structural engineering that he has testified
It is customary for civil engineers to form is authoritative.
professional opinions based on these reports.
The photographs are of the collapsed stairway (C) Testimony regarding the repair of the bridge
and of an identical stairway located in another one week after the collapse, indicating
part of the mall. Neither the photographs nor the that it is now equipped with special safety
report has been admitted into evidence, but the features that were not installed prior to the
expert is willing to disclose to the jury the facts collapse.
on which he relied in forming his opinion.
(D) Statements made to him by a former
Should the expert’s testimony be ruled admis- maintenance employee of the shopping mall
sible? indicating that the management knew the
bridge needed repairs but delayed doing
(A) Yes, because other civil engineers ordinar- so because the busy holiday season was
ily reasonably rely on structural engineers’ approaching.
reports in forming professional opinions.

(B) Yes, because the expert will disclose to


the jury the facts on which he based his
opinion.

(C) No, because the expert based his opinion in


part on the structural engineer’s opinion.

(D) No, because the photographs and report


were commissioned solely for the purpose
of preparing for litigation and were not
admitted into evidence.
EVIDENCE - PRESENTATION OF TESTIMONIAL EVIDENCE QUESTIONS 7.

Question 13 Question 14

In a criminal trial, the prosecution called The key witness in the prosecution’s case
an expert witness to the stand. The prosecutor against the defendant is the only eyewitness
conducted a direct examination of the witness to have seen the commission of the felony for
that lasted one-half hour. The defense attorney which the defendant is being charged. The
cross-examined the witness for three days and witness, a recent immigrant, has a total hearing
told the court that he planned to spend at least impairment and is mute. In addition, the system
another day in cross-examination to develop of “signing” for the deaf is different in the
testimony brought up on direct examination. The witness’s country of origin from the method
prosecutor moved that the cross-examination be used in the United States. The only person in
terminated by the court. the county conversant with the witness’s signing
method is a clerk in the county prosecutor’s
May the court approve the prosecutor’s office. The clerk had assisted the police in their
motion? questioning of the witness prior to the defen-
dant’s arrest, and also when the witness identi-
(A) Yes, unless the testimony is relevant. fied the defendant in a lineup.
(B) Yes, because the defendant had an adequate Should the court allow the witness to testify
opportunity for meaningful cross-examina- using the clerk as an interpreter?
tion.
(A) Yes, because the clerk is qualified.
(C) No, because the testimony relates to the
subject matter of direct. (B) Yes, if the clerk takes an oath to make a
true translation.
(D) No, because in a criminal trial the consider-
ation of judicial economy is outweighed by (C) No, because as a result of her employment
due process. and previous activities, the clerk is inher-
ently biased.

(D) No, unless the clerk discloses to the jury her


employment and previous activities in this
case.
8. EVIDENCE - PRESENTATION OF TESTIMONIAL EVIDENCE QUESTIONS

Question 15 Question 16

An engineer is called as a witness in a profes- The plaintiff filed a personal injury action
sional malpractice action. During direct exami- against a restaurant. The plaintiff alleges that
nation by the attorney for the party who called she suffered food poisoning after consuming the
her, she is asked leading questions. The opposing restaurant’s daily special—extra-strong coffee
party objects. and chili over eggs. The restaurant defends by
claiming that the special on the day in question
Which of the following will have no bearing was an exotic tea served with a yogurt and fruit
on whether the trial court will permit leading blend and that the extra-strong coffee drink was
questions? not even on the menu that day. Counsel for the
restaurant called a former waitress to testify as
(A) The question is necessary to refresh recol- to whether the extra-strong coffee drink was
lection. on the menu that day. The former waitress,
who lives with the plaintiff, testified that it was
(B) The questions relate to preliminary infor- always on the menu. The restaurant’s attorney
mation, such as name, address, etc. then asked, “Didn’t you in fact, at your deposi-
tion, state that the extra-strong coffee drink
(C) The engineer was called as an adverse was not on the menu that day?” The plaintiff’s
witness. attorney objects.
(D) The questions relate directly to an issue of Is the question proper?
ultimate fact in the lawsuit.
(A) Yes, because the former waitress is hostile.

(B) Yes, because it constitutes refreshing the


witness’s recollection.

(C) No, because this is a leading question on


direct examination.

(D) No, because the restaurant cannot impeach


its own witness.
EVIDENCE -
PRESENTATION OF
TESTIMONIAL EVIDENCE
ANSWERS
EVIDENCE - PRESENTATION OF TESTIMONIAL EVIDENCE ANSWERS 1.

EVIDENCE - PRESENTATION OF TESTIMONIAL EVIDENCE ANSWERS

Answer to Question 1

(A) The witness’s testimony is admissible. This question involves testimony by a witness whose
recollection has been refreshed by reference to a document (her diary). Under the Federal Rules,
any materials can be used to refresh one’s recollection, and the Rules do not prohibit the use of
such materials before trial. Thus, (A) is correct. (B) can quickly be discarded as incorrect. The
contents of the witness’s diary are not remotely protected under the work product rule. The work
product rule involves a product, such as a document, prepared in anticipation of litigation or
preparation for trial, by or on behalf of a party. The witness is not a party, and her diary was not
prepared in anticipation of litigation or preparation for trial. It is, presumably, merely a collection
of private thoughts transcribed for the witness’s personal edification. (D) is also incorrect. The
best evidence rule does not literally require a party to produce the best evidence possible to prove
a point. Rather, it requires the production of the original document when attempting to prove the
contents of a document. [Fed. R. Evid. 1002] Although the witness used a document to refresh
her recollection and enable her to testify, her testimony concerned the details of an auto accident,
not the contents of the document. Because the witness was testifying about an event, as opposed
to the contents of her diary, her testimony was proper, notwithstanding the failure to produce
the diary. (C) is a tempting answer, but ultimately incorrect. If a witness’s recollection has been
refreshed prior to trial by reference to a document, the court has discretion to require that the
document be disclosed to the opposing party. [Fed. R. Evid. 612] The court is not required to
order disclosure. In this case, there is no indication that the court ordered disclosure of the diary
or that the plaintiff failed to comply with an order to produce the diary. All that is known is that,
upon learning through cross-examination of the witness that she had consulted her diary the night
before, the defendant’s attorney immediately moved that the witness’s testimony be stricken from
the record. Because there is nothing improper about a witness’s refreshing her recollection prior
to trial, the defendant’s motion should be denied and the witness’s testimony should remain on the
record.

Answer to Question 2

(D) Asking the eyewitness on direct as to whether the light was red is objectionable as leading. A
question is leading and generally objectionable when it suggests to the witness the fact that the
examiner expects and wants to have confirmed. Leading questions are generally not permitted on
direct examination of a disinterested witness. An objection is most likely to be sustained for such
a question. Each of the other choices presents a situation wherein leading questions are gener-
ally permissible, at the discretion of the court: (A) is a preliminary matter as to which there is no
dispute; courts are more likely to permit leading questions on direct examination in matters such
as this. (B) is cross-examination, in which leading questions are permitted. (C) is examination of
a very young child; a court may allow leading questions when the witness needs aid to respond
because of immaturity.

Answer to Question 3

(B) The wife is competent to testify. Persons interested in a lawsuit are not disqualified as witnesses.
(A) is wrong because the wife need not be the personal representative of her husband’s estate
to testify. (C) is incorrect because she is testifying as to what she saw, not as to her opinion. In
any event, a lay witness may testify as to an opinion if she is a percipient witness. (D) is wrong
because the common law spousal incapacity has been abolished.
2. EVIDENCE - PRESENTATION OF TESTIMONIAL EVIDENCE ANSWERS

Answer to Question 4
(B) The physician’s opinion is admissible. Under the Federal Rules, an expert witness may learn
the basis for the opinion at or before the hearing. (A) is wrong because the basis for an expert’s
opinion need not be personal observation; here, the physician was asked his opinion about the
beekeeper’s testimony. (C) is wrong because under the Federal Rules, an expert witness may
testify in the form of an opinion without prior disclosure of the underlying facts or data. In
addition, here the basis of the expert’s testimony was based on evidence already introduced at
trial. (D) is wrong because expert opinion may indeed be based on information provided by lay
witnesses.
Answer to Question 5
(A) The court should strike the statement as unresponsive to any question asked. An unresponsive
answer by a witness is subject to a motion to strike by examining counsel, but not by opposing
counsel. Thus, examining counsel can adopt an unresponsive answer if it is not objectionable on
some other ground. Here, the defense attorney asked the witness a question that was very specific
and called for a specific answer (i.e., whether there was broken glass on the pavement at the time
and place of the accident). Thus, the witness should only have stated whether there was any glass.
Her volunteered information regarding the speed of the cars bore no connection to the question
posed and was totally unresponsive to that question (or to any other question asked). Therefore,
the defense attorney, as examining counsel, is entitled to move to strike the statement, and this
motion should be granted. (B) is incorrect for two reasons: (i) The witness’s comment should
be stricken as unresponsive regardless of whether she could have known how fast the cars were
traveling. Even if she was highly experienced in estimating vehicle speeds, her comment was still
not in response to any question. (ii) It is not true that the witness could not offer an opinion on
the speed of the cars (if she were asked a question on this). A lay witness is permitted to estimate
in miles per hour the speed of a moving vehicle if it is shown that she has some experience in
observing the rate of speed of moving objects. Thus, if the witness can establish such experience,
her statement would be admitted into evidence (providing it was made in response to a question
posed by examining counsel). (C) is incorrect because it misstates the concept of “opening the
door.” One who introduces evidence on a particular subject thereby asserts its relevance and
cannot complain, except on grounds other than relevance, if her adversary thereafter offers
evidence on the same subject. This is what is meant by “opening the door.” The defense attorney
is not complaining of evidence being offered by opposing counsel. Rather, the motion to strike
is directed at a totally unsolicited comment from a witness. (D) is incorrect because the preju-
dicial nature (or lack thereof) of an unresponsive answer does not form the basis for a motion to
strike. It is true that, in most cases, an attorney would move to strike only if the witness has made
a statement harmful to his case. However, as noted above, the option of moving to strike rests
entirely with the examining attorney, and he may move to strike based solely on the unresponsive
character of the statement, without showing any prejudice.
Answer to Question 6
(C) The engineer’s testimony is admissible because it is based on knowledge gained by him at trial.
Facts or data upon which expert opinions are based may be derived from presentation at trial. One
acceptable method of doing this is to have the expert attend the trial and hear testimony estab-
lishing the facts. Thus, (B) is incorrect. (A) is incorrect. Under the Federal Rules, a hypothetical
question is not required to elicit an expert’s opinion. (D) is incorrect because an expert may also
base his opinion on facts supplied to him outside the courtroom, including types of facts not
admissible into evidence, as long as they are reasonably relied on by experts in the field.
EVIDENCE - PRESENTATION OF TESTIMONIAL EVIDENCE ANSWERS 3.

Answer to Question 7

(A) The basis of an expert’s opinion need not be admissible into evidence for the expert’s testimony
to be admissible, provided that the information is of a type reasonably relied upon by experts in
the particular field. [Fed. R. Evid. 703(B)] (B) is incorrect because the biochemist need not offer
the reports into evidence in order to base his opinion on them. In fact, the proponent of the expert
opinion is not permitted to disclose the reports if they are inadmissible, unless the court deter-
mines that their probative value in assisting the jury to evaluate the expert’s opinion substantially
outweighs their prejudicial effect. (C) is wrong because the Federal Rules reject the traditional
prohibition on opinions embracing the ultimate issue in the case, as long as the opinion is helpful
to the trier of fact. (D) is wrong because the biochemist is only relying on the reports as the basis
of making his opinion; he is not offering the reports themselves into evidence. Hence, they would
not be governed by the hearsay rule.

Answer to Question 8

(B) The driver’s answer could only reflect his opinion of what the boy did or thought, and is thus
impermissible opinion evidence. A layperson’s opinion is admissible if it is rationally based on the
perception of the witness, helpful to a clear understanding of the witness’s testimony on the deter-
mination of a fact in issue, and not based on scientific, technical, or other specialized knowledge.
The driver’s opinion does not meet those requirements; he is speculating rather than testifying to
his own perceptions. (A) is wrong because the question does not call for the driver to testify about
an out-of-court declaration. (C) is wrong because, although relevant, the answer would still be
improper. (D) is wrong because the form of the question is improper.

Answer to Question 9

(A) The court should permit the cab driver to review her notes. Under the rule of present recollec-
tion refreshed, a witness may be shown any writing or other thing that may refresh her memory
of an event. The writing is not authenticated, is not in evidence, and may be used solely to refresh
her recollection. (B) is wrong because the doctrine of past recollection recorded applies when a
party is seeking to introduce a writing into evidence. To have the writing read into evidence, a
foundation must be laid. Here, the motorcyclist’s attorney is not seeking to introduce the notes
into evidence; he merely wants the cab driver to review them. (C) is a misstatement of the law
and makes no sense in this situation. (D) is wrong because the cab driver is not testifying to the
contents of her statement, but to her recollection of the accident.

Answer to Question 10

(A) The witness’s testimony should be admitted as evidence of the extent of the plaintiff’s injury,
because it relates to his firsthand observations and is otherwise admissible. Before or while testi-
fying, a witness may use any writing or thing for the purpose of refreshing his present recollec-
tion. He usually may not read from the writing while he actually testifies, since the writing is
not authenticated, is not in evidence, and may be used solely to refresh his recollection. Here, the
witness properly refreshed his recollection before testifying. (B) is wrong because a past recollec-
tion recorded is a writing that is actually read into evidence. Here, the witness is not reading from
the records as he testifies, and the record is not being introduced. In addition, a proper foundation
has not been laid for the introduction of the records as a past recollection recorded. (C) is wrong
because the best evidence rule applies when a party is trying to prove the contents of a writing.
Here, the witness’s testimony is about what he saw, not about the contents of the records. (D) is
4. EVIDENCE - PRESENTATION OF TESTIMONIAL EVIDENCE ANSWERS

wrong because anything (even hearsay statements) can be used by a witness to refresh his recol-
lection.

Answer to Question 11

(A) The expert’s testimony is admissible because other civil engineers ordinarily reasonably rely on
structural engineers’ reports in forming professional opinions. Expert testimony is admissible
if the subject matter is one where scientific, technical, or other specialized knowledge would
assist the jury in understanding the evidence or determining a fact in issue. [Fed. R. Evid. 702]
The proper design of a spiral staircase would not be a matter of common knowledge, and the
testimony of an expert would be of assistance in determining whether the design was faulty. The
expert’s opinion may be based on facts not in evidence that were supplied to the expert out of
court, and which facts are of a type reasonably relied upon by experts in the particular field in
forming opinions on the subject. [Fed. R. Evid. 703] The expert may therefore give an opinion
based on the photographs and the structural engineer’s report if such photographs and reports
are of a type reasonably relied upon by civil engineers in forming opinions on structural design.
Federal Rule 703 allows the expert’s testimony even though the photographs and report are not
in evidence. (B) is incorrect because Federal Rule 703 does not require an expert to disclose the
facts on which he relied in forming his opinion. In fact, the proponent of the expert opinion must
not disclose those facts to the jury (because they may be of a type not admissible in evidence)
unless the court determines that their probative value in assisting the jury to evaluate the expert’s
opinion substantially outweighs their prejudicial effect. (C) is incorrect because the Federal Rules
allow the expert to base his opinion on opinions in the structural engineer’s report as long as such
opinions in reports would be reasonably relied upon by civil engineers in forming opinions. An
expert traditionally is not permitted to rely on the opinions of others as a predicate for his own
opinion. However, Federal Rule 703 significantly expands the traditional rule and permits an
expert to base his opinion on the opinion of others if they are of the type reasonably relied upon
by experts in the field, as discussed above. Therefore, if civil engineers ordinarily reasonably
rely on such opinions in reports from structural engineers, the Federal Rules allow the expert to
base his opinion on opinions in the report. (D) is incorrect because the fact that the report and
photographs were commissioned solely for the purpose of litigation and were not admitted into
evidence is irrelevant when an expert is basing an opinion on facts reasonably relied upon by
experts in that field. When determining whether a report comes under the business records excep-
tion to the hearsay rule, the courts sometimes look at whether the report was prepared for litiga-
tion in determining if it was a record maintained in conjunction with a business activity. [Palmer
v. Hoffman (1943)] Here, whether the report is hearsay is not at issue because an expert may base
his opinion on facts reasonably relied upon by experts in that particular field, even if those facts
are inadmissible hearsay.

Answer to Question 12

(B) The jury may consider statements from an authoritative text as substantive evidence. Rule 803(18)
provides that an expert may base an opinion on facts supplied to him outside the courtroom. One
such source is authoritative texts and treatises. Statements from a treatise established as reliable
(which may be done by the expert’s own testimony) may be introduced on direct examination of
the expert and read into the record as substantive evidence under an exception to the hearsay rule.
[Fed. R. Evid. 803(18)] Hence, the jury may consider the testimony in (B) as substantive evidence.
(A) is incorrect because the evidence may not be considered by the jury as substantive evidence.
Rule 703 provides that where an expert bases his opinion on facts made known to him outside the
courtroom, the facts need not be of a type admissible in evidence as long as the facts are of a kind
EVIDENCE - PRESENTATION OF TESTIMONIAL EVIDENCE ANSWERS 5.

reasonably relied on by experts in the particular field. However, the expert will only be permitted
to disclose such facts if the court determines that their probative value in assisting the jury to
evaluate the expert’s opinion substantially outweighs their prejudicial effect. The jury would only
be permitted to consider them as the basis for the expert’s opinion; the jury could not consider
the facts as substantive evidence unless they were independently admissible. Here, the statements
made to the expert by witnesses to the collapse are a proper basis of his opinion because they are
of a kind reasonably relied on by experts in his field. However, they do not appear to be admis-
sible under any exception to the hearsay rule and therefore could not be considered by the jury
as substantive evidence. (C) is incorrect because it is too broad. Generally, evidence of repairs or
other precautionary measures made after an incident is inadmissible if used to prove negligence,
culpable conduct, product or design defects, or a need for a warning or instruction. Such evidence
may, however, be admitted to prove ownership or control, to rebut any claim that precautionary
measures were not feasible prior to the incident, or to prove destruction of evidence. Here, admis-
sibility of the expert’s testimony regarding the subsequent repair of the bridge will be contingent
on the purpose of the testimony, and (C) is incorrect because it does not indicate any purpose.
(D) is incorrect. Generally, a statement made by an agent concerning a matter within the scope
of his agency is admissible against the principal as a vicarious admission if it was made during
the existence of the employment relationship. In this case, the statement was made by a former
employee of the mall, and therefore it was not made during the scope of the employment relation-
ship. Because it does not fall within any of the hearsay exceptions, the statement is inadmissible.

Answer to Question 13

(B) Although a party is entitled as of right to some cross-examination, the extent or scope of cross-
examination is a matter of judicial discretion. The judge may exercise reasonable control over the
examination of witnesses to aid the effective ascertainment of truth, to avoid wasting time, and to
protect witnesses from harassment or undue embarrassment. Specifically, the trial court has the
authority to cut off cross-examination when it determines there has been an adequate opportunity
for meaningful cross-examination. Here, the defense attorney has the right to cross-examine the
adverse witness. However, carrying on such cross-examination for three days, with the prospect of
at least one more day, would probably be considered excessive in light of the limited direct exami-
nation of the witness. Cross-examination is limited to: (i) matters brought out on direct examina-
tion and inferences naturally drawn therefrom; and (ii) matters affecting the credibility of the
witness. [Fed. R. Evid. 611(b)] (A) is incorrect because, even if the testimony is relevant, the court
may terminate or otherwise control the cross-examination to avoid wasting time or harassing a
witness. (C) is incorrect for a similar reason. Although matters relating to the subject matter of
direct examination may properly be brought out on cross-examination, the court has the discre-
tion to terminate such questioning to avoid wasting of time or harassment or embarrassment of a
witness. Therefore, it is incorrect to state that, if the questioning relates to the subject matter of
direct examination, the court lacks the authority to terminate the cross-examination. (D) is incor-
rect because, although due process considerations are of undisputed significance, the facts here do
not appear to present any real conflict between due process and judicial economy. If the defense
attorney has had an adequate opportunity for cross-examination of the witness, then there is no
deprivation of due process if principles of judicial economy now dictate that the questioning be
cut short.

Answer to Question 14

(B) The court should allow the witness to testify if the clerk takes an oath to make a true translation.
The services of an interpreter may be used where a witness, due to language problems or other

一 信:liuxue119118 , 们 信免 供
6. EVIDENCE - PRESENTATION OF TESTIMONIAL EVIDENCE ANSWERS

reasons, would otherwise have difficulty communicating. Under Federal Rule 604, an interpreter
must meet the qualifications required of an expert witness (i.e., by reason of knowledge, skill,
experience, training, or education, she is capable of providing a true translation). Also, an inter-
preter must take an oath or affirmation that she will make a true translation (i.e., that she will
communicate exactly what the witness is expressing in his testimony). Here, the witness will have
extreme difficulty communicating, due to the fact that he uses a signing method different from
that which is used in the United States. Thus, the circumstances allow the use of an interpreter to
assist the witness in communicating at the trial. Here, the facts establish that the clerk is the only
person qualified to act as an interpreter for the witness. If, as (B) states, the clerk takes an oath to
communicate what the witness expresses in his testimony, then the requirements of Rule 604 are
met. (A) is incorrect because it omits the requirement of taking an oath or affirming to make a
true translation. (C) is incorrect because there is no principle of law that renders the clerk “inher-
ently biased” simply because she works for the prosecutor and has assisted the police in their
communications with the witness. If the clerk takes the required oath, she is bound to render a
true translation, regardless of her affiliation with the prosecutor’s office. (D) is incorrect because
there is no requirement that the clerk disclose this information to the jury. Determining whether
an interpreter is qualified is a matter within the judge’s discretion.

Answer to Question 15

(D) The substantive relation of the questions to the issues of the lawsuit has no bearing on whether
leading questions are permitted. Use of leading questions on direct examination to refresh the
recollection of a witness or to establish preliminary matters is permitted in the discretion of the
trial court. Thus, (A) and (B) are incorrect. Where a witness is hostile to the party calling him,
the examination is treated as cross-examination, and leading questions may be used. Thus, (C) is
incorrect.

Answer to Question 16

(A) The question is proper. For the purpose of impeaching the credibility of a witness, a party may
show that the witness has, on another occasion, made statements that are inconsistent with some
material part of her present testimony. Under the Federal Rules, any party may impeach any
witness (even its own witness), so (D) is incorrect. (C) is incorrect because leading questions are
permitted on direct examination when a witness is hostile or identified with an adverse party.
Thus, leading questions are permitted because the former waitress lives with the plaintiff. (B) is
incorrect because the restaurant’s attorney is not trying to refresh the former waitress’s recollec-
tion of whether the drink was on the menu; rather, he is talking about a deposition and is trying to
impeach her with an inconsistent statement.
EVIDENCE - RELEVANCY,
PRIVILEGES AND
WRITINGS QUESTIONS
EVIDENCE - RELEVANCY, PRIVILEGES, AND WRITINGS QUESTIONS 1.

EVIDENCE - RELEVANCY, PRIVILEGES, AND WRITINGS QUESTIONS

Question 1 Question 2

A brother and a sister were arrested on the A plaintiff sued a defendant for personal
federal charge of tax evasion in connection with injuries, claiming that while the plaintiff was
the family business. Prior to trial, the prosecutor driving through an intersection at the posted
told the sister that he believed he could get her speed limit, the defendant failed to stop at a stop
sentence reduced to probation if she pleaded sign and struck her car. At trial, the plaintiff
guilty to a lesser charge and agreed to testify calls the defendant’s friend to testify to the fact
against her brother; the sister reluctantly agreed. that the defendant never stops at the stop sign at
During the jury trial, the sister is called by the the accident intersection and invariably “runs”
prosecution. On cross-examination, the defense every stop sign.
attorney brings out the fact that the sister was
arrested on the same charge. The attorney then Should the defendant’s objection to the testi-
asks her whether it is true that after her arrest, mony be sustained?
the prosecutor told her that if she testifies against
her brother her sentence can be reduced to (A) Yes, because it is not the best evidence.
probation. The prosecutor objects.
(B) Yes, because character evidence is inadmis-
How should the court rule on the objection? sible in a civil case.

(A) Sustained, because it is against public (C) No, because it is evidence of habit.
policy to reveal information about plea
bargains to a jury. (D) No, because it is self-serving.

(B) Sustained, because it calls for hearsay.

(C) Overruled, because the question goes to


bias or interest.

(D) Overruled, because the sister waived the


attorney-client privilege by testifying.
2. EVIDENCE - RELEVANCY, PRIVILEGES, AND WRITINGS QUESTIONS

Question 3 Question 4

A victim was struck by a car and taken to In litigation over whether a holographic will
the emergency room (“ER”) immediately after and codicil were prepared by the testator, the
the accident. He was treated by an ER doctor defendant, in a challenge to the validity of the
and released 30 minutes later. The victim later instruments, calls the testator’s former business
went to his own physician for a follow-up. The partner to testify that he has seen the testator’s
victim filed suit against the driver of the car handwriting on many business documents and
that struck him. At trial, the victim testified that that the handwriting on the will and codicil is
he suffered from lower back pains and sought that of the testator. The plaintiff objects to the
damages from the driver therefor. The defense testimony of the business partner.
attorney subpoenaed the ER doctor and put her
on the witness stand. After a line of questioning How should the court rule?
establishing who the ER doctor is and where she
is employed, the defense attorney asked the ER (A) The testimony is inadmissible because the
doctor to describe the victim’s condition when witness has not been qualified as a hand-
she examined the victim immediately after the writing expert.
accident. The victim’s attorney objected on the
grounds that his client wished to invoke the (B) The testimony is inadmissible because a
jurisdiction’s physician-patient privilege. proper foundation has not been laid.

How should the court rule on the objection? (C) The testimony is admissible because the
witness is familiar with the testator’s
(A) Sustained, because the patient has the right handwriting.
to invoke the privilege.
(D) The testimony is admissible because the
(B) Sustained, because the ER doctor’s testi- witness is not interested in the outcome
mony is irrelevant to the victim’s present of the case and therefore is competent to
condition. testify.

(C) Overruled, because the victim is suing for


personal injuries.

(D) Overruled, because the ER doctor was not


the victim’s physician prior to the accident.
EVIDENCE - RELEVANCY, PRIVILEGES, AND WRITINGS QUESTIONS 3.

Question 5 Question 6

A 60-year-old employee who was fired by a A homeowner is suing a motorcyclist, alleging


corporation after 25 years of employment filed that the motorcyclist destroyed her rose garden
an age discrimination suit against the corpora- by riding his motorcycle through her yard. The
tion. While the corporation’s excuse was that the homeowner seeks to introduce in evidence a
reorganization and merger required a trimming photograph of her garden to depict the nature
of personnel, at trial the employee seeks to have and extent of the damage done. The photograph
a board member testify that the chairman of was taken by an insurance adjuster, who was not
the board had convinced the board to fire the called as a witness, four days after the alleged
employee because he “didn’t fit our corporate incident.
image of youthful vigor.” It was typical practice
that all directors’ meetings be recorded, and that How should the judge rule on the photograph?
the corporate secretary use the recording to type
up a formal transcript of the proceedings. The (A) Admissible, because the homeowner can
meeting at which the employee’s dismissal was testify that it fairly and accurately portrays
discussed was no exception. the condition of the garden after the dam-
age was done.
If the defense objects to the board member’s
proposed testimony, how should the court rule? (B) Admissible, because the photograph was
taken within a week after the alleged occur-
(A) The testimony is admissible, because it rence.
is based on the board member’s firsthand
knowledge. (C) Inadmissible, because the homeowner
failed to call the photographer to testify
(B) The testimony is inadmissible, because it is concerning the circumstances under which
hearsay not within any recognized excep- the photograph was taken.
tion to the hearsay rule.
(D) Inadmissible, because it is possible to
(C) The testimony is inadmissible, because the describe the damage to the garden through
audio recording of the directors’ meeting is direct oral testimony.
the best evidence.

(D) The testimony is inadmissible, because the


corporate secretary’s transcription of the
directors’ meeting is the best evidence.
4. EVIDENCE - RELEVANCY, PRIVILEGES, AND WRITINGS QUESTIONS

Question 7 Question 8

A tool producer was in the business of The plaintiff brought suit against his former
manufacturing and selling various hardware. boss for a libelous letter received by the
The owner of a retail hardware store ordered plaintiff’s present employer. The former boss
1,000 hammers from the tool producer, using allegedly sent the letter. The authenticity and
the standard form that the tool producer’s sales contents of the letter are disputed at the jury
personnel used. Shortly after the hardware trial. In response to a question from the plain-
store placed their order, the tool producer’s tiff’s attorney, the present employer states that:
headquarters caught fire. All the purchase and “I received a letter from the plaintiff’s former
sale records were destroyed in the fire. After- boss, which I seem to have lost, but I remember
wards, the hardware store owner asserted that exactly what it said.”
the salesperson had agreed that the price for the
hammers would be $5,000. The tool producer The plaintiff’s present employer will be
maintained that the price was $5,500. The tool allowed to testify as to the contents of the letter
producer then sued the hardware store. At the if:
trial, the tool producer’s director of the order
department took the stand. After establishing (A) The plaintiff’s attorney lays a proper foun-
that the original order had been destroyed by dation by proving the present employer’s
fire, the director sought to testify that the price good character for truth.
of the order was $5,500.
(B) The judge finds that the original letter is
How should the court rule on the admissibility lost.
of such testimony?
(C) The jury is satisfied that the original letter
(A) Inadmissible, because it is self-serving and is unavailable.
untrustworthy.
(D) The present employer can recall the
(B) Inadmissible, because of the best evidence contents of the letter from memory without
rule. looking to any other document.

(C) Inadmissible, because it is not the best


degree of secondary evidence.

(D) Admissible.
EVIDENCE - RELEVANCY, PRIVILEGES, AND WRITINGS QUESTIONS 5.

Question 9 Question 10

In a real property dispute over a decedent’s The plaintiff was injured when she slipped at
vacation home, the plaintiff offers a deed to the defendant restaurant. The defense attorney
show that the home had been transferred to her asked the manager on duty at the time of the
two months before the decedent died. The defen- incident to prepare a report of the accident. This
dant, the decedent’s heir, disputes the plaintiff’s report was given to the defense attorney prior
claim and alleges that the decedent’s signature to trial. During discovery, the plaintiff demands
on the deed was forged. The defendant testifies that a copy of the report be produced.
that he is familiar with the decedent’s signature
and the signature on the deed is not his. Will the court order the defendant to produce
the report?
How should the judge rule on this testimony?
(A) Yes, because business reports are not privi-
(A) Inadmissible, because the defendant is not leged.
a handwriting expert.
(B) Yes, because it is the best evidence.
(B) Inadmissible, because the defendant has
a stake in the outcome and his opinion is (C) No, because it is covered by the attorney-
unreliable. client privilege.

(C) Admissible, because the defendant knows (D) No, because it is hearsay.
the decedent’s signature.

(D) Admissible, because he is disputing the


genuineness of the document, not seeking
to establish it.
6. EVIDENCE - RELEVANCY, PRIVILEGES, AND WRITINGS QUESTIONS

Question 11 Question 12

A plumbing contractor sued a homeowner, A singer has denied his purported signature
alleging that the homeowner refused to pay for on a letter that has become critical in a breach of
extensive pipe repairs performed on her home contract suit between him and a record producer.
by an employee of the contractor. The contractor At trial, the record producer’s counsel calls a
called the employee to the stand as a witness. teacher who testifies that she taught the singer
The employee, under oath, testified that he did mathematics in school 10 years earlier, knows
not perform any work at the homeowner’s home. his signature, and proposes to testify that the
The employee also denied writing a letter to a signature to the letter is that of the singer. The
friend telling the friend that he was going to singer’s counsel objects.
do plumbing work on the homeowner’s house.
Without releasing the employee as a witness, the The trial judge should:
contractor offers in evidence the letter written by
the employee to his friend. (A) Sustain the objection on the ground that
identification of handwriting requires ex-
Which of the following is NOT a proper basis pert testimony and the teacher does not, per
for admitting the employee’s letter? se, qualify as an expert.

(A) Testimony by the employee’s wife that she (B) Sustain the objection on the ground that the
recognizes the employee’s handwriting. best evidence of the singer’s handwriting
would be testimony by a person who had
(B) The friend’s testimony that the statements examined his writing more recently than 10
in the letter are responsive to a prior letter years ago.
from the friend to the employee.
(C) Overrule the objection on the ground that a
(C) Comparison by the jury of the letter schoolteacher qualifies as an expert witness
with another letter that the employee has for the purpose of identifying handwriting.
admitted writing.
(D) Overrule the objection on the ground that a
(D) In-court comparison by the friend, a nonex- lay person may identify handwriting if he
pert, of the letter with another letter that the has seen the person in question write, and
employee has admitted writing. has an opinion concerning the writing in
question.
一 信:liuxue119118 , 们 信免 供

EVIDENCE - RELEVANCY, PRIVILEGES, AND WRITINGS QUESTIONS 7.

Question 13 Question 14

A plaintiff applied for a life insurance policy A plaintiff was injured as a result of a
and was required to submit to a physical exami- defendant’s negligence. The plaintiff hired an
nation to qualify for the policy. During the attorney, who sent the plaintiff to see a physician
course of the examination, the plaintiff told the for the purpose of examining the plaintiff prior
physician, who was approved by the life insur- to trial and assessing the extent of his injuries.
ance company and had never seen the plaintiff During the course of the examination, the
before, “I used to have some back trouble, but plaintiff made some statements to the physician
that’s all cleared up now.” A few weeks after the indicating that he was not completely free from
examination, the defendant’s automobile struck negligence when the accident occurred. The
the rear end of a car in which the plaintiff was defendant seeks to call the physician to testify to
riding as a passenger. The plaintiff now claims the statements the plaintiff made to the physi-
that he suffers persistent lower back pain and cian. The attorney objects.
sues the defendant for damages. After laying a
proper foundation that the plaintiff is attempting How should the court respond?
to perpetrate a fraud, the defendant calls the
physician as a witness and seeks to have her (A) Overrule the objection, because the plain-
testify as to the plaintiff’s statement to her. The tiff made an admission.
plaintiff’s attorney objects on the ground of the
jurisdiction’s physician-patient privilege. (B) Overrule the objection, because a physician
qualifies as an expert witness.
Should the court allow the physician to testify
about the plaintiff’s statement? (C) Sustain the objection, because the attorney-
client privilege applies.
(A) Yes, because a proper foundation is laid
establishing that the plaintiff is attempting (D) Sustain the objection, because the jurisdic-
to perpetrate a fraud. tion’s physician-patient privilege applies.

(B) Yes, because the plaintiff was not seeking


diagnosis or treatment.

(C) No, because the statement was made to a


physician who was attending the plaintiff in
a professional capacity.

(D) No, because the statement was made prior


to the accident and is therefore irrelevant.
8. EVIDENCE - RELEVANCY, PRIVILEGES, AND WRITINGS QUESTIONS

Question 15 Question 16

A building contractor sued a homeowner for An artist was on trial for burglary, defined
failure to pay on a small cost-plus construction in the jurisdiction as breaking and entering the
contract. At trial, the contractor, who person- dwelling of another with the intent to commit
ally supervised all of the work, seeks to testify a felony. The artist’s defense was that he was
to what he remembers about the amount of pipe merely retrieving his own property, because
used, the number of workers used on the job, and he had sold a painting to a customer but the
the number of hours spent. customer had refused to pay him for it or return
it. A neighbor of the customer testified for the
The homeowner objects on the ground that the prosecution that he saw the artist carrying a
contractor had routinely recorded these facts in painting from the customer’s house. The artist
notebooks, which are in the contractor’s posses- attempts to call to the stand a witness who was
sion. in the gallery at the time of the sale to testify
to the transaction between the artist and the
Is the contractor’s testimony admissible? customer. The prosecution objects to the testi-
mony, claiming it is irrelevant.
(A) Yes, as a report of regularly conducted
business activity. The judge should rule that the witness’s testi-
mony is:
(B) Yes, as based on firsthand knowledge.
(A) Admissible, because it is relevant in de-
(C) No, because it violates the best evidence termining whether the artist had the intent
rule. required for burglary.
(D) No, because a summary of writings cannot (B) Admissible, because it is relevant in estab-
be made unless the originals are available lishing the neighbor’s credibility.
for examination.
(C) Inadmissible as irrelevant, because it would
tend to impeach the neighbor’s testimony
on a collateral matter.

(D) Inadmissible as irrelevant, because the


probative value of the witness’s testimony
is substantially outweighed by the danger
of unfair prejudice, confusion of the issues,
waste of time, or needless presentation of
cumulative evidence.
EVIDENCE - RELEVANCY, PRIVILEGES, AND WRITINGS QUESTIONS 9.

Question 17 Question 18

A plaintiff sued a defendant for injuries A woman is suing a fast-food restaurant for
arising out of a collision between vehicles driven injuries she suffered when she fell after slipping
by the parties. The plaintiff alleged that the on a wet spot on the floor of the restaurant.
defendant ran a red light when he struck the At trial, the woman introduced evidence that
plaintiff’s vehicle in an intersection. The plain- someone had spilled a drink on the floor and
tiff wishes to call a witness to the stand who was that an employee had just finished mopping up
near the intersection at the time of the accident. the spill before she walked over the wet spot and
The witness is prepared to testify that the defen- fell, injuring her back. The woman then calls the
dant offered to pay the witness $500 to testify restaurant’s manager, as a hostile witness, and
falsely in the defendant’s favor. attempts to elicit testimony that, after her fall,
the restaurant instituted a policy requiring that
Is the testimony admissible? bright yellow signs with red warnings be placed
after mopping the floor.
(A) Yes, as substantive evidence of the weak-
ness of the defendant’s case. Will this evidence be permitted?

(B) Yes, for the limited purpose of impeach- (A) Yes, to establish that the restaurant was
ment by specific bad conduct. negligent in not warning customers that the
floor was wet.
(C) No, because it is irrelevant to the case.
(B) Yes, to show that the restaurant was aware
(D) No, because although relevant, such that a wet floor creates a hazardous condi-
evidence is misleading and prejudicial. tion.

(C) No, because its admission would discourage


other tortfeasors from taking remedial
measures.

(D) No, because it is not a proper form of


impeachment.
EVIDENCE - RELEVANCY,
PRIVILEGES AND
WRITINGS ANSWERS
EVIDENCE - RELEVANCY, PRIVILEGES, AND WRITINGS ANSWERS 1.

EVIDENCE - RELEVANCY, PRIVILEGES, AND WRITINGS ANSWERS

Answer to Question 1

(C) The prosecutor’s objection should be overruled because the question goes to the witness’s bias
or interest. Evidence that a witness is biased or has an interest in the outcome of the suit tends
to show that the witness has a motive to lie. A witness may always be impeached by evidence
of interest or bias, either on cross-examination or, if a proper foundation is laid, by extrinsic
evidence. In a criminal case, it is proper for the defense to ask a prosecution witness whether she
has been promised immunity from punishment or a reduction of punishment for testifying. This
shows a motive for the witness to curry favor with the state. Here, the defense attorney is trying
to impeach the sister by showing that because she was offered an attractive sentence, she has a
motive to curry favor with the prosecution. This is perfectly proper. Note that there is no need
for a foundation because the attorney is eliciting this evidence on cross-examination, rather than
attempting to introduce extrinsic evidence of the deal. (A) is wrong because it misapplies and
misstates the rule with regard to plea bargains. Under Federal Rule 410, withdrawn guilty pleas,
pleas of nolo contendere, offers to plead guilty, and evidence of statements made in negotiating
such pleas are inadmissible against the defendant who made the plea or was a participant in the
plea discussions. This rule does not apply in this case because it does not apply to accepted guilty
pleas, and the sister is not the defendant. The rule applies only to offers and withdrawn pleas.
After the plea is accepted, it is admissible. (B) is wrong for two reasons: (i) the statement by the
prosecutor is not hearsay because it is not being offered for the truth of the matter asserted, but
rather to show its effect on the hearer; and (ii) even if the statement were hearsay, it would not
make the question improper because evidence that is substantively inadmissible may be admitted
for impeachment purposes if relevant to show bias or interest. (D) is wrong because no attorney-
client privilege arises with respect to communications between the prosecutor and the sister.
The attorney-client privilege requires that the attorney-client relationship exist at the time of the
communication. To be covered, the client must be seeking the professional services of the attorney
at the time of the communication. The sister was not seeking the services of the prosecutor. The
prosecutor could not in any way be considered to be her attorney; they are clearly adversaries.
Furthermore, even if this were not the case, the client holds the privilege and may waive it. Thus,
the question would be proper, and it would be up to the client-witness to decide whether to waive
the privilege.

Answer to Question 2

(C) The testimony is admissible as evidence of habit. Habit describes a person’s regular response
to a repeated specific situation. Evidence of a person’s habit may be admitted to prove that on a
particular occasion the person acted in accordance with that habit. [Fed. R. Evid. 406] According
to the testimony, the defendant regularly fails to obey the stop sign at the intersection at which
the collision occurred, and in fact, he regularly disregards any stop sign. This regular response
to a specific circumstance constitutes a habit. Consequently, the testimony, which is evidence of
this habit, is admissible, and so the defendant’s objection should be overruled. (A) is incorrect
because the best evidence rule is inapplicable to this question. Under the best evidence rule, where
the terms of a writing are material, the original writing must be produced in proving the terms
of the writing. Here, there is no writing material to the case; thus, the best evidence rule does not
come into play. (B) is incorrect because the offered testimony is not character evidence. Character
describes one’s disposition with respect to traits or general patterns of behavior. If the testimony
were that the defendant is generally a careless driver, it would be inadmissible character evidence.
The testimony, however, describes a repeated response by the defendant to repeated specific
2. EVIDENCE - RELEVANCY, PRIVILEGES, AND WRITINGS ANSWERS

circumstances, which is admissible habit evidence. (D) is incorrect because the fact that an objec-
tion is self-serving does not form a basis for overruling (or sustaining) the objection. In a sense,
all objections are self-serving to the party making them, just as the evidence to which an objection
is made is self-serving to the party offering it.

Answer to Question 3

(C) The court should overrule the objection because the victim is suing for personal injuries. A person
cannot invoke the physician-patient privilege, which prohibits the doctor from divulging informa-
tion acquired while attending a patient, where that person has put his physical condition in issue
(e.g., by suing for personal injuries). The victim is suing the driver for personal injuries allegedly
incurred as a result of being struck by the car driven by the driver. Therefore, the victim himself
has put his physical condition in issue and cannot avail himself of the physician-patient privilege.
While it is true that the physician-patient privilege belongs to the patient (i.e., the patient decides
to claim or waive the privilege), (A) is incorrect because, as discussed above, the privilege does
not apply in this situation. (B) is incorrect because the testimony of the ER doctor is relevant
to the present condition of the victim. Evidence is relevant if it tends to make the existence of
any fact that is of consequence to the determination of an action more probable than it would be
without the evidence. [Fed. R. Evid. 401] The testimony of the ER doctor will indicate whether
the physical condition of the victim immediately after the accident would suggest that the victim
had suffered lower back injuries. For example, if the ER doctor’s testimony would indicate the
absence of a condition of injury at the time of the observation, then this would make it more
probable than not that any current pain experienced by the victim was not caused by the accident.
(D) is incorrect because the privilege does not require that the physician be the patient’s personal
physician prior to an accident or other cause of injury. It is sufficient if the physician attended the
patient in a professional capacity, even if she has never seen the patient before. Thus, the fact that
the ER doctor was not the victim’s personal physician prior to the accident will not constitute a
ground for overruling the objection.

Answer to Question 4

(C) The court will allow the witness to testify as to the testator’s handwriting simply because he is
familiar with it by virtue of his former business relationship with the testator. As long as such a
foundation is laid to show familiarity with the handwriting, a lay opinion is permissible. Thus,
because the witness became familiar with the testator’s handwriting when they worked together,
he will be permitted to testify that the handwriting on the instruments is that of the testator. (A)
is incorrect because expert testimony is not required for handwriting identification. Because the
witness’s testimony is based on his previous familiarity with the testator’s handwriting, it will
be admissible as nonexpert opinion testimony. (B) is incorrect because a proper foundation has
been laid. To lay a foundation for nonexpert opinion testimony, it must be shown that the witness
had the opportunity to observe the event that forms the basis of the opinion. The defense has
established that the witness was familiar with the testator’s handwriting because he had seen it
on various business documents. This is sufficient foundation for his testimony. (D) is incorrect
because his interest in the outcome of the will is irrelevant to his competency as a witness (though
it may affect the weight and credibility given to his testimony).

Answer to Question 5

(A) The board member’s testimony should be admissible because it is relevant nonhearsay based
on firsthand knowledge. The testimony is relevant to the employee’s age discrimination suit
EVIDENCE - RELEVANCY, PRIVILEGES, AND WRITINGS ANSWERS 3.

because it is being offered to prove that the board’s motivation in firing the employee was his
age. It is not hearsay, even though the board member is repeating the statement of an out-of-court
declarant (the chairman of the board), because it is not being offered to prove the truth of what
the chairman was asserting (i.e., that the employee was in fact too old to fit the corporate image).
Rather, it is being offered to show its effect on the board; i.e., it is being offered as circumstantial
evidence of the board’s motivation in deciding to fire the employee, which is the critical issue in
the case. Finally, the board member is competent to testify to the chairman’s statement since he
heard it firsthand, and no other restrictions on the admissibility of relevant evidence are appli-
cable in this case. (B) is incorrect because, as discussed above, the testimony as to the chairman’s
statement is not hearsay because it is not being offered to prove the truth of that statement. Even
if it were being offered to prove its truth, it would not be hearsay under the Federal Rules because
it qualifies as a vicarious admission by a party-opponent. The chairman made the statement in
his capacity as chairman of the board; hence, it will be admissible against the corporation as an
admission. (C) and (D) are incorrect because the best evidence rule applies only when the writing
or recording is a legally operative or dispositive instrument or the knowledge of the witness comes
from reading the document or listening to the recording. The best evidence rule does not apply
where, as here, the fact to be proved exists independently of any writing or recording, and the
witness testifying to the fact has knowledge of the fact independent of the audio recording or the
transcription.

Answer to Question 6

(A) The judge should rule the photograph admissible. Photographs are admissible only if identified
by a witness as a portrayal of certain facts relevant to the issue and verified by the witness as a
correct representation of those facts. The witness who identifies the photograph need only be
familiar with the scene or object that is depicted. It is not necessary to call the photographer to
authenticate the photograph. The homeowner is familiar with the garden that is depicted. Thus,
she may testify to the photograph as an accurate portrayal of the condition of the garden after
the damage was done. (B) is incorrect because, if the photograph accurately depicts the damage,
the photograph need not have been taken within a week after the occurrence. The fact that it was
taken within a week does not establish that it was an accurate portrayal of the damage to the
garden. (C) is incorrect because, as noted above, it is not necessary to call the photographer. (D) is
incorrect because, as long as the photograph is properly authenticated, it is admissible.

Answer to Question 7

(D) The testimony is admissible. The best evidence rule requires the production of the original
document when attempting to prove the contents of the document. [Fed. R. Evid. 1002] The
director of the tool producer’s order department seeks to testify that the price of the hardware
store’s order was $5,500. It appears that this testimony is based solely on the director having
seen the standard tool producer’s form on which the hardware store placed their order. Thus, the
testimony is about the contents of a document—the tool producer’s form that the store owner
completed to order hammers. At first glance, therefore, it would appear that (B) is the correct
answer, that the director’s testimony would be inadmissible because of the best evidence rule.
However, that rule does not require production of the original if the original has been lost or
destroyed in good faith. [Fed. R. Evid. 104(1)] It appears that the order form was destroyed in
good faith in a fire at the tool producer’s headquarters. Thus, (B) is incorrect. (C) is also incorrect.
If the original of a document cannot be produced to prove its contents, any form of secondary
evidence can be substituted for the original. There is no requirement, for example, that a copy
of the original be produced. Testimony as to the contents of the document is equally permis-
4. EVIDENCE - RELEVANCY, PRIVILEGES, AND WRITINGS ANSWERS

sible. From the standpoint of the best evidence rule, then, (D) is the correct answer. The proposed
testimony of the director of the tool producer’s order department is admissible. (A) is incor-
rect because it confuses the question of admissibility of evidence with that of the weight that a
fact finder should give to the evidence. That the proposed testimony is self-serving may render
it unpersuasive, but not inadmissible. Also, if the self-serving nature of testimony rendered it
inadmissible, then virtually any time a party testified in support of his proposition, the testimony
would have to be excluded. The law of evidence has for centuries rejected such a broad rule
requiring disqualification of witnesses. In addition, (A) states that the proposed testimony of the
director of the tool producer’s order department is untrustworthy. There is no factual basis in the
question for reaching that conclusion.

Answer to Question 8

(B) This is a best evidence rule question; it is an attempt to prove the contents of a document that
is the basis of the suit. The best evidence rule applies and therefore the party must produce the
original document or offer a satisfactory excuse for not producing it. Under Rule 1004, if the
original is lost, that is considered a reasonable excuse for not producing the original. Whether the
letter is in fact “lost” would be a preliminary question of fact to be determined by the court under
Rule 104(a), rather than the jury. Thus, (B) is correct and (C) is incorrect. (A) is incorrect; there is
no such requirement under the Rules. (D) is incorrect; there is no such rule of law.

Answer to Question 9

(C) The judge should rule this testimony admissible because the defendant knows the decedent’s
signature. Lay opinion testimony is permissible and often essential to identify telephone voices
and handwriting. Any lay witness who is familiar with the signature of a person may testify as to
his opinion as to its genuineness. In such a case, a foundation must first be laid to show familiarity
with the handwriting, as was done here by the defendant’s testimony. Therefore, (A) is wrong.
(B) is wrong because it goes to the weight of the testimony, not the admissibility. (D) is wrong
because the lay witness may testify in support of, or against, the genuineness.

Answer to Question 10

(C) The court will not order the defendant to produce the report. Communications between an
attorney and client, made during professional consultation, are privileged from disclosure. A
business report prepared as a communication from client to attorney is privileged. Here, the
manager prepared the report at the request of the restaurant’s attorney. As such, the report
constitutes a privileged communication between an attorney and client and its production is not
required. Thus, (C) is correct and (A) is incorrect. (B) is incorrect because the best evidence rule
is inapplicable here. That rule requires that where the terms of a writing are material, the original
or its duplicate must be produced. Here, the copy of the report would have the same effect as the
original, but the report is privileged and therefore does not have to be produced. (D) is incorrect
because a report may be discovered by the opposing party even if it contains hearsay.

Answer to Question 11

(D) The friend’s comparison of the letters is not a proper basis for authenticating the employee’s
letter. Before a writing may be received into evidence, the writing must be authenticated by
proof showing that the writing is what the proponent claims it is. The Federal Rules list several
examples of proper methods of authentication through evidence of the genuineness of the
EVIDENCE - RELEVANCY, PRIVILEGES, AND WRITINGS ANSWERS 5.

handwriting of a letter writer. None of them, however, would permit an in-court comparison
by the friend of the contested letter with another letter established to be the employee’s. Under
Federal Rule of Evidence 901(b), an expert witness or the trier of fact can determine the genuine-
ness of a writing by comparing the questioned writing with another writing proved to be genuine.
Thus, (C) would be a proper basis for admitting the employee’s letter, because the jury, as the
trier of fact, can compare the letter with another letter that the employee has admitted writing.
The friend, however, cannot undertake that comparison because he is not an expert witness.
While Rule 901 does not limit the methods of authentication, the rule governing opinion testi-
mony by lay witnesses [Fed. R. Evid. 701] would preclude the friend’s in-court comparison. To
be admissible, opinion testimony by lay witnesses must be (i) rationally based on the perception
of the witness; (ii) helpful to a clear understanding of his testimony or to determination of a fact
in issue; and (iii) not based on scientific, technical, or other specialized knowledge. Here, the
friend can add nothing to the jury’s determination of the authenticity of the letter because the
jury can compare for itself that letter with a letter the employee has admitted writing. Because
the friend’s in-court comparison would not be helpful to the jury here, it would not be a proper
basis for authenticating the employee’s letter. (A) is incorrect because a lay witness who has
personal knowledge of the handwriting of the supposed writer may state her opinion as to whether
the document is in that person’s handwriting. Therefore, the employee’s wife could properly
authenticate the letter by testifying that she recognizes his handwriting. (B) is incorrect because
the friend’s testimony is proper circumstantial evidence of authentication under the reply letter
doctrine. Under this doctrine, a letter may be authenticated by evidence that it was written in
response to a communication sent to the claimed author. The content of the letter must make it
unlikely that it was written by anyone other than the claimed author of the writing. Here, the
employee’s letter to the friend can be properly authenticated by the friend’s testimony that it was a
reply to a letter that he sent to the employee.

Answer to Question 12

(D) The judge should overrule the objection because lay opinion is permissible (and often essen-
tial) to identify handwriting. A foundation must first be laid to establish familiarity with the
handwriting. (A) is wrong because expert testimony is not necessary to identify handwriting. (B)
is wrong because only a proper foundation is required for the admission of testimony identifying
handwriting. The fact that there are other individuals who may be more familiar with or in more
recent contact with the handwriting does not, of itself, preclude admissibility of the teacher’s
testimony. The fact that the teacher has not seen the singer’s handwriting for 10 years goes to the
credibility of her testimony but not to its admissibility. (C) is wrong because expert testimony is
not required for handwriting identification and, therefore, the witness need not be qualified as an
expert.

Answer to Question 13

(B) The physician’s testimony regarding the plaintiff’s statement is admissible because an examina-
tion for insurance purposes is not considered to be for diagnosis and treatment. To be privileged,
the information must be acquired by the physician in the course of treatment. Some states have
expanded this to include a consultation for diagnosis, but an insurance examination would not
qualify as diagnostic either. Thus, the plaintiff’s statement to the physician is not privileged. Since
it is not privileged and qualifies as an admission by a party-opponent for hearsay purposes, the
statement is admissible. (A) is wrong because an attempt by the plaintiff to perpetrate a fraud
is not the only way that the physician could testify to the plaintiff’s statement. It is true that the
physician-patient privilege does not apply if the physician’s services were sought or obtained
6. EVIDENCE - RELEVANCY, PRIVILEGES, AND WRITINGS ANSWERS

in aid of planning a crime or tort, but that is not the only time it is inapplicable. Moreover, it is
nearly impossible to believe that the plaintiff could have been attempting to lay the groundwork
for a fraudulent claim, given the facts that the insurance examination occurred before the accident
and the statement was against his interest. As noted above, the privilege never attached since the
plaintiff was not seeking diagnosis or treatment. Even if it had, the mere fact that the plaintiff
has put his physical condition in issue by suing for injuries is sufficient to abrogate the privilege.
There would be no need to resort to proof of an attempted fraud. (C) is wrong because the plain-
tiff’s statement, despite having been made while the physician was attending to the plaintiff in a
professional capacity, is not privileged for two reasons: (i) the plaintiff was not seeking diagnosis
or treatment, and (ii) he put his physical condition in issue. (D) is wrong because the preexisting
back injury is relevant on issues of causation and damages.

Answer to Question 14

(C) The objection should be sustained because the attorney-client privilege applies. A client has a
privilege to refuse to disclose, and to prevent others from disclosing, confidential communications
between himself (or his representative) and his lawyer (or her representative). A “representa-
tive of a lawyer” is one who is employed to assist in the rendition of legal services. If a physician
examines a client at the request of the attorney (e.g., to assess the extent of injury), the attorney-
client privilege applies to communications made to the physician because the physician is deemed
to be a representative of the attorney. (A) is incorrect because the statements are privileged under
the attorney-client privilege. Absent this privilege, the statements would be admissible as state-
ments of a party-opponent (commonly called admissions). An admission is a statement made by
a party and offered against that party. The plaintiff’s statements to the physician offered against
the plaintiff fall under this rule. (B) is incorrect because whether the physician qualifies as an
expert is irrelevant. This situation does not call for expert testimony. Expert opinion testimony
is appropriate when the subject matter is such that scientific, technical, or other specialized
knowledge would assist the finder of fact in understanding the evidence or determining a fact in
issue. The physician is not being called to give her opinion on some matter that calls for special-
ized knowledge, such as matters pertaining to medicine. Rather, she is being called to testify
that the plaintiff acknowledged his negligence. Thus, the principles of expert opinion testimony
are inapplicable to these facts. (D) is incorrect for two reasons: (i) The physician-patient privi-
lege protects only information that is necessary to enable the physician to act in her professional
capacity. Thus, if information given by the patient deals with a nonmedical matter, the informa-
tion is not privileged. The plaintiff’s statements concerning his negligence were not necessary to
enable the physician to treat or diagnose his condition. Therefore, this information obtained by the
physician is not covered by the privilege. (ii) The privilege is not applicable where the patient has
put his physical condition in issue by, e.g., suing for personal injuries. Here, the plaintiff is suing
the defendant for personal injuries. Consequently, even if the plaintiff’s statements to the physician
constituted information that would be deemed privileged, the privilege is not applicable in this
case.

Answer to Question 15

(B) The contractor can testify as to any firsthand knowledge he has and need not rely on any written
records if he presently remembers the facts. Firsthand knowledge is considered to be reliable
testimony. (A) is wrong because the records themselves are not being introduced. (C) is wrong
because the best evidence rule applies only in situations where the content of the writing is in
issue. Here, the contractor is testifying about facts he perceived, facts that exist apart from the
writing. The notebooks merely describe what the contractor saw and knows personally. (D) is
EVIDENCE - RELEVANCY, PRIVILEGES, AND WRITINGS ANSWERS 7.

wrong because no such summary is sought to be introduced, and even if it were, (D) is an incor-
rect statement of the law.

Answer to Question 16

(A) The judge should admit the witness’s testimony. Evidence that the artist was retrieving his own
painting from the customer’s wrongful possession would help negate the intent required for
burglary. Therefore, it would be relevant. The testimony would have no effect on the neighbor’s
credibility, so (B) and (C) are incorrect. (D) is not a very good answer. First, the objection is on
relevancy, not on Rule 403. In addition, rejecting the evidence on Rule 403 grounds would be
extremely unlikely on the facts presented.

Answer to Question 17

(A) Testimony regarding the defendant’s attempt to bribe the witness is admissible as substantive
evidence against the defendant. Under the Federal Rules, a statement made by a party and offered
against that party (commonly called an admission) is not hearsay. Various kinds of conduct,
including attempts to bribe witnesses, may be held to manifest an awareness of liability or guilt.
Because the defendant’s liability is the issue (i.e., a relevant fact) in the case, his attempt to bribe
the witness is admissible as a statement of a party-opponent. (B) is wrong because, as discussed
above, the offer is also admissible as a statement of a party, and thus is not limited to impeach-
ment. Moreover, the witness’s testimony would not be admissible as evidence of a specific
instance of misconduct to impeach the defendant. A specific act of misconduct can be elicited
only on cross-examination; extrinsic evidence is not permitted. The facts do not even indicate
that the defendant has testified. (C) is wrong because the offer to bribe the witness is relevant.
Evidence is relevant if it tends to make the existence of a fact of consequence to the action more
probable or less probable than it would be without the evidence. Evidence that the defendant tried
to bribe a witness to testify falsely makes it more probable that the accident was his fault; thus,
it is relevant. (D) is wrong because the fact that relevant evidence is misleading or prejudicial is
not a sufficient reason, by itself, to exclude the evidence. Under Federal Rule 403, the judge has
discretion to exclude otherwise admissible, relevant evidence if its probative value is substantially
outweighed by the danger of unfair prejudice, confusion of the issues, misleading the jury, or by
considerations of undue delay, waste of time, or needless presentation of cumulative evidence.
Here, the choice states only that the evidence is misleading and prejudicial; it does not state
that the probative value of the evidence is substantially outweighed by these facts. Furthermore,
bribing a witness is highly probative of guilt, so it is unlikely to be misleading. Although the
evidence is prejudicial, it is not at all clear that the prejudice would be unfair since there are few
reasons other than guilt for bribing a witness.

Answer to Question 18

(C) The evidence should not be admitted. Under the Federal Rules of Evidence, evidence of subse-
quent remedial measures is inadmissible to show negligence. This is based on the public policy of
safety. The law wants to encourage people to make repairs or take other remedial measures after
an accident without being held back by a fear of liability. Thus, (A) and (B) are incorrect. (D) is
wrong because the woman is not trying to impeach the manager or anyone else.
REAL PROPERTY -
ESTATES, LEASES, CO-
TENANCIES QUESTIONS

l
REAL PROPERTY - ESTATES, LEASES, CO-TENANCIES QUESTIONS 1.

REAL PROPERTY - ESTATES, LEASES, CO-TENANCIES QUESTIONS

Question 1 Question 2

A landowner developed and built a large A brother and a sister held real property
apartment house on property he owned. After as joint tenants. The sister was involved in an
the apartment house was completed, the automobile accident and was sued by a motorist
landowner conveyed the building and the land who had received serious bodily injuries. The
to his son for life, remainder to his grand- jury ruled against the sister and assessed a large
daughter; subject, however, to a bank’s mortgage. damages award that the sister was unable to
The bank held a mortgage on the building for pay in full. Therefore, the motorist went back
$17,000, payable in annual installments of into court and secured a statutory lien on the
$2,000 plus interest. property. Shortly thereafter, the sister died.

The son began to manage the day-to-day What are the respective interests of the
maintenance of the building as well as to collect brother and the motorist in the property?
rents. Despite the son’s best efforts, the vacancy
factor reached an unanticipated high and the (A) The brother is the sole owner of the proper-
rental income was not sufficient to cover the ty, but the property is subject to the motor-
annual mortgage payment. ist’s statutory lien.

Who is responsible for the mortgage payments (B) The brother is the sole owner of the
and in what amount? property, and the property is not subject to
the motorist’s statutory lien.
(A) The son pays all.
(C) The brother and the motorist own the
(B) The granddaughter pays all. property as tenants in common.

(C) The son pays the interest and the grand- (D) The brother and the motorist own the
daughter pays the principal. property in joint tenancy.

(D) The granddaughter pays the interest and the


son pays the principal.
2. REAL PROPERTY - ESTATES, LEASES, CO-TENANCIES QUESTIONS

Question 3 Question 4

A grandfather conveyed a parcel of land “to A rancher deeded his ranch “to my son so
my granddaughter and her heirs, but should long as he lives, and on his death to my son’s
my granddaughter or her successors attempt to widow for the remainder of her life, and on her
convey the property, then to my grandson and death to each of her lineal descendants who are
his heirs.” living on her death, to share and share alike.”
The deed was validly executed, delivered, and
How is the grandson’s interest in the property recorded. At the time of the conveyance, the
best described during his lifetime? son was married but had no children. Three
years later, the rancher died. His will left all
(A) An executory interest. of his estate to his sister. Two years after the
rancher’s death, the son was killed in an automo-
(B) A contingent remainder. bile accident, leaving his widow with twin
girls. Upon the widow’s death many years later,
(C) A vested remainder. she was survived by both girls, her only lineal
descendants.
(D) No interest.
Who now owns the ranch?

(A) The girls are tenants in common, with each


owning an undivided one-half; their inter-
est vested on the son’s widow’s death, well
within the period of lives in being plus 21
years.

(B) The girls are tenants in common, with


each owning an undivided one-half; they
received a vested remainder, which is not
subject to the Rule Against Perpetuities.

(C) The sister owns the ranch in fee simple,


because the Rule Against Perpetuities
voided the gift over to the son’s widow and
the following gift to her lineal descendants.

(D) The sister owns the ranch in fee simple,


which she received upon the death of the
son’s widow.
REAL PROPERTY - ESTATES, LEASES, CO-TENANCIES QUESTIONS 3.

Question 5 Question 6

A landlord leased residential property to Two brothers inherited a tract of land as


a tenant. The written lease was for a period joint tenants from their parents. Only half of
of one year, with the monthly rent of $1,000 the acreage was planted with oranges, and the
payable on or before the first of each month. older brother decided that to increase produc-
The termination date set out in the lease was tivity, the balance of the land should be planted
October 1. On August 10 of the first year of with a different variety of oranges. The younger
her tenancy, the tenant received a letter from brother refused to participate in the project, and
the landlord along with a new lease form. The the older brother brought an action for declara-
lease was for a period to terminate on October tory relief alleging that he was entitled to be
1 of the following year, and the rent stated in reimbursed for half of the cost of the improve-
the new lease was $1,200 per month. Both the ment.
rent increase and the notice given were in full
compliance with relevant state statutes. An Which of the following correctly describes the
accompanying letter, signed by the landlord, result in the action?
asked the tenant to sign the lease on the line
marked “tenant.” On September 15, the tenant (A) The older brother will be denied the re-
sent the lease back to the landlord unsigned. quested judgment.
On September 20, the tenant sent a letter to the
landlord by certified mail. The landlord signed (B) The older brother will be granted the
the return receipt, which the post office duly sent requested judgment.
to the tenant. Enclosed with the tenant’s letter
was a check for $1,000 for “next month’s rent.” (C) The result depends upon whether the
The landlord deposited the check into his bank younger brother will receive an economic
account. With the landlord’s acquiescence, the benefit.
tenant remained in possession after October 1.
(D) The result depends upon whether the
Which of the following statements is most improvements will produce sufficient
accurate? income to amortize their cost.

(A) The tenant has a month-to-month tenancy


at $1,000 monthly rent.

(B) The tenant has a month-to-month tenancy


at $1,200 monthly rent.

(C) The tenant has an annual tenancy at $1,200


per month rent.

(D) The tenant has a tenancy at will.


4. REAL PROPERTY - ESTATES, LEASES, CO-TENANCIES QUESTIONS

Question 7 Question 8

While dating, a man and woman purchased By mutual agreement, a brother and sister
a parcel of land for $100,000, each contributing purchased a 10-acre parcel of land and took
half the purchase price, and took title as joint title “as joint tenants with right of survivorship.”
tenants with right of survivorship. Two years Three years after the purchase, the brother asked
after the couple were married, they separated. if he could build an apartment house on his half
The man then quitclaimed all of his interest in of the property; the sister agreed. He then built
the land to his brother, who duly recorded the an apartment house on the eastern five acres of
deed. The jurisdiction has no applicable statute. the property. Two years later, the brother died,
leaving his entire estate to his son.
The land is now held by:
In an action for partition, what is the best
(A) The woman and brother, as joint tenants argument in favor of the brother’s estate being
with right of survivorship. judged the owner of the eastern five acres?

(B) The woman and brother, as tenants in (A) The sister’s conduct during the brother’s
common. lifetime estopped her from asserting title to
the eastern half of the property.
(C) The man and woman, as tenants by the
entirety. (B) The taking of title as joint tenants does not
conclusively presume that the property is
(D) The man and woman, as joint tenants with held as joint tenants.
right of survivorship.
(C) The joint tenancy was terminated at the
time the oral agreement was made.

(D) A joint tenant may will away his interest in


property to a lineal descendant.
REAL PROPERTY - ESTATES, LEASES, CO-TENANCIES QUESTIONS 5.

Question 9 Question 10

A landowner and his friend owned a tract A landowner devised her parcel of land to her
of land as joint tenants with right of survivor- daughter, her heirs, and assigns, “so long as the
ship. The landowner executed a deed conveying property is used for residential purposes, then to
his interest in the land to his grandson. The my niece, her heirs, and assigns.” The remainder
landowner gave the deed to his attorney with of the landowner’s property passed through the
instructions to deliver it to the grandson upon the residuary clause of her will to her grandson. The
landowner’s death. The grandson first learned of daughter lived on the land for 25 years; then, on
the deed at the landowner’s funeral the following her death, ownership passed to her husband. In
year. The next day, the grandson recorded the the meantime, the niece had also died, leaving
deed. her entire estate to her son. The husband has
leased the land to a developer, who has obtained
Who owns the land? the necessary permits to build a shopping center
on it. The grandson and the niece’s son both
(A) The friend and the grandson, as joint ten- file quiet title and ejectment actions against the
ants. husband, and the cases are consolidated.
(B) The friend and the grandson, as tenants in How should the court, applying common law,
common. rule as to ownership of the land?
(C) The friend. (A) For the husband.
(D) The grandson. (B) For the niece’s son.

(C) For the grandson, because he received a


right of reversion from his grandmother.

(D) For the grandson, because he received


a possibility of reverter from his grand-
mother.
6. REAL PROPERTY - ESTATES, LEASES, CO-TENANCIES QUESTIONS

Question 11 Question 12

A father owned a piece of property located A tenant rented an apartment in a large


near the state university. His son was a student multi-unit building. One day vandals broke into
at the university, and the father allowed the several of the building’s apartments, including
son and the son’s classmate to live in the large the tenant’s, and smeared excrement into the
house on the property rent free. On the son’s carpets and on the walls, and broke out all of the
20th birthday, the father handed the son a windows. The jurisdiction provides by statute
signed instrument of conveyance containing that if a tenant notifies her landlord in writing of
the following language: “I give the property to a repair that is needed to keep the premises in
my dearly beloved son on the condition prece- a habitable condition and the landlord does not
dent that he receive a college degree before he repair it within 15 days, the tenant may, at her
reaches the age of 30, and if he does not receive option, either repair it herself and withhold the
the degree by his 30th birthday, to my beloved expenses from rent, or consider herself construc-
daughter.” The son promptly recorded the deed. tively evicted and terminate her tenancy. The
Shortly thereafter, the son asked his classmate tenant wrote a letter to the landlord informing
to start paying him rent. An argument evolved him that her walls, carpets, and windows had
out of the son’s request and the son told the been damaged and needed repair, and after eight
classmate to get off his property. The classmate days she received a letter in reply stating that
refused to leave and the son filed suit to evict such damages to her apartment were her respon-
him. sibility to repair.

How will the court rule? After waiting another week, the tenant paid
to have her carpets and walls cleaned and to
(A) The son loses, because the son is attempt- have her windows replaced. She then withheld
ing a retaliatory eviction. the entire next month’s rent of $400, because
the cleaning and repair bills had totaled $750.
(B) The son loses, because the classmate is the After sending her the required statutory notices,
father’s tenant and not the son’s. the landlord commenced unlawful detainer
litigation, seeking to have the tenant evicted for
(C) The son wins, because he has a fee simple nonpayment of rent.
subject to divestment.
How should the court rule?
(D) The son wins, because the father’s convey-
ance cuts off the classmate’s tenancy. (A) For the landlord, because the damage was
the result of the criminal acts of a third
party.

(B) For the landlord, because the damage


was to a private apartment and not to the
common areas of the apartment complex.

(C) For the tenant, if she can show that the


landlord was negligent in connection with
the vandalism.

(D) For the tenant, because she satisfied the


requirements of the statute.
REAL PROPERTY - ESTATES, LEASES, CO-TENANCIES QUESTIONS 7.

Question 13 Question 14

A college student assigned her apartment A renter rented a house from a landlord. The
rental lease, by written agreement, to one of renter and the landlord entered into a written
her sorority sisters. The sorority sister, in turn, lease providing that the renter was to pay a
assigned the lease to a classmate. fixed monthly rent plus all taxes. A month later,
the renter was offered a job in another state,
If the classmate fails to pay the rent, can the and the renter assigned her lease to a friend by
landlord bring suit against the sorority sister to written agreement. The renter forgot to tell the
recover this money? friend that he was liable to pay the taxes on the
residence during the period of the lease. As a
(A) Yes, because the sorority sister remains in result, the friend failed to pay the taxes on the
privity of estate with the landlord. house. The landlord was informed that there was
a tax lien on the residence. She paid the lien and
(B) Yes, because the sorority sister remains in brought suit against the friend for the amount.
privity of contract with the landlord.
Who will prevail?
(C) No, because the sorority sister is no longer
in privity of estate with the landlord. (A) The friend, because under the terms of his
assignment with the renter he is not liable
(D) No, because the fact that the college student for the taxes.
was allowed to assign the lease means that
the sorority sister is allowed to assign it (B) The friend, because the agreement to pay
also. taxes was collateral, and thus is not a
covenant running with the land.

(C) The friend, because in order for the agree-


ment to be a covenant running with the land
it must be expressly stated in the original
agreement.

(D) The landlord, because payment of property


taxes touches and concerns the land.
8. REAL PROPERTY - ESTATES, LEASES, CO-TENANCIES QUESTIONS

Question 15 (D) The landlord was not obligated to attempt


to re-rent the property to help lower the
A tenant moved out of the dormitory at a state tenant’s damages, and under these facts the
college and rented a one-bedroom apartment not tenant is only liable for the unpaid rent up
far from campus. She signed a one-year lease to the time the landlord retook the apart-
and moved in on September 1, two weeks before ment, and is not liable for any unpaid rent
classes began. Rent was specified in the lease up to the re-renting, nor for any deficiency
to be $750 per month. When classes began, she in rent.
started walking from her apartment building
to the college, and quickly found that the walk, Question 16
much of which was on a busy street with no
sidewalks, was much more difficult than she had An attorney was staying at a hotel while
thought it would be. After a month, the tenant traveling to work on a case. The normal room
moved out when she found another apartment. charges were $150 per night. The attorney made
She sent the landlord the keys, together with a a deal with the hotel management to pay $700
note apologizing for the sudden departure. per week for his room for an indefinite period.
The attorney never told the hotel management
The landlord immediately cleaned up the exactly how long he planned to stay or how he
apartment and placed an ad in the paper. Since would pay, but each Sunday he would present
school had already started, it was difficult to the cashier with a check for $700, plus any
find someone to take the apartment, and when additional taxes and room charges, to pay for
the landlord found someone, the rent had to be the previous week’s lodging. He did this for four
lowered to $620. weeks, and then the case he was working on was
settled. On a Thursday morning, before check-
Which of the following is the most accurate out time for that day, the attorney tendered the
statement concerning the landlord’s rights? hotel cashier a check for $400 plus additional
charges incurred from Sunday through
(A) The landlord was obligated to attempt to Wednesday nights. The cashier promptly took
re-rent the property to help lower the ten- the check and then demanded that the attorney
ant’s damages, but will be able to recover pay an additional $1,000 plus taxes for 10 more
his costs of the re-rental, any unpaid rent days’ lodging, covering the rest of the week
until he re-rented the apartment, and $130 plus an additional week because he did not give
per month for the balance of the lease. a week’s notice that he planned to vacate. The
attorney refused to pay the additional charges. If
(B) The landlord was not obligated to attempt the hotel sues the attorney and the court deter-
to re-rent the property to help lower the mines that the parties created a tenancy, for
tenant’s damages, but since he did he will how many additional days’ lodging will he be
be able to recover the costs of re-rental, any required to pay?
unpaid rent until he re-rented the apart-
ment, and $130 per month for the balance (A) None, because a tenancy at will was cre-
of the lease. ated.
(C) The landlord was obligated to attempt (B) Three days, because a periodic tenancy was
to re-rent the property to help lower the created.
tenant’s damages, but will be unable to
recover the costs of the re-rental since it (C) Seven days, because a periodic tenancy was
was mandatory; however, he will be able to created.
recover the difference in rent between what
the tenant promised to pay and the amount (D) Ten days, because a periodic tenancy was
secured on the re-rental. created.
REAL PROPERTY - ESTATES, LEASES, CO-TENANCIES QUESTIONS 9.

Question 17 Question 18

A landlord owns a six-unit strip mall. He A landlord leased an apartment to a tenant


offered to lease one of the units to a barista, for five years. The lease provided that the
who planned to open a gourmet coffee shop. landlord will: (i) keep the apartment building at
Because the mall is located on a busy street, the a comfortable temperature 24 hours per day, and
barista insisted that the landlord install a metal (ii) have the carpets cleaned once a year. Two
fence around the outdoor seating area that would years later, the landlord began turning off the air
protect her customers. The landlord agreed to conditioning at 10 p.m. The tenant’s apartment
do so, and the barista signed a 10-year lease. A became hot and stuffy, and she demanded that
term in the lease stated, “The landlord agrees to the landlord honor the covenant. The landlord
maintain all structures on the property in good refused. The following month, the pipes burst
repair.” Eight years later, the landlord sold the in the tenant’s only bathroom, rendering it
strip mall to a buyer. The buyer did not agree unusable. The resultant flooding soiled some
to perform any obligations under the leases. As of the carpeting, which had not been cleaned
instructed, the barista began paying rent to the in the past 12 months. The tenant reported the
buyer. The following year, the metal fence was problems to the landlord, who did not return the
in desperate need of repairs. Citing the lease tenant’s phone calls.
terms, the barista asked the buyer to repair the
fence, but the buyer continually refused to do so. Which of the following are valid reasons for
The barista finally repaired the fence herself at a the tenant to terminate the lease?
cost of $3,000 and then brought an appropriate
lawsuit to recover the money. (A) Only that the landlord did not keep the
apartment building at a comfortable tem-
Absent any other facts, what is the barista perature 24 hours per day.
likely to recover?
(B) Only that the landlord did not fix the
(A) $3,000 from either the landlord or the bathroom pipes.
buyer, because they are both in privity with
the barista. (C) That the landlord did not keep the apart-
ment building at a comfortable tempera-
(B) $2,700 from the landlord and $300 from ture 24 hours per day and did not fix the
the buyer, because that represents their pro bathroom pipes.
rata shares.
(D) That the landlord did not keep the apart-
(C) $3,000 from the landlord only, because the ment building at a comfortable temperature
buyer did not assume the lease obligations. 24 hours per day, did not have the carpets
cleaned, and did not fix the bathroom pipes.
(D) $3,000 from the buyer only, because a
covenant to repair runs with the land.
REAL PROPERTY -
ESTATES, LEASES, CO-
TENANCIES ANSWERS
REAL PROPERTY - ESTATES, LEASES, CO-TENANCIES ANSWERS 1.

REAL PROPERTY - ESTATES, LEASES, CO-TENANCIES ANSWERS

Answer to Question 1

(C) The son is responsible for the interest payments on the mortgage and the granddaughter is respon-
sible for the principal. The doctrine of waste governs the obligations between a life tenant and the
holder of the remainder regarding the payment of a mortgage on the property. Under this doctrine,
a life tenant is obligated to pay interest on any encumbrances on the land, but he does not have to
pay anything on the principal of the debt; reversioners or remaindermen must pay the principal in
order to protect their interests. (C) correctly states that rule, and (A), (B), and (D) are necessarily
wrong. Note, however, that a life tenant’s duty to pay carrying charges, such as mortgage interest,
is limited to the extent of the income or profits derived from the land (or if there is no actual
income or profit, to the extent of the reasonable rental value of the land). Thus, if the apartment
building is not generating enough rent to cover the interest, the son is liable only to the extent of
the rent.

Answer to Question 2

(B) The motorist’s lien was extinguished at the sister’s death, leaving the brother as the sole owner of
the property. The distinguishing characteristic of a joint tenancy, which is how the brother and
the sister held the property, is the right of survivorship. When one joint tenant dies, the property
is freed of her concurrent interest and the survivor continues to retain an undivided right in the
property no longer subject to the interest of the deceased co-tenant. The survivor does not succeed
to the decedent’s interest; he holds free of it. Hence, as long as the joint tenancy is still intact, the
decedent’s devisees, heirs, and judgment creditors have no claim on the joint tenancy property
that the decedent held. For the joint tenancy to remain intact, the unities of time, title, interest,
and possession that were necessary to create the tenancy must be undisturbed. Any disturbance
of the unities causes a severance of the joint tenancy, and thereafter the parties whose unities are
disturbed hold as tenants in common. In most jurisdictions, however, the fact that a creditor has
obtained a lien on one joint tenant’s interest does not by itself result in severance; there must also
be at least a judicial sale of the property. Thus, judgment and other lien creditors easily can lose
their security interests by the death of the debtor. That is what happened to the motorist in this
case. He obtained a statutory lien on the property through the sister’s interest as a joint tenant.
This did not cause a severance of the joint tenancy; the brother and the sister remained joint
tenants. When the sister died, the property was freed of her concurrent interest and any claims
arising through that interest. The motorist, not having caused a judicial sale of the property, loses
his security interest. Thus, (A) is incorrect because the brother is the sole owner of the property,
and the property is not subject to the motorist’s statutory lien. (C) would be the correct answer
if the motorist had foreclosed on his lien. The sheriff’s deed issued to the buyer at the judicial
sale would have severed the joint tenancy. However, a transferee takes his interest as a tenant in
common and not as a joint tenant. Thus, (D) is incorrect.

Answer to Question 3

(D) The grandson has no interest in the land. The language of the grant creates a fee simple subject
to a condition subsequent in the granddaughter and an executory interest in the grandson. The
condition subsequent, however, attempts to restrict the transferability of a fee simple. As a general
rule, any restriction on the transferability of a legal interest in property is void: The restriction
violates the common law Rule Against Restraints on Alienation. “Restraint on alienation” means
an express restriction on the transferability of property. Like the Rule Against Perpetuities, this
2. REAL PROPERTY - ESTATES, LEASES, CO-TENANCIES ANSWERS

is a rule of public policy that is designed to prevent property from being tied up and taken out of
commerce. There are three types of restraints on alienation: (i) disabling restraints, under which
any attempted transfer is ineffective; (ii) forfeiture restraints, under which an attempted transfer
results in a forfeiture of the interest; and (iii) promissory restraints, under which an attempted
transfer breaches a covenant. Any total restraint on a fee simple—either forfeiture, disabling, or
promissory—is void. The provision here is a total forfeiture restriction on a fee simple and there-
fore is void. When the condition is stricken, the granddaughter has a fee simple absolute, and the
grandson is left with nothing. Thus, (A) is incorrect because, as explained, the grandson’s interest
is void. (B) and (C) are incorrect for the same reason, and also because a remainder cannot follow
a fee simple interest; it can only follow a fee tail or a life estate. Here, the granddaughter was
conveyed a fee simple estate.

Answer to Question 4

(D) The sister owns the ranch, because the gift over to the son’s widow’s descendants was void. This
is the case of the unborn widow. Remember, the Rule Against Perpetuities is measured and
enforced at the time of the grant, not later. If there is any way, at the time of the grant, that a gift
over may vest later than any life in being plus 21 years, it is void. Take the grants one at a time,
in order. The gift to the son is valid; he was alive at the time of the grant. The gift to the son’s
widow also is valid. Although her life estate is a contingent remainder (because we would not
know the identity of the son’s widow until the son dies), we would know her identity (and thus the
remainder will become vested) at the moment of the son’s death, clearly within the time period
of the Rule. But the gift to the widow’s descendants violates the Rule Against Perpetuities. The
son’s widow was not necessarily born and in existence at the time of the conveyance, and we will
not know who the widow’s descendants are until she dies, which may be well outside the lives
in being at the time of the rancher’s transfer, plus 21 years. Since the gift to the widow’s descen-
dants is void, the grant effectively reads: “to my son for life, then to my son’s widow for life.”
That would have left a reversion in the rancher, which the sister inherited under the rancher’s
will. The fact that the son’s widow was in fact alive at the time of the rancher’s grant does not
matter because the Rule Against Perpetuities is applied at the time of the grant. Subsequent facts
are irrelevant. (A) is incorrect. If we cared what actually happened after the grant in question,
this might be a good answer, but we do not. The validity of interests is measured as of the time
of the conveyance, and if it later turns out that the facts show a vesting within the Rule, it does
not matter if there was any way that the interests could vest outside the Rule at the moment the
interests were created. (B) is incorrect. The girls do not have a vested remainder at the time of the
grant. Their interest is a contingent remainder (they are not born yet) and it does not even become
vested on the son’s death, because we will not know which of the widow’s lineal descendants will
survive her until she dies; thus, this vesting could come outside any life in being at the time of the
grant, plus 21 years. (C) is incorrect. The sister does own the ranch in fee simple, but not because
the Rule Against Perpetuities voids the gift to the son’s widow. It does not. Even if she is not yet
born at the time of the grant, we will know who she is and her interest will vest on the son’s death.
Because the son is alive at the time of the grant, his widow’s interest will clearly vest within the
Rule. It is the gift to the widow’s descendants after she dies that violates the Rule.

Answer to Question 5

(B) The tenant has a month-to-month tenancy at $1,200 per month. When a tenant continues in
possession after termination of her right to possession, the landlord may bind the tenant to a
new periodic tenancy. While the terms and conditions of the expired tenancy generally apply to
the new tenancy, if the landlord notifies the tenant before termination that occupancy after the
REAL PROPERTY - ESTATES, LEASES, CO-TENANCIES ANSWERS 3.

termination date will be at an increased rent, the tenant will be held to have acquiesced to the new
terms if she does not surrender. This is so even if the tenant objects to the increased rent, as long
as the rent increase is reasonable. (A) is therefore incorrect. (C) is also incorrect. In commercial
leases, where the original lease term was for a year or more, a year-to-year tenancy results from
holding over. In residential leases, however, most courts would rule that the tenant is a month-to-
month tenant, irrespective of the term of the original lease. Hence, the tenancy would be month-
to-month rather than annual. (D) is incorrect because a tenancy at will generally arises from a
specific understanding between the parties that either party may terminate the tenancy at any
time. Unless the parties expressly agree to a tenancy at will, the payment of regular rent will cause
a court to treat the tenancy as a periodic tenancy.

Answer to Question 6

(A) The older brother will be denied the requested judgment. A co-tenant has no duty to improve the
property and cannot force the other co-tenants to contribute to the cost of improvements made
by him. Only in an action for partition can the value of improvements be recouped. Thus, (B) is
wrong. (C) and (D) are wrong because the results of an improvement are irrelevant; a co-tenant
cannot force his other co-tenants to contribute to the improvement.

Answer to Question 7

(B) An inter vivos conveyance by one joint tenant of his interest in the property severs the joint
tenancy and changes it to a tenancy in common. (A) is wrong because, as discussed above, sever-
ance results in a tenancy in common. The woman and brother could not hold as joint tenants
because the unity of time is lacking. (C) is incorrect because a tenancy by the entirety can be
created only in a husband and wife. The couple was not married until after the creation of the
tenancy, and their marriage does not change the nature of their title to the land. (D) is wrong
because the man’s conveyance to his brother severed the joint tenancy.

Answer to Question 8

(A) Because the sister informed her brother that he could develop the eastern five acres of the land,
and because he reasonably relied on her statement to his detriment, she may be estopped to
deny her brother’s likely intended effect, i.e., to develop the property and pass on the benefit of
the improved portion to his son. (B) is incorrect. Where the language of the grant is clear and a
joint tenancy is created, no presumption is needed or applies. The joint tenancy cannot later be
changed by subsequent informal action. (C) is wrong because an oral agreement is not effective to
terminate a joint tenancy. (D) is an incorrect statement of the law. A joint tenant may not pass an
interest to anyone at death, due to the right of survivorship.

Answer to Question 9

(C) The friend owns the land. A joint tenancy is a concurrent estate in land in which each co-tenant
has an undivided right in the property and a right of survivorship—when one joint tenant dies,
the property is freed of his concurrent interest and the survivor retains an undivided right in the
property that is no longer subject to the interest of the deceased co-tenant. An inter vivos convey-
ance by one joint tenant, even a “secret” deed that is to take effect only upon the grantor’s death,
severs a joint tenancy. However, although acceptance (presumed or otherwise) usually “relates
back” to the date of delivery of the deed in escrow, many courts refuse to relate back an accep-
tance where it would defeat the rights of intervening third parties, such as surviving joint tenants.
4. REAL PROPERTY - ESTATES, LEASES, CO-TENANCIES ANSWERS

Thus, the grantee’s acceptance of the deed after the grantor’s death does not relate back to defeat
the right of survivorship. Here, the grandson did not accept the deed until after the landowner’s
death. In the meantime and because of that death, the friend’s right to the whole property had
accrued to her as the surviving joint tenant. Thus, the grandson has no interest in the land, and
(A), (B), and (D) are incorrect. Note that if the landowner’s conveyance had been effective, (B)
rather than (A) would have been the correct answer. Upon severance of a joint tenancy by inter
vivos conveyance, the new tenant holds as a tenant in common with the remaining joint tenant,
and not as a joint tenant.

Answer to Question 10

(D) The court should rule for the grandson because he received a possibility of reverter from his
grandmother. The landowner attempted to give the daughter and her successors a fee simple
subject to an executory interest, with the niece and her successors holding the executory interest.
However, the attempted gift to the niece and her successors fails under the Rule Against Perpetu-
ities because the niece’s interest could vest in possession more than 21 years after a life in being.
Thus, (B) is incorrect. After the void interest is stricken, the daughter and her successors have a
fee simple determinable and the landowner retained a possibility of reverter, which passed to the
grandson through the residuary clause in the landowner’s will. When the husband, the daughter’s
successor, ceased using the property for residential purposes, the possibility of reverter matured,
leaving ownership in the grandson. (A) is therefore incorrect. (C) is incorrect because a possibility
of reverter, not a right of reversion, is the interest left in the grantor when a fee simple determin-
able is created.

Answer to Question 11

(B) The son cannot evict the classmate because he is not yet the owner of the property. The instru-
ment of conveyance signed by the father conveyed only a future interest and not a present estate.
A future interest is an estate that does not entitle the owner thereof to possession, but it will or
may in the future become a present interest. Because the father’s estate is not a life estate, the
future interest conveyed to the son is an executory interest. Furthermore, it is a springing execu-
tory interest because it divests the estate of the transferor, the father (i.e., the father’s fee simple
will be cut short if and when the son receives his degree before age 30). Because the father has
retained his present estate, which is a fee simple subject to the springing executory interests of his
son and his daughter, only the father would have the power to evict the classmate. The son, as the
holder of a future interest, has no right to possession of the property and therefore has no cause of
action against another in possession. (A) is incorrect. If the son had acquired a present possessory
interest in the property, nothing in the facts would have prevented him from evicting the class-
mate, regardless of his motivation. (C) is incorrect because the language of the father’s convey-
ance clearly indicates that he is conveying a future interest rather than a present interest. (C)
would have been correct if the father had used the following language in his conveyance: “to my
son; provided, however, that if he does not receive a college degree by his 30th birthday, title shall
pass to my daughter.” The son would have had a fee simple subject to divestment by the daughter’s
executory interest. Here, however, the father expressly stated that the son’s receipt of a college
degree was a condition precedent to the transfer of the property. Hence, the son has only a future
interest and not a fee simple of any kind. (D) is incorrect. If the father had conveyed his present
interest in the property, the classmate’s tenancy would have been terminated by operation of law
because it was, at most, a tenancy at will. However, the father conveyed only a future interest to
the son, so the conveyance had no effect on the classmate’s tenancy.
REAL PROPERTY - ESTATES, LEASES, CO-TENANCIES ANSWERS 5.

Answer to Question 12

(D) The tenant will win because she had a right under the statute to withhold the rent. The general
rule at common law was that the landlord was not liable to the tenant for damages caused by the
landlord’s failure to maintain the premises during the period of the leasehold. Today, however,
a majority of jurisdictions, usually by statute, provide for an implied warranty of habitability
for residential tenancies. The statute in this question allows the tenant to make the repairs and
withhold the cost of the repairs 15 days after notifying the landlord in writing. The statute is
applicable because the damage done by the vandals makes the apartment unfit for habitation under
whatever standard the court would apply. The tenant has complied with the terms of the statute;
she therefore cannot be evicted for nonpayment of rent. (A) is incorrect. The fact that the damage
was caused by a third party would be relevant only if the tenant were relying on the judicially
developed remedy of constructive eviction, which requires that the damage making the premises
uninhabitable have been caused by the landlord. The warranty of habitability is not limited in this
way. (B) is incorrect because the statute has extended the common law duty of the landlord, which
applied only to the common areas of a multiunit building. (C) is incorrect because the tenant does
not need to show that the landlord was negligent.

Answer to Question 13

(C) The landlord cannot bring suit against the sorority sister. Absent an express assumption, an
assignee is not liable on the original covenants once she reassigns. Hence, (A) is wrong. (B)
is wrong because an assignee is not in privity of contract with the lessor unless the assignee
expressly assumes the lease obligations. (D) is incorrect because the sorority sister’s ability
to subsequently assign the lease is unrelated to the issue of liability. The sorority sister, as an
assignee of the college student, is liable for covenants running with the land on the basis of privity
of estate. Once the sorority sister assigned the lease to her classmate, the privity of estate ends and
the sorority sister’s liability is terminated.

Answer to Question 14

(D) Judgment should be for the landlord, because payment of property taxes touches and concerns
the land. An assignee is in privity of estate with the lessor, and is liable for those covenants in
the original lease that run with the land. Hence, (A) is wrong. It does not matter that the terms
of the friend’s agreement did not mention the taxes. The friend accepted the assignment of the
renter’s lease. By doing so, the friend obtained both the benefit and the burden of the terms of the
assigned lease that ran with the land. (B) and (C) are wrong because an agreement to pay taxes
touches and concerns the land, and thus runs with the land.

Answer to Question 15

(A) The landlord was obligated to attempt to re-rent the property but may recover all of his damages.
When a tenant wrongfully abandons the leasehold, the landlord has two choices: (i) treat the
abandonment as an offer of surrender and accept by retaking the property, thus ending the
leasehold and terminating the tenant’s obligations from that moment; or (ii) attempt to relet the
property on the tenant’s account and hold the tenant liable for any difference between what the
lease specified and what the landlord can now get on the reletting. For the landlord to choose
option (ii), the mitigation of the tenant’s damages and holding the tenant liable for the difference,
the landlord’s retaking and reletting must be done to mitigate the tenant’s damages. (B) and (D)
are incorrect, since the landlord is under an obligation to mitigate the tenant’s damages. (C) is
6. REAL PROPERTY - ESTATES, LEASES, CO-TENANCIES ANSWERS

incorrect. Since a reletting was being attempted, the landlord could recover all losses, including
the costs of reletting, the unpaid rent, and the difference in rent.

Answer to Question 16

(D) The attorney must pay 10 days’ lodging because his notice to terminate will not become effec-
tive until then. In the usual case, a hotel guest is treated as a licensee rather than a tenant. Here,
however, the court found that the parties created a tenancy because they specifically agreed to a
week-to-week arrangement at a special weekly rate. A periodic tenancy arises when the parties
do not fix the duration of the tenancy. It continues from period to period and is automatically
renewed for another period until terminated by the giving of proper notice. The period is gener-
ally based on an express understanding between the parties or an implied understanding based
on the payment of rent. The attorney’s arrangement with the hotel constitutes a periodic week-
to-week tenancy because it was of indefinite duration and “rent” was paid each week. To termi-
nate a periodic tenancy of less than one year, a full period in advance of the period in question
is required by way of notice. For a week-to-week tenancy, notice at least one full week prior to
vacating would have to be given prior to the beginning of the period. Thus, the attorney’s notice
had no effect on his liability for that week’s rent and would operate as one week’s notice starting
at the beginning of the next week. (A) is incorrect. If the parties had expressly agreed that either
party could terminate at any time, a tenancy at will would have been created and the attorney
would be liable for nothing. However, the payment of rent on a regular periodic basis will cause
a court to treat the tenancy as a periodic tenancy. (B) is incorrect because it does not take the one
week’s notice requirement into account. (C) is incorrect because the notice must fix the last day
of the period as the date of termination rather than some intervening day. Thus, the notice period
does not begin to run until the end of the current week.

Answer to Question 17

(A) The barista may recover the cost of the repairs from either the landlord or the buyer. A landlord’s
promise in a lease to maintain the property does not terminate because the property is sold.
Although no longer in privity of estate, the original landlord and tenant remain in privity of
contract, and the original landlord remains liable on the covenant unless there is a novation. A
novation substitutes a new party for an original party to the contract. It requires the assent of
all parties, and completely releases the original party. Because neither the barista nor the buyer
has agreed to a novation, the landlord remains liable for the covenant because he and the barista
remain in privity of contract even after the sale. Thus, the promise to repair can be enforced
against the landlord. When leased property is sold, the purchaser is liable to the tenants for
performance of all covenants made by the original landlord in the lease, provided that those
covenants run with the land. A covenant runs with the land if the parties to the lease so intend
and the covenant touches and concerns the land. Generally, promises to do a physical act, such
as maintain or repair the property, are considered to run with the land. Thus, the buyer is liable
because he is in privity of estate with the barista and the covenant to repair runs with the land.
Consequently, both the landlord and the buyer are potentially liable to the barista for the repairs.
(B) is incorrect because the barista may recover the full amount from either the landlord or the
buyer. (C) is incorrect because the burdens of covenants that touch and concern the land run with
the landlord’s estate and become the burdens of the new landlord, regardless of whether the new
landlord assumes the lease obligations. (D) is incorrect because, although it is true that a covenant
to repair touches and concerns the land and runs with it upon assignment, the landlord as well as
the buyer can be held liable.
REAL PROPERTY - ESTATES, LEASES, CO-TENANCIES ANSWERS 7.

Answer to Question 18

(B) The tenant will be successful in terminating the lease because the landlord breached the implied
warranty of habitability by failing to fix the bathroom pipes. The general rule at common law
was that the landlord was not liable to the tenant for damages caused by the landlord’s failure to
maintain the premises during the period of the leasehold. Today, however, a majority of jurisdic-
tions, usually by statute, provide for an implied warranty of habitability for residential tenan-
cies. In the absence of a local housing code, the standard applied is whether the conditions are
reasonably suitable for human residence. If the landlord breaches the implied warranty, the tenant
may: (i) terminate the lease, (ii) make repairs and offset their cost against future rent, (iii) abate
rent, or (iv) seek damages. Here, a court is likely to consider the lack of a functioning bathroom
as making the premises unsuitable for human residence, allowing the tenant to terminate the
lease. (A) is therefore incorrect. (C) would be a stronger answer if the tenant had vacated the
premises within a reasonable time. The doctrine of constructive eviction provides that where
a landlord does an act or fails to perform some service that he has a legal duty to provide, and
thereby makes the property uninhabitable, the tenant may terminate the lease and seek damages.
However, a tenant cannot claim a constructive eviction unless: (i) the injurious acts were caused
by the landlord, (ii) the premises are uninhabitable, and (iii) the tenant vacates the premises
within a reasonable time. Here, the landlord’s failing to keep the apartment building at a comfort-
able temperature 24 hours per day meets conditions (i) and perhaps (ii), but the tenant remains in
possession. Therefore, the tenant cannot claim constructive eviction and (C) is incorrect. (D) is
incorrect for the same reason.
REAL PROPERTY -
COVENANTS,
EASEMENTS, FIXTURES
QUESTIONS
REAL PROPERTY - COVENANTS, EASEMENTS, FIXTURES QUESTIONS 1.

REAL PROPERTY - COVENANTS, EASEMENTS, FIXTURES QUESTIONS

Question 1 According to common law principles, which


of the following statements is correct?
A developer created an exclusive residential
subdivision. In his deed to each lot, the following (A) If the developer, still owning unsold lots,
language appeared: sues the accountant to have him cease
operating the tax preparation business, the
Grantee agrees for himself and assigns accountant would win because there is no
to use this property solely as a single- privity between the developer and the ac-
family residence, to pay monthly fees countant.
as levied by the homeowners’ associa-
tion for upkeep and security guard (B) If the homeowners’ association sues the
services, and that the backyard of this barber to collect the monthly fees for
property shall remain unfenced so upkeep and security guard services, the
that bicycle paths and walkways may homeowners’ association would win
run through each backyard, as per the because the covenant regarding fees is
subdivision master plan [adequately enforceable in equity against the barber.
described], for use by all residents of
the subdivision. (C) If the barber sues the chiropractor to obtain
removal of her backyard fence, the barber
The developer sold lots to an actuary, a baker, would win because the covenant regarding
and a coroner. All deeds were recorded. The fencing is enforceable in equity against the
subdivision was developed without backyard chiropractor.
fences, with bicycle paths and walkways in
place in accordance with the general plan. The (D) If the chiropractor sues the accountant to
actuary in turn sold to an accountant by a deed have him cease operating the tax prepara-
that omitted any mention of the covenants above, tion business, the chiropractor would win
and the accountant had no actual knowledge because the covenant regarding single-
thereof. Shortly thereafter, the accountant started family use is enforceable at law against the
operating a tax preparation business out of his accountant.
home. The baker in turn sold to a barber, who
knew of, but refused to pay, the monthly fees
levied by the homeowners’ association. The
coroner leased her property for 10 years to a
chiropractor, who erected a fence around the
backyard, unaware of the covenant against such
fencing.
2. REAL PROPERTY - COVENANTS, EASEMENTS, FIXTURES QUESTIONS

Question 2 (C) The purchaser will prevail, because he can


show that a common development scheme
The owner of a large tract of land in fee had been established for the entire subdivi-
simple subdivided 100 acres into 250 lots. sion.
She obtained all the necessary governmental
approvals and recorded a plat of the planned (D) The purchaser will prevail, unless the
development, and in a 12-year span sold 175 evidence shows that the resort was not
of the lots. The purchaser of one of the lots aware of this provision at the time of its
received a deed containing a provision that was agreement with the owner.
in all the deeds to these 175 lots. It provided that
the property conveyed shall be used for a single- Question 3
family dwelling only and that no structure, other
than a single-family dwelling, shall be erected or A developer owned a large parcel of land,
maintained. It further provided that occupancy which she subdivided into 25 separate lots. The
in any dwelling built on the property shall be by developer sold lots 1 - 24, retaining lot 25 for
a single family for residential purposes only and herself. The deeds for each of the 24 lots sold
that the agreement is specifically made binding by the developer restricted the use of the land to
on the grantee and grantee’s heirs, their assigns residential purposes only. The purchasers of lots
and successors. 1 - 24 each built residences on their property.
The developer lived on lot 25 in the subdivi-
Two years ago, the owner contracted with a sion until her death, when it was sold without
water sport resort to sell an additional 100 acres any restrictions to a buyer. Several years ago,
that she owned contiguous to these lots. As part the buyer sold lot 25 to a merchant, without any
of this agreement, the owner also conveyed to restrictions. Last month, the merchant decided to
the resort the 75 lots she had not previously sold. build a convenience store on lot 25. An adjoining
Nothing in the deeds for these 75 lots restricted owner in the subdivision informed the merchant
their use to single-family residences, and in fact, of the restriction and told him that he would be
the resort was planning to use all the property unable to build the convenience store.
purchased for a resort and for multi-family
dwellings. If the restriction to use the land for residential
purposes only is held to apply to lot 25, what
If the purchaser of the first lot brought suit will be the most likely reason?
against the owner to establish that all the
original 250 lots, including the 75 she had agreed (A) The developer could not equitably add the
to sell to the resort, had to be used only for restriction to the other 24 lots but not her
single-family dwellings in a proper proceeding, own.
what would be the most likely result?
(B) The restriction runs with the land at law.
(A) The owner will prevail, because the provi-
sion in the deed only binds the grantee. (C) The restriction created a reciprocal negative
servitude.
(B) The owner will prevail, because the
remaining 75 deeds did not contain this (D) The restriction created a general plan.
provision.
REAL PROPERTY - COVENANTS, EASEMENTS, FIXTURES QUESTIONS 3.

Question 4 Question 5

The owner of 50 acres of unimproved A developer divided his tract of land into four
property prepared and recorded a subdivision standard lots, which he conveyed to a doctor, a
plan calling for 80 home sites on one-half acre pilot, a carpenter, and an athlete, respectively.
each. Purchasers were required to build their Each deed granted by the developer contained
homes using one of five different approved a covenant requiring that the property be used
plans. Each deed, which referred to the recorded only for single-family housing. All deeds were
plan, stated that “no residence shall be erected duly recorded in the office of the county recorder
on any lot that has not been approved by the of deeds. The doctor and the pilot proceeded
homeowners’ association.” An investor who to build single-family houses on their lots. The
bought one of the lots but did not build on carpenter and the athlete did not develop their
it resold it to a purchaser a few years later. properties immediately.
The deed from the investor to the purchaser
did not contain any reference to the recorded The doctor later sold her property to a nurse
plan nor the obligation regarding approval by and included the covenant limiting use to single-
the homeowners’ association. The purchaser family dwellings in the deed. The pilot sold
began to build a very modernistic house on her his property to a flight attendant, but did not
one-half acre. A neighbor who had built a house include the covenant in the deed. The carpenter
using one of the approved plans brought an sold her property to an electrician, and the deed
action to enjoin the construction. contained the restriction. The athlete sold his
property to a physical therapist, and the deed did
Who is likely to prevail in this action? not contain the restriction. All but the electri-
cian’s deed were duly recorded.
(A) The purchaser, because her deed contained
no restrictive covenants. Subsequently, the nurse died and her property
passed by will to her daughter. The flight
(B) The purchaser, because any restrictive attendant gave her land to her son “for life.”
covenant can only be enforced by the The electrician sold his property for value to
opposite party to the covenant or that a plumber. All three transfers of title were
person’s successor in title. recorded, but none of the deeds mentioned the
covenant.
(C) The neighbor, because the recorded subdi-
vision plan, taken with the fact that all lots Which of the current owners below is not
were similarly restricted and the purchaser bound by the covenant?
had notice of this, gave him the right to
enforce the covenant on the purchaser’s (A) The nurse’s daughter, who received the
property. property by will.

(D) The neighbor, because his deed contained (B) The flight attendant’s son, who received the
the restrictive covenant. property for life.

(C) The plumber, who purchased the property


from the electrician.

(D) The physical therapist, who purchased the


property from the athlete.
4. REAL PROPERTY - COVENANTS, EASEMENTS, FIXTURES QUESTIONS

Question 6 Question 7

A developer subdivided a 25-acre tract of A landowner owned two adjoining parcels of


land into 100 quarter-acre lots. On each lot she land. The landowner sold the western parcel to
built a two-unit townhouse. The deeds to each a buyer by a deed that contained the following
of the purchasers contained a covenant that “the clause: “Grantee promises for himself, his heirs,
grantee, his heirs and assigns” would use the successors, and assigns to not erect a structure
property only for single-family use. All deeds over two stories on the land.” The buyer recorded
were promptly and properly recorded. Subse- the deed and built a two-story house on the
quently, the zoning laws were amended to allow property and lived there for 30 years, after which
multifamily use within the subdivision. Six he sold the land to a movie star by a deed that
months later, a social worker offered to purchase did not contain the structure height restriction.
an original owner’s unit that was for sale. The The movie star decided to tear down the existing
social worker informed the owner that she house on the land and erect a three-story house.
planned to operate a halfway house out of the Her neighbor, who purchased the eastern parcel
unit, an activity in conformity with the appli- from the landowner 10 years earlier, discovers
cable zoning regulations. Therefore, the owner that the movie star’s house will be larger than his
did not include the single-family restriction in and files suit to enforce the covenant.
the deed to the social worker.
Who will prevail?
If a neighbor, who purchased his lot from the
developer, seeks to enjoin the operation of the (A) The neighbor, because the restrictive cov-
halfway house, will he succeed? enant runs with the land.

(A) No, because the deed from the owner to the (B) The neighbor, because privity is not
social worker did not refer to the covenant. required to enforce an equitable servitude.

(B) No, because the social worker relied on the (C) The movie star, because her deed did not
zoning regulations when purchasing the contain the restrictive covenant.
unit.
(D) The movie star, because she had no notice
(C) Yes, because the social worker had notice of the structure height restriction.
of the restrictive covenant.

(D) Yes, but only if the neighbor can establish a


common scheme for development.
REAL PROPERTY - COVENANTS, EASEMENTS, FIXTURES QUESTIONS 5.

Question 8 Question 9

A farmer owned a 70-acre farm on the A homeowner owned a parcel of land on


outskirts of a rapidly growing town. She which she built a single-family residence. To
conveyed the farm to a developer by a deed that pay for the construction, she obtained financing
contained no restrictions. The developer divided from a bank in exchange for a mortgage on the
the property into two tracts, a 40-acre parcel and land. The bank promptly and properly recorded
a 30-acre parcel. He then subdivided the 40-acre its mortgage. When the house was completed,
parcel into 80 half-acre lots and put them on the except for the absence of an oven in the kitchen,
market. Under the town’s liberal land-use provi- the homeowner leased the house to a tenant for
sions, there was no zoning code limitation on a three-year term. There was no provision in the
the use of either parcel, nor was the developer lease agreement regarding kitchen appliances.
required to file a plat map with local officials. The homeowner bought an oven from an appli-
However, the deed that each purchaser of a half- ance company and had it installed in the space
acre lot received contained a clause restricting provided around the built-in cabinets in the
the use of the lot to “no more than one single- kitchen. To make the purchase, the homeowner
family dwelling and one garage.” In addition, signed a security agreement with the appli-
the developer gave every prospective purchaser a ance company granting it a security interest in
brochure depicting the 80-lot subdivision on its the oven in exchange for financing. The appli-
completion, with a drawing of a house on each ance company did not file or record its security
lot. The lots sold briskly and soon a number of interest in the oven.
homes were constructed thereon.
By the end of the lease term, the homeowner
Two years later, an original owner of one of was in serious default on her mortgage payments
the lots sold his home to a buyer. The buyer’s to the bank and to the appliance company. In
deed did not contain the restrictive covenant. preparing foreclosure proceedings against the
Soon after, when the last subdivision lot was homeowner, the bank learned that the tenant was
sold, the developer sold the adjacent 30-acre planning to remove the oven and take it with
parcel to a car dealer by a deed that contained him when he moved out within the next few
no restrictions as to the use of the parcel. Two weeks. The bank filed an action against the tenant
weeks after the buyer moved into her home, the claiming ownership of the oven, and joined the
car dealer began building an auto mall on his homeowner and the appliance company as parties.
parcel. The buyer was angry that the busy mall
was to be right next door to her home. She filed Which party has a superior claim to the oven?
suit against the car dealer to enjoin his construc-
tion of the auto mall. (A) The bank, because its mortgage interest
attaches to all fixtures on the real estate and
Which of the following is the best argument in it has priority over the appliance company.
the car dealer’s defense?
(A) The developer did not expressly promise in (B) The tenant, because removal of the oven
writing that he would restrict the remaining will not cause substantial damage to the
lots when he conveyed the lot to the owner. real estate.

(B) The buyer and the car dealer are not in each (C) The homeowner, because the oven
other’s chain of title. was annexed to the real estate after the
mortgage was given.
(C) The buyer’s deed contained no restrictive
covenant. (D) The appliance company, because it has
a valid security interest in the oven even
(D) The 30-acre parcel was not part of a though it was not recorded.
common scheme of development.
6. REAL PROPERTY - COVENANTS, EASEMENTS, FIXTURES QUESTIONS

Question 10 Question 11

A landlord leased a vacant commercial A farmer asked his adjoining neighbor if he


building to a tenant for a 10-year term. On could build an irrigation ditch from the neigh-
taking possession, the tenant installed a bar, bor’s natural fresh-water spring to his property
booths, special lighting, and a raised dance to provide water for his cattle. Because the
floor. The bar and booths were simply placed on spring supplied more than enough water to meet
the floor and were not secured to it. The lights the neighbor’s needs, she agreed, provided that
were installed by a qualified electrician and the farmer construct the ditch in such a manner
were directly wired into the building’s electrical that it would need the least maintenance possible
system. The special dance floor was bolted to so that he did not need to continually enter her
the building’s cement floor by installation crews land. The farmer constructed a concrete irriga-
from the company that made the dance floor. tion ditch from the spring to the land at a cost of
After installation of the above, the tenant then $25,000. The only maintenance required on the
operated her business for almost 10 years. At ditch was a semiannual cleaning. Three years
that time, she decided that business was so good later, the neighbor informed the farmer that her
that she would move to a larger space down the water needs had increased, and he could no
street. She told the landlord that she would not longer take water from the spring. In addition,
be renewing her lease and that she would be the neighbor did not allow the farmer onto her
removing the lights, booths, bar, and dance floor. land to do the semiannual cleaning, resulting in
The landlord told her that none of the installa- the blocking of the ditch. The farmer wishes to
tions could be removed because they are now keep water flowing through the irrigation ditch
part of the building. to his land.

If a court were called upon to resolve this Which of the following would be the farmer’s
dispute, how would it likely rule? strongest argument?

(A) The tenant may remove the dance floor, (A) The farmer owns a valid easement appurte-
bar, and lights, but not the booths. nant to the neighbor’s property.

(B) The tenant may remove the bar and the (B) Because the neighbor has allowed the
booths, but not the dance floor and the farmer to construct the irrigation ditch,
lights. the neighbor would be estopped from
preventing the farmer from coming onto her
(C) The tenant may remove the bar, booths, land.
dance floor, and lights.
(C) The farmer, although a licensee, has
(D) The tenant may not remove the bar, booths, expended such a substantial sum of money
dance floor, or lights. in constructing the irrigation ditch that the
neighbor may not terminate the farmer’s
license now.

(D) The farmer, although a licensee, may


continue to enter the neighbor’s property to
clean and maintain the ditch until he is able
to acquire another source of water.
REAL PROPERTY - COVENANTS, EASEMENTS, FIXTURES QUESTIONS 7.

Question 12 Question 13

An owner of a large parcel of land divided the A landowner granted an easement over his
parcel into two halves. The front parcel abutted front yard to a railway several decades ago for
a public highway. The shortest route from the the purpose of operating a cable car system,
back parcel to the highway was over a small which the railway then built. The railway used
private road that crossed the front parcel. There the system for many years, but ceased its opera-
was another route from the back parcel to the tions when the system became rundown and
highway that did not involve crossing the front unprofitable. Over 10 years ago, the railway
parcel, but it wound through the woods for over removed the cables and tracks because they
four miles. The owner sold the back parcel to had become an eyesore. Three years ago, the
a purchaser who planned to open an inn on the landowner planted the area with grass.
property, and included an express easement in
the deed permitting her to access the highway The applicable statute provides for a prescrip-
via the private road across the front parcel. The tive period of 10 years. Last year, the railway
purchaser never properly recorded her deed to received a state subsidy to reopen its system and
the back parcel. Although the purchaser built the is now planning to lay new tracks and cables
inn, she never opened it to the public because of over its right-of-way. The landowner has brought
financial circumstances and only rarely used the an action to quiet title and to restrain the railway
road across the front parcel. from building on the landowner’s land.
Fifteen years after that transaction, the If the landowner wins, what will be the likely
purchaser sold the land to a buyer who planned reason?
to open the inn to the public. The buyer promptly
recorded her deed to the property from the (A) The railway’s removal of its cable and
purchaser, but the buyer’s deed made no mention tracks combined with many years of
of the right to cross the front parcel via the nonuse indicates an intent to abandon the
private road. Shortly after the buyer took posses- easement.
sion of the back parcel, she learned of the provi-
sion in the original deed. The buyer told the (B) The railway’s plan to build the new system
owner, who still owned the front parcel, that she will unreasonably interfere with the
planned to use the road across his property when landowner’s use of his fee simple.
she opened the inn, but the owner refused to
grant permission to do so. (C) The railway’s easement terminated when
it was no longer practical to utilize the
Does the buyer have a right to cross the front easement for the purpose for which it was
parcel? granted.
(A) No, because the easement is not mentioned
in the buyer’s deed, and the purchaser’s (D) The railway’s nonuse of the easement for
deed containing the easement was not a period beyond the prescription period
recorded. terminated the easement.

(B) No, because the buyer’s opening of the inn


would increase the use of the easement.
(C) Yes, because the purchaser exercised her
right to use the easement when she owned
the back parcel.
(D) Yes, because the easement was not extin-
guished.
8. REAL PROPERTY - COVENANTS, EASEMENTS, FIXTURES QUESTIONS

Question 14 Question 15

An investor owned a large tract of land. To the A vintner owned a vineyard, which adjoined
east of the land was a city-owned parking lot. a public road. His neighbor owned the adjoining
The investor used a driveway across the eastern undeveloped lot, which was from a different
part of the land to provide the western part with original tract. The easiest practical access to
access to the parking lot. Subsequently, the the undeveloped lot was from the public road
investor sold the western parcel to a bakery and adjoining the vineyard. This required, however,
the eastern parcel to a coffee shop. By a written that the neighbor cross the vineyard to reach
instrument, the coffee shop granted the bakery the road. The vintner granted an easement to
and its customers the right to use the driveway the neighbor, allowing her to “pass through” the
to access the parking lot. The only other way to vineyard on her journeys between the undevel-
enter the parking lot is over land to the north of oped lot and the road. The neighbor also wanted
the western parcel. The bakery has been unsuc- to develop her lot but no power lines had been
cessful in obtaining access to the parking lot installed to provide it with utility service. The
over this land. neighbor consulted the vintner, but he refused
to alter the wording of the easement. Although
The bakery’s right to use the driveway is: a power company had a statutory power of
eminent domain, it refused to run power lines
(A) An easement in gross. to the neighbor’s property without the vintner’s
permission.
(B) An easement appurtenant.
Can the neighbor require the vintner to allow
(C) An easement by necessity. the power lines to be installed through the
vineyard?
(D) An easement by implied reservation.
(A) Yes, by exercising the power company’s
right of eminent domain.

(B) Yes, because the need for utility service is


a sufficient basis for an easement by neces-
sity.

(C) No, because the neighbor’s easement is for


a different purpose.

(D) No, because the property is undeveloped


and there is no established necessity for
power service.
REAL PROPERTY - COVENANTS, EASEMENTS, FIXTURES QUESTIONS 9.

Question 16 Question 17

A retiree purchased a rustic cabin on a small Twenty-five years ago, a property owner
plot of land near the center of a landowner’s large placed a large sewer line (to service a single-
parcel of land. The deed to the land, which the family house he built on the property) across
landowner delivered to the retiree for fair consid- a neighbor’s property without the neighbor’s
eration, did not specifically grant an easement permission. Four years ago, the owner tore down
over the landowner’s property to reach the public the house in preparation for the construction of a
highway bordering her land. There were two larger house on the land, and made an agreement
means of access to the cabin from the public with municipal authorities to take an easement
roads: a driveway from the county road on the across the neighbor’s property and install a
south, and a private road from the highway on the new sewer line to service the house. After a
east. The landowner told the retiree that he could long delay, the municipal authorities failed to
use the private road from the highway. Twice perform their agreement, and the owner finally
during his first two years at the cabin, the retiree constructed the house last year using the existing
took the driveway from the county road instead; sewer line. The state has a 20-year statute for
at all other times he used the private road. acquiring property interests by adverse use.
At the end of his second year at the cabin, If the neighbor attempts to enjoin the use of
the retiree began reading tarot cards to supple- the sewer line to service the new house, will she
ment his retirement income. He had a steady prevail?
stream of clients coming to his home at all
hours of the day and night. Most of the clients (A) Yes, because the agreement with the mu-
came in on the driveway from the county road, nicipal authorities estopped the owner from
which ran close to the landowner’s home. The using the existing line to service the new
landowner objected, and told the retiree that house.
neither he nor his clients had any right to use
that driveway and that they must use the private (B) Yes, because the owner’s tearing down of
road from the highway. The retiree refused, and the house and nonuse of the sewer line for
he and his clients continued to use the driveway several years constituted an abandonment
from the county road for three years. Finally, of the easement.
the landowner began blocking off the driveway
from the county road. The retiree brought suit to (C) No, because the original sewer line was
enjoin this practice. The prescriptive period in a license which ripened into an implied
this jurisdiction is five years. easement.
Who will most likely prevail? (D) No, because the use was within the scope of
the prescriptive easement acquired.
(A) The landowner, because the tarot business
has changed the nature of the use of the
easement by necessity.
(B) The landowner, because she may select the
location of the easement.
(C) The retiree, because he has a valid
easement by necessity in the driveway from
the county road.
(D) The retiree, because he has acquired an
easement by prescription in the driveway
from the county road.
10. REAL PROPERTY - COVENANTS, EASEMENTS, FIXTURES QUESTIONS

Question 18

An owner of a 60-acre parcel of land bordered


by a lake on its east side sold the western 50
acres to a fisherman. The recorded deed recited
that the fisherman, his heirs and assigns had the
right to use the single lane gravel road to the
lake that ran along the southern border of the
owner’s property. Subsequently, the fisherman
died and his son inherited the 50-acre parcel.
The son wishes to build a resort on the property
but would need to pave and widen the lake road
so it could bear the increased traffic. The owner
files an action for declaratory judgment seeking
to enjoin improvement of the road.

If the owner prevails, what is the reason?

(A) The proposed improvement exceeds the


scope of an easement by necessity.

(B) The proposed improvement constitutes a


burden that exceeds the scope of an express
easement.

(C) The servient owner has the obligation for


maintenance of an easement for right-of-
way, and hence can control the nature of its
improvement.

(D) The proposed resort so changes the nature


of the use of the dominant tenement that
any easement has been abandoned.
REAL PROPERTY -
COVENANTS,
EASEMENTS, FIXTURES
ANSWERS
REAL PROPERTY - COVENANTS, EASEMENTS, FIXTURES ANSWERS 1.

REAL PROPERTY - COVENANTS, EASEMENTS, FIXTURES ANSWERS

Answer to Question 1

(C) If the barber sues the chiropractor to remove her backyard fence, the barber would win because
the covenant regarding fencing is enforceable against the chiropractor as an equitable servitude.
An equitable servitude is a covenant that, regardless of whether it runs with the land at law, equity
will enforce against the assignees of the burdened land who have notice of the covenant. The
benefit of an equitable servitude runs to successors if: (i) the original parties so intended, and (ii)
the servitude touches and concerns the land. The burden runs if (i) and (ii) are met and (iii) the
subsequent purchaser has actual or constructive notice of the covenant. Privity of estate is not
needed to enforce an equitable servitude because it is enforced not as an in personam right against
the owner of the servient tenement, but as an equitable property interest in the land itself. Here,
the original parties intended for the fencing covenant to be enforceable by and against assignees,
as shown by the specific language of the covenant (“Grantee agrees for himself and assigns”) and
its purpose to provide bicycle paths and walkways running through each backyard for the use of
all subdivision residents. The benefit of the covenant touches and concerns the barber’s property
because it increases his enjoyment thereof by providing him with such paths and walkways.
Therefore, the barber is entitled to enforce the covenant. The burden of the covenant touches and
concerns the land occupied by the chiropractor because it restricts the landholder in her use of
the parcel (i.e., her rights in connection with the enjoyment of the land are diminished by being
unable to fence in the backyard). The chiropractor will be deemed to have inquiry notice of the
restriction because the subdivision is sufficiently developed in accordance with a general plan
for the subdivision. Moreover, any neighbor in a subdivision can enforce a covenant contained in
a subdivision deed if a general plan existed at the time he purchased his lot. As has been noted,
the maintenance of access to all backyards for use as bike paths and walkways was part of such a
general plan. Finally, the fact that the chiropractor did not succeed to the coroner’s entire estate,
but rather a leasehold interest, is irrelevant because privity is not required to enforce an equitable
servitude. Therefore, all of the requirements are in place for the existence of an equitable servi-
tude, which can be enforced by the barber against the chiropractor. (A) is incorrect because
there is privity between the developer and the accountant. There was horizontal privity between
the original covenanting parties because, at the time the actuary entered into the covenant with
the developer, they shared an interest in the land independent of the covenant (i.e., they were in
a grantor-grantee relationship). The accountant holds the entire interest held by the actuary at
the time the actuary made the covenant; thus, there is vertical privity. (B) is incorrect because
the remedy sought is the payment of money. Breach of a real covenant, which runs with the
land at law, is remedied by an award of money damages, whereas breach of an equitable servi-
tude is remedied by equitable relief, such as an injunction or specific performance. Because the
homeowners’ association seeks to obtain from the barber the payment of money, it is inaccurate to
refer to this as a situation involving an equitable servitude. (D) is incorrect because, as explained
above, if equitable relief is sought, the covenant must be enforced as an equitable servitude rather
than a real covenant.

Answer to Question 2

(C) The buyer will prevail. Because the owner had a scheme for an exclusively residential subdivi-
sion that included these lots when the sales began (as evidenced by the recorded plat), a court
will imply a reciprocal negative servitude limiting the remaining lots to the same use. (A) is
incorrect because the owner also covenanted to restrict the lots to single-family use. (B) is incor-
rect, because if there is a reciprocal negative covenant, it would apply whether or not the deeds
2. REAL PROPERTY - COVENANTS, EASEMENTS, FIXTURES ANSWERS

specifically contained the provision. The resort’s actual knowledge is not the issue; the resort must
have had actual notice, record notice, or, as is likely here, inquiry notice (e.g., the neighborhood
appears to conform to common restriction). Thus, (D) is not the best answer.

Answer to Question 3

(C) If the restriction applies to lot 25, it will be because the restriction is a reciprocal negative servi-
tude. When a developer subdivides land into several parcels and some of the deeds contain
negative covenants, but some do not, negative covenants or equitable servitudes, binding all of the
parcels in the subdivision, may be implied under the doctrine of reciprocal negative servitudes.
To enforce a reciprocal negative servitude, the court will need to find: (i) a common scheme for
development, and (ii) notice of the covenants. A common scheme may be evidenced by a general
pattern of prior restrictions. Notice may be actual (i.e., direct knowledge of covenants in prior
deeds), inquiry (i.e., neighborhood appears to conform to common restrictions), or record (i.e.,
prior deeds containing covenants are in grantee’s chain of title). Here, although there were no
restrictions on lot 25, all of the other parcels were subject to a negative covenant limiting their use
to residential purposes only. Thus, a court would likely determine that all parcels in the subdivi-
sion were developed according to a common scheme. It does not appear that the merchant had
actual notice of the restriction or that it was in his chain of title; however, the merchant will be
deemed to have inquiry notice of the restriction because of the uniform residential character of the
other lots in the subdivision. Thus, the restriction may be enforced as a reciprocal negative servi-
tude. (A) is incorrect because the developer could have added the restriction to the other 24 lots
but not her own, so long as she did so in a way that did not imply that her own lot was also bound
by the restriction. (B) is incorrect because the remedy sought here is an injunction (i.e., prohib-
iting the merchant from building a convenience store), and a breach of a real covenant is remedied
by an award of damages, not an injunction. (D) is incorrect because it is incomplete. In order to
enjoin the merchant from building the convenience store there must not only be a general plan, but
he must have had notice of the restriction.

Answer to Question 4

(C) The neighbor will likely prevail because the purchaser had inquiry and constructive notice of the
restriction. When a subdivision is created with similar covenants in all deeds, there is a mutual
right of enforcement (each lot owner can enforce against every other lot owner) if two things are
satisfied: (i) a common scheme for development existed at the time that sales of parcels in the
subdivision began; and (ii) there was notice of the existence of the covenant to the party sued.
Here, there was a common scheme evidenced by the recorded plan, and the fact that the covenant
was in the purchaser’s chain of title gave her constructive notice of the restriction. Therefore, not
only does the covenant apply to the purchaser’s land, but the neighbor (or any other lot owner) can
enforce it as a reciprocal negative servitude. (A) is incorrect. While it is true that the purchaser’s
deed had no restrictions, those restrictions are binding if they are in her chain of title so as to give
her notice of them. The restriction was in the deed from the owner to the investor, so the fact that
it was omitted in the deed from the investor to the purchaser is of no significance. (B) is incor-
rect. While a covenant is normally only enforceable by the party receiving the promise (here, the
owner), this is a situation of mutual rights of enforcement within a geographically defined area,
which gives every lot owner in the area the right of enforcement, even though they did not directly
receive the benefit of the promise. (D) is incorrect. The fact that gives the neighbor the right of
enforcement is not just that his deed contains the covenant, but that the same covenant was in all
of the deeds from the owner, including the one to the purchaser’s predecessor in title.
REAL PROPERTY - COVENANTS, EASEMENTS, FIXTURES ANSWERS 3.

Answer to Question 5

(B) The covenant may not be enforced at law against the son. If all requirements are met for the
burden of a covenant to run, the successors in interest to the burdened estates will be bound by
the arrangement entered into by their predecessors as effectively as if they had expressly agreed
to be bound. The requirements are: (i) The covenanting parties must have intended that succes-
sors in interest to the covenantor be bound by the terms of the covenant. The requisite intent
may be inferred from circumstances surrounding creation of the covenant. This requirement is
satisfied because the developer’s deed to each of the grantees contained a covenant requiring
that the property be used only for single-family housing. (ii) By virtue of the recording statutes,
a subsequent purchaser of the promisor’s land must have actual, inquiry, or constructive (record)
notice of the arrangement at the time she purchased the land; otherwise, she is not bound. Here,
the daughter, the son, and the physical therapist have at least constructive notice of the restric-
tion because it is in their chain of title. Because the electrician’s deed was never recorded, the
plumber had a duty to inquire of the electrician where he obtained the property. Thus, the notice
requirement is met. (iii) The covenant must “touch and concern” the land; i.e., the performance
of the burden must diminish the landowner’s rights, privileges, and powers in connection with
her enjoyment of the land. The current owners’ rights as landowners are diminished because they
cannot use their land to construct multifamily dwellings. (iv) Finally, there must be horizontal
and vertical privity. Horizontal privity requires that, at the time the promisor entered into the
covenant with the promisee, the two shared some interest in the land independent of the covenant.
The developer and each of the four original owners, as grantor and grantees, shared an interest in
the land independent of the covenant. Vertical privity exists when the successor in interest to the
covenanting party holds the entire interest that was held by the covenantor at the time she made
the covenant. Here, the daughter, the plumber, and the physical therapist took the entire interest
(fee simple absolute) from their predecessors. However, the son possesses only a life estate in the
property, which is less than the fee simple absolute held by the flight attendant. Thus, vertical
privity is lacking and the son cannot be bound by the covenant. (A) is incorrect because the
daughter is bound by the covenant as discussed above. The fact that she took the property by will
is irrelevant. In fact, taking in this manner increases the chance that the daughter will be bound
because it means she is not a bona fide purchaser for value and thus not protected by the notice
requirement of the recording act. (C) is incorrect because the plumber also is bound because he
is charged with inquiry notice of the electrician’s unrecorded deed, which contained the single-
family restriction. (D) is incorrect because even though the physical therapist’s deed did not
contain the restriction, she will be charged with constructive notice of it. The restriction appears
in the developer’s deed to the athlete and thus is in the physical therapist’s chain of title. Hence,
she also is bound by the covenant.

Answer to Question 6

(C) The neighbor will succeed in enjoining the operation of the halfway house because the social
worker had notice of the restrictive covenant. A covenant runs with the land to a subsequent
purchaser with notice of the covenant if it touches and concerns the land and is intended to run.
Notice may be actual or constructive. Here, the social worker was on record notice of the covenant
because the original owner’s deed was recorded. Restricting land to single-family use touches
and concerns the land, and it is evident that the developer and the original owners, including the
neighbor, intended it to run with the land by use of the language “grantee, his heirs and assigns.”
The social worker thus will be bound even though her deed did not refer to the covenant. Thus,
(A) is incorrect. (B) is incorrect because compliance with zoning regulations does not excuse
noncompliance with an enforceable covenant; both must be complied with. (D) is incorrect
4. REAL PROPERTY - COVENANTS, EASEMENTS, FIXTURES ANSWERS

because the neighbor can prevail without needing to show a servitude implied from a common
scheme, which comes into play when a developer subdivides land into several parcels and some
of the deeds contain negative covenants and some do not. Here, the covenant relating to single-
family use was in all of the original deeds and, as discussed above, it runs with the land. A
covenant that runs with the land may be enforced as an equitable servitude if the assignees of the
burdened land have notice of the covenant; the usual remedy is an injunction. Here, the social
worker had record notice of the covenant and it runs with the land, so the neighbor can enforce
the covenant as an equitable servitude without resort to implying a reciprocal negative servitude.

Answer to Question 7

(A) The neighbor will prevail because the restrictive covenant runs with the neighbor’s land, which is
benefited, and with the movie star’s land, which is burdened. A covenant at law will run with the
land and be enforceable against subsequent grantees if: (i) the contracting parties intended it to
run; (ii) there is privity of estate between the original promisor and promisee (horizontal privity),
as well as between the promisor and his successor (vertical privity); (iii) the covenant touches
and concerns the property; and (iv) the burdened party has notice of the covenant. Here, the use
of the words “heirs, successors, and assigns” in the covenant shows the intent for the covenant
to run. The original parties were in horizontal privity because at the time the buyer entered into
the covenant, he and the landowner shared an interest in the land independent of the covenant—
as grantor and grantee. Because the neighbor and the movie star hold the entire interest in the
parcels held by the landowner and the buyer, respectively, there is vertical privity. The covenant
touches and concerns the land because it diminishes the movie star’s rights in connection with
her enjoyment of the western parcel. Finally, the movie star has notice of the covenant because it
was recorded in her chain of title (in the landowner-buyer deed). Thus, the covenant runs with the
land. (B) is incorrect because although privity is not required to enforce an equitable servitude,
the covenant here runs with the land at law. Moreover, privity does exist under these facts, as
explained above. (C) is incorrect. The fact that the structure height restriction was not contained
in the movie star’s deed does not shield her from enforcement of the covenant because it was
contained in a previous deed relating to the property (record notice). (D) is incorrect because the
movie star had at least record notice of the restriction.

Answer to Question 8

(D) The car dealer’s best argument would be that the buyer cannot enforce the restriction against him
because the 30-acre parcel was not part of the common scheme of development. Because neither
the car dealer’s deed to the 30-acre parcel nor any deed in his chain of title contains the restrictive
covenant, to succeed the buyer must show the existence of an implied reciprocal negative servi-
tude that can be enforced on the 30-acre parcel. When a developer subdivides land into several
parcels and some of the deeds contain negative covenants but some do not, equitable servitudes
(negative covenants enforceable by injunction) binding on all of the parcels may be implied if:
(i) there is a common scheme of development, and (ii) the grantee has notice of the restric-
tive covenant. In this case, both requirements are lacking. A common scheme of development
is proven by evidence that the developer intended that all parcels in a subdivision be developed
within the terms of the negative covenant. The key in this case is that the 30-acre parcel was not
part of the subdivision intended to be so restricted. The same evidence that proves the common
plan for the subdivision shows that the 30-acre parcel was not intended to be included; i.e., the
brochure shows only the 80-lot subdivision with single-family homes and nothing at all about
the 30-acre parcel. In addition, the mere fact that the developer bothered to divide the property
into two parcels before he subdivided the 80 lots shows that he intended to develop the parcels
REAL PROPERTY - COVENANTS, EASEMENTS, FIXTURES ANSWERS 5.

separately. Last, the 30-acre parcel’s disproportionate size—30 acres compared with half-acre
lots—further shows that, at the time the owner bought his lot (and the buyer’s rights are derived
from the owner), the developer did not intend to include the 30-acre parcel in the subdivision.
Therefore, the common scheme requirement is not met. Also, it is unlikely that the car dealer
had the requisite notice to be bound by the covenant. He did not have record notice (because the
restriction was not in his chain of title); nothing in the facts shows that he had actual notice; and
because his parcel does not appear to be part of the neighborhood (being 30 acres and on the
periphery), he did not have inquiry notice. Thus, the car dealer is not bound by the single-family
home restriction in the subdivision deeds. (A) is wrong for the above reasons, but would be wrong
even if the 30-acre parcel were within the common scheme. A prior purchaser of a burdened
lot generally cannot enforce the promise against a subsequent purchaser unless the developer
promised the prior purchaser that he would restrict his remaining lots. This promise need not be
in writing, however; it may be implied from many things, including: a map or plat shown to the
purchaser, oral representations, or sales literature. In this case, the brochure, as well as the overall
appearance of the development, would suffice to infer a promise on the part of the developer to
restrict his remaining subdivision lots, but not the 30-acre parcel. (B) is wrong because, had the
30-acre parcel been part of the common scheme, the fact that the buyer and the car dealer are
not in each other’s chain of title would not matter. That is not a requirement for enforcing an
equitable servitude; in fact, it is almost never the case that the party seeking enforcement is in the
other party’s chain of title. If that were true, only the developer could enforce the restraints. (C)
is wrong because the fact that the buyer’s deed did not contain the restriction is irrelevant. The
important thing is that the buyer’s property is bound by the restriction. The restriction is in her
record chain of title; thus, the buyer took subject to the restriction and is bound by it.

Answer to Question 9

(A) The bank will win because it recorded its mortgage on the property. In a fixture case involving
common ownership, the majority rule is that the annexor’s-intention test applies regardless of
whether the owner makes the annexation before or after mortgaging the land. The mortgage
attaches to all fixtures on the real estate in the absence of an agreement to the contrary. As
between a mortgagee and the holder of a security interest in chattel affixed to the land, whichever
interest is first recorded in the local real estate records wins. While a purchase money security
interest can gain priority if it is recorded within 20 days after affixation, the appliance company
never filed. (B) is wrong because it states the standard for divided ownership cases, i.e., cases in
which the person who brings the chattel onto the land does not own the land. Here, the tenant has
no interest in the oven; the competing interests are those of the homeowner and the creditors. (C)
is wrong because, as discussed above, the mortgagee’s interest applies to the homeowner’s fixtures
regardless of the time of affixation. (D) is wrong because, as discussed above, the appliance
company did not record a fixture filing.

Answer to Question 10

(C) All of the items listed may be removed either because they were not affixed to the real property
or because they were used in the tenant’s business and thus may properly be termed trade
fixtures. As long as the tenant can remove trade fixtures with little damage to the real property,
the tenant may always take them when the tenant moves. (A) is incorrect. For a chattel to be
deemed a fixture (and thus remain as a part of the realty) it must be affixed to the real property.
The booths, along with the bar, were not even attached to the realty; thus, they cannot under any
circumstances be considered fixtures, and the tenant may always take them. (B) is incorrect. If
these were not trade fixtures, an argument could be made that the tenant could take the items not
6. REAL PROPERTY - COVENANTS, EASEMENTS, FIXTURES ANSWERS

affixed (bar and booths) but not the ones affixed (lights and dance floor). But even items strongly
attached to the realty may be detached by the tenants who installed them if, as here, they are tools
of the trade. (D) is incorrect. The booths and bar can be removed because they were not affixed
to the building, and even though the lights and dance floor were attached, they can be removed as
trade fixtures.

Answer to Question 11

(C) The farmer’s strongest argument would be that he has an irrevocable license. The original grant
by the neighbor to the farmer was a license, which is a personal privilege to go upon the land
belonging to the licensor. When the farmer expended a substantial sum of money in reliance on
the license, the license became irrevocable under the doctrine of estoppel. (A) is wrong because
the neighbor was estopped from terminating the license, not just from preventing the farmer from
coming onto her land. (B) is wrong because the estoppel is based not on the mere construction of
the ditch but the expenditure of a substantial sum of money in reliance on the license. Therefore,
(C) is a better answer than (B). (D) is wrong because the duration of the irrevocable license is
based on what the parties contemplated would be the duration when the oral license was granted
and the facts do not indicate that they intended it to exist until the farmer acquired another source
of water.

Answer to Question 12

(D) The buyer has an easement to cross the front parcel because the easement was never extinguished.
The original easement granted to the purchaser was an easement appurtenant, the benefit of which
passes with a transfer of the benefited land. An easement is deemed appurtenant when the right of
special use benefits the easement holder in her physical use or enjoyment of another tract of land.
The land subject to the easement is the servient tenement, while the land having the benefit of the
easement is the dominant tenement. The benefit of an easement appurtenant passes with transfers
of the benefited land, regardless of whether the easement is mentioned in the conveyance. All who
possess or subsequently succeed to title to the dominant tenement are entitled to the benefit of the
easement. The easement granted to the purchaser was an easement appurtenant because the right
to use the private road across the front parcel (the servient tenement) benefited the purchaser in
her use and enjoyment of the back parcel (the dominant tenement) by providing her with the most
convenient access to the public highway. Thus, when the purchaser sold the benefited land to the
buyer, the benefit of the easement also passed to the buyer as an incident of possession of the back
parcel. (A) is wrong because, as explained above, this benefit passed to the buyer despite the fact
that the deed to the buyer made no mention of the easement. The failure to record does not affect
the validity of the easement. Recordation is not essential to the validity of a deed, but only serves
to protect the interests of a grantee against subsequent purchasers. Here, the dispute is between
the original grantor and the successor of the original easement holder. (B) is incorrect because
the buyer’s use of the easement would not be a change in its use. This choice goes to the scope of
the easement. The key for determining the scope is the reasonable intent of the original parties,
including the reasonable present and future needs of the dominant tenement. Here, because the
owner knew of the purchaser’s plans to open an inn, he knew that she and her guests would use
the road across the front parcel. The buyer’s use of the easement would be the same—her use and
that of her guests. This is not a change in intended use sufficient to allow the owner to legally
prevent the buyer’s use of the easement. (C) is incorrect because nonuse does not extinguish
an easement. Abandonment, which does terminate an easement, requires a physical act by the
easement holder that manifests an intent to permanently abandon the easement (e.g., erecting a
building that blocks access to an easement of way). Since there is no indication of such an act by
REAL PROPERTY - COVENANTS, EASEMENTS, FIXTURES ANSWERS 7.

the purchaser, the easement would have continued to benefit the back parcel even if the purchaser
had never used it.

Answer to Question 13

(A) If the landowner wins it will be because the railway intended to abandon its easement. An
easement can be abandoned provided two things are satisfied: nonuse of the easement together
with some affirmative act taken on the property which would indicate an intent to abandon the
easement. Mere nonuse would not be enough, no matter how long the easement was not used. It is
the act taken on the property that shows the intent to abandon that is crucial. Here, the railway’s
acts in removing the cables and tracks can show the necessary intent. (B) is incorrect. The
landowner had granted the easement to the railway, and if the easement has not been terminated,
any interference claimed by the landowner is irrelevant. There is no indication that the railway is
planning to use the easement in any way different from the use contemplated in the original grant.
(C) is incorrect because easements are not terminated simply because the purpose is no longer
“practical.” (D) is incorrect. Nonuse of an easement will not terminate the easement unless there
is also some physical act taken on the property itself that would show an intent to abandon the
easement.

Answer to Question 14

(B) The bakery’s right to use the driveway is an easement appurtenant. An easement is deemed
appurtenant when the right of special use benefits the holder of the easement in its physical use
or enjoyment of another tract of land. For an easement appurtenant to exist, there must be two
tracts of land: the dominant tenement has the benefit of the easement, and the servient tenement
is subject to the easement right. Here, the bakery’s easement is appurtenant because its use and
enjoyment of the western parcel is benefited by the right to use the driveway; thus, the easement is
appurtenant. (A) is incorrect because an easement in gross is acquired independent of the owner-
ship and enjoyment of another tract of land; there is no dominant tenement. (C) and (D) are incor-
rect because the bakery’s easement is in writing and is therefore express rather than implied. An
easement by necessity is created by implied grant or reservation when the owner of a tract of land
sells a part of the tract and by this division deprives one lot of access to a public road or utility
line. An easement implied from an existing use (quasi-easement) exists if, prior to the time a tract
is divided, a use exists on the “servient part” that is reasonably necessary for the enjoyment of the
“dominant part” and the parties intended the use to continue after division of the property. Here,
although the western and eastern parcels were once a unified tract of land, there is no need to
resort to an implied easement because the coffee shop expressly granted the bakery an easement
in the driveway.

Answer to Question 15

(C) The neighbor cannot require the vintner to permit power lines to be installed on the vineyard. A
basic change in the nature of use (scope) of an easement is not allowed without the consent of the
owner of the land burdened by the easement. Thus, a power line cannot be added to the neigh-
bor’s private easement of way across the vineyard. (A) is incorrect because the neighbor cannot
exercise the power company’s right of eminent domain. Eminent domain can only be exercised by
government entities or organizations (such as power companies) that provide public services and
are empowered by government entities to use eminent domain. A landowner cannot use eminent
domain on behalf of a power company. (B) is incorrect because easements by necessity arise only
when the owner of a single tract divides it, leaving a portion of the tract with no access to a public
8. REAL PROPERTY - COVENANTS, EASEMENTS, FIXTURES ANSWERS

road or utility line. The facts in this question do not indicate that the vintner ever owned both the
vineyard and the undeveloped lot; therefore, no easement by necessity can arise. (D) is incorrect
because prior necessity is irrelevant. Necessity is determined at the time the parcel is divided. As
noted above, the facts here indicate that the land was not part of a unitary tract; thus, a necessity
analysis is impossible.

Answer to Question 16

(B) The landowner will prevail in a suit because she, as the holder of the servient estate, has the right
to choose the location of an easement by necessity. An easement by necessity arises when the
owner of a tract of land sells a part of the tract and by this division deprives one lot of access to
a public road or utility line. The owner of the servient parcel has the right to locate the easement,
provided the location is reasonably convenient. The landowner has chosen the private road from
the highway; thus, the retiree has no right to use the driveway from the county road. Both (A)
and (C) are incorrect because the retiree has no easement by necessity in the driveway. As stated
above, the owner of the servient parcel (the landowner) has located the easement in the private
road; thus, no easement in the driveway exists. When the owner of an easement uses it in a way
that exceeds its legal scope (i.e., the easement is surcharged), the servient landowner may enjoin
the excess use and possibly collect damages. If the easement by necessity had been located in
the driveway, the excess use from the tarot business could have been the basis for the court’s
ruling in the landowner’s favor. However, as stated above, the easement is in the private road
from the highway. (D) is incorrect because the retiree’s use has not been continuous for the five-
year period. To acquire an easement by prescription, the use must be: (i) open and notorious, (ii)
adverse, and (iii) continuous and uninterrupted for the statutory period. Continuous adverse use
does not mean constant use. Periodic acts that put the owner on notice of the claimed easement
fulfill the requirement. In this case, however, two uses in the first two years would not be suffi-
cient to put the landowner on notice that the retiree intended to claim an easement in the driveway.
Therefore, the retiree has not acquired a prescriptive easement in the driveway from the county
road.

Answer to Question 17

(D) The neighbor will not prevail. The owner has made an adverse use of the neighbor’s land since
the sewer line was first constructed across it. The fact that the owner tore down the house on his
property does not stop the prescriptive period from running, because the mere existence of the
sewer line was an adverse use and there was no intent to abandon it. Therefore, the owner’s use
ripened into a prescriptive easement 20 years after that use first began. Moreover, the use of the
line to service the new house is within the scope of the easement by prescription because the
same pipe in the same location will be used for the new house. (A) is incorrect. By the time the
owner constructed the new house, he had maintained the sewer line across the neighbor’s property
for the statutory period and therefore had acquired an easement by prescription to continue to
use that line. Once the owner obtained an easement by prescription over the neighbor’s property,
that property right persists despite any actions by the municipal authorities with regard to the
neighbor’s property, or any actions by the owner with regard to the authorities. (B) is incorrect.
An easement is not terminated merely because it is not used for a long period of time. It can only
be extinguished when the owner clearly demonstrates by physical action an intent to permanently
abandon it. Here, abandonment could have arisen if the owner had been able to connect to a new
sewer line, but that did not happen. (C) is incorrect. The owner never had a license. A license
is a permissive use of property. The owner placed the sewer line across the neighbor’s property
without her permission. Moreover, the use never ripened into an implied easement. Such an
REAL PROPERTY - COVENANTS, EASEMENTS, FIXTURES ANSWERS 9.

easement can only occur when there is a division of a large parcel of property into smaller parcels
and there is a reasonable necessity for the owner of one of the parcels to continue to use rights
over another part of the larger parcel. There is no such division of property here. Instead, the
adverse use by the owner ripened into an easement by prescription.

Answer to Question 18

(B) If the owner prevails it will be because the proposed easement constitutes an excessive burden.
The grant was for use of a single lane gravel road, not for a wider, paved road, and a court could
easily find this burden excessive. (A) is incorrect because the easement involved is an express
easement, not an easement by necessity. (C) is an incorrect statement of law; the owner of a
servient estate does not have the obligation to maintain an easement and cannot unilaterally
control the nature of its improvement. (D) is incorrect because a surcharging of the easement does
not terminate the easement; it merely gives the owner of the servient estate the right to stop the
additional use.
REAL PROPERTY - DEEDS,
WILLS, RECORDING
QUESTIONS
REAL PROPERTY - DEEDS, WILLS, RECORDING QUESTIONS 1.

REAL PROPERTY - DEEDS, WILLS, RECORDING QUESTIONS

Question 1 Question 2

A rancher owned a 100-acre tract of land. An elderly man wrote a note to a friend
Eighty acres had been devised to him by his stating that he wanted her to have his house and
father, and the rancher had acquired title to an land. The man then went to his lawyer and had
adjacent 20 acres from a prospector by adverse her draft a deed conveying the property to his
possession. The rancher entered into a land sale friend. The man validly executed the deed and
contract in which he promised to convey the gave both the note and the executed deed to the
100 acres to an investor. The contract listed the lawyer, telling her to give them to the friend on
description of the property and was otherwise his death.
definite and certain as to the transaction, but
the contract did not state the nature of the title The man continued to live on the property
that the rancher was to convey to the investor. until his death one year later. Shortly thereafter,
At the time of closing, the investor paid the his will was admitted to probate; it left all of
purchase price and accepted the deed conveyed his property to a cousin. Prior to the probate of
by the rancher. Six months later, the prospector the estate, the friend received the note and deed
returned to the area and brought an action in from the lawyer, and promptly recorded the
ejectment against the investor for the 20 acres. deed. After probate, she brought an appropriate
The investor now sues the rancher. action to quiet title to the property conveyed by
the deed.
Which of the following statements would most
accurately describe the investor’s rights? In that action, the court should find for:

(A) The investor’s rights are based on the im- (A) The friend, because the deed as delivered
plied covenant contained in a marketable constituted a valid conveyance of the prop-
title. erty.

(B) The investor could bring an action for refor- (B) The friend, because the man’s note to
mation of the deed with an abatement of the her constituted a valid conveyance of the
price of the land. property.

(C) The terms of the investor’s deed control the (C) The cousin, because the deed conveying
rancher’s liability. the property to the friend was not recorded
until after the man’s death and thus was not
(D) The investor could bring an action against effective.
the rancher for fraud.
(D) The cousin, because the fact that the man
remained in possession of the property
rendered the conveyance in the deed to the
friend ineffective.
2. REAL PROPERTY - DEEDS, WILLS, RECORDING QUESTIONS

Question 3 Question 4

A woman purported to convey a vacant parcel A landowner conveyed a vacant tract of land
of land she did not own to a buyer by a general to a developer by quitclaim deed. The devel-
warranty deed. The buyer immediately recorded oper did not record its deed. Prior to the start
the deed, built a house on the land, and has of construction, the developer abandoned the
resided there ever since. Six months later, the project and returned the deed to the landowner.
woman acquired title to the parcel of land from The landowner then executed and delivered a
the true owner. The woman immediately sold general warranty deed conveying the land to a
the land to an investor by a general warranty buyer. The buyer promptly and properly recorded
deed, which the investor promptly recorded. his deed. Subsequently, the developer decided
The investor paid the woman a reasonable value to proceed with its original development of the
for the parcel of land. Because the investor land it had purchased from the landowner. The
purchased the parcel of land solely as an invest- jurisdiction has the following recording act: “A
ment and never visited it, she never knew that conveyance of an interest in land shall not be
the buyer was living there. valid against any subsequent purchaser for value,
without notice thereof, unless the conveyance is
Who has title to the parcel of land? recorded.”

(A) The buyer, and the investor has no enforce- In a suit between the developer and the buyer,
able claim to the parcel of land. the buyer will prevail, if at all, because:

(B) The buyer, but the investor can sue to (A) The buyer recorded his deed.
recover title under the title covenants.
(B) The developer’s interest terminated when it
(C) The investor, and the buyer has no enforce- returned its deed to the landowner.
able claim to the parcel of land.
(C) A general warranty deed has priority over a
(D) The investor, but the buyer can sue to quitclaim deed.
recover title on an estoppel theory.
(D) The buyer is a bona fide purchaser.
REAL PROPERTY - DEEDS, WILLS, RECORDING QUESTIONS 3.

Question 5 rendered.

A father drew up a deed conveying his land A landowner conveyed a lot in that state to
to his son. The father never recorded the deed his aunt, who had had a judgment lien recorded
and left it in the top drawer of his desk in his against her two years earlier in the county in
study. Two years later, the father died. He left a which the land was located. One year later, the
will, which declared that all of his property be aunt conveyed the property to a buyer by general
divided equally between the son and the father’s warranty deed. The deed did not mention the
daughter. lien, but the buyer was aware of it. Two years
later, the buyer conveyed the property to a
While going through his father’s personal creditor by special warranty deed. The creditor
effects, the son discovered the deed to the land. was not aware of the lien and her deed also made
He showed the deed to his sister and the two of no mention of it. One year after that transaction,
them agreed not to record the deed. The son put the creditor conveyed the property to a devel-
the deed in a desk drawer in his home. A year oper by general warranty deed. The developer’s
later, the son died. As the executor perused the deed did not mention the lien but the developer
son’s personal papers, he came across the deed was aware of it. The next year, the developer
and promptly recorded it. He then entered into a entered into a contract to convey the property to
contract to sell the land to a buyer. The daughter an entrepreneur. The entrepreneur’s title search
discovered this and promptly filed suit, claiming disclosed the judgment lien against the aunt,
an interest in the land. A statute of the jurisdic- and the entrepreneur refused to proceed with
tion provides: “No conveyance is good against a the transaction because title was not market-
subsequent purchaser for value, without notice, able. The developer brought an action against the
who first records.” entrepreneur for specific performance and was
denied relief. He then brought an action against
How will the court rule? the aunt, the buyer, and the creditor for breach of
warranty.
(A) In favor of the daughter, because there was
no proper delivery of the deed to the land. Assuming that all transactions concerning the
property were promptly and properly recorded,
(B) In favor of the daughter, because the and that the party holding the judgment lien has
executor violated his fiduciary duty when taken no action as of yet to enforce it, which
he recorded. parties, if any, will be liable to the developer?
(C) Against the daughter, because the executor (A) No one, because the developer had actual
and the buyer are protected by the knowledge of the lien when he purchased
recording act. the property.
(D) Against the daughter, because she is not a (B) The creditor only, because the party holding
bona fide purchaser. the judgment lien has taken no action as of
yet to enforce it.
Question 6
(C) The creditor and the aunt, because the
A state statute provides as follows: buyer conveyed by special warranty deed.
Any judgment properly filed shall, for (D) The buyer and the aunt, because they
10 years from the date of filing, be a were aware of the judgment lien but did
lien on the real property then owned not mention it in their deeds, but not the
or subsequently acquired by any creditor, because she was not aware of the
person against whom the judgment is lien.
4. REAL PROPERTY - DEEDS, WILLS, RECORDING QUESTIONS

Question 7 Question 8

A woman and her friend lived together in A man gave a friend a deed purporting to give
the woman’s home for 20 years. Subsequently, to the friend “My property known as Twelve
the woman became disabled because of a heart Oaks, with its five acres of land and the stable
ailment and the friend had to take care of her. and dressage course located thereon.” The man
The woman told the friend that she wanted to be told the friend that he was giving the property
sure that the friend got her house after she died, to her, but because he did not have a better
so she gave the friend a quitclaim deed. The description, he wanted to keep the deed until
friend did not record the deed, but put it in his his attorney could review it. The friend agreed
safe deposit box. and gave it back to him. Unfortunately, the man
suffered a heart attack the next day and died
Four months later, the woman’s son found out without seeing his attorney. However, when the
about this and told his mother that if she would friend spoke with the administrator of the man’s
sell the house to him, she could live there for estate, she learned that the man had, as part of
the rest of her life. The woman, who wanted another deed 12 years before, sold that part of
the money, agreed and carried out the transac- Twelve Oaks on which was located the stable
tion. She told the friend that she had changed and dressage course.
her mind and decided to leave the home to her
children. The friend promised to destroy the The man’s heirs bring an action for declara-
deed, and the next day, he did. Several days tory relief against the friend, asserting that the
later, however, as the friend and the woman deed to Twelve Oaks is void because it contained
were driving to the store, their car was hit by a an inaccurate and ambiguous description.
train and they both died. The woman’s and the
friend’s heirs claim title to the house. How will the trial court rule?

The friend's heirs bring an appropriate action (A) Against the friend, because the deed failed
to resolve the dispute. Which party will the court to give an accurate description of the prop-
likely find owned the house? erty owned by the man at the time of his
death.
(A) The friend, because the woman did not tell
him the truth about why she was revoking (B) Against the friend, because the man
her agreement. informed the friend that he did not know
the description of his property and he
(B) The friend, because he did not retransfer wanted his attorney to prepare the deed.
title to the woman.
(C) Against the friend, because the deed
(C) The woman, because the friend agreed to purports to transfer more property than the
return the title, and did in fact destroy the man owned and thus is void as a matter of
deed. law.

(D) The woman, because the friend, as a donee, (D) For the friend, because the error in descrip-
would not be able to prevail against the son, tion was not sufficient to put in doubt what
who was a bona fide purchaser. the man intended to convey to her.
REAL PROPERTY - DEEDS, WILLS, RECORDING QUESTIONS 5.

Question 9 Question 10

A mother properly executed a deed, A landowner conveyed a parcel of land to


purporting to convey a parcel of land to her a buyer by warranty deed. The buyer did not
son. The mother gave the deed to her son for record the deed. One year later, the landowner
his birthday. A week later, the son decided that entered into a written contract with a friend, who
he should not have accepted such an expen- had performed many favors for the landowner
sive gift and told his mother so, suggesting that over the years, to convey the land to the friend
she devise the land to him by will rather than for “one dollar.” On the closing date provided in
by deed. Instead, the mother executed a will, the contract, the friend handed the landowner
devising the land to “my son and daughter, share a certified check for one dollar. The landowner
and share alike.” Two years later, the mother handed the friend a quitclaim deed conveying
died. The next day, the son found the deed in his the land to him. The friend promptly recorded
desk drawer and recorded it. the deed.

Who has title to the land? A statute of the state in which the land is
located provides: “Any conveyance of an interest
(A) The son and the daughter, because the son in land, other than a lease for less than one year,
did not pay valid consideration for the land. is not valid against any subsequent purchaser for
value without notice thereof whose conveyance
(B) The son and the daughter, because the will is first recorded.”
is evidence that the mother did not intend to
pass title to the son only. What is the friend’s greatest obstacle to
prevailing in an action to establish ownership of
(C) The son, because the deed was recorded by the land?
him.
(A) A quitclaim deed is not a valid transfer of
(D) The son, because the deed was delivered to title.
him.
(B) The transaction must be shown to be a
transfer for value.

(C) Warranty deeds are superior in right to


quitclaim deeds.

(D) A quitclaim deed is not a recordable instru-


ment.
6. REAL PROPERTY - DEEDS, WILLS, RECORDING QUESTIONS

Question 11 Question 12

A landowner owned a large tract of mineral- A landowner borrowed $30,000 from a


rich land in a sparsely populated area. He bank, secured by a mortgage on his land. The
entered into a lease with a prospector who was mortgage papers were signed by the landowner
interested in developing the land for mining. The and by the chief loan officer as agent for the
term of the lease was two years and gave the bank on March 18. The loan officer filled out the
prospector an option to buy the property at any appropriate recording form and gave it to a bank
time after the first year. The prospector did not clerk on March 19, instructing him to file the
record the lease. Six months later, the prospector papers at the county recorder’s office. The bank
left the land for a period of time to prospect in clerk inadvertently misplaced the papers. He
Mexico, leaving no goods on the land that would discovered the papers on April 10 and filed them
identify him. The landowner then conveyed the with the county recorder.
property in fee simple to a developer, who had
inspected the property while the prospector was At the recorder’s office, the bank clerk discov-
in Mexico and was unaware of the prior transac- ered a conveyance of the landowner’s land
tion. The developer did not immediately record from the landowner to a buyer dated April 5
her deed. After three months in Mexico, the and recorded on April 8. Subsequent inquiry
prospector returned to the land and encountered revealed that the buyer paid the landowner
the developer. $150,000 for the land after a diligent title search
and that the buyer had no knowledge of the
A statute in the jurisdiction provides, in part: mortgage on the property until the loan officer
“No conveyance or mortgage of an interest in contacted her on April 11. The jurisdiction in
land, other than a lease for less than one year, is which the land is located follows the lien theory
valid against any subsequent purchaser for value of mortgages, and has a statute providing: “Any
without notice thereof whose conveyance is first conveyance of an interest in land shall not be
recorded.” valid against any subsequent purchaser for value
If the developer brings an action to quiet title, without notice thereof who first records.”
how should the court rule?
If the bank seeks a declaration from the court
(A) The developer takes title subject to the that the buyer owns the land subject to a $30,000
prospector’s leasehold interest and his op- mortgage with the bank, is the bank likely to
tion to purchase, because the prospector prevail?
acquired his interest first.
(A) Yes, because the bank’s interest was ac-
(B) The developer takes title subject to the quired for value prior to the date when the
prospector’s leasehold interest but not his buyer recorded.
option to purchase, because he does not yet
have the power to exercise the option. (B) Yes, because the mortgage was merely
(C) The developer takes title subject to the security for a loan.
prospector’s leasehold interest and option
(C) No, because the buyer recorded first.
to purchase regardless of whether she now
records, because she will have had notice of (D) No, because the jurisdiction follows the lien
the prospector’s interest before recording. theory.
(D) The developer takes title free of the
prospector’s leasehold interest and option,
because the prospector failed to record
before the developer purchased the property
without notice of his interest.
REAL PROPERTY - DEEDS, WILLS, RECORDING QUESTIONS 7.

Question 13 Question 14

A businesswoman owned an office building. A landlord leased an office building to a


She sold the building to an investor, who paid tenant for 10 years. The tenant, a second-
her a fair price for the property. The investor year law student, was familiar with the state’s
failed to record the deed and left for an extended recording act, which provided:
trip. Two days after he departed, the business-
woman died suddenly. Her will devised the No conveyance is valid against any
office building to her daughter. The daughter subsequent purchaser for value without
knew nothing about the sale of the building notice unless the conveyance is record-
to the investor, and she properly recorded her ed. No lease for three years or more is
title to the building in the county recorder of valid against a subsequent purchaser
deeds office. The daughter, who had no prior for value without notice unless the
experience in business, applied to a lender for lease has been recorded.
a $50,000 loan to start up her own business,
offering a mortgage on the office building as Believing it would be obvious to any prospec-
collateral. The lender checked the recorder’s tive purchaser that the tenant was in possession
records showing the daughter to have sole title of the property, she failed to record the lease.
to the building, and the loan and mortgage were
executed. Shortly thereafter, the landlord entered into
a contract to sell the leased property to a buyer.
Unfortunately, the daughter’s business failed Before purchasing the property, the buyer merely
and she was unable to repay her loan to the drove by it, and thus did not notice the tenant’s
lender. The lender instituted foreclosure proceed- occupancy. The standard title search did not
ings. Just as the building was about to be sold, reveal the lease because it was unrecorded.
the investor returned from his trip, deed in hand. The buyer tendered the purchase money to the
The state in which the building is located has the landlord, and the landlord conveyed to the buyer
following statute: “No conveyance or mortgage the property by warranty deed. The buyer subse-
of an interest in land is valid against any subse- quently found the tenant in possession of the
quent purchaser for value without notice thereof, premises and ordered her to vacate. The tenant
unless it is recorded.” refused and asked the buyer where she should
send the rent checks.
May the lender successfully take the office
building? In an action by the buyer to evict the tenant,
how should the court rule?
(A) Yes, because the lender succeeds to the
daughter’s rights in the building. (A) The tenant wins, because the buyer’s drive-
by inspection will be deemed to confer
(B) Yes, because the lender is a mortgagee for actual notice on him.
value.
(B) The tenant wins, because the buyer had a
(C) No, because the daughter never owned the duty to properly inspect the property.
building.
(C) The buyer wins, because the buyer is a
(D) No, because the lender was imprudent in subsequent purchaser for value and the
lending the money in the first place. tenant failed to record.

(D) The buyer wins, because the tenant knew of


the statute and willfully failed to record.
8. REAL PROPERTY - DEEDS, WILLS, RECORDING QUESTIONS

Question 15 In a suit between the friend and the woman,


which of the following statements most
A large tract of land was located in a jurisdic- accurately describes the probable outcome?
tion that has adopted the following statute:
(A) The friend would prevail, because the
No conveyance or mortgage of an money he paid for the property, along with
interest in land is valid against a sub- the money expended since then, was far in
sequent purchaser for value without excess of what the woman paid, and under
notice thereof whose conveyance is equity, the friend would be deemed the
first recorded. owner; however, he would have to reim-
burse the woman for what she paid for the
The man who owned the land owed money property.
to a woman, and in satisfaction of this debt, the
man conveyed the property to her. Although (B) The friend would prevail, because under the
the woman intended to have the deed recorded, doctrine of equitable conversion, his “right”
she mistakenly failed to do so. Two years later, to the property preceded the woman’s
the man borrowed money from a bank and, to recordation, and thus whatever right she
secure the loan, executed a mortgage deed on may have had would have been terminated
the property. The bank promptly recorded this before she could record.
mortgage. Three months later, the man, just
before he died, donated the property by general (C) The friend would prevail, because he
warranty to his son, who did not know about purchased from the son, whose deed was
the prior events. The son recorded the deed and recorded before the woman’s deed.
entered into a contract with his friend to sell
him the property. The next month, the woman (D) The woman would prevail, because she
discovered that the deed in her safe was not recorded first.
recorded, and so, without notice of any of the
prior transactions, the woman recorded the deed.
A month after that, the friend paid the son full
value for the property, and without actual knowl-
edge of any of the other transactions regarding
the property, the friend had the deed duly
recorded. By the end of the next year, the friend
had expended substantial sums of money on the
property. However, when he put up the property
as security for a loan from the bank, he learned
for the first time of the woman’s claim.
REAL PROPERTY - DEEDS, WILLS, RECORDING QUESTIONS 9.

Question 16 Question 17

A woman conveyed her real property to a A woman owned a tract of land. The jurisdic-
buyer for $90,000. The buyer never recorded tion in which the land was located had a statute
his deed. A year later, the woman drew up a providing that no conveyance or mortgage of
deed of conveyance for the property and gave an interest in land is valid against a subsequent
it to her son as a gift. The son knew nothing of purchaser for value without notice thereof whose
the prior conveyance to the buyer, and the son conveyance is first recorded.
recorded his deed. The following year, the son
sold the property to a friend via an installment The following events occurred:
land sale contract. The full purchase price was
$100,000, and the friend made her first install- December 2007: The woman owed money
ment payment of $40,000 to the son. The friend to a man, and in satisfaction of this debt, the
received a deed from the son and promptly and woman conveyed the property to him. Although
properly recorded it. According to the terms the man intended to have the deed recorded, he
of the installment agreement, the friend was to mistakenly failed to do so.
make two further payments of $30,000 each.
August 2008: The woman borrowed money
The first of these payments was due a year after
from a bank and, to secure the loan, executed
the closing date and the second payment a year
a mortgage deed on the property. The bank
after that. The buyer learned about the son’s
promptly recorded this mortgage.
conveyance to the friend and filed suit against
the friend, seeking to oust her from the property December 2008: The man discovered that the
and to quiet title in the buyer. The recording deed from the woman’s property was still in his
statute in the state where the property is located safe and had not yet been recorded. He immedi-
and where all the parties reside reads in relevant ately went and recorded it.
part: “No conveyance shall be good against a
subsequent purchaser for value, without notice, In February 2009, the woman defaulted on her
unless it be recorded.” loan, and the bank therefore instituted proceed-
ings to foreclose on the property.
Which of the following judicial orders would
it NOT be appropriate for the court to make? The bank will:
(A) Award the friend a 40% share of the prop- (A) Not prevail, because the woman conveyed
erty as a tenant in common with the buyer. her entire interest to the man, and therefore
had no property left that she could have
(B) Grant the property to the buyer, but order mortgaged.
the buyer to pay the friend $40,000.
(B) Prevail, because to not enforce the bank’s
(C) Grant the property to the friend, but order claim would reward the woman for perpe-
the friend to pay the buyer the remaining trating a fraud on the bank.
$60,000 in payments.
(C) Prevail, because the bank loaned the
(D) Grant the property solely to the friend, woman the money without notice of the
based on the recording statute, without any man’s interest, and it recorded the mortgage
compensation to the buyer. deed first.

(D) Prevail, because the instrument that is


recorded first under this type of statute
always prevails over the unrecorded or
subsequently recorded instrument.
10. REAL PROPERTY - DEEDS, WILLS, RECORDING QUESTIONS

Question 18

A mother executed a will, devising a vacant


parcel of land to her daughter. The daughter
moved her mobile home onto the land. Two
years later, the mother conveyed the land to an
investor for $25,000 by a quitclaim deed. The
investor, knowing nothing of the mother’s will,
recorded the deed but has never visited the land.
The following year, the mother died, survived
by her daughter. The jurisdiction in which the
land is located has a five-year limitations period
for adverse possession along with the following
statute:

No conveyance of an interest in land,


other than a lease for less than one
year, shall be valid against any sub-
sequent purchaser for value, without
notice thereof, unless the conveyance
is recorded.

If the daughter sues to quiet title to the land,


for whom should the court rule?

(A) The daughter, because a quitclaim deed


conveys whatever interest the seller has.

(B) The daughter, because the investor was not


a bona fide purchaser.

(C) The investor, because the daughter’s mobile


home is not a fixture.

(D) The investor, because the daughter has no


interest in the land.
REAL PROPERTY - DEEDS,
WILLS, RECORDING
ANSWERS
REAL PROPERTY - DEEDS, WILLS, RECORDING ANSWERS 1.

REAL PROPERTY - DEEDS, WILLS, RECORDING ANSWERS

Answer to Question 1

(C) The terms of the deed control the rancher’s liability. If a general warranty deed was used, the
rancher may have to defend the investor against the lawsuit and compensate the investor in the
event of an adverse judgment. If a quit claim deed was used, the investor has no rights with
respect to the rancher because a quit claim deed creates no warranties of title. (A) is incorrect
because of the merger doctrine. The law implies that a marketable title must be conveyed at a
closing and title based on adverse possession is not marketable, although it may be good, so the
investor could have refused to perform at closing. But once the investor accepted a deed, the
contract is merged into the deed. Any contract provisions for quality of title, express or implied,
are no longer effective. The investor must look to the terms of the deed for his rights. (B) is incor-
rect because there was no mistake, so the court will not rewrite the deed. The facts do not indicate
that the rancher made any misrepresentations, so (D) is incorrect.

Answer to Question 2

(A) The court should find for the friend. A grantor may deliver a deed to an escrowee with instruc-
tions that it be delivered to the grantee when certain conditions (e.g., death of the grantor) are met.
When the conditions occur, title passes automatically to the grantee and relates back to the date
of delivery to the escrowee. Thus, (D) is incorrect. (B) is not as good an answer because the facts
given are insufficient to conclude that the note is a valid conveyance. To work a conveyance, the
note would have to include the names of the grantor and grantee, operative words of conveyance
(questionable here), a description of the land, and the grantor’s signature. (C) and (D) are wrong
because deeds may be effective to convey property although recorded after the grantor’s death or
although the grantor remains in possession after executing the deed.

Answer to Question 3

(A) The buyer has title to the parcel of land. Under the estoppel by deed theory, if a grantor purports
to convey an estate in property that she does not then own, her subsequent acquisition of title to
the property will automatically inure to the benefit of the grantee, but only as against the grantor.
If the grantor transfers her after-acquired title to a bona fide purchaser for value (“BFP”), the
BFP gets good title. Here, when the woman acquired title to the parcel of land from the true
owner, that title passed automatically to the buyer by operation of law. Although the investor
paid a reasonable value for the parcel of land, she is not a BFP because she had at least inquiry
notice of the buyer’s interest in the land. Under the majority view, a recorded deed obtained from
a grantor who had no title at that time but who afterwards obtains title is not in the chain of title
and so does not give constructive (or record) notice to a subsequent purchaser. However, a subse-
quent purchaser is charged with knowledge of whatever an inspection of the property would have
disclosed and anything that would have been disclosed by inquiring of the possessor. Here, an
examination of possession would have revealed the buyer’s interest in the parcel of land. Thus, the
investor has no enforceable claim to the land and (C) is wrong. (B) is wrong because the investor’s
cause of action against the woman for breach of the covenants of seisin (i.e., the grantor has the
estate or interest that she purports to convey) and right to convey (i.e., the grantor has the power
and authority to make the grant) does not affect the buyer’s title to the parcel of land. (D) is wrong
because it states the minority position under estoppel by deed. Under the majority view, title
passes to the buyer automatically; he need not bring suit to recover title.
2. REAL PROPERTY - DEEDS, WILLS, RECORDING ANSWERS

Answer to Question 4

(D) If the buyer prevails it will be because he is a bona fide purchaser. The jurisdiction’s recording
act is a notice statute. Under a notice statute, a subsequent bona fide purchaser (i.e., a person who
gives valuable consideration and has no actual, record, or inquiry notice of the prior instrument)
prevails over a prior grantee who failed to record. Here, the developer failed to record its deed.
Thus, the buyer will prevail if he is a bona fide purchaser. The buyer did not have actual notice of
the developer’s interest in the land, nor record notice of the developer’s unrecorded deed. Because
the land was still vacant, the buyer also did not have inquiry notice. Thus, assuming the buyer
paid value for the land, he is a bona fide purchaser. (A) is wrong because the recording act is a
notice statute. The buyer would prevail against the developer even if the buyer had not recorded
(in contrast to the result under a race-notice statute). (B) is wrong because the developer’s return
of its deed does not affect its interest. The developer loses under the recording act because the
buyer is a bona fide purchaser. (C) is a misstatement of law. A general warranty deed is one in
which the grantor covenants against title defects created both by herself and all prior titleholders,
while a quitclaim deed conveys whatever interest, if any, the grantor has in the property. Neither
has priority over the other.

Answer to Question 5

(A) The daughter will prevail because the deed executed by the father to the son was never properly
delivered. A deed is not effective unless it is delivered. Unless there is some clear expression of
intent that the grantor envisioned the passage of title to the grantee (i.e., that the grantor intended
to relinquish control over the property), the continued possession of the deed raises a presumption
of nondelivery. The father did not do anything to indicate an intent to pass immediate title to the
son. The presumption of nondelivery is not rebutted and the father retained title. Therefore, the
land was part of the father’s estate and the daughter and the son inherited it as tenants in common.
(Multiple grantees are presumed to take as tenants in common.) The daughter has an undivided
one-half interest in the land. (B) is wrong because whether the executor violated his fiduciary duty
has no bearing on the ownership of the land. That fact would be relevant only if the daughter were
seeking damages from the executor. The executor’s recording of the void deed did not damage the
daughter because it did not affect her rights. (C) is wrong for two reasons: (i) The buyer cannot
be protected by the recording act because he has not yet recorded and this is a race-notice juris-
diction. To prevail in a race-notice jurisdiction, a party must be a subsequent purchaser for value,
without notice of an adverse claim, and must record first. The buyer has not yet recorded. It is not
clear from the facts whether he has even received the deed. If he has not, he cannot qualify as a
bona fide purchaser because he now has notice of the daughter’s claim. (ii) The executor cannot
claim the protection of the recording act because he is acting as the son’s agent. The son was a
donee with notice and as such was outside the protective provisions of the recording act. (D) is
wrong because, as explained above, the daughter need not be a bona fide purchaser (i.e., need
not turn to the recording act) to prevail. She inherited an undivided one-half interest in the land,
which has not been cut off by the subsequent acquisition and recording by a bona fide purchaser.

Answer to Question 6

(B) The developer will prevail only against the creditor because only the creditor has committed
an actionable breach of the covenant against encumbrances. A grantor making a conveyance by
general warranty deed generally makes five covenants for title, and warrants against title defects
created both by herself and by all prior titleholders. The covenant of seisin, the covenant of right
to convey, and the covenant against encumbrances are present covenants and are breached, if at
REAL PROPERTY - DEEDS, WILLS, RECORDING ANSWERS 3.

all, at the time of conveyance. The covenant for quiet enjoyment and the covenant of warranty
are future covenants and are breached only on interference with the possession of the grantee or
his successors. Unlike the future covenants, the present covenants do not “run” with the grantee’s
estate and cannot be enforced against the covenantor by successive grantees in most jurisdictions.
Here, because the party holding the judgment lien has not taken any action to enforce it, there
is no disturbance of possession and the future covenants have not been breached. Since the only
covenant that has been breached was the covenant against encumbrances, only the developer’s
grantor, the creditor, is liable. (A) is incorrect because most jurisdictions hold that a covenant
against encumbrances is breached even if the grantee knew of the encumbrance, particularly if it
is an encumbrance on title, such as a mortgage or lien, rather than a physical encumbrance such
as an easement. (C) is incorrect, even though it is true that the buyer will not be liable because
she conveyed by special warranty deed, which covenants only that the grantor herself did not
create any title defects. As discussed above, the aunt is not liable to the developer because any
future covenants she made have not been breached. (D) is incorrect because, as discussed above,
neither the aunt nor the buyer has breached any covenant owed to the developer, regardless of their
knowledge of the lien. On the other hand, the creditor has breached her covenant against encum-
brances despite her ignorance of the judgment lien.

Answer to Question 7

(B) The court will most likely find that the friend was still the owner of the house because he did not
retransfer title to the woman. The deed, once delivered, merely evidences title to the property, and
its destruction has no effect on the title. Thus, the friend was still the owner, and (C) is incorrect.
(A) is incorrect because the woman cannot affect the ownership of the property after she delivered
the deed to the friend, regardless of the truth or falsity of her subsequent reasons for the attempted
revocation. (D) is incorrect because the son was not a bona fide purchaser: He was aware of
the friend’s deed. Also, it is only the subsequent bona fide purchaser, a purchaser for value and
without notice, who gets the protection of the recording act. The friend was the first grantee and,
thus, his status as a donee would be irrelevant to the applicability of the recording act.

Answer to Question 8

(D) The court should find for the friend, because the error in description was not sufficient to put
in doubt what the man intended to convey to her. In land contracts and deeds, property may be
described in various ways; i.e., by reference to a government survey, by metes and bounds, by
courses and angles, by references to a recorded plat, by reference to adjacent properties, by the
name of the property, or by a street and number system. A description in a deed is sufficient if
it furnishes a good lead as to the identity of the property. Hence, (A) is wrong. (C) is incorrect
because a deed purporting to convey more land than the grantor owns does not invalidate the
deed. (B) fails to conform to the facts.

Answer to Question 9

(D) The son has title to the land because the deed was delivered to him. A deed must be delivered to
be valid. Delivery is presumed if the deed is: (i) handed to the grantee, or (ii) acknowledged by
the grantor before a notary and recorded. Title passes to the grantee upon effective delivery. Thus,
destroying the deed or returning it to the grantor has no effect; such an act constitutes neither
a cancellation nor a reconveyance. Moreover, there must be acceptance by the grantee in order
to complete a conveyance. Most courts will infer the grantee’s acceptance if the conveyance is
beneficial to him. However, all courts will consider evidence that is contrary to the presumption or
4. REAL PROPERTY - DEEDS, WILLS, RECORDING ANSWERS

inference. Here, the son’s statement that he should not have accepted such a gift occurred after the
conveyance was complete, i.e., after the deed was handed to the son and he did not object. Answer
(A) is incorrect because a deed may validly convey land by inter vivos gift (i.e., without consider-
ation) as long as there is donative intent, delivery, and acceptance, all of which are present here.
Answer (B) is incorrect because delivery refers to the grantor’s intent that the deed have some
present operative effect. Here, the mother possessed the requisite intent at the time of the convey-
ance. The fact that she changed her mind later is irrelevant, because delivery cannot be cancelled.
Answer (C) is incorrect because recording the deed had no effect on its validity; delivery had
occurred (thereby making the deed valid) two years earlier. Here, the deed to the son is valid
because it was delivered to him and he accepted it.

Answer to Question 10

(B) To be a bona fide purchaser for value, and thus entitled to the protection of the recording statute,
the purchaser must prove that real—not merely nominal—consideration was paid. In other words,
the friend must prove that he is a purchaser rather than a donee. The consideration need not be
adequate or the market value, but it must be of substantial pecuniary value. Generally, past consid-
eration (e.g., favors) is not sufficient. Thus, the friend will have a difficult time proving that one
dollar meets this test. (A) is wrong because title can be conveyed by quitclaim deed. (C) is wrong
because a warranty deed is not superior in right to a quitclaim deed. The only difference between
the two is that a warranty deed normally contains covenants for title, the breach of which gives
rise to a cause of action against the grantor, and a quitclaim deed contains no assurances by the
grantor. (D) is wrong because all instruments affecting title to real property are recordable and
should be recorded.

Answer to Question 11

(A) The developer’s ownership of the land is subject to the prospector’s interest because the prospec-
tor’s interest was first in time. The common law rule that priority is given to the grantee who was
first in time still applies unless operation of the jurisdiction’s recording statute changes the result.
The statute in this question is a race-notice statute, under which a subsequent bona fide purchaser
is protected only if she records her interest before the prior grantee does. While the developer
is a bona fide purchaser, she must still win the race to the recording office to prevail over the
prospector’s prior interest. Since neither party has recorded in this fact pattern, the developer will
take the property subject to the prospector’s prior interest. (B) is wrong because the prospector’s
option is treated as an interest in land just like his leasehold interest, regardless of the fact that it
cannot yet be exercised. (C) is wrong because the developer probably would prevail if she were
to record now. Under the recording statute, it is irrelevant that the developer became aware of the
prospector’s interest at a later point; at the time of the conveyance to her, she did not have notice
of his interest and therefore qualifies as a bona fide purchaser. Had she recorded when she first
encountered the prospector, she would have prevailed. Even if she were to record now, she could
still prevail if the prospector does not record first. (D) is wrong because it states the result under
a notice statute. In contrast to a notice statute, the race-notice statute in the question would still
permit the prospector to prevail even though he failed to record before the developer purchased
the property, as long as the developer does not record before he does.

Answer to Question 12

(C) The bank’s position will not be upheld because the buyer, a bona fide purchaser, recorded first.
The applicable recording statute is a race-notice statute, under which a subsequent purchaser for
REAL PROPERTY - DEEDS, WILLS, RECORDING ANSWERS 5.

value without notice of any prior conveyance is protected if she records before the prior grantee.
Here, the bank, as mortgagee, is a grantee of an interest in the land prior to the buyer. However,
the buyer purchased the land for valuable consideration and without notice (either actual, record,
or inquiry) of the prior conveyance to the bank. The buyer recorded her conveyance on April 8,
prior to the time the bank clerk filed on behalf of the bank. Consequently, the buyer satisfies the
statutory requirements, and she is protected against the bank’s claim. (A) is incorrect because,
to prevail, the bank must have actually recorded prior to the buyer. It is of no significance that
the bank acquired its interest in the property—even for value—prior to the date of the buyer’s
recordation. The bank is not protected by the recording statute because it recorded its interest
after recordation by another bona fide purchaser. (B) is incorrect because, although a mortgage
is a security interest for a loan, it is still an instrument creating an interest in land. As such, a
mortgage comes within the scope of the recording acts, so that a grantee thereof must record in
order to give notice of the conveyance to subsequent purchasers. Having failed to record in time,
the bank will lose against a subsequent bona fide purchaser who records first. (D) is incorrect
because whether the jurisdiction is a lien theory or a title theory state is irrelevant to this question.
Under either theory, the bank’s mortgage is an instrument creating an interest in land, and is thus
subject to the recording statute.

Answer to Question 13

(B) The lender may successfully take the office building. A purchaser for value without notice of
the prior conveyance at the time of the transaction is protected by the recording statute, which
is a “notice” type of statute. Under a notice statute, a subsequent bona fide purchaser (i.e., one
who gives valuable consideration and has no notice of the prior instrument) prevails over a prior
grantee who failed to record. Mortgagees are considered “purchasers”; thus, the lender is a bona
fide purchaser (“BFP”) and is entitled to the protection of the recording act. Because the investor
was a prior grantee who failed to record his deed, and because the lender did not have any notice
of the investor’s interest in the office building, the lender will prevail over the investor under
the notice statute. (A) is wrong because the daughter’s rights would be insufficient to protect the
lender because she was not a BFP and would not be protected by the notice statute. The daughter
was not a BFP because she did not pay value for the building; she inherited it under the business-
woman’s will. (C) is wrong because, even though the investor could have successfully challenged
the daughter’s right to ownership of the building because she was not a BFP for value, a subse-
quent mortgagee for value and without notice is protected by the recording statutes. (D) is wrong
because it is irrelevant to a determination of one’s interest under any of the recording acts. Even
if the lender was imprudent in loaning money to the daughter in the first place, when it lent the
money it satisfied the requirements of the notice statute to acquire a priority interest in the office
building.

Answer to Question 14

(B) The tenant will prevail because the buyer did not properly inspect the property. A title search is
not complete without an examination of possession. If the possession is unexplained by the record,
the purchaser is obligated to make inquiry. The purchaser is charged with knowledge of whatever
an inspection of the property would have disclosed and anything that would have been disclosed
by the possessor. Thus, the buyer is on constructive notice of the tenant’s possession and anything
that would have been disclosed by inquiring of the tenant. Because the buyer had this notice,
he is not protected by the notice recording statute, and he will take subject to the tenant’s lease.
(A) is wrong because “actual notice” means that the buyer was aware of the tenant’s possession.
The facts make clear that the buyer did not know of the tenant’s possession. Furthermore, the
6. REAL PROPERTY - DEEDS, WILLS, RECORDING ANSWERS

act of driving by has no legal consequence; thus, it would not be deemed to confer anything on
the buyer. (C) is wrong because the buyer had constructive notice of the tenant’s possession and,
therefore, is not protected by the recording statute. (D) is wrong because the tenant’s state of mind
is irrelevant for purposes of applying the recording statute.

Answer to Question 15

(D) The woman would prevail because she recorded her deed before the friend recorded his deed.
The jurisdiction in this question has a race-notice statute, under which a subsequent bona fide
purchaser is protected only if she records before the prior grantee. While the friend was a bona
fide purchaser, he did not record his interest in the property before the woman did; thus the
woman will prevail. (A) is incorrect because priority under a race-notice recording act, such as the
one in this question, is determined by the subsequent purchaser’s status as a bona fide purchaser
and on the basis of who records first. Courts do not determine ownership by balancing the
equities on the basis of who spent the most money. (B) is wrong because the doctrine of equitable
conversion, wherein equity regards the purchaser in a land sale contract as the owner of the real
property, does not change the result under the recording statute. The woman will prevail because
she recorded first. (C) is incorrect because the friend cannot rely on the son’s recording of his
deed; the woman recorded her interest before the friend recorded his. Furthermore, even if the
issue were whether the friend had actual notice of the woman’s interest, the friend could not rely
on the “shelter rule” that protects transferees from a bona fide purchaser, because the son was not
a purchaser for value and therefore not protected by the recording statute.

Answer to Question 16

(D) It would not be appropriate to grant the property solely to the friend without compensation to
the buyer. When a purchaser has paid only part of the purchase price under an installment land
contract, most courts hold that the purchaser is protected by the recording acts only to the extent
of payment made. Depending on the equities involved in the case, the court has three options:
(i) create a tenancy in common in the property, with the contract purchaser receiving a share of
the property equal to the proportion of payments made; (ii) award the land to the prior claimant,
but give the contract purchaser a right to recover the amount she paid (with interest), secured by
a lien on the property; or (iii) award the land to the contract purchaser, but require the contract
purchaser to make the remaining payments to the prior claimant. This last obligation is also
secured by a lien on the property. Thus, (A), (B), and (C) are all appropriate actions for the court.
(D) is not appropriate because it misstates the law. When only partial payment of the purchase
price has been paid, the subsequent purchaser is protected only to the extent of payment. Thus,
even if the equities favored the friend and she was allowed to retain the property, the buyer would
be entitled to the remaining payments.

Answer to Question 17

(C) The bank will prevail under the recording statute. The jurisdiction in this question has a race-
notice statute, under which a subsequent bona fide purchaser is protected only if he records before
the prior grantee or mortgagee. Here, the bank recorded its mortgage prior to the man’s recording
of his deed; thus, the man will take the property subject to the bank’s mortgage. (A) is incorrect
because even though at common law the woman would not have had any title to mortgage to the
bank after the conveyance to the man, the race-notice statute nullifies that priority as to the subse-
quent bona fide purchaser who first records. (B) is wrong because the issue is not whether the
woman would be rewarded, but which innocent party will be protected by the recording act. (D)
REAL PROPERTY - DEEDS, WILLS, RECORDING ANSWERS 7.

is incorrect because it more accurately describes a pure race jurisdiction. In a race-notice jurisdic-
tion, only the bona fide purchaser for value will prevail over a prior unrecorded deed or mortgage
interest; hence, a purchaser who records first under this statute would not prevail if he had actual
notice of a prior unrecorded interest.

Answer to Question 18

(D) The court should rule for the investor because the daughter has no interest in the land. If property
is specifically devised or bequeathed in the testator’s will, but the testator no longer owns that
property at the time of death, the gift is adeemed; i.e., it fails. Here, the mother specifically
devised the land to the daughter in her will. However, the mother conveyed the same land to the
investor prior to her death. Thus, the land was no longer in the mother’s estate at her death, so the
daughter takes no interest in the land under the will. Moreover, the daughter has not obtained title
to the land by adverse possession because she has only been in possession for three years, two
years less than that required by the statute. (A) is a correct statement of the law—quitclaim deeds
do convey whatever interest (if any) the seller has—but here the seller (the mother) owned the
land in fee simple, and thus conveyed a fee simple interest to the investor. (B) is incorrect because
the daughter has no interest in the land and because the investor was a bona fide purchaser. A
bona fide purchaser is a purchaser who takes without actual, constructive, or inquiry notice of a
prior conveyance and pays valuable consideration. Because a will takes effect only at the testa-
tor’s death, there was no prior conveyance here. Thus, the investor will not be held to have inquiry
notice of the daughter’s interest due to her being in possession of the land. (C) is incorrect because
whether the mobile home is a fixture is not at issue here. However, if the mobile home is a fixture
(i.e., a chattel that has been so affixed to land that it has ceased being personal property and has
become part of the realty), the trespassing daughter risks losing it to the investor. Trespassers
normally lose their annexations whether installed in good faith or not. Moreover, the trespasser
can be held liable for the reasonable rental value of the property on which she annexed the item
for the period she illegally occupied the land.
REAL PROPERTY - LAND
SALE CONTRACTS
QUESTIONS
REAL PROPERTY - LAND SALE CONTRACTS QUESTIONS 1.

REAL PROPERTY - LAND SALE CONTRACTS QUESTIONS

Question 1 Question 2

A buyer purchased a house from a seller. It A seller owned a two-acre tract of land, on
turned out that the concrete used to pour the which he built a single-family residence. The
foundation had been improperly mixed and the seller entered into a contract to sell the land
foundation was crumbling. The buyer discovered to a buyer for $200,000. One week before
that the cost of repairing the defective foundation closing, the buyer had a survey of the property
would be over $10,000. She filed suit against the conducted. It revealed that a portion of the
seller for the cost of repairs. seller’s house was 5.98 feet from the sideline.
The applicable zoning ordinance requires a
If the court rules in the buyer’s favor, what is six-foot sideline setback. The buyer refused to
the likely reason? go ahead with the purchase of the land on the
ground that the seller’s title was not marketable.
(A) The crumbling foundation makes the house
unsafe or uninhabitable. If the seller brings suit against the buyer for
specific performance, will he prevail?
(B) The seller was the builder of the house.
(A) Yes, because any suit against the seller con-
(C) The buyer took title to the house by cerning the setback would be frivolous.
warranty deed.
(B) Yes, because the setback violation is de
(D) The buyer had no knowledge of the defect minimis.
when she purchased the house, and the
defect was not reasonably apparent. (C) No, because any variation, however small,
amounts to a breach of contract.

(D) No, because the seller’s title is unmarket-


able.
2. REAL PROPERTY - LAND SALE CONTRACTS QUESTIONS

Question 3 Question 4

A seller entered into an enforceable written The fee simple owner of an unimproved
agreement to sell her home to a buyer for parcel of wooded land orally agreed to sell it
$150,000. The agreement provided that escrow to a buyer under an installment land contract.
would close on March 31, and on that date The buyer agreed to pay $5,000 down and $100
the seller would provide good and marketable a month for the next 10 years, and the owner
title to the house, free and clear of all encum- would retain the deed until the buyer finished
brances. On March 10, the seller was notified paying the installments.
by her insurance company that she had to renew
her insurance policy by March 15. The seller After making the down payment, the buyer
immediately notified the company that she did moved onto the property and began clearing
not want the insurance renewed at that time. some of the trees for a road and a cabin. He
Consequently, when the house was destroyed by regularly made the payments for several months
fire on March 25, it was uninsured. On March but then was killed by a falling tree. His properly
31, the buyer refused to close and the seller executed will conveyed his real estate to a friend
immediately brought an action against him for and the remainder of his estate, consisting of
specific performance. personal property and $200,000 in cash, to his
son. During the next several months, his estate
In this jurisdiction, which has no applicable failed to make payments on the installment
statute to govern this situation, the most probable contract. The owner then notified the estate that
result of this action would be: he was rescinding the deal and asserting owner-
ship of the parcel, and offered to return the
(A) The buyer prevails, because an implied amount the buyer had paid him, less expenses, as
term of all conveyances is that the property restitution.
at the time of closing will be in substantial-
ly the same condition as it was at the time The buyer’s estate initiated a quiet title action,
the contract was entered into. naming the owner, the friend, and the son as
parties. The estate’s filings indicated that it
(B) The buyer prevails, because as the house was prepared to complete the conveyance and
was destroyed, the seller would have redeem the land by paying the accelerated full
nothing to “sell” and, therefore, could not balance of the contract from the proceeds of the
convey marketable title. estate. The friend believes he should receive title
to the parcel free of any obligation on the install-
(C) The seller prevails, because under the ment contract.
doctrine of equitable conversion, the risk of
loss was on the buyer. Which of the following doctrines is inappli-
cable?
(D) The seller prevails, but since the house was
destroyed, she is only entitled to recover the (A) The doctrine of equitable conversion.
fair market value of the land itself.
(B) The doctrine of part performance.

(C) The equitable mortgage doctrine.

(D) The doctrine of exoneration.


一 信:liuxue119118 , 们 信免 供

REAL PROPERTY - LAND SALE CONTRACTS QUESTIONS 3.

Question 5 date as May 24. The buyer contracted to tender


the $200,000 on that date and the accountant
A seller entered into a written contract to contracted to convey “marketable title, free of
sell a tract of land to a buyer. The buyer was to encumbrances.”
pay $1,500 per month for five years, at which
time the seller would deliver a warranty deed. On May 15, the buyer’s title search in the
The contract was silent as to the quality of title county recorder’s office revealed the mortgage
to be conveyed. After making 12 payments, on the property. The buyer immediately
the buyer discovered that a neighbor had an contacted the accountant, who said that he
easement of way over the land, which was not planned to use the money he obtained at closing
discussed at the time the seller and buyer entered to pay off the mortgage. The buyer found
into the contract. The neighbor had not used this unacceptable and failed to appear at the
the easement over the previous year because appointed time and place of closing, although
she had been out of the country. On the basis the accountant was there on time with a deed of
of the easement, the buyer wishes to cancel the conveyance in hand.
contract.
If the accountant files an appropriate suit
Which party is more likely to prevail? against the buyer demanding specific perfor-
mance and wins, what is the most likely reason?
(A) The seller, because the neighbor’s easement
has been extinguished. (A) A mortgage used to secure a debt does not
constitute a legal “encumbrance.”
(B) The seller, because the buyer has no basis
on which to rescind the contract. (B) The vendor of real property need not
have marketable title until the time of the
(C) The buyer, because the obligation to convey closing.
marketable title is implied.
(C) The mortgage was unenforceable ab initio,
(D) The buyer, because the seller has breached because a preexisting debt is not adequate
the covenant against encumbrances. consideration.

Question 6 (D) A mortgage of the type described does not


follow the land, so the buyer would have
An accountant borrowed $30,000 from a taken the land free of the mortgage even if
bank to help set up a small business. There the accountant did not pay it off.
was an acceleration clause in the loan agree-
ment, which the bank could exercise any time
after six months unless the accountant provided
security for the loan. A few years later, the bank
invoked the acceleration clause. The accoun-
tant did not have the cash on hand to repay the
loan, so he offered the bank a $30,000 mortgage
on property he owned. The bank accepted the
mortgage and duly recorded it. There were no
other mortgages on the property, which the
accountant had inherited from his parents. The
following year, a buyer offered the accountant
$200,000 for the property, and the accoun-
tant promptly accepted. They entered into a
written land sale contract, setting the closing

一 信:liuxue119118 , 们 信免 供
4. REAL PROPERTY - LAND SALE CONTRACTS QUESTIONS

Question 7 Question 8

A landowner devised her campground in her A buyer agreed to purchase a portion of a


will “to my niece, her heirs, and assigns, so seller’s parcel for $25,000. The buyer and the
long as it is used for camping and recreational seller orally agreed that the property included
purposes; if used for any other purpose during in the purchase would be the westerly third of
her lifetime, then to the Girl Scouts of America.” the parcel, and the eastern boundary would be
Subsequently, the landowner died. The residuary a stone fence that ran from the northern border
clause of her will left all property not devised of the parcel to the southern boundary. Due to a
in the remainder of the will to her daughter and clerical error by the seller’s secretary, when the
sole heir. Soon thereafter, the daughter died agreement was reduced to writing, the eastern
intestate, her only heir being her son. boundary was stated to be the picket fence,
which is 275 yards east of the stone fence.
Last month, the niece entered into a contract
to sell the campground to a buyer for its reason- If the buyer sues for specific performance of
able market value. After the buyer received the the contract conveying the additional strip of
title report called for in the contract, he refused land, will the buyer likely prevail?
to proceed with the purchase, claiming that the
niece could not convey good title. The niece, (A) No, because the court will reform the con-
the Girl Scouts of America, and the buyer then tract.
execute a new contract calling for the former
two parties to sell the property at the same (B) No, because natural monuments prevail
price to the latter. The jurisdiction follows the over artificial monuments.
common law Rule Against Perpetuities, and a
statute provides that future estates and interests (C) Yes, because the language in the contract
are alienable, and may be devised or inherited, controls.
all in the same manner as possessory estates or
interests. (D) Yes, because evidence of the oral agree-
ment is inadmissible under the Statute of
Should the buyer proceed with the new Frauds.
purchase transaction?

(A) Yes, because good title can now be con-


veyed by the sellers.

(B) Yes, if the Girl Scouts of America promises


never to use its right of entry should the
buyer use the property for other than
camping and recreational purposes.

(C) No, because the daughter’s son has not been


included as a party selling the property.

(D) No, because no one can convey good title to


the property during the niece’s lifetime.
REAL PROPERTY - LAND SALE CONTRACTS QUESTIONS 5.

Question 9 Question 10

A seller entered into a written land sale A seller received a written offer in the mail
contract with a buyer on May 20, whereby the signed by a buyer to purchase the seller’s land for
seller agreed to sell a home to the buyer for $50,000. The written offer was legally sufficient
$60,000. The closing date was set at August 1. to form a written contract for the sale of the land.
The buyer put up $6,000 as earnest money, as The seller called the buyer and said that the offer
provided by the contract. The contract stated was acceptable, but that she wanted her attorney
that if the buyer failed to perform by tendering to review it. The seller asked her attorney to
the balance due on the house on August 1, the prepare a formal contract for the sale of the
$6,000 could be treated as liquidated damages land according to the same terms and condi-
“at the option of the seller.” tions in the written offer. When the attorney
had finished, the seller signed the contract and
On July 21, the week before the closing, the mailed it to the buyer. Later that day, before the
house burned to the ground because of a freak buyer had received the contract, a developer
lightning strike during a thunderstorm. When called the seller and offered to buy the land for
August 1 arrived, the buyer refused to tender $60,000, which the seller accepted immediately
$54,000 to the seller. The buyer asked the seller over the phone. The seller called the buyer and
for the refund of his earnest money because the told him that she had received and accepted a
house had been destroyed. The seller refused higher offer. The seller then signed a written
and filed suit, asking for specific performance. contract to sell the land to the developer. When
The buyer countersued, demanding refund of the the developer received the contract, he signed
$6,000 earnest money. it and then promptly and properly recorded it,
and sent the seller the specified down payment.
How should the court rule on the suits? The buyer received the written contract from the
seller the next day. The recording statute in the
(A) The court will order specific performance jurisdiction provides: “Any conveyance of an
by the buyer. interest in land, other than a lease for less than
one year, shall not be valid against a subsequent
(B) The court will order the seller to return the purchaser for value, without notice thereof,
$6,000 to the buyer because of frustration whose conveyance is first recorded.”
of purpose of the contract.
In an appropriate action brought by the buyer
(C) The court will award the seller $6,000 as against the seller and the developer for specific
liquidated damages. performance and to quiet title, will the buyer
win?
(D) The court will award neither party the relief
sought because of mutual mistake. (A) Yes, because the buyer’s written offer satis-
fies the Statute of Frauds.

(B) Yes, because the contract of sale prepared


by the attorney satisfies the Statute of
Frauds.

(C) No, because the buyer never entered into a


binding contract with the seller.

(D) No, because the recording statute protects


the developer.
6. REAL PROPERTY - LAND SALE CONTRACTS QUESTIONS

Question 11 Question 12

A businessman entered into a contract to sell A seller entered into a written contract to
his office complex to a purchaser for $1 million. sell his factory to a manufacturer. Before the
The purchaser paid the businessman $100,000 closing date, the manufacturer found an alternate
in earnest money. The day before the date set for site that was better suited to her business. The
closing, the purchaser died intestate, leaving her manufacturer notified the seller that she would
niece as her only heir. The niece showed up at not be going through with the closing. The seller
the closing with a certified check for $900,000. sued the manufacturer for specific performance.

Which of the following is correct? Which fact, if true, would cause the court to
rule in favor of the manufacturer?
(A) The niece may specifically enforce the
agreement. (A) The factory does not comply with the city
building code.
(B) The businessman may return the $100,000
down payment and cancel the contract. (B) The factory violates the setback require-
ment of the zoning ordinance by one foot.
(C) Death terminates the agreement.
(C) The zoning ordinance forbids the use of the
(D) Any title acquired would be unmarketable type of truck that the manufacturer uses in
by reason of the purchaser’s death. shipping her goods.

(D) Access to the factory is by way of an


easement over adjoining property.
REAL PROPERTY - LAND SALE CONTRACTS QUESTIONS 7.

Question 13 Question 14

A seller contracted to convey her property to A mother purchased 80 acres of desert land
a buyer for $75,000. A title search revealed the over 30 years ago. The deed was properly
following: (i) There were 25 years left on a lease recorded. Although her family had never even
of the property, which was recorded. The buyer visited the land, the mother had described to
agreed to take title subject to the lease but was them the little two-room cabin that sat in the
not aware that the lease gave the lessee, his heirs, middle of the parcel near the dry streambed. Ten
and assigns an option to purchase the land. (ii) years ago, the mother’s son found what he was
The roof of the garage on the property extended certain was the little cabin, and over the next few
approximately one-half inch across the property years he built a barn, a greenhouse, and some
line into the airspace of an adjoining neighbor. corrals, all enclosed by a sturdy wire mesh fence.
The garage did not interfere with any current or The area bounded by the fence, containing all
future use of the adjoining lot. (iii) The home on the structures, occupied about two acres of the
the property was subject to a $5,000 lien arising 80 owned by the mother.
from a dispute involving some remodeling
work. The seller promised to pay off the lien at Three years ago, the mother died, validly
closing with the proceeds from the sale. (iv) The devising the 80 acres to the son. The son
property was subject to an easement by necessity entered into a contract for sale of the two acres,
in favor of the adjoining neighbor. Last month, describing it in detail with reference to the struc-
the city extended the main road to the neighbor’s tures and nearby landmarks. The purchaser’s
land, but the neighbor planned to continue to use surveyor discovered that the son had settled
the easement because it was more convenient. onto a completely different parcel from the one
owned by the mother. The purchaser immedi-
In a jurisdiction that has a standard race- ately announced that he would not proceed with
notice recording statute and maintains the the sale contract. The state’s statutory period for
common law Rule Against Perpetuities without establishing adverse possession is five years.
any modern statutory reformation, which encum-
brance renders the seller’s title unmarketable? If the son brings an action for specific perfor-
mance of the sale contract, for whom should the
(A) The lessee’s option. court rule?

(B) The encroachment of the garage’s roof. (A) The purchaser, because the son does not
own the land he is purporting to sell.
(C) The $5,000 lien.
(B) The purchaser, because the son does not
(D) The easement. have marketable title to the land he is
purporting to sell.

(C) The son, if he conveys by quitclaim deed.

(D) The son, because the description in the


contract of sale is sufficient to identify the
property and need not be as accurate as one
contained in a deed conveying land.
REAL PROPERTY - LAND
SALE CONTRACTS
ANSWERS
REAL PROPERTY - LAND SALE CONTRACTS ANSWERS 1.

REAL PROPERTY - LAND SALE CONTRACTS ANSWERS

Answer to Question 1

(B) If the court rules in the buyer’s favor it will be because the seller was the builder of the house.
Generally, a conveyance of real property contains no warranties of quality or fitness for the
purpose intended, but there is a recognized exception for the sale of a new house by the builder.
There is an implied warranty that the new house is designed and constructed in a reasonably
“workmanlike” manner and suitable for human habitation. Thus, in this case, the buyer would
appear to have no claim against the seller unless he was the builder, in which case she could claim
that the house was not constructed in a reasonably “workmanlike” manner. (A) is wrong because
the fact that the house was unsafe and uninhabitable would not by itself result in the seller’s
liability unless he was the builder. (C) is wrong because the covenants contained in a warranty
deed are covenants for title (i.e., they protect the purchaser against competing claims for the title
to the property); they offer no protection against defects on the property. (D) is wrong because the
buyer’s knowledge of the defect is not relevant to the seller’s liability under these facts. While a
seller may be liable if he purposely conceals defects on the property or, in many states, if he does
not disclose serious defects that he is aware of, he is not generally liable for defects in the absence
of these circumstances. Because the facts do not indicate that the seller acted to conceal the
crumbling foundation, or was even aware of the problem, he will not be liable for the conditions
unless he was the builder.

Answer to Question 2

(D) The seller will not prevail because his title was unmarketable. There is an implied covenant in
every land sale contract that at closing the seller will provide the buyer with title that is market-
able. It need not be perfect title, but it must be free from questions that might present an unreason-
able risk of litigation. Because the placement of the seller’s house violated the zoning ordinance,
the buyer could be subject to suit. (A) is incorrect because the location of the house violates the
ordinance, and the local government has the power to enforce the ordinance strictly. (B) is incor-
rect for the same reason. It is probably unlikely that the local government would insist on the strict
enforcement of the zoning ordinance, but it has the power to do so, which makes title unmarket-
able, barring specific enforcement of the contract. As a practical matter, if the parties are acting in
good faith, this problem would probably be dealt with by the seller applying to the zoning authority,
usually a city or county zoning board, for a zoning variance. The board would probably grant
a permanent variance, allowing this particular piece of property to violate the six-foot rule by a
fraction of an inch. Once that variance is granted, title would be marketable because the property
would no longer violate any zoning rules applicable to the property. This solution would take some
time, but the parties could push the closing date back in order to make time. So, even though the de
minimis nature of the setback violation would probably allow for a solution to the problem, it would
not give the seller a right to specific performance. (C) is an incorrect statement of law. Building the
house too close to the sideline did not breach a contract; it violated an ordinance.

Answer to Question 3

(C) The seller prevails because the doctrine of equitable conversion places the risk of loss on the
purchaser as soon as the enforceable contract is entered into. (B) is wrong because “marketable”
title does not refer to whether the seller would be able to sell a destroyed home or not. It refers to a
deed free of any possible dispute as to who is the owner of the property. (A) is wrong because the
doctrine of equitable conversion applies to the risk of loss and, thus, there is no such implied term
2. REAL PROPERTY - LAND SALE CONTRACTS ANSWERS

in the conveyance or contract. (D) is wrong because the seller is entitled to receive the contract
price in the specific performance action.

Answer to Question 4

(C) The court will not apply the equitable mortgage doctrine in making its determination. A
landowner needing to raise money may “sell” the land to a person who will pay cash, giving the
“buyer” an absolute deed rather than a mortgage. If the court concludes that the deed was really
given as security for a loan, rather than a true conveyance, it will treat the deed as an “equitable”
mortgage and require that the lender foreclose it by judicial action, like any other mortgage. Here,
there was no such security arrangement between the owner and the buyer; rather, they entered
into an installment land contract arrangement. Thus, the equitable mortgage doctrine would
not support the court’s determination that the friend receive title to the parcel. (A) is incorrect
because equitable conversion is applicable here. Under that doctrine, once a contract is made
and each party is entitled to specific performance, equity regards the purchaser as the owner of
the real property. If the purchaser dies before title has passed, his interest is characterized as
real property in his estate and will go to the taker of the estate’s real property on closing, which
in this case is the friend. (B) is incorrect. Although the Statute of Frauds applies to land sale
contracts, the doctrine of part performance allows a court to order specific performance despite
the absence of a writing if additional facts are present. In most jurisdictions, part performance
can be established by two of the following: (i) possession of the land by the purchaser; (ii) making
of substantial improvements; and/or (iii) payment of all or part of the purchase price. Here, the
buyer’s estate could be entitled to specific performance of the contract because he had (i) moved
onto the property, (ii) begun clearing trees, and (iii) made several payments before his death. (D)
is incorrect. The court must apply the doctrine of exoneration, which is followed by a few states,
for the friend to receive the parcel free of the obligation on the installment contract. The doctrine
provides that when a testator makes a devise of real property that is subject to a mortgage, the
devisee is entitled to have the mortgage satisfied out of the testator’s residuary estate. Thus, if the
court applies exoneration, it will order that the mortgage be paid from the $200,000 cash resid-
uary that was left to the buyer’s son.

Answer to Question 5

(B) The seller is more likely to prevail because the buyer has no basis on which to rescind the
contract. Absent a provision to the contrary, a contract for the sale of land contains an implied
promise by the seller that she will deliver to the buyer a marketable title at the time of closing.
This promise imposes on the seller an obligation to deliver a title that is free from reasonable
doubt; i.e., free from questions that might present an unreasonable risk of litigation. Title is
marketable if a reasonably prudent buyer would accept it in the exercise of ordinary prudence.
An easement that reduces the value of the property (e.g., an easement of way for the benefit of a
neighbor) generally renders title unmarketable. If the buyer determines, prior to closing, that the
seller’s title is unmarketable, he must notify the seller and allow a reasonable time to cure the
defect. If the seller is unable to acquire title before closing, so that title remains unmarketable, the
buyer can rescind, sue for damages caused by the breach, or obtain specific performance with an
abatement of the purchase price. However, the buyer cannot rescind prior to closing on grounds
that the seller’s title is unmarketable. Where an installment land contract is used, the seller’s
obligation is to furnish marketable title when delivery is to occur, e.g., when the buyer has made
his final payment. Thus, a buyer cannot withhold payments or seek other remedies on grounds
that the seller’s title is unmarketable prior to the date of promised delivery. Here, there is a valid
easement on the property (see below), but the seller has four years in which to cure this defect.
REAL PROPERTY - LAND SALE CONTRACTS ANSWERS 3.

Thus, the buyer cannot yet rescind on grounds that title is unmarketable. (A) is incorrect because
the neighbor’s easement has not been extinguished. An easement can be extinguished where the
owner of the privilege demonstrates by physical action an intention to permanently abandon the
easement. Mere nonuse is not sufficient to terminate an easement, unless the nonuse is combined
with other evidence of intent to abandon it. Here, the fact that the neighbor did not use the
easement for a year because she was out of the country does not establish her intent to abandon
the easement. (C) is incorrect because, although the law implies in every land sale contract a
covenant that title will be marketable, the seller has until the time of delivery to cure the defect.
(D) is incorrect because the deed has not yet been delivered. The covenant against encumbrances
is a covenant contained in a general warranty deed which assures that there are neither visible
encumbrances (e.g., easements) nor invisible encumbrances (e.g., mortgages) against the title or
interest conveyed. This covenant is breached, if at all, at the time of conveyance. Here, the deed
has not yet been delivered, and thus this covenant has not yet been breached.

Answer to Question 6

(B) If the accountant is granted specific performance, it will be because he was prepared to deliver
marketable title to the buyer at closing. The seller of real property need not have marketable title
until the closing; i.e., the buyer cannot rescind prior to that date on grounds that the seller’s title
is not marketable. In fact, the seller has the right to satisfy and eliminate title defects, such as a
mortgage, at the closing with the proceeds from the sale. As long as the purchase price is suffi-
cient and the mortgage is paid simultaneously with the transfer of title (e.g., by use of an escrow),
the buyer cannot claim that the title is unmarketable. The closing will result in marketable title. In
this case, the accountant intended to pay off the $30,000 mortgage at closing with a portion of the
$200,000 purchase price. Thus, the buyer cannot claim that the accountant’s title is unmarketable;
she must proceed to closing. (A) is wrong because any mortgage or lien constitutes an encumbrance
on the property. An encumbrance is any interest in land that diminishes its value. (C) is wrong
because the fact that the mortgage was given for a preexisting debt does not mean it is unenforce-
able. The only possible negative effect of granting the mortgage for an antecedent debt is that
the mortgagee is not considered a purchaser for value and, thus, is not protected by the recording
statute. In this case, however, the recording statute does not even come into play. Even if it did, the
buyer had notice of and could not cut off the bank’s interest. The bank’s loss of protection would
only be a problem as against prior (of which there are none), rather than subsequent, grantees.
Furthermore, it is not clear from the facts that the mortgage was granted merely in exchange for an
antecedent debt. The bank agreed to forgo the immediate payment of the debt in exchange for the
mortgage on the property. (D) is wrong because all mortgages follow the transfer of land. Had the
mortgage not been paid off and the sale proceeded, the buyer would not have been personally liable
on the mortgage, but the property would remain subject to foreclosure for default.

Answer to Question 7

(C) Good title cannot be obtained without the daughter’s son’s inclusion in the conveyance. All
contracts for the sale of land contain, unless the contract expressly provides otherwise, an implied
covenant by the seller that she will deliver to the buyer a marketable title at the date of closing.
Private restrictions or encumbrances, including executory interests and possibilities of reverter,
will render title unmarketable unless the holders of those interests join in the transaction. Here,
the Girl Scouts have a valid executory interest that does not violate the Rule Against Perpetu-
ities because their interest must vest, if at all, during the niece’s lifetime. However, the niece’s
fee simple determinable is not limited in duration like the interest of the Girl Scouts. Thus, the
niece’s heirs will only have a fee simple determinable rather than a fee simple absolute. Because
4. REAL PROPERTY - LAND SALE CONTRACTS ANSWERS

the Girl Scouts’ interest vanishes on the niece’s death, and no other provision was made for the
property if it should thereafter be used for noncamping or nonrecreational purposes, the transferor
(the landowner) retained a possibility of reverter. This interest passed to the daughter and then
to the daughter’s son, and so the son’s interest must be included in the conveyance. (A) is wrong
because the buyer has contracted to receive good title to a fee simple absolute without any restric-
tions on the use of the property. Unless the son joins in the contract to convey his possibility of
reverter, good title cannot be conveyed. (B) is wrong because the son’s interest is the one blocking
good title; the Girl Scouts have already agreed to sell their interest. Furthermore, their interest is
not a right of entry but a shifting executory interest that would automatically divest the buyer’s
interest if he violated the use restriction, and so their promise would be meaningless. (D) is wrong
because the niece, the Girl Scouts, and the daughter’s son together can validly convey a fee simple
absolute to the buyer.
Answer to Question 8
(A) The buyer will not prevail in a suit for specific performance of the contract conveying the
additional strip of land because the court will reform the contract. Reformation is the remedy
whereby the writing setting forth the agreement between the parties is changed to make it
conform to the original intent of the parties. Reformation may be available where there is a
mutual mistake (i.e., the writing does not conform to the original agreement and the parties are
not aware of the discrepancy). As long as the parties were in agreement as to the terms before
the contract was reduced to writing, reformation can be had regardless of whether both parties
signed the contract without noticing the deviation from the oral agreement or one party knew of
the deviation and the other did not. Here, both parties intended that the stone fence constitute the
eastern boundary of the buyer’s parcel. There was no mistake in the oral agreement, merely in
putting it in writing. Thus, the court will reform the contract so as to indicate the stone fence as
the eastern boundary of the buyer’s parcel. (B) is incorrect because there is no mistake or incon-
sistency in the written description (e.g., two different measurements) that would warrant applying
the rules of construction to carry out the parties’ probable intent. Here, the parties’ intent that the
stone fence serve as the eastern border of the buyer’s parcel is clear. (C) is incorrect because if the
contract does not express what the parties agreed to, the court will reform it to conform to their
original intent. (D) is incorrect because the general rule is that parol evidence is admissible to
explain or supplement a written description or to clear up an ambiguity.
Answer to Question 9
(A) The court should order specific performance by the buyer. Because land is considered unique,
specific performance is always appropriate for the enforcement of a valid land sale contract. This
option is available to either the buyer or the seller. The contract gave the seller the option of using
the liquidated damages provision if the seller wished, but the seller has sued for specific perfor-
mance. Because that is appropriate, it will be granted. It is not of significance that the house
burned to the ground. When a contract for the sale of land is signed, equitable conversion takes
place and it is, for all practical purposes, the buyer’s land and the buyer’s risk. Here, the risk of
loss shifted to the buyer upon the signing of the contract. (B) is incorrect because frustration of
purpose is not applicable. The doctrine of equitable conversion shifted the risk of loss to the buyer
when the contract of sale was signed. The fact that an improvement on the property (the house) is
no longer present is not relevant to the grant of specific performance. (C) is incorrect. Based upon
these facts, the seller could request as a remedy either specific performance or liquidated damages
of $6,000. Because the seller elected the specific performance remedy, the court will not award
liquidated damages. (D) is also incorrect because there are no facts which would indicate mutual
mistake. The burning down of the house would not qualify.
REAL PROPERTY - LAND SALE CONTRACTS ANSWERS 5.

Answer to Question 10

(D) The buyer will lose because the race-notice recording statute protects the developer. The devel-
oper paid a fair price for the land and had no knowledge of the buyer’s claim to the land at the
time he purchased the property. He would thus qualify as a bona fide purchaser for value and,
because he was the first to record, he would have priority over the buyer. (A) and (B) are incorrect
because the recording act determines priority among purchasers of property while the Statute of
Frauds deals only with the validity of an individual contract. (C) is incorrect because the seller’s
signing the contract, which contained the same terms and conditions as the buyer’s offer, consti-
tuted an acceptance, which became effective upon dispatch under the “mailbox rule” of contract
law.

Answer to Question 11

(A) The niece may specifically enforce the agreement. Under the doctrine of equitable conversion, if
the buyer dies, the takers of her real property can demand a conveyance of the land at the closing
of the contract. (B) is wrong because the death of either the seller or the buyer does not render
the agreement cancellable at the will of either party. (C) is wrong because a real estate contract
survives the death of either party unless the agreement itself provides otherwise. (D) is wrong
because marketable title is title reasonably free from doubt. Generally, this involves either defects
in the chain of title or encumbrances that might present an unreasonable risk of litigation. Such
problems are not present in these facts.

Answer to Question 12

(B) If the factory violates the setback allowance in the zoning ordinance, the seller cannot success-
fully sue for specific performance because title is unmarketable. There is an implied covenant in
every land sale contract that at closing the seller will provide the buyer with marketable title, i.e.,
title reasonably free from doubt. Generally, zoning restrictions do not affect the marketability of
title, but an existing violation of a zoning ordinance does render title unmarketable. (A) is incor-
rect because it is generally held that the violation of subdivision, housing, or building codes does
not constitute an encumbrance on title. Zoning is treated differently. (C) is incorrect because only
an existing violation of the zoning ordinance, not a potential one, will render title unmarketable.
Moreover, the truck restriction will not make the manufacturer’s intended use impossible (she can
use different trucks); thus, that contract argument also fails. (D) is incorrect because although an
easement is an encumbrance that can render title unmarketable, this easement does not encumber
the factory, but rather the adjoining property.

Answer to Question 13

(A) The lessee’s option to purchase renders the seller’s title unmarketable. Ordinarily, an option of this
duration would violate the Rule Against Perpetuities and be stricken. An interest violates the Rule
Against Perpetuities if there is any possibility, however remote, that it will vest more than 21 years
after some life in being at the creation of the interest. Here, the relevant measuring life would
be the lessee. The lessee could die within the next three years, and the option could be exercised
by his successors more than 21 years after his death. There is a special exception to the Rule,
however, for options to purchase attached to leaseholds. Because the one who holds the option in
this case is the current lessee, the Rule does not apply. Thus, the option is valid, and it renders the
seller’s title unmarketable. (B) is incorrect because only a significant encroachment will render
title unmarketable. A one-half-inch encroachment on airspace would not be considered signifi-
6. REAL PROPERTY - LAND SALE CONTRACTS ANSWERS

cant, particularly because it does not interfere with the use of the adjoining property. Thus, this
encroachment will not affect marketability. (C) is incorrect because a lien on property will not
render title unmarketable if the seller pays the lien at closing. Unless the contract provides other-
wise, the seller need not provide marketable title until closing. A seller has the right to satisfy a
lien at the closing with the proceeds of the sale. Therefore, as long as the purchase price is suffi-
cient and the lien is satisfied simultaneously with the transfer of title (e.g., by using escrows), the
buyer cannot claim that the title is unmarketable. The closing will result in marketable title. In
this situation, the $75,000 purchase price is clearly sufficient to satisfy the $5,000 lien. Thus, the
seller may satisfy the lien at the closing and convey marketable title to the buyer. (D) is incorrect
because although an easement that reduces the value of the property renders title unmarketable,
this easement has been terminated. An easement by necessity terminates as soon as the necessity
ends. Because the main road now provides access to the neighbor’s land, she no longer needs the
easement over the seller’s land and it is extinguished.

Answer to Question 14

(B) The purchaser prevails because the son’s acquisition of the land by adverse possession does not
satisfy his implied covenant to deliver a marketable title. Unless expressly provided otherwise,
all contracts for the sale of land contain an implied covenant by the seller that he will deliver to
the buyer at closing a marketable title. Title is marketable if a reasonably prudent buyer, ready
and able to purchase, would accept it in the exercise of ordinary prudence. Generally, inability
to establish a record chain of title will render title unmarketable. If a seller attempts to rely
on adverse possession to establish marketable title, many courts will hold that such title is not
marketable until the adverse possessor has perfected it by a judgment quieting title. In other
words, the buyer is not required to “buy a lawsuit.” Other states require only that the seller
provide written evidence or some other proof that the buyer can use in court to defend any lawsuit
challenging title. Even under the latter approach, it does not appear that the son can provide
enough evidence to make his title marketable. Thus, although the son has acquired title to the land
by occupying it in an open, notorious, hostile, and continuous manner, for a period exceeding that
prescribed by statute, the fact that he has acquired title only by adverse possession renders his title
unmarketable. Because the son has thus breached his covenant of marketable title, implied in the
sale contract, his action to specifically enforce that contract will fail. (A) is incorrect because the
son does own the two acres he is trying to sell. The son’s fencing of the land and building struc-
tures on it qualifies as open and notorious possession because it is such as the usual owner would
make of the land and puts the true owner or the community on notice of the fact of possession.
His possession was continuous for more than the five-year statutory period, and it was hostile
because it was without the permission of the true owner. Under the majority view, the son’s good
faith belief that he was possessing the land described by the mother’s deed is irrelevant. Thus,
the son acquired title by adverse possession. However, as explained above, such title, as far as
the purchaser is concerned, is not marketable. (C) is incorrect because the type of deed by which
title is transferred does not affect the seller’s covenant of marketable title, which is implied in
the contract. It is true that the implied covenant of marketability is no longer assertable once a
deed has been delivered (absent fraud or mistake), so that if the son delivered a quitclaim deed
(i.e., without making any assertions relative to the title being transferred), the purchaser could no
longer assert the implied contractual covenant of marketable title. However, no deed has yet been
delivered; the covenants under the contract are still in effect. (D) is incorrect because the fact that
a contract describes land with sufficient specificity does not establish the marketability of title to
that land.
REAL PROPERTY -
MORTGAGES QUESTIONS
REAL PROPERTY - MORTGAGES QUESTIONS 1.

REAL PROPERTY - MORTGAGES QUESTIONS

Question 1 Question 2

A developer owned an office building An architectural historian bought a house,


subject to a first mortgage with a creditor in the financing $150,000 of the purchase price with a
amount of $1 million. Subsequently, the devel- loan from a bank, secured by a mortgage on the
oper borrowed $100,000 from a bank secured property. The bank recorded its mortgage. Ten
by a second mortgage on the building to help years later, the historian borrowed $5,000 from
pay the first mortgage and other expenses of a finance company, using the house as security.
the building. The developer’s financial condi- The finance company recorded its mortgage
tion worsened, and he was unable to make the on the property. Five years later, the historian
required payments on the first mortgage to the obtained a $40,000 mortgage from a savings
creditor. The developer approached the creditor and loan association to pay for an addition to the
and offered to give her a deed to the building house. The savings and loan association did not
in satisfaction of all of his obligations to the record its mortgage. Subsequently, the historian
creditor. The developer delivered to the creditor lost her job and was unable to make payments
a quitclaim deed to the building, which recited on either the finance company or the savings
as consideration the release of the developer and loan mortgages. The finance company
from all liability on the mortgage to the creditor. filed foreclosure of its mortgage and joined the
The deed was duly recorded. savings and loan association in the action, and
the house was sold to a buyer at the foreclosure
Shortly thereafter, the office market greatly sale.
improved, and the building was worth $1.5
million. The developer then brought an action After acquiring the property at the sale, what
against the creditor, claiming that the deed was is the buyer’s obligation regarding the bank’s and
an equitable mortgage, and the bank served the savings and loan association’s mortgages?
notice on the creditor that it was preparing to
foreclose its mortgage on the building. (A) The buyer takes the property subject to
both mortgages.
Against which parties will the creditor
prevail? (B) The buyer takes the property subject to
neither mortgage.
(A) The developer only.
(C) The buyer takes the property subject to the
(B) The bank only. savings and loan association’s mortgage, but
not subject to the bank’s mortgage.
(C) Both the developer and the bank.
(D) The buyer takes the property subject to
(D) Neither the developer nor the bank. the bank’s mortgage, but not subject to the
savings and loan association’s mortgage.
2. REAL PROPERTY - MORTGAGES QUESTIONS

Question 3 deed from the owner to the farmer was promptly


and properly recorded, but due to an oversight
A landowner needed money. A neighbor the mortgage from the financing company was
agreed to loan him the money he needed if the not immediately recorded. A few months later,
landowner would give adequate security for the the farmer approached the financing company
loan. The landowner made out a note payable about getting a second mortgage. The financing
to the neighbor and secured it with a first trust company turned him down, so he contacted a
deed on his house, which the landowner owned bank. Not having knowledge of the previous
free and clear of monetary encumbrances. The mortgage on the property, the bank agreed to
neighbor recorded the trust deed. Subsequently, loan the farmer $300,000 secured by a mortgage
the landowner sold his house to a buyer, who on the land, which it promptly and properly
took the house subject to the mortgage. A few recorded. One day later, the financing company,
months later, the buyer needed money to build a having discovered that its original mortgage had
swimming pool and borrowed it from a lender, not been recorded, properly recorded it.
securing the loan with a second trust deed on the
house. This trust deed was recorded. The jurisdiction’s recording statute provides:
“Any conveyance or mortgage of an interest in
Several years later, the buyer got into finan- land, other than a lease for less than a year, shall
cial trouble and was unable to make payments not be valid against any subsequent purchaser for
to either the neighbor or the lender on the notes value, without notice thereof, whose conveyance
they held. Both the neighbor and the lender is first recorded.”
commenced foreclosure proceedings. To protect
his credit, the landowner paid the neighbor, and The farmer struggled to keep up with his
the neighbor assigned him the note and first trust mortgage payments, and finally stopped making
deed. The landowner recorded the assignment of payments altogether on both mortgages. The
the trust deed. bank began foreclosure proceedings, but did not
include the financing company as a party. At
Which of the following arguments offers the
the foreclosure sale, a buyer purchased the land,
LEAST support for the landowner’s position that
having no actual knowledge of the mortgage
his interest has the first priority?
with the financing company. Soon after, the
(A) The landowner is subrogated to the rights financing company declared its loan in default
of the neighbor to the extent of the land- and sought to foreclose on the land.
owner’s payment to the neighbor.
May the financing company foreclose against
(B) The landowner, as assignee, may enforce the buyer?
the note and trust deed that were owned by
the neighbor. (A) Yes, because the holder of a senior mort-
gage interest is unaffected by foreclosure
(C) If the landowner were denied a first priority, of a junior interest.
the buyer or the lender would be unjustly
enriched. (B) Yes, because the holder of a junior
mortgage interest is a necessary party
(D) Even though the landowner was the maker that must be included in a foreclosure
of the note, the landowner’s payment to the proceeding by the holder of a senior
neighbor did not discharge the note. interest.
Question 4 (C) No, because its failure to promptly record
extinguished its rights against all parties
The owner in fee simple of a tract of land except the farmer, the original mortgagor.
sold it to a farmer for $850,000. To finance the
purchase, the farmer obtained a mortgage loan (D) No, because the buyer succeeds to the
from a financing company for $600,000. The farmer’s right of redemption.
REAL PROPERTY - MORTGAGES QUESTIONS 3.

Question 5 (C) The bank’s mortgage is extinguished


because when it was modified it became
A chef purchased a restaurant for $100,000. junior to the financing company’s mortgage.
As part of his financing, he obtained a purchase
money mortgage from a bank for $60,000. Due (D) The bank’s mortgage is extinguished
to a clerical error by the bank, the mortgage was regardless of the modification because
not recorded in the county recorder’s office. it had not recorded before the financing
company obtained its mortgage interest.
A statute in the jurisdiction provides: “No
conveyance of an interest in land, other than Question 6
a lease for less than one year, shall be valid
against any subsequent purchaser for value, A merchant had a serious cash flow problem
without notice thereof, whose conveyance is first and needed cash to buy the inventory required to
recorded.” fill orders. A friend offered to loan the merchant
$50,000 if he would put up adequate collateral
After the chef’s restaurant had been in opera- to assure her that she would not lose her money.
tion for five years, business dropped dramati- To guarantee the loan, the merchant gave the
cally. To stay in business, the chef obtained a friend a deed to a property worth $100,000 that
mortgage from a financing company for $30,000. he had inherited. The friend recorded the deed.
The financing company was not informed by The friend gave the merchant $50,000, and the
the chef of the mortgage held by the bank. The merchant signed a promissory note agreeing
next day, the chef contacted the bank about to repay the $50,000 within eight months. The
renegotiating its mortgage. Checking its records, merchant continued to occupy the property, and
the bank discovered that the original mortgage the friend agreed to reconvey the property to the
was not recorded and immediately recorded it. merchant as soon as he repaid the $50,000. The
Later that day, the financing company recorded merchant was unable to pay the friend when the
its mortgage. A few days later, the chef and the note came due. He asked the friend for an exten-
bank agreed to a modification of their mortgage sion, but she refused.
agreement to allow the chef to make lower
monthly payments in exchange for a higher If the friend seeks to take possession of the
interest rate and a longer period of repayment. property, may she do so?

Despite this agreement, the chef was unable (A) Yes, because she has clear record title.
to make payments on the financing company
mortgage. The financing company instituted a (B) Yes, but only if she institutes foreclosure
foreclosure action six months later, but failed proceedings and is the successful purchaser
to include the bank as a party to the foreclosure at the foreclosure sale.
action.
(C) No, because $50,000 does not reflect the
If the financing company takes title to the fair market value of the property.
restaurant at the foreclosure sale, which of the
following statements most correctly describes (D) No, the friend is limited to a contract claim
the bank’s interest? against the merchant because the law does
not recognize this type of security agree-
(A) The bank’s mortgage on the restaurant sur- ment.
vives under its original terms.

(B) The bank’s mortgage on the restaurant


survives under its modified terms.
4. REAL PROPERTY - MORTGAGES QUESTIONS

Question 7 Question 8

A bank sold one of its mortgages and the A landlord owned an apartment building.
accompanying note to a finance company. Needing money to make some repairs and
Shortly thereafter, the finance company sold improvements, he went to a bank and applied for
both the mortgage and the note to a brokerage a loan. The bank loaned the landlord $100,000,
firm. The brokerage firm duly recorded the secured by a mortgage on the apartment
assignment in the offices of the county recorder building. Last month, the landlord defaulted
of deeds, as prescribed by state statute. However, on his mortgage payments. The bank instituted
the brokerage firm decided to use the finance foreclosure proceedings and wishes to take
company as its collection agent for the payments possession of the apartment building in order to
as they came due. Therefore, the brokerage firm collect the rents from the property.
left the mortgage and note documents in the
hands of the finance company. Does the bank have the right to take posses-
sion of the apartment building before foreclo-
The finance company developed cash-flow sure?
and liquidity problems. To try to save the finance
company from bankruptcy, its president sold the (A) Yes, if the jurisdiction follows the lien
mortgage and accompanying note to an investor. theory.
This transaction was not enough to save the
finance company from insolvency. During the (B) Yes, if the jurisdiction follows the title
winding up of the finance company’s affairs, the theory.
brokerage firm discovered the finance company’s
sale of the mortgage and note to the investor. (C) No, because the landlord has a right to
The brokerage firm also learned that the finance redeem the property before the foreclosure
company never told the investor about the sale.
brokerage firm’s interests in the mortgage and
note. The brokerage firm files suit against the (D) No, unless the landlord consents to the
investor for the return of the mortgage and note. possession.

What will the court likely decide?

(A) The brokerage firm owns both the mort-


gage and the note.

(B) The investor owns both the mortgage and


the note.

(C) The brokerage firm owns the mortgage, but


the investor owns the note.

(D) The investor owns the mortgage, but the


brokerage firm owns the note.
REAL PROPERTY - MORTGAGES QUESTIONS 5.

Question 9 Question 10

An entrepreneur purchased several acres A real estate investor purchased a home by


of scrub-covered land of little apparent value. taking out a $160,000 mortgage. After a few
Shortly thereafter, an international conglomerate months, he sold the home to a buyer. According
announced plans to develop a theme park on a to the terms of the agreement signed by the
tract immediately adjacent to the entrepreneur’s parties, the buyer took the house “subject to the
scrubland. The plan caused real estate values mortgage” and agreed to “assume payment of”
in the surrounding area to skyrocket. Rather the mortgage. A recession struck the area and
than resell his land for a substantial profit, the the buyer found himself in financial difficul-
entrepreneur decided to build a variety of tourist- ties. The buyer sold the home to a company
oriented facilities on the land. To finance his that buys otherwise unsellable houses. Under
project, the entrepreneur obtained a loan from a the terms of the agreement signed by the
bank in exchange for a $20,000 mortgage on his parties, the company agreed to take “subject
land. The bank promptly recorded the mortgage. to the mortgage.” All deeds and the mortgage
A few days later, the entrepreneur went to a were properly recorded. After two months, the
credit union and took out a $15,000 mortgage company ceased making mortgage payments.
on the land. The credit union knew of the bank’s The bank that held the mortgage unsuccessfully
mortgage, and the credit union promptly recorded demanded payments from the company, the
its own mortgage. A few weeks after that, the buyer, and the investor. The bank properly insti-
entrepreneur went back to the bank and, after tuted proceedings to foreclose, and the house
full disclosure of the credit union’s mortgage, was put up for judicial sale. Because the reces-
obtained another advance of $15,000 from the sion had severely depressed property values, the
original bank mortgage, increasing the amount house sold for only $120,000. After the $120,000
borrowed against the mortgage from $20,000 to was applied to the mortgage, $37,800 was still
$35,000. The bank promptly recorded the change. owing on the principal amount.
After spending much of these funds on retainers
for architects, builders, and attorneys, the entre- From whom can the bank seek judgment to
preneur was upset to learn that the county council cover the deficiency?
summarily rejected the theme park proposal. The
entrepreneur made no more mortgage payments (A) The real estate investor only.
to the credit union, but continued to make
payments to the bank. The credit union brought (B) The real estate investor and the buyer.
a foreclosure action against the entrepreneur and
included the bank as a party. The value of the (C) The buyer and the company.
land in the area had plummeted, so the proceeds
at the foreclosure sale were just $18,000 after (D) The real estate investor, the buyer and the
attorneys’ fees and court costs. company.

How should the proceeds be divided?


(A) $18,000 to the bank and nothing to the
credit union.
(B) $9,000 to the bank and $9,000 to the credit
union.
(C) $3,000 to the bank and $15,000 to the
credit union.
(D) Nothing to the bank, $15,000 to the credit
union, and $3,000 to the entrepreneur.
6. REAL PROPERTY - MORTGAGES QUESTIONS

Question 11 Question 12

An investor financed the purchase of a lot A woman arranged with a bank to take out a
through a mortgage on the property held by a loan for $30,000, secured with a mortgage on
bank. The mortgage contained a “due-on-sale her home. On June 3, the woman executed the
clause,” requiring the mortgagor to pay off the note and the mortgage, and the bank gave her
mortgage when the property is sold. The investor a certified check for $30,000. On June 4, the
made all of his mortgage payments in a timely woman sold her home to a wealthy buyer for
manner. Five years later, he sold the property to $150,000 in cash. The buyer knew nothing about
a buyer. The investor told the buyer that there the mortgage. On June 5, the buyer recorded her
was a mortgage on the property but he did not deed to the property. Two hours after the buyer
mention the due-on-sale clause. The buyer paid recorded, the woman fled the country.
the investor the asking price for the property.
The investor pocketed the proceeds without On the evening of June 8, which was a
paying off the bank mortgage. Saturday, the buyer presented her niece with a
deed to the property as a gift. At 10 a.m. on June
Which of the following best states the effect 10, the bank recorded its mortgage. At 2 p.m.
of the due-on-sale clause on the buyer’s interest on June 10, the niece recorded her deed. After
in the property? the woman missed her first mortgage payment
on July 1, the bank employees went to the title
(A) The bank can foreclose on the property. office. They discovered the deeds to the buyer
and her niece. The bank demanded that the niece
(B) The buyer must agree to assume the satisfy the $30,000 mortgage. The niece filed an
mortgage on the property. appropriate suit to determine the various inter-
ests in the property.
(C) The only effect of the clause is that the
buyer is personally liable on the mortgage. The recording statute in the jurisdiction reads,
in relevant part: “A conveyance of an estate in
(D) The clause has no effect, because due-on- land shall not be valid against any subsequent
sale clauses are generally void as contrary purchaser for value, without notice thereof,
to public policy. unless the conveyance is recorded.”

How should the court rule?

(A) The niece owns the property subject to the


bank’s mortgage, because the niece is a
donee.

(B) The niece owns the property subject to


the bank’s mortgage, because the bank
recorded before the niece.

(C) The niece owns the property free of the


bank’s mortgage, because the buyer was
a bona fide purchaser for value without
notice.

(D) The niece owns the property free of the


bank’s mortgage, because the bank does not
qualify as a bona fide purchaser for value.
REAL PROPERTY - MORTGAGES QUESTIONS 7.

Question 13 by a mortgage on his land, which the bank


promptly and properly recorded. A few months
A buyer bought a home from a real estate later, the owner obtained another loan of $60,000
developer for $700,000. The buyer paid from a lender, in exchange for a promissory note
$100,000 of the purchase price herself. The secured by a mortgage on the land, which the
buyer’s employer provided $100,000 of the lender promptly and properly recorded. Subse-
purchase price by giving the buyer a loan quently, the owner sold the land to a buyer for
and taking a mortgage. The developer loaned $150,000 and conveyed a warranty deed. The
$500,000 to the buyer to finance the remainder buyer expressly agreed with the owner to assume
of the purchase price, and in return took a both mortgages, with the consent of the bank and
mortgage on the property. One week later, a the lender. A few years later, the bank loaned the
bank obtained a judgment against the buyer buyer an additional $50,000 in exchange for an
for a delinquent credit card balance. The bank increase in the interest rate and principal amount
properly recorded its judgment as a lien against of its mortgage on the land. At that time, the
the property. Another month after that, the buyer balance on the original loan from the bank was
incurred some extraordinary medical expenses, $50,000. Shortly thereafter, the buyer stopped
and asked the employer for another $100,000, making payments on both mortgages and disap-
which the employer provided and added onto the peared. After proper notice to all appropriate
principal balance the buyer owed on the loan. parties, the bank instituted a foreclosure action
Finally, six months later, the buyer asked the on its mortgage, and purchased the property at
developer to change the terms of the loan, so the foreclosure sale. At that time the principal
that the buyer would have more time to pay. The balance on the lender’s mortgage loan was
developer and the buyer agreed that the buyer $50,000. After fees and expenses, the proceeds
could have an additional five years to pay the from the foreclosure sale totaled $80,000.
balance of the loan in exchange for an increase
in the principal of the loan. Shortly thereafter, Assuming that the jurisdiction permits
the buyer lost his job and defaulted on all of his deficiency judgments, which of the following
payments. The employer brought an action to statements is most accurate?
foreclose its mortgage. All mortgages and liens
were promptly and properly recorded. (A) The bank keeps the entire $80,000 and can
proceed personally against the owner for
Regarding the distribution of the proceeds of its deficiency, while the lender’s mortgage
an eventual sheriff’s sale of the property, which remains on the land.
of the following statements is true?
(B) The bank keeps the entire $80,000, the
(A) The bank is paid in full before the devel- lender’s mortgage on the land is extin-
oper is paid in full. guished, and both the bank and the lender
can proceed personally against the owner
(B) The employer is paid in full before the bank for their deficiencies.
receives any proceeds.
(C) The bank keeps $50,000, the lender is
(C) The developer is paid in full before the entitled to $30,000, and only the lender can
employer receives any proceeds. proceed personally against the owner for its
deficiency.
(D) The developer is paid in full before the
employer is paid in full. (D) The bank keeps $50,000, the lender is
Question 14 entitled to $30,000, and neither the bank
nor the lender can proceed personally
An owner obtained a loan of $60,000 from a against the owner for their deficiencies.
bank in exchange for a promissory note secured
REAL PROPERTY -
MORTGAGES ANSWERS
REAL PROPERTY - MORTGAGES ANSWERS 1.

REAL PROPERTY - MORTGAGES ANSWERS

Answer to Question 1

(A) The creditor will be successful against the developer only. The developer initiated an agreement
with the creditor to convey the property to the creditor in satisfaction of the developer’s obligation
to the creditor. The deed was meant to convey absolute title to the creditor, and was not intended
to be another form of the mortgage that already existed. Therefore, the developer has no further
interest in the property. (B) is incorrect. The creditor will not prevail over the bank. The convey-
ance from the developer to the creditor discharged the first mortgage on the property because
the conveyance was in satisfaction of the mortgage obligation. The first mortgage, however, was
never foreclosed and therefore did not wipe out the second mortgage. When the first mortgage was
discharged by the conveyance, the second mortgage became a first mortgage and is still a valid
encumbrance on the property. (C) and (D) are incorrect because the creditor will prevail against
the developer but not against the bank.

Answer to Question 2

(D) The buyer takes subject to the bank’s mortgage. A foreclosure sale wipes out all junior mortgages
(those that came later in time than the mortgage that was foreclosed) but does not wipe out
senior mortgages (those that came earlier). Because the bank’s mortgage preceded the finance
company’s, it is senior and is not wiped out. The buyer takes subject to this mortgage. Thus, (B)
is wrong. Although the buyer is not personally liable on this debt (he did not sign the note, the
historian did), he must pay the mortgage or face foreclosure by the bank. Because the savings and
loan association’s mortgage came later than the finance company’s, it is junior and is wiped out.
Thus, (A) and (C) are wrong. If, after paying the cost of the foreclosure and paying off the finance
company’s mortgage, there is money left over from the sale, it will first go to paying off the
savings and loan association’s mortgage. But regardless of whether the savings and loan associa-
tion is paid off or not, the buyer takes completely free of this obligation.

Answer to Question 3

(D) The least support for the landowner’s position that his interest has the first priority is the argument
that his payment to the neighbor did not discharge the note. The note is considered to embody
the security obligations, and payment of the note by the maker discharges the note. The best way
to get the correct answer is to use a process of elimination. (A) is wrong because it supports the
landowner’s position: The purchaser of a note and trust deed is subrogated to (i.e., takes over) the
interest of the seller, thus giving the landowner the neighbor’s priority. (B) is wrong because it is
saying essentially the same thing as (A). (C) is wrong because it is the policy reason behind the
rules stated in (A) and (B). Thus, (A), (B), and (C) are wrong because they do offer support for
the landowner’s priority position, while the argument in (D) would not support the landowner’s
position.

Answer to Question 4

(B) The financing company may foreclose against the buyer because it was not included as a party
in the foreclosure proceeding brought by the bank. Generally, the priority of a mortgage is deter-
mined by the time it was placed on the property. When a mortgage is foreclosed, the buyer at the
sale will take title as it existed when the mortgage was placed on the property. Thus, foreclosure
will terminate interests junior to the mortgage being foreclosed but will not affect senior interests.
2. REAL PROPERTY - MORTGAGES ANSWERS

However, the junior mortgagee has the right to pay the senior interest off (i.e., redeem it) in order
to avoid being wiped out by its foreclosure. Thus, those with interests subordinate to those of
the foreclosing party are necessary parties to the foreclosure action. Failure to include a neces-
sary party results in the preservation of that party’s interest despite foreclosure and sale. Here,
even though the financing company created its mortgage first, its interest was junior to the bank’s
interest by virtue of the recording statute. Hence, when the bank brought its foreclosure action, it
should have included the financing company as a necessary party. Because the financing company
was not included, its mortgage interest on the land is preserved and it may bring a foreclosure
action. (A) is incorrect because the financing company did not hold the senior mortgage interest
at the time of the foreclosure by the bank. Mortgagees for value such as the bank are treated as
“purchasers” under recording statutes. By failing to record its mortgage before the bank executed
and recorded its mortgage for value, the financing company became the junior mortgagee under
the jurisdiction’s race-notice recording statute. Thus, had it been made a party to the foreclosure
proceeding by the bank, its interest would have been wiped out. (C) is incorrect because the
financing company’s delay in recording only affected its rights vis-à-vis the bank. By the time of
the foreclosure sale, the financing company’s mortgage interest was recorded; hence, the buyer
will take subject to it because he had constructive (record) notice of it. (D) is incorrect because
the fact that the buyer obtained the right to redeem the property when he purchased it does not
preclude the financing company from beginning foreclosure proceedings. To redeem the property,
the buyer must pay off the financing company’s mortgage prior to the date set for the foreclosure
sale.

Answer to Question 5

(B) The bank’s mortgage survives under its modified terms because the financing company did not
include the bank in the foreclosure action. The general rule is that when a mortgage is foreclosed,
the buyer at the sale will take title as it existed when the mortgage was placed on the property.
Thus, foreclosure generally will destroy all interests junior to the mortgage being foreclosed,
but will not discharge senior interests. However, those with interests subordinate to those of the
foreclosing party are necessary parties to the foreclosure action. Failure to include a necessary
party results in the preservation of that party’s interest despite foreclosure and sale. Here, the
bank’s original mortgage was senior to the financing company’s mortgage. However, where the
landowner enters into a modification agreement with the senior mortgagee, raising its interest rate
or otherwise making it more burdensome, the junior mortgagee will be given priority over the
modification. Thus, the bank’s modification would not have priority over the financing company’s
mortgage. Nevertheless, because the financing company failed to include the bank in its foreclo-
sure action, the bank’s mortgage interest survives under its modified terms, even though the
modification did not have priority. (B) is therefore correct and (A) is incorrect. (C) is incorrect
because the modification of a senior mortgage does not nullify its original senior status; it only
means that the junior mortgage will be given priority over the modification. Because the buyer
at the foreclosure sale ordinarily will take title as it existed when the mortgage was placed on the
property, the senior mortgage ordinarily survives in its original form. (As noted above, here the
mortgage survives in its modified form despite its junior status because of the failure to include
the bank in the foreclosure action.) (D) is incorrect because the recording statute applicable here
is a race-notice statute rather than a notice statute. Under a race-notice statute, a subsequent bona
fide purchaser (including a mortgagee) is protected only if it records before the prior grantee
or mortgagee. Here, the bank recorded its mortgage before the financing company recorded its
mortgage. The fact that the financing company had no notice of the bank’s interest at the time it
granted the mortgage on the restaurant does not help the financing company because it did not
record first.
REAL PROPERTY - MORTGAGES ANSWERS 3.

Answer to Question 6

(B) The friend may take possession of the property only if she institutes foreclosure proceedings
and is the successful purchaser at the foreclosure sale. The friend must institute foreclosure
proceedings because the deed is, in reality, an equitable mortgage. A landowner needing to raise
money may “sell” the land to a person who pays cash and may give the lender an absolute deed
rather than a mortgage. However, if a court concludes that the deed was really given for security
purposes, it will treat it as an equitable mortgage and require that the creditor foreclose it by
judicial action, like any other mortgage. The following factors indicate an equitable mortgage: (i)
the existence of a debt or promise of payment by the deed’s grantor; (ii) the grantee’s promise to
return the land if the debt is paid; (iii) the fact that the amount advanced to the grantor/debtor was
much lower than the value of the property; (iv) the degree of the grantor’s financial distress; and
(v) the parties’ prior negotiations. Here, the merchant has agreed by his promissory note to repay
the friend the amount she purportedly paid to purchase the property, and the friend has promised
to return the property upon repayment. The sum of money given by the friend to the merchant is
only one-half of the value of the property. In addition, the merchant was in fairly serious finan-
cial condition at the time of his dealings with the friend. Finally, the discussions between the
merchant and the friend prior to the exchange of money and the deed indicate quite clearly that
the parties intended that the property serve as security for the friend’s loan to the merchant. These
facts give rise to the conclusion that the deed, absolute on its face, is really an equitable mortgage,
thus requiring the friend to foreclose it by judicial action. At a foreclosure sale, the highest bidder
takes the property. Therefore, to take possession of the property, the friend must not only institute
foreclosure proceedings but also be the successful bidder at the sale. (A) is incorrect because,
pursuant to the foregoing analysis, the deed given to the friend will not be dealt with by the
court as an absolute deed conveying title, but rather as an equitable mortgage giving the friend a
security interest in the property. Consequently, the friend does not have clear record title. (C) is
incorrect because, while the fact that the friend gave the merchant a sum of money lower than the
market value of the property will be considered as one of several factors indicating that this is an
equitable mortgage situation, it will not, by itself, preclude the friend’s taking possession of the
property. If this were a situation involving a true absolute conveyance of the property, the convey-
ance would not be set aside simply because the purchase price is less than the market value. No
consideration is necessary for a valid deed. (D) is incorrect because, as discussed above, the court
will treat this as an equitable mortgage.

Answer to Question 7

(A) The brokerage firm owns both the mortgage and the note. A mortgage is a security interest in
property and a note is evidence of the underlying debt. Physical possession of the mortgage and
note is not required for ownership. Thus, because the brokerage firm bought the mortgage and
note and recorded its interest, the brokerage firm is the owner of both, even though it left posses-
sion of the documents with the finance company. The investor has no interest in the mortgage and
note because he had record notice of the brokerage firm’s interest (because the brokerage firm
recorded the mortgage). Having notice of the brokerage firm’s interest, the investor cannot claim
the protection of the recording act. Also, the investor cannot claim holder in due course status,
because that status requires no notice of any other claims to the property. Therefore, the investor
has no interest in the mortgage and note. (B) is wrong because, even absent the protection of the
recording act or holder in due course status, the first in time rule governs. The brokerage firm was
the first to purchase the mortgage and note, and so it prevails. (C) and (D) are wrong because, as
stated above, the brokerage firm owns both the mortgage and note, which were never separated.
4. REAL PROPERTY - MORTGAGES ANSWERS

Answer to Question 8

(B) The bank may take possession of the apartment building before foreclosure if the jurisdiction
follows the title theory. Under the title theory, title is in the mortgagee (lender) until the mortgage
has been satisfied or foreclosed. Thus, the mortgagee is entitled to possession on demand at
any time. (A) is incorrect because under the lien theory, title remains in the mortgagor (debtor)
and the mortgagee holds only a security interest in the property. Thus, the mortgagee may not
have possession before foreclosure in these jurisdictions. (C) is incorrect because, although
the mortgagor has the right to redeem the property by paying off the amount due before the
foreclosure sale, the equitable right of redemption does not affect the bank’s right to possession
before foreclosure. (D) is incorrect because, although the mortgagee may take possession if the
mortgagor gives his consent, the bank would be entitled to possession before foreclosure in a title
theory jurisdiction without the landlord’s consent.

Answer to Question 9

(C) The credit union should have its mortgage fully satisfied from the proceeds of the foreclosure sale,
and the remainder should go toward satisfying the bank’s junior interest. As a general rule, the
priority of a mortgage is determined by the time it was placed on the property. When a mortgage
is foreclosed, the buyer at the sale will take title as it existed when the mortgage was placed on the
property. Thus, foreclosure will destroy interests junior to the placing of the mortgage but will not
discharge senior interests, which remain on the property. However, where a landowner enters into
a modification agreement with the senior mortgagee that makes the mortgage more burdensome,
the junior mortgage will be given priority over the modification. In particular, if a junior mortgage
is placed on the property and the senior lender later makes an optional advance while having
notice of the junior lien, the advance will lose priority to the junior lien. (An optional advance
is one that the senior lender is not contractually bound to make.) Here, the senior mortgage of
the bank was modified by the additional advance after creation of the junior mortgage with the
credit union; hence, the increase in the debt to the bank will not have priority over the credit
union’s interest. Therefore, the proceeds of the foreclosure sale will be applied first to satisfy the
$15,000 mortgage of the credit union that is being foreclosed, and the balance of $3,000 will go
toward the $15,000 modification of the bank’s mortgage. This leaves a $12,000 deficiency, for
which the entrepreneur might be liable, and leaves the original $20,000 mortgage of the bank,
which remains on the property in the hands of the foreclosure sale buyer. (A) is wrong because
the bank’s senior interest on the property is not affected by the credit union’s foreclosure action;
since the entrepreneur is not in default on that mortgage, the bank is not entitled to recover
foreclosure proceeds for it. (B) is wrong because the foreclosure proceeds are not divided propor-
tionally between the interests affected by the foreclosure; they are first applied to fully satisfy
the foreclosed mortgage, and whatever is left over is applied toward any junior interests in order
of priority. (D) is wrong because the bank’s additional advance is junior to the credit union’s
mortgage and is wiped out by its foreclosure; hence, the balance of the proceeds after the credit
union’s mortgage is satisfied will be paid to partially satisfy the bank’s junior interest.

Answer to Question 10

(B) The bank can require either the investor or the buyer to pay the deficiency. When a mortgage is
foreclosed and the proceeds of the foreclosure sale are insufficient to satisfy the mortgage debt,
the mortgagee can bring a personal action against the mortgagor for the deficiency. The mortgagor
remains personally liable on the mortgage loan regardless of any subsequent transfers of the
mortgaged property. Hence, the real estate investor remains liable to the bank for the deficiency
REAL PROPERTY - MORTGAGES ANSWERS 5.

because he was the original mortgagor. The buyer is also liable for the deficiency because he
“assumed payment of” the mortgage. When a grantee signs an assumption agreement, promising
to pay the mortgage loan, he becomes primarily liable to the lender (who is a third-party benefi-
ciary of the assumption agreement), while the original mortgagor becomes secondarily liable as a
surety. Thus, the bank can seek judgment against either the real estate investor or the buyer for the
deficiency, making (B) correct and (A) incorrect. (C) and (D) are incorrect because the company
is not liable for the deficiency. When a mortgagor sells property and conveys a deed, the grantee
takes subject to the mortgage, which remains on the land (unless the proceeds of the sale are used
to pay off the mortgage). However, a grantee who does not sign an assumption agreement does
not become personally liable on the loan. Instead, the original mortgagor remains primarily and
personally liable. Here, the company only took “subject to the mortgage,” without assuming it.
Thus, it cannot be required to pay the deficiency.

Answer to Question 11

(A) Unless the buyer pays off the entire loan, the bank can foreclose on the property. When a
mortgagor sells his mortgaged property and gives a deed, the grantee takes subject to the
mortgage, which remains on the land. A mortgage containing a “due-on-sale” clause allows the
lender to demand full payment of the loan if the mortgagor transfers an interest in the property
without the lender’s consent. Under federal law, due-on-sale clauses are generally enforceable.
Here, the bank apparently did not consent to the transfer and the investor did not pay off the
mortgage. The mortgage remained on the property and will allow the bank to foreclose unless
the buyer extinguishes the mortgage by paying off the entire loan. (B) is incorrect because the
bank cannot require the buyer to assume the mortgage, which would make the buyer personally
liable on the loan (i.e., make him liable for the balance of the loan if the bank forecloses and the
foreclosure sale does not bring in enough to pay off the loan balance). However, the bank can
foreclose even if the buyer does not agree to assume the mortgage. While the original mortgagor
remains primarily and personally liable, if the mortgagor has not paid off the mortgage and the
grantee (the buyer) does not pay, the mortgage may be foreclosed because it remains on the land.
(C) is incorrect because there is no indication that the buyer signed an assumption agreement. The
due-on-sale clause will allow the bank to foreclose but will not make the buyer personally liable
for any deficit after the foreclosure sale. (D) is incorrect. While some states refused to enforce
these clauses unless the lender’s security was endangered by sale to a poor credit risk, federal
statute makes due-on-sale clauses enforceable even if the lender is using it simply as an opportu-
nity to raise the interest rate when the property is sold.

Answer to Question 12

(C) The niece owns the property free of the mortgage because the buyer’s status as a bona fide
purchaser without notice brings the “shelter rule” into play. In general, a person who takes from a
bona fide purchaser will prevail against any interest that the transferor-bona fide purchaser would
have prevailed against, even if the person taking the property has actual or record notice of the
prior interest. Hence, the buyer’s status as a bona fide purchaser without notice “shelters” her
niece from the bank’s interest, and (C) is correct. (A) is incorrect because the niece’s “sheltered”
status as a transferee of a bona fide purchaser makes the fact that the recording act does not
directly protect donees irrelevant. (B) is incorrect because a pure notice recording statute requires
actual or constructive notice at the time of the conveyance. When the buyer conveyed to her
niece, the bank had not yet recorded. (D) is incorrect because mortgagees for value are treated as
“purchasers” under the recording act.
6. REAL PROPERTY - MORTGAGES ANSWERS

Answer to Question 13

(A) The bank will be paid in full before the developer is paid in full. Generally, the priority of
mortgages is chronological. A number of other factors, however, may affect priority. Where a
seller of property receives a mortgage as part of the purchase price, a purchase money mortgage
results. Purchase money mortgages may also arise when a third party lends money to the buyer
for the purchase of property and takes a mortgage on the property in return. In general, the seller’s
purchase money mortgage will take priority over the third-party purchase money mortgage.
Purchase money mortgages, however, are subject to later liens by virtue of recording acts. In the
case where a mortgage is modified by agreement between the parties, any increase in the debt
resulting from the modification will be subject to a junior lien, even if the original mortgage
itself had priority over the junior lien. In the same way, an optional (as opposed to an obligatory)
advance that is made after the junior lien will have a lower priority than the junior lien. Again,
this is the case even if the original mortgage is first in priority. Therefore, the distribution of
sale proceeds in this case would be: (i) the original amount of the employer’s purchase money
mortgage, (ii) the bank’s judgment, (iii) the $100,000 advance by the employer, and finally, (iv)
the amount of the increase in the debt to the developer due to the agreed modification of the
principal of the original loan. The original unmodified purchase money mortgage of the developer
would remain on the land because it was senior to the mortgage being foreclosed (the employer’s).
(B) is incorrect because the employer would not be paid in full before the bank received payment.
(C) is incorrect because the developer would initially receive only the increased amount of the
debt according to the modified loan terms. Also, the developer would have to wait until the bank
judgment was satisfied and the employer was paid in full before the developer’s claim for the
modification amount could be paid. (D) is incorrect because the developer would not be paid in
full—its original purchase money mortgage would remain on the property after the foreclosure.

Answer to Question 14

(C) The bank’s original mortgage has priority in the proceeds, followed by the lender’s mortgage,
and only the lender can proceed against the owner because the bank modified its mortgage after
the owner had transferred to the buyer. Generally, the priority of a mortgage is determined by the
time it was placed on the property, and the proceeds of a foreclosure sale will be used to pay off
the mortgages in the order of their priority. However, if the landowner enters into a modification
agreement with the senior mortgagee, raising its interest rate or otherwise making the agree-
ment more burdensome, the junior mortgage will be given priority over the modification. Thus, if
the first mortgage debt is larger because of the modification, the second mortgage gains priority
over the increase in the debt. Here, the bank and the buyer modified the original mortgage by
increasing the principal amount and the interest rate. This modification is not given priority over
the lender’s mortgage, and foreclosure proceeds will not be applied against it because the senior
lender’s mortgage was not fully satisfied from the proceeds. With regard to the deficiency, the
owner is liable to the lender because when a grantee signs an assumption agreement, becoming
primarily liable to the lender, the original mortgagor remains secondarily liable on the promis-
sory note as a surety. Here, the buyer assumed the lender’s mortgage and became primarily
liable; however, the owner remained secondarily liable as surety and can be required to pay off
the rest of the lender’s mortgage loan. On the other hand, the owner will not be liable to pay off
the balance of the bank’s loan, because when a mortgagee and an assuming grantee subsequently
modify the original obligation, the original mortgagor is completely discharged of liability. The
owner had nothing to do with the modification agreed to by the bank and the buyer that increased
the amount of the mortgage debt, and will not be even secondarily liable for that amount. (A)
and (B) are incorrect because the bank is not entitled to the entire $80,000 in proceeds from
REAL PROPERTY - MORTGAGES ANSWERS 7.

the sale and because the owner is not liable to the bank for more than the original loan amount.
(D) is incorrect because, as discussed above, the owner is secondarily liable to the lender for the
$20,000 deficiency on its mortgage.
TORTS - INTENTIONAL
TORTS QUESTIONS
TORTS - INTENTIONAL TORTS QUESTIONS 1.

TORTS - INTENTIONAL TORTS QUESTIONS

Question 1 Question 2

A repair technician employed by a photo- At the request of local police, security officials
copier company was called to a bank to fix a for an intercity passenger train made random
copy machine. It was Friday afternoon, and searches of passenger luggage for contraband
the technician planned to be on vacation the as it was being sent to the baggage claim area,
next week, so he decided to try to complete the although they did not have legal authority to
repairs on the spot even if it required working search bags without a warrant. The searches
past the normal end of the work day at 5 p.m., were conducted so that there was no delay in
when he was supposed to leave. He believed that the luggage being released to those claiming
he would be able to exit the building when he their bags. A traveler went to the baggage
was finished. Thus, he continued to work after claim area but his luggage, which contained a
the bank closed without alerting anyone as to number of valuables but no contraband, did not
his presence. At 5:30 p.m., the bank’s security appear. It had been selected for a search but the
guard activated its electronic security system, security officers were having difficulty getting it
which automatically locked the locks in every unlocked to search it. When the traveler inquired
door in the building. The guard then went home about the luggage, he was told that it was being
for the weekend. When the technician completed inspected and that he would have to remain in
his work at 5:45 p.m., he attempted to leave the the area if he wanted to claim it when it was
photocopy room, but discovered that the door released, and that its return could not be guaran-
was locked. There was no phone in the room and teed if he was not around when it was released.
his cell phone was dead, and he realized that no About 30 minutes later, the luggage was returned
one would be back at the bank until Monday. to the traveler with an apology for the delay. The
Noticing a smoke detector on the ceiling, he delay caused the traveler to miss his commuter
dumped a file of papers into a waste basket and train to the suburbs, so he had to pay for cab
set them ablaze. Eventually, the detector picked fare.
up the burning particles from the fire and set off
a fire alarm. The fire department arrived soon Assuming there are no issues of governmental
after and freed the technician. The papers were immunity, can the traveler bring an action
copies of documents for which the bank had the against the security officials for false imprison-
originals. ment?

What cause or causes of action may the bank (A) Yes, because the traveler suffered harm as a
assert against the technician? result of the delay in releasing his luggage.

(A) Conversion and trespass to land. (B) Yes, because the traveler reasonably
believed that he would not get his luggage
(B) Conversion but not trespass to land because back if he left the baggage area.
the technician believed that he would be
able to get out when he was finished. (C) No, because the traveler was not restrained
from leaving the baggage claim area.
(C) Trespass to land but not conversion because
the bank had original copies of the papers (D) No, because the delay in releasing the
that the technician burned. luggage was not done for the purpose of
restraining the traveler.
(D) Neither conversion nor trespass to land.
2. TORTS - INTENTIONAL TORTS QUESTIONS

Question 3 Question 4

A patient needing ankle surgery signed Late one night, a man was walking up a very
standard consent forms and liability waivers narrow city street when he noticed that a large
covering the surgeon scheduled to perform the garbage truck parked near the top of the hill
surgery. Two hours before the operation was had begun to roll, driverless, down the street
scheduled to be performed, one of the surgeon’s toward him. Seeing that he could not proceed
patients was brought into the emergency room forward and escape the truck, and if he turned
with numerous orthopedic injuries that required and ran downhill, he would soon be overtaken
immediate attention. The surgeon requested the and crushed, he pushed open a window in the
head of orthopedic surgery, who was the leading apartment building he was passing and climbed
authority on ankle surgery, to perform the ankle inside. The truck narrowly missed him as it
surgery for him so he could go to the emergency careened down the street. The elderly tenant in
room. By the time the head surgeon arrived the apartment, who had a serious heart condi-
in the operating room, the patient was already tion and lived in fear of intruders, thought that
sedated. He performed the operation with his the man was a local gang member trying to
usual skill and the operation was a complete burglarize his apartment and suffered a major
success. heart attack.

If the patient sues the head surgeon for If the elderly tenant sues the man, is the
battery, will she likely prevail? elderly tenant likely to recover damages?

(A) Yes, but she may be entitled only to nomi- (A) Yes, because of the man’s negligent inflic-
nal damages. tion of his emotional distress.

(B) Yes, because the head surgeon is vicari- (B) Yes, because of the man’s trespass on his
ously liable for the original surgeon’s property.
obtaining a replacement without the
patient’s consent. (C) No, because the man had no alternative to
avoid the garbage truck.
(C) No, because the head surgeon performed
the operation competently and the patient (D) No, if a reasonable person would not have
suffered no harm. been frightened.

(D) No, because a reasonable person similarly


situated would have consented to the opera-
tion.
TORTS - INTENTIONAL TORTS QUESTIONS 3.

Question 5 Question 6

A missile company was engaged in research A motorcyclist had just bought a new motor-
and development of an interplanetary space cycle. One week later, while still learning how
shuttle, under contract with the United States to ride the motorcycle, he saw a classmate from
government. Over a period of years, it developed school whom he did not like walking along the
the prototype of a huge, solid-fuel rocket engine sidewalk. He tried to scare him by swerving
for use in this program. To evaluate the perfor- onto the sidewalk at a driveway, planning to
mance of this engine, it conducted a static test of swerve back onto the street at the next driveway
the engine at a remote desert test site. The rocket just in front of the classmate. As the motorcy-
engine was mounted on a concrete test stand, clist attempted to swerve back onto the street,
with the thrust of the engine directed downward the motorcycle’s front tire blew because of a
into the ground. When the engine was fired up, latent defect, causing him to lose control of the
huge clouds of flame and smoke filled the air, steering. He attempted to apply the brakes, but
and particles of debris from the rocket fell onto due to his inexperience, hit the accelerator by
an adjoining farm. mistake. The motorcycle struck and seriously
injured the classmate. The classmate sues the
If the farmer files an action against the motorcyclist for battery.
company for trespass, which of the following
facts, if proved, would be most helpful to the Who is likely to prevail?
company in avoiding liability?
(A) The classmate, because the motorcyclist
(A) The farmer bought and operated his farm intended to frighten the classmate.
knowing that the company used the adjoin-
ing property for testing its rocket engines. (B) The classmate, unless the motorcyclist’s
negligence in hitting the accelerator was the
(B) Neither the company nor anyone in its proximate cause of the accident.
employ set foot upon the farmer’s land.
(C) The motorcyclist, because he did not intend
(C) The company had no reason to anticipate to inflict bodily harm on the classmate.
that the tests would cause any of the results
that occurred. (D) The motorcyclist, because the injury was
proximately caused by the defective front
(D) The rocket testing program is essential to tire.
national security, so that the company’s
conduct was completely privileged as a
public necessity.
4. TORTS - INTENTIONAL TORTS QUESTIONS

Question 7 Question 8

A wife discovered that her husband was A pedestrian was walking home when she saw
having an affair with his assistant and decided a man waving what looked to be a police badge
to seek revenge. One night, the wife drove to the pursuing another man with a wallet in his hand.
assistant’s house, drove onto her lawn, and began The pursuer yelled, “Don’t let him get away. I’m
blowing her horn and yelling obscenities out her a police officer and he just mugged someone!”
window. The wife was so loud that the assistant’s The pedestrian immediately put out her leg and
family and all of her neighbors were awakened tripped the man, injuring him. The pursuing
and heard the wife’s accusations. police officer arrested the man for the mugging,
a felony in the jurisdiction. Subsequent inves-
If the assistant asserts a claim based on inten- tigation revealed that the man had just picked
tional infliction of emotional distress against the the victim’s pocket, a misdemeanor, rather than
wife, will the assistant prevail? mugged him, as the victim had told the officer.

(A) Yes, because the wife’s conduct was ex- If the man sues the pedestrian for battery, will
treme and outrageous. the pedestrian have a valid defense?

(B) Yes, because the wife was intruding on the (A) Yes, because she saw the badge and there-
assistant’s property. fore had reasonable grounds to believe that
the pursuer was a police officer.
(C) No, because the assistant suffered no
physical harm. (B) Yes, because the police officer had reason-
able grounds to arrest the man.
(D) No, because the assistant was guilty of
adultery. (C) No, because the pedestrian did not witness
the crime.

(D) No, because a felony had not in fact been


committed.
TORTS - INTENTIONAL TORTS QUESTIONS 5.

Question 9 Question 10

A developer owned a 10-acre tract of land that While practicing their target shooting at
was covered by a number of small trees. The the firing range, a man and woman got into
first step in his development was the removal of an argument that almost erupted into physical
the trees. The developer’s neighbor knew that combat, except that they were restrained and
the developer intended to remove the trees from separated by bystanders. Later, in the parking
his property and either burn them or send them lot of the range, the man shot the woman in the
to a landfill. The day before development was shoulder. Bystanders who rushed to the scene
to start, the neighbor went onto the developer’s immediately after hearing the man’s shot found
land, cut down a number of trees for firewood the woman on the pavement with a black flash-
and moved them onto his property. The devel- light in her hand. The woman’s pistol was in
oper brought an appropriate action against the her locker at the firing range. At the trial of the
neighbor for damages. woman’s civil action for battery against the man,
the woman established that the man intention-
What is the most likely result? ally shot her. In defense, the man testified that
the woman approached him, saying, “We’ll settle
(A) The developer will recover only nominal this once and for all, right now,” and raised an
damages, because the value of the land was object toward the man. He testified that he feared
not diminished. that the woman was about to shoot him with a
pistol, so he fired in self-defense.
(B) The developer will recover nominal
damages and the value of the trees Assuming that the jury decides that the man is
removed. telling the truth, what else must the jury find for
him to prevail?
(C) The developer will not recover, because
removal of the trees saved him money in (A) No additional facts.
the development of the land.
(B) That a reasonable person in the same
(D) The developer will not recover, because the circumstances would have believed that the
value of the land was not diminished by the woman was about to shoot.
removal of the trees.
(C) That the woman was at fault in raising a
black object toward the man while threat-
ening him.

(D) That the woman was the original aggressor.


6. TORTS - INTENTIONAL TORTS QUESTIONS

Question 11 Question 12

A company that provided electrical and A man with a long history of mental illness
communication services to industries deter- was subject to recurrent hallucinations and
mined that its main trunk line of copper conduit delusions involving a belief that aliens from
needed to be replaced. The line ran through an another planet were about to attack him. When
industrial park, and the company had access to the man saw a pedestrian walking on the
it by an easement. At the end of the workday on sidewalk in his direction, he pulled a bottle from
Friday, the copper conduit that had not yet been his shopping bag and hurled it at her, striking her
disconnected was left exposed in the trench. In in the head and causing her injury. The pedes-
accordance with the company’s policy, intended trian sued the man for battery.
to discourage theft of the copper, the conduit
was still electrically charged. Which of the following, if true, provides the
best defense for the man?
The next morning, a man was walking by the
industrial park and saw the construction site. (A) The man had no desire to harm the pedes-
Ignoring the “no trespassing” signs, he came up trian.
to the trench and saw the copper conduit, which
he decided to try to steal. He climbed over the (B) Because of his mental illness, the man did
barriers and climbed partway into the trench to not understand that his act was wrongful.
try to pull out some of the conduit. He received
an electric shock as he made contact with the (C) The man did not know that he was striking
conduit, causing him to fall into the trench and a person.
suffer severe electrical burns.
(D) The man thought that the pedestrian was
If the man sues the company for his injuries, about to attack him.
is he likely to prevail?

(A) Yes, because the company used unreason-


able force to protect its property.

(B) Yes, because force applied by mechanical


devices may not be used to protect property
alone.

(C) No, because the company owed no duty to a


trespasser.

(D) No, because the man assumed the risk.


TORTS - INTENTIONAL TORTS QUESTIONS 7.

Question 13 Question 14

A sailor steering his sailboat through a A boat owner gave his brother-in-law permis-
channel was nearly swamped by a large cabin sion to use his boat for a few hours for fishing.
cruiser. The sailor made an obscene gesture and The brother-in-law instead kept the boat
shouted epithets at the captain of the larger boat, overnight, and the next day the boat was struck
who responded by swinging his boat around and and seriously damaged by a large tree that fell
heading at high speed directly at the sailboat’s on it at the boat launch. It was dark when the
bow. The sailor was convinced that the boats brother-in-law finally brought the boat home, and
would collide, so he steered close to the edge the owner immediately went over to the brother-
of the channel and abruptly ran aground on in-law’s house to retrieve it. The owner acciden-
a shallow sand bar. The sailor was extremely tally ran over a corner of the brother-in-law’s
upset but otherwise uninjured. His boat was not garden when he backed his truck up to the boat
damaged by hitting the sand bar. trailer in the dark. However, when he saw the
damage to the boat up close, the owner left the
If the sailor brings an appropriate action boat there and went home.
against the captain for damages, what is the
probable outcome? Assuming that the boat was worth $18,000
before it was damaged and sustained $9,000
(A) The sailor will win, because he suffered worth of damage, what should the owner be
severe emotional distress from the captain’s entitled to recover in an action against the
conduct. brother-in-law?

(B) The sailor will win, because he believed (A) $9,000 and damages for loss of use.
that the captain’s maneuvers threatened
imminent danger of harm to him. (B) $9,000 and damages for loss of use, offset
by the damage to the brother-in-law’s
(C) The captain will win, because the sailor garden.
suffered no physical injury or property
damage. (C) $18,000.

(D) The captain will win, because the sailor (D) $18,000, offset by the damage to the
was responsible for provoking the captain brother-in-law’s garden.
during the relevant events.
8. TORTS - INTENTIONAL TORTS QUESTIONS

Question 15 Question 16

A homeowner heard a loud bang against his A pet owner left his dog in his yard while he
window one evening. He looked out the window went to work. The dog’s constant barking greatly
and saw a 12-year-old boy from the neighbor- annoyed his neighbor. When the pet owner came
hood packing a large snowball. Fearful that he home that evening he found the dead body of his
would break the window with another throw, the beloved dog in the yard, with blood around its
homeowner went outside and said, “Come here, nose and mouth. He was very upset because the
I want to talk to you.” The boy ran in the other dog had been his pet for many years. A subse-
direction and jumped over the fence belonging to quent investigation revealed that his neighbor
a neighbor. Because it was dark, the boy landed had given the dog a treat with rat poison in it
on a birdbath and knocked it over, breaking during the day because she could not stand the
it. The neighbor brought an action against the barking.
homeowner for trespass.
If the pet owner brings an action against the
If the homeowner prevails, what is the most neighbor to recover for his emotional distress, is
likely reason? he likely to prevail?

(A) The homeowner confronted the boy in (A) Yes, if he suffered physical injury from his
order to defend his property. distress.

(B) The homeowner did not enter onto the (B) Yes, if the neighbor was aware that it was
neighbor’s land. very likely that the pet owner would suffer
severe emotional distress.
(C) The boy was the one who made the
decision to jump over the fence. (C) No, unless the neighbor intended to cause
the pet owner severe emotional distress.
(D) The homeowner did not intend to frighten
the boy onto the neighbor’s property. (D) No, because the neighbor did not kill the
dog in the presence of its owner.
TORTS - INTENTIONAL
TORTS ANSWERS
TORTS - INTENTIONAL TORTS ANSWERS 1.

TORTS - INTENTIONAL TORTS ANSWERS

Answer to Question 1

(A) The bank may assert a cause of action based on both of these theories. Trespass to land requires
(i) an act of physical invasion of plaintiff’s real property by defendant, (ii) intent on the part of
defendant to bring about a physical invasion of plaintiff’s real property, and (iii) causation. A
trespass to land may exist when defendant remains on plaintiff’s land after an otherwise lawful
right of entry has lapsed. Thus, because the technician intended to stay beyond the bank’s normal
hours and did so without permission, he has arguably committed a trespass. Conversion consists
of (i) an act by defendant interfering with plaintiff’s right of possession in the chattel, (ii) intent to
perform the act bringing about the interference with plaintiff’s right of possession, (iii) causation,
and (iv) damages—an interference that is serious enough in nature or consequence to warrant that
the defendant pay the full value of the chattel. Because the technician did intend to destroy the
bank documents that he burned, he is liable for conversion. Thus, (A) is correct and (D) is incor-
rect. (B) is incorrect because the technician’s belief that he would be able to get out is irrelevant to
his liability for trespass—he had the intent to stay in the bank knowing he was not permitted to do
so. (C) is incorrect because the fact that the bank had originals of the papers does not change his
liability for conversion. Because he intentionally destroyed the papers, he is liable for conversion
of those papers even if the bank’s damages were minimal.

Answer to Question 2

(B) The traveler can bring an action for false imprisonment because the delay in releasing his luggage
had the effect of confining him to the baggage area against his will. To establish a prima facie
case for false imprisonment, a plaintiff must prove (i) an act or omission to act on defendant’s
part that confines or restrains plaintiff to a bounded area, (ii) intent on the part of defendant to
confine or restrain plaintiff to a bounded area, and (iii) causation. The act or omission can be
directed against plaintiff’s property if its effect is to restrain plaintiff from leaving. Here, security
officials, who had no legal authority to conduct a search of the traveler’s luggage, were under a
duty to release it to him when he requested it. Requiring him to remain in the area to claim it
when it was released was a sufficient confinement or restraint for purposes of false imprisonment.
(A) is wrong because harm is not an element of the prima facie case for false imprisonment. The
traveler could recover at least nominal damages even if he had suffered no harm from the delay in
releasing his luggage. (C) is wrong because the seizure of the traveler’s luggage did have the effect
of restraining him because he believed that he would not get it back if he left the baggage claim
area. (D) is wrong because, for purposes of intentional torts, an actor “intends” the consequences
of his conduct if he knows with substantial certainty that these consequences will result. Even if
the delay in releasing the luggage was not done for the purpose of restraining the traveler, it was
substantially certain that he would remain in the area rather than risk losing his luggage. Hence,
the security officials had a sufficient intent for false imprisonment liability.

Answer to Question 3

(A) The patient can establish a prima facie case for battery. The fact that the operation was a success
and that she may not be able to prove actual damages will not bar her recovery. The prima facie
case for battery requires: (i) an act by defendant that brings about a harmful or offensive contact
to plaintiff; (ii) intent on the part of defendant to do the act; and (iii) causation. Here, the head
surgeon’s performing the operation on the patient’s ankle would be harmful or offensive contact
because the patient had selected another surgeon to perform the operation and did not consent
2. TORTS - INTENTIONAL TORTS ANSWERS

to the head surgeon’s participating in any way. Even if evidence of her distress is not adequate to
prove actual damages, she will still be entitled to a judgment in her favor and nominal damages,
because damages is not an element of the prima facie case for battery. (B) is wrong because the
head surgeon is directly liable to the patient for battery. The patient does not need to establish
vicarious liability based on the original surgeon’s conduct. (C) is wrong because the fact that
the head surgeon performed the operation competently is irrelevant because the patient did not
consent to his involvement at all. As discussed above, the prima facie case for battery does not
require harm (damages) to be shown. Even if she cannot prove actual harm, she will be entitled
to a judgment for nominal damages. (D) is wrong because the fact that a reasonable person would
have consented is irrelevant in a nonemergency situation. If the patient had been brought into the
emergency room requiring immediate surgery, her consent to the operation would be implied
by law if she was incapable of consenting and a reasonable person similarly situated would have
consented. Here, however, the operation was not an emergency and the patient’s express consent
should have been obtained.

Answer to Question 4

(B) The tenant can recover damages because of the man’s trespass. Under the defense of necessity, a
person may interfere with the real or personal property of another when the interference is reason-
ably and apparently necessary to avoid threatened injury from another force and the threatened
injury is substantially more serious than the invasion that is undertaken. However, when the act is
done not for the general public good but for the benefit of a limited number of people, the defense
is qualified; i.e., the actor must pay for any injury he causes. Here, although the man was privi-
leged to enter the tenant’s apartment to prevent his injury or death (private necessity), the privilege
does not extend to the infliction of damages. Thus, he must pay for any injury he caused, including
the tenant’s damages from his heart attack. (A) is wrong because negligent infliction of emotional
distress requires that the distress be caused by negligent conduct by the defendant. These facts
give no indication that the man acted negligently. (C) is wrong because, as stated above, the man’s
privilege is limited. While he cannot be forced off the premises as a trespasser, he is liable for the
injury he caused. (D) is wrong because it is irrelevant what a reasonable person would have felt.
If the man’s actions caused the tenant’s injury, he is liable because the defense of private necessity
provides only an incomplete privilege.

Answer to Question 5

(C) If the company had no reason to anticipate that the tests would cause the results that occurred,
then it cannot be said that the company intended to commit the act constituting trespass. Absent
such intent, there is no cause of action for trespass. A prima facie case for trespass to land consists
of: (i) an act of physical invasion of the plaintiff’s real property by the defendant; (ii) intent on the
defendant’s part to bring about a physical invasion of the plaintiff’s real property; and (iii) causa-
tion. The intent required is not intent to trespass; thus, mistake as to the lawfulness of an entry
onto another’s land is no defense as long as the defendant intended the entry upon that particular
piece of land. Here, the company, in firing the rocket engine, caused debris to fall onto the
farmer’s property. This would be a sufficient physical invasion for purposes of a trespass action.
However, maintenance of this action requires a showing that the company intended to send this
debris onto the land of the farmer. If (as stated in (C)) the company had no reason to anticipate
that its rocket engine tests would cause the debris to fall onto the farmer’s property, then the
company did not intend to make any entry onto the farmer’s property (i.e., the company neither
acted with the goal of sending debris onto the farmer’s land nor did it know with substantial
certainty that such a consequence would result from its tests of the engine). Consequently, under
TORTS - INTENTIONAL TORTS ANSWERS 3.

the circumstances set forth in (C), the element of intent would be missing, thereby precluding
the farmer from establishing a prima facie case for trespass. (A) is incorrect because the fact that
the farmer bought and operated the farm with knowledge that the company used the adjoining
land for engine tests will not allow the company to avoid liability for an act that would otherwise
be characterized as a trespass. The farmer’s knowledge of the use to which the company put the
adjoining property cannot be taken as implied consent to the scattering of debris on the farmer’s
land. (B) is incorrect because a physical invasion does not require that the defendant person-
ally set foot on the land. There is a trespass if, e.g., the defendant floods the plaintiff’s land or, as
here, causes debris to settle on the plaintiff’s land. (D) is incorrect because the defense of public
necessity arises where the public good is threatened with injury, and the defendant’s actions are
reasonably and apparently necessary to avoid such injury. Also, the threatened injury must be
substantially more serious than the defendant’s interference with the plaintiff’s property that seeks
to avoid such injury. This defense presupposes a situation in which immediate action is required
by the defendant to ward off an imminent threat to the public good. Testing rocket engines for
eventual military use does not rise to the level of conduct necessary to avoid an impending injury
to the public good. Thus, the company cannot successfully claim that its conduct is privileged as a
public necessity.

Answer to Question 6

(A) The classmate is likely to prevail. The motorcyclist intended to assault the classmate, because he
drove at the classmate with the intent to scare him. Under the doctrine of transferred intent, the
intent to assault is sufficient to establish a battery if a touching results. A battery requires: (i) a
harmful or offensive touching to the plaintiff’s person; (ii) intent; and (iii) causation. The causa-
tion element is satisfied here because the motorcyclist set in motion the force which brought about
the assault and subsequent battery. (C) is incorrect because the motorcyclist’s intent to commit an
assault is sufficient. (B) is incorrect because the motorcyclist’s potential negligence in hitting the
accelerator would not be sufficient to cut off liability for his battery. (D) is incorrect because the
fact that the tire may have been defective would not qualify as an intervening force sufficient to
cut off the motorcyclist’s liability. The classmate may have a separate claim against the motorcycle
manufacturer on a products liability theory, but the motorcyclist remains liable for the battery.

Answer to Question 7

(A) The assistant will likely prevail. Extreme and outrageous conduct is an element of the prima facie
case for intentional infliction of emotional distress. Because the wife’s conduct was extreme and
outrageous, intentional, and likely caused the assistant severe distress, the assistant will probably
prevail. (B) is incorrect because mere intrusion on the plaintiff’s property does not constitute
intentional infliction of emotional distress. (C) is incorrect because physical injury is not required
to recover for this tort. (D) is incorrect because, even though the assistant may be guilty of
adultery, this would not justify the wife’s extreme and outrageous behavior.

Answer to Question 8

(B) The pedestrian will have a valid defense to her use of force. A felony arrest without a warrant by
a police officer or a private citizen acting at the officer’s direction is privileged if the officer has
reasonable grounds to believe that a felony has been committed and that the person arrested has
committed it. In such a case, force reasonably necessary to make the arrest is permitted. Here,
the police officer had reason to believe he was chasing someone who had committed a felony, and
the pedestrian can rely on the officer’s privilege because she was acting at the officer’s direction.
4. TORTS - INTENTIONAL TORTS ANSWERS

(A) is incorrect because her belief that the pursuer was a police officer is not relevant here. The
pursuer was a police officer, so the pedestrian was entitled to the same privilege as he had because
she was acting at his direction. (C) is wrong because the pedestrian need not have been a witness
to make an arrest for a felony, and can rely on the police officer’s privilege even though the crime
committed was a misdemeanor. (D) is wrong because the pedestrian was acting at the officer’s
direction, and the officer need only have a reasonable belief that a felony has been committed.
Had she been acting on her own initiative, she would not have had a defense because the man in
fact did not commit a felony.

Answer to Question 9

(B) The developer can recover nominal damages and the value of the trees removed. The neighbor
trespassed onto the developer’s land and took the trees, which belonged to the developer. However,
the developer is entitled to receive only nominal damages for the act of trespass, because the
trespass caused no damage. The developer is also entitled to receive the value of the trees removed
by the neighbor since this amounted to a taking of the developer’s property. (A) and (D) are there-
fore incorrect. (C) is incorrect because the fact that the neighbor’s actions benefited the developer
would not diminish his right of recovery. The fact that the developer was not planning to sell the
trees does not mean that they had no value; the trier of fact will determine the fair market value of
the trees.

Answer to Question 10

(B) If the man prevails, it will be because the jury determined that he acted reasonably under the
circumstances. One may act in self-defense not only where there is real danger but also where
there is a reasonable appearance of danger. An honest but mistaken belief that the woman was
about to shoot would justify the use of deadly force by the man if a reasonable person would
have acted similarly under those circumstances. The test is an objective one—an honest belief
alone is not sufficient. Thus, (A) is incorrect. (C) is incorrect because the woman’s fault is not the
determining factor—the reasonableness of the man’s belief governs for self-defense. (D) is incor-
rect because it does not resolve whether the man had the right to use deadly force. Even if the
man started the altercation at the range, he would have the right to use deadly force if the woman
escalated the fight with deadly force.

Answer to Question 11

(A) The man will prevail because the company used unreasonable force to protect its property.
One may use reasonable force to prevent the commission of a tort against one’s property, real
or personal. However, force that will cause death or serious bodily harm may not be used. In
addition, indirect deadly force may not be used when such force could not lawfully be directly
used. Here, the company kept the power running to prevent the theft of its copper. In effect, this
amounts to the use of indirect deadly force as a means of preventing a tort to personal property.
As explained above, use of such force to protect property is not permitted. Thus, the man will
prevail in his suit against the company. (B) is incorrect because it is too broad. Force may be used
to protect only property if such force is reasonable, regardless of whether it is directly applied
or indirectly applied by mechanical devices. The problem here is that the force used by the
company was unreasonable because it was deadly force. Regarding (C), it is true that an owner
or occupier of land owes no duty to an undiscovered trespasser. However, the company is not an
owner or occupier of the land on which the line runs, but simply the owner of the line itself and
the holder of an easement across the land. Thus, the company is held to the general duty of due
TORTS - INTENTIONAL TORTS ANSWERS 5.

care regardless of the man’s status, rather than the more limited duty owed by a landowner or
occupier to a trespasser. (D) is incorrect because the facts do not indicate that the man assumed
any risk. To have assumed a risk, a plaintiff must have known of the risk and must have volun-
tarily gone ahead in the face of the risk. Here, the facts do not indicate that the man knew that
electrical current was still running through the line. In fact, because it was under construction
and exposed, the man likely assumed that the power was not running. Thus, he did not know of
the risk and could not have voluntarily assumed such a risk. Note also that, while the question
does not indicate the theory on which the man is basing his cause of action, it may be based on a
theory of intentional tort (specifically battery), in which case assumption of risk would not be an
appropriate defense.

Answer to Question 12

(C) If the defendant did not know that he was striking a person, he did not have the required intent
for battery. To establish a prima facie case for an intentional tort such as battery, the plaintiff
must prove (i) an act by the defendant, (ii) intent, and (iii) causation. The intent of the actor that
is relevant for purposes of intentional torts is the intent to bring about the consequences that are
the basis of the tort. An actor “intends” these consequences when his purpose is to bring them
about or when he knows with substantial certainty that they will result from his actions. The
intent required for battery is an intent to bring about harmful or offensive contact to the plain-
tiff’s person. [Restatement (Second) of Torts §13(a)] If the defendant did not know that he was
striking a person, he could not have intended or known that a harmful or offensive contact with
the person would result. Therefore, he will not have the intent required for battery. (A) is incor-
rect because the intent required is not an intent to cause harm to the person, but rather an intent to
bring about the harmful or offensive contact. (“Offensive” contact is contact that the plaintiff has
not expressly or impliedly consented to.) Thus, as long as the defendant at least knew that he was
bringing about an offensive contact with the plaintiff, he will be liable for battery even though he
had no desire to harm the plaintiff. (B) is incorrect because the defendant’s mental illness does
not preclude him from possessing the requisite intent for battery. As long as he was capable of
intending the consequences of his conduct, i.e., intending to bring about the harmful or offensive
contact, his inability to understand that his act was wrongful is irrelevant. (D) is incorrect because
the defense of self-defense is only available when a person has reasonable grounds to believe that
he is being attacked or is about to be attacked. This is an objective test—how the situation would
have looked to a reasonable person under the circumstances. The facts here make it clear that a
reasonable person would not have believed that he was about to be attacked. The defendant there-
fore will not be able to successfully claim self-defense.

Answer to Question 13

(B) The sailor will win because the captain’s actions constituted an assault. A prima facie case for
assault consists of: (i) an act by the defendant creating a reasonable apprehension in the plaintiff of
immediate harmful or offensive contact; (ii) intent on the part of the defendant to bring about that
apprehension; and (iii) causation. Here, if the sailor reasonably believed that the captain’s boat was
about to hit his boat (and thus cause a harmful or offensive contact with him), there is a basis for
assault because the other elements (intent on the part of the captain and causation) were present.
The only issue is whether the belief was reasonable. In determining whether the apprehension was
reasonable, the courts usually apply a reasonable person test. Here, a reasonable person certainly
could believe that the captain’s actions constituted a threat of an immediate harmful contact (e.g.,
a crash). Thus, there would be a basis for assault. (A) is wrong because the sailor can prevail on
an assault claim regardless of whether he can establish all the elements of a claim for intentional
6. TORTS - INTENTIONAL TORTS ANSWERS

infliction of emotional distress. That tort will require the sailor to prove not only that he suffered
emotional distress but also that the captain’s conduct was extreme and outrageous. While a trier
of fact might find that to be the case, intentional infliction of emotional distress should be consid-
ered a fallback tort position, to be chosen only if there is no other tort available from the facts
and answer choices. (C) is wrong because damages are recoverable for assault without the plain-
tiff’s suffering actual physical injury. Assault is the causing of apprehension of contact; it does
not require actual contact. The sailor can recover damages for his emotional distress through
his assault claim. (D) is wrong because, unless the sailor’s initial actions were such as to justify
the captain’s acting in self-defense, it is immaterial that the sailor made insults provoking the
captain’s tortious conduct. Self-defense is permitted when a person reasonably believes he is being
or is about to be attacked. Here, it is clear that the sailor’s conduct would not justify the captain’s
actions in self-defense. Also, self-defense would not apply because the captain’s conduct was
clearly not defensive; he came after the sailor even though the sailor was not a threat at that point.

Answer to Question 14

(C) The owner can recover $18,000 from the brother-in-law for conversion. A conversion is an inter-
ference with the plaintiff’s possessory rights in a chattel that is so serious as to warrant that the
defendant pay full value for the chattel. Here, the brother-in-law’s permission to use the boat had
long since expired when the boat was severely damaged by the tree. While the brother-in-law
would not have been responsible for that damage had it occurred while he was still rightfully in
possession of the boat, the combination of the wrongful detention and the severe damage amounts
to a conversion. Hence, the owner is entitled to recover damages for the fair market value of the
boat at the time and place of conversion, in effect creating a forced sale of the boat. (A) and (B)
are wrong because the owner can recover more than the actual damage to the boat and damages
for dispossession; those remedies are appropriate for trespass to chattels, which is a less serious
interference with the plaintiff’s possessory rights in a chattel. (B) and (D) are wrong because the
owner is not liable for the damage to the brother-in-law’s garden when he went to retrieve the boat.
When a chattel is located on the land of a wrongdoer, the owner is privileged to enter on the land
and reclaim it in a reasonable manner. Unlike an entry onto the land of an innocent party, reason-
able entry onto the land of a wrongdoer is completely privileged; i.e., the chattel owner does not
have to pay for any actual damage caused by the entry.

Answer to Question 15

(D) The best basis for the homeowner to prevail is that he did not intend to frighten the boy onto
the neighbor’s property. For the neighbor to succeed in his trespass suit, he must show that
the homeowner intended to bring about a physical invasion of the neighbor’s property. The
homeowner did not chase the boy onto the neighbor’s yard, nor did the homeowner intend or
know with substantial certainty that the boy would enter onto the neighbor’s yard as a result of
the homeowner’s actions. (A) is incorrect because a landowner is not automatically privileged to
chase or otherwise cause third persons to enter onto another’s land to prevent the commission of
a tort against his property. While the landowner may have a qualified defense if the trespass was
reasonable and apparently necessary to protect his property from destruction or serious injury,
the interference with the neighbor’s property here did not result from necessity. (B) is incorrect
because it is not necessary to establish a prima facie case for trespass to land that the defendant
personally came onto the land; e.g., trespass exists where the defendant floods the plaintiff’s land,
throws rocks onto it, or chases third persons upon it. (C) is incorrect because even though the
boy made the decision to go over the fence, the homeowner could still be liable for trespass if the
homeowner acted with the intention of causing the boy to enter onto the neighbor’s land.

一 信:liuxue119118 , 们 信免 供
TORTS - INTENTIONAL TORTS ANSWERS 7.

Answer to Question 16

(B) The pet owner will prevail if the neighbor was aware that it was very likely that the pet owner
would suffer severe emotional distress. An action for intentional infliction of emotional distress
requires (i) an act by the defendant amounting to extreme and outrageous conduct, (ii) intent on
the part of the defendant to cause the plaintiff to suffer severe emotional distress, or recklessness
as to the effect of the defendant’s conduct, (iii) causation, and (iv) damages—i.e., severe emotional
distress. Here, intentionally killing someone’s pet by giving it poison would be considered extreme
and outrageous conduct by most courts, and the pet owner appears to have suffered severe
emotional distress. If, as (B) states, the neighbor was aware that it was very likely that the pet
owner would suffer severe distress, she has acted with the mental state of recklessness, completing
the prima facie case. (A) is wrong because a showing of physical injury is not necessary for
intentional infliction of distress—severe emotional distress will suffice. (C) is incorrect because
it is too narrow. It is not essential for the pet owner to show an intent to cause severe emotional
distress—a reckless disregard of a high probability that emotional distress will result satisfies the
state of mind element. (D) is incorrect because it is not essential to recover for this tort that the
neighbor have killed the dog in the pet owner’s presence. Given the circumstances of the dog’s
death and the pet owner’s discovery of the dog’s body, the conduct is likely to be found extreme
and outrageous.
一 信:liuxue119118 , 们 信免 供

TORTS - NEGLIGENCE
CAUSATION AND PARTIES
QUESTIONS
TORTS - NEGLIGENCE CAUSATION AND PARTIES QUESTIONS 1.

TORTS - NEGLIGENCE CAUSATION AND PARTIES QUESTIONS

Question 1 Question 2

A driver struck a 10-year-old boy who had A bicyclist was riding his bicycle in the
darted into the road during a game of tag. After street when a negligently driven car struck the
the accident, the boy’s mother refused to take bike, knocking the bicyclist off the bike and
him to a physician because of moral scruples breaking his right ankle. The driver of the car
against the medical profession. As a result, the immediately stopped and went to his assistance.
boy’s injuries were more severe than they would She got him to his feet and was slowly moving
otherwise have been. him toward the curb when a negligently driven
taxicab struck him in the left leg. The bicyclist
What is the boy’s best argument for recov- required surgery on both his right ankle and his
ering all of his injuries in an action against the left leg.
driver?
If the bicyclist sues the driver and the cabbie,
(A) The doctrine of avoidable consequences which of the following best states his right to
at most bars recovery for the aggravation recover?
of the injury but not for the original injury
itself. (A) He can recover from either the driver or the
cabbie for all of his injuries because the
(B) Any negligence on the mother’s part is not driver and the cabbie are jointly and sever-
to be imputed to her child. ally liable.

(C) Victims have no duty to take steps for their (B) He can recover from the driver only for the
own safety after the accident. injury to his right ankle and recover from
the cabbie only for the injury to his left leg.
(D) Defendants must take their victims as they
find them, including their mothers’ attitudes (C) He can recover from either the driver or
toward physicians. the cabbie for the injury to his left leg and
recover from the driver only for the injury
to his right ankle.

(D) He cannot recover against the driver for


the injury to his left leg unless the jury
determines that the driver acted negligently
when she came to his aid.
2. TORTS - NEGLIGENCE CAUSATION AND PARTIES QUESTIONS

Question 3 Question 4

An employee of a construction company A motorist was negligently driving close to


working in a high crime neighborhood negli- the shoulder of a highway when his vehicle
gently cut through power cables. The accident skidded and hit a support column of a bridge
knocked out power to the area and disabled that crossed over the highway. The impact from
a home’s security system. The homeowner, the car caused structural damage to the support
who was out of town, received an alert from column, which caused the bridge to drop 18
the security company advising her to arrange inches. The sag of the bridge was clearly visible
for someone to go to the house and restart from the highway. The motorist died in his
the system with a backup power source. The heavily damaged car as a result of the accident.
homeowner got distracted soon after getting the A rescuer, who had been five miles away at the
message and neglected to contact anyone. That time of the accident, came on the scene and
evening, a burglar broke into the home while pulled his car off the road to see if he could
power was still off in the neighborhood. Because render assistance. Shortly thereafter, a trucker
the security system was not working and the approached the scene of the accident. The
alarm did not go off, the burglar was able to trucker saw that an accident had occurred, and
steal jewelry and other valuables from the home had adequate time to slow down or stop, but he
and escape. The homeowner sued the construc- proceeded ahead without reducing speed. Under
tion company for the loss of her valuables in a ordinary circumstances, his truck could have
jurisdiction that has adopted pure comparative passed easily under the bridge, but the 18-inch
negligence rules. drop caused the top of the truck to strike the
bridge. A chunk of concrete fell from the bridge,
Is the homeowner likely to recover? striking the rescuer in the head and seriously
injuring him.
(A) Yes, because the homeowner’s negligence
contributed the least to her loss. If the rescuer sues the motorist’s estate, who
will prevail?
(B) Yes, because the conduct of the construc-
tion company’s employee created the oppor- (A) The motorist’s estate, because the trucker’s
tunity for the burglar to steal the valuables. actions caused the rescuer’s injuries.

(C) No, because the homeowner’s negligence (B) The motorist’s estate, because the rescuer
was a superseding cause of her loss. was five miles away when the initial
accident occurred and therefore not a
(D) No, because the burglar committed a foreseeable plaintiff.
criminal act that was a superseding cause
of the loss. (C) The rescuer, because he stopped to render
assistance.

(D) The rescuer, because the motorist’s negli-


gence was the proximate cause of the
rescuer’s injuries.
TORTS - NEGLIGENCE CAUSATION AND PARTIES QUESTIONS 3.

Question 5 Question 6

A state statute required that any freight train A farmer employed a 16-year-old high school
operating within the city limits be able to stop student for a summer agricultural labor job. One
within 200 yards of applying its brakes. No fixed afternoon, a violent storm suddenly erupted as
speed limit was established or particular type of the farmer was driving a tractor up a hill in an
braking mechanism required, but through either open field with the student in the wagon behind.
lowered speed or braking power, the 200-yard When loud claps of thunder erupted, the farmer
limit was required of all trains. Another statute stopped his tractor, jumped off without saying
prohibited vehicles from being within the anything, and ran swiftly down the hill toward
railroad crossing when the lights on the warning the low ground, which he knew would be safer.
signs are flashing or when the gates are lowered. The student, who lived in a nearby city and had
One day, as a freight train was entering the never seen an electrical storm in open country
city limits, the engineer saw a car stalled at a (except as a passenger inside an automobile), had
street crossing ahead. He immediately applied never been told how to act safely during such a
full braking power, but was unable to stop the storm. Once the storm began, the student was
train before it had hit and demolished the car. struck by lightning and seriously injured as he
The driver of the car had gotten clear before the stood at the crest of the hill watching the farmer
impact, but brought suit against the freight line run.
for property damage to the $25,000 car. At trial,
the parties stipulated that the car was stalled Is the farmer liable to the student for the
within the crossing while the warning lights injuries caused by lightning?
were flashing. Evidence at trial established that
the distance from the point at which the engineer (A) Yes, because the student was an employee,
applied the train’s brakes to the point of impact acting within the scope of his employment.
was 150 yards, and from the braking point to
the point at which the train finally stopped was (B) Yes, because the student was a minor.
225 yards. No other evidence of negligence was
presented by the driver. At the end of the driver’s (C) No, because the student was injured by an
case, the freight line moved for a directed act of God.
verdict.
(D) No, because lightning is never foreseeable.
Should the court grant the motion?

(A) No, because the freight line was negligent


per se.

(B) No, because the freight line was strictly


liable for its violation of the braking statute.

(C) Yes, because the driver’s car was on the


freight line’s tracks in violation of the
crossing statute.

(D) Yes, because the freight line’s violation of


the braking statute was not the cause in fact
of the accident.
4. TORTS - NEGLIGENCE CAUSATION AND PARTIES QUESTIONS

Question 7 Question 8

A chemical plant and a steel mill located on Two companies manufacture pesticide and
a river both negligently discharged toxic waste have plants located along the same river. During
into the river during the same time period. A a specific 24-hour period, each plant discharged
farmer downstream who used water from the pesticide into the river because of negligence
river for irrigation suffered substantial crop in their operations. A rancher operated a cattle
losses as a result of using the poisoned water. He ranch downstream from the plants. The rancher’s
brought suit against the chemical plant. At trial, cattle drank from the river and were poisoned by
the evidence established that the discharge from the pesticide. The amount of the discharge from
either plant alone was sufficient to have caused either plant alone would not have been sufficient
the farmer’s crop losses. to cause any harm to the rancher’s cattle.

How much of the farmer’s damages should he If the rancher asserts a claim against the two
recover from the chemical plant? companies, what, if anything, will the rancher
recover?
(A) All of his damages, because the chemical
plant’s negligence was a substantial factor (A) Nothing, because neither company dis-
in causing the farmer’s damage. charged enough pesticide to cause harm to
the rancher’s cattle.
(B) Half of his damages, because he did not
present evidence to allow the court to (B) Nothing, unless the rancher can establish
reasonably apportion damages between the how much pesticide each plant discharged.
two tortfeasors.
(C) One-half of the rancher’s damages from
(C) None of his damages, because he needed to each company.
join the other tortfeasor in the litigation.
(D) The entire amount of the rancher’s
(D) None of his damages, because the harm damages, jointly and severally, from the
would have occurred even in the absence of two companies.
the chemical plant’s negligence.
TORTS - NEGLIGENCE CAUSATION AND PARTIES QUESTIONS 5.

Question 9 Question 10

The plaintiff was badly injured in a three- In a negligence action against two joint
car accident and suffered $100,000 worth of tortfeasors, the jury determined that the plaintiff
damages. The trier of fact determined that the suffered $100,000 in damages and that she was
plaintiff was 30% at fault, the defendant was 10% at fault. The first defendant was judged to
30% at fault, and the third driver was 40% at be 60% at fault and to have suffered $200,000
fault. in damages. The second defendant was judged to
be 30% at fault and to have suffered no damages.
How much can the plaintiff recover from the
defendant? If all parties assert their respective valid
claims and the plaintiff ends up with an award
(A) $70,000, but the defendant can compel the of $40,000 from the first defendant, which of the
third driver to pay him $40,000 as contribu- following statements is most likely to be true?
tion.
(A) The jurisdiction follows traditional joint
(B) $40,000, but the defendant can compel the and several liability and traditional contri-
third driver to contribute $10,000. bution rules.

(C) $30,000, because the defendant was 30% at (B) The jurisdiction follows traditional joint
fault. and several liability rules and comparative
contribution rules.
(D) Nothing, because the defendant was no
more at fault than the plaintiff. (C) The jurisdiction has abolished joint and
several liability.

(D) The jurisdiction has not adopted pure


comparative negligence.
6. TORTS - NEGLIGENCE CAUSATION AND PARTIES QUESTIONS

Question 11 Question 12

During a personal injury case, the jury deter- A driver picked up his cousin and his best
mined that the plaintiff was 30% at fault for his friend to go to a concert. The friend sat in the
own injuries, the defendant was 30% at fault, front seat, and the cousin sat in back. On the way
and a third party was 40% at fault. The jury to the concert, the friend pulled out a water pipe
further found that the plaintiff suffered $100,000 with which to smoke marijuana. He handed it to
worth of damages. The jurisdiction has a partial the driver, and then grabbed the steering wheel
comparative negligence statute that bars a plain- in order to steer the vehicle while the driver
tiff’s recovery if his fault was greater than that lit up the water pipe. The driver continued to
of the defendant, and has abolished joint and operate the brake and gas pedals with his feet.
several liability. As the driver smoked the water pipe, the steering
wheel slipped from the friend’s hand, and the
How much can the plaintiff collect from the car smashed into a tree. The cousin in the back
defendant? seat was injured, incurring a broken wrist and a
broken ankle. She sued both the driver and the
(A) $100,000, because the partial comparative friend for damages. Following a trial, the jury
negligence statute does not bar his recov- found the driver 60% negligent and the friend
ery. 40% negligent, and awarded the cousin a total
of $10,000 for medical expenses and $100,000
(B) $70,000, because the plaintiff was 30% at for pain and suffering. Because the driver is
fault. insolvent, the cousin wants to collect all of the
damages from the friend.
(C) $30,000, because the defendant was 30% at
fault. In the absence of a statute on the issue, may
she do so?
(D) Nothing, because the defendant was no
more at fault than the plaintiff. (A) No, because tortfeasor liability is propor-
tional to fault in the case of noneconomic
damages.

(B) No, because the driver and the friend did


not act in concert.

(C) Yes, because the injuries to the cousin are


divisible.

(D) Yes, because the friend is jointly and sever-


ally liable.
TORTS - NEGLIGENCE CAUSATION AND PARTIES QUESTIONS 7.

Question 13 Question 14

A company that owned a tract of land believed A company let an employee borrow one of
to be rich in mineral deposits contracted with its company cars for a cross-country vacation
a licensed excavator for the removal of soil trip which the employee had planned for his
from the property and delivery of the soil to the family. While driving through a remote stretch
company’s laboratories. While one of the excava- of farmland, the employee decided to see how
tor’s trucks was on the way to the laboratory, the much power the car really had, and was driving
rear gate broke loose, dumping three tons of soil in excess of 90 m.p.h. when he came to a curve.
onto the highway. A motorist who was driving He applied the brakes and attempted to slow
a short but safe distance behind the truck was down, but the car went across the double line
unable to stop in time and collided with the soil, and struck head-on a minivan coming in the
causing her serious injury. The rear gate had opposite direction. The driver of the minivan
been negligently secured by one of the excava- was killed in the accident and the minivan was
tor’s employees. destroyed.

If the motorist sues the company for his A “permissive use” statute is in effect making
injuries and does not prevail, what is the most the bailor of an automobile liable for personal
likely reason? injury, death, or property damage caused by
any person operating the automobile with his
(A) The rear gate was secured by the excava- consent, up to a maximum of $25,000. The juris-
tor’s employee. diction follows traditional contribution rules.

(B) The excavator had a license to transport soil If the driver’s estate files suit against the
on the highway. company pursuant to the “permissive use”
statute, and recovers the full $25,000, what
(C) The company’s duty in respect to the rights, if any, would the company have against
movement of its soil on the highway was the employee?
delegable.
(A) None, unless there was a written agreement
(D) The transportation of soil on the highways with the employee that obligated him to
was a common practice in the area where assume any such liability imposed on the
the accident occurred. company.

(B) The company may obtain contribution from


the employee to the extent of $12,500, but
not indemnity.

(C) The company may obtain indemnity against


the employee for the full $25,000.

(D) The company may obtain contribution from


the employee to the extent of the employee’s
relative fault, but not indemnity.
8. TORTS - NEGLIGENCE CAUSATION AND PARTIES QUESTIONS

Question 15

An electrician was employed by an electrical


services company that had contracts with a
number of large office and condominium build-
ings to provide emergency electrical services and
repairs at any hour of the day or night. Hence, he
was required to be “on call” 24 hours a day and
to drive his company van, which had all of his
tools, to his home each night. One afternoon, the
electrician left the company’s office at 4 p.m. as
usual. However, when he left the main highway,
he did not turn left toward his home but instead
turned right toward the supermarket a few
blocks away to pick up some items for dinner.
While leaving the supermarket parking lot, the
electrician drove negligently and struck a pedes-
trian. The pedestrian suffered serious injuries
and required several operations and a lengthy
hospital stay. The pedestrian filed suit against the
company for $100,000.

Is the pedestrian likely to recover from the


company?

(A) Yes, because the electrician’s trip to the


market was only a slight deviation from the
direct route to his home.

(B) Yes, but only if the company knew that the


electrician had proclivities to drive negli-
gently.

(C) No, because turning in the opposite direc-


tion from his home constituted a “frolic” by
the electrician.

(D) No, because an employer is not liable for


the torts of an employee traveling to and
from work.
TORTS - NEGLIGENCE
CAUSATION AND PARTIES
ANSWERS
TORTS - NEGLIGENCE CAUSATION AND PARTIES ANSWERS 1.

TORTS - NEGLIGENCE CAUSATION AND PARTIES ANSWERS

Answer to Question 1

(B) The boy’s best argument is that his mother’s refusal to take him to a physician, if deemed to
be negligent, is not imputed to him. A plaintiff has a duty to take reasonable steps to mitigate
damages. Thus, in personal injury cases, there is a duty to seek appropriate treatment to effect
healing and to prevent aggravation. Failure to do so will preclude recovery for any particular item
of injury that occurs or is aggravated due to the failure to mitigate (this is the avoidable conse-
quences rule). Thus, the boy’s not consulting a doctor could limit his recovery to the damages for
the original injury only. However, the boy is a child and his mother decided not to seek medical
help for the boy. If this was negligence on her part, is it imputed to the boy? In actions against
a third party, a parent’s negligence is not imputed to the child. Thus, the negligence of the boy’s
mother will not be imputed to the boy, and so the boy should receive a full recovery for all of his
injuries. On the other hand, if the boy uses the argument in (A), then he will probably not recover
for the aggravated injuries. (A) presents an accurate statement of law, relative to the effect of the
avoidable consequences rule. If the boy avails himself of the avoidable consequences rule, then
he will succeed in salvaging merely his right to recover for the original injury. Thus, (A) does
not give the boy a chance to recover for all of his injuries, as does (B). (C) is incorrect because it
directly contradicts the rule that a plaintiff must take all reasonable measures to mitigate damages
after the original injury is inflicted. (D) is incorrect because it misstates the concept of “taking
your victim as you find him.” This concept refers to the physical or mental condition of the
victim at the time of the injury (e.g., the “eggshell skull plaintiff”); it does not cover the victim’s
relationship to others and their attitudes or actions. Thus, the attitude of a victim’s mother toward
physicians is not included in “taking your victim as you find him.”

Answer to Question 2

(C) The bicyclist can recover from either party for the left leg injury but only from the driver for the
right ankle injury. When two or more tortious acts combine to proximately cause an indivisible
injury to a plaintiff, each tortfeasor is jointly and severally liable to the plaintiff for the entire
damage incurred. Joint and several liability applies even though each tortfeasor acted entirely
independently. However, if the actions are independent, plaintiff’s injury is divisible, and it is
possible to identify the portion of injuries caused by each defendant, then each will be liable only
for the identifiable portion. Here, the cabbie would not be liable for the injury to the right ankle,
because the cabbie did not cause the injury. (A) is therefore incorrect. With regard to the left leg,
the cabbie was not the only cause of that injury. The original tortfeasor is liable for harm caused
by the negligence of third persons when such negligence was a foreseeable risk created by the
original tortfeasor’s conduct. Here, as a result of the driver’s original negligence, the bicyclist was
in a position of danger while he was still in the street. The negligence of the cabbie in striking
the bicyclist was a foreseeable risk while the bicyclist was in the street; it is therefore a foresee-
able intervening force that will not cut off the driver’s liability. Hence, both the driver and the
cabbie will be jointly and severally liable for that injury. (B) is therefore incorrect. (D) is incorrect
because the driver remains responsible for the foreseeable consequences of her original negligence
in striking the bicyclist, regardless of whether she acted with due care when she came to his aid.

Answer to Question 3

(B) The homeowner is likely to recover from the construction company. The general rule of proximate
cause is that the defendant is liable for all harmful results that are the normal incidents of and
2. TORTS - NEGLIGENCE CAUSATION AND PARTIES ANSWERS

within the increased risk caused by his acts. In indirect cause cases, an independent intervening
force may be foreseeable where the defendant’s negligence increased the risk that these forces
would cause harm to the plaintiff. Even a criminal act by a third party will not cut off the defen-
dant’s liability if the defendant’s negligence created a foreseeable risk that a third person would
commit the crime. Here, the construction company negligently cut the power in a high crime
neighborhood, increasing the risk of criminal conduct occurring. But for the power being cut,
the homeowner’s valuables would not have been stolen. A jury is likely to find that the burglar’s
intervening act was sufficiently foreseeable that the construction company will be held liable for
at least some of the damages suffered by the homeowner. (A) is incorrect because, under pure
comparative negligence rules, the homeowner could recover some of her damages even if her
negligence was deemed to be greater than that of the defendant. (C) is incorrect because super-
seding cause analysis does not apply to the plaintiff’s negligence. The homeowner’s failure to have
the security system reset is an issue of contributory negligence, which under the jurisdiction’s pure
comparative negligence rules is not a complete defense. (D) is incorrect because, as discussed
above, the burglar’s conduct probably would be deemed foreseeable and therefore not a super-
seding force that cuts off the construction company’s liability.

Answer to Question 4

(D) The rescuer will prevail against the motorist’s estate because he can establish a prima facie case
of negligence. One of the elements of negligence is a showing that defendant’s breach of duty
was the actual (cause in fact) and proximate cause of plaintiff’s injury. An act or omission to
act is the cause in fact of an injury when the injury would not have occurred but for the act. The
motorist’s accident was the actual cause of the rescuer’s injury because, but for the accident, the
rescuer would not have stopped to assist the motorist. The motorist’s accident was also a proxi-
mate cause of the rescuer’s injury. The general rule of proximate cause is that defendant is liable
for all harmful results that are the normal incidents of and within the increased risk caused by
his acts. This is an indirect cause case because an independent intervening force (the truck)
came into motion after defendant’s negligent act and combined with it to cause plaintiff’s injury.
Independent intervening forces are foreseeable (and thus do not cut off defendant’s liability)
where defendant’s negligence increased the risk that these forces would cause harm to the plain-
tiff. Once the motorist negligently put himself in peril on the highway, he created a foreseeable
risk that a rescuer would be injured in some way by the act of another motorist while the rescuer
was assisting the motorist. Thus, the trucker’s negligence was a foreseeable intervening force that
combined with the motorist’s negligence to create a foreseeable harmful result to the rescuer.
The motorist’s estate, therefore, is not relieved of liability by the trucker’s conduct. (A) is incor-
rect because even if the trucker’s actions caused the rescuer’s injuries, they were a foreseeable
risk created by the motorist’s conduct and thus do not constitute a superseding intervening force
that would cut off the motorist’s liability to the rescuer. (B) is incorrect. The motorist owed a
duty of care to the rescuer under the general rule that a rescuer is a foreseeable plaintiff as long
as the rescue is not reckless. Hence, a defendant will be liable if he negligently puts himself in
peril and the plaintiff is injured in attempting a rescue. The fact that, at the time the motorist’s car
struck the bridge support, the rescuer was five miles from the bridge does not make the rescuer an
unforeseeable plaintiff. He could still be considered a foreseeable rescuer. (C) is incorrect because
the fact that the rescuer stopped to render assistance merely establishes him as a foreseeable plain-
tiff. The critical issue is whether he can establish proximate cause.

Answer to Question 5

(D) The court should grant the motion because the driver did not establish the cause-in-fact element
TORTS - NEGLIGENCE CAUSATION AND PARTIES ANSWERS 3.

of his prima facie case against the freight line. The primary test for cause in fact (actual cause) is
the “but for” test: An act is the cause in fact of an injury when the injury would not have occurred
but for the act. Even though the freight line had a duty created by the statute to be able to stop its
train within 200 yards of first braking, and breached that duty (establishing the first two elements
of the driver’s prima facie case), it must still be shown that the collision would not have occurred
in the absence of the breach. Because the car was only 150 yards from the point of braking, even
a train in compliance with the statute would have struck it. Since no other evidence of negligence
has been presented, the motion should be granted. (A) is incorrect because establishing the freight
line’s “negligence per se” through violation of the statute only establishes a conclusive presump-
tion of duty and breach of duty; the plaintiff must still prove causation. (B) is incorrect because
generally violation of a statute does not create strict liability; even if it did in this case, the plain-
tiff would still have to prove causation as part of the prima facie case for strict liability. (C) is
not correct because the court will not reach the issue of the plaintiff’s contributory negligence in
this case because the prima facie case for the defendant’s negligence has not been established.
Furthermore, establishing the plaintiff’s contributory negligence by violation of a statute uses
the same rules that govern whether a statute can establish the defendant’s negligence. Hence, the
driver’s violation of the crossing statute may be excused if the trier of fact determines that compli-
ance was beyond his control because his car stalled.

Answer to Question 6

(A) As an employer, the farmer breached his duty of care owing to the student and therefore is liable
for the student’s injuries on a negligence theory. To establish a prima facie case for negligence,
the following elements must be proved: (i) the existence of a duty on the part of defendant to
conform to a specific standard of conduct for the protection of the plaintiff against an unreason-
able risk of injury; (ii) breach of that duty by defendant; (iii) the breach of the duty by defendant
was the actual and proximate cause of plaintiff’s injury; and (iv) damage to the plaintiff’s person
or property. The first issue raised by these facts is whether the farmer owed a duty of care to
his employee. As a general matter, no legal duty is imposed upon any person to affirmatively
act for the benefit of others. However, the existence of a special relationship between the parties
may create a duty. Modern cases extend the duty to employers when employees are injured in
the course of employment. Thus, the farmer owed the student a duty to protect him against an
unreasonable risk of injury while he was acting within the scope of his employment. The farmer
breached this duty by not warning and instructing the student in how to act safely during an
electrical storm. The breach of that duty was the cause in fact and proximate cause of the student’s
injuries. An act or omission to act is the cause in fact of an injury when the injury would not
have occurred but for the act. The “but for” test applies where several acts combine to cause the
injury, but none of the acts standing alone would have been sufficient. But for any of the acts, the
injury would not have occurred. Thus, but for the farmer’s failure to instruct the student on how
to act during an electrical storm, the student would not have been injured. The farmer’s failure to
instruct is also the proximate cause of the student’s injuries. The general rule of proximate cause
is that defendant is liable for all harmful results that are the normal incidents of and within the
increased risk caused by his acts. This is an indirect cause case because an independent inter-
vening force (the lightning) came into motion after the farmer’s negligent conduct and combined
with it to cause the student’s injury. Independent intervening forces are foreseeable (and thus do
not cut off defendant’s liability) where defendant’s negligence increased the risk that these forces
would cause harm to the plaintiff. The farmer’s negligent failure to instruct the student about
the need to seek low ground during an electrical storm greatly increased the risk that the student
would be struck by lightning when the storm came up. Because the lightning was foreseeable
and brought about a foreseeable harmful result to the student, it was not a superseding force that
4. TORTS - NEGLIGENCE CAUSATION AND PARTIES ANSWERS

would cut off the farmer’s liability for the student’s injuries. (B) is incorrect because the student’s
minority does not create a duty toward him by the farmer. The duty of care arises out of the
employer/employee relationship. (C) is incorrect because, as noted above, the act of God (the
lightning) would not be a superseding intervening force since it was foreseeable. Here, the farmer
was negligent in not seeking to minimize the chances of the student’s being struck by lightning,
when the farmer knew that such danger existed and owed the student such duty as a result of his
relationship (employer/employee) with the student. (D) is similarly incorrect because lightning
can be foreseeable and was foreseeable here. The rain and loud claps of thunder were a clear
signal that lightning might occur, and the farmer’s failure to warn the student created a foresee-
able risk that the lightning would strike him.

Answer to Question 7

(A) The farmer should recover all of his damages against the chemical plant. To establish the element
of actual cause in a negligence action, the “but for” test is ordinarily used. However, when several
causes commingle and bring about an injury, and any one alone would have been sufficient to
cause the injury, the “but for” test is inadequate to determine actual cause. In these cases, it is
sufficient if the defendant’s conduct was a “substantial factor” in causing the injury. Here, the
farmer cannot establish that, but for the chemical plant’s negligence, he would not have suffered
damage, because either discharge was sufficient to cause his injury. However, he can establish
actual cause because the chemical plant’s negligence was a substantial factor in causing his injury;
hence, he can recover against the chemical plant. (B) is incorrect because the farmer does not
need to present evidence to apportion damages. Under joint and several liability, when two or
more tortious acts combine to proximately cause an indivisible injury to a plaintiff, each tortfeasor
is liable to the plaintiff for the entire damage incurred. Thus, he can collect all of his damages
from the chemical plant; it will be up to the chemical plant to seek apportionment of damages in
a contribution action against the steel mill. (C) is similarly incorrect. The farmer does not need to
join the steel mill in the litigation to recover. (D) is incorrect because that choice applies the “but
for” test. As discussed above, the appropriate test in this case is the “substantial factor” test.

Answer to Question 8

(D) The rancher can recover all of his damages. Where two or more tortious acts combine to proxi-
mately cause an indivisible injury to a plaintiff, each tortfeasor will be jointly and severally liable
for that injury. This is so even though each defendant acted entirely independently. Here, the
tortious acts of the two companies combined to proximately cause the poisoning of the ranch-
er’s cattle. Thus, each company will be jointly and severally liable for the entire amount of the
rancher’s damages. Thus, (D) is correct. (A) is wrong. Where two or more acts combine to cause
the injury, but none of the acts standing alone would be sufficient, each of the acts is an actual
cause of the injury (because but for either of the acts, the injury would not have occurred). Thus,
the rancher can recover because the combined actions of the defendants caused his injury. (B) is
wrong because the rancher need not show the amount of fault of each defendant to recover; they
are both joint tortfeasors. (C) is wrong because, as mentioned, in this type of case the defendants
are jointly and severally liable for the entire injury.

Answer to Question 9

(A) The defendant is jointly and severally liable for the entire amount of the plaintiff’s damages that
the plaintiff himself was not responsible for, but can obtain contribution from the third driver in
proportion to the third driver’s fault. Comparative negligence jurisdictions allow a plaintiff to
TORTS - NEGLIGENCE CAUSATION AND PARTIES ANSWERS 5.

recover some percentage of his damages even though he was contributorily negligent in causing
the accident. Pure comparative negligence allows a plaintiff some recovery no matter how great
his negligence was. On the MBE, examinees are instructed to assume that pure comparative
negligence, with joint and several liability, is the applicable rule unless stated otherwise. The
doctrine of joint and several liability makes each of several tortfeasors liable to the plaintiff for
the entire amount of damages to which the plaintiff is entitled. However, if one tortfeasor does
have to pay the plaintiff the entire judgment, the rule of contribution allows the paying tortfeasor
to recover the excess over his share from the nonpaying tortfeasor. Contribution in most compara-
tive negligence jurisdictions is based on comparative fault, whereby the nonpaying tortfeasor is
required to contribute an amount in proportion to his relative fault. Here, the plaintiff suffered
$100,000 in damages but was 30% at fault, allowing him a net recovery of $70,000—$100,000
minus 30% ($30,000). He can recover the entire $70,000 from the defendant because the juris-
diction retains the doctrine of joint and several liability. However, under a system of contribu-
tion based on comparative fault, the defendant can compel the third driver to pay him $40,000
as contribution because the third driver was 40% at fault in causing the $100,000 of damages
incurred by the plaintiff. (B) is incorrect. If the defendant did only pay $40,000 to the plaintiff, he
could compel the third driver to pay $10,000 as contribution (because the defendant’s proportional
share of the damages is only $30,000). However, because the defendant is jointly and severally
liable, the plaintiff can collect up to $70,000 from him. (C) is incorrect because the defendant is
liable for more than his proportional share of the damages under joint and several liability rules.
(D) is incorrect because a pure comparative negligence jurisdiction allows recovery no matter how
great a plaintiff’s negligence is compared with the defendant’s.

Answer to Question 10

(C) The jurisdiction has most likely abolished joint and several liability. Under joint and several
liability, each tortfeasor is liable to the plaintiff for the entire damage incurred. In the absence
of joint and several liability, each tortfeasor is liable for only the amount of damages that is
proportional to his fault. Applied to these facts, the first defendant would be liable to the plaintiff
for 60% of her damages, or $60,000. This would be offset by the plaintiff’s liability for 10% of
the first defendant’s damages, or $20,000. This leaves the plaintiff with a net award of $40,000
from the first defendant. (A) and (B) are incorrect because if joint and several liability applied,
the plaintiff would be entitled to an award of $90,000 against the first defendant (or $10,000 if
the first defendant were allowed to offset his entire amount of damages—$80,000—against the
plaintiff under joint and several liability). Note that the type of contribution rule is irrelevant to
the plaintiff’s recovery against the defendant. Contribution comes into play only after distribu-
tion of the plaintiff’s award under joint and several liability rules; it allows any tortfeasor required
to pay more than his share of damages to have a claim against the other jointly liable parties for
the excess. (D) is incorrect because if partial comparative negligence applied, the first defendant
would be able to recover nothing from the plaintiff because he was more than 50% at fault, and
so the plaintiff’s damage award against the first defendant would not be offset by any amount.
Therefore, only under pure comparative negligence can the first defendant assert a claim against
the plaintiff that would reduce the plaintiff’s award to $40,000.

Answer to Question 11

(C) The plaintiff can collect $30,000 from the defendant. In the absence of joint and several liability,
the defendant is liable only for the portion of the plaintiff’s damages for which he was respon-
sible. Joint and several liability makes each of several negligent actors liable to the plaintiff for the
entire amount of damages to which plaintiff is entitled. Because the jurisdiction has abolished this
6. TORTS - NEGLIGENCE CAUSATION AND PARTIES ANSWERS

rule, the defendant is liable to the plaintiff for only 30% of his damages ($30,000), because the
defendant was 30% at fault in causing the accident. (A) is incorrect because the partial compara-
tive negligence statute still applies. Even though it does not bar the plaintiff’s recovery, it will
still reduce his recovery corresponding to the percentage of his fault. (B) is incorrect because
the defendant is liable only for $30,000, because he was only 30% at fault in causing the plain-
tiff’s injuries and joint and several liability has been abolished. The defendant is not liable for
the injuries for which the third party is at fault. (D) is incorrect because this partial comparative
negligence jurisdiction allows a plaintiff to recover if his fault was equal to that of the defendant.
(D) probably also would be incorrect in a partial comparative negligence jurisdiction that allows
a plaintiff to recover only if he is less at fault than the defendant. Most comparative negligence
jurisdictions use a “combined comparison” approach when several tortfeasors are involved,
comparing a plaintiff’s negligence with the total negligence of all of the defendants combined.
Because here the plaintiff was only 30% at fault and the defendant and the third party together
were 70% at fault, the plaintiff could recover against the defendant even in that type of partial
comparative negligence jurisdiction.

Answer to Question 12

(D) The cousin can collect all of her damages from the friend. When two tortfeasors combine to
proximately cause an indivisible injury to a plaintiff, the tortfeasors each will be jointly and sever-
ally liable for the entire injury. This means that each is liable to the plaintiff for the entire damage
incurred. Here, the driver and his friend acted negligently and the cousin suffered injuries as a
result; thus, she can recover all of her damages from the friend. (A) is incorrect. Some states have
enacted statutes abolishing joint and several liability for pain and suffering damages; however,
there is no such statute in this case. (B) is incorrect. While the driver and the friend may not have
acted in concert (i.e., by agreement), that is not necessary to create joint and several liability for
an indivisible injury. It is enough that the negligence of the two tortfeasors combined to cause
the plaintiff’s injuries. (C) is incorrect because an injury can be considered divisible only if it
is possible to identify the portion of injury that is caused by each defendant. In this case, the
accident was a single occurrence that caused a single set of injuries to the cousin.

Answer to Question 13

(C) The strongest basis for the motorist not prevailing is the absence of a nondelegable duty. The
general rule is that a principal will not be liable for tortious acts of its agent if the agent is an
independent contractor. However, a broad exception will impose liability on the principal if
the duty is nondelegable because of public policy considerations. As long as the company was
not subject to a nondelegable duty, it would not be liable for the negligence of the excavator’s
employee in the transportation of its soil. (A) is not as good an answer as (C) because the fact
that the accident was caused by the negligence of the independent contractor’s employee does not
necessarily excuse the company from liability. (C) supplies the additional factor that enables the
company to avoid liability. (B) is incorrect because the possession of a license by the excavator
would not excuse the company from liability. (D) is incorrect because the fact that the transpor-
tation of soil was common to the area is relevant only for a strict liability action for abnormally
dangerous activities, and the transport of soil by truck is not such an activity.

Answer to Question 14

(C) The company may obtain indemnity from the employee. One who is held liable for damages
caused by another simply because of his relationship to that person may seek indemnification
TORTS - NEGLIGENCE CAUSATION AND PARTIES ANSWERS 7.

from the person whose conduct actually caused the damage. The company has been held liable
for the damages caused by the employee solely because it loaned a car to the employee. Such
vicarious liability (imposed pursuant to the permissive use statute) being imposed on the company
will entitle it to be indemnified by the employee, whose conduct actually caused the damage.
Thus, the entire loss will be shifted from the company to the employee. (A) is incorrect because
a written agreement to indemnify is not the only means by which a right of indemnification can
come into existence. For example, a right of indemnity exists as a matter of law in the circum-
stances here, as detailed above. Thus, the absence of any obligation of the employee contained in
a written agreement will not mean that the company has no rights against the employee. (B) is
incorrect because contribution is a device whereby responsibility is apportioned among those who
are at fault. Contribution allows any defendant required to pay more than his share of damages to
have a claim against any other jointly liable parties for the excess. The company is not really at
fault in bringing about the harm here. Rather, the company can only be held liable by virtue of the
permissive use statute. There is no responsibility to be apportioned here; instead, the entire loss
should be shifted from the company to the person who actually caused the damage. Thus, indem-
nity is available, and contribution is not the appropriate remedy. (D) is incorrect for the same
reasons that (B) is incorrect. In addition, (D) incorrectly provides for contribution to the extent of
the employee’s relative fault (comparative contribution). Traditional contribution rules require all
defendants to pay equal shares regardless of their respective degrees of fault.

Answer to Question 15

(A) The company probably will be vicariously liable to the pedestrian because the electrician’s devia-
tion did not take him outside the scope of the employment relationship. Under the doctrine of
respondeat superior, an employer will be vicariously liable for tortious acts committed by its
employee if the tortious acts occur within the scope of the employment relationship. What the
scope of employment is in a particular case is a question of fact determined by factors such as
the specific authorization by the employer, the employee’s motivation, and the normal routines
of the employee. Ordinarily, an employee heading home after work is no longer within the scope
of employment. Here, however, the electrician was required to be “on call” 24 hours a day, and
was required to drive the company van to his home so he would be ready to provide emergency
service whenever a call would come in. Most likely, then, the electrician was still within the scope
of his employment when he was driving the van home. The next issue is whether his deviation
from his route home took him outside the scope of his employment. Most courts today consider
the foreseeability of the deviation to be the most important factor in determining whether the
employee was still within the scope of employment or was on a “frolic” of his own. Thus, minor
deviations in time and geographic area from the employer’s business are still within the scope of
employment because they are foreseeable. Here, the electrician’s deviation of a few blocks from
his normal route home to pick up some groceries was not a substantial enough departure from his
employment purposes so as to be unforeseeable, and therefore the company can be held vicari-
ously liable for the electrician’s negligence. (B) is incorrect because the pedestrian can recover
even without showing that the company knew of the electrician’s potential for negligence. While
that might make the company independently liable for its own negligence in allowing the electri-
cian to drive a company van, the company is vicariously liable even without the assumption stated
in (B). (C) is incorrect because, as discussed above, a minor deviation of a few blocks is not
considered a “frolic” by most courts unless it is unforeseeable. (D) is incorrect even though it is a
true statement as a general rule. While an employee traveling to and from work ordinarily is not
acting within the scope of his employment, the electrician was “on call” for his employer under
the facts in this question even while he was driving home from work.
TORTS - NEGLIGENCE
DUTY AND BREACH
QUESTIONS
TORTS - NEGLIGENCE DUTY AND BREACH QUESTIONS 1.

TORTS - NEGLIGENCE DUTY AND BREACH QUESTIONS

Question 1 Question 2

A patient went to a dermatologist for treat- In response to a number of accidents involving


ment of a skin condition on his face that had pedestrians in the local business district, a city
resisted standard treatment. The dermatologist enacted a statute making it illegal for a pedes-
prescribed a new topical antibiotic cream that trian to walk through the central business district
was recommended by her associate. She gave anywhere other than on the sidewalk. The city
the patient instructions on how and when to also enacted a statute making it illegal for any
apply the cream but did not discuss potential business to obstruct the sidewalk in front of its
side effects. The patient purchased the cream at establishment. As a man was walking along
his local pharmacy and applied it as instructed. the sidewalk in the central business district, he
Shortly thereafter his skin turned a distinct discovered that a shopkeeper had stacked a pile
shade of green and he felt a strong burning of boxes in front of his establishment in such
sensation when he tried to wash it off. The color a way that it totally obstructed the sidewalk.
took almost a week to fade away, during which The man stepped into the street to walk around
time he avoided going out in public and took the obstruction of boxes and was struck by a
time off from work. cab driver driving negligently. The jurisdiction
follows traditional contributory negligence rules.
The cream was packaged with a lengthy
printed insert that detailed a number of possible If the man asserts a claim against the cab
side effects of varying degrees of probability. driver, the man’s act of leaving the sidewalk and
A “green pallor” and “irritation” were listed as walking into the street will have which of the
uncommon side effects. The patient sued the following effects?
dermatologist for prescribing the medicine and
established the above facts. He also testified that (A) It will bar the man’s recovery as a matter of
he would not have taken the medicine had he law.
been informed of all of the potential side effects.
(B) It will bar the man’s recovery unless the
If the dermatologist is found to be not liable to cab driver saw the man in time to avoid the
the patient, it will be because: impact.

(A) A reasonable person in the patient’s posi- (C) It may be considered by the trier of fact on
tion would have used the cream even when the issue of the cab driver’s liability.
told of the potential side effects.
(D) It is not relevant in determining the man’s
(B) The printed insert that came with the cream rights.
listed possible side effects similar to the
reaction the patient experienced.

(C) The severity of the reaction the patient


experienced was unforeseeable.

(D) The jurisdiction does not apply a “national”


standard of care to the dermatologist.
2. TORTS - NEGLIGENCE DUTY AND BREACH QUESTIONS

Question 3 Question 4

A 12-year-old snowboarder at a ski resort An 18-year-old had been refused a driver’s


lost control while he was going down a slope license due to a seizure disorder not controlled
and collided with a middle-aged skier who was by medication. Nevertheless, he borrowed a
ahead of him. The collision caused the skier to friend’s car for a short drive. While he was
fall hard and sustain multiple fractures of her driving, he suffered a seizure that caused him to
leg. The injuries required surgery and several lose control of the car. He struck a bicyclist with
months of rehabilitation before the skier could the car, causing him serious bodily injury.
walk again.
If the bicyclist sues the driver for negligence
If the skier prevails in a personal injury action and prevails, what will be the reason?
against the snowboarder, what is the most likely
reason? (A) The driver failed to exercise ordinary and
reasonable care under the circumstances.
(A) The skier had not signed a ski resort waiver
assuming the risk of any injuries suffered (B) The driver failed to exercise the amount of
while skiing. care that an l8-year-old of like education,
intelligence, and experience would have
(B) The severity of the skier’s injuries dictated exercised.
that an adult standard of care be applied to
the snowboarder’s activity. (C) The driver failed to exercise the ordinary
and reasonable care that a person with his
(C) A 12-year-old of like education, intelli- disability would have exercised.
gence, and experience would have avoided
the accident in the exercise of reasonable (D) The driver failed to exercise the care of a
care. reasonable person in an emergency situa-
tion.
(D) The snowboarder had used a fake pass to
gain entrance to the ski resort.
TORTS - NEGLIGENCE DUTY AND BREACH QUESTIONS 3.

Question 5 Question 6

When a hurricane suddenly increased its At the end of the season, the owner of a
intensity and changed its path to strike the private beach stacked up his rental canoes onto
oceanfront town that a tourist was visiting, a trailer, and arranged for them to be moved the
she took refuge at the hotel where she was a next day into a storage shed for the winter. That
guest. As the winds increased and the storm evening, two nine-year-old boys came onto the
surge began to cause the water to rise, the hotel owner’s property even though they knew that
manager advised everyone still in the building the lake was closed to the public for the season.
to move off of the ground floor. However, the Both of them had used the canoes (with an adult)
tourist panicked at being trapped by the water several times during the past summer. They
and decided to flee the hotel and seek higher unhooked one of the canoes from the rack, lifted
ground. She got almost to the main gate of the it down, and pushed it into the water. Although
hotel when she was swept away by the floodwa- the life vests were sitting in an open bin nearby,
ters and drowned. neither boy put one on. When they were out
in the middle of the lake with the canoe, they
In an action by the tourist’s estate against the tried to switch seats and caused the canoe to
hotel, what is the likely result? capsize. They both tried to swim to shore. One
was able to make it, but unfortunately the other
(A) The tourist’s estate will prevail because the boy could not make it and he drowned. Had he
tourist was a guest at the hotel. been wearing a life vest, he would have survived.
The boy’s parents bring a wrongful death action
(B) The tourist’s estate will prevail because the against the beach owner.
hotel manager did not prevent the tourist
from leaving. If the beach owner prevails, what will be the
likely reason?
(C) The hotel will prevail because it did not
breach any duty it owed to the tourist. (A) Children of the boy’s age, intelligence, and
education would not likely take the canoe
(D) The hotel will prevail because the hurricane out without a life vest.
was an unforeseeable act of God.
(B) The owner took precautions to make the
canoes inaccessible.

(C) The boy appreciated the risk of taking the


canoe out onto the lake without a life vest.

(D) The boy was not lured onto the owner’s


property by the canoes.
4. TORTS - NEGLIGENCE DUTY AND BREACH QUESTIONS

Question 7 Question 8

An only daughter whose father died after a A mountain lodge catered to cross-country
long illness arranged with a mortuary to bury skiing enthusiasts. Every winter, the owner of
him next to his wife’s grave. The daughter the lodge, along with his assistants, marked
selected the most ornate casket available because and groomed trails on his extensive lands. He
she wanted it to remain closed during the wake, charged a fee for a day’s use of the trails. He
and she could not bring herself to view the body also supplied equipment for those who needed
before the funeral. At the cemetery, however, it and provided lodging and meals for skiers
she decided to view the body just before it staying more than one day. A woman on her first
was buried. She was horrified to discover that cross-country skiing outing was striding along a
the body in the casket was dressed in a clown groomed trail when, with a loud crack, the rotten
costume and a bright orange wig. In fact, it limb of a huge oak under which the trail passed
was not her father but a popular circus enter- separated from the tree because of the load of ice
tainer who had died the same day as her father and snow it bore. It came crashing down on top
and had requested to be buried in his costume. of the woman, killing her instantly. The woman’s
Although the mortuary was able to retrieve heirs brought a wrongful death action against the
her father’s body and bury it, the daughter was lodge owner in a jurisdiction applying traditional
greatly distressed by the episode and suffered rules for landowners and possessors of land.
nightmares as a result. However, she did not seek
medical or psychiatric care because of it. The If the lodge owner prevails, what is the most
mortuary apologized for its error in switching likely explanation?
the bodies, but insisted that the daughter pay all
of the agreed-to charges for the funeral. (A) The oak limb was a natural condition of the
land.
If the daughter brings action against the
mortuary to recover for her emotional distress, (B) The lodge owner did not know that the oak
can she recover damages? limb was rotten and was likely to fall.

(A) No, because the daughter did not have to (C) The lodge owner could not reasonably have
obtain medical or psychiatric care. discovered the rotten limb by inspecting his
property.
(B) No, because the daughter suffered no
physical injury. (D) The ice and snow was an intervening force
that caused the limb to fall.
(C) Yes, because of the known sensitivity of
people concerning the death of a family
member.

(D) Yes, because the mortuary is requiring


the daughter to pay the bill for the funeral
expenses.
TORTS - NEGLIGENCE DUTY AND BREACH QUESTIONS 5.

Question 9 Question 10

A salesman was staying at a hotel for a A homeowner invited guests over for an
convention. Hotels in the area near the conven- evening pool party. One of the guests jumped
tion had been experiencing a series of robberies into the pool without seeing that a swimmer
where the robber would break into the locked was in the water right below him. He struck
hotel room and then rob his victim by threat- the swimmer on the head, knocking her uncon-
ening the victim with a gun. In response, the scious. Other guests pulled the swimmer out
hotel where the salesman was staying had of the water and attempted to revive her. She
recently re-inspected its rooms to ensure that its survived but suffered permanent injury.
hotel room locks were secure and in compliance
with state requirements. Nevertheless, one night The swimmer brought an action against the
during the convention, the robber broke into homeowner for her injuries in a jurisdiction
the salesman’s room in the same manner as his that applies the traditional liability rules for
other robberies. Instead of threatening to shoot, landowners and possessors of land. At trial,
however, the robber hit the salesman over the evidence established that when the guest jumped
head with a blackjack, seriously injuring him. into the pool, he did not see the swimmer in
part because a light in the pool near the point
If the salesman sues the hotel for his injuries of impact was not working at the time, but
and does not prevail, what is the likely reason? the homeowner was not aware that it was not
working. No other evidence was presented
(A) The hotel’s management reasonably be- regarding the condition of the pool. Other
lieved that the hotel room locks were evidence established that the rescue efforts of
adequate. the other guests caused greater injury to the
swimmer than the initial impact. At the end of
(B) The hotel is not vicariously liable for the the presentation of the evidence, the homeowner
intentional torts of unrelated third parties. moved for a directed verdict.
(C) The use of force by the robber was unfore- Should the homeowner’s motion be granted?
seeable and therefore a superseding inter-
vening cause. (A) Yes, because the swimmer’s injuries were
caused by the conduct of third persons.
(D) The hotel was in compliance with a state
statute setting minimum standards for hotel (B) Yes, because the homeowner was not aware
room locks. that the light in the pool was not working.

(C) No, because the jury could determine that


the homeowner failed to exercise reason-
able care in activities on the property.

(D) No, because the jury could find that any


negligence on the part of the rescuers was
foreseeable and therefore does not cut off
the homeowner’s liability.
6. TORTS - NEGLIGENCE DUTY AND BREACH QUESTIONS

Question 11 Question 12

A young boy was hospitalized after he A landowner’s piece of property was located
accidentally swallowed a coin while playing. The on the corner of a busy intersection. People
boy recovered, but his mother began suffering walking past the intersection often cut across
feelings of anxiety, worrying that her child might the landowner’s property rather than walk along
be injured again. The mother went to a movie to the sidewalk, despite a “keep off” sign that the
relax and ordered some popcorn. While handling landowner erected. The landowner had captured
her change, the concession stand attendant a young bobcat several months earlier and
carelessly dropped a quarter into the popcorn, had decided to keep him as a pet. In an effort
which the mother did not notice until she bit the to stop people from cutting across his yard,
quarter. She did not swallow the quarter, nor did the landowner erected a large sign that read,
she bite on it hard enough to injure her teeth or “Beware of Bobcat.” Knowing that a bobcat
gums, but the feel of the coin brought back all attacks using its claws, he took it to a vet to have
her anxieties about the boy’s injury. The mother its claws removed. The vet failed to remove one
sued the movie theater for her emotional distress. of the bobcat’s claws, but the landowner was not
aware of this. The landowner kept the bobcat
If the jury finds that a reasonable person on a leash in the front yard. The leash was long
would not have suffered similar distress under enough to allow the bobcat access to most of the
the circumstances, is the mother likely to landowner’s property; however, the bobcat could
prevail? not reach the sidewalk that ran adjacent to the
landowner’s property. The next day, a woman
(A) Yes, because the concession stand attendant was walking home from the market. When
was negligent. she came to the intersection, she cut across the
landowner’s property rather than staying on the
(B) Yes, because she suffered emotional sidewalk. The bobcat charged at the woman and
distress. slashed her legs severely with its one remaining
claw.
(C) No, because a reasonable person would not
have suffered similar distress under the If the woman brings suit against the
circumstances. landowner for her injuries caused by the bobcat
and establishes that she did not see the warning
(D) No, because she suffered no actionable sign, the woman will:
harm.
(A) Recover, because the bobcat was not a
domesticated animal.

(B) Recover, because the woman was not aware


of the sign on the landowner’s property.

(C) Not recover, because the woman was a


trespasser.

(D) Not recover, because the vet was the cause


of the injury.
TORTS - NEGLIGENCE DUTY AND BREACH QUESTIONS 7.

Question 13 Question 14

A shopping center contracted with a security A rancher owned a large parcel of vacant land
service to provide nighttime monitoring of mall that motorcyclists frequently used for impromptu
property. An employee of the security service, motorcross competitions on weekends. The
without the manager’s permission, began placing rancher was aware of this use of his land but had
several of his pet rattlesnakes in the shopping no immediate plans for the parcel and so took no
center at night as a deterrent to burglars, action against the motorcyclists. One weekend,
and posted signs at night saying, “Beware of the rancher drove out to the parcel to do some
poisonous snakes.” target shooting. He was shooting when a rider
arrived at the area with his bike and began
The shopping center was conducting a contest riding on the trails. The rider, who was intoxi-
to guess the number of jelly beans that filled a cated, tried a particularly dangerous stunt on his
car. A student at a local college broke into the bike just as one of the rancher’s shots struck a
shopping center to take a sample of the jelly rock and ricocheted. The shot passed through
beans in a given volume of the car’s interior so the rider’s leg, severing an artery. Fortunately,
that, using the manufacturer’s figures for interior prompt medical attention saved his life.
volume, he could calculate the number of beans
in the car. As the student was carefully opening If the rider brings an action for negligence
a vent window to take his sample of jelly beans, against the rancher, he will probably:
one of the security service employee’s rattlers
that had been lying under the car bit him on (A) Recover all of his damages.
the ankle. The student, who had not seen the
warning signs about snakes, was hospitalized for (B) Recover a portion of his damages, because
two weeks and had to miss a semester of school he was negligent in driving his motorcycle
as a result of the snakebite. while drunk and attempting the dangerous
stunt.
If the student brings an action against the
shopping center, will he likely recover? (C) Not recover, because driving while drunk is
gross negligence or worse.
(A) No, because he was a trespasser on shop-
ping center property. (D) Not recover, because he was a trespasser on
the rancher’s land.
(B) No, because he was guilty of breaking and
entering.

(C) Yes, because the use of poisonous snakes


for security amounted to unreasonable
force.

(D) Yes, because he had not seen the signs


warning of the poisonous snakes.
8. TORTS - NEGLIGENCE DUTY AND BREACH QUESTIONS

Question 15 Question 16

A swimmer went to a privately owned lake A small print shop on the second floor of an
resort whose owner charged a fee for admis- older two-story industrial building kept its front
sion. The beach had a roped-in swimming area door locked for security reasons, so most visitors
and large signs directing swimmers not to swim gained access to the shop by using the shop’s
anywhere but within the ropes. The lifeguards freight elevator in the rear of the building. This
regularly enforced this rule. The resort also elevator did not have a call-button for use by the
rented canoes and rowboats to its patrons, who public, so anyone wanting to use it had to wait
could take them anywhere on the lake. The for one of the shop’s employees to send down
swimmer and two of his friends had rented the elevator. A city fire inspector inspecting the
a canoe and started to paddle out toward the building informed the shop foreman that he was
other side of the lake when the swimmer saw going to inspect the lower floor, the trash area,
a volleyball game starting on the beach that he and the elevator shaft. The foreman said that he
wanted to join. He left his friends in the canoe would turn the elevator off until the inspector
and started swimming to shore. He was only called up from downstairs that he was finished
a few yards outside of the roped-in swimming with his inspection. About 10 minutes later, as
area when he started, but he angled away from the inspector was under the elevator in the shaft,
the swimming area toward the area of the beach the freight elevator suddenly started descending.
where the volleyball net was set up. Although the The inspector tried to get out of the way, but his
lifeguard on duty saw him, she did not warn him leg was severed by the elevator.
to return to the swimming area. When the depth
of the water was about four feet, he put his foot The inspector sued the print shop for the loss
down and was severely cut by the jagged edge of of his leg. At the time of the accident, there
a rusted metal stake protruding a few inches out were three employees in the shop, but all deny
of the bottom of the lake. The swimmer had not that they pushed the button that would send
seen the stake even though the water was clear the elevator to the first floor. Evidence at trial
and it was visible if he had looked down. indicated that the foreman had turned the switch
to “off” as he agreed to do. The inspector’s
If the swimmer sues the resort in a jurisdiction expert testified that there was nothing wrong
that applies the traditional rules for landowners with the elevator that would cause it to malfunc-
and possessors of land, is he likely to recover? tion and operate when it was turned off.

(A) No, because the stake could have been seen Is the inspector likely to prevail?
by the swimmer.
(A) No, because the evidence indicated that the
(B) No, because he was swimming outside of foreman had turned the switch to “off.”
the roped-in area.
(B) No, because, as a fire inspector, the
(C) Yes, because the lifeguard on duty saw inspector was merely a licensee and the
him and did not warn him to return to the foreman took steps to protect him from
swimming area. injury.

(D) Yes, because he is a public invitee of the (C) Yes, because the foreman must have
resort. forgotten to turn the switch to “off.”

(D) Yes, because the printing shop’s elevator


could not be controlled from the first floor.
TORTS - NEGLIGENCE DUTY AND BREACH QUESTIONS 9.

Question 17 Question 18

A nursing home, as a matter of policy, The class president invited his class to his
admitted patients known for violent behavior. home to celebrate homecoming. When the
Each night before the patients went to bed, all sun began to set, a student built a bonfire in
doors were locked so that intruders could not the backyard. The student continued to feed
enter the premises without forcing an entry. One the flames until the bonfire was quite large.
morning an elderly and docile patient was found Suddenly, a gust of wind blew the flames to a
beaten in her bed. The doors had been locked, neighboring property, igniting the neighbor’s
and there were no signs of forced entry into the shed.
building during the previous night.
If the neighbor sues the student for the
The patient sued the nursing home for her damage to his shed on a theory of negligence,
injuries and established the above facts. At the under which of the following arguments, if
close of the patient’s case, the nursing home sustained by the facts, would the student most
moved for a directed verdict. likely be able to avoid liability?

How should the court rule? (A) The lighting of bonfires on homecoming is
an accepted custom in the community.
(A) Deny the motion, because the jury could
find that an occurrence such as this would (B) The bonfire was positioned by the student
not ordinarily happen unless the nursing in the center of the backyard to avoid
home was negligent. harming neighboring property.

(B) Deny the motion, because the nursing home (C) The fire that started would have burned
is required to exercise a very high degree of itself out but for the fact that the neighbor’s
care towards the patients in its care. shed was built out of substandard, highly
flammable material.
(C) Grant the motion, because the patient did
not establish that the beating was caused by (D) The student was a guest on the class presi-
an employee of the nursing home. dent’s property and entitled to the same
restricted scope of liability as the property
(D) Grant the motion, because the beating was owner.
caused by the criminal act of a third party.
TORTS - NEGLIGENCE
DUTY AND BREACH
ANSWERS
TORTS - NEGLIGENCE DUTY AND BREACH ANSWERS 1.

TORTS - NEGLIGENCE DUTY AND BREACH ANSWERS

Answer to Question 1

(A) If the patient does not prevail, it will be because a reasonable person in the patient’s position
would have used the cream even after being told of the potential side effects. As part of the physi-
cian’s duty of care, a doctor proposing a course of treatment has a duty to provide the patient with
enough information about its risks to enable the patient to make an informed consent to the treat-
ment. If an undisclosed risk was serious enough that a reasonable person in the patient’s position
would have withheld consent to the treatment, the doctor has breached her duty. Here, whether the
dermatologist breached her duty to the patient by not warning him of possible side effects would
be determined ultimately by the trier of fact using a reasonable person standard. A determina-
tion that a reasonable person in the patient’s position would have consented to using the cream
even after being informed of the potential side effects would preclude the patient from prevailing.
Answer (B) is incorrect. It is not as good a choice as (A) because the listing of side effects on a
lengthy and detailed printed insert does not necessarily satisfy the dermatologist’s duty to provide
enough information for the patient to make an informed consent. Information that is buried in a
lengthy insert may not be read through by a reasonable person, so the dermatologist may have had
a duty to verbally disclose side effects, and she will have breached that duty if a reasonable person
in the patient’s position would have withheld consent to the treatment if given the information.
Answer (C) is incorrect because the dermatologist could be liable regardless of the severity of the
reaction the patient experienced. If she should have warned about the particular side effects the
patient experienced even though they are ordinarily less severe, the fact that the patient suffered
an unforeseeably severe reaction to the medicine does not relieve her from liability. Answer (D) is
incorrect. Even if the jurisdiction requires only that the dermatologist exercise the knowledge and
skill of a physician in good standing in similar communities, rather than apply a national standard
most jurisdictions use, that does not establish whether the duty was breached here. The duty
issue here is whether she provided the patient with enough information to allow him to make an
informed consent, for which the factor in (A) is determinative.

Answer to Question 2

(C) The man’s conduct may be considered by the trier of fact. Whether the cab driver is liable
depends on whether the man can establish a prima facie negligence case; i.e., whether the cab
driver breached a duty to him and thereby was the actual and proximate cause of his injuries. The
general rule is that a duty of care is owed only to foreseeable plaintiffs. Here, while the cab driver
was driving negligently, the facts do not indicate what his negligent conduct was. More facts are
needed to determine whether the man was in the foreseeable zone of danger from the cab driver's
negligence. Because the man was not where the cab driver might have expected him to be, it
would be relevant in judging the reasonableness of the cab driver’s conduct. For this reason, (D)
is wrong. (A) and (B) are wrong because the man’s violation would probably be excused under
the circumstances. A plaintiff’s contributory negligence may be established by his violation of a
statute under the same rules that govern whether a statute can establish a defendant’s negligence.
Ordinarily, the violation of an applicable statute establishes that the plaintiff was contributorily
negligent as a matter of law. However, violation of a statute will be excused when compliance
with the statute is not possible. Here, the trier of fact could determine that the obstruction excused
compliance with the statute barring walking in the street. While (B) raises the issue of last clear
chance, which would negate a contributory negligence defense, the facts do not establish that the
man was contributorily negligent, as discussed above.
2. TORTS - NEGLIGENCE DUTY AND BREACH ANSWERS

Answer to Question 3

(C) The skier will prevail if a 12-year-old snowboarder acting reasonably would have been able to
avoid the accident. A child is required to conform to the standard of care of a child of like age,
education, intelligence, and experience in the usual case. This permits a subjective evaluation of
whether that particular defendant acted reasonably under the circumstances. If the snowboarder
acted unreasonably under this standard, the skier will recover. (A) is incorrect because even
if a waiver had been signed by the skier, it would allow only the ski resort to raise the defense
of express assumption of risk if sued; the snowboarder could not rely on a waiver to which he
was not a party. Thus, the skier’s failure to sign a waiver is irrelevant. (B) is wrong because the
severity of the skier’s injuries do not dictate whether an adult standard of care applies. Courts
will require children to conform to an adult standard of care when they are engaged in activities
that normally only adults engage in, such as driving an automobile. Snowboarding would not be
considered that type of activity, so an adult standard of care would not apply regardless of the
severity of the skier’s injuries. (D) is incorrect because the fact that the snowboarder had used
a fake pass to gain entrance to the resort does not indicate whether he was complying with the
applicable standard of care when he struck the skier, which is the critical question here.

Answer to Question 4

(C) If the bicyclist prevails it will be because the driver failed to exercise ordinary care, given his
physical disability. In a lawsuit based on negligence, the usual standard of care is ordinary and
reasonable care under the circumstances. The standard changes when the defendant has a major
physical disability such as a seizure disorder. In that situation, the standard becomes ordinary and
reasonable care for a person with that disability, which in this case would have been to refrain
from driving. Therefore, (C) is a more accurate answer than (A). (B) is wrong. It correctly states
the standard to be applied to children, but that standard would not be applied to an 18-year-old car
driver. (D) is incorrect. The existence of an emergency ordinarily may be considered in deter-
mining whether the defendant acted as a reasonable person under the circumstances, but not if
the emergency was of the defendant’s own making. Regardless of how he acted once his seizure
started, his conduct in driving the car knowing he was subject to seizures would be the breach of
duty here.

Answer to Question 5

(C) The hotel will prevail because it did not breach any duty owed to the tourist. The tourist’s estate
may claim that the hotel owed the tourist an innkeeper’s high duty of care to its guests or a land
occupier’s duty to make the premises safe for invitees, and breached that duty when the tourist
drowned on the hotel’s property. However, even though the tourist was a guest of the hotel and an
invitee of the hotel, the facts do not establish any breach of duty by the hotel. The hotel manager
advised those still in the hotel to move to a higher floor, which may have been the only reason-
able course of action under the circumstances, and the danger of leaving the building should have
been obvious to the tourist. Hence, the tourist’s estate will not prevail. (A) is incorrect because,
as discussed above, the hotel did not breach its innkeeper’s high duty of care. (B) is incorrect
because the hotel had neither the duty nor the right to prevent the tourist from leaving, even
though it was an unwise decision. (D) is incorrect because the hotel’s liability does not turn on
whether the hurricane was foreseeable. Even if the hurricane were foreseeable, nothing in the facts
indicates that the hotel breached its duty to the tourist.
TORTS - NEGLIGENCE DUTY AND BREACH ANSWERS 3.

Answer to Question 6

(C) If the beach owner is not liable to the boy’s parents, it will be because the boy appreciated the risk
of using a canoe without a life vest. A landowner owes a higher duty of care to a child trespasser
than to an adult trespasser. Under the “attractive nuisance” doctrine, a landowner has a duty to
exercise ordinary care to avoid reasonably foreseeable risk of harm to children caused by artificial
conditions on his property. To assess this special duty on the landowner, the following elements
must be shown: (i) there is a dangerous condition on the land of which the owner is or should be
aware; (ii) the owner knows or should know that children frequent the vicinity of this dangerous
condition; (iii) the condition is dangerous because the child is unable to appreciate the risk; and
(iv) the expense of remedying the situation is slight compared with the magnitude of the risk.
The third element would be negated by showing that the boy appreciated the risk of taking the
canoe out onto the lake without a life vest. In that case, the attractive nuisance doctrine would not
apply and the boy would be treated like an adult trespasser, and leaving the canoes out would not
constitute a breach of the owner’s limited duties to adult trespassers. (A) is incorrect even though
showing that children of like age, etc., would likely use a life vest is some evidence that the
canoes might not be an attractive nuisance. However, (C) is the stronger answer because it focuses
on the boy’s own appreciation of the risk. If he had sufficient familiarity with the canoes to
appreciate the risk of not using a life vest, the condition would not be an attractive nuisance as to
him, as discussed above. (B) is incorrect because the precautions that the owner took might not be
sufficient to avoid liability under the attractive nuisance doctrine. The owner would have to show
that the expense of taking further precautions to remedy the situation was so great as to outweigh
the magnitude of the risk. (D) is incorrect because most jurisdictions do not require a showing
that the child was lured onto the property by the dangerous condition. Foreseeability of harm is
the true basis of liability; the element of attraction is important only insofar as it indicates that the
presence of children should have been anticipated by the landowner. Hence, it is not relevant that
the boy was not lured onto the property by the canoes.

Answer to Question 7

(C) The daughter can recover damages for her emotional distress, even though she suffered no
physical injury and did not require medical care, because of the known sensitivity of people
concerning the death of a family member. In the usual case, the duty to avoid negligent inflic-
tion of emotional distress is breached when defendant creates a foreseeable risk of physical injury
to plaintiff, typically by causing a threat of physical impact that leads to emotional distress,
and plaintiff can recover for physical injury caused solely by the distress. In special situations,
however, courts have permitted plaintiff to recover in the absence of physical symptoms where
defendant’s negligence creates a great likelihood of severe emotional distress. One of these situa-
tions is the mishandling of a relative’s corpse, because it is certainly foreseeable that a person will
suffer severe emotional distress if the corpse of a family member is negligently mishandled. In
this case, the mortuary was negligent in putting the wrong body in the casket, creating a foresee-
able risk of severe emotional distress to the daughter under the circumstances. Despite the fact
that she suffered no physical injury, she can recover damages from the mortuary. (C) is therefore
correct and (B) is incorrect. (A) is incorrect because the fact that the daughter did not obtain
medical or psychiatric care does not prevent her from recovering damages. She can establish proof
of her emotional distress through her own testimony and the testimony of others. (D) is incorrect
because the fact that the mortuary is charging the daughter for the funeral is irrelevant. Regard-
less of the mortuary’s conduct regarding the bill, its negligence in handling the body of her father
makes it liable for her emotional distress.
4. TORTS - NEGLIGENCE DUTY AND BREACH ANSWERS

Answer to Question 8

(C) If the lodge owner prevails, it will be because he made a reasonable inspection without discov-
ering the danger. In a jurisdiction applying traditional rules for landowners and possessors of
land, a landowner owes a duty to invitees to warn of or otherwise make safe dangerous natural
conditions of which he knows or could reasonably discover by inspection. (A) is wrong because
the fact that the tree limb was a natural condition does not relieve the lodge owner of this duty.
(B) is wrong because actual knowledge is not necessary; it is sufficient to impose liability if the
dangerous natural condition would have been discovered by a reasonable inspection. (D) is wrong
because the fact that other factors contributed to the danger does not negate the landowner’s
liability because those factors were foreseeable.

Answer to Question 9

(A) If the salesman does not prevail, it will be because the hotel’s management did not breach its
duty of care. Innkeepers have a duty to use a high degree of care to aid or assist their guests and
to prevent injury to them from third persons. Included in this duty is the duty to take reasonable
precautions against foreseeable criminal acts of third parties. As stated in (A), the fact that the
hotel’s management had reason to believe that the locks were adequate after inspection is a basis
for finding that the hotel has satisfied its duty of care. (B) is incorrect because any liability on the
part of the hotel would not be vicarious liability, but rather direct liability for its failure to take
security precautions. (C) is incorrect because the robbery would be a superseding intervening
force only if it was unforeseeable. Given the string of robberies, the fact that this robber used a
blackjack does not make the crime a superseding force. If the hotel’s management had reason to
believe the locks were inadequate, any criminal conduct by the robber would likely be deemed to
be a foreseeable risk of the inadequate locks. (D) is incorrect because compliance with the statute
is not determinative. The statute sets forth a minimum standard of conduct regarding installation
of locks in hotel rooms. Despite compliance with this statute, a careful hotel proprietor might be
required to take additional precautions to satisfy the duty of care under the circumstances.

Answer to Question 10

(B) The homeowner’s motion should be granted because the homeowner was not aware that the light
was not working and there is no other evidence of negligence on his part. The swimmer, a social
guest, had the status of a licensee on the homeowner’s property. In a jurisdiction that applies
the traditional rules for landowners and possessors of land, an owner or occupier of land owes
licensees a duty to warn of or make safe a dangerous condition known to the owner or occupier
that creates an unreasonable risk of harm to the licensee and that the licensee is unlikely to
discover. The owner has no duty to inspect for defects. Here, the only potentially dangerous condi-
tion on the premises was the pool light not working, but the homeowner was not aware of that.
The swimmer has presented no other evidence of breach of duty on the homeowner’s part; hence,
the homeowner’s motion should be granted. (A) is incorrect because the fact that the swimmer’s
injuries were caused by third persons does not establish that the landowner was not liable. If the
landowner were aware of a dangerous condition on the property that was a cause of the injury, the
fact that third persons were also a cause of the injury would not cut off his liability. (C) is incor-
rect because there is no evidence that the landowner failed to meet his duty to exercise reason-
able care in activities on the property. The only evidence pertaining to the homeowner’s duty was
the pool light not working, which is a condition on the land rather than an activity. Because the
homeowner was not aware of the condition, he has not been shown to have breached a duty to his
guest. (D) is incorrect even though negligence of rescuers is often foreseeable. The swimmer has
TORTS - NEGLIGENCE DUTY AND BREACH ANSWERS 5.

not established that the homeowner breached a duty, so the homeowner has no liability regardless
of the conduct of the rescuers.

Answer to Question 11

(D) The mother will lose because she will not be able to establish a prima facie case for negligent
infliction of emotional distress. An action for negligent infliction of emotional distress requires the
plaintiff to show that the defendant’s conduct created a foreseeable risk of physical injury to the
plaintiff, such as by threat of physical impact, and, in the usual case, that the emotional distress
caused by this conduct also manifests itself in physical symptoms. Here, the first element is estab-
lished because the negligence of the theater employee created a foreseeable risk that the mother
would be injured by biting down on the coin, but the second element is not established because
the mother’s distress does not amount to an actionable physical injury, and she suffered no other
injury by biting the coin. While many courts do not require physical symptoms if the defendant's
negligence creates a great likelihood of substantial emotional distress, these facts do not rise to
that level. Note that if she had suffered physical injury by biting the coin, she would have a prima
facie case against the theater for negligence. She then would have been able to recover for her
emotional distress as an additional element of her noneconomic damages. (A) is incorrect because
the concession stand attendant’s negligence only establishes the first element of the case for negli-
gent infliction of emotional distress; the element of physical symptoms, which most courts would
require under these facts, is not established. (B) is incorrect because emotional distress alone is
generally not a sufficient injury where the defendant’s conduct was only negligent. If the conces-
sion stand attendant instead had purposely put the coin in the popcorn with knowledge or reckless
disregard of the likely effect of his conduct, the mother’s emotional distress probably would be
sufficient for establishing a prima facie case of intentional infliction of emotional distress. (C) is
incorrect because the element of emotional distress in this tort does not depend on foreseeability
or a reasonable person standard; only the initial threat of physical injury need be foreseeable. Had
the mother otherwise established a prima facie case by proving physical symptoms, the fact that a
reasonable person would not have suffered similar distress is irrelevant.

Answer to Question 12

(C) The woman will not recover, because she was a trespasser on the landowner’s land. The general
rule is that one who possesses an animal not customarily domesticated in that area is strictly
liable for all harm done by the animal as a result of its harmful or dangerous characteristics. For
trespassers, however, strict liability is not imposed against landowners. Trespassers cannot recover
for injuries inflicted by the landowner’s wild animals in the absence of negligence, such as where
the landowner knows that trespassers are on the land and fails to warn them of the animal. Under
this standard, even though the landowner could anticipate that trespassers like the woman would
cross his property, he will not be liable because he exercised reasonable care by posting a sign
warning about the bobcat and by attempting to make the animal less dangerous. (A) is incor-
rect because, as discussed above, the landowner is not strictly liable to the woman, a trespasser.
(B) is incorrect because the woman cannot recover regardless of whether she saw the sign. With
regard to the landowner’s exercise of reasonable care, it does not matter that the woman did not
see the sign as long as the sign was noticeable enough that it could have been seen. (D) is incor-
rect because the vet’s negligent conduct is the type of foreseeable intervening cause that would not
supersede any fault on the landowner’s part. If the landowner were negligent, he could be jointly
liable with the vet for the woman’s injuries.
6. TORTS - NEGLIGENCE DUTY AND BREACH ANSWERS

Answer to Question 13

(A) The student’s status as a trespasser on shopping center property precludes his recovery against
the shopping center. This question does not identify the basis for the student’s action against the
shopping center; therefore, you have to consider all of the shopping center’s potential grounds for
liability. Because the security service is an independent contractor, something more than simple
vicarious liability is needed to make the shopping center liable for the security service employee’s
torts. And because the shopping center did not authorize the use of snakes to guard the stores, it
is not liable for an intentional tort, even though the use of poisonous snakes would otherwise be
unreasonable force in defense of property. Another potential basis for liability is the shopping
center’s status as owner or occupier of the property. The general rule is that a landowner owes no
duty to a trespasser whose presence is undiscovered to warn of or make safe concealed dangerous
conditions or dangerous activities on the land. However, trespassers who are discovered or whom
the landowners should anticipate because they habitually and routinely enter the land are owed a
higher duty. The landowner will be liable for concealed, dangerous, artificial conditions known to
the landowner and negligently conducted active operations, including those of third persons whom
the landowner has the ability and authority to control. Here, the student was neither discovered
nor anticipated. No one actually noticed him before his injury, and nothing in the facts indicates
that people habitually broke into the mall—the mere fact that the shopping center had a security
guard is not sufficient to make someone breaking in an anticipated trespasser. Thus, even if the
shopping center should have supervised the security service employee more closely, it owed no
duty to the student because he was a trespasser. (B) is incorrect because the fact that the student
was guilty of breaking and entering is irrelevant. Had the student been a discovered or antici-
pated trespasser, the shopping center might be liable to the student for, e.g., negligent supervision
of the security service, even though the student broke into the building. (C) is incorrect because
the shopping center did not authorize use of the poisonous snakes. If it had, (C) would be correct
because a landowner may only use reasonable force to defend his property, and may not use
indirect force that will cause death or serious bodily harm, such as a spring gun or deadly animal,
when he could not use such force directly, such as here. (D) is incorrect because it is irrelevant.
Undiscovered trespassers need not be warned of the dangers on the land; the landowner owes no
duty to them.

Answer to Question 14

(A) The rider probably will recover all of his damages. The rancher owed the rider a duty of reason-
able care in conducting activities on his land because the rancher was (or should have been) aware
of the rider’s presence, making the rider a discovered (or anticipated) trespasser. Hence, (D) is
incorrect. The rancher breached this duty of care by shooting while the rider was in the vicinity,
and this breach was an actual and proximate cause of the rider’s injury. (B) and (C) are incor-
rect because the rider’s negligence, even if it was gross negligence, was not a substantial factor in
causing his injuries because it did not create a foreseeable risk of being shot by the rancher (i.e., a
sober, careful rider who was in the same place at the same time would still have been shot). The
rider’s damages stem just from the bullet piercing the artery in his leg, which does not appear to
be attributable to his negligence. Thus, his recovery from the rancher would be total.

Answer to Question 15

(B) The swimmer cannot recover from the resort because he did not have invitee status when he
was injured. In jurisdictions following the traditional rules for landowners and possessors of
land, the nature of a duty of an owner or occupier of land to those on the premises depends on
TORTS - NEGLIGENCE DUTY AND BREACH ANSWERS 7.

the legal status of the plaintiff in regard to the property, i.e., whether the plaintiff is a trespasser,
licensee, or invitee. An invitee is a person who enters onto the premises in response to an express
or implied invitation of the landowner. Those who enter as members of the public for a purpose
for which the land is held open to the public and those who enter for a purpose connected with
the business or other interests of the landowner are considered invitees. However, a person will
lose his status as an invitee if he exceeds the scope of the invitation—if he goes onto a portion of
the property where his invitation cannot reasonably be said to extend. Here, the swimmer was an
invitee of the resort in the areas to which it allowed its patrons to go. However, the resort clearly
identified the boundaries of the area held open to swimmers, and the swimmer could not reason-
ably have believed that he was invited to swim in the area where he was injured. Because the
swimmer was at most a licensee when he was injured, the resort did not owe him a duty to make
reasonable inspections of that area to discover dangerous conditions and make them safe. At
most, the resort had a duty only to warn the swimmer of known dangerous conditions that create
an unreasonable risk of harm to him and that he is unlikely to discover, and nothing in the facts
indicates that any employees of the resort knew of the stake under the water. The swimmer there-
fore cannot recover against the resort. (A) is not as good a choice as (B). While a landowner is not
liable for a dangerous condition that is obvious to the entrant on the land, the fact that the stake
was visible does not establish that it was obvious, given that the swimmer was looking forward
rather than down. Whether a danger is obvious is determined by all of the surrounding circum-
stances, not just whether the danger is visible. The better reason why the swimmer cannot recover
is because he was no longer an invitee. (C) is incorrect because the lifeguard’s failure to direct the
swimmer to the swimming area would not constitute an invitation to swim in the restricted area;
at most, it would establish only that the swimmer was a licensee rather than a trespasser when he
swam in that area. A licensee is one who enters onto land with the possessor’s permission, express
or implied, for his own purpose or business rather than for the possessor’s benefit. The lifeguard’s
conduct may have constituted implied permission for the swimmer to exit the lake in a nonswim-
ming area for his own benefit, but it does not establish that he reasonably believed that he was
invited to swim in that area. (D) is incorrect because the swimmer lost his status as an invitee
when he exceeded the scope of his invitation by swimming in an area where swimming was not
permitted.

Answer to Question 16

(D) The inspector will likely prevail. Under the doctrine of res ipsa loquitur, the trier of fact is
permitted to infer the defendant’s breach of duty when the facts strongly indicate that the plain-
tiff’s injuries resulted from the defendant’s negligence. The facts here indicate that, because the
elevator was not controllable from the first floor, the elevator would not have descended unless
someone from the second floor changed it to “on” and pushed the down button. The fact that
someone started the elevator while the inspector was in the shaft is circumstantial evidence that
either the foreman was negligent (e.g., by failing to apprise his co-workers of the inspector’s
presence in the shaft) or one of his co-workers was negligent (e.g., by forgetting the foreman’s
warning and turning the elevator back on). Thus, under the doctrine of res ipsa loquitur, the
inspector would be able to show that, through respondeat superior, the print shop breached a duty
owing to him. Thus, (A) is incorrect. (B) is incorrect because the inspector’s status is irrelevant.
A possessor of land owes a duty of reasonable care in the exercise of all active operations on the
property, regardless of whether the plaintiff was a licensee or invitee. Thus, regardless of the
inspector’s status, the print shop owed a duty of reasonable care in its employees’ operation of the
elevator. (C) is incorrect because it may have been another employee who was negligent.
8. TORTS - NEGLIGENCE DUTY AND BREACH ANSWERS

Answer to Question 17

(A) The court should deny the motion because the nursing home could be found liable under the
doctrine of res ipsa loquitur. This doctrine would apply because the injury ordinarily would not
have occurred in the absence of negligence on the part of the nursing home, such as by accepting
violent patients without providing adequate security for other patients. The nursing home has
assumed the duty of ordinary care for the safety and security of its patients. Because the doors
were locked, the injury occurred as a result of either a nursing home employee or a patient,
and the nursing home would be liable in either case. Admitting patients known for their violent
behavior created a foreseeable risk of injury to all the patients, and the exercise of ordinary
care would require that the nursing home provide adequate security to protect the patients. The
patient’s beating establishes enough evidence of the breach of duty by the nursing home here to
withstand the motion and send the case to the jury. (B) is incorrect. There is no rule of law that
imposes a high degree of care in this situation—just ordinary care under the circumstances. Thus,
the nursing home is only liable because it has breached its duty of ordinary care for the safety and
security of the patients. (C) is incorrect. The patient need not prove that the beating was caused
by an employee in order to impose liability on the nursing home, because violence by another
patient was foreseeable. The nursing home has assumed the duty of ordinary care for the safety
and security of its patients. (D) is incorrect. An intervening criminal act of a third party will not
cut off liability if it was foreseeable and the defendant owed a duty of care to the plaintiff. The
nursing home has assumed the duty of ordinary care for the safety and security of its patients, and
here criminal acts were foreseeable because the nursing home accepted violent patients.

Answer to Question 18

(B) The student probably will be able to avoid liability if he can establish that he built the bonfire
to avoid harming neighboring property. The element of breach of duty in a negligence case
requires a showing that the defendant acted unreasonably, which is a question for the trier of
fact. Assuming that the student owed a duty to avoid harm to the neighbor’s shed, the student
will be able to argue that he did not breach that duty because he positioned the bonfire to avoid
damaging the neighbor’s shed. If the facts support this argument, the student probably will be able
to avoid liability to the neighbor. (A) is incorrect because customary methods of conduct do not
conclusively establish the standard for determining whether a defendant’s conduct amounted to
negligence. Although it may have been an “accepted custom” in the community to light bonfires
at homecoming, this custom may in fact present an unreasonable risk of injury to persons in the
position of the neighbor. Thus, the argument in (A) does not enable the student to avoid liability.
(C) is incorrect because all of the direct consequences of a negligent act are viewed as proxi-
mately caused by that act. If it was reasonably foreseeable that lighting the bonfire would cause
damage to the neighbor’s shed, then the student’s conduct was negligent. If some damage to the
shed from the bonfire was foreseeable, the student is liable for the total destruction that occurred,
despite the fact that the shed would not have burned down if it were not built of highly flammable
material (i.e., the student must “take his plaintiff as he finds him”). (D) is incorrect because there
really is no restricted scope of liability for the class president with respect to persons outside his
property. A landowner has a duty to exercise reasonable care with respect to his own activities on
the land so as to avoid unreasonable risk of harm to others outside the property. Thus, the class
president would be liable to the neighbor if he had built the bonfire and thereby created an unrea-
sonable risk of harm to the neighbor’s shed, resulting in damage to the shed. The student’s status
as a guest on the property does not excuse the student from the duty to refrain from creating
an unreasonable risk of harm to the person or property of a foreseeable plaintiff, such as the
neighbor. Therefore, (D) presents nothing that will enable the student to avoid liability.
TORTS - NUISANCE,
DEFAMATION, AND
PRIVACY QUESTIONS
TORTS - NUISANCE, DEFAMATION, AND PRIVACY QUESTIONS 1.

TORTS - NUISANCE, DEFAMATION, AND PRIVACY QUESTIONS

Question 1 Question 2

A well-known Oscar-winning producer who A store security guard who reasonably but
owned the nation’s leading special effects studio mistakenly thought that a shopper had tried to
employed a creative designer who was recog- steal a scarf directed her to accompany him
nized throughout the industry as one of the top to the manager’s office, which had an interior
talents in the special effects field. A competitor window overlooking the sales floor. Because the
of the producer who wanted to hire the designer blinds were up on the window, the occupants
told him that that the producer was in negotia- of the office could be seen from the sales floor.
tions to sell his studio to a major Hollywood film After the security guard described what he had
company. In fact this statement was not true, and seen, the manager began to berate her for trying
the competitor made the statement without any to steal the scarf and threatened to prosecute
knowledge of its truth or falsity. The designer, her as a shoplifter. However, the manager had
whose employment contract with the producer neglected to make sure that the public address
was terminable at will by either party, agreed to system that he used to announce specials was
be hired by the competitor. The producer was turned off, and his statements were broadcast to
very upset when he learned of the competitor’s everyone in the store.
action because he knew that the designer would
be almost impossible to replace. The producer In an action against the store for defamation,
brought suit against the competitor. will the shopper likely recover?

If the producer establishes the above facts and (A) No, because the manager was speaking
that he suffered damages, which of the following directly to the shopper.
is likely to be true?
(B) No, because the manager did not intend for
(A) He will recover for intentional interfer- others to hear his statements.
ence with business relations, because the
competitor used improper means to hire the (C) Yes, because the manager should have
designer away from the producer. checked that the public address system was
not on.
(B) He will recover for defamation, because the
producer can show actual damages from the (D) Yes, because the manager’s belief that the
competitor’s statement. shopper had stolen the scarf was reasonable
based on the security guard’s information.
(C) He will not recover for intentional interfer-
ence with business relations, because the
designer was an at-will employee of the
producer.

(D) He will not recover for defamation, because


the producer did not show that the compet-
itor knew that his statement about the
producer’s plans was false.
2. TORTS - NUISANCE, DEFAMATION, AND PRIVACY QUESTIONS

Question 3 Question 4

A privately held manufacturing corpora- An attorney won a large medical malpractice


tion was the major employer in a local town. verdict against a doctor for a botched liposuction
The chief executive officer (“CEO”) of the procedure performed on a patient. The patient
marketing division of the corporation was a was unaware that the doctor was board-certified
respected figure with a good reputation in the only in dermatology but not in plastic surgery. In
community. The CEO was suddenly fired by the a televised interview in which he discussed the
executive vice president of the main corporation, trial, the attorney stated that “doctors who are
prompting rumors in the town about the finan- not board-certified need to notify their patients
cial health of the marketing division. A reporter regarding their true qualifications.” The doctor
from the town newspaper interviewed the vice brought a defamation action against the attorney
president and asked him why the CEO had been based on the statement.
dismissed. The vice president said: “The CEO
was fired because he was a bad manager and the Who will prevail?
marketing division lost money because of the
CEO’s stewardship.” The vice president’s state- (A) The attorney, because the attorney’s state-
ment was printed in the town newspaper, and ment is related to judicial proceedings and
was picked up by business-oriented publications. thus is privileged.

If the CEO sues the town newspaper for (B) The attorney, because the doctor was not
defamation, which of the following statements board-certified in plastic surgery.
with regard to damages is correct?
(C) The doctor, because he was board-certified
(A) To prevail, the CEO must plead and prove in dermatology.
pecuniary damages, such as an inability to
find a position with another company. (D) The doctor, because the statement clearly
referred to him.
(B) To prevail, the CEO must show evidence of
actual injury, such as mental distress.

(C) Damages are presumed because the written


repetition of a slander is characterized as
libel.

(D) Damages are presumed if the court deter-


mines that the CEO is not a public figure.
TORTS - NUISANCE, DEFAMATION, AND PRIVACY QUESTIONS 3.

Question 5 Question 6

Two mothers were talking outside of a school A man working at a clothing store discovered
one afternoon. One of them asked the other for that his girlfriend, a coworker, had been taking
the phone number of the woman she employed money from the cash register. Not wanting to be
in her home to take care of her children. That a party to the situation, he ended the relation-
mother, knowing that reliable childcare is hard to ship and found another job. Not long after this,
find, did not want to share the information with the man’s new boss, who knew why the man had
the other woman, so she instead said, “You don’t quit, came into the clothing store. He asked the
want her number, we discovered that she was girlfriend if she missed her boyfriend working
intoxicated last week when we came home from with her at the store. She replied, “Yes, but when
work.” This accusation was false, but it quickly we found that he was stealing from the cash
spread throughout the neighborhood. register, we had no choice but to let him go.”

If the woman sues the mother for defamation If the man sues his former girlfriend for
and establishes the above facts, what else must defamation, the fact that the new boss knew the
the woman show? truth of why the man had left his job at the store
will have what result?
(A) That she suffered special damages by rea-
son of the mother’s oral statement. (A) It will act as a complete defense to an ac-
tion for defamation.
(B) That she lost other childcare jobs because
of the statement. (B) It will establish that the man has not
suffered any actual injury.
(C) That the statement was extreme and outra-
geous because any drinking of alcohol is (C) It may diminish the damages that the man
against the woman’s strict moral code. would be entitled to recover.

(D) No additional facts. (D) It proves that the girlfriend had no reason-
able ground for believing that the man was
fired for dishonesty.
4. TORTS - NUISANCE, DEFAMATION, AND PRIVACY QUESTIONS

Question 7 Question 8

The wife of the president of a small but presti- The academic dean of a college was well liked
gious private college was also an instructor at and considered a competent scholar and a fine
the college. While researching an article about teacher. However, six months after her appoint-
the college president, a reporter discovered and ment as dean, she received a certified letter from
revealed in a published news story that while the the provost of the college, summarily dismissing
wife was dating the president, she had falsified her. No reasons were given in the letter for her
her academic credentials on her application for a dismissal. She had her lawyer contact the provost
position with the college. As a result of the news to discover the basis for her dismissal. Two
story, the wife was subject to verbal attacks and days later, the lawyer received a letter from the
innuendo among her colleagues. She asserted provost stating, in relevant part, “The dean was
a cause of action against the newspaper for dismissed from her employment at our college
defamation and established at trial that the story because I received an anonymous telephone call
about her was not accurate. informing me that she had purchased drugs from
a student.” The provost had, in fact, received
What else will the wife need to show to such a phone call, but the basis of the statement
prevail? was untrue, because the dean had never even
used illegal drugs, much less bought them from
(A) She suffered special (economic) damages a student.
as a result of the story.
If the dean files suit against the provost for
(B) The story revealed facts about her private libel:
affairs not generally known to the public.
(A) The dean will lose, because the provost was
(C) The newspaper published the story with merely repeating the defamatory communi-
knowledge that it was false or with reckless cation of another.
disregard for its truth or falsity.
(B) The dean will lose, because by having her
(D) The newspaper was negligent in publishing attorney ask the reason for the dismissal,
the story. the dean impliedly consented to the state-
ment in the letter.

(C) The dean will win, because the provost


should have verified the anonymous state-
ment before repeating it to the lawyer.

(D) The dean will win, because the dean was


not buying drugs.
TORTS - NUISANCE, DEFAMATION, AND PRIVACY QUESTIONS 5.

Question 9 Question 10

The sales representative of a hot dog company A boy was arrested for shoplifting. His mother
gave the proprietor of a corner hot dog stand a was informed and arrived at the police station,
poster of an actress endorsing those hot dogs, where a reporter for a daily newspaper took her
which the proprietor put in the window. Sales picture. The photograph appeared on the front
picked up, but the actress filed suit against the page of the next day’s edition of the paper, above
proprietor for invasion of privacy because, in a story about an increase in shoplifting arrests.
fact, she was a spokesperson for a rival hot dog A caption to the photograph identified her as the
company. mother of a boy arrested for shoplifting. Later
that week, the mother lost her job as a result of
Will the actress prevail? the story in the paper.

(A) No, because the actress is a public figure. Will the mother recover if she asserts a claim
against the newspaper for invasion of privacy?
(B) No, because the proprietor was reasonably
justified in believing the poster was autho- (A) Yes, because she was not involved in the
rized. events that led to the boy’s arrest.

(C) Yes, because the actress had not given her (B) Yes, because the photograph and news
permission to use the poster. story caused her to be discharged from her
employment.
(D) Yes, because the proprietor gained profit
from using the poster. (C) No, because her photograph was taken in a
public place.

(D) No, because the caption was accurate.


6. TORTS - NUISANCE, DEFAMATION, AND PRIVACY QUESTIONS

Question 11 Question 12

A husband and wife were going through a A physics teacher at the local college was
nasty divorce. The wife hired a private detective also a model rocket enthusiast. He owned a
to spy on her husband. The detective followed large parcel of land in the country, and on many
the husband to a hotel, where he saw the husband occasions he would launch one of his rockets
meet a woman and go into her hotel room. The from the back area of his property. Although
detective checked into the adjoining room, none of the teacher’s rockets ever came near his
placed an electronic listening device on the wall, neighbor’s property, the neighbor complained to
and listened to and recorded the activities of the the teacher several times about his hobby and
husband and the woman in the next room. He the fact that the teacher stored flammable fuels
did nothing with the information he gathered, in his house. Once, his neighbor complained to
however, because the wife’s retainer check to the the county sheriff, whereupon the teacher was
detective bounced. Both adultery and electronic charged with violating a local ordinance that
eavesdropping are crimes under state law. prohibits the improper storage of flammable
liquids on residential property. He was given
If the husband sues the detective for invasion a warning and told that he must have proper
of privacy, what is the likely result? storage permits and facilities if he intended
to keep the fuels for his model rockets on his
(A) The husband will win, because he had an property. Although the teacher obtained the
expectation of privacy in the woman’s hotel proper permits to build underground storage
room. tanks for his fuels, he continued to store them in
55-gallon drums in a shed located on the edge of
(B) The husband will win, because the detec- his property farthest away from his neighbor.
tive’s electronic eavesdropping was illegal.
Eventually, his neighbor brought a suit based
(C) The husband will lose, because adultery is on public nuisance against the teacher, seeking
illegal. an injunction against the teacher to prevent him
from storing flammable liquids on his property
(D) The husband will lose, because the detec- and launching model rockets.
tive published nothing about the husband’s
activities. What is the teacher’s best defense?

(A) He obtained a permit from the city to build


storage tanks for the fuels.

(B) There is no showing that his neighbor


suffered any special damage.

(C) This is not a residential neighborhood.

(D) There is no specific ordinance that prohibits


the teacher from launching model rockets
on his own property.
一 信:liuxue119118 , 们 信免 供

TORTS - NUISANCE, DEFAMATION, AND PRIVACY QUESTIONS 7.

Question 13 Question 14

A “wind farm” was constructed on the open Growers of potatoes in the state recently
land next to a gas station where a mechanic began spraying a pesticide onto their crops to
is employed. The huge windmills generated prevent the spread of a pest that can destroy
electrical power and oscillated at low speeds, young potato plants. The pesticide is manufac-
resulting in very low-frequency but high-ampli- tured exclusively by one company at its plant.
tude sound waves that caused extreme discom- When the plant is producing the pesticide
fort in a substantial minority of people. The it emits a fine, sticky, harmless mist as a
mechanic is one of the people adversely affected byproduct. The mist often drifts over a farmer’s
by such sound waves; the vibrations from the property, which is adjacent to the company’s
windmills give him severe headaches and upset plant. Although the company uses the best
his immune system. technology available, it is unable to prevent
the release of the mist. The farmer brings suit
If the mechanic brings an action for nuisance against the company on the theory of private
against the owners of the wind farm and loses, nuisance to enjoin the production of the pesticide
what is the most likely explanation? at the company’s plant.

(A) Sound waves are not the kind of physical Which of the following facts, if established,
phenomenon on which liability for nui- will be most helpful to the company’s defense?
sance can be based.
(A) The company commenced the manufacture
(B) The owners of the wind farm cannot of the pesticide at its plant three years be-
generate power without producing the fore the farmer acquired the land adjacent
offending sound waves, despite their best to the plant.
efforts to find an alternative technology.
(B) Federal, state, and local agencies approved
(C) At least six out of 10 people are not affected the design of the plant and equipment used
by low-frequency sound waves. to produce the pesticide.

(D) The mechanic neither owns nor rents the (C) The principal users of the pesticide are state
gas station at which he works. and federal departments of agriculture.

(D) The pesticide is the only type of pesticide


that can safely and effectively kill the pest,
which, if not controlled, would destroy the
state’s potato crop, its principal product.

一 信:liuxue119118 , 们 信免 供
8. TORTS - NUISANCE, DEFAMATION, AND PRIVACY QUESTIONS

Question 15 Question 16

An artist purchased a warehouse in an indus- An investor and a sports club owned adjacent
trial district that she converted into a studio parcels of land in an area that was zoned for
many years ago. A few months ago, a record commercial use. The sports club constructed
company purchased a warehouse next door to the four tennis courts on its property and equipped
artist’s studio and converted it into a recording them with bright lights for night use. The
studio and rehearsal rooms. The recording tennis court operated 365 days a year and was
studio frequently stays open long past midnight, kept open until 2 a.m. Five years later, a new
particularly on the weekends. Although the five shopping center was built one mile from their
recording studios are thoroughly soundproofed, parcels of land, making them highly desirable
the various rehearsal rooms are not. When bands for residential purposes. The investor decided to
are using them, the sound can easily be heard construct several residential units on his property
by the artist when she is working in her studio. and had his property rezoned for residential use,
The artist had always done her work at night, but his architect informed him that, because of
but since the recording studio next door opened, the lights, the residences facing the tennis courts
she has found it very difficult to concentrate on would have to be equipped with light-proof
her work. She is distracted by all the noise and draperies that would have to be kept closed until
activity from the recording studio. after the tennis courts close each night.

In a private nuisance suit against the record If the investor asserts a claim based on
company, is the fact that the artist owns her nuisance against the sports club, who is likely to
studio and has used it for 15 years controlling? prevail?

(A) Yes, on the issue of whether the record (A) The investor, because the residences will
company’s use of its property is reasonable. constitute higher use of the property than
the tennis courts.
(B) No, but it is relevant to the issue of whether
the record company’s use of its property is (B) The investor, because the glare from the
reasonable. lights interferes with the use and enjoyment
of the investor’s property.
(C) No, and it is irrelevant because the
surrounding area was used exclusively for (C) The sports club, because the area was
industry and manufacturing when the artist zoned for commercial use when the tennis
bought her property. courts were built.

(D) No, and it is irrelevant because a landowner (D) The sports club, because the tennis courts
cannot establish the reasonableness of the were built before the investor planned his
use of property simply by being the first in residential development.
the neighborhood.
TORTS - NUISANCE, DEFAMATION, AND PRIVACY QUESTIONS 9.

Question 17

A manufacturer of insecticides vital to the


local agriculture generated very foul-smelling
fumes during the operation of its plant.
When the plant was built many years ago, the
surrounding area was completely agricultural,
but now much of the area around the plant is
residential, although the plant itself is zoned
for manufacturing activities. In response to
complaints, the manufacturer’s engineers
investigated whether it would be possible to
install machinery to filter and scrub the plant
emissions, but determined that the project would
be too costly. Residents of nearby homes brought
an action for nuisance against the manufacturer.

If the manufacturer prevails, what will be the


likely reason?

(A) The plant was there first, and the residents


came to the nuisance.

(B) The area is zoned for commercial and


manufacturing activities.

(C) The fumes from the plant do not unreason-


ably interfere with the residents’ use and
enjoyment of their property.

(D) The insecticides are vital to the local


agriculture.
TORTS - NUISANCE,
DEFAMATION, AND
PRIVACY ANSWERS
TORTS - NUISANCE, DEFAMATION, AND PRIVACY ANSWERS 1.

TORTS - NUISANCE, DEFAMATION, AND PRIVACY ANSWERS

Answer to Question 1

(A) The producer can recover for intentional interference with business relations because the compet-
itor used improper means to hire the designer away. To establish a prima facie case for interfer-
ence with contract or prospective economic advantage, the plaintiff must prove: (i) the existence
of a valid contractual relationship between the plaintiff and a third party or a valid business
expectancy of the plaintiff; (ii) the defendant’s knowledge of the expectancy; (iii) intentional inter-
ference by the defendant that induces a breach or termination of the relationship or expectancy;
and (iv) damage to plaintiff. While an interferer’s conduct may be privileged where it is a proper
attempt to obtain business for the interferer, not only the ends but also the means of persuasion
used must be proper. Here, the competitor made an intentional misrepresentation with the intent
to induce the designer to leave the producer’s employment, and the producer has shown that he
suffered actual damage from the designer’s departure. (B) is incorrect because the producer
cannot recover for defamation even though he can show actual damages. Defamatory language is
language that tends to adversely affect one’s reputation in the community, such as impeaching the
individual’s honesty, integrity, virtue, sanity, or the like. The statement that the producer is negoti-
ating to sell his business is not, on the facts presented, defamatory. While the competitor could be
liable for the tort of injurious falsehood, he is not liable for defamation. Conversely, if the compet-
itor’s statement were defamatory, the producer would not need to show actual damages to recover.
The producer, who likely is a public figure, can recover presumed damages under common law
rules if he establishes that the competitor made the statement with malice, i.e., knowledge that
the statement was false or reckless disregard as to its truth or falsity, which is suggested by the
facts. (C) is incorrect because even though the competitor did not induce the designer to breach
his contract, because it was terminable at will by either party, the competitor did induce a termi-
nation of the contractual relationship and interfered with the producer’s business expectancy that
the designer would stay in his employ. While the competitor would have been privileged to try to
hire the designer away using legitimate means, the privilege does not extend to making intentional
misrepresentations about the designer’s current employer. (D) is incorrect because, as discussed
above, the producer need not show that the competitor knew that the statement was false, as long
as he shows that it was made with reckless disregard of its truth or falsity. As discussed above, he
cannot recover for defamation because he cannot establish the defamatory language element of the
prima facie case.

Answer to Question 2

(C) The shopper can recover against the store for defamation because the store manager negligently
communicated his defamatory statements to third persons. A prima facie case for defamation at
common law consists of (i) defamatory language by defendant (ii) of or concerning the plaintiff,
(iii) publication of the defamatory language by the defendant to a third person, and (iv) damage to
the reputation of the plaintiff. The publication requirement is satisfied when there is a communica-
tion of the defamatory statement to a third person who understood it. The communication to the
third person may be made either intentionally or negligently. Here, the store manager’s statements
were defamatory, they were of or concerning the shopper because those hearing the statement
could see that the manager was talking to her, and damage to the shopper’s reputation is presumed
by law because the allegation that she was a thief is slanderous per se. Because the manager
neglected to check that the public address system was not on, his broadcasting of the statements
to third persons was negligent. Hence, the publication requirement is satisfied. The store, as the
employer of the store manager, will be vicariously liable for the manager’s defamation because it
2. TORTS - NUISANCE, DEFAMATION, AND PRIVACY ANSWERS

was committed within the scope of his employment. (A) and (B) are wrong because a publication
may occur even when the defendant is not speaking to third persons or does not intend that third
persons hear his statements, as long as he was negligent in letting third persons overhear him. (D)
is wrong because the manager’s reasonable belief in the truth of his statements is irrelevant in a
common law defamation action. Because the statements here did not involve a matter of public
concern, the shopper does not need to establish that the manager was negligent in his belief as to
the truth or falsity of his statements.

Answer to Question 3

(B) To prevail, the CEO must show evidence of actual injury. When a defamatory statement involves
a matter of public concern, the plaintiff must provide competent evidence of actual injury (i.e.,
presumed damages are not permitted absent a showing of knowledge of falsity or reckless disre-
gard of truth). Actual injury is not limited to out-of-pocket loss; it may include impairment of
reputation, personal humiliation, and mental anguish. Here, the reasons for the firing of the CEO
of a town’s largest employer, and the financial health of that employer, are matters of public
concern for the readership of the town newspaper. Hence, the CEO will have to prove actual
injury to prevail. (A) is incorrect because pecuniary damages are not necessary in libel cases.
If not for the fact that a matter of public concern was involved, damages would be presumed.
(C) is wrong even though it is a true statement. The common law rule of presumed damages is
supplanted by the constitutional rules because a matter of public concern is involved. (D) is wrong
because even if the CEO is not a public figure, the article involved a matter of public concern, so
damages will not be presumed.

Answer to Question 4

(B) The attorney will prevail. Defamation is actionable if the plaintiff proves defamatory language,
“of or concerning” the plaintiff, publication by the defendant to a third person, and damage to
the plaintiff’s reputation. The statement at issue may be defamatory on its face, or the defama-
tory meaning may be established by additional facts (innuendo). In the present case, the doctor is
alleging that the attorney implied that he failed to inform the patient that he was not board-certi-
fied when he did the surgery, making the bare statement, “Doctors who are not board-certified
need to notify their patients regarding their true qualifications” defamatory by innuendo. However,
the truth of the statement is a complete defense to a defamation action. Even though it is true that
the doctor is board-certified in dermatology, the context in which the attorney made his statement
was the interview concerning malpractice in a cosmetic surgery procedure, and the doctor did not
have a board certification in plastic surgery. With all the additional facts considered, the attor-
ney’s statement is not actionable defamation. Therefore, (C) is incorrect. (A) is incorrect because,
although the statement did concern a judicial proceeding, it was not made as part of the proceed-
ings, and thus is not privileged. (D) is incorrect because, as explained above, the doctor also must
show defamatory language, publication, and damage to reputation in order to establish a prima
facie case for defamation, and the attorney can raise truth as a defense.

Answer to Question 5

(D) The woman need show no additional facts. A defamatory statement communicated to another
constitutes slander per se if it adversely reflects on a person’s abilities in her business or profes-
sion. At common law, damages are presumed for slander per se. The mother’s statement to the
other mother was defamatory and slander per se because she alleged that the woman was intoxi-
cated while on the job, which adversely reflects on her reliability and trustworthiness in her child
TORTS - NUISANCE, DEFAMATION, AND PRIVACY ANSWERS 3.

care work. Hence, the woman need not prove special damages, such as loss of child care jobs.
Thus, (A) and (B) are incorrect. (C) might be relevant in an action for intentional infliction of
distress, but it is irrelevant in a defamation action against the mother.

Answer to Question 6

(C) The new boss’s knowledge of the true circumstances behind the man’s departure from the store
may diminish the man’s recovery. The girlfriend is liable for defamation because she made a
defamatory statement about the man to a third person. As long as it is understood in its defama-
tory sense, an accusation need not be believed to be actionable. Because the statement that he was
stealing at his job constituted slander per se, damages are presumed, completing the prima facie
case. Nevertheless, the stated fact will diminish the amount of damages that the man will recover
because his reputation was not likely harmed in the eyes of his boss. (A) is wrong because, as
stated above, the man can establish a prima facie case for defamation even though the defamatory
statement was not believed. (B) is wrong because actual injury encompasses not only damage to
reputation but also humiliation and mental distress, for which the man could recover even if his
reputation was not damaged. (D) is wrong because the fact that the new boss did not believe the
statement does not prove lack of basis for the girlfriend to have made it.

Answer to Question 7

(D) For the wife to prevail, she will need to show that the newspaper was negligent. Although at
common law defamation liability could be strict, a number of Supreme Court decisions based
on the First Amendment now impose a fault requirement in cases involving public figures or
matters of public concern. A defendant may not be held liable for defamation on a matter of public
concern not involving a public figure unless, in addition to publishing a false story, it was at least
negligent in ascertaining the truth or falsity of its facts. Here, a story about an instructor at a
prestigious college falsifying her academic credentials, while dating the president of the college,
likely is a matter of public concern because it affects the perceived integrity of the institution. (C)
is incorrect because knowledge or reckless disregard is the standard applicable to public figures,
and the wife does not qualify as such merely because she is married to someone who may be
a public figure. (A) is incorrect because libel does not require proof of special damages; actual
damages (which includes noneconomic damages) is enough. (B) is incorrect because it describes a
type of invasion of privacy—public disclosure of private facts. Invasion of privacy is not relevant
to a defamation action.

Answer to Question 8

(B) The dean will lose because the provost can raise consent as a complete defense to her libel action.
A defamation action at common law required the plaintiff to prove (i) defamatory language on
the defendant’s part that is (ii) “of or concerning” the plaintiff, (iii) publication of the defama-
tory language to a third person, and (iv) damage to the reputation of the plaintiff. (Because the
defamation here does not refer to a public figure or likely involve a matter of public concern, the
elements of falsity and fault do not need to be proved.) In this case, the provost uttered a state-
ment to a third person, the lawyer, that linked the dean to the purchase of drugs; this satisfies the
first three elements of the prima facie case. Because the statement was libel (uttered in a writing),
general damages will be presumed by law; the dean does not need to show actual damages from
the statement. Hence, the dean can establish a prima facie case for libel against the provost. As
with all torts, however, consent, whether express or implied, is a complete defense to a defama-
tion action. Implied consent includes apparent consent, which is consent that a reasonable person
4. TORTS - NUISANCE, DEFAMATION, AND PRIVACY ANSWERS

would infer from the plaintiff’s conduct. Here, it was reasonable for the provost to infer that the
dean’s dispatch of her lawyer to request the reasons for her dismissal included a consent for the
provost to make defamatory statements to the lawyer as long as they related to the reasons for
the dean’s dismissal. Thus, the dean impliedly consented to the statement in the letter and will
not prevail in her suit against the provost. (A) is incorrect because the fact that the provost did
not originate the statement does not provide a defense. One who repeats a defamatory statement
will be held liable on the same general basis as the primary publisher even though the repeater
states the source or makes it clear that he does not believe the defamation. (C) is incorrect even
though the provost should have verified the statement before repeating it. His defense is that the
dean impliedly consented to the statement rather than that he did not originate the statement. (D)
is incorrect because whether the dean was buying drugs only affects whether the provost can also
raise the defense of truth in this defamation action. The provost will win by raising the defense of
consent; the fact that the dean was not buying drugs does not change this result.

Answer to Question 9

(C) The actress can recover for the unauthorized use of her picture for the proprietor’s commercial
advantage. There are four branches of the tort of invasion of privacy: (i) appropriation of the
plaintiff’s picture or name for the defendant’s commercial advantage; (ii) intrusion upon plain-
tiff’s affairs or seclusion; (iii) publication of facts that place plaintiff in a false light; and (iv)
public disclosure of private facts about plaintiff. Of these four branches, the only one applicable to
these facts is the first. To establish a prima facie case for this type of invasion of privacy, a plain-
tiff need only prove the unauthorized use by the defendant of the plaintiff’s picture or name for
the defendant’s commercial advantage. Generally, liability is limited to the use of the plaintiff’s
picture or name in connection with the promotion or advertisement of a product or service. The
proprietor has used the actress’s picture to promote the sale of hot dogs. The actress did not in any
way consent to this use by the proprietor of her picture. Therefore, the elements that the actress
needs to prove are present, and she will prevail. (A) is wrong because the actress’s status as a
public figure is irrelevant to this type of privacy tort. Public figure status could affect other types
of invasion of privacy (e.g., publication of facts placing the plaintiff in a false light and public
disclosure of private facts about the plaintiff) if the published matter was of legitimate public
interest. If so, the plaintiff would have to establish that the defendant acted with actual malice
(i.e., with knowledge of falsity or reckless disregard for the truth). If the actress’s action were
based on one of these other types of privacy tort, her status as a public figure might be relevant.
However, public figure status does not alter the elements needed to establish a case for unauthor-
ized use of the actress’s picture, nor does it impose upon her the necessity of proving actual
malice. (B) is wrong because mistake (even if reasonable) as to whether consent was given is not
a valid defense. Thus, the actress will prevail regardless of whether the proprietor was reasonably
justified in believing that the poster was authorized. (D) is wrong because there is no requirement
that the defendant have actually profited from using the picture. The proprietor is liable for this
tort simply because he used the actress’s picture without her permission to promote and advertise
the hot dogs.

Answer to Question 10

(C) The facts do not make out a claim for invasion of privacy in any of the four forms that invasion of
privacy takes. The photograph was not an appropriation for commercial purposes because it was
incidental to a legitimate news story and was not used in an advertisement. The photograph did
not involve intrusion because it was taken in a public place. The news feature did not involve false
light because the facts do not suggest that the photograph conveyed a false impression. Finally,
TORTS - NUISANCE, DEFAMATION, AND PRIVACY ANSWERS 5.

the mother’s appearance at the police station simply was not a private fact, because it is generally
agreed that anything visible in a public place may be recorded and given circulation by means
of a photograph. (A) and (B) are therefore incorrect because those facts do not establish that an
invasion of privacy occurred. (D) is incorrect because truth is not a defense to most invasion of
privacy actions. Even for false light invasion of privacy, the fact that the caption was true does not
preclude recovery if the photograph otherwise conveyed a false impression. Thus, (C) is a better
choice than (D).

Answer to Question 11

(A) The detective is liable for the type of invasion of privacy of intrusion upon the plaintiff’s private
affairs or seclusion. The husband could reasonably expect that what he did in the woman’s hotel
room would be free from intrusion. The detective violated the husband’s seclusion by listening to
the activities in the room. This intrusion would be highly offensive to a reasonable person. (D) is
wrong because the viability of this cause of action does not depend on publication by the defen-
dant; the interest protected by this type of invasion of privacy is the plaintiff’s right to be let alone,
rather than his interest in not having the information disseminated. (B) is wrong because illegality
of the defendant’s actions is not an element of the prima facie case for this form of invasion of
privacy. Even if the eavesdropping were legal, the detective’s conduct would still be an action-
able intrusion upon the husband’s private domain. Finally, the fact that the husband may have
been engaging in illegal adulterous conduct does not justify a private individual’s intrusion on his
solitude. (The situation might be different if the detective were a law enforcement officer acting
pursuant to previously obtained judicial authorization.) Thus, (C) is wrong.

Answer to Question 12

(B) The best defense is that there is no showing of special injury. A public nuisance exists when a
property owner is using his property in such a manner that it creates an unreasonable risk to
the public in general. The owner need not be violating a specific statute in order to be a public
nuisance; hence, (D) is incorrect. Nor is it determinative that the owner has a permit to engage
in the activity if the owner’s use otherwise constitutes a public nuisance, making (A) incorrect.
(Albeit, in many cases it would be hard to show a public nuisance if the owner did have a permit
to use his property in a certain manner. But the facts of this case show that the teacher had not
complied with the provisions of the permit.) The teacher would be correct in arguing that it is not
a nuisance per se in storing the flammable liquids on his property because this is not a residential
neighborhood; however, (C) is not the best answer because it still could be shown that under this
particular situation it was a nuisance. Nevertheless, it is generally the duty of the local district
attorney to bring actions to enjoin public nuisances, and private citizens will generally have no
standing to bring such suit unless the private citizen can show that he suffers some special injury
in addition to that sustained by the general public. The facts in this case fail to show such special
injury, so (B) would be the teacher’s best defense.

Answer to Question 13

(D) If the mechanic loses, it will be because he does not own or rent the affected property. A private
nuisance is a substantial and unreasonable interference with the use and enjoyment of real
property. Because the mechanic does not own or rent the property where he works, he may not
pursue a claim based on interference with that real property’s use and enjoyment. (A public
nuisance is an interference with the rights of the community at large—a situation not presented
by the fact pattern.) (A) is wrong because sound waves may be a basis for a nuisance action.
6. TORTS - NUISANCE, DEFAMATION, AND PRIVACY ANSWERS

(C) is wrong because, if 40% of the people are bothered by the sound waves, they probably are
a nuisance. It is not necessary for a majority of the population to be affected for there to be a
nuisance, but more than one must be affected. The choice between (B) and (D) is a difficult one.
Nuisance requires an unreasonable interference with the property, and if the facts in (B) were
true, the defendants could arguably be acting reasonably. However, (D) is more clearly a basis for
the mechanic to lose than (B).

Answer to Question 14

(D) The importance of the pesticide to a critical crop is most helpful to the company. A private
nuisance action requires a showing that the defendant’s interference with the use and enjoyment
of the plaintiff’s property was unreasonable. To be characterized as unreasonable, the severity of
the inflicted injury must outweigh the utility of the defendant’s conduct. Here, the fact that the
pesticide is the only means of preventing destruction of the state’s principal agricultural product
would be the most persuasive additional fact for the company’s defense. (A) is incorrect because
"coming to the nuisance" is generally not a good defense to a nuisance action. A purchaser of land
is entitled to reasonable use or enjoyment of the land to the same extent as any other owner as
long as the land was purchased in good faith and not for the sole purpose of a harassing lawsuit.
(B) and (C) are incorrect even though they state factors which may be considered by a court in
determining whether an injunction should be issued. The main question is whether the severity of
the injury outweighs the utility of the defendant’s conduct, and these two factors do not speak as
well to this point as (D).

Answer to Question 15

(B) The artist’s ownership and use of her studio is relevant but not controlling. The fact that one type
of land use was entered into before another is relevant but not conclusive evidence of the reason-
ableness of the use in a private nuisance action. Hence, (A) is incorrect. (C) is not correct because
it does not matter what use the neighborhood was put to many years before if the facts show that,
subsequently, the neighborhood has changed to another use that would be inconsistent with the
record company’s use of its property. This rule applies as well in "coming to the nuisance" cases,
where the plaintiff arrives in the neighborhood after the defendant has established its use of the
property—the defendant may not condemn the surrounding premises to endure the nuisance just
because its use of the property was first. (D) is incorrect because the fact that the artist was there
first is still relevant to the issue of the reasonableness of the record company’s use of its property.

Answer to Question 16

(C) The sports club is likely to prevail because the area was zoned for commercial use when the
tennis courts were built. Private nuisance is a substantial, unreasonable interference with another
private individual’s use or enjoyment of property he actually possesses or to which he has a right
of immediate possession. While not conclusive, conduct consistent with what a zoning ordinance
permits is relevant evidence that the use is not a nuisance. (A) is incorrect because higher use of
property may be a factor considered when the courts balance the respective uses, but it would not
outweigh the legislative authority established by the zoning ordinance. (B) is incorrect because
the interference with another’s use and enjoyment must be substantial and unreasonable to be
actionable. (D) is incorrect because coming to the nuisance is not a good defense. The prevailing
rule is that a defendant may not condemn surrounding premises to endure the nuisance; i.e., the
purchaser is entitled to reasonable use or enjoyment of his land to the same extent as any other
owner.
TORTS - NUISANCE, DEFAMATION, AND PRIVACY ANSWERS 7.

Answer to Question 17

(C) If the manufacturer prevails, it will be because the fumes from the plant do not constitute
an unreasonable interference with the residents’ use and enjoyment of their property. Private
nuisance is a substantial, unreasonable interference with another private individual’s use or enjoy-
ment of his property. A substantial interference is one that is offensive, inconvenient, or annoying
to an average person in the community. For an interference to be characterized as unreason-
able, the severity of the inflicted injury must outweigh the utility of the defendant’s conduct.
In balancing these respective interests, courts take into account that every person is entitled to
use his own land in a reasonable way, considering the neighborhood, the values of the respec-
tive properties, the cost to the defendant to eliminate the condition complained of, and the social
benefits from allowing the condition to continue. Here, the interference is substantial because the
horrible smells are offensive and annoying to average residents who do not appear to be hypersen-
sitive. However, if the fumes are deemed not to constitute an unreasonable interference with the
residents’ use and enjoyment of their property, the requirements for establishing a private nuisance
have not been satisfied and the manufacturer will prevail. (A) is incorrect. The mere fact that an
activity creating a nuisance existed before the plaintiff came within its scope is ordinarily not a
defense. The prevailing rule is that, in the absence of a prescriptive right, the defendant may not
condemn surrounding premises to endure the nuisance; i.e., the purchaser is entitled to reason-
able use or enjoyment of his land to the same extent as any other owner as long as he buys in
good faith and not for the sole purpose of a harassing lawsuit. (B) is incorrect. Conduct consistent
with what a zoning ordinance or other legislative license permits is relevant but not conclusive
evidence that the use was not a nuisance. Thus, even though the area is zoned for commercial and
manufacturing activities, courts will still balance the hardships to determine if injunctive relief
should issue. (D) is incorrect because the fact that the insecticides are vital to the local agriculture
is just one factor among several that the courts must weigh in determining whether a defendant’s
conduct is unreasonable. (C) is a better choice because it states the legal standard that will enable
the manufacturer to avoid liability.
TORTS - STRICT LIABILITY
AND PRODUCTS
LIABILITY QUESTIONS
TORTS - STRICT LIABILITY AND PRODUCTS LIABILITY QUESTIONS 1.

TORTS - STRICT LIABILITY AND PRODUCTS LIABILITY QUESTIONS

Question 1 (C) Grant the manufacturer’s motion, because


the purchaser was in violation of the law
A motorcycle enthusiast who lived in a state when he drove off of the road.
wilderness area with rugged terrain purchased a
motorcycle that was promoted as an all-terrain (D) Grant the purchaser’s motion, because the
motorcycle in advertisements showing it going shock absorbers were dangerously inade-
over very rugged terrain. However, the shock quate for the off-road conditions shown in
absorbers that were sold with the motorcycle as the motorcycle’s advertisements.
standard equipment were not designed for rough
terrain and would not provide a safe ride under Question 2
these conditions. The owner’s manual that came
with the motorcycle stated that it should not be A woman purchased a new toaster, manufac-
driven over rough terrain without equipping tured by a well-known appliance manufacturer,
it with heavy-duty shock absorbers designed from an appliance store. The first morning the
for that purpose. The next day, the purchaser woman had the toaster, the toaster burnt the
took his motorcycle to the wilderness area and woman’s toast, so she took the toaster back to
rode onto a trail. He crested a hill and landed the store to complain. The store’s technician
hard, causing his shock absorbers to fail. The adjusted the toaster’s darkening dial and assured
purchaser lost control and crashed, suffering the woman that the toaster should work properly
serious and permanent injuries. in the future. The next morning, the woman’s
sister came over for breakfast and decided to try
The purchaser brought a strict products the new toaster. When the sister went to retrieve
liability action against the motorcycle’s manufac- her toast from the toaster, she received an
turer in a jurisdiction that does not apply its electrical shock and was injured.
comparative negligence rules to these actions.
At trial, he presented evidence of the advertise- The sister sued the appliance manufacturer
ments and the fact that the shock absorbers on a theory of strict liability for her injuries. At
installed on the motorcycle were danger- trial, the parties stipulated to the above facts.
ously inadequate under off-road conditions. The sister presented evidence that the electrical
The manufacturer presented evidence that the shock was caused by a defective power cord. At
purchaser had received the owner’s manual with the close of evidence, the manufacturer moved
the warning about the shock absorbers and had for a directed verdict.
disobeyed a posted state statute in the wilderness How should the court rule?
area forbidding motorized vehicles from leaving
the roadway. At the close of the evidence, both (A) Deny the motion, because manufacturers
parties move for a directed verdict. are strictly liable for injuries that result
from defects in their products.
What should the court do?
(B) Deny the motion, because the trier of fact
(A) Deny both motions, because the jury could could find that the toaster was defective
determine that the purchaser’s use of the when it left the manufacturer’s control.
motorcycle over rough terrain was foresee-
able. (C) Grant the motion, because the technician at
the store made adjustments to the toaster.
(B) Grant the manufacturer’s motion, because
the owner’s manual adequately warned of (D) Grant the motion, because the sister
the unsuitability of the shock absorbers for presented no evidence that that model of
off-road use. toaster had a history of electrical defects.
2. TORTS - STRICT LIABILITY AND PRODUCTS LIABILITY QUESTIONS

Question 3 Question 4

A manufacturer built a speedboat that it sold The owner of a small dry cleaning business
through an independent dealership. One day, the wanted to expand his business to include
boat’s owner was racing the boat dangerously tailoring jobs. A local sewing machine retailer
close to shore in a misguided effort to impress recommended a certain model of sewing
some sunbathers on the beach. The owner machine for that purpose to the business owner,
was heading straight for the beach and then so the owner purchased the machine from the
attempted to turn quickly away, but the steering retailer for the use of one of his employees, an
failed and the boat crashed onto shore, injuring experienced seamstress. The machine had a
a sunbather. The sunbather asserts a claim based defect that caused an injury to his employee’s
on negligence against the manufacturer. At trial, fingers, putting her out of work for several
she presents evidence of the above facts and weeks. As a result, the business had to turn away
evidence that the steering failure resulted from all tailoring requests for that time period. The
a defect in the boat that was present when it left business owner asserted a claim based on strict
the manufacturer. At the end of the sunbather’s liability in tort against the retailer of the sewing
case, the manufacturer moves for a directed machine, seeking damages for loss of business
verdict. because of the injury to his worker.

How should the court rule? Is the business owner likely to recover?

(A) Grant the motion, because the sunbather (A) Yes, because the sewing machine was the
failed to present evidence of negligence on actual and proximate cause of the business
the part of the manufacturer. owner’s loss of business.

(B) Grant the motion, because the sunbather’s (B) Yes, because the retailer knew that the
uncontroverted evidence established that sewing machine was to be used in the
the boat owner’s negligence was a super- tailoring business.
seding cause of the accident.
(C) No, because economic loss from injury to
(C) Deny the motion, because the defect is not an employee is not within the scope of the
something that would ordinarily occur in retailer’s duty.
the absence of negligence.
(D) No, because the retailer was not the
(D) Deny the motion, because the manufacturer manufacturer of the sewing machine.
placed a dangerously defective boat into the
stream of commerce.
TORTS - STRICT LIABILITY AND PRODUCTS LIABILITY QUESTIONS 3.

Question 5 Question 6

The owner of a gardening service looking to A homeowner purchased a new electric garage
purchase a lawnmower for his business asked a door opener directly from the manufacturer. A
retailer if he would install a lightweight blade on representative of the manufacturer installed the
a particular lawnmower that he liked, but which opener and tested it after installation. The opener
had a heavy duty blade on it designed for cutting had a safety sensor that was supposed to detect
thick weeds. The retailer agreed and installed when anything was in the path of the door and
a lightweight blade designed for lawn cutting, automatically stop and retract. One day as the
but the blade was not designed to be used on homeowner was backing out of her garage, she
that style of lawnmower. Two weeks later, while accidently hit the garage door button too early,
a worker for the gardening service was using causing the garage door to come down and crush
the lawnmower to cut through a patch of heavy the hood of her car. The homeowner brought a
weeds, the blade broke and a piece of it flew off negligence action against the manufacturer to
the lawnmower and severely cut the worker’s leg. recover her damages.
Assume that the jurisdiction follows traditional
contributory negligence rules. Evidence at trial showed that the manufac-
turer had tested a sample of the garage door
If the worker asserts a claim based on strict openers in the same series as the one installed in
liability in tort against the lightweight blade’s the homeowner’s garage, and had conducted its
manufacturer, which of these is the strongest normal tests with regard to the installation. The
defense? particular opener installed in the homeowner’s
garage, however, had a defective circuit that
(A) The worker was not in privity of contract disabled the safety sensor.
with the blade manufacturer.
To establish a prima facie case, which of the
(B) The worker was contributorily negligent following best states what the homeowner must
in using the lightweight blade to cut heavy establish?
weeds.
(A) The safety sensor was defective, and the
(C) The blade manufacturer used every avail- manufacturer failed to inspect the safety
able means to inspect and discover defects sensor on the homeowner’s opener.
in the blade.
(B) The safety sensor was defective.
(D) The blade manufacturer could not foresee
that the grass blade might be used on heavy (C) The safety sensor was defective, and the
weeds. manufacturer inspected the safety sensor
but failed to discover the defect.

(D) The safety sensor was defective, and the


manufacturer could have discovered the
defect if it had properly inspected the safety
sensor.
4. TORTS - STRICT LIABILITY AND PRODUCTS LIABILITY QUESTIONS

Question 7 Question 8

During the sale of a car, the seller informed A homeowner purchased a product that
the purchaser that the seller had previously makes it easy to remove vinyl tile by chemically
bought four new tires for the car from a tire breaking down the bonding properties of the
shop and had put 2,000 additional miles on the adhesive. The label on the product instructed
car before deciding to sell it. The purchaser users to allow several hours for the product to be
of the car immediately took it on a long trip, absorbed before attempting removal of the tile
often driving it over the posted limit. As he and warned not to use the product other than in
approached his destination, he was driving 10 compliance with the instructions and not to use
m.p.h. over the speed limit when his left front any other product in conjunction with it. The
tire suddenly blew out because of a defect. The homeowner believed that she could hasten the
car went out of control, crashed into another absorption process by heating the tile, so she
vehicle, and the purchaser was severely injured. used an industrial strength blow dryer set on
The defect in the tires could have been discov- high shortly after applying the product. The heat
ered by the seller by a reasonable inspection at caused toxic fumes to be released by the product,
the time she selected them. damaging her lungs.

If the purchaser sues the seller on a strict The homeowner brought an action against
liability theory, who is likely to prevail? the manufacturer of the product based on strict
liability in tort for failure to warn. At trial, the
(A) The purchaser, because the tire was de- homeowner presented evidence of her use of the
fective when the seller sold the car to the product and its manufacture by the manufac-
purchaser. turer, the injury she suffered, and that the label
bore no warning regarding applying heat to the
(B) The purchaser, because the seller could product. The manufacturer presented evidence
have discovered the defect with a reason- that it had received no reports of persons injured
able inspection. by fumes from the product in the 10 years that
the product had been on the market.
(C) The seller, because the purchaser misused
If the manufacturer moves for a summary
the tire by exceeding the posted speed limit
judgment at the close of all of the evidence, what
on his trip.
should the court do?
(D) The seller, because the seller is not a (A) Deny the motion, because a jury could find
commercial supplier. that the product was in a defective condi-
tion unreasonably dangerous to users based
on the absence of warnings regarding the
use of heat.
(B) Deny the motion, because the homeowner
has established that she suffered injury as a
result of her use of the product.
(C) Grant the motion, because the homeowner’s
failure to follow the instructions constituted
contributory negligence as a matter of law.
(D) Grant the motion, because the long history
of no injuries demonstrates that the
warnings on the label were adequate and
the product was not defective.
TORTS - STRICT LIABILITY AND PRODUCTS LIABILITY QUESTIONS 5.

Question 9 Question 10

A motorist driving her brand new SUV was The owner of a self-propelled mower started
exceeding the posted speed limit by 15 m.p.h. on the engine in preparation to mow when the
a curving road when she lost control. The vehicle clutch of the mower suddenly engaged, and it
rolled over, causing her to suffer severe injuries. jerked forward rapidly. The owner was unable
She brought an action against the manufacturer to grab the handle in time before the mower
of the SUV based on strict liability in tort. At was out of reach and heading for the street. As
trial, the facts of the accident were entered into he caught up with the mower and attempted to
evidence. The motorist also presented expert restrain it in the street, a motorist driving down
testimony that stability control technology, the street swerved to avoid the mower and struck
which was available in other similar sized SUVs, a tree. The motorist was injured and her car
could have been incorporated into the model damaged. The motorist brought an action against
she owned and likely would have prevented the owner of the mower and was awarded
the vehicle from tipping over. The manufac- damages. The owner then sought indemnifica-
turer presented evidence that incorporating the tion from the manufacturer of the mower in
stability control technology would have added a jurisdiction following traditional indemnity
almost $2,000 to the cost of manufacturing each rules.
vehicle. At the close of all of the evidence, the
manufacturer moved for a directed verdict. If the owner prevails, it will be because:

How should the court rule on the manufac- (A) The owner was found to have taken reason-
turer’s motion? able care in maintaining the mower.

(A) Grant the motion, because incorporating (B) The mower clutch engaged because of some
the stability control technology would have defect in manufacture.
increased the cost of the vehicle.
(C) The clutch assembly was manufactured and
(B) Grant the motion, because the motorist’s assembled by the manufacturer.
speeding constituted contributory negli-
gence per se. (D) The jurisdiction apportions damages based
on relative fault in contribution cases.
(C) Deny the motion, because the jury could
find that the vehicle was defectively
designed if a feasible alternative design
could have incorporated stability control
technology.

(D) Deny the motion, because the jury could


find that the manufacturer should have
known that the vehicle was not safe without
the stability control technology.
6. TORTS - STRICT LIABILITY AND PRODUCTS LIABILITY QUESTIONS

Question 11 Question 12

A parent purchased for her teenage daughter Ten years ago, a chemical manufacturer
a refurbished tablet from an online auction site decided that it needed a safe place to store toxic
so she could listen to music during the times her waste that was a byproduct of its manufacturing
cell phone was taken away from her. The seller processes. A highly reputable engineering
of the tablet had purchased it from a recycling company was hired and an exhaustive survey
center and reconditioned it with cheaper parts was performed to find a site. Once the site
from a different manufacturer. One day while was selected, the chemicals were stored there
the teenager was listening to music through the underground, using state-of-the-art techniques.
tablet’s original headphones, the device shorted However, the surveyors failed to discover a crack
out and caused injury to the teenager’s eardrum. in the rock of the storage area that was too small
for their instruments to detect. Over a period of
If the teenager asserts a claim based on strict years the chemicals began to seep through the
liability in tort against the tablet’s manufacturer, crack and into the water table below the surface.
the teenager will probably: Tests disclosed that a nearby city’s water supply,
all of which came from local wells that tapped
(A) Recover, because the manufacturer put the into the water table, contained unacceptably high
device in the stream of commerce. levels of chemical contaminants. The city was
required to spend $5 million on a complex filtra-
(B) Recover, because the headphones were a tion system to remove the dangerous chemicals
part of the device when it was new. from its water.
(C) Not recover, because the device had been The city brought an action against the
refurbished by the seller. chemical manufacturer to recover the cost of
building the filtration system. At the end of the
(D) Not recover, because the teenager was not plaintiff’s case establishing the above facts, the
in privity of contract with the manufacturer. manufacturer moved for a directed verdict.

Should the court grant the manufacturer’s


motion?

(A) Yes, because the city has not presented any


evidence that the manufacturer breached a
duty owed to the city.

(B) Yes, if the manufacturer also established


that the city’s water wells were drilled after
the manufacturer stored the chemicals.

(C) No, if the court determines as a matter of


law that storage of the chemicals creates a
foreseeable risk of serious harm even when
reasonable care is exercised by all actors.

(D) No, because the trier of fact could deter-


mine that storage of the chemicals creates a
foreseeable risk of serious harm even when
reasonable care is exercised by all actors.
TORTS - STRICT LIABILITY AND PRODUCTS LIABILITY QUESTIONS 7.

Question 13 Question 14

A trucker owned and operated a small truck A landowner operated a honey farm on her
which he used commercially to haul dynamite to property adjacent to a busy state highway.
construction sites. Unbeknownst to the trucker, The landowner had numerous hives for her
there was a hidden defect in the latch that held honeybees that she carefully maintained and
the rear panel of the truck. The trucker was operated in compliance with all appropriate
hauling a load of dynamite one morning and regulations. A motorist was driving his sports
exceeding the speed limit when his truck struck car on the highway at a high rate of speed after
a bump in the road, the latch malfunctioned, and a rain shower when he lost control on the wet
the rear panel of the truck flew open. One box of pavement. His car crossed in front of a motor-
dynamite fell out of the truck and struck a pedes- cyclist who was going in the opposite direction,
trian, breaking her foot. causing the motorcyclist to crash into a ditch on
the side of the road. The motorist’s car continued
If the pedestrian sues the trucker under strict off the road onto the landowner’s property and
liability for her injuries, will the pedestrian win? smashed into one of the beehives, driving a
swarm of bees out of the hive. The motorcyclist,
(A) Yes, because hauling dynamite is an abnor- who suffered only a few bruises when his motor-
mally dangerous activity. cycle crashed, saw the swarm of bees and started
to run across the road to get away from them.
(B) Yes, because the trucker was speeding He stumbled and was struck by a truck, causing
while driving with the dynamite. him to suffer several broken bones and serious
internal injuries.
(C) No, because the defect in the latch was not
discoverable upon reasonable inspection. Can the motorcyclist recover any damages
from the landowner?
(D) No, because the dynamite did not explode.
(A) Yes, because the landowner is strictly liable
for injury caused by the honeybees.

(B) Yes, because the motorcyclist was a traveler


on a public road.

(C) No, because the honeybees did not directly


inflict injury on the motorcyclist.

(D) No, because the landowner exercised due


care in her operation of the beehives.
8. TORTS - STRICT LIABILITY AND PRODUCTS LIABILITY QUESTIONS

Question 15 produce this liquid, and the manufacturing plant


secured an expert opinion on how to dispose of
Several cars of a freight train transporting the hazardous chemical byproduct. The expert
nuclear waste derailed as the train neared a concluded that the earth beneath the disposal site
street crossing. One of the cars struck a motor- was impermeable, and that there was no danger
ist’s car as it was waiting at the crossing gate, of contaminating the underground waters if
seriously injuring the motorist. The area around the chemical were buried. Based on this expert
the accident was immediately evacuated, but opinion, the hazardous chemical was buried in
fortunately none of the freight cars ruptured in a depression on the land because the head of
the derailment. the manufacturing plant reasonably believed
that it was safe. The chemical, nonetheless,
In an action alleging strict liability against seeped through the underlying soil strata, and
the railway that operated the freight train, was carried by the flow of percolating water to
the motorist established the above facts and a neighboring well used by the adjacent sheep
presented evidence of her injuries. The railway farm to water the sheep. The chemical rendered
presented evidence that the derailment was the water in the well unfit for consumption by
caused by a hidden defect in the spikes that sheep.
anchored the rails to the track. The spikes were
manufactured by its regular supplier and had not The sheep farmer had bought the farm after
previously caused any problems. The railroad the plant was built. While he was unaware of
also presented evidence that the local authori- the hazardous chemical disposal underground
ties were supposed to restrict access to roads when he bought the farm, the sheep farmer was
crossing the freight line while that particular later told his well may be contaminated, and he
train was in transit, but they had failed to do so. did nothing about it. The sheep were harmed
by drinking from the contaminated well, and
In this action, is the motorist likely to prevail? the sheep farmer asserts a claim against the
manufacturing plant for damages to the sheep in
(A) Yes, because the spikes were in a defective a jurisdiction that follows traditional contribu-
condition that made them unreasonably tory negligence rules.
dangerous.
Which of the following is the manufacturing
(B) Yes, because the railway was engaged in an plant’s best defense?
abnormally dangerous activity.
(A) Many companies converted their power
(C) No, because the injury did not arise from plants so that they could utilize the syn-
the dangerous propensity of the activity. thetic liquid developed by the manufactur-
ing plant.
(D) No, because the negligence of the local
authorities in failing to restrict access to (B) The sheep farmer did not do what a reason-
roads crossing the freight line was a super- able person would have done to prevent
seding cause of the motorist’s injuries. harm to his sheep after he learned that the
well was contaminated.
Question 16
(C) The sheep farmer was contributorily negli-
A new synthetic liquid was created that could gent.
safely double the output of electrical power
plants. One byproduct of the production of the (D) The manufacturing plant was in place
liquid was a hazardous chemical that was not and in operation before the sheep farmer
biodegradable in the environment. A state- purchased his property.
of-the-art manufacturing plant was built to
TORTS - STRICT LIABILITY
AND PRODUCTS
LIABILITY ANSWERS
TORTS - STRICT LIABILITY AND PRODUCTS LIABILITY ANSWERS 1.

TORTS - STRICT LIABILITY AND PRODUCTS LIABILITY ANSWERS

Answer to Question 1
(A) The court should deny both motions because the jury should determine whether the purchaser’s
misuse of the motorcycle was foreseeable. A strict products liability action requires plaintiff to
establish that: (i) the defendant is a commercial supplier, (ii) the defendant produced or sold a
defective product, (iii) the defective product was the actual and proximate cause of the plaintiff’s
injury, and (iv) the plaintiff suffered damage to person or property. If the product was dangerous
beyond the expectation of the ordinary consumer or a less dangerous alternative or modifica-
tion was economically feasible, the supplier has supplied a defective product. Furthermore, while
some products may be safe if used as intended, they may involve serious dangers if used in other
ways. Courts require suppliers to anticipate reasonably foreseeable uses even if they are misuses
of the product. In this case, the manufacturer has supplied its motorcycle with standard shock
absorbers that are probably safe for use on the road. However, the advertisements promoted use
of the motorcycle for off-road purposes, and purchasers may not have taken note of the warning
in the owner’s manual. It is a question for the trier of fact to determine whether the manufac-
turer should have foreseen that purchasers would use the motorcycle on rough terrain without
buying different shock absorbers. (B) is wrong because a simple warning of danger in an owner’s
manual is not sufficient under the “feasible alternative” approach if it would not be effective to
deter users of the motorcycle from using it on rough terrain. The jury will need to determine
whether the manufacturer should have provided different shock absorbers or changed the way
the motorcycle was advertised. (C) is wrong. The fact that the purchaser was in violation of the
law when he drove off of the road may establish that his conduct was contributorily negligent.
However, ordinary contributory negligence such as failing to discover a defect or guard against
its existence is not a defense to a products liability action based on strict liability in jurisdictions
that do not apply their comparative negligence rules to strict products liability actions. The type of
contributory negligence where one voluntarily and unreasonably encounters a known risk, which
is essentially assumption of risk, would be a defense to strict liability, but there is no indication
that the purchaser learned of the risk (such as by reading the owner’s manual) and decided to take
a chance anyway with the shock absorbers that he had. Hence, the purchaser’s driving off the
road does not warrant granting the manufacturer’s motion. (D) is wrong because the fact that the
shock absorbers were not safe for off-road use does not establish that they were so defective as to
be unreasonably dangerous. The jury will need to determine whether the off-road use was suffi-
ciently foreseeable to make the motorcycle unreasonably dangerous to users.
Answer to Question 2
(B) The court should deny the motion. To hold a manufacturer strictly liable for a defect in a product,
the product must have reached the consumer without substantial change in the condition in which
it was supplied. The jury could find that the toaster was defective when it left the manufacturer’s
control, so that the manufacturer produced a product that is so defective as to be unreasonably
dangerous. This defective product actually and proximately caused the sister’s injuries; hence,
the manufacturer’s motion should be denied. (A) is wrong because, unlike (B), it ignores the
requirement that the defect be attributable to the manufacturer. (C) is wrong because the sister’s
strict liability action is based on a defect in the electrical cord of the toaster. The facts do not
establish that the technician’s adjusting of the darkening dial had any effect on the toaster’s power
cord. Thus, the technician’s input does not preclude the success of the sister’s action against the
manufacturer. (D) is wrong because a history of defects in this model of toaster would show that
the manufacturer should have had notice of the problem. Such notice would be relevant to a negli-
gence action, but not to an action for strict liability.
2. TORTS - STRICT LIABILITY AND PRODUCTS LIABILITY ANSWERS

Answer to Question 3

(C) The court should deny the motion because the sunbather can rely on res ipsa loquitur to get the
case to the jury. To prevail on a negligence claim, the sunbather must show negligent conduct by
the manufacturer, leading to the supplying of a dangerously defective product by the company.
The failure to exercise reasonable care, which is the critical distinction between a products
liability action based on negligence and one based on strict liability, can be established by res ipsa
loquitur in a case such as this, because the steering defect at the manufacturing stage would not
usually occur without some negligence on the part of the manufacturer. Hence, (C) is correct and
(A) is wrong. (B) is wrong because any negligence on the part of the owner is reasonably foresee-
able and will not relieve the manufacturer of the consequences of its negligence. (D) is wrong
because simply placing a defective boat into the stream of commerce would present grounds for a
strict liability action, but not one for negligence. Some negligence must be shown.

Answer to Question 4

(C) The business owner will not recover, because economic loss from injury to an employee is not
within the scope of the retailer’s duty. Most courts have refused to extend strict products liability
to cases in which plaintiff suffers only economic loss when not accompanied by physical harm
to the plaintiff, requiring the plaintiff instead to bring an action for breach of warranty to recover
such damages. (In contrast, the employee would probably be successful in her strict liability
action because the product’s defect caused her injuries.) Here, the business owner’s only loss is
economic; hence, he cannot recover in a strict liability action. (A) is therefore wrong. (B) is wrong
because it would have no bearing on a strict liability in tort action; it would be relevant only in
an action based on breach of implied warranty. (D) is wrong because strict liability extends to
retailers, because they are commercial suppliers of products.

Answer to Question 5

(D) The blade manufacturer’s best defense is that it could not foresee that the grass blade might be
used on heavy weeds. Manufacturers must foresee reasonably foreseeable misuses of a product,
but a product manufacturer is not liable if the injury results from an unintended and unforeseeable
misuse of the product. (A) is wrong because strict liability extends to foreseeable users; absence
of privity is not a defense. (B) is wrong because the facts do not indicate that the worker knew
about the lightweight blade, and, in any case, states retaining traditional contributory negligence
rules hold that ordinary contributory negligence, including reasonably foreseeable misuse, is not a
defense to strict products liability. (C) is wrong because the care exercised by the manufacturer is
irrelevant in a strict liability action; the supplying of an unreasonably dangerous defective product
satisfies the breach element of the tort.

Answer to Question 6

(D) The plaintiff must show that the manufacturer could have discovered the defect. In a products
liability action based on negligence, the plaintiff must show negligent conduct on the part of the
manufacturer that led to the supplying of a product with an unreasonably dangerous defect. The
standard of care in a negligence action is reasonable care, and the plaintiff must show that the
defendant has failed to exercise such care. The manufacturer’s failure to discover the defect when
a proper inspection would have revealed it establishes the failure to exercise due care. (A) is wrong
because it does not refer to the standard of care. The failure to inspect must be shown to be a
breach of the duty of care, which would only be the case if the defect could have been discovered
TORTS - STRICT LIABILITY AND PRODUCTS LIABILITY ANSWERS 3.

by an inspection. (C) is similarly wrong because it does not show that the manufacturer’s standard
of conduct was unreasonable; only if the defect could have been discovered by a reasonable
inspection, as indicated by (D), would the failure to do so constitute a breach of duty. (B) is wrong
because it does not refer to the manufacturer’s conduct at all, which is critical to an action based
on negligence.

Answer to Question 7

(D) The seller is not strictly liable to the purchaser because the seller is not a commercial supplier of
tires. The purchaser’s lawsuit is alleging that the seller is strictly liable for selling him a car with
a defective tire. To establish a prima facie case for a defective product based on strict liability, the
plaintiff must show: (i) the defendant is a commercial supplier; (ii) production or sale of a defec-
tive product; (iii) actual and proximate cause; and (iv) damages. While the purchaser might be
able to establish the other elements, he cannot establish that the seller is a commercial supplier of
either tires or automobiles. A casual seller who is not in the business of manufacturing, distrib-
uting, or selling the product is not strictly liable to subsequent purchasers; thus, the purchaser will
not recover against the seller. (A) is incorrect because the fact that the tire was defective would
establish liability only if the seller were a commercial supplier. Because she is only a casual seller,
she is not strictly liable to the purchaser. (B) is incorrect because the seller’s failure to discover the
defect, even if negligent, has no relevance to whether she is strictly liable. (C) is incorrect because
misuse that is reasonably foreseeable, such as exceeding the posted speed limits, is not a defense
to a strict products liability action. If the seller were otherwise liable to the purchaser, the purchas-
er’s conduct would not preclude his recovery.

Answer to Question 8

(A) The court should deny the motion because a jury could find that the product was defective. To
establish a prima facie case in products liability based on strict liability in tort, the following
elements must be proved: (i) the defendant is a commercial supplier (ii) that produced or sold a
defective product (iii) that actually and proximately caused (iv) damage to the plaintiff’s person
or property. To establish the second element, the plaintiff need prove only that the product in fact
is so defective as to be “unreasonably dangerous.” Here, the homeowner has presented evidence
that she was injured by a product supplied by the manufacturer, and alleged that the product was
in a defective condition unreasonably dangerous to users because of inadequate warnings. In
failure to warn cases, the plaintiff is entitled to a presumption that an adequate warning would
have been read and heeded, although the jury could very well reject that presumption based on the
homeowner’s conduct. Nevertheless, whether inadequate warnings made the product so defective
as to be unreasonably dangerous is a question of fact for the jury; hence, the motion should be
denied. (B) is wrong because the homeowner must also establish as part of her prima facie case
that the product was defective. (C) is wrong because even if the homeowner’s failure to follow
instructions constituted contributory negligence, it would only reduce the homeowner’s recovery
by the amount of fault the jury assigns to her; it would not warrant granting a motion to take the
case away from the jury. (D) is wrong because the absence of previous injuries does not establish
the adequacy of the warnings. While it may help the manufacturer’s case, it is not sufficient to
take that determination away from the jury.

Answer to Question 9

(C) The manufacturer’s motion should be denied because the jury could conclude that the vehicle
was defectively designed if it agrees with the motorist’s contention that a reasonable alternative
4. TORTS - STRICT LIABILITY AND PRODUCTS LIABILITY ANSWERS

design could have incorporated the stability control technology. To establish a prima facie case
in products liability based on strict liability in tort, the following elements must be proved: (i)
the defendant is a commercial supplier (ii) that produced or sold a defective product (iii) that
actually and proximately caused (iv) damage to the plaintiff’s person or property. To establish
liability in a strict liability action, the plaintiff need prove only that the product is defective.
For design defects, the plaintiff usually must show a reasonable alternative design, i.e., that a
less dangerous modification or alternative was economically feasible. Here, the motorist has
made a prima facie case sufficient to withstand a motion for a directed verdict. It is a question
for the jury whether a design incorporating the stability control technology was a reasonable
alternative and, if so, whether the existing design of the vehicle was defective and unreasonably
dangerous. (A) is incorrect because the increased cost of a safety measure does not mean that
it is not economically feasible. The trier of fact weighs various factors to make that determina-
tion; hence, a directed verdict on that issue is not appropriate under these facts. (B) is incorrect.
While the manufacturer may be able to establish that the motorist was contributorily negligent
per se based on her violation of the statutory speed limit, that will just reduce the motorist’s
recovery; it does not justify granting a motion for a directed verdict. (D) is incorrect because
whether the manufacturer should have known that the vehicle was unsafe is an issue of breach of
duty in a negligence action; in a strict liability case, negligence need not be proved and the jury
will not be instructed to make that determination.

Answer to Question 10

(B) If the owner prevails, it will be because the mower clutch engaged because of some defect in
manufacture. To recover in an action for indemnity, the owner must show that the manufacturer
breached a duty that caused the injuries to the motorist. Indemnity involves shifting the entire loss
between tortfeasors. One of the circumstances in which indemnity is available allows one joint
tortfeasor to recover against a fellow joint tortfeasor due to a considerable difference in degree
of fault. Where the tortfeasor is a retailer or user of a product who negligently failed to discover
or guard against a product's defect, he can receive indemnification from the manufacturer who
was liable for the defect on either a negligence or a strict liability theory. Here, if the claim of
the motorist was valid, then the owner must have been at fault in allowing the mower to reach
the street. However, if the mower clutch engaged due to some defect in manufacture, then the
manufacturer would be a joint tortfeasor. In such an instance, the manufacturer's conduct, either
in negligently manufacturing the mower or in placing into commerce a mower that was so defec-
tive as to be unreasonably dangerous, actually and proximately caused the injuries suffered by the
motorist. Such wrongful conduct on the part of the manufacturer is considerably more culpable
than that of the owner, who apparently was merely careless in losing control of the mower. Conse-
quently, under these circumstances, the owner is entitled to indemnity from the manufacturer,
which is the "more wrongful" tortfeasor. (A) is incorrect because, as explained above, the owner
is entitled to indemnity if his negligence is appreciably less than any wrongful conduct by the
manufacturer. Thus, the owner's recovery of indemnity is not dependent on a finding that he
exercised reasonable care in maintaining the mower. (C) is incorrect because the manufacturer
would be liable even if it hired another company to manufacture or assemble the clutch assembly.
The manufacturer would be liable for a defect in the clutch assembly because the manufacturer
put the mower into the stream of commerce. (D) is incorrect because it would be a reason why the
owner would not prevail. Most jurisdictions adopting comparative contribution, whereby damages
are apportioned based on relative fault, no longer apply indemnity rules to cases based on differ-
ences in degree of fault.
TORTS - STRICT LIABILITY AND PRODUCTS LIABILITY ANSWERS 5.

Answer to Question 11

(C) The teenager will probably not recover because the device had been refurbished by the seller. To
establish a prima facie case in products liability based on strict liability in tort, the plaintiff must
prove: (i) the defendant is a commercial supplier, (ii) the defendant produced or sold a defective
product, (iii) the product was actual and proximate cause of the plaintiff’s injuries, and (iv) the
plaintiff suffered damages to person or property. To hold the commercial supplier strictly liable
for a product defect, the product must be expected to, and must in fact, reach the user or consumer
without substantial change in the condition in which it is supplied. A product manufacturer is not
strictly liable for defects that were not present when the product left the manufacturer’s control.
(A) is therefore wrong. (B) is wrong because the seller altered the device by refurbishing it with
cheaper parts, and the facts indicate that the device shorted out; there was no apparent defect in
the headphones. (D) is wrong because courts do not require privity of contract in a strict liability
in tort action.

Answer to Question 12

(C) The court should deny the chemical manufacturer’s motion if it determines that its storage of
chemicals was an abnormally dangerous activity, making the manufacturer strictly liable for
any harm caused by the chemicals. The city can bring a private nuisance action against the
chemical manufacturer for the substantial and unreasonable interference with the use of its
water. Nuisances may be based on intent, negligence, or strict liability. A strict liability standard
for engaging in an abnormally dangerous activity would apply where the activity (i) creates a
foreseeable risk of serious harm even when reasonable care is exercised by all actors; and (ii) is
not a matter of common usage in the community. Whether an activity is abnormally dangerous
is a question of law that the court can decide on a motion for a directed verdict. Because the city
has presented no evidence of negligence on the chemical manufacturer’s part, the only way that
the city can survive the manufacturer’s directed verdict motion is if the court finds that a strict
liability standard applies, as suggested by (C). (A) is incorrect. While no evidence of a breach
of duty owed in negligence has been shown, the city has presented enough evidence to estab-
lish liability if the court finds that the manufacturer was engaged in an abnormally dangerous
activity. The city has a cause of action because its use of the water table adjacent to the storage
site makes it a foreseeable plaintiff. (B) is incorrect because the fact that the city drilled its wells
after the manufacturer stored its chemicals would not bar the city from recovering and would
have no effect on the manufacturer’s motion. This defense, called “coming to the nuisance,”
is generally rejected by the courts. (D) is incorrect because, as discussed above, whether the
elements making an activity abnormally dangerous are present is a question of law for the court
rather than a question of fact.

Answer to Question 13

(D) The pedestrian will lose in this suit. The defendant is strictly liable for engaging in certain
activities when the dangerous propensity of that activity injures the plaintiff. Although hauling
dynamite is an unusually dangerous activity, it was not the dynamite’s dangerous propensities
that caused the accident. Instead, it was a defective latch in the truck. Thus, (A) is incorrect. (B)
and (C) can be eliminated because they deal with negligence issues and the pedestrian is suing
under a strict liability theory. (D) is the best answer. Note that if the dynamite had exploded,
after falling out due to the defective latch, there would have been liability under a strict liability
theory.
6. TORTS - STRICT LIABILITY AND PRODUCTS LIABILITY ANSWERS

Answer to Question 14

(D) The landowner will prevail because there is no evidence that she was negligent in her operation
or maintenance of her beehives, and she is not strictly liable for the bees getting loose. Honey-
bees kept in a hive on a honey farm are domestic animals just like other farm animals, so strict
liability does not apply. In contrast to keepers of wild animals, the owner of a domestic animal
is not strictly liable for the injuries it causes. Strict liability would only apply if the owner has
knowledge of that particular animal’s dangerous propensities (i.e., propensities more dangerous
than normal for that species). Here, while honeybees as a class can inflict harm by stinging, there
is no indication that any of these particular honeybees were more aggressive or dangerous than
normal. Hence, (A) is incorrect because the landowner would not be strictly liable for the injury
to the motorcyclist. (B) is incorrect because the fact that the motorcyclist is a traveler on a public
road establishes only that the landowner, as the adjacent landowner, owes a duty of ordinary care
as to dangerous conditions and active operations on her property. Here, the facts do not establish
that the landowner acted unreasonably in the placement of her beehives. (C) is incorrect because
another vehicle on the highway striking the motorcyclist was a foreseeable intervening force that
did not break the causal connection between the release of the bees and the motorcyclist’s injury.
If strict liability were applicable here, the harm must result from the “normally dangerous propen-
sity” of the animal involved, but fleeing from the perceived danger is part of the risk that the
dangerous propensity creates. Similarly, if the landowner were negligent, the fact that the bees did
not reach the motorcyclist would not cut off the landowner’s liability to the motorcyclist.

Answer to Question 15

(C) The motorist is not likely to prevail in a strict liability action because her injury did not arise
from the abnormally dangerous propensity of the railway’s activity. The railway’s transport of
nuclear waste likely qualifies as an abnormally dangerous activity because: (i) it creates a foresee-
able risk of serious harm even when reasonable care is exercised by all actors; and (ii) the activity
is not a matter of common usage in the community. However, the scope of liability extends only
to the dangers that would be anticipated from the activity involved; strict liability does not apply
to harms that were not caused by the normally dangerous propensity of the activity. Here, the
railway’s activity is subject to strict liability because of the danger of radioactivity inherent in
nuclear waste, but not from a derailment by itself. Because the motorist’s injuries were not caused
by the release of radioactivity, strict liability does not apply. The railway would be liable for the
injuries from the derailment only if the motorist established negligence. (A) is incorrect because
it states the standard for strict liability for defective products, which requires that the defendant
be a commercial supplier of the defective product, i.e., someone in the chain of distribution of the
product. Here, the railway is not a commercial supplier of the spikes that failed; rather, it is the
purchaser or consumer of the spikes. (B) is incorrect because, as discussed above, strict liability
does not apply to the harm that occurred here. (D) is incorrect. It is questionable whether the
negligence by the local authorities could be considered an intervening force, which must come
into play after the culpable conduct by the defendant. Even if it were an intervening force, it likely
would not be considered so extraordinary as to be an unforeseeable intervening force. Hence,
it would not constitute a superseding force that would break the causal connection between the
motorist’s injury and the railway’s actions.

Answer to Question 16

(B) Under the “avoidable consequences” rule, a plaintiff has a duty to mitigate damages to avoid
further injuries from the defendant’s conduct. Because the sheep farmer’s property was damaged
TORTS - STRICT LIABILITY AND PRODUCTS LIABILITY ANSWERS 7.

in this situation, the sheep farmer’s claim would be based on strict liability. As such, simple
contributory negligence would not be a good defense in jurisdictions following traditional contrib-
utory negligence rules. (C) is therefore incorrect. But if the plaintiff discovers the existence of the
danger and fails to act reasonably to prevent further harm from occurring, the defendant would
have a good defense, making (B) the best defense. (A) is incorrect because there is no balancing
of utility and risk where abnormally dangerous activities are involved. (D) is incorrect because
the sheep farmer would have had to have known of and appreciated the risk involved when he
purchased the property to constitute assumption of the risk. Thus, (B) is the only correct answer.

You might also like